You are on page 1of 574

EBD_7809

• Corporate Office : 45, 2nd Floor, Maharishi Dayanand Marg, Corner Market,
Malviya Nagar, New Delhi-110017
Tel. : 011-49842349 / 49842350

Typeset by Disha DTP Team

DISHA PUBLICATION
ALL RIGHTS RESERVED

© Copyright Publisher

No part of this publication may be reproduced in any form without prior permission of the publisher. The author and the
publisher do not take any legal responsibility for any errors or misrepresentations that might have crept in. We have
tried and made our best efforts to provide accurate up-to-date information in this book.

For further information about the books from DISHA,

Log on to www.dishapublication.com or email to info@dishapublication.com


Contents
TREND ANALYSIS iv

SOLVED PAPERS.
JEE Advanced – 2019 (Paper I & II) Solved Paper 1 - 48
2019-

JEE Advanced – 2018 (Paper I & II) Solved Paper 1 - 40


2018-

JEE Advanced – 2017 (Paper I & II) Solved Paper 1 - 36


2017-

JEE Advanced – 2016 (Paper I & II) Solved Paper 1 - 36


2016-

JEE Advanced – 2015 (Paper I & II) Solved Paper 1 - 40


2015-

JEE Advanced – 2014 (Paper I & II) Solved Paper 1 - 40


2014-

JEE Advanced – 2013 (Paper I & II) Solved Paper 1 - 36


2013-

IIT-JEE – 2012 (Paper I & II) Solved Paper 1 - 36


2012-

IIT-JEE – 2011 (Paper I & II) Solved Paper 1 - 44


2011-

IIT-JEE – 2010 (Paper I & II) Solved Paper 1 - 46


2010-

IIT-JEE – 2009 (Paper I & II) Solved Paper 1 - 44


2009-

IIT-JEE – 2008 (Paper I & II) Solved Paper 1 - 48


2008-

IIT-JEE – 2007 (Paper I & II) Solved Paper 1 - 40


2007-

IIT-JEE – 2006 Solved Paper 1 - 36


2006-

(iii)
EBD_7809
TREND ANALYSIS JEE ADVANCED
Chapters as per Physics Chemistry Mathematics
NCERT No. of Questions
2019

2018

2017

2016

2015

2014

2013

2019

2018

2017

2016

2015

2014

2013

2019

2018

2017

2016

2015

2014

2013
Ch. No.

1 – – – – – – – – – – – – 2 – – – – – – – –
2 2 4 1 3 4 2 3 3 – 3 1 1 1 1 – 1 – – – – 1
3 – – – – – 1 – – – – 1 – – – 1 2 1 3 1 2 1
4 1 1 1 – – 1 1 1 – 1 1 1 3 1 – – – – – – –
5 1 1 – – – 1 – 1 – – 2 1 2 – 2 1 3 1 2 1 1
6 2 3 1 – 1 3 7 1 4 3 1 1 2 4 – – – – – – –
7 1 4 6 5 3 3 1 1 2 – 2 3 – 3 1 2 1 1 1 3 1
8 1 1 1 – 2 1 1 – – – – – 1 1 1 1 – 1 1 2 1
9 1 1 – – 1 – 1 – – – – – – – 2 1 1 1 1 2 1
10 1 2 1 1 2 4 1 – – – – – – – – – – – – – 1
11 2 1 2 4 1 1 2 – – 1 1 1 1 – 4 4 6 6 5 4 4
12 1 3 3 1 – 1 1 1 – 3 – 1 1 2 – – – – – – –
13 3 – – – 2 2 1 1 – – – 3 1 – 1 – 1 1 1 1 –
14 – – – 1 1 – – – – – – – – – – – – – – – –
15 3 2 2 1 1 2 2 – 1 1 1 1 – 1 – – – – – – –
16 2 3 1 – 3 3 2 2 2 2 2 1 1 – – 2 1 – – – –
17 3 1 – – 1 1 1 – 2 2 1 2 1 1 – 1 1 – 1 1 –
18 – 0 – 3 2 4 – 3 2 1 2 1 1 2 2 2 – – 1 1 2
19 1 3 5 1 3 4 2 1 – 1 – 1 – 1 – – 1 2 1 – 1
20 – – – – – – – 2 1 – 1 2 1 2 5 2 2 3 1 2 1
21 1 – 2 2 – – 2 4 3 6 3 2 3 5 2 4 1 3 2 4 –
22 1 1 4 2 – – 2 4 3 2 3 4 5 3 3 1 6 2 2 1 8
23 – – – – – – – 1 4 1 2 3 2 3 4 4 4 4 8 8 2
24 4 2 2 4 5 3 2 1 – 1 2 – 2 2 1 2 1 1 1 1 3
25 1 0 1 1 1 1 1 2 3 – 2 2 4 2 1 2 1 1 2 1 1
26 2 1 1 2 3 2 2 4 5 6 4 4 3 4 1 2 3 1 4 3 3
27 – 1 1 2 – – 1 1 3 2 1 3 2 – 3 2 1 2 2 1 4
28 2 1 1 3 4 – 4 1 1 – 2 1 1 1 – – – – – – –
29 – – – – – – – 1 – – 1 1 – 1 2 – 1 3 3 2 4
30 – – – – – – – – – – – – – –
Total 36 36 36 36 40 40 40 36 36 36 36 40 40 40 36 36 36 36 40 40 40

(iv)
EBD_7809
EBD_7809
EBD_7809
EBD_7809
EBD_7809
EBD_7809
EBD_7809
EBD_7809
EBD_7809
EBD_7809
EBD_7809
EBD_7809
EBD_7809
EBD_7809
EBD_7809
EBD_7809
EBD_7809
EBD_7809
EBD_7809
EBD_7809
EBD_7809
EBD_7809
EBD_7809
EBD_7809
JEE ADVANCED 2018
PAPER - 1

PHYSICS 2. Consider a body of mass 1.0 kg at rest at the origin at time


r
t = 0. A force F = (at iˆ + bˆj ) is applied on the body, where
SECTION - I (MAXIMUM MARKS: 24) a = 1.0 Ns–1 and b = 1.0 N. The torque acting on the body
r
• This section contains SIX (06) questions. about the origin at time t = 1.0 s is t . Which of the following
• Each question has FOUR options for correct answer(s). ONE statements is (are) true?
OR MORE THAN ONE of these four option(s) is (are) correct r 1
option(s). (A) | t |= Nm
3
• For each question, choose the correct option(s) to answer r
(B) The torque t is in the direction of the unit vector + k̂
the question.
• Answer to each question will be evaluated according to the r 1
(C) The velocity of the body at t = 1 s is v = (iˆ + 2 ˆj )ms –1
following marking scheme: 2
Full Marks : +4 If only (all) the correct option(s) is (are) 1
chosen. (D) The magnitude of displacement of the body at t = 1s is m
6
Partial Marks : +3 If all the four options are correct but 3. A uniform capillary tube of inner radius r is dipped vertically
ONLY three options are chosen. into a beaker filled with water. The water rises to a height h
Partial Marks : +2 If three or more options are correct but in the capillary tube above the water surface in the beaker.
ONLY two options are chosen, both of which are correct The surface tension of water is s. The angle of contact
options. between water and the wall of the capillary tube is q. Ignore
Partial Marks : +1 If two or more options are correct but the mass of water in the meniscus. Which of the following
ONLY one option is chosen and it is a correct option. statements is (are) true?
Zero Marks : 0 If none of the options is chosen (i.e. the (A) For a given material of the capillary tube, h decreases
question is unanswered). with increase in r
Negative Marks : –2 In all other cases. (B) For a given material of the capillary tube, h is
• For Example: If first, third and fourth are the ONLY three independent of s
correct options for a question with second option being an (C) If this experiment is performed in a lift going up with a
incorrect option; selecting only all the three correct options constant acceleration, then h decreases
will result in +4 marks. Selecting only two of the three correct (D) h is proportional to contact angle q
options (e.g. the first and fourth options), without selecting 4. In the figure below, the switches S1 and S2 are closed
any incorrect option (second option in this case), will result simultaneously at t = 0 and a current starts to flow in the
in +2 marks. Selecting only one of the three correct options circuit. Both the batteries have the same magnitude of the
(either first or third or fourth option), without selecting any electromotive force (emf) and the polarities are as indicated
incorrect option (second option in this case), will result in +1 in the figure. Ignore mutual inductance between the
marks. Selecting any incorrect option(s) (second option in inductors. The current I in the middle wire reaches its
this case), with or without selection of any correct option(s) maximum magnitude Imax at time t = t. Which of the following
will result in –2 marks. statements is (are) true?

1. The potential energy of a particle of mass m at a distance r R L R 2L


from a fixed point O is given by V(r) = kr2/2, where k is a
positive constant of appropriate dimensions. This particle
is moving in a circular orbit of radius R about the point O. If
V I V
v is the speed of the particle and L is the magnitude of its
angular momentum about O, which of the following
statements is (are) true? S1 S2

k k
(A) v = R (B) v = R V V
2m m (A) Imax = (B) Imax =
2R 4R
mk 2 L 2L
(C) L = mk R 2 (D) L = R (C) t = ln 2 (D) t = ln 2
2 R R
EBD_7809
2018 - 2 JEE Advanced 2018 Solved Paper
5. Two infinitely long straight wires lie in the xy-plane along 8. Two men are walking along a horizontal straight line in the
the lines x = ±R. The wire located at x = +R carries a constant same direction. The man in front walks at a speed 1.0 ms–1
current I1 and the wire located at x = –R carries a constant and the man behind walks at a speed 2.0 ms–1. A third man
current I2. A circular loop of radius R is suspended with its is standing at a height 12 m above the same horizontal line
centre at (0,0, 3R ) and in a plane parallel to the xy-plane. such that all three men are in a vertical plane. The two
This loop carries a constant current I in the clockwise walking men are blowing identical whistles which emit a
direction as seen from above the loop. The current in the sound of frequency 1430 Hz. The speed of sound in air is
wire is taken to be positive if it is in the + ĵ direction. 330 ms–1. At the instant, when the moving men are 10 m
Whichr of the following statements regarding the magnetic apart, the stationary man is equidistant from them. The
field B is (are) true?r frequency of beats in Hz, heard by the stationary man at
(A) If I1 = I2, then B cannot be equal to zero at the origin this instant, is __________.
(0, 0, 0) 9. A ring and a disc are initially at rest, side by side, at the top
r
(B) If I1 > 0 and I2 < 0, then B can be equal to zero at the of an inclined plane which makes an angle 60° with the
origin (0, 0, 0) horizontal. They start to roll without slipping at the same
r
(C) If I1 < 0 and I2 > 0, then B can be equal to zero at the instant of time along the shortest path. If the time difference
origin (0, 0, 0) between their reaching the ground is (2 – 3) / 10 s, then
(D) If I1 = I2, then the z-component of the magnetic field at
the height of the top of the inclined plane, in metres, is
æ m0 I ö __________. Take g = 10 ms–2.
the centre of the loop is ç – ÷
è 2R ø 10. A spring-block system is resting on a frictionless floor as
6. One mole of a monatomic ideal gas undergoes a cyclic shown in the figure. The spring constant is 2.0 Nm–1 and
process as shown in the figure (where V is the volume and the mass of the block is 2.0 kg. Ignore the mass of the
T is the temperature). Which of the statements below is spring. Initially the spring is in an unstretched condition.
(are) true?
Another block of mass 1.0 kg moving with a speed of 2.0
T
ms–1 collides elastically with the first block. The collision is
II such that the 2.0 kg block does not hit the wall. The distance,
in metres, between the two blocks when the spring returns
to its unstretched position for the first time after the collision
I III is _________.

IV

V
(A) Process I is an isochoric process –1
2 ms
(B) In process II, gas absorbs heat
1 kg 2 kg
(C) In process IV, gas releases heat
(D) Processes I and III are not isobaric
11. Three identical capacitors C1, C2 and C3 have a capacitance
SECTION - II (MAXIMUM MARKS: 24) of 1.0 mF each and they are uncharged initially. They are
• This section contains EIGHT (08) questions. The answer connected in a circuit as shown in the figure and C1 is then
to each question is a NUMERICAL VALUE. filled completely with a dielectric material of relative
• For each question, enter the correct numerical value (in permittivity er. The cell electromotive force (emf) V0 = 8V.
decimal notation, truncated/rounded-off to the second First the switch S1 is closed while the switch S2 is kept
decimal place; e.g. 6.25, 7.00, –0.33, –.30, 30.27, –127.30) open. When the capacitor C3 is fully charged, S1 is opened
using the mouse and the on-screen virtual numeric keypad and S2 is closed simultaneously. When all the capacitors
in the place designated to enter the answer. reach equilibrium, the charge on C3 is found to be 5 mC.
• Answer to each question will be evaluated according to The value of er = ____________.
the following marking scheme:
Full Marks : +3 If ONLY the correct numerical value is
entered as answer.
V0
Zero Marks : 0 In all other cases. S2

r r r r
7. Two vectors A and B are defined as A = aiˆ and B = a C1 C3

S1
(cos wt iˆ + sin wt ˆj ), where a is a constant and w = p/6 C2
r r r r
rad s–1. If | A + B |= 3 | A – B | at time t = t for the first time,
the value of t, in seconds, is __________.
JEE Advanced 2018 Solved Paper 2018 - 3
12. In the xy-plane, the region y > 0 has a uniform magnetic PARAGRAPH “X”
field B1kˆ and the region y < 0 has another uniform magnetic In electromagnetic theory, the electric and magnetic phenomena
field B kˆ . A positively charged particle is projected from
2
are related to each other. Therefore, the dimensions of electric
the origin along the positive y-axis with speed v0 = p ms–1 at and magnetic quantities must also be related to each other. In the
t = 0, as shown in the figure. Neglect gravity in this problem. questions below, [E] and [B] stand for dimensions of electric and
Let t = T be the time when the particle crosses the x-axis magnetic fields respectively, while [e0] and [m0 ] stand for
from below for the first time. If B2 = 4B1, the average speed dimensions of the permittivity and permeability of free space
of the particle, in ms–1, along the x-axis in the time interval respectively. [L] and [T] are dimensions of length and time
T is __________. respectively. All the quantities are given in SI units.
y
15. The relation between [E] and [B] is
(A) [E] = [B][L][T] (B) [E] = [B] [L]–1 [T]
(C) [E] = [B] [L] [T]–1 (D) [E]=[B] [L]–1 [T]–1
B1
16. The relation between [e0] and [m0] is
–1
v0 = p ms (A) [m0] = [e0] [L]2 [T]–2 (B) [m0] = [e0] [L]–2 [T]2
x
(C) [m0] = [e0]–1 [L]2 [T]–2 (D) [m0] = [e0]–1 [L]–2 [T]2
B2 PARAGRAPH “A”
If the measurement errors in all the independent quantities are
known, then it is possible to determine the error in any dependent
13. Sunlight of intensity 1.3 kW m–2 is incident normally on a quantity. This is done by the use of series expansion and
thin convex lens of focal length 20 cm. Ignore the energy truncating the expansion at the first power of the error. For example,
loss of light due to the lens and assume that the lens aperture consider the relation z = x/y. If the errors in x, y and z are Dx, Dy
size is much smaller than its focal length. The average
and Dz, respectively, then
intensity of light, in kW m–2, at a distance 22 cm from the
lens on the other side is __________. –1
x ± Dx x æ Dx ö æ Dy ö
14. Two conducting cylinders of equal length but different radii z ± Dz = = ç1 ± ÷ ç1 ± ÷ .
are connected in series between two heat baths kept at y ± Dy yè x øè y ø
temperatures T1 = 300 K and T2 = 100 K, as shown in the –1
figure. The radius of the bigger cylinder is twice that of the æ Dy ö
The series expansion for ç 1 ± ÷ , to first power in Dy/y, is
smaller one and the thermal conductivities of the materials è y ø
of the smaller and the larger cylinders are K1 and K2 1 m (Dy/y). The relative errors in independent variables are always
respectively. If the temperature at the junction of the two added. So the error in z will be
cylinders in the steady state is 200 K, then K1 /K2 =
__________. æ Dx Dy ö
Dz = z ç + ÷.
è x y ø
Insulating material
The above derivation makes the assumption that Dx/x = 1,
T1 K1 K2 T2 Dy/y = 1. Therefore, the higher powers of these quantities are
neglected.
L
(1 – a )
L 17. Consider the ratio r = to be determined by
(1 + a )
SECTION - III (MAXIMUM MARKS: 12) measuring a dimensionless quantity a. If the error in the
• This section contains TWO (02) paragraphs. Based on measurement of a is Da (Da/a = 1, then what is the error
each paragraph, there are TWO (02) questions. Dr in determining r?
• Each question has FOUR options. ONLY ONE of these
four options corresponds to the correct answer. Da 2D a 2D a 2a D a
(A) 2 (B) 2 (C) 2 (D)
• For each question, choose the option corresponding to (1 + a ) (1 + a ) (1 – a ) (1 – a 2 )
the correct answer.
• Answer to each question will be evaluated according to 18. In an experiment the initial number of radioactive nuclei is
the following marking scheme: 3000. It is found that 1000 ± 40 nuclei decayed in the first
Full Marks : +3 If ONLY the correct option is chosen. 1.0 s. For |x| = 1, ln(1 + x) = x up to first power in x. The
Zero Marks : 0 If none of the options is chosen (i.e. the error Dl, in the determination of the decay constant l, in
question is unanswered). s–1, is
Negative Marks : –1 In all other cases. (A) 0.04 (B) 0.03 (C) 0.02 (D) 0.01
EBD_7809
2018 - 4 JEE Advanced 2018 Solved Paper

CHEMISTRY 22. In the following reaction sequence, the correct


structure(s) of X is (are)
SECTION - I (MAXIMUM MARKS: 24) Me N3
1) PBr3, Et 2O
• This section contains SIX (06) questions. X ¾¾¾¾¾®
2) NaI, Me 2CO
• Each question has FOUR options for correct answer(s). ONE
3) NaN3, HCONMe2 enantiomerically pure
OR MORE THAN ONE of these four option(s) is (are) correct
option(s).
(A) Me OH (B) Me OH
• For each question, choose the correct option(s) to answer
the question.
• Answer to each question will be evaluated according to the
following marking scheme:
Full Marks : +4 If only (all) the correct option(s) is (are)
chosen. (C) (D)
Partial Marks : +3 If all the four options are correct but Me Me OH
OH
ONLY three options are chosen.
Partial Marks : +2 If three or more options are correct but
ONLY two options are chosen, both of which are correct 23. The reaction (s) leading to the formation of
options. 1,3,5-trimethylbenzene is (are)
Partial Marks : +1 If two or more options are correct but
ONLY one option is chosen and it is a correct option. O Conc. H2SO4
(A) ¾¾¾¾¾®
Zero Marks : 0 If none of the options is chosen (i.e. the D
question is unanswered).
heated iron tube
Negative Marks : –2 In all other cases. (B) Me H ¾¾¾¾¾¾®
873 K
• For Example: If first, third and fourth are the ONLY three
correct options for a question with second option being an O
incorrect option; selecting only all the three correct options
1) Br2 , NaOH
will result in +4 marks. Selecting only two of the three correct +
2) H 3O
options (e.g. the first and fourth options), without selecting ¾¾¾¾¾®
(C) 3) sodalime, D
any incorrect option (second option in this case), will result
in +2 marks. Selecting only one of the three correct options O O
(either first or third or fourth option), without selecting any
incorrect option (second option in this case), will result in +1 CHO
marks. Selecting any incorrect option(s) (second option in
this case), with or without selection of any correct option(s) Zn/Hg, HCl
(D) ¾¾¾¾¾®
will result in –2 marks. OHC CHO
19. The compound(s) which generate(s) N2 gas upon thermal 24. A reversible cyclic process for an ideal gas is shown below.
decomposition below 300 °C is (are) Here, P, V, and T are pressure, volume and temperature,
(A) NH4NO3 (B) (NH4)2Cr2O7 respectively. The thermodynamic parameters q, w, H and U
(C) Ba(N3)2 (D) Mg3N2 are heat, work, enthalpy and internal energy, respectively.
20. The correct statement(s) regarding the binary transition metal
carbonyl compounds is (are) (Atomic numbers: Fe = 26, Ni = 28) A (P1, V1, T1) C (P2, V1, T2)
(A) Total number of valence shell electrons at metal centre
Volume (V)

in Fe(CO)5 or Ni(CO)4 is 16
(B) These are predominantly low spin in nature
(C) Metal–carbon bond strengthens when the oxidation
state of the metal is lowered
(D) The carbonyl C–O bond weakens when the oxidation B (P2, V2, T1)
state of the metal is increased
21. Based on the compounds of group 15 elements, the correct Temperature (T)
statement(s) is (are) The correct option(s) is (are)
(A) Bi2O5 is more basic than N2O5 (A) qAc = DUBC and wAB = P2(V2 – V1)
(B) NF3 is more covalent than BiF3
(B) wBC = P2(V2 – V1) and qBC = DHAC
(C) PH3 boils at lower temperature than NH3
(D) The N–N single bond is stronger than the P–P single (C) DHCA < DUCA and qAC = DUBC
bond (D) qBC = DHAC and DHCA > DUCA
JEE Advanced 2018 Solved Paper 2018 - 5
SECTION - II (MAXIMUM MARKS: 24) separating the two compartments is fixed and is a perfect
• This section contains EIGHT (08) questions. The answer heat insulator (Figure 1). If the old partition is replaced by
to each question is a NUMERICAL VALUE. a new partition which can slide and conduct heat but does
• For each question, enter the correct numerical value (in NOT allow the gas to leak across (Figure 2), the volume
decimal notation, truncated/rounded-off to the second (in m3) of the compartment A after the system attains
decimal place; e.g. 6.25, 7.00, –0.33, –.30, 30.27, –127.30) equilibrium is_______.
using the mouse and the on-screen virtual numeric keypad
in the place designated to enter the answer.
• Answer to each question will be evaluated according to 1 m3, 5 bar,
400 K 3 m 3, 1 bar, 300 K
the following marking scheme: B
A
Full Marks : +3 If ONLY the correct numerical value is
entered as answer.
Figure 1
Zero Marks : 0 In all other cases.

25. Among the species given below, the total number of


diamagnetic species is_______. A B

H atom, NO2 monomer, O2– (superoxide), dimeric sulphur in


vapour phase, Mn 3O 4, (NH4) 2[FeCl 4],(NH4) 2[NiCl 4],
Figure 2
K2MnO4, K2CrO4
30. Liquids A and B form ideal solution over the entire range of
26. The ammonia prepared by treating ammonium sulphate with composition. At temperature T, equimolar binary solution
calcium hydroxide is completely used by NiCl2.6H2O to form of liquids A and B has vapour pressure 45 Torr. At the same
a stable coordination compound. Assume that both the temperature, a new solution of A and B having mole fractions
reactions are 100% complete. If 1584 g of ammonium xA and xB, respectively, has vapour pressure of 22.5 Torr.
sulphate and 952 g of NiCl 2.6H2O are used in the The value of xA/xB in the new solution is________.
preparation, the combined weight (in grams) of gypsum
and the nickel-ammonia coordination compound thus (given that the vapour pressure of pure liquid A is 20 Torr at
produced is________. temperature T)
31. The solubility of a salt of weak acid (AB) at pH 3 is Y×10–3
(Atomic weights in g mol–1: H = 1, N = 14, O = 16, S = 32,
mol L–1. The value of Y is________.
Cl = 35.5, Ca = 40, Ni = 59)
(Given that the value of solubility product of AB
27. Consider an ionic solid MX with NaCl structure. Construct
a new structure (Z) whose unit cell is constructed from the (Ksp) = 2×10–10 and the value of ionization constant of HB
unit cell of MX following the sequential instructions given (Ka) = 1×10–8)
below. Neglect the charge balance. 32. The plot given below shows P — T curves (where P is the
(i) Remove all the anions (X) except the central one pressure and T is the temperature) for two solvents X and
Y and isomolal solutions of NaCl in these solvents. NaCl
(ii) Replace all the face centered cations (M) by anions
completely dissociates in both the solvents.
(X)
(iii) Remove all the corner cations (M) 1 2 34
(iv) Replace the central anion (X) with cation (M)
760
æ ö
Pressure (mmHg)

The value of çç number of anions ÷÷ in Z is____.


çè number of cations ø÷÷ 1. solvent X
2. solution of NaCl in solvent X
28. For the electrochemical cell, 3. solvent Y
Mg(s) | Mg2+ (aq, 1 M) || Cu2+ (aq, 1 M) | Cu(s) 4. solution of NaCl in solvent Y

the standard emf of the cell is 2.70 V at 300 K. When the


360
362

367
368

concentration of Mg2+ is changed to x M, the cell potential


changes to 2.67 V at 300 K. The value of x is_______. Temperature (K)

F On addition of equal number of moles of a non-volatile


(given, < 11500K V,1 , where F is the Faraday constant solute S in equal amount (in kg) of these solvents, the
R
and R is the gas constant, ln (10=2.30) elevation of boiling point of solvent X is three times that of
solvent Y. Solute S is known to undergo dimerization in
29. A closed tank has two compartments A and B, both filled these solvents. If the degree of dimerization is 0.7 in solvent
with oxygen (assumed to be ideal gas). The partition Y, the degree of dimerization in solvent X is______.
EBD_7809
2018 - 6 JEE Advanced 2018 Solved Paper
SECTION - III (MAXIMUM MARKS: 12) 1) H2/Pd-C
• This section contains TWO (02) paragraphs. Based on 2) NH3/D 1) H2/Pd-C 1) HCl
each paragraph, there are TWO (02) questions. 3. Br2/NaOH 2) SOCl 2
2) Mg/Et2O
S P ¾¾¾¾¾¾¾¾ ® Q ¾¾¾¾¾¾¾¾ ® R
• Each question has FOUR options. ONLY ONE of these 4) CHCl 3, KOH, D 3) MeMgBr, CdCl2 3) CO2 (dry ice)
four options corresponds to the correct answer. 5) H2/ Pd-C 4) NaBH4 4) H3O
+

• For each question, choose the option corresponding to


the correct answer. 35. The compound R is
• Answer to each question will be evaluated according to
the following marking scheme:
Full Marks : +3 If ONLY the correct option is chosen. (A) CO2H
Zero Marks : 0 If none of the options is chosen (i.e. the
question is unanswered).
Negative Marks : –1 In all other cases. HO2C

PARAGRAPH “X”
(B)
Treatment of benzene with CO/HCl in the presence of anhydrous
AlCl 3/CuCl followed by reaction with Ac2O/NaOAc gives
compound X as the major product. Compound X upon reaction
with Br2/Na2CO3, followed by heating at 473 K with moist KOH
CO2H (D) CO2H
furnishes Y as the major product. Reaction of X with H2/Pd-C, (C)
followed by H3PO4 treatment gives Z as the major product.

33. The compound Y is


36. The compound S is
OH
COBr Br
(A) (B)
HO (A) (B)
O
NH2 HN
Br
COBr
(C) (D) H
Br N
(C) NH2 (D)
34. The compound Z is

MATHEMATICS
(A) (B)
SECTION - I (MAXIMUM MARKS: 24)
O O
• This section contains SIX (06) questions.
• Each question has FOUR options for correct answer(s). ONE
OH OR MORE THAN ONE of these four option(s) is (are) correct
option(s).
(C) (D) • For each question, choose the correct option(s) to answer
the question.
• Answer to each question will be evaluated according to the
O
following marking scheme:
PARAGRAPH “A” Full Marks : +4 If only (all) the correct option(s) is (are)
chosen.
An organic acid P (C11H12O2) can easily be oxidized to a dibasic
Partial Marks : +3 If all the four options are correct but
acid which reacts with ethylene glycol to produce a polymer ONLY three options are chosen.
dacron. Upon ozonolysis, P gives an aliphatic ketone as one of Partial Marks : +2 If three or more options are correct but
the products. P undergoes the following reaction sequences to ONLY two options are chosen, both of which are correct
furnish R via Q. The compound P also undergoes another set of options.
reactions to produce S.
JEE Advanced 2018 Solved Paper 7 2018 -
Partial Marks : +1 If two or more options are correct but (D) If P3 is the plane passing through the point (4, 2, – 2)
ONLY one option is chosen and it is a correct option. and perpendicular to the line of intersection of P1 and
Zero Marks : 0 If none of the options is chosen (i.e. the P2, then the distance of the point (2, 1, 1) from the
question is unanswered). 2
Negative Marks : –2 In all other cases. plane P3 is
3
• For Example: If first, third and fourth are the ONLY three
correct options for a question with second option being an 40. For every twice differentiable function f : ¡ ® [ -2, 2] with
incorrect option; selecting only all the three correct options
( f (0))2 + ( f ¢(0)) 2 = 85 , which of the following statement(s)
will result in +4 marks. Selecting only two of the three correct
options (e.g. the first and fourth options), without selecting is (are) TRUE?
any incorrect option (second option in this case), will result (A) There exist r , s Î ¡ , where r < s, such that f is one-one
in +2 marks. Selecting only one of the three correct options on the open interval (r, s)
(either first or third or fourth option), without selecting any (B) There exists x0 Î (-4,0) such that | f ¢( x0 ) | £ 1
incorrect option (second option in this case), will result in +1
marks. Selecting any incorrect option(s) (second option in (C) lim f ( x) = 1
x ®¥
this case), with or without selection of any correct option(s)
will result in –2 marks. (D) There exists a Î (-4, 4) such that f (a ) + f ¢¢( a) = 0
and f ¢(a ) ¹ 0
37. For a non-zero complex number z, let arg(z) denote the
principal argument with -p < arg( z ) £ p . Then, which of 41. Let f : ¡ ® ¡ and g : ¡ ® ¡ be two non-constant
the following statement (s) is (are) FALSE? differentiable functions. If

p f ¢( x ) = (e ( f ( x)- g ( x)) ) g ¢( x ) for all x Î ¡ ,


(A) arg(-1 - i) = , where i = -1
4 and f (1) = g (2) = 1 , then which of the following
(B) The function f : ¡ ® (-p, p] , defined by statement (s) is (are) TRUE ?
f (t ) = arg(-1 + it ) for all t Î ¡ , is continuous at all (A) f (2) < 1 - loge 2 (B) f (2) > 1 - log e 2
points of ¡ , where i = -1 (C) g (1) > 1 - loge 2 (D) g (1) < 1 - log e 2
(C) For any two non-zero complex numbers z1 and z2, 42. Let f :[0, ¥) ® ¡ be a continuous function such that
æz ö x
ò0 e
x -t
arg ç 1 ÷ - arg( z1 ) + arg( z2 ) f ( x) = 1 - 2 x + f (t ) dt
è z2 ø
is an integer multiple of 2p for all x Î[0, ¥) . Then, which of the following statement (s)
(D) For any three given distinct complex numbers z1, z2 is (are) TRUE?
and z3, the locus of the point z satisfying the condition (A) The curve y = f ( x ) passes through the point (1, 2)
æ ( z - z1 )( z2 - z3 ) ö (B) The curve y = f ( x ) passes through the point (2, –1)
arg ç ÷ = p , lies on a straight line
è (z - z3 )(z 2 - z1 ) ø (C) The area of the region
38. In a triangle PQR, let ÐPQR = 30° and the sides PQ and QR
have lengths 10 3 and 10, respectively. Then, which of
{(x, y) Î[0,1]´ ¡ : f ( x) £ y £ }
1 - x 2 is
p-2
4
the following statement(s) is (are) TRUE? (D) The area of the region
(A) ÐQPR = 45°
(B) The area of the triangle PQR is 25 3 and ÐQRP = 120°
{(x, y)}Î[0,1] ´ ¡ : f ( x) £ y £ }
1 - x 2 is
p -1
4
(C) The radius of the incircle of the triangle PQR is
SECTION - II (MAXIMUM MARKS: 24)
10 3 - 15 • This section contains EIGHT (08) questions. The answer
(D) The area of the circumcircle of the triangle PQR is to each question is a NUMERICAL VALUE.
100 p. • For each question, enter the correct numerical value (in
39. Let P1 : 2 x + y - z = 3 and P2 : x + 2 y + z = 2 be two planes. decimal notation, truncated/rounded-off to the second
Then, which of the following statement(s) is (are) TRUE? decimal place; e.g. 6.25, 7.00, –0.33, –.30, 30.27, –127.30)
(A) The line of intersection of P1 and P2 has direction using the mouse and the on-screen virtual numeric keypad
ratios 1, 2, – 1 in the place designated to enter the answer.
• Answer to each question will be evaluated according to
3x - 4 1 - 3 y z the following marking scheme:
(B) The line = =
9 9 3 Full Marks : +3 If ONLY the correct numerical value is
is perpendicular to the line of intersection of P1 and entered as answer.
P2 Zero Marks : 0 In all other cases.
(C) The acute angle between P1 and P2 is 60°.
EBD_7809
2018 - 8 JEE Advanced 2018 Solved Paper
1 1 • For each question, choose the option corresponding to
the correct answer.
43. The value of ((log 2 9) 2 ) log 2 (log2 9)
´ ( 7) log 4 7
• Answer to each question will be evaluated according to
is __________. the following marking scheme:
44. The number of 5 digit numbers which are divisible by 4, with Full Marks : +3 If ONLY the correct option is chosen.
digits from the set {1, 2, 3, 4, 5} and the repetition of digits is Zero Marks : 0 If none of the options is chosen (i.e. the
allowed, is _________. question is unanswered).
45. Let X be the set consisting of the first 2018 terms of the Negative Marks : –1 In all other cases.
arithmetic progression 1, 6, 11, ..., and Y be the set consisting
of the first 2018 terms of the arithmetic progression 9, 16, 23, PARAGRAPH - X
.... . Then, the number of elements in the set X È Y is ____.
Let S be the circle in the xy-plane defined by the equation
46. The number of real solutions of the equation
x2 + y2 = 4.
æ ¥ ¥
æxö

p æ ¥ i ¥ ö
÷ = - cos -1 ç æç - ö÷ -
x
sin -1 ç
ç åx i +1
å
-x ç ÷
2 ÷ 2 ç è 2ø å ( i
-x) ÷
÷ å 51. Let E1E2 and F1F2 be the chords of S passing through the
è i =1 i =1 è ø ø è i =1 i =1 ø point P0(1, 1) and parallel to the x-axis and the y-axis,
respectively. Let G1G2 be the chord of S passing through
æ 1 1ö P0 and having slope –1. Let the tangents to S at E1 and E2
lying in the interval ç - , ÷ is ______.
è 2 2ø meet at E3, the tangents to S at F1 and F2 meet at F3, and
(Here, the inverse trigonometric functions sin –1 x and the tangents to S at G1 and G2 meet at G3. Then, the points
p p E3, F3, and G3 lie on the curve
cos–1x assume values in é - , ù and [0, p], respectively.) (A) x + y = 4
êë 2 2 úû
(B) (x – 4)2 + (y – 4)2 = 16
47. For each positive integer n, let
(C) (x – 4)(y – 4) = 4
1 (D) xy = 4
1
yn = (n + 1)(n + 2)...(n + n) n
n 52. Let P be a point on the circle S with both coordinates being
For x Î ¡ , let [x] be the greatest integer less than or equal positive. Let the tangent to S at P intersect the coordinate
to x. If lim yn = L , then the value of [L] is ______. axes at the points M and N. Then, the mid-point of the line
n ®¥ segment MN must lie on the curve
r r r r (A) ( x + y )2 = 3xy
2/3
(B) x + y = 2
2/3 4/3
48. Let a and b be two unit vectors such that a × b = 0 . For
r r r r r r
some x, y Î ¡ , let c = xa + yb + (a ´ b ) . If | c |= 2 and the (C) x 2 + y 2 = 2 xy
2 2 2 2
(D) x + y = x y
r r r
vector c is inclined at the same angle a to both a and b , PARAGRAPH - A
then the value of 8 cos2 a is ________.
49. Let a, b, c be three non-zero real numbers such that the There are five students S1, S2, S3, S4 and S5 in a music class and
for them there are five seats R1, R2, R3, R4 and R5 arranged in a
equation : é p pù
3a cos x + 2b sin x = c, x Î ê - , ú , has two row, where initially the seat Ri is allotted to the student Si, i = 1, 2,
ë 2 2û 3, 4, 5. But, on the examination day, the five students are randomly
p allotted the five seats.
distinct real roots a and b with a + b = . Then, the value
3 53. The probability that, on examination day, the student S1
b gets the previously allotted seat R1, and NONE of the
of is _______.
a remaining students gets the seat previously allotted to him/
50. A farmer F1 has a land in the shape of a triangle with vertices her is
at P(0, 0), Q(1, 1) and R(2, 0). From this land, a neighbouring
farmer F2 takes away the region which lies between the side 3 1 7 1
(A) (B) (C) (D)
40 8 40 5
PQ and a curve of the form y = x n (n > 1) . If the area of the
region taken away by the farmer F2 is exactly 30% of the 54. For i = 1, 2, 3, 4, let Ti denote the event that the students Si
area of DPQR, then the value of n is ______. and Si + 1 do NOT sit adjacent to each other on the day of
the examination. Then, the probability of the event
SECTION - III (MAXIMUM MARKS: 12)
T1 Ç T2 Ç T3 Ç T4 is
• This section contains TWO (02) paragraphs. Based on
each paragraph, there are TWO (02) questions. 1 1 7 1
• Each question has FOUR options. ONLY ONE of these (A) (B) (C) (D)
15 10 60 5
four options corresponds to the correct answer.
JEE Advanced 2018 Solved Paper 2018 - 9

PAPER - 2
PHYSICS (C) The tangential (shear) stress on the floor of the tank
increases with m0
(D) The tangential (shear) stress on the plate varies linearly
SECTION - I (MAXIMUM MARKS: 24) with the viscosity h of the liquid
• This section contains SIX (06) questions. 3. An infinitely long thin non-conducting wire is parallel to
• Each question has FOUR options for correct answer(s). ONE the z-axis and carries a uniform line charge density l. It
OR MORE THAN ONE of these four option(s) is (are) correct pierces a thin non-conducting spherical shell of radius R
option(s). in such a way that the arc PQ subtends an angle 120° at the
• For each question, choose the correct option(s) to answer centre O of the spherical shell, as shown in the figure. The
the question. permittivity of free space is Î0. Which of the following
• Answer to each question will be evaluated according to the statements is (are) true?
following marking scheme: l Z
Full Marks : +4 If only (all) the correct option(s) is (are)
chosen. P
Partial Marks : +3 If all the four options are correct but R
ONLY three options are chosen. o O
Partial Marks : +2 If three or more options are correct but 120
ONLY two options are chosen, both of which are correct
Q
options.
Partial Marks : +1 If two or more options are correct but
ONLY one option is chosen and it is a correct option.
Zero Marks : 0 If none of the options is chosen (i.e. the
(A) The electric flux through the shell is 3Rl /Î0
question is unanswered). (B) The z-component of the electric field is zero at all the
Negative Marks : –2 In all other cases. points on the surface of the shell
• For Example: If first, third and fourth are the ONLY three (C) The electric flux through the shell is 2Rl /Î0
correct options for a question with second option being an
incorrect option; selecting only all the three correct options (D) The electric field is normal to the surface of the shell at
will result in +4 marks. Selecting only two of the three correct all points
options (e.g. the first and fourth options), without selecting 4. A wire is bent in the shape of a right angled triangle and is
any incorrect option (second option in this case), will result placed in front of a concave mirror of focal length f, as
in +2 marks. Selecting only one of the three correct options shown in the figure. Which of the figures shown in the four
(either first or third or fourth option), without selecting any options qualitatively represent(s) the shape of the image
incorrect option (second option in this case), will result in +1 of the bent wire? (These figures are not to scale.)
marks. Selecting any incorrect option(s) (second option in
this case), with or without selection of any correct option(s)
will result in –2 marks. o
45
1. A particle of mass m is initially at rest at the origin. It is f f
subjected to a force and starts moving along the x-axis. Its 2
kinetic energy K changes with time as dK/dt = gt, where g is
a positive constant of appropriate dimensions. Which of
the following statements is (are) true? ¥
(A) The force applied on the particle is constant
(B) The speed of the particle is proportional to time (A) a > 45o (B)
a
(C) The distance of the particle from the origin increases
linearly with time
(D) The force is conservative ¥
2. Consider a thin square plate floating on a viscous liquid in 0< a <45 (D)
o
(C) a
a large tank. The height h of the liquid in the tank is much
less than the width of the tank. The floating plate is pulled 232
5. In a radioactive decay chain, 90Th nucleus decays to
horizontally with a constant velocity m0. Which of the
following statements is (are) true? 212
82 Pb nucleus. Let Na and Nb be the number of a and
(A) The resistive force of liquid on the plate is inversely -
b particles, respectively, emitted in this decay process.
proportional to h Which of the following statements is (are) true?
(B) The resistive force of liquid on the plate is independent (A) Na = 5 (B) Na = 6 (C) Nb = 2 (D) Nb = 4
of the area of the plate
EBD_7809
2018 - 10 JEE Advanced 2018 Solved Paper
6. In an experiment to measure the speed of sound by a 11. A steel wire of diameter 0.5 mm and Young’s modulus
resonating air column, a tuning fork of frequency 500 Hz is 2 × 1011 N m–2 carries a load of mass M. The length of the
used. The length of the air column is varied by changing wire with the load is 1.0 m. A vernier scale with 10 divisions
the level of water in the resonance tube. Two successive is attached to the end of this wire. Next to the steel wire is
resonances are heard at air columns of length 50.7 cm and a reference wire to which a main scale, of least count 1.0
83.9 cm. Which of the following statements is (are) true? mm, is attached. The 10 divisions of the vernier scale
(A) The speed of sound determined from this experiment correspond to 9 divisions of the main scale. Initially, the
is 332 ms–1 zero of vernier scale coincides with the zero of main scale.
If the load on the steel wire is increased by 1.2 kg, the
(B) The end correction in this experiment is 0.9 cm
vernier scale division which coincides with a main scale
(C) The wavelength of the sound wave is 66.4 cm
division is __________. Take g =10 m s–2 and p = 3.2.
(D) The resonance at 50.7 cm corresponds to the 12. One mole of a monatomic ideal gas undergoes an adiabatic
fundamental harmonic expansion in which its volume becomes eight times its initial
value. If the initial temperature of the gas is 100 K and the
SECTION - II (MAXIMUM MARKS: 24)
universal gas constant R = 8.0 J mol –1 K –1, the decrease in
• This section contains EIGHT (08) questions. The answer its internal energy, in Joule, is__________.
to each question is a NUMERICAL VALUE. 13. In a photoelectric experiment a parallel beam of
• For each question, enter the correct numerical value (in monochromatic light with power of 200 W is incident on a
decimal notation, truncated/rounded-off to the second perfectly absorbing cathode of work function 6.25 eV. The
decimal place; e.g. 6.25, 7.00, –0.33, –.30, 30.27, –127.30) frequency of light is just above the threshold frequency so
using the mouse and the on-screen virtual numeric keypad that the photoelectrons are emitted with negligible kinetic
in the place designated to enter the answer. energy. Assume that the photoelectron emission efficiency
• Answer to each question will be evaluated according to is 100%. A potential difference of 500 V is applied between
the following marking scheme: the cathode and the anode. All the emitted electrons are
Full Marks : +3 If ONLY the correct numerical value is incident normally on the anode and are absorbed. The anode
entered as answer. experiences a force F = n × 10–4 N due to the impact of the
Zero Marks : 0 In all other cases. electrons. The value of n is __________. Mass of the
electron me = 9 ×10–31 kg and 1.0 eV = 1.6 ×10–19 J.
7. A solid horizontal surface is covered with a thin layer of oil. 14. Consider a hydrogen-like ionized atom with atomic number
A rectangular block of mass m = 0.4 kg is at rest on this Z with a single electron. In the emission spectrum of this
surface. An impulse of 1.0 Ns is applied to the block at time atom, the photon emitted in the n = 2 to n = 1 transition has
t = 0 so that it starts moving along the x -axis with a velocity energy 74.8 eV higher than the photon emitted in the n = 3
n(t) = n0e–t/t, where n0 is a constant and t = 4s. The to n = 2 transition. The ionization energy of the hydrogen
displacement of the block, in metres, at t = t is __________. atom is 13.6 eV. The value of Z is __________.
Take e –1 = 0.37.
8. A ball is projected from the ground at an angle of 45o with SECTION - III (MAXIMUM MARKS: 12)
the horizontal surface. It reaches a maximum height of 120 m • This section contains FOUR (04) questions.
and returns to the ground. Upon hitting the ground for the • Each question has TWO (02) matching lists: LIST-I and
first time, it loses half of its kinetic energy. Immediately LIST-II.
after the bounce, the velocity of the ball makes an angle of • FOUR options are given representing matching of elements
30o with the horizontal surface. The maximum height it from LIST-I and LIST-II. ONLY ONE of these four options
reaches after the bounce, in metres, is ___________. corresponds to a correct matching.
9. A particle, of mass 10–3 kg and charge 1.0 C, is initially at rest.
• For each question, choose the option corresponding to
At time t = 0,rthe particle comes under the influence of an
the correct matching.
electric field E (t) = E 0 sin w tiˆ, where E0 = 1.0 NC–1 and
• For each question, marks will be awarded according to the
w =103 rad s–1. Consider the effect of only the electrical force
following marking scheme:
on the particle. Then the maximum speed, in ms–1, attained by
Full Marks : +3 If ONLY the option corresponding to the
the particle at subsequent times is ____________.
correct matching chosen.
10. A moving coil galvanometer has 50 turns and each turn has
Zero Marks : 0 If none of the options is chosen (i.e. the
an area 2 ×10–4 m2. The magnetic field produced by the
question is unanswered).
magnet inside the galvanometer is 0.02 T. The torsional
constant of the suspension wire is 10–4 N m rad–1. When a Negative Marks : –1 In all other cases.
current flows through the galvanometer, a full scale 15. The electric field E is measured at a point P (0, 0, d) generated
deflection occurs if the coil rotates by 0.2 rad. The resistance due to various charge distributions and the dependence of E
of the coil of the galvanometer is 50 W. This galvanometer on d is found to be different for different charge distributions.
is to be converted into an ammeter capable of measuring List-I contains different relations between E and d. List-II
current in the range 0 –1.0 A. For this purpose, a shunt
describes different electric charge distributions, along with
resistance is to be added in parallel to the galvanometer.
their locations. Match the functions in List-I with the related
The value of this shunt resistance, in ohms, is __________.
charge distributions in List-II.
JEE Advanced 2018 Solved Paper 2018 - 11
LIST–I LIST–II LIST–I LIST–II
P. E is independent of d 1. A point charge Q at the origin P. In process I 1. Work done by the gas is zero
Q. E µ 1/d 2. A small dipole with point Q. In process II 2. Temperature of the gas
charges Q at (0, 0, l) and – Q remains unchanged
at (0, 0, –l). Take 2l << d R. In process III 3. No heat is exchanged between
R. E µ 1/d2 3. An infinite line charge the gas and its surroundings
coincident with the x-axis, S. In process IV 4. Work done by the gas is 6 P0V0
with uniform linear charge (A) P ® 4; Q ® 3; R ® 1; S ® 2
density l
(B) P ® 1; Q ® 3; R ® 2; S ® 4
S. E µ 1/d3 4. Two infinite wires carrying
uniform linear charge density (C) P ® 3; Q ® 4; R ® 1; S ® 2
parallel to the x - axis. The one (D) P ® 3; Q ® 4; R ® 2; S ® 1
along (y = 0, z = l) has a charge 18. In the List-I below, four different paths of a particle are
density +l and the one along given as functions of time. In these functions, a and b are
(y = 0, z = –l) has a charge positive constants of appropriate dimensions and a ¹ b In
density – lTake 2l << d each case, the force acting on the particle is either zero or
5. Infinite plane charge conservative. In List-II, five physical quantities of the
coincident with the xy-plane r r
with uniform surface charge particle are mentioned p is the linear momentum, L is the
density angular momentum about the origin, K is the kinetic energy,
(A) P ® 5; Q ® 3, 4; R ® 1; S ®2 U is the potential energy and E is the total energy. Match
(B) P ® 5; Q ® 3; R® 1, 4; S ® 2 each path in List-I with those quantities in List-II, which
(C) P® 5; Q ® 3; R ® 1, 2; S ® 4 are conserved for that path.
(D) P ® 4; Q ® 2, 3; R ® 1; S ®5 LIST–I LIST–II
r r
16. A planet of mass M, has two natural satellites with masses P. r(t) = a t iˆ+ b t ˆj 1. p
m1 and m2. The radii of their circular orbits are R1 and R2 r r
respectively, Ignore the gravitational force between the Q. r(t) = a cos wt iˆ+ b sin wt ˆj 2. L
satellites. Define v1, L1, K1 and T1to be, respectively, the r
orbital speed, angular momentum, kinetic energy and time R. r(t) = a(cos wt iˆ+ sin wt ˆj) 3. K
period of revolution of satellite 1; and v2, L2, K2, and T2 to be r b 2
the corresponding quantities of satellite 2. Given m1/m2 = 2 S. r(t) = at iˆ+ t ˆj 4. U
2
and R1/R2 = 1/4, match the ratios in List-I to the numbers in
5. E
List-II.
(A) P ® 1, 2, 3, 4, 5; Q ® 2, 5; R ® 2, 3, 4, 5; S ® 5
LIST–I LIST–II
P. v1 /v 2 1. 1/8 (B) P ® 1, 2, 3, 4, 5; Q ® 3, 5; R ® 2, 3, 4, 5; S ® 2, 5
Q. L1/L2 2. 1 (C) P ® 2, 3, 4; Q ® 5; R ® 1, 2, 4; S ® 2, 5
R. K1/K2 3. 2 (D) P ® 1, 2, 3, 5; Q ® 2, 5; R ® 2, 3, 4, 5; S ® 2, 5
S. T1/T 2 4. 8
(A) P ® 4; Q ® 2; R ® 1; S ® 3 CHEMISTRY
(B) P ® 3; Q ® 2; R ® 4; S ®1
(C) P ® 2; Q ® 3; R ® 1; S ® 4
SECTION - I (MAXIMUM MARKS: 24)
(D) P ® 2; Q ® 3; R ® 4; S ® 1
17. One mole of a monatomic ideal gas undergoes four • This section contains SIX (06) questions.
thermodynamic processes as shown schematically in the • Each question has FOUR options for correct answer(s). ONE
PV-diagram below. Among these four processes, one is OR MORE THAN ONE of these four option(s) is (are) correct
isobaric, one is isochoric, one is isothermal and one is option(s).
adiabatic. Match the processes mentioned in List-I with • For each question, choose the correct option(s) to answer
the corresponding statements in List-II. the question.
• Answer to each question will be evaluated according to the
P following marking scheme:
Full Marks : +4 If only (all) the correct option(s) is (are)
II chosen.
3P0
Partial Marks : +3 If all the four options are correct but
IV III ONLY three options are chosen.
Partial Marks : +2 If three or more options are correct but
I ONLY two options are chosen, both of which are correct
P0
options.
V Partial Marks : +1 If two or more options are correct but
V0 3V0 ONLY one option is chosen and it is a correct option.
EBD_7809
2018 - 12 JEE Advanced 2018 Solved Paper
Zero Marks : 0 If none of the options is chosen (i.e. the 22. The Fischer presentation of D-glucose is given below.
question is unanswered). CHO
Negative Marks : –2 In all other cases. H OH
• For Example: If first, third and fourth are the ONLY three HO H
H OH
correct options for a question with second option being an H OH
incorrect option; selecting only all the three correct options
will result in +4 marks. Selecting only two of the three correct CH2OH
options (e.g. the first and fourth options), without selecting D-glucose
any incorrect option (second option in this case), will result The correct structure(s) of b-L-glucopyranose is (are)
in +2 marks. Selecting only one of the three correct options
H H
(either first or third or fourth option), without selecting any O
incorrect option (second option in this case), will result in +1 HO CH OH OH O
2 HO CH OH OH
marks. Selecting any incorrect option(s) (second option in 2
(A) H H HO H (B)
this case), with or without selection of any correct option(s) H H H H
will result in –2 marks. OH H OH OH
19. The correct option(s) regarding th e complex CH2OH
H
[Co(en)(NH3)3(H2O)]3+ (en = H2NCH2CH2NH2) is (are) O O
H OH HO
(A) It has two geometrical isomers H CH2OH H
(B) It will have three geometrical isomers if bidentate ‘en’ (C) HO OH HO H (D)
H H HO OH
is replaced by two cyanide ligands H H
(C) It is paramagnetic OH H
23 . For a first order reaction A(g) ® 2B(g) + C(g) at constant
(D) It absorbs light at longer wavelength as compared to volume and 300 K, the total pressure at the beginning (t = 0)
[Co(en)(NH3)4]3+ and at time t are P0 and Pt, respectively. Initially, only A is
20. The correct option(s) to distinguish nitrate salts of Mn 2+ present with concentration [A]0, and t1/3 is the time required
and Cu2+ taken separately is (are) for the partial pressure of A to reach 1/3rd of its initial value.
The correct option(s) is (are) (Assume that all these gases
(A) Mn2+ shows the characteristic green colour in the flame behave as ideal gases)
test ­ ­
(B) Only Cu2+ shows the formation of precipitate by
passing H2S in acidic medium
ln(3P0 –Pt)

(C) Only Mn2+ shows the formation of precipitate by

t1/3
passing H2S in faintly basic medium (A) (B)
(D) Cu 2+ /Cu has higher reduction potential than
® ®
Mn2+/Mn (measured under similar conditions) Time [A]0
21. Aniline reacts with mixed acid (conc. HNO3 and conc. H2SO4) ­ ­
at 288 K to give P (51 %),Q (47%) and R (2%). The major
Rate constant

product(s) of the following reaction sequence is (are)


ln(P0 –Pt)

1)Ac2 O,pyridine 1)Sn / HCl


2)Br ,CH CO H 2)Br / H O(excess) (C) (D)
R ¾¾¾¾¾¾¾¾¾¾
2
+
3 2 ® S ¾¾¾¾¾¾¾¾¾¾
2 2 ®
3)H3O 3)NaNO 2 ,HCl / 273- 278 K
4)NaNO2 ,HCl / 273- 278 K 4)H3PO2 ® ®
Time [A]0
5)EtOH, D
major product(s) 24. For a reaction, A ƒ P, the plots of [A] and [P] with time
at temperatures T1 and T2 are given below.
Br Br
­ 10
­
[A]/(mol L–1)

Br
[A]/(mol L–1 )

Br 10
T1
(A) (B) 5 T2
5
Br Br T2
T1 ®
Br Br ®
Time Time

Br If T 2 > T 1 , the correct statement(s) is (are)


Br
(Assume DH° and DS° are independent of temperature and
(C) (D) T2
Br Br ratio of lnK at T 1 to lnK at T2 is greater than . Here H,
Br Br T1
Br S,G and K are enthalpy, entropy, Gibbs energy and
equilibrium constant, respectively.)
JEE Advanced 2018 Solved Paper 2018 - 13
(A) DH° < 0, DS° < 0 (B) DG° < 0, DH° > 0 O
(C) DG° < 0, DS° < 0 (D) DG° < 0, DS° > 0
. NaOBr NH ,D
¾¾¾¾ ® A ¾¾¾¾
3 ® B
SECTION - II (MAXIMUM MARKS: 24) +
H3O (60%) (50%)
• This section contains EIGHT (08) questions. The answer
to each question is a NUMERICAL VALUE. Br /KOH Br (3 equiv)
¾¾¾¾¾
2 ® C ¾¾¾¾¾¾
2 ® D
• For each question, enter the correct numerical value (in (50%) AcOH (100%)
decimal notation, truncated/rounded-off to the second 30. The surface of copper gets tarnished by the formation of
decimal place; e.g. 6.25, 7.00, –0.33, –.30, 30.27, –127.30) copper oxide. N2 gas was passed to prevent the oxide
using the mouse and the on-screen virtual numeric keypad formation during heating of copper at 1250 K. However, the
in the place designated to enter the answer. N2 gas contains 1 mole % of water vapour as impurity. The
• Answer to each question will be evaluated according to water vapour oxidises copper as per the reaction given
the following marking scheme: below: 2Cu(s) + H2O(g) ® Cu2O(s) + H2(g)
Full Marks : +3 If ONLY the correct numerical value is pH is the minimum partial pressure of H2 (in bar) needed to
2
entered as answer. prevent the oxidation at 1250 K. The value of ln(pH )
2
is_____.
Zero Marks : 0 In all other cases.
(Given: total pressure = 1 bar, R (universal gas constant) =
25. The total number of compounds having at least one 8 J K–1 mol–1, ln (10) = 2.3. Cu (s) and Cu2O (s) are mutually
bridging oxo group among the molecules given below immiscible.
is______. N2O3, N2O5, P4O6, P4O7, H4P2O5, H5P3O10, At 1250 K : 2 Cu(s) + ½O2(g) ® Cu2O(s) ;
H2S2O3, H2S2O5 DG° = –78,000 J mol–1
26. Galena (an ore) is partially oxidized by passing air through H2(g) + ½O2(g) ® H2O(g); DG° = –1,78,000J mol–1 ;
it at high temperature. After some time, the passage of air is (G is the Gibbs energy)
stopped, but the heating is continued in a closed furnace 31. Consider the following reversible reaction,
such that the contents undergo self-reduction. The weight A(g) + B(g)®AB(g).
(in kg) of Pb produced per kg of O2 consumed is______ . The activation energy of the backward reaction exceeds
(Atomic weights in g mol–1: O = 16, S = 32, Pb = 207) that of the forward reaction by 2RT (in J mol –1). If the
27. To measure the quantity of MnCl2 dissolved in an aqueous pre-exponential factor of the forward reaction is 4 times
solution, it was completely converted to KMnO4 using the that of the reverse reaction, the absolute value of DG°
reaction, (in J mol–1) for the reaction at 300 K is_____.
MnCl2 + K2S2O8 + H2O® KMnO4 + H2SO4 + HCl (Given; ln(2) = 0.7, RT = 2500 J mol–1 at 300 K and G is the
(equation not balanced). Gibbs energy)
Few drops of concentrated HCl were added to this solution 32. Consider an electrochemical cell:
and gently warmed. Further, oxalic acid (225 mg) was added A(s) | An+ (aq, 2 M) |B2n+ (aq, 1 M) | B(s).
in portions till the colour of the permanganate ion The value of DH° for the cell reaction is twice that of DG° at
disappeared. The quantity of MnCl2 (in mg) present in the 300 K. If the emf of the cell is zero, the DS° (in J K–1 mol–1)
initial solution is ______. of the cell reaction per mole of B formed at 300 K is______.
(Atomic weights in g mol–1: Mn = 55, Cl = 35.5) (Given: ln(2) = 0.7, R (universal gas constant)
28. For the given compound X, the total number of optically = 8.3 J K–1 mol–1. H, S and G are enthalpy, entropy and
active stereoisomers is ______. Gibbs energy, respectively.)

This type of bond indicates SECTION - III (MAXIMUM MARKS: 12)


that the configuration at the • This section contains FOUR (04) questions.
HO HO specific carbon and the • Each question has TWO (02) matching lists: LIST-I and
geometry of the double bond is fixed
LIST-II.
• FOUR options are given representing matching of elements
This type of bond indicates from LIST-I and LIST-II. ONLY ONE of these four options
that the configuration at the
HO HO corresponds to a correct matching.
specific carbon and the geometry
of the double bond is NOT fixed • For each question, choose the option corresponding to
X the correct matching.
• For each question, marks will be awarded according to the
29. In the following reaction sequence, the amount of D (in g) following marking scheme:
Full Marks : +3 If ONLY the option corresponding to the
formed from 10 moles of acetophenone is ______.
correct matching chosen.
(Atomic weights in g mol–1: H = 1, C = 12, N = 14, O = 16, Zero Marks : 0 If none of the options is chosen (i.e. the
Br = 80. The yield (%) corresponding to the product in each question is unanswered).
Negative Marks : –1 In all other cases.
step is given in the parenthesis)
EBD_7809
2018 - 14 JEE Advanced 2018 Solved Paper
33. Match each set of hybrid orbitals from LIST-I with Ph
Br H
complex(es) given in LIST-II
S. Ph OH + AgNO3 4. HCHO, NaOH
List - I List- II
Me
P. dsp 2 1. [FeF6]4– 5. NaOBr
Q. sp3 2. [Ti(H2O)3Cl3] The correct option is
R. sp3d 2 3. [Cr(NH3)6]3+
(A) P-1; Q-2, 3 ; R-1, 4; S-2, 4
S. d2sp 3 4. [FeCl4]2–
5. Ni(CO)4 (B) P-1, 5 ; Q-3, 4 ; R-4, 5; S-3
6. [Ni(CN)4]2– (C) P-1, 5; Q-3, 4 ; R-5; S-2, 4
The correct option is (D) P-1, 5; Q-2, 3 ; R-1, 5; S-2, 3
(A) P-5; Q-4, 6 ; R-2, 3; S-1 35. LIST-I contains reactions and LIST-II contains major
(B) P-5, 6; Q-4; R- 3; S-1, 2 products.
(C) P-6; Q-4, 5 ; R-1; S-2, 3
(D) P-4, 6; Q-5, 6 ; R-1, 2; S-3 List - I List- II
34. The desired product X can be prepared by reacting the
major product of the reactions in LIST-I with one or more P. + ¾® 1.
appropriate reagents in LIST-II. (given, order of migratory ONa Br OH
aptitude: aryl > alkyl > hydrogen)
O Q. + HBr ¾® 2. Br
OMe
Ph
OH R. + NaOMe ¾® 3.
Me Ph Br OMe
X
List - I List- II S. + MeBr ¾® 4.
Ph ONa
HO Me O
5.
P. Ph OH + H2SO4 1. I2, NaOH
Me Match the reaction in LIST-I with one or more products in
Ph LIST-II and choose the correct option.
H2N H
Q. Ph OH + HNO2 2. [Ag(NH3)2]OH (A) P-1, 5; Q-2 ; R-3; S-4
Me
Ph (B) P-1, 4 ; Q-2 ; R-4; S-3
HO Ph
R. (C) P-1, 4; Q-1, 2 ; R-3, 4; S-4
Me OH + H2SO4 3. Fehling solution
Me (D) P-4, 5; Q-4 ; R-4 ; S-3, 4
36 . Dilution processes of different aqueous solutions, with water, are given in LIST-I. The effects of dilution of the solutions on [H +]
are given in LIST-II.
(Note: Degree of dissociation (a) of weak acid and weak base is << 1; degree of hydrolysis of salt <<1; [H+] represents the
concentration of H+ ions)
LIST-I LIST-II
P. (10 mL of 0.1 M NaOH + 20 mL of 0.1 M acetic acid) 1. the value of [H+] does not change on dilution
diluted to 60 mL
Q. (20 mL of 0.1 M NaOH + 20 mL of 0.1 M acetic acid) 2. the value of [H+] changes to half of its initial value on dilution
diluted to 80 mL
R. (20 mL of 0.1 M HCl + 20 mL of 0.1 M ammonia solution) 3. the value of [H+] changes to two times of its initial
diluted to 80 mL value on dilution
1
S. 10 mL saturated solution of Ni(OH)2 in equilibrium 4. the value of [H+] changes to times of its initial value
2
with excess solid Ni(OH)2 is diluted to 20 mL on dilution
(solid Ni(OH)2 is still present after dilution). 5. the value of [H+] changes to 2 times of its initial value on
dilution
JEE Advanced 2018 Solved Paper 2018 - 15
MATHEMATICS such that S1 and S2 touch each other at a point, say, M. Let
E1 be the set representing the locus of M as the pair (S1, S2)
varies in F1. Let the set of all straight line segments joining
SECTION - I (MAXIMUM MARKS: 24) a pair of distinct points of E1 and passing through the point
• This section contains SIX (06) questions. R(1, 1) be F2. Let E2 be the set of the mid-points of the line
• Each question has FOUR options for correct answer(s). ONE segments in the set F2. Then, which of the following
OR MORE THAN ONE of these four option(s) is (are) correct statement(s) is (are) TRUE?
option(s). (A) The point (–2, 7) lies in E1
• For each question, choose the correct option(s) to answer
the question. æ4 7ö
(B) The point ç , ÷ does NOT lie in E2
• Answer to each question will be evaluated according to the è5 5ø
following marking scheme:
æ1 ö
Full Marks : +4 If only (all) the correct option(s) is (are) (C) The point ç ,1÷ lies in E2
chosen. è2 ø
Partial Marks : +3 If all the four options are correct but
æ 3ö
ONLY three options are chosen. (D) The point ç 0, ÷ does NOT lie in E1
Partial Marks : +2 If three or more options are correct but è 2ø
ONLY two options are chosen, both of which are correct
options. éb1 ù
ê ú
Partial Marks : +1 If two or more options are correct but 39. Let S be the set of all column matrices êb2 ú such that b1,
ONLY one option is chosen and it is a correct option. êëb3 úû
Zero Marks : 0 If none of the options is chosen (i.e. the
question is unanswered). b2, b3¸ Î ¡ and the system of equations (in real variables)
Negative Marks : –2 In all other cases. –x + 2y + 5z = b1
• For Example: If first, third and fourth are the ONLY three 2x – 4y + 3z = b2
correct options for a question with second option being an x – 2y + 2z = b3
incorrect option; selecting only all the three correct options has at least one solution. Then, which of the following
will result in +4 marks. Selecting only two of the three correct system(s) (in real variables) has (have) at least one solution
options (e.g. the first and fourth options), without selecting
éb1 ù
any incorrect option (second option in this case), will result ê ú
in +2 marks. Selecting only one of the three correct options for each êb2 ú Î S?
(either first or third or fourth option), without selecting any êëb3 úû
incorrect option (second option in this case), will result in +1
marks. Selecting any incorrect option(s) (second option in (A) x + 2y + 3z = b1, 4y + 5z = b2 and x + 2y + 6z = b3
this case), with or without selection of any correct option(s) (B) x + y + 3z = b1, 5x + 2y + 6z = b2 and –2x – y – 3z = b3
will result in –2 marks. (C) –x + 2y – 5z = b1, 2x – 4y + 10z = b2 and x – 2y + 5z = b3
(D) x + 2y + 5z = b1, 2x + 3z = b2 and x + 4y – 5z = b3
37. For any positive integer n, define fn : (0, ¥) ® ¡ as
40. Consider two straight lines, each of which is tangent to
æ 1 ö
f n (x) = å nj = 1 tan –1 ç
è 1 + ( x + j ) ( x + j -1) ÷ø
for all both the circle x2 + y2 =
1
2
and the parabola y2 = 4x. Let

x Î (0, ¥). these lines intersect at the point Q. Consider the ellipse
whose center is at the origin O (0, 0) and whose semi-major
æ Here, theinverse trigonometricfunction tan -1 x assumes ö axis is OQ. If the length of the minor axis of this ellipse is
ç ÷
ç æ p pö ÷ 2 , then which of the following statement(s) is (are) TRUE?
ç valuesin - ,
ç 2 2÷ . ÷
è è ø ø 1
(A) For the ellipse, the eccentricity is and the length
Then, which of the following statement(s) is (are) TRUE? 2
of the latus rectum is 1
(A) å 5j =1 tan2 ( fj (0)) = 55 (B) For the ellipse, the eccentricity is
1
and the length of
2
(B) å 10j =1 (1 + fj¢ (0)) sec2 ( fj (0)) = 10 1
the latus rectum is
1 2
(C) For any fixed positive integer n, lim tan (fn (x)) = (C) The area of the region bounded by the ellipse between
x ®¥ n
1 1
(D) For any fixed positive integer n, lim sec2 (fn (x)) = 1 the lines x = and x = 1 is ( p - 2)
x ®¥ 2 4 2
38. Let T be the line passing through the points P(–2, 7) and (D) The area of the region bounded by the ellipse between
Q(2, –5). Let F1 be the set of all pairs of circles (S1, S2) such 1 1
that T is tangent to S1 at P and tangent to S2 at Q, and also the lines x = and x = 1 is (p – 2)
2 16
EBD_7809
2018 - 16 JEE Advanced 2018 Solved Paper
41. Let s, t, r be non-zero complex numbers and L be the set dy
= (2 + 5y) (5y – 2),
of solutions z = x + iy ( x, y, Î ¡, i = -1) of the equation dx
sz + t z + r = 0, where z = x – iy. Then, which of the then the value of lim f ( x) is _____.
x®-¥
following statement(s) is (are) TRUE? 47. Let f : ¡ ® ¡ be a differentiable function with f (0) = 1 and
(A) If L has exactly one element, then |s| ¹ |t| satisfying the equation
(B) If |s| = |t|, then L has infinitely many elements f (x + y) = f(x) f ¢ (y) + f ¢ (x) f (y) for all x, y Î ¡
(C) The number of elements in L Ç {z : |z – 1 + i| = 5} is at Then, the value of loge (f (4)) is _____.
most 2 48. Let P be a point in the first octant, whose image Q in the
(D) If L has more than one element, then L has infinitely plane x + y = 3 (that is, the line segment PQ is perpendicular
many elements to the plane x + y = 3 and the mid-point of PQ lies in the
42. Let f : (0, p) ® ¡ be a twice differentiable function such that plane
f ( x) sin t - f (t )sin x x + y = 3) lies on the z-axis. Let the distance of P from the
lim = sin 2 x for all x Î (0, p). x-axis be 5. If R is the image of P in the xy-plane, then the
t ®x t-x
length of PR is _____ .
p p
If f æç ö÷ = - , then which of the following statement(s) 49. Consider the cube in the first octant with sides OP, OQ and
6
è ø 12 OR of length 1, along the x-axis, y-axis and z-axis,
is (are) TRUE?
æ1 1 1ö
respectively, where O (0, 0, 0) is the origin. Let S ç , , ÷
p p
(A) f æç ö÷ = è2 2 2ø
è4ø 4 2 be the centre of the cube and T be the vertex of the cube
x4 opposite to the origin O such that S lies on the diagonal OT.
(B) f ( x) < - x 2 for all x Î (0, p) r uur r uuur r uur r uuur
6 If p = SP , q = SQ , r = SR and t = ST , then the value of
(C) There exists a Î (0, p) such that f ¢ (a) = 0 r r r r
| ( p ´ q ) ´ ( r ´ t ) | is _____ .
æpö æpö
(D) f ¢¢ ç ÷ + f ç ÷ = 0 50. Let
2
è ø è2ø X = (10C1)2 + 2(10C2)2 + 3(10C3)2 + ... + 10(10C10)2,
SECTION - II (MAXIMUM MARKS: 24) where 10Cr, r Î {1, 2, ..., 10} denote binomial coefficients.
• This section contains EIGHT (08) questions. The answer 1
Then, the value of X is _____ .
to each question is a NUMERICAL VALUE. 1430
• For each question, enter the correct numerical value (in
decimal notation, truncated/rounded-off to the second SECTION - III (MAXIMUM MARKS: 12)
decimal place; e.g. 6.25, 7.00, –0.33, –.30, 30.27, –127.30) • This section contains FOUR (04) questions.
using the mouse and the on-screen virtual numeric keypad • Each question has TWO (02) matching lists: LIST-I and
in the place designated to enter the answer. LIST-II.
• Answer to each question will be evaluated according to • FOUR options are given representing matching of elements
the following marking scheme: from LIST-I and LIST-II. ONLY ONE of these four options
Full Marks : +3 If ONLY the correct numerical value is corresponds to a correct matching.
entered as answer. • For each question, choose the option corresponding to
Zero Marks : 0 In all other cases. the correct matching.
• For each question, marks will be awarded according to the
43. The value of the integral following marking scheme:
Full Marks : +3 If ONLY the option corresponding to the
1 correct matching chosen.
2
1+ 3 Zero Marks : 0 If none of the options is chosen (i.e. the
ò (( x +1)2 (1- x)6 ) 1
4
dx is _____ .
question is unanswered).
0 Negative Marks : –1 In all other cases.
44. Let P be a matrix of order 3 × 3 such that all the entries in P
are from the set {–1, 0, 1}. Then, the maximum possible ì x ü
value of the determinant of P is _____ . 51. Let E1 = í x Î ¡ : x ¹ 1 and > 0ý
î x -1 þ
45. Let X be a set with exactly 5 elements and Y be a set with
exactly 7 elements. If a is the number of one-one functions ìï -1 æ æ x öö üï
from X to Y and b is the number of onto functions from Y to and E2 = í x Î E1 :sin ç log e ç ÷ ÷ is a real number ý .
îï è è x -1 øø þï
X, then the value of 1 (b – a) is _____ .
5!
(Here, the inverse trigonometric function sin -1 x
46. Let f : ¡ ® ¡ be a differentiable function with f (0) = 0. If
y = f(x) satisfies the differential equation é p p ùö
assumes valuesin ê - , ú ÷ .
ë 2 2 ûø
JEE Advanced 2018 Solved Paper 2018 - 17
Let f : E 1 ® ¡ be the function defined by
x2 y2
æ x ö 53. Let H : - = 1 , where a > b > 0, be a hyperbola in the
f (x) = loge ç ÷ and g : E2 ® ¡ be the function defined a 2 b2
è x -1 ø
xy-plane whose conjugate axis LM subtends an angle of 60°
æ æ x öö at one of its vertices N. Let the area of the triangle LMN be
by g(x) = sin–1 ç log e ç ÷÷ .
è è x -1 ø ø
4 3.
LIST - I LIST - II LIST - I LIST - II
P. The length of the conjugate axis of H is 1. 8
æ 1 ù é e ö
P. The range of f is 1. ç - ¥, úÈê , ¥÷ 4
è 1 - e û ë e - 1 ø Q. The eccentricity of H is 2.
3
Q. The range of g contains 2. (0, 1)
2
é 1 1ù R. The distance between the foci of H is 3.
R. The domain of f contains 3. ê- 2 , 2 ú 3
ë û S. The length of the latus rectum of H is 4. 4
S. The domain of g is 4. (– ¥, 0) È (0, ¥)
The correct option is:
5. æ e ù
(A) P ® 4; Q ® 2; R ® 1; S ® 3
ç -¥, e - 1 ú
è û (B) P ® 4; Q ® 3; R ® 1; S ® 2
æ1 e ù (C) P ® 4; Q ® 1; R ® 3; S ® 2
6. (– ¥, 0) È ç , ú
è 2 e -1û (D) P ® 3; Q ® 4; R ® 2; S ® 1
The correct option is: æ p pö æ p ö
54. Let f1 : ¡ ® ¡, f2 : ç - , ÷ ® ¡, f3 : çè - 1, e - 2÷ø ® ¡
2
(A) P ® 4; Q ® 2; R ® 1; S ® 1 è 2 2ø
(B) P ® 3; Q ® 3; R ® 6; S ® 5
and f4 : ¡ ® ¡ be functions defined by
(C) P ® 4; Q ® 2; R ® 1; S ® 6
(D) P ® 4; Q ® 3; R ® 6; S ® 5 æ 2 ö
(i) f1 (x) = sin ç 1 - e - x ÷ ,
52. In a high school, a committee has to be formed from a group è ø
of 6 boys M1, M2, M3, M4, M5, M6 and 5 girls G1, G2, G3,
G4, G5. ì | sin x |
ï -1 if x ¹ 0
(i) Let a1 be the total number of ways in which the (ii) f 2 (x) = í tan x , where the inverse
committee can be formed such that the committee has ï 1 if x = 0
î
5 members, having exactly 3 boys and 2 girls. trigonometric function tan –1 x assumes values in
(ii) Let a2 be the total number of ways in which the
æ p pö
committee can be formed such that the committee has ç- 2 , 2 ÷,
at least 2 members, and having an equal number of è ø
boys and girls. (iii) f3 (x) = [sin (loge (x + 2))], where, for t Î ¡, [t] denotes
(iii) Let a3 be the total number of ways in which the the greatest integer less than or equal to t,
committee can be formed such that the committee has
5 members, at least 2 of them being girls.
ìï x 2 sin
(iv) f4 (x) = í
( 1x ) if x ¹ 0
.
(iv) Let a4 be the total number of ways in which the ïî 0 if x = 0
committee can be formed such that the committee has LIST - I LIST - II
4 members, having at least 2 girls and such that both P. The function f1 is 1. NOT continuous at x = 0
M1 and G1 are NOT in the committee together. Q. The function f2 is 2. continuous at x = 0 and
LIST - I LIST - II NOT differentiable at x = 0
P. The value of a1 is 1. 136 R. The function f3 is 3. differentiable at x = 0 and
Q. The value of a2 is 2. 189 its derivative is NOT
continuous at x = 0
R. The value of a3 is 3. 192
S. The function f4 is 4. differentiable at x = 0 and
S. The value of a4 is 4. 200 its derivative is
5. 381 continuous at x = 0
6. 461 The correct option is:
The correct option is: (A) P ® 2; Q ® 3; R ® 1; S ®4
(A) P ® 4; Q ® 6; R ® 2; S ® 1
(B) P ® 4; Q ® 1; R ® 2; S ®3
(B) P ® 1; Q ® 4; R ® 2; S ® 3
(C) P ® 4; Q ® 6; R ® 5; S ® 2 (C) P ® 4; Q ® 2; R ® 1; S ®3
(D) P ® 4; Q ® 2; R ® 3; S ® 1 (D) P ® 2; Q ® 1; R ® 4; S ®3
EBD_7809
2018 - 18 JEE Advanced 2018 Solved Paper

SOLUTIONS
Paper - 1
Physics
2s cos q
3. (A, C) We know that h =
dv d é kr 2 ù r rg eff
1. (B, C) We know that, F = = ê ú = kr
dr dr ëê 2 ûú As 'r' increases, h decreases
Q Potential energy, V(r) = Kr2/2 given [all other parameter remaining constant]
For r = R, F = kR Also h µ s
Further if lift is going up with an acceleration 'a'
mv 2 then geff = g + a. As geff increases, 'h' decreases.
Also F = (circular motion)
R Also h is not proportional to 'q' but h µ cos q
4. (B, D) Here I + I2 = I1 \ I = I1 – I2
mv 2 k
\ = kR \ v= ´R
R m é - Rt ù é - Rt ù
Vê V
\ I= 1 - e 2 L ú - ê1 - e L ú
æ k ö Rê ú Rê ú
Angular momentum L = mvR = m çç R ÷ R = kmR 2 ë û ë û
÷
è m ø
r r
md v é - Rt - Rt ù
2. (A, C) Given F = tiˆ + ˆj \ = t iˆ + ˆj Vê L
- e 2L ú
dt Þ I= e
Rê ú
r ë û
\ dv = t dt iˆ + dt ˆj [Q m = 1]
v r t t R L R 2L
\ ò0 dv = ò0 t dt iˆ + ò0 dt ˆj
r t2
v = iˆ + tjˆ I
V
2
r 1 1
At t = 1s, v = iˆ + ˆj = (iˆ + 2 ˆj )ms -1
2 2 V I1 I2
r
dr t 2 ˆ ˆ dI
Further = i + tj For I to be maximum, =0
dt 2 dt
r t2 é - Rt - Rt ù
\ dr = dt iˆ + t dt ˆj V ê -R L æ - R ö 2L ú
2 \ e -ç ÷e =0
R êë L è 2L ø úû
r r t t2 t
\ ò0 dr = ò0 2 dt iˆ + ò0 t dt ˆj - Rt
1 æ Rö
\ e 2L = Þ ç ÷ t = ln 2
3 2 2 è 2L ø
r t t
Þ r = iˆ + ˆj
6 2 2L
\ t= ln 2
r 1 1 r 1 1 10 R
At t = 1, r = iˆ + ˆj \ r = + =
6 2 36 4 36 This is the time when I is maximum
r r r æ 1 ˆ 1 ˆö ˆ ˆ é - R æ 2L ö -R æ 2L öù
t = r ´ F = ç i + j ÷ ´ (i + j ) (At t = 1s) V ê L çè R l n 2 ÷ø ç l n 2÷
è6 2 ø = e - e 2L è R øú
Further Imax
Rê ú
iˆ ˆj kˆ ëê ûú
1 1 æ1 1ö 1 V é1 1ù
= 0 = iˆ(0 - 0) - ˆj (0 - 0) + kˆ ç - ÷ = - kˆ Þ I max = ê - ú
6 2 è6 2ø 3 R ë4 2û
1 1 0
V
\ I max =
r 1 4R
\ t = Nm
3
JEE Advanced 2018 Solved Paper 2018 - 19
5. (A, B, D) If I1 = I2, then the magnetic fields due to I1 and I2 7. (2.00)
at origin 'O' will cancel out each other. But the r r r r
A+ B = 3 A- B
magnetic field at 'O' due to the circular loop will be
\ a iˆ + a cos wt iˆ + a sin wt ˆj = 3 a iˆ - a cos wt iˆ - a sin wt ˆj
present. Therefore 'A' is correct.
If I1 > 0 and I2 < 0, then the magnetic field due to both Þ (1 + cos wt )iˆ + sin w t ˆj = 3 (1 - cos w t )iˆ - sin w t ˆj
current will be in + Z direction and add-up. The
magnetic field due to current I will be in – Z direction 2 + 2cos wt = 3 2 - 2cos wt
and if its magnitude is equalr to the combined \ 1 + cos wt = 3(1 - cos wt )
magnitudes of I1 and I2, then B can be zero at the
1 p
origin. Therefore option 'B' is correct. Þ 4cos wt = 2 \ cos wt = or , wt =
2 3
X I1 p p
\ ´t= \ t = 2.00 seconds
6 3
R 8. (5.00) Observer/listener
I2 I
P é v ù 13 m
nA = n ê
ë v - 2cos q úû
12 m
O Z 2 cos q
–R é v ù q 2 ms–1 q 1 ms
–1
nB = n ê
ë v + 1cos q úû
A
Y 5m 5m B q
1 cos q
Beat frequency = n éê ù é ù
v v
If I1 < 0 and I2 > 0 then their net magnetic field at \ ú - nê
origin will be in –Z direction and the magnetic field ë v - 2cos q û ë v + cos q úû
due to I at origin will also be in –Z direction. Therefore é 1 1 ù
r = n vê -
B at origin cannot be zero. Therefore 'C' is incorrect. ë v - 2cos q v + cos q úû
If I1 = I2 then the resultant of the magnetic field BR at é 1 1 ù
P (the centre of the circular loop) is along +X direction. = 1430 ´ 330 ê -
5 5ú
Therefore the magnetic field at P is only due to the ê 330 - 2 ´ 330 + ú
ë 13 13 û
current I which is in –Z direction and is equal to
r m I é 1 1 ù
= 1430 ´ 330 ´ 13 ê -
B = 0 (-kˆ) . ë 330 ´13 - 10 330 ´ 13 + 5 úû
2R
é 1 1 ù
= 1430 ´ 330 ´ 13 ê - » 5 Hz
ë 4280 4295 úû
X
BR 9. (0.75)
The time taken to reach the ground is given by
B2 B1
I1 1 2h æ I ö
t= ç 1+ C 2 ÷
O P Z sin q g è MR ø
I
1 2h æ MR 2 ö 4 h
Y I2 For ring t1 = ç1 + ÷ =
sin 60° g è MR 2 ø 3 g

Therefore option 'D' is correct. æ 1 ö


MR 2
6. (B, C, D) In process I, volume is changing. Therefore it is 1 2h ç 2 ÷ 2 3h
not isochoric. Therefore 'A' is incorrect. For disc t 2 = ç 1+ ÷ =
sin 60° g 2
MR ÷ 3 g
çè ø
In process II, q = DU + W. DU = 0 as temperature is
constant. Therefore q = W. Here W = P(Vf – Vi) is
positive therefore q is positive i.e., gas absorbs heat. 2- 3
Given t1 - t2 =
Therefore 'B' is correct. 10
For process IV, q = DU + W. Here DU = 0 and W is
4 h 2 3h 2 - 3
negative (volume decreases). Therefore q is negative \ - =
i.e., gas releases heat. 'C' is correct. 3 g 3 g 10
For an isobaric process, V µ T i.e., we will get a
h 3h ( 2 - 3 ) æ 3 ö
straight inclined line in T-V graph. Therefore I and II 2 - = ç ÷
are NOT isobaric. 'D' is correct. 10 10 10 è 2 ø
EBD_7809
2018 - 20 JEE Advanced 2018 Solved Paper

3 But V0 = p
2 h - 3h = 3 - \ Average speed along X-axis = 2.00
2 13. (130.00)
0.232 DAFB and DCFD are similar
h (2 – 1.732) = 1.732 – 1.5 \ h=
0.268 d 2 1
= = A
\ h » 0.75 m D 20 10
C
10. (2.09) 2
d 1 D d
(m - m2 )u1 2m2u2 (1 - 2)2 -2 \ Ratio of area = = F
v1 = 1 + = = ms -1 D 2 100 D
m1 + m2 m1 + m2 1+ 2 3 20 cm
As there is no energy loss B 2 cm
(m - m1 )u2 2m1u1 2 ´ 1 ´ 2 4 -1 \ Average intensity of light at a distance 22 cm
v2 = 2 + = = ms
m1 + m2 m1 + m2 1+ 2 3
1.3 ´ p D 2 4
The time period of mass 2 kg after attaining velocity is = = 1.3 × 100 = 130.00 kWm–2
pd2 4
m 2 14. (4.00)
T = 2p = 2p = 2p
k 2 The intermediate temperature is given by the formula

u1 = 2 ms
–1
u2 = 0 k1 A1T1 k2 A2T2
+
l l2
T= 1
1 kg 2 kg k1 A1 k 2 A2
+
l1 l2
Therefore the time taken to return the original position by Here, T = 200 k, T1 = 300 k, T2 = 100 k
2 kg mass is p sec. l1 = l2 and A1 = pr2, A2 = 4pr2
\ Distance between the two blocks
2 2 22 k1pr 2 ´ 300 + k2 p (4r 2 )100
= ´p = ´ = 2.09 m \ 200 =
3 3 7 k1pr 2 + k2 p (4r 2 )
11. (1.50)
Initially 300k1 + 400k2
\ 200 =
The charge on C3 is q3 = C3V = 1 × 8 µC = 8 µC k1 + 4k 2
\ 200 k1 + 800 k2 = 300 k1 + 400 k2

++ k
8V 1 µF = C3 Þ 400 k2 = 100 k1 \ 1 = 4.00
– – k2
E
Finally 15. (C) We know that, C = where C = speed of light
B
As the charge on C3 is found to be 5 µC therefore charges \ E = CB = LT–1B
on C1 and C2 are 3 µC each 16. (D) We know that
Applying Kirchhoff loop law 3 µC
+ + 1 1
5 3 3 C1 = er µF
– – + +
5 µC
C= \ C2 =
- - =0 m 0e0 m 0e0
1 er 1 – – C3 = 1 µF

\ m0 = e0-1L-2T 2
é 1ù
\ 5 = 3 ê1 + ú
ë e rû
+ +
1- a dr (1 + a )(-1) - (1 - a) -2
C2 = 1 µF – –3 µC 17. (B) r= \ = =
1+ a da (1 + a) 2
(1 + a )2
1 5 2
\ = -1= \ e r = 1.5
er 3 3 2 Da
\ Dr =
12. (2.00) (1 + a ) 2

Average speed along X-axis = D1 + D2 = 2( R1 + R2 ) 18. (C) We know that, N = N 0e -lt


t1 + t 2 t1 + t2
Taking log on both sides loge N = loge N0 – lt
differentiating with respect to 'l' we get
é mv0 mv0 ù
ê qB + q (4 B ) ú 1 dN dN 40
= 2ê 1 1 ú = 0-t \ dl = = = 0.02
ê pm + pm ú N dl tN 1 ´ 2000
êë qB1 q (4B1 ) úû [Q N = 3000 – 1000 = 2000]
JEE Advanced 2018 Solved Paper 2018 - 21
Chemistry
19. (B,C) (B) Me H ¾¾¾¾¾®
(A) NH 4 NO 3 ¾¾¾¾¾®
below 300°C
N 2 O + 2H 2 O

N 2O ¾¾¾¾¾¾
® N 2 + O2 O COOH
above 600°C
(C)
D
(B) (NH 4 ) 2 Cr2 O 7 ¾¾ ® N 2 + Cr2 O3 + 4H 2 O ¾ ¾¾ ¾®
D HOOC COOH
(C) Ba(N 3 ) 2 ¾¾¾®
180°C
Ba + 3N 2 O O
(D) Mg3 N 2 ¾¾¾¾¾¾ ® 3Mg + N 2 ¾ ¾¾¾ ®
above 700°C
Hence only (NH4)2Cr2O7 and Ba(N3)2 can provide N2
gas on heating below 300°C
CHO
20. (B, C) (D)
(A) [(Fe(CO5)] & [Ni(CO)4] complexes have 18-electrons in
¾¾¾¾¾®
their valence shell.
OHC CHO
(B) Due to strong ligand field, carbonyl complexes are
predominantly low spin complexes.
24. (B, C)
(C) As electron density increases on metals (with lowering
oxidation state on metals), the extent of synergic A– C Þ isochoric process
bonding increases. Hence M-C bond strength increases A– B Þ isothermal process
(D) While positive charge on metals increases and the ex-
B– C Þ isobaric process
tent of synergic bond decreases and hence C–O bond
becomes stronger. Þ qAC = DUAC = nCV,m (T2 – T1) = D UBC
21. (A, B, C)
(A) Basic character of oxides increase on moving down æV ö
the group therefore Bi2O5 is more basic than N2O5. Þ WAB = –nRT1 ln ç 2 ÷
(B) Covalent nature depends on electronegativity è V1 ø
difference between bonded atoms. In NF3 , N and F are Þ WBC = –P2(V1–V2) = P2(V2–V1)
non-metals but in BiF3, Bi is metal while F is non metal Þ qBC = DHBC = nCP,m (T2–T1) = DHAC
therefore NF3 is more covalent than BiF3.
(C) In PH3 hydrogen bonding is absent but in NH3 Þ DHCA = nCP,m(T1–T2)
hydrogen bonding is present, therefore PH3 boils at Þ DUCA = nCV,m(T1–T2)
lower temperature than NH3.
DHCA < DUCA since both are negative (T1 < T2)
(D) Due to small size in N–N single bond l.p. – l.p.
repulsion is more than P–P single bond therefore N–N
single bond is weaker than the P–P single bond. 25. (1)
22. (B) • H-atom =1s1 Paramagnetic
Me OH Me Br
PBr3
¾¾¾¾®
Et 2O(S N2)
• NO 2= || N odd electron species Paramagnetic
Me O O
I
NaI
¾¾¾® • O2– (superoxide) = One unpaired electron
acetone
(S N2) in p* M.O. Paramagnetic
NaN3(DMF) • S2 (in vapour phase) = same as O2, two
(S N2)
unpaired e–s are present in p* M.O. Paramagnetic
Me N3 +2 +4
• Mn3O4 = 2MnO . MnO 2 Paramagnetic

23. (A, B, D) • (NH4)2[FeCl4] = Fe2+ = 3d 6 4s0 Paramagnetic

O • (NH4)2 [NiCl4] = Ni = 3d 8 4s2


(A) ¾¾¾¾® Ni2+ = 3d 8 4s0 Paramagnetic
D
EBD_7809
2018 - 22 JEE Advanced 2018 Solved Paper
29. (2.22) P1 = 5 bar P2 = 1 bar
é O– ù
| V1 = 1 m3 V2 = 3 m3
ê ú
Mn T1 = 400 K T2 = 300 K
• K2MnO4 = 2K êO
+ ú
|| O– , Mn6+ = [Ar] 3d1
êë úû

|
O 5 3
n1 = n2 =
400R 300R
Paramagnetic Let volume be (V + x) V = (3 – x)

é O ù PA PB
|| =
ê ú TA TB
Cr
• K2CrO4 = 2K+ ê | – –ú , Cr = [Ar] 3d
6+ 0
êëO O O úû
|

n b1 ´ R n b2 ´R
Þ =
Vb1 Vb2
Diamagnetic
5 3
26. (2992) Þ 400 ∋ 4´ x ( = 300R ∋3 , x (

(NH 4 ) 2 SO 4 + Ca(OH) 2 ® CaSO 4 .2H 2 O + 2NH 3 Þ 5(3 – x) = 4 + 4x


1584 g gypsum(M =172) 24 mole
=12 mole 12 mole 11
Þx=
9
NiCl2 .6H 2O + 6NH 3 ® [Ni(NH3 ) 6 ]Cl 2 + 6H 2 O
952 g = 4 mole 24 mole (M = 232) 11 æç 20 ö÷
4 mole V= 1+x =1 + = çç ÷÷ = 2.22
9 è9ø
Total mass = 12 × 172 + 4 × 232 = 2992 g
30. (19) PT = p°AXA + p°BXB
27. (3) As per given information, cations form fcc lattice and
anions occupy all the octahedral voids. 45 = 20(0.5) + poB (0.5)
So M+ X– Formula MX p oB = 70
4 ions 4 ions 22.5 = 20 XA + 70(1 – XA)
After step I 4 ions 1 ion 50XA = 47.5
After step II 1 ion 4 ions
4.75
After step III 0 ion 4 ions XA = = 0.95
5
After step IV 1 ion 3 ions
XB = 0.05
No.of anions 3
So ratio of = XA
No.of cations 1
X B = 19

28. (10) ® Mg 2 + (aq) + 2e -


Mg (s) ¾¾ æ é H∗ ù ö÷
çç ê úû ÷÷ æ10,3 ö÷
Cu 2+ (aq) + 2e - ¾¾
® Cu (s) Ksp çç ë ∗ 1÷÷ = ç
20´10,10 çç ∗ 1÷÷÷
31. (4.47) S = çç K a ÷÷ ,
çè10 ø÷
8
èç ø÷
Mg (s) + Cu 2+ (aq) ¾¾
® Mg 2+ (aq) + Cu (s)
; 2´10,5 = 4.47 × 10 M
–3
RT
E cell = E°cell - lnx 32. (0.05) From graph
nF
For solvent ‘X’ DTb(x) = 362 – 360 = 2
RT x DTb(x) = mNaCl × Kb(x)
E = 2.67 = 2.7 - ln
nF 1 ...(1)
For solvent ‘Y’ DTb(y) = 368 – 367 = 1
300
0.03 = lnx DTb(y) = mNaCl × Kb(y)
2 ´ 11500
...(2)
2.3 = lnx
Dividing equation (1) by (2)
x = 10
JEE Advanced 2018 Solved Paper 2018 - 23

K b∋x ( æ a ö
DTb(y)(s) = ç1 - 2 ÷ K b(y)
Þ K =2 è 2 ø
b∋ y(

For solute S Given DTb(x)(s) = 3DTb(y)(s)


Given solute S dimerizes in solvent. Hence,
æ a1 ö æ a2 ö
2(S) ® S2 ç1 - ÷ K b(x) = 3 ´ ç 1 - ÷ ´ K (b)(y)
è 2 ø è 2 ø
1
(1-a ) a2
æ a ö æ a ö
2 ç1 - 1 ÷ = 3ç1 - 2 ÷
è 2 ø è 2 ø
i = (1 - a 2)
a2 = 0.7
æ a ö
DTb(x)(s) = ç1 - 1 ÷ K b(x) so a1 = 0.05
è 2 ø

CHO CH CH COOH

CO + HCl Ac2O
¾¾¾¾®
33. (C) AlCl /CuCl
3 AcONa
(X)

Br2/Na2CO3
Br Br

C CH CH CH
moist KOH
473 K COONa
(Y)

CHO CH CH COOH

34. (A) CO + HCl Ac2O H2/Pd


¾¾¾¾®
AlCl /CuCl
3 AcONa
HOCO
(X)
H3PO4

(Z)
O

35. (A) 36. (B)

Sol. 35 & 36
COOH
OH
[O] OH
C11H12O2 ¾¾® ¾ ¾¾
+
® dacron
(P) H

COOH
EBD_7809
2018 - 24 JEE Advanced 2018 Solved Paper

COOH

¾
¾
¾H2/Pd-C
¾
H2/Pd-C ¾
®
COOH COOH COC1
SOCl2
¾¾¾®

NH3/D
MeMgCl/CdCl2
CONH2
O
C Me
Br2 ,NaOH
NH2
NaBH4

D CHCl3/KOH OH
NC NH – CH3
H
¾¾®
2
(Q)
Pd-C

(S) HCl

COOH 1.CO2 MgCl Mg/Et2O Cl


+
2.H3O
(R)

Mathematics ( z - z1 )( z2 - z3 )
= k, kÎR
Þ
37. (A, B, D) ( z - z3 )( z2 - z1 )

-3p
(A) arg(-1 - i) = æ z - z1 ö æ z2 - z1 ö
4 Þ ç ÷ = kç ÷
è z - z3ø è z2 - z3 ø
\ (A) is false
Þ z, z1, z2, z3 are concyclic. i.e. z lies on a circle.
é p - tan -1 (t ), t ³ 0 \ (D) is false.
(B) f (t ) = arg(-1 + it ) = ê
êë -p + tan -1 (t ), t < 0 38. (B, C, D)
P
PQ 2 + QR 2 - PR 2 10 3
lim f (t ) = -p and lim f (t ) = p (A) cos 30° =
t ®0- t ®0+ 2PQ × QR 30°
LHL ¹ RHL Þ f is discontinuous at t = 0 Q 10 R
\ (B) is false. 3 (10 3) 2 + 10 2 - PR 2
Þ =
2 2 ´ 10 3 ´10
æz ö
(C) arg ç 1 ÷ - arg z1 + arg z2 Þ PR2 = 100 or PR = 10
è z2 ø \ Ð P = Ð Q = 30°
\ (A) is false.
= 2np + arg z1 - arg z2 - arg z1 + arg z2
= 2np, multiple of 2p 1
(B) Area of DPQR = PQ ´ QR ´ sin30°
\ (C) is true. 2

æ ( z - z1 )( z2 - z3 ) ö 1 1
= ´ 10 3 ´ 10 ´ = 25 3
(D) arg ç ÷=p 2 2
è ( z - z3 )( z2 - z1 ) ø
Also Ð R = 180° – 30° – 30° = 120°
\ (B) is true.
JEE Advanced 2018 Solved Paper 2018 - 25

D 25 3 25 3 5 3 f (0) - f ( -4)
(C) r= = = = f ¢ ( x0 ) =
s æ 10 3 + 10 + 10 ö 10 + 5 3 2 + 3 0 - (-4)
çç ÷÷
è 2 ø f (0) - f (-4)
Þ | f ¢( x0 ) | =
4
= 5 3(2 - 3) = 10 3 - 15
\ (C) is true. Q -2 £ f ( x ) £ 2

abc 10 3 ´ 10 ´10 \ -4 £ f (0) - f (-4) £ 4


(D) R= = = 10
4D 4 ´ 25 3 Þ | f ¢( x0 ) | £ 1
\ (B) is true.
\ Area of circumcircle = pR 2 = 100 p
\ (D) is true. (C) If we consider f ( x) = sin( 85 x) then f (x) satisfies
39. (C,D)
(A) Direction ratios of line of intersection of two planes the given condition [f (0)]2 + [f ¢(0)]2 = 1
r r
will be given by n1 ´ n2 . But lim (sin 85x) does not exist
x ®¥
$i $j k$ \ (C) is false.
r r
n1 ´ n2 = 2 1 -1 = 3$i - 3 $j + 3k$ (D) Let us consider g(x) = [f(x)]2 + [f ¢ (x)]2
1 2 1
By Lagrange’s Mean Value theorem
\ dr’s of line of intersection of P1 and P2 are 1, – 1, 1 | f ¢(x) | £ 1
\ (A) is false.
Also | f(x1 ) |£ 2 as f ( x) Î[ -2, 2]
(B) Given line can be written as
\ g( x1 ) £ 5, for same x1 Î (-4, 0)
4 1
x- y-
3= 3=z Similarly g(x 2 ) £ 5, for same x2 Î (0, 4)
3 -3 3 Also g(0) = 85
Clearly this line is parallel to line of intersection of P1 Hence g(x) has maxima in (x1, x2) say at a such that
and P2 g¢(a) = 0 and g(a) ³ 85
\ (B) is false.
(C) If q is the angle between P1 and P2 then g ¢( a ) = 0 Þ 2f (a )f ¢ (a ) + 2f ¢ (a )f ¢¢ (a ) = 0

2 ´ 1 + 1 ´ 2 + ( -1) ´ 1 3 1 Þ 2f ¢(a)[f(a ) + f ¢¢(a)] = 0


cos q = = =
6 6 6 2
If f ¢(a) = 0 Þ g(a) = [f (a)]2
\ q = 60°
Hence (C) is true. and [f (a )]2 £ 4
(D) Equation of plane P3 :
1(x – 4) – 1(y – 2) + 1(z + 2) = 0 \ g( a) ³ 85 (is not possible.)
or x – y + z = 0 Hence f (a ) + f ¢¢ ( a ) = 0 for a Î (x1, x 2 ) Î (-4, 4)
2 - 1+ 1 2 \ (D) is true.
Distance of (2, 1, 1) from P3 = =
1+ 1 + 1 3 41. (B, C)
\ (D) is true. Given f ¢(x) = e(f(x) -g(x)) × g¢(x)
40. (A, B, D)
-f (x)
(A) f(x) being twice differentiable, it is continuous but can’t Þ e f ¢(x) = e -g(x) g ¢(x)
be constant throughout the domain.
Integrating both sides, we get
\ We can find x Î (r , s ) such that f(x) is one one.
-e -f (x) = -e - g(x) + c
Hence (A) is true.
(B) By Lagrange’s Mean Value theorem for f(x) in [–4, 0], Þ - e - f (x) + e - g(x) = c
there exists
Þ -e -f (1) + e- g(1) = -e -f (2) + e -g(2)
x0 Î (-4,0) such that
But given that f (1) = g(2) = 1
EBD_7809
2018 - 26 JEE Advanced 2018 Solved Paper

\ -e -1 + e -g(1) = -e -f (2) + e -1 1 1
2 log 2 (log 2 9) log 4 7
43. (8) ((log 2 9) ) ´ ( 7)
2
Þ e -f (2) + e -g(1) =
e 1
´ log 7 4
= ( log 2 9 )2 ´ log (log 2 9) 2 ´ 7 2
2 2
Þ e -f (2) < and e -g(1) <
e e = ( log 9 )log(log 2 9) 4 ´ 7log7 2
2
Þ -f (2) < ln 2 - 1 and -g(1) < ln 2 - 1 = 4 ´ 2 = 8.
Þ f (2) > 1 - ln 2 and g(1) > 1 - ln 2 44. (625) The last 2 digits, in 5-digit number divisible by 4, can
be 12, 24, 32, 44 or 52.
\ (B) and (C) are True.
42. (B, C) Also each of the first three digits can be any of
{1, 2, 3, 4, 5}
x
f (x) = 1 - 2x + ò e x - t f (t) dt Hence 5 options for each of the first three digits and
0
total 5 options for last 2-digits
x \ Required number of 5 digit numbers are
Þ f (x) = 1 - 2x + e x ò e - t f (t)dt
0 = 5 × 5 × 5 × 5 = 625
45. (3748)
x
Þ f ¢(x) = -2 + e x ò e- t f (t) dt + e x [e - x f (x)] The given sequences upto 2018 terms are
0
1, 6, 11, 16, ........., 10086
Þ f ¢(x) = -2 + [f(x) - 1 + 2x] + f(x) and 9, 16, 23, ........., 14128
Þ f ¢(x) - 2f(x) = 2x - 3 The common terms are
Its a linear differential equation. 16, 15, 86, ........... upto n terms, where Tn £ 10086

IF = e ò
-2dx
= e -2x Þ 16 + (n – 1) 35 £ 10086
Þ 35n – 19 £ 10086
Solution: f (x) ´ e -2x = ò e-2x (2x - 3) dx
10105
Þ n£ = 288.7
-2x -2x 35
e e
f (x) ´ e -2x = (2x - 3) - ò ´ 2 dx \ n = 288
-2 -2
\ n(XÈ Y) = n(X) + n(Y) - n(X Ç Y)
-2x -2x = 2018 + 2018 – 288 = 3748
e e
e -2x f (x) = (2x - 3) + +c
-2 -2
æ x ö æ x ö
ç 2 ÷ -
3 1 -1 x 2 -1 ç 2 -x ÷
f (x) = - x + + + ce2x 46. (2) sin ç -x×

= sin ç - ÷
2 -2 ç1- x 1- ÷ ç 1+ x 1+ x ÷
è 2ø è 2 ø
f (x) = - x + 1 + ce 2x
From definition of function, f(0) = 1 x2 x2 -x x
Þ - = +
\1=1+c Þ c=0 1- x 2 - x 2 + x 1+ x
\ f(x) = 1 – x
Clearly curve y = 1 – x, does not pass through (1, 2) but it x2 x x x2
Þ - + - =0
passes through (2, – 1) 1- x 1+ x 2 + x 2 - x
\ (A) is false and (B) is true
Also the area of the region B x(x + x 2 - 1 + x) x(2 - x - 2 x - x 2 )
Þ 2
+ =0
1- x 4 - x2
1 - x £ y £ 1 - x 2 , is shown in
the figure, is given by x(x 2 + 2 x - 1) x(2 - 3x - x 2 )
O A Þ + =0
= Area of quadrant – Area DOAB 1- x2 4 - x2
1 1 p-2 2 2 2 2
= ´ p´12 - ´ 1´ 1 = Þ x[(x + 2 x - 1)(4 - x ) + (1 - x )(2 - 3x - x )] = 0
4 2 4
\ (C) is true and (D) is false.
JEE Advanced 2018 Solved Paper 2018 - 27
3 2
Þ x[x + 2x + 5x - 2] = 0 \ 3 a cos a + 2b sin a = c ... (1)
Þ x = 0 or x3 + 2x2 + 5x – 2 = 0 = p(x) (say) and 3 a cos b + 2b sin b = c ... (2)
æ1ö subtracting equation (2) from (1) we get
We observe p(0) < 0 and p ç ÷ > 0
è2ø 3 a (cos a – cos b) + 2b (sin a – sin b) = 0
a +b a -b a+b a -b
\ One root of p(x) = 0 lies in æç 0, ö÷ .
1 - 3 a2sin + 2b.2cos =0
Þ sin sin
è 2ø 2 2 2 2

p p
1 1 Þ -2 3 a sin + 4b cos = 0
Thus two solutions lie between , and . 6 6
2 2
1/n 1 3
æ n +1 n + 2 n + 3 n+nö Þ -2 3 a ´ + 4b =0
47. (1) yn = ç . . ..... ÷ 2 2
è n n n n ø
b 1
n Þ = = 0.5
1 æ rö a 2
Þ log yn =
n å log ç1 + ÷
è nø
r =0
50. (4) y
n
n y=x
æ rö
)
1
Þ
( lim y n = lim
n ®¥ n ®¥ n
å
r=0
log ç1 + ÷
è nø Q(1,1)

1 1 x
ò0 log(1 + x)dx = [x log(1 + x)]0 - ò0 1 + x dx
1
Þ log L =

P(0,0) R(2,0) x
= log 2 - [ x - log |1 + x |]0
1

= log 2 – 1 + log 2 = 2 log 2 – 1


30
Shaded area = ´ Ar( DPQR)
æ4ö 100
= log 4 – log e = log ç ÷
èeø
1 3 1
ò0 (x - x
n
Þ )dx = ´ ´ 2 ´1
4 é4ù 10 2
\ L= Þ [ L] = ê ú = 1
e ëeû 1
r r rr r æ x 2 x n +1 ö 3
48. (3) Given | a | = | b | = 1, a.b = 0, | c | = 2 Þ çç - ÷÷ =
è 2 n + 1 ø0 10
r r r
c makes angle a with both a and b
r r r r r 1 1 3
Also, c = x a + y b + a ´ b Þ - =
2 n + 1 10
rr
c.a = 2 cos a Þ x = 2 cos a
1 1 3 1
rr Þ = - = Þ n=4
c.b = 2 cos a Þ y = 2 cos a n + 1 2 10 5
r2 r r r r r r2
c = c.c = (2cos a)a + (2cos a) b + a ´ b y
51. (A) (0,2) G G3
r r
2

Þ (2)2 =4 cos2 a + 4 cos2 a + | a ´ b |2 F1 E1


E2
r r
P(

( )
1,
1)
Þ 4 = 8 cos2 a + 1 Q | a ´ b | = 1 ´ 1 ´ sin 90° = 1 1 x
x O G1(2,0)
Þ 8 cos2 a =
3
49. (0.5) Given that the equation
F2
3 a cos x + 2b sin x = c
y1
p
has two roots a and b, such that a + b =
3
EBD_7809
2018 - 28 JEE Advanced 2018 Solved Paper
Equation of E1 E2 : y = 1 53. (A) No. of dearrangements for 4 students
Equation of F1 F2 : x = 1
æ 1 1 1 1ö
Equation of G1 G2 : x + y = 2 = 4!ç1 - + - + ÷
è 1! 2! 3! 4! ø
By symmetry, tangents at E1 and E2 will meet
= 12 – 4 + 1 = 9
on y-axis and tangents at F1 and F2 will meet
Total no. of arrangements of seating of 5 students
on x-axis
= 5! = 120
E1 º ( ) (
3, 1 & F1 º 1, 3 ) 9 3
\ required probability = =
120 40
Equation of tangent at E1 : 3x + y = 4
54. (C) Total cases = 5! = 120
Equation of tangent at F1 : x + 3y = 4 Favourable cases :
\ Points E3 (0, 4) and F3(4, 0) 1, 3, 5, 2, 4 ü
ý2
Tangents at G1 and G2 are x = 2 and y = 2 1, 4, 2, 5, 3þ
intersecting each other at G3(2, 2). 2, 4, 1, 3, 5ü
Clearly E3, F3 and G3 lie on the curve x + y = 4. ï
2, 5, 3, 1, 4 ý 3
52 (D) Let point P be (2 cos q, 2 sin q)
2, 4, 1, 5, 3ïþ
Tangent at P : x cos q + y sin q = 2
3, 1, 4, 2, 5ü
æ 2 ö ï
\ Mç ,0 ÷ . and N æ 0, 2 ö 3, 5, 2, 4, 1ý 3
è cos q ø ç ÷
è sin q ø 3, 1, 5, 2, 4 ïþ
æ 1 1 ö 4, 2, 5, 1, 3ü
Mid point of MN = ç , ÷ ï
è cos q sin q ø 4, 2, 5, 3, 1ý 3
For locus of mid point (x, y) of MN, 4, 1, 3, 5, 2 ïþ
1 1 5, 2, 4, 1, 3 ü
x= , y= ý2
cos q sin q 5, 3, 1, 4, 2 þ
\ favourable cases = 14
1 1 14 7
Þ 2
+ 2 =1 \ required probability = =
x y 120 60
Þ x2 + y2 =x2y2
JEE Advanced 2018 Solved Paper 2018 - 29

Paper - 2
Physics
dk d æ1 ö A' ¥
1 mV 2 ÷ = gt
= gt and k = mV 2 \
dt çè 2
1. (A, B, D) A
dt 2 ø

m dV dV B' P Bf/2 f C
Þ ´ 2V = gt \ mV = gt
2 dt dt

V t
m ò VdV = g ò tdt mV 2 gt 2
\ Þ = f
0 0 2 2 The object distance decreases from to f. Therefore the
2
g final result is (D).
\V= ´ t. i.e., V µ t
m
232 – 212 20
As V is proportional to 't', distance cannot be proportional 5. (A, C) No. of a-particles = = =5
4 4
to 't'.
232 212 4 0
\ Th Pb + 5 He + 2 b
dV dé g ù g 90 82 2 –1
Now F = ma = m =m ê ´ tú = m = gm = constant
dt dt ë m û m
l
6. (A, B, C) Given (2n + 1) = 50.7 + e
4
u
2. (A, C, D) We know that |F| = hA 0
h l
and (2n + 3) = 83.9 + e
u 4
where 0 = velocity gradient
h 3l / 4 50.7 + e
If n = 1, = \ 3 × 83.9 + 3e = 5 × 50.7 + 5e
|F| u 5l / 4 83.9 + e
Also =h 0 \ 2e = 1.8 Þ e = 0.9 cm
A h
3. (A, B) According to Gauss's Law, 3l
\ = 50.7 + 0.9 = 51.6 \ l = 66.4 cm
4
1 1 3lR
Electric flux, f = qin = [l × 2 R sin 60°] = Also V = vl = 500 × 0.664 ms–1 = 332.0 ms–1
Î0 Î0 Î0
7. (6.30) Impulse = Change in linear momentum
Further electric field is perpendicular to the wire therefore
its z-component will be zero. J 1
\ J = mV0 or V0 = = = 2.5 ms–1
m 0.4
z
l ds
Also V = v0e–t/t \ = v0e–t/t Þ ds = v0e–t/t dt
A R dt
AB = R sin 60°
60° t
B –t / t
\ s = v0 ò e dt = v0t (1 – e–1) = 2.5 × 4 × 0.63 = 6.30 m
0

æ1ö
u2 ç ÷
u 2 sin 2 q è2ø
8. (30.00) H= Þ 120 =
2g 2g
4. (D) The image of AB will be A'B' as AB lies between pole
and focus. Further as the object is moved towards the focus \ u2 = 480 g
the image also moves away.
1
\ K.Einitial = mu 2 = 240 mg
2
EBD_7809
2018 - 30 JEE Advanced 2018 Solved Paper
13. (24.00) Number of electrons emitted per second
1
K.Efinal = (240 mg) = 120 mg
2
200 W
1 2 =
\ mv = 120 mg \ v2 = 240 g 6.25 ´ 1.6 ´ 10 -19 J
2
Force = Rate of change of linear momentum = N 2mk
æ1ö
240 g ´ ç ÷ 200
v 2 sin 2 q è4ø = × 2 ´ 9 ´10 –31 ´1.6 ´ 10 –19 ´ 500
\ H¢ = = = 30 m 6.25 ´1.6 ´ 10–19
2g 2g
[Q K = eV: e = 1.6 × 10–19 = V = 500]
9. (2.00) Given E = sin 103 t iˆ = 24.00
F = ma 14. (3.00) DE2–1 = 74.8 + DE3–2

dv qEdt q sin1000t iˆ é 1ù é1 1ù
\ qE = m \ dv = = dt 13.6z2 ê1 – ú = 74.8 + 13.6z2 ê4 – 9ú
dt m m ë 4û ë û
\ z= 3
v p/w 2p ù
q é T
\ ò dv = ò sin1000t dt ê max. speed is at = kQ kp
m ë 2 w´ 2 úû 15. (B) For a point charge E = and for a dipole E =
0 0 d2 d3
p/ w 2k l
q é cos1000t ù 1[cos1000t ]0p / w Further for an infinite long line charge E =
d
and for
\ V= – ê = – –3 ´
m ë 1000 úû 0 10 1000 s
infinite plane charge E =
2 Î0
é p ù
\ V = – êcos1000 ´ – cos 0 ú = –[–1–1] = 2 ms–1 2k l
ë 1000 û r
10. (5.56) We know that Cq = NBA Ig a

Cq 10 – 4 ´ 0.2
\ Ig = = = 0.1A 2k l
NBA 50 ´ 2 ´10 –4 ´ 0.02 r
r
Further for a galvanometer d
Ig × G = (I – Ig) S
I gG
+l l –l
0.1 ´ 50 50
\ S= I – I = = W Also for two infinite wires carrying uniform linear charge
g 1 – 0.1 9 density.
Fl 2k l 2k l l 2k l l
E= cos a = ´ =
11. (3.00) We know that Dl = r 2 2 2 2 d 2 + l2
AY d +l d +l
GM V1 R2 2
1.2 ´ 10 ´ 1 16. (B) Vo= , \ =
= ; 0.3 mm R V2 R1 = 1
2
æ 5 ´ 10 –4 ö
pç ÷÷ ´ 2 ´ 10
11 L1 m1v1 R1 2 ´ 2 ´1 1
ç 2 Further L = m v R ==
è ø 2 1 2 2 1´1´ 4 1
The third marking of vernier scale will coincide with the GMm k1 m1 R1 2´ 4 8
main scale because least count is 0.1 mm. Also K.E. = . Therefore k = m ´ R = =
R 2 2 2 1´ 1 1
12. (900.00) For an adiabatic process TVg–1 = T2 (8V)g–1 3/ 2
T1 æ R1 ö 1
Further T2 µ R3 =ç ÷ \ =
5 T T2 è R2 ø 8
where g = \ T2 =
3 4 17. (C) Process 1 is adiabatic therefore DQ = 0
Process 2 is isobaric therefore W = P(V2 – V1)
æf ö nfR æ –3T ö = 3P0(3V0–V0) = 6P0V0
Further DV = nCvDT = n ç R ÷ DT = ç ÷
è2 ø 2 è 4 ø Process 3 is isochoric therefore W = P(V2 – V1) = 0
Process 4 is isothermal therefore temperature is
1´ 3 ´ 8 3 constant, Du = 0
\ DV = – ´ ´100 = –900J
2 4
JEE Advanced 2018 Solved Paper 2018 - 31
r As the force is central therefore total energy remains
r dr
18. (A) P ® v = = aiˆ + bˆj which is constant constant.
dt
r
r dr
\ a =0 R ® vr = = a[– w sin wt iˆ + w cos wt ˆj ]
r dt
r
Further P = mv is constant \ v = aw i.e., speed is constant
1 r
and K = mv2 is constant r dv
2 a = dt = –aw2[cos wt iˆ + sin wt ĵ ]

r æ ¶U ˆ ¶U r r
F = –ç i+ ˆj ö÷ = 0 (Q ar is constant)
\ a = – w2 r
è ¶x r = rr ´ Fr = 0
¶y ø
t
Þ U = constant
Force is central in nature
Also E = K + U
U and K are also constant.
r
dL r r r r r
\ = t= r´F =0 \ L = constant dr
dt S ® vr = = at iˆ + bt ˆj \ V = f(t)
dt
r
r dr r
Q® v = = -aw(sin wt ) ˆi + bw (cos wt ) ˆj a = bĵ i.e., constant
dt r r
r F = ma constant
r dv r
\ a= = -w2 [a cos wt iˆ + b sin wt ˆj] = -w 2 r t
dt r r – mb2t 2
DU = – ò F × dr = – m ò bˆj × (aiˆ + btjˆ)dt =
r r r r r 2
Also t = r ´ F = 0 (\ r and F are parallel) 0

r uur r 1 2 1
mw2 r 2 K= mv = m(a 2 + b 2 t 2 )
DU = - ò F .dr = + ò mw2rdr = \ U µ r2 2 2
2
0
1
Also E = K + U = ma 2 which is constant.
Also r = a 2 cos2 wt + b 2 sin 2 wt \ r = f (t) 2

Chemistry
19. (A, B, D)
(A) [Co(en)(NH3)3(H2O)]3+ has 2 geometrical isomers.
3+ 3+
NH3 NH3
NH3 NH3
en Co en Co
OH2 NH3
NH3 H2O

(B) Compound [Co(CN)2(NH3)3(H2O)]+ will have three geometrical isomers.


+ + +
H2O H2O H2O
H3N NH3 H3N CN H3N CN
Co Co Co
H3N CN H3N CN NC NH3
CN NH3 NH3

(C) [Co(en)(NH3)3(H2O)]3+ is diamagnetic


(D) [Co(en)(NH3)4]3+ has larger gap between eg and t2g than [Co(en)(NH3)3(H2O)]3+ . So [Co(en)(NH3)3(H2O)]3+absorbs light
at longer wavelength as compared to [Co(en)(NH3)4]3+.
EBD_7809
2018 - 32 JEE Advanced 2018 Solved Paper
20. (B, D)
(A) Cu2+ shows characteristic green colour in the flame test whereas Mn 2+ shows the pale colour in flame test.
(B) Only Cu2+ can give black precipitate of CuS in acidic medium on passing H2S.
(C) Both Cu2+ and Mn2+ show the formation of precipitate by passing H2S in faintly basic medium.

(D) E°Cu 2+ /Cu (+ 0.34V) > E °Mn 2+ /Mn (– 1.18V) as per electrochemical series.

NH2 NH2 NH2 NH2


Conc. HNO
NO2
21. (D) ¾ ¾¾ ¾ ®
3
+ +
Conc. H2SO4
NO2
NO2
(P) 51% (Q) 47% (R) 2%

O O
NH2 NH C CH3 NH C CH3
NO2 NO2 NO2
Ac O, Pyridine Br
¾¾¾¾¾®
2
¾¾¾¾®
2
CH 3COOH
(R)
Br
¾¾®
+
H3O

N2+ Cl– NH2


NO2 NO2 NO2
EtOH NaNO , HCl
¬¾¾¾ ¬¾¾¾¾¾
2
D 273-278 K

Br Br Br
(S)
Br
NO2 NH2 NH2
Sn/HCl Br
¾¾¾® ¾¾¾¾®
2
H2O (excess)
Br Br
Br Br Br
(S)
¾¾®

NaNO2, HCl

Br Br
H3PO4
N2+ Cl–
¬¾¾¾
Br Br Br Br
Br Br
22. (D) Structure of b-L-glucopyranose is First order
A(g) ¾¾¾¾¾
® 2B(g) + C(g) V = constant,T = 300K
H t=0 P0
O
HO
CH2OH H æ 2P0 ö 4P0 2P0
t = t1/3 çè P0 - ÷
3 ø 3 3
H H HO OH P0
=
OH H 3
t=t P0 – x 2x x
23. (A, D) So, Pt = P0 – x + 2x + x = P0 + 2x
or 2x = Pt – P0
JEE Advanced 2018 Solved Paper 2018 - 33
1 P0 O O O
t= ln
k (P0 - x) H 5P 3O 10 = HO P O P O P OH

1 P0 1 2P0 OH OH OH
or t= ln = ln S
k (Pt - P0 ) k 2P0 - Pt + P0
P0 -
2 H2S2O3=HO S OH
2P0 O
or kt = ln , kt = ln 2P0 - ln(3P0 - Pt )
3P0 - Pt O O
or ln(3P0–Pt) = – kt + ln 2P0 H2S2O5 = HO S S OH
Graph between ln(3P0 – Pt) vs ‘t’ is a straight line with O
negative slope. 26. (6.47)
Since rate constant is a constant quantity and independent PbS + O2 ¾® Pb + SO2
of initial concentration.
So graph (A) and (D) are correct. 103
No. of moles of O2 =
24. (A, C) 32
On increasing temperature, concentration of product 103
decreases Moles of Pb formed =
Hence reaction is exothermic Þ DH°<0 32
lnK T1 103
> 1 Þ lnK T1 > lnKT2 so, K T > K T \ Mass of Pb formed = ´ 207 = 6468.75 g
lnK T2 1 2 32
= 6.46875 kg
lnK T1 T = 6.47 kg
Also, > 2 27. (126)
lnK T2 T1 The balanced equations are
T1lnKT1 > T2 lnKT2 Þ -RT1lnK T > -RT2lnKT 2MnCl2 + 5K2S2O8 + 8H2O ¾®
or 1 2 2KMnO4 + 4K2SO4 + 6H2SO4 + 4HCl ...(1)
2KMnO4 + 5H2C2O4 + 3H2SO4 ¾®
or DGT°1 < DG°T2 (Q DG = –RT lnK)
K2SO4 + 2MnSO4 + 8H2O + 10CO2 ...(2)
or DH°–T 1DS <DH –
° ° T2DS° Mass of oxalic acid added = 225 mg
(Also DG°=DH°–TDS°: Gibbs Helmholtz equation) 225
Milimoles of oxalic acid added = = 2.5
As DGT°1 < DGT° 2 , this is possible only when DS° < 0 90
From equation (2)
O Milimoles of KMnO4 used to react with oxalic acid = 1
N N
25. (6) N2O3= O N N O (5 m mole H2C2O4 = 2 m mole of KMnO4)
O and milimoles of MnCl2 required initially = 1
O
O O \ Mass of MnCl2 required initially = 1 ´ 126 = 126 mg
O
N2O5= (Molar mass of MnCl2 = 126)
N N
28. (7)
O O 29. (495) Moles of D formed = 10 × 0.6 × 0.5 × 0.5 × 1 = 1.5
P Mass of D formed = 1.5 × 330 = 495 g
O O 30. (– 14.6)
O
P4O6= P 1
O P ® Cu 2O (s) : D G ° = -78 kJ/mol
(i) 2Cu (s) + O2 (g) ¾¾
2
O P O 1
® H 2O(g), D G ° = -178 kJ/mol
(ii) H 2 (g) + O2 (g) ¾¾
2
P O (i) – (ii) then
O
O
P4O7= P 2Cu (s) + H 2 O (g) ¾¾
® Cu 2 O (s) + H 2 (g)
O P=O
DG° = –78 + 178 = 100 kJ/mol = 105 J/mol
O P O Now for the above reaction

O O æ PH ö
DG = DG° + RTln ç 2
÷
P P è PH2 O ø
H4P2O5 = H O H To prevent the above reaction:
OH OH DG > 0
EBD_7809
2018 - 34 JEE Advanced 2018 Solved Paper

æ PH ö Anode ® A n + + ne - ) ´ 2
(A ¾¾
DG° + RTln ç 2
÷ ³0
P
è H2 O ø Cathode B2n + + 2ne - ¾¾
®B
æ PH ö Overall reaction :
2
105 + 8 × 1250 ln ç ÷ ³0 2A + B2n + ¾¾
® 2A n + + B
è PH 2O ø
RT
104 (lnPH 2 - ln PH2O ) ³ -105 E = E° - lnQ
2nF
lnPH 2 ³ -10 + ln PH 2O RT [A n + ]2
Now, 0 = E° - ln
2nF [B2n + ]
PH 2O = X H 2O ´ Ptotal = 0.01 ´ 1 = 10-2
RT 22
> –10 – 2ln10 E° = ln
> –10 – 2×2.3 (Given ln10 = 2.3) 2nF 1
ln PH 2 ³ -10 - 4.6 RT
E° = ln 4
2nF
ln PH 2 ³ -14.6 Now,
\ Minimum ln PH 2 = -14.6 2nFRT
DG° = -2nFE° = - ln 4 = - RTln 4
31. (– 8500) 2nF
ˆˆ† AB(g) DG° = DH° – TDS° = 2DG° – TDS° (Given DH° = 2DG°)
A (g) + B(g) ‡ˆˆ
TDS° = DG°
Ea b = Ea f + 2RT & Af = 4A b DGº - RTln 4
DS° = = = - Rln 4
Now, T T
= –8.3 × 2 × 0.7 = –11.62 JK–1 mol–1
Rate constant of forward reaction k f = Af e - Ea f /RT
33. (c) P – 6; Q – 4, 5; R – 1; S – 2, 3
Rate constant of reverse reaction K b = A b e - E ab /RT 1. [FeF6]4–, Fe2+ = 3d6 & F– is weak field ligand
Equilibrium constant \ Hybridization is sp3d2 (high spin complex)
2. [Ti(H2O)3Cl3], Ti3+ = 3d 1 (No effect of ligand field
K f Af - (Eaf - E ab )/RT strength)
K eq = = e
Kb Ab \ Hybridization is d 2sp3
3. [Cr(NH3)6]3+, Cr 3+ = 3d3 (No effect of ligand field
K eq = 4e 2RT /RT = 4e 2 strength)
Now, DG° = –RT ln Keq = –2500 ln (4e2) \ Hybridization is d 2sp3
= –2500 (ln 4 + ln e2) 4. [FeCl4]2–, 3d6 & Cl– is weak field ligand
= –2500 (1.4 + 2)
\ Hybridization is sp3
= –2500 ´ 3.4
= –8500 J/mol. 5. Ni (CO)4, Ni = 3d10 & CO is strong field ligand
32. (– 11.62) \ Hybridization is sp3
A (s) | An+ (aq, 2M) || B2n+ (aq, 1M) | B (s) 6. [Ni(CN)4]2–, Ni2+ = 3d 8 & CN is strong field ligand
Reactions \ Hybridization is dsp2

34. (D)
Ph OH CH3 CH3
Ph O O
Ch3 Ph
(P) H2SO4 –H + I2/NaOH
Ph ¾¾¾® C+ CH3 ¾¾¾®
(1) H3C ¾¾¾®
(1)
Ph
OH Ph
H3C Ph CH3 OH
OH
.. Ph
..

(P)
O
Ph
NaOBr
¾¾¾¾¾® OH
(5)
Ph
CH3
JEE Advanced 2018 Solved Paper 2018 - 35
Ph NH2 H O CH3 O
Ph
(Q) H Ph
HNO2 + .. –H + Fehling solution
Ph ¾¾¾® C . ¾¾¾® H3C
.OH ¾¾¾¾¾® Ph
(3)
H3C OH Ph
Ph H H
Me Ph
O
Ph
[Ag(NH ) ]OH
¾¾¾¾¾¾¾®
3 2
OH
(2)
Ph
CH3

Ph OH Ph O CH3 O
Ph Ph
Ph
H2SO4 .. –H + I2/NaOH
(R)
Me ¾ ¾¾ ® C+ . ¾ ¾¾ ® H3C
.OH ¾¾¾®
(1)
Ph
Me OH Me Ph CH3 OH
Me Ph

O
Ph
NaOBr
¾ ¾¾ ¾
(5)
¾®
OH
Ph
CH3

Ph Br H O CH3 O
Ph
(S) H Ph
AgNO3 + .. –H + [Ag(NH 3)2]OH
Ph ¾ ¾¾ ® C . ¾ ¾¾ ® H3C
.OH ¾¾¾¾¾® Ph
(2)
Me OH Ph
Ph H OH
CH3 Ph
O
Ph
Fehling solution
¾ ¾¾ ¾ ¾¾¾ ® OH
(3)
35. (B) P – 1, 4; Q – 2; R – 4; S – 3 Ph
CH3
H3C H3C H3C
CH3 CH3 CH3 CH2
P: + E2
¾¾®
+
H3C H3C H3C
ONa Br H3C CH3
OH
(1) (4)

CH3
H3C CH3
Q : H3C O + HBr ¾¾® + MeOH

H3C Br
CH3 Me
(2)

H3C
CH3 CH 2

R: MeO Na
¾¾¾¾®
+

E2
H3C H 3C CH 3
Br (4)
EBD_7809
2018 - 36 JEE Advanced 2018 Solved Paper
n
H3C
H3C
CH3 CH3 = å[tan -1(x + j) - tan -1(x + j-1)]
S 2 j=1
S: + MeBr ¾¾®
N
Þ fn(x) = tan–1(x + n) – tan–1 (x)
H3C H3C
ONa OMe -1 æ n ö
= tan ç ÷
(3) è 1 + x(n + x) ø
36. (D) P – 1; Q – 5; R – 4; S – 1 1 1
Þ f n¢ (x) = -
(P) NaOH + CH3COOH ¾® CH3COONa + H2O 1 + (x + n) 1 + x 2
2
m mole 10 × 0.1 20 × 0.1 and fn(0) = tan–1(n) \ tan2(tan–1n) = n2
= 1 m.mol = 2 m.mol Here x = 0 is not in the given domain, i.e., x Î (0, µ).
\ Solution contains 1 m. mol CH3COOH & 1 m.mol \ Options (A) & (B) are not correct options.
CH3COONa in 30 mL solution.
It is a Buffer solution. Hence pH does not change with æ n ö
(C) lim tan(f n (x)) = lim ç ÷=0
dilution. x ®¥ x ®¥ è 1 + x(n + x) ø
(Q) NaOH + CH3COOH ¾® CH3COONa + H2O
m mole 20 × 0.1 20 × 0.1 (D) lim sec 2 (fn (x)) = nlim 1 + tan 2 (f n (x))
= 2 m.mol = 2 m.mol x ®¥ ®¥
\ Solution contains 2 m. mol of CH3COONa in 40 mL
solution (salt of weak acid and strong base) = 1 + lim tan 2 (f n (x)) = 1
x ®¥
For salts of weak acid and strong base :
38. (B, D)
K w Ka P A Q
[H+]initial = T
C (1, 1)B
C
On dilution upto 80 mL, new conc. will be = M
2 R C(0, 1)
K w Ka
\ [H ]new =
+ = [H+]initial × 2 common tangent
C/ 2
(radical axis)
(R) HCl + NH3 ¾® NH4Cl
m mole 20 × 0.1 20 × 0.1 Here, AP = AQ = AM
= 2m.mol = 2 m.mol \ locus of M is a circle having PQ as its
\ Solution contains 2 m. mol of NH4Cl in 40 mL solution diameter and is given as :
(salt of strong acid and weak base) E1 : (x – 2) (x + 2) + (y – 7)(y + 5) = 0 and x ¹ ±2
For salts of strong acid and weak base and its centre is (0, 1)
Locus of B (midpoint) is a circle having RC as its diameter
Kw C
[H+]initial = and is given as :
Kb E2 : x(x – 1) + (y – 1)2 = 0
C (A) Option (A) is incorrect although it satisfies E1 otherwise
On dilution upto 80 mL, new conc. will be = . the line T would touch the second circle on two points.
2 (B) Also (4/5, 7/5) satisfies E2 but again in this case one
K w C [H∗ ]initial end of the chord would be (–2, 7) which is not included
\ [H+]new = Kb 2 = 2 æ4 7ö
in E1. Therefore ç , ÷ does not lie in E2.
(S) Ni(OH)2(s) ¾¾
® Ni2+ + 2OH– è5 5ø
(C) (1/2, 1) does not satisfy E2 therefore it does not lie in
Q it is sparingly soluble salt E2.
(D) (0, 3/2) does not satisfy E1, so it does not lie in E1.
\ On dilution [OH–] conc. in saturated solution of 39. (A, D) Here D= 0 so for at least one solution, we have
Ni(OH)2 remains constant
D1 = D 2 = D3 = 0
\ [H+]new = [H+]initial Þ b1 + 7b2 = 13b3 (i)

Mathematics (A) D ¹ 0
\ The equations have unique solution
n \ Option (A) is correct.
-1 æ 1 ö
37. (D) f n (x) = å tan ç ÷ (D) D ¹ 0
j=1 è 1 + (x + j)(x + j- 1) ø \ The equations have unique solution
n
\ option (D) is correct
é (x + j) - (x + j - 1) ù
= å tan -1 ê 1 + (x + j)(x + j- 1) ú (C) D = 0
Þ equations are x – 2y + 5z = – b1
j=1 ë û
JEE Advanced 2018 Solved Paper 2018 - 37
(C)
b2
x - 2y + 5z = y
2
x - 2y + 5z = b3
The planes given in option (c) are parallel so they must be
coincident
b x
Þ -b1 = 2 = b3 ...(ii) (–1, 0) (1, 0)
2
\ Equation (ii) satisfies equation (i) for all b1, b2, b3, but
not every value of (b1, b2, b3) of equation (i) satisfies 1
x=
equation (ii) 2
\ Option (C) is not correct.
1 1
1 1 3 1 1 1 \ Required area = 2 × ò 1 × 1 - x 2 dx
2 2
(B) D = 5 2 6 = -3 5 2 2 = 0
1
-2 -1 -3 2 1 1 éx 1 ù
= 2 ê 1 - x 2 + sin -1 x ú
ë2 2 û1/ 2
-1 1 3
Also D1 = 2 2 6 b3 = 0 é p æ 1 p öù æp 1ö p-2
= 2 ê - ç + ÷ú = 2 ç - ÷ =
1 -1 -3 ë 4 è 4 8 øû è8 4ø 4 2
For infinite solutions, D2 and D3 must be 0 41. (A, C, D)
sz + tz + r = 0 ...(i)
1 b1 3 ...(ii)
s z+ t z+ r =0
Þ 5 b2 6 =0 Adding (i) and (ii), we get :
-2 b3 -3 (t + s )z + (s+ t) z + (r + r) = 0 (1)
Þ b1 + b2 + 3b3 = 0 which does not satisfy (i) for all b1, b2, b3 Subtracting (ii) from (i), we get:
so option (B) is incorrect. (t - s) z + (s- t ) z + (r - r) = 0 (2)
40. (A, C)
Equation (1) and (2) represent set of lines.
For equation (1) and (2) to have unique solution, we have:
1 t+ s s+ t
¹
2 t -s s-t
Q (0, 0) On solving the above equation we get
|t|¹|s|
\ Option (A) is correct
For equation (1) and (2) to have infinitely many solutions,
1 we have :
Let the equation of common tangent is : y = mx +
m t+ s t +s r + r
= = Þ |t|=|s|
1 t -s s- t r - r
0+0+
m = 1 and tr - t r + sr - sr = sr + s r - t r - t r
\
1 + m2 2 Þ 2 tr = 2s r Þ tr = s r
\ | t || r |=| s || r |
Þ m4 + m2 – 2 = 0 Þ m = ±1
\ Equation of common tangents are Þ | t || r |=| s || r | Þ | t |=| s |
\ If | t | = | s |, lines will be parallel for sure but may not be
y = x + 1 and y = - x - 1 coincident (i.e., does not have infinitely many solutions).
\ Q º (–1, 0) (C) : Locus of Z is a null set or singleton set or a line, in all
x 2 y2 threse cases it will intersect given circle at most two points.
\ Equation of ellipse is : + =1 (D) : in this case locus of Z is a line so L has infinte elements.
1 1/ 2 42. (B, C, D)
1 1 f (x)sin t - f (t)sin x
(A) e = 1 - = lim = sin 2 x
2 2 t ®x t-x
2 f (x) cos t - f ¢(t)sinx
æ 1 ö Þ lim = sin 2 x
2ç ÷ 2 t ®x 1
2b
= è

and latus rectum = =1 (Using L’Hospital’s Rule)
a 1
EBD_7809
2018 - 38 JEE Advanced 2018 Solved Paper

Þ f (x) cos x - f ¢(x)sinx = sin 2 x 1/3


(1 + 3) dt - (1 + 3) -2
1/3

æ f ¢(x)sin x - f (x)cos x ö
\ I= ò -2t 6/4
=
2
´
t 1
Þ -ç ÷ =1 1
è sin 2 x ø
(
= 1+ 3 )( )
3 -1 = 2
æ f (x) ö f (x)
Þ -d ç ÷ =1 Þ = -x + c
è sin x ø sin x é a1 a 2 a3 ù
p p p 44. (4) Suppose P = êê b1 b2 b3 úú
Now, Put x = also it is given that f æç ö÷ = -
6 6
è ø 12 êë c1 c2 c3 úû
-p a1 a2 a3
p -p -p
\ 12 = - + c Þ = +c So, det (P) = b1 b2 b3
1 6 12 12
c1 c2 c3
2
Þ c = 0 Þ f(x) = – x sin x = a1 (b2c3 – b3c2) – a2 (b1c3 – b3c1) + a3 (b1c2 – b2c1)
æ p ö -p 1 Maximum value can be 6 when a1 = 1, a2 = –1, a3 = 1 and
(A) f ç ÷ =
è4ø 4 2 b2c3 = b1c3 = b1c2 = 1 & b3c2 = b3c1 = b2c1 = –1.
(B) f(x) = –x sin x So, (b2c3) (b3c1) (b1c2) = – 1
x3 and (b1c3) (b3c2) (b2c1) = 1.
as sin x > x - "x Î (0, p)
6 Therefore b1b2b3c1c2c3 has two values 1 and –1
which is not possible.
x4
\- x sin < - x 2 + Contradiction also occurs if a1 = 1, a2 = 1, a3 = 1 and
6 b2c2 = b3c1 = b1c2 = 1 & b3c2 = b1c3 = b1c2 = –1.
4 For maximum value to be 5 one of the terms should be
\ f (x) < - x 2 + x "x Î (0, p) zero but this will make 2 terms zero therefore answer
6
should not be 5.
(C) f ¢(x) = - sin x - x cos x
1 0 1
f ¢(x) = 0 Þ tan x = - x
As -1 1 1 = 4.
Þ there exist a Î (0, p) for which f ¢(a)=0
-1 -1 1
Hence maximum value of the determinant of P = 4.
45. (119) Here n (X) = 5 and n (Y) = 7
Number of one-one function = a = 7C5 × 5!
and Number of onto function Y to X is given as:

a1
(D) Here, f ¢¢(x) = -2cos x + x sin x
a2 b2
\ f ¢¢ æç p ö÷ = p and f æç p ö÷ = - p . .
è2ø 2 è2ø 2 . .
æpö æpö . .
Þ f ¢¢ ç ÷ + f ç ÷ = 0
è2ø è2ø a7 b5
1 1, 1, 1, 1, 3 1, 1, 1, 2, 2
2
(1 + 3) dx
43. (2) Let I = ò é(1 + x)2 (1 - x)6 ù
1/4 b=
7!
3! 4!
× 5! +
7!
(2!)3 3!
× 5! = (7C3 + 3 × 7C3) 5!
0 ë û
= 4 × 7C3 × 5!
1
2 b-a
(1 + 3) dx Þ = 4 × 7C3 – 7C5 = 4 × 35 – 21 = 119
= ò é (1 - x)6 ù
1/4 5!
0
(1 + x)2 ê ú dy æ 2ö æ 2ö
6 46. (0.4) = (5y + 2) (5y – 2) = 25 ç y + ÷ çy - ÷
ëê (1+ x) ûú dx è 5ø è 5ø
1- x -2dx 1 dy
Now put
1+ x
=t Þ
(1 + x) 2
= dt Þ
25 ò æ y + 2 ö æ y - 2 ö = ò dx
ç ÷ç ÷
è 5øè 5ø
JEE Advanced 2018 Solved Paper 2018 - 39
49. (0.5) y
1 5é 1 1 ù
Þ
25 ò -
4 ê y- 2 y+ 2 ú
ê ú
dy = ò dx
Q (0, 1, 0)
ë 5 5û

2
y- T (1, 1, 1)
1 5 5 = x + c, where c is constant of
Þ ´ ln . S ( 12 , 12 , 12 )
25 4 2
y+
5 O (0, 0, 0) P (1, 0, 0) x
integration.
1 5y - 2 R
Þ ln =x +c (0, 0, 1)
20 5y + 2
Z
As, f(0) = 0 Þ at x = 0, y = 0 Þ 0 = 0 + c Þ c = 0 r uur ˆi - ˆj - kˆ
Here, p = SP =
5y - 2 2
Therefore, = e20x
5y + 2 r uuur - ˆi + ˆj - kˆ
q = SQ =
5 f (x) - 2 2
Þ lim = lim e 20x = e–¥ = 0 uuu
r
x ® -¥ 5 f (x) + 2 x ® -¥ r - i - ˆj + kˆ
ˆ
r = SR =
2
2
Þ 5 lim f(x) – 2 = 0 Þ lim f(x) = = 0.4 r uur ˆ
i + j + kˆ
ˆ
x ® -¥ x ® -¥ 5 t = ST =
2
47. (2) f (x + y) = f (x) f ¢ (y) + f ¢ (x) f (y) (1)
After putting x = y = 0, we get ˆi ˆj kˆ
r r 1 2iˆ + 2ˆj ˆi + ˆj
1 So, p ´ q = 1 -1 -1 = =
f (0) = 2f ¢ (0) f (0) Þ f ¢ (0) = [Q f (0) = 1] 4 4 2
2 -1 1 -1
Now putting y = 0 in equation (1), we get
ˆi ˆj kˆ
f (x) = f (x) f ¢ (0) + f ¢ (x) f (0)
r r 1 - 2iˆ + 2ˆj -ˆi + ˆj
f (x) f ¢ (x) 1 and, r ´ t = -1 -1 1 = =
Þ f ¢(x) =
2
Þ ò f (x)
dx =
2 ò dx 4
1 1 1
4 2

[Q f(0) = 1 and f ¢(0) = 1 ] ˆi ˆj kˆ


2 r r r r 1 kˆ
Hence, (p ´ q) ´ (r ´ t ) = 1 1 0 =
x 4 2
Þ loge f (x) = + loge c -1 1 0
2
Þ f (x) = cex/2 Þ f (x) = ex/2 (Q f (0) = 1) r r r r 1
Þ (p ´ q) ´ (r ´ t ) = = 0.5
x 2
Þ loge (f (x)) = Þ loge (f (4)) = 2 50. (646)
2
48. (8) Suppose coordinates of P are (a, b, c). n n
So, coordinates of Q are (0, 0, c) and coordinates of R å r(n Cr )2 = n å n Cr n -1Cr -1
are (a, b, –c). r =0 r =0
Here, PQ is perpendicular to the plane x + y = 3. n
So, PQ is parallel to the normal of given plane = n å n Cn - r n -1Cr -1 = n2n–1C
n–1
r =1
i.e. (aiˆ + bj)
ˆ is parallel to (iˆ + ˆj) So, X = (10C1)2 + 2(10C2)2 + 3(10C3)2 + ...+10(10C10)2
Þ a=b 10
As mid-point of PQ lies in the plane x + y = 3, so = å r(10 Cr ) 2 = 1019 C9
n =0
a b
+ = 3Þa +b= 6Þa= 3=b X 1 19
2 2 Hence, = C9 = 646
1430 143
Therefore, distance of P from the x-axis = b2 + c 2 = 5 x
51. (A) For E1, > 0 and x ¹ 1 Þ x Î ( -¥,0) È (1, ¥)
(given) x -1
Þ b2 + c2 = 25 Þ c2 = 25 – 9 = 16 æ x ö 1 x
Þ c=±4 For E 2 , - 1 £ log e ç ÷ £1 Þ £ £e
Hence, PR = |2c| = 8 è x -1 ø e x -1
1 1
Þ £ 1+ £e
e x -1
EBD_7809
2018 - 40 JEE Advanced 2018 Solved Paper
1 1 æ e ù é 1 ö 2
-1 £ £ e - 1 Þ (x - 1) Î ç -¥, È ,¥ Þ The eccentricity of H= e = and
e x -1 è 1 - e ûú ëê e - 1 ø÷ 3
The distance between the foci of H = 2ae
æ 1 ù é e ö
Þ x Î ç -¥, ú Èê ,¥ ÷ 2
è e - 1 û ë e - 1 ø = 2´ 2 3 ´ =8
3
x
For E1, Î (0, ¥) - {1} 2b2 2 ´ 4 4
x -1 and length of latus ractum of H = = =
æ x ö a 2 3 3
Þ log e ç ÷ Î ( -¥, ¥ ) - {0} 54. (D)
è x -1ø
é 2 ù
Þ f (x) Î (-¥,0) È (0, ¥ ) sin 1 - e - h - 0 ú
(i) f '1 (0) = lim ê
æ æ x öö é p p ù h ®0 ê h ú
g(x) = sin -1 ç loge ç ÷ ÷ Î - , - {0} ëê úû
è è x - 1 ø ø ëê 2 2 ûú
52. (C) Here, a committee has to be formed form a group of 6 é 2 2 ù
sin 1 - e - h sin 1 - e- h hú
Boys and 5 girls. = lim ê ´ ´
Total number of ways for selecting exactly 3 boys and h ®0 ê -h 2 h2 hú
2 girls = 6C3 × 5C2 = 20 × 10 = 200 = a1. ëê 1 - e úû
Total number of ways for selecting at least 2 members é hù h é sin x ù
with equal number of boys and girls = lim ê1´ 1´ ú = lim
h ®0 ë h û h ®0 h êëQ xlim
®0 x
= 1ú
û
= (6C1 × 5C1) + (6C2 × 5C2) + (6C3 × 5C3 ) + (6C4 × 5C4)
+ (6C5 × 5C5) = 11C5 – 1 = 461 = a2. which does not exit.
Total number of ways for selecting 5 members having So for (P), (2) is correct.
at least 2 girls = 11C5 – 6C5 – 6C4 × 5C1 = 11C5 – 81 = 381 é sin x ù
= a3 (ii) lim f 2 (x) = lim ê ú
x ®0 x ®0 ë tan –1 x û
Total number of ways for selecting 4 memebrs having
at least 2 girls M1 and G1 are not selected together é sin x x xù
= n(M1 selected & G1 not selected) + n(G1 selected & lim ê ´ ´ ú
x ®0 ë x –1 x
M1 not selected) + n(M1 and G1 both not selected) tan x û
= (4C2 × 5C1 + 4C3) + (4C1 × 5C2 + 4C2 × 5C1+ 4C3) é xù x é x ù
+(4C4 + 4C3 × 5C1 + 4C2× 5C2) lim ê1´1´ ú = lim êQ lim = 1ú
x x - 1
x ®0 ë û x ®0 ë x ®¥ tan x û
= 34 + 4 + 81 = 189 = a 4 which does not exist, so for Q, (1) is correct.
53. (B) y
(iii) lim f3 (x) = lim [sin(log e (x + 2))]
x ®0 x ®0
L (0, b)
if x ® 0 Þ (x + 2) ® 2
Þ log e (x + 2) ® log e 2 < 1
90°
30° N Þ 0 < lim sin(log e (x + 2) < sin1
x x ®0
(–a, 0) O 30° (a, 0)
Þ lim [sin(log e (x + 2))] = 0
x®0

f3 (x) = 0 "x Î [ -1, e p /2 - 2)


M (0, –b)
Þ f '3 (x) = 0 "x Î ( -1, ep /2 - 2)
Area of DLMN = 4 3 (given)
Þ f ''3 (x) = 0 "x Î ( -1, ep /2 - 2)
1
Þ ´ LM ´ ON = 4 3 So for (R), (4) is correct.
2
æ 2 1ö 2æ 1ö
1
Þ (2b)( 3 b) = 4 3 (iv) lim f 4 (x) = lim ç x sin ÷ = lim x ç sin ÷ = 0
x ®0 x ®0 è x ø x ®0 è xø
2
æ1ö
Þ b2 = 4 Þ b = 2 h 2 sin ç ÷ - 0
So, length of the conjugate axis of H = 2b = 4 èhø æ1ö
f '4 (0) = lim = lim h sin ç ÷ = 0
OL b x ®0 h x ®0 è4ø
tan 30° = = Þ a = 3b Þ a = 2 3
ON a 1 1
f '4 (x) = - cos + 2x sin , x ¹ 0
\ b2 = a2 (e2 – 1) x x
Þ 4 = 12(e2–1)
é 1 1ù 1
1 4 lim f 4¢ (x) = lim ê - cos + 2x sin ú = - lim cos
Þ e2 = 1 + = x ®0 x ®0 ë x x û x ®0 x
3 3 which does not exist
So for (S), (3) is correct.
JEE ADVANCED 2017
1. The question paper consists of three parts (Physics, Chemistry and Mathematics). Each part consists of three
sections.
2. Section I contains 7 multiple choice questions. Each question has four choices (a), (b), (c) and (d) out of which
ONE OR MORE THAN ONE is/are correct.
3. Section II contains 5 questions. The answer to each of the questions is a single-digit integer ranging from 0 to 9
(both inclusive).
4. Section III contains 6 questions of Matching type, contains two tables each having 3 columns and 4 rows. Each
question has four choices (a), (b), (c) and (d). ONLY ONE of these four options is correct.

PAPER - 1

PHYSICS mR
(A) The position of the point mass m is: x = - 2
M+m
SECTION - I
2gR
This section contains 7 questions. Each question has 4 options m
(A), (B), (C) and (D). ONE or MORE THAN ONE of these four (B) The velocity of the point mass m is: v = 1+
M
options is (are) correct.
(C) The x component of displacement of the center of mass
1. A flat plate is moving normal to its plane through a gas
under the action of a constant force F. The gas is kept at a mR
of the block M is: -
very low pressure. The speed of the plate v is much less M+m
than the average speed u of the gas molecules.
Which of the following options is/are true? m
(D) The velocity of the block M is: V = - 2gR
(A) The pressure difference between the leading and M
trailing faces of the plate is proportional to uv 3. A block M hangs vertically at the bottom end of a uniform
(B) The resistive force experienced by the plate is rope of constant mass per unit length. The top end of the
proportional to v rope is attached to a fixed rigid support at O. A transverse
(C) The plate will continue to move with constant non- wave pulse (Pulse 1) of wavelength l0 is produced at point
zero acceleration, at all times
O on the rope. The pulse takes time TOA to reach point A. If
(D) At a later time the external force F balances the resistive
force the wave pulse of wavelength l0 is produced at point A
2. A block of mass M has a circular cut with a frictionless (Pulse 2) without disturbing the position of M it takes time
surface as shown. The block rests on the horizontal TAO to reach point O. Which of the following options is/are
frictionless surface of a fixed table. Initially the right edge of correct?
the block is at x = 0, in a co-ordinate system fixed to the table.
A point mass m is released from rest at the topmost point of
the path as shown and it slides down. O Pulse 1
When the mass loses contact with the block, its position is
x and the velocity is v. At that instant, which of the following
options is/are correct?

R
Pulse 2
A M
m
R
y (A) The time TAO = TOA
(B) The velocities of the two pulses (Pulse 1 and Pulse 2)
are the same at the midpoint of rope
x M (C) The wavelength of Pulse 1 becomes longer when it
reaches point A
x=0 (D) The velocity of any pulse along the rope is
independent of its frequency and wavelength
EBD_7809
2017-2 JEE Advanced 2017 Solved Paper
4. A human body has a surface area of approximately 1 m2. The 6. In the circuit shown, L = 1 mH, C = 1 mF and R = 1 kW. They
normal body temperature is 10 K above the surrounding are connected in series with an a.c. source V = V0 sin wt as
room temperature T0. Take the room temperature to be shown. Which of the following options is/are correct?
T0 = 300 K. For T0 = 300 K, the value of sT04 = 460 Wm-2 L = 1mH C = 1mF R = 1kW
(where s is the Stefan-Boltzmann constant). Which of the
following options is/are correct?
(A) The amount of energy radiated by the body in 1 second ~ V0 sin wt
is close to 60 joules
(B) If the surrounding temperature reduces by a small
amount DT0 << T0, then to maintain the same body (A) The current will be in phase with the voltage if w = 104
temperature the same (living) human being needs to rad.s-1
(B) The frequency at which the current will be in phase
radiate DW = 4sT03 DT0 more energy per unit time with the voltage is independent of R
(C) Reducing the exposed surface area of the body (e.g. (C) At w ~ 0 the current flowing through the circuit
by curling up) allows humans to maintain the same becomes nearly zero
body temperature while reducing the energy lost by (D) At w >> 106 rad. s-1, the circuit behaves like a capacitor
radiation 7. For an isosceles prism of angle A and refractive index m, it is
(D) If the body temperature rises significantly then the found that the angle of minimum deviation dm= A.
peak in the spectrum of electromagnetic radiation Which of the following options is/are correct?
emitted by the body would shift to longer wavelengths (A) For the angle of incidence i1 = A, the ray inside the
5. A circular insulated copper wire loop is twisted to form two prism is parallel to the base of the prism
loops of area A and 2A as shown in the figure. At the point (B) For this prism, the refractive index µ and the angle of
of crossing the wires remain electrically insulated from each 1 -1 æ m ö
other. The entire loop lies in the plane (of the paper). A uniform prism A are related as A = cos ç ÷
2 è2ø
®
magnetic field B points into the plane of the paper. At t = 0, (C) At minimum deviation, the incident angle i1 and the
the loop starts rotating about the common diameter as axis refracting angle r 1 at the first refracting surface are
with a constant angular velocity w in the magnetic field. related by r1 = (i1/2)
Which of the following options is/are correct? (D) For this prism, the emergent ray at the second surface
will be tangential to the surface when the angle of
incidence at the first surface is
× × × × × × × ×
B -1 é 2 A ù
× × × × × × × × i= - 1 - cos A ú
1 sin êsin A 4 cos
× × × × × × × × ë 2 û
× × × ×area
× A× × ×
× × × × × × × × SECTION - II
× × × × × × × × This section contains 5 questions. The answer to each question
× × × × × × × × is a SINGLE DIGIT INTEGER ranging from 0 to 9, both inclusive.
× × × × × × × ×
area 2A 8. A drop of liquid of radius R = 10–2 m having surface tension
× × × × × × × ×
× × × × × × × × 0.1
× × × × × × × × S= Nm-1 divides itself into K identical drops. In this
4p
× × × × × × × × process the total change in the surface energy DU = 10–3 J.
w
If K = 10a then the value of a is
9. An electron in a hydrogen atom undergoes a transition from
an orbit with quantum number ni to another with quantum
(A) The emf induced in the loop is proportional to the sum number nf. Vi and Vf are respectively the initial and final
of the areas of the two loops
(B) The amplitude of the maximum net emf induced due to Vi
potential energies of the electron. If V = 6.25, then the
both the loops is equal to the amplitude of maximum f
emf induced in the smaller loop alone smallest possible nf is
(C) The net emf induced due to both the loops is 10. A monochromatic light is travelling in a medium of refractive
proportional to cos wt index n = 1.6. It enters a stack of glass layers from the bottom
(D) The rate of change of the flux is maximum when the side at an angle q = 30°. The interfaces of the glass layers are
plane of the loops is perpendicular to plane of the parallel to each other. The refractive indices of different glass
paper
JEE Advanced 2017 Solved Paper 2017-3

layers are monotonically decreasing as nm= n – mDn, where 11. A stationary source emits sound of frequency f0 = 492 Hz.
nm is the refractive index of the mth slab and Dn = 0.1 (see The sound is reflected by a large car approaching the source
the figure). The ray is refracted out parallel to the interface with a speed of 2 ms–1. The reflected signal is received by
between the (m – 1)th and mth slabs from the right side of the the source and superposed with the original.
stack. What is the value of m? What will be the beat frequency of the resulting signal in
Hz? (Given that the speed of sound in air is 330 ms–1 and the
m n – mDn car reflects the sound at the frequency it has received).
m–1 n – (m – 1)Dn 12. 131I is an isotope of Iodine that B decays to an isotope of

Xenon with a half-life of 8 days. A small amount of a serum


labelled with 131I is injected into the blood of a person. The
activity of the amount of 131I injected was 2.4 × 105 Becquerel
3 n – 3Dn (Bq). It is known that the injected serum will get distributed
2 n – 2Dn uniformly in the blood stream in less than half an hour. After
1 n – Dn 11.5 hours, 2.5 ml of blood is drawn from person's body, and
n gives an activity of 115 Bq. The total volume of blood in the
q
person¡¦s body, in liters is approximately
(you may use ex » 1 + x for |x| < < 1 and ln2 » 0.7).

SECTION - III
This section contains 6 questions of MATCHING TYPE, contains two tables each having 3 columns and 4 rows. Based on each table,
there are three questions. Each question has four options (A), (B), (C) and (D) ONLY ONE of these four options is correct.
Answer (Qs. 13-15) : By appropriately matching the information given in the three columns of the following table.
®
A charged particle (electron or proton) is introduced at the origin (x = 0, y = 0, z = 0) with a given initial velocity v . A uniform
® ® ® ® ®
electric field E and a uniform magnetic field B exist everywhere. The velocity v, electric field E and magnetic field B are given in
columns 1, 2 and 3, respectively. The quantities E0, B0 are positive in magnitude.
Column 1 Column 2 Column 3
® E0 ® ®
(I) Electron with v = 2 xˆ (i) E = E 0 zˆ (P) B = - B xˆ
B0 0

® E0 ® ®
(II) Electron with v = yˆ (ii) E = - E 0 yˆ (Q) B = B0 xˆ
B0

® ® ®
(III) Proton with v = 0 (iii) E = - E 0 xˆ (R) B = B0 yˆ

® E0 ® ®
(IV) Proton with v = 2 xˆ (iv) E = E 0 xˆ (S) B = B0 zˆ
B0

13. In which case will the particle move in a straight line with constant velocity?
(A) (III) (ii) (R) (B) (IV) (i) (S) (C) (III) (iii) (P) (D) (II) (iii) (S)
14. In which case will the particle describe a helical path with axis along the positive z direction?
(A) (IV) (i) (S) (B) (II) (ii) (R) (C) (III) (iii) (P) (D) (IV) (ii) (R)
15. In which case would the particle move in a straight line along the negative direction of y-axis (i.e., move along – ŷ )?
(A) (II) (iii) (Q) (B) (III) (ii) (R) (C) (IV) (ii) (S) (D) (III) (ii) (P)

Answer (Qs. 16-18) : By appropriately matching the information given in the three columns of the following table.
An ideal gas is undergoing a cyclic thermodynamic process in different ways as shown in the corresponding P–V diagrams in
column 3 of the table. Consider only the path from state 1 to state 2. W denotes the corresponding work done on the system. The
equations and plots in the table have standard notations as used in thermodynamic processes. Here Y is the ratio of heat capacities at
constant pressure and constant volume. The number of moles in the gas is n.
EBD_7809
2017-4 JEE Advanced 2017 Solved Paper
Column 1 Column 2 Column 3
1
(I) W1®2 = (P2 V2 - P1V1 ) (i) Isothermal (P) P 1
g -1 2

(II) W1®2 = -PV2 + PV1 (ii) Isochoric (Q) P


1

P 1 2

(III) W1®2 = 0 (iii) Isobaric (R)

P
1

æV ö
(IV) W1®2 = -nRT ln ç 2 ÷ (iv) Adiabatic (S)
è V1 ø 2

16. Which of the following options is the only correct representation of a process in which DU = DQ – PDV?
(A) (II) (iv) (R) (B) (III) (iii) (P) (C) (II) (iii) (S) (D) (II) (iii) (P)
17. Which one of the following options is the correct combination?
(A) (IV) (ii) (S) (B) (III) (ii) (S) (C) (II) (iv) (P) (D) (II) (iv) (R)
18. Which one of the following options correctly represents a thermodynamic process that is used as a correction in the determination
of the speed of sound in an ideal gas?
(A) (I) (ii) (Q) (B) (IV) (ii) (R) (C) (III) (iv) (R) (D) (I) (iv) (Q)
JEE Advanced 2017 Solved Paper 2017-5

CHEMISTRY 22. The colour of the X2 molecules of group 17 elements changes


gradually from yellow to violet down the group. This is due
to
SECTION - I (A) The physical state of X2 at room temperature changes
This section contains 7 questions. Each question has 4 options from gas to solid down the group
(A), (B), (C) and (D). ONE or MORE THAN ONE of these four (B) Decrease in ionization energy down the group
options is (are) correct. (C) Decrease in p* - s* gap down the group
(D) Decrease in HOMO-LUMO gap down the group
19. An ideal gas is expanded from (p1, V1, T1) to (p2, V2, T2) 23. Addition of excess aqueous ammonia to a pink coloured
under different conditions. The correct statement(s) among aqueous solution of MCl2.6H2O(X) and NH4Cl gives an
the following is(are) octahedral complex Y in the presence of air. In aqueous
(A) The work done on the gas is maximum when it is solution, complex Y behaves as 1:3 electrolyte. The reaction
compressed irreversibly from (p2, V2) to (p1, V1 ) of X with excess HCl at room temperature results in the
against constant pressure p1 formation of a blue coloured complex Z. The calculated spin
(B) If the expansion is carried out freely, it is simultaneously only magnetic moment of X and Z is 3.87 B.M., whereas it is
both isothermal as well as adiabatic zero for complex Y. Among the following options, which
(C) The work done by the gas is less when it is expanded statement(s) is(are) correct?
reversibly from V1 to V2 under adiabatic conditions as (A) Addition of silver nitrate to Y gives only two
compared to that when expanded reversibly from V1 equivalents of silver chloride
to V2 under isothermal conditions (B) The hybridization of the central metal ion in Y is d2sp3
(D) The change in internal energy of the gas is (i) zero, if it (C) Z is a tetrahedral complex
is expanded reversibly with T1 = T2, and (ii) positive, if (D) When X and Z are in equilibrium at 0°C, the colour of
it is expanded reversibly under adiabatic conditions the solution is pink
with T1 ¹ T2 24. The IUPAC name(s) of the following compound is(are)
20. For a solution formed by mixing liquids L and M, the vapour
pressure of L plotted against the mole fraction of M in
solution is shown in the following figure. Here xL and xM H3C Cl
represent mole fractions of L and M, respectively, in the
solution. The correct statement(s) applicable to this system (A) 1-chloro-4-methylbenzene
is (are) (B) 4-chlorotoluene
(C) 4-methylchlorobenzene
(D) 1-methyl-4-chlorobenzene
Z 25. The correct statement(s) for the following addition reactions
is(are)
pL
H3C H
Br 2/CHCl 3
M and N
(i)
H CH3
1 xM 0
H3C CH3
(A) The point Z represents vapour pressure of pure liquid Br 2/CHCl 3
(ii) O and P
M and Raoult's law is obeyed from xL= 0 to xL= 1
(B) The point Z represents vapour pressure of pure liquid H H
L and Raoult's law is obeyed when xL ® 1 (A) O and P are identical molecules
(C) The point Z represents vapour pressure of pure liquid (B) (M and O) and (N and P) are two pairs of diastereomers
M and Raoult's law is obeyed when xL ® 0 (C) (M and O) and (N and P) are two pairs of enantiomers
(D) Attractive intermolecular interactions between L-L in
(D) Bromination proceeds through trans-addition in both
pure liquid L and M-M in pure liquid M are stronger
the reactions
than those between L-M when mixed in solution.
21. The correct statement(s) about the oxoacids, HClO4 and SECTION - II
HClO, is(are)
(A) The central atom in both HClO4 and HClO is sp3 This section contains 5 questions. The answer to each question
hybridized is a SINGLE DIGIT INTEGER ranging from 0 to 9, both inclusive.
(B) HClO4 is more acidic than HClO because of the 26. A crystalline solid of a pure substance has a face-centred
resonance stabilization of its anion cubic structure with a cell edge of 400 pm. If the density of
(C) HClO4 is formed in the reaction between Cl2 and H2O the substance in the crystal is 8 g cm-3, then the number of
(D) The conjugate base of HClO4 is weaker base than H2O
atoms present in 256 g of the crystal is N × 1024. The value
of N is
EBD_7809
2017-6 JEE Advanced 2017 Solved Paper
27. The conductance of a 0.0015 M aqueous solution of a weak (Atomic numbers: H = 1, He = 2, Li = 3, Be = 4, B = 5, C = 6,
monobasic acid was determined by using a conductivity N = 7, O = 8, F = 9)
cell consisting of platinized Pt electrodes. The distance 30. Among the following, the number of aromatic compound(s)
between the electrodes is 120 cm with an area of cross section is
of 1 cm2. The conductance of this solution was found to be
5 × 10–7S. The pH of the solution is 4. The value of limiting
o
( )
molar conductivity L m of this weak monobasic acid in
aqueous solution is Z × 102 S cm-1 mol-1 . The value of Z is
28. The sum of the number of lone pairs of electrons on each
central atom in the following species is
[TeBr6]2-, [BrF2]+, SNF3 and [XeF3]–
(Atomic numbers: N = 7, F = 9, S = 16, Br = 35, Te = 52,
Xe = 54)
29. Among H2, He2+, Li2, Be2, B2, C2, N2,O2– and F2, the number
of diamagnetic species is

SECTION - III
This section contains 6 questions of MATCHING TYPE, contains two tables each having 3 columns and 4 rows. Based on each table,
there are three questions. Each question has four options (A), (B), (C) and (D) ONLY ONE of these four option is correct.
(Qs. 31-33) : By appropriately matching the information given in the three columns of the following table.
The wave function, yn,l,ml is a mathematical function whose value depends upon spherical polar coordinates (r, q, f) of the
electron and characterized by the quantum numbers n, l and ml. Here r is distance from nucleus, q is colatitude and f is azimuth. In the
mathematical functions given in the table, Z is atomic number and a0 is Bohr radius.
Column 1 Column 2 Column 3

3 æ Zr ö
æ Z ö 2 -ç a ÷
yn,l,ml,(r)

(i) 1s orbital (i) y n,l,ml µ ç ÷ e è 0 ø (P)


è a0 ø 0
r
a0

1
(ii) 2s orbital (ii) One radial node (Q) Probability density at nucleus µ
a 30

5 æ Zr ö
æ Z ö 2 -ç 2a ÷
(iii) 2pz orbital (iii) y n,l,ml µ ç ÷ re è 0 ø cos q (R) Probability density is maximum at nucleus
è a0 ø

(iv) 3d orbital (iv) xy-plane is a nodal plane (S) Energy needed to excite electron from n = 2
z2

27
state to n = 4 state is
times the energy
32
needed to excite electron from n = 2 state to
n = 6 state
31. For the given orbital in Column 1, the only CORRECT combination for any hydrogen-like species is
(A) (I) (ii) (S)
(B) (IV) (iv) (R)
(C) (II) (ii) (P)
(D) (III) (iii) (P)
JEE Advanced 2017 Solved Paper 2017-7

32. For hydrogen atom, the only CORRECT combination is 39. Let [x] be the greatest integer less than or equals to x. Then,
(A) (I) (i) (S) at which of the following point(s) the function
f(x) = x cos(p(x + [x])) is discontinuous?
(B) (II) (i) (Q)
(A) x = – 1 (B) x = 0
(C) (I) (i) (P) (C) x = 1 (D) x = 2
(D) (I) (iv) (R) 40. Let f : R ® (0,1) be a continuous function. Then, which of
33. For He+ ion, the only INCORRECT combination is the following function(s) has(have) the value zero at some
point in the interval (0, 1)?
(A) (I) (i) (R)
(A) x9 – f(x)
(B) (II) (ii) (Q)
p
(C) (I) (iii) (R) -x
(B) x-ò2 f (t) cos t dt
(D) (I) (i) (S) 0

(Qs. 34-36) : By appropriately matching the information given in x


(C) ex - ò f (t) sin t dt
the three columns of the following table. 0
Columns 1, 2 and 3 contain starting materials, reaction
p
conditions, and type of reactions, respectively.
(D) f (x) + ò 2 f (t) sin t dt
0
Column 1 Column 2 Column 3
(I) Toluene (i) NaOH/Br2 (P) Condensation 41. Which of the following is(are) not the square of a 3 × 3 matrix
(II) Acetophenone (ii) Br 2/hv (Q) Carboxylation with real entries?
(III) Benzaldehyde (iii) (CH3CO)2O/ (R) Substitution
é1 0 0ù é1 0 0 ù
CH3COOK ê0 1 0ú ê0 1 0 ú
(IV) Phenol (iv) NaOH/CO2 (S) Haloform (A) ê ú (B) ê ú
êë0 0 1 úû êë 0 0 -1úû
34. For the synthesis of benzoic acid, the only CORRECT
combination is
(A) (II) (i) (S) (B) (IV) (ii) (P) é1 0 0 ù é -1 0 0 ù
(C) (I) (iv) (Q) (D) (III) (iv) (R) ê 0 -1 0 ú ê 0 -1 0 ú
(C) ê ú (D) ê ú
35. The only CORRECT combination that gives two different êë 0 0 -1úû êë 0 0 -1úû
carboxylic acids is
(A) (II) (iv) (R) (B) (IV) (iii) (Q) 42. Let a, b, x and y be real numbers such that a – b = 1 and
(C) (III) (iii) (P) (D) (I) (i) (S) y ¹ 0. If the complex number z = x + iy satisfies
36. The only CORRECT combination in which the reaction
proceeds through radical mechanism is æ az + b ö
Im ç
è z + 1 ÷ø
= y, then which of the following is(are) possible
(A) (III) (ii) (P) (B) (IV) (i) (Q)
(C) (II) (iii) (R) (D) (I) (ii) (R) value(s) of x ?

MATHEMATICS (A) -1 + 1 - y 2

SECTION - I (B) -1 - 1 - y 2
This section contains 7 questions. Each question has 4 options (C) 1 + 1 + y2
(A), (B), (C) and (D). ONE or MORE THAN ONE of these four
options is (are) correct.
(D) 1 - 1 + y2

x2 y2 1 1
37. If 2x – y + 1 = 0 is a tangent to the hyperbola - = 1, 43. Let X and Y be two events such that P(X) = , P(X|Y) =
a2 16 3 2
then which of the following cannot be sides of a right angled
triangle? 2
and P (Y|X) = . Then
(A) a, 4, 1 (B) a, 4, 2 5
(C) 2a, 8, 1 (D) 2a, 4, 1
4 1
38. If a chord, which is not a tangent, of the parabola y2 = 16x (A) P(Y) = (B) P(X¢ | Y) =
has the equation 2x + y = p, and midpoint (h, k), then which 15 2
of the following is(are) possible value(s) of p, h and k ? 1 2
(A) p = –2, h = 2, k = –4 (C) P(X Ç Y) = (D) P(X È Y) =
(B) p = –1, h = 1, k = –3 5 5
(C) p = 2, h = 3, k = –4
(D) p = 5, h = 4, k = –3
EBD_7809
2017-8 JEE Advanced 2017 Solved Paper
SECTION - II
é1 a a 2 ù é x ù é 1ù
This section contains 5 questions. The answer to each question ê ú
a 1 a ú êê y úú = êê - 1úú
is a SINGLE DIGIT INTEGER ranging from 0 to 9, both inclusive. 46. For a real number a, if the system ê
ê 2 ú
ëa a 1 û êë z úû êë 1úû
44. For how many values of p, the circle x2 + y2 + 2x + 4y – p = 0
and the coordinate axes have exactly three common points? of linear equations, has infinitely many solutions,
45. Let f : R ® R be a differentiable function such that f(0) = 0, then 1 + a + a2 =
47. Words of length 10 are formed using the letters A, B, C, D, E,
æpö F, G, H, I, J. Let x be the number of such words where no letter
f ç ÷ = 3 and f ¢(0) = 1.
è2ø is repeated; and let y be the number of such words where
exactly one letter is repeated twice and no other letter is
p
2
y
If g(x) = ò [f ¢(t)cosec t – cot t cosec t f(t)]dt for repeated. Then, =
x 9x
48. The sides of a right angled triangle are in arithmetic
æ pù progression. If the triangle has area 24, then what is the
x Î ç 0, ú , then lim g(x) =
è 2û x®0 length of its smallest side?

SECTION - III
This section contains 6 questions of matching type. This section contains two tables each having 3 columns and 4 rows. Based on
each table, there are three questions. Each question has four options (A), (B), (C) and (D) only ONE OF these four option is correct.
(Qs. 49-51) : By appropriately matching the information given in the three columns of the following table.
Column 1, 2, and 3 contain conics, equations of tangents to the conics and points of contact, respectively.
Column 1 Column 2 Column 3

æ a 2a ö
(I) x2 + y2 = a2 (i) my = m2x + a (P) ç 2, ÷
èm m ø

æ - ma a ö
(II) x2 + a2y2 = a2 (ii) y = mx + a (Q) ç , ÷
m2 + 1 ç 2 2 ÷
è m +1 m +1 ø

æ -a 2 m 1 ö
(III) y2 = 4ax (iii) y = mx + 2 2 (R) ç , ÷
a m -1 ç 2 2 2 2 ÷
è a m +1 a m +1 ø

æ -a 2 m -1 ö
(IV) x2 – a2y2 = a2 (iv) y = mx + 2 2 (S) ç , ÷
a m +1 ç 2 2 2 2 ÷
è a m -1 a m -1 ø

49. For a = 2 , if a tangent is drawn to a suitable conic (Column 1) at the point of contact (– 1, 1), then which of the following
options is the only correct combination for obtaining its equation?
(A) (I) (i) (P) (B) (I) (ii) (Q) (C) (II) (ii) (Q) (D) (III) (i) (P)
50. If a tangent to a suitable conic (column 1) is found to be y = x + 8 and its point of contact is (8, 16), then which of the following
options is the only correct combination?
(A) (I) (ii) (Q) (B) (II) (iv) (R) (C) (III) (i) (P) (D) (III) (ii) (Q)
JEE Advanced 2017 Solved Paper 2017-9

æ 1ö
51. The tangent to a suitable conic (Column 1) at ç 3, ÷ is found to be 3x + 2y = 4, then which of the following options is the
è 2ø
only correct combination?
(A) (IV) (iii) (S) (B) (IV) (iv) (S) (C) (II) (iii) (R) (D) (II) (iv) (R)

(Qs. 52-54) : By appropriately matching the information given in the three columns of the following table.
Let f(x) = x + loge x – x loge x, x Î (0, ¥)
Column 1 contains information about zeros of f(x), f ¢(x) and f ¢¢(x) .
Column 2 contains information about the limiting behaviour of f (x), f ¢(x) and f ¢¢(x) at infinity.
Column 3 contains information about increasing/decreasing nature of f (x) and f ¢(x) .
Column 1 Column 2 Column 3

(I) f (x) = 0 for some x Î (1, e2) (i) lim f (x) = 0 (P) f is increasing in (0, 1)
x®¥

(II) f ¢ (x) = 0 for some x Î (1, e) (ii) lim f (x) = -¥ (Q) f is increasing in (e, e2)
x®¥

(III) f ¢(x) = 0 for some x Î (0, 1) (iii) lim f ¢(x) = -¥ (R) f ¢ is increasing in (0, 1)
x ®¥

(IV) f ¢¢(x) = 0 for some x Î (1, e) (iv) lim f ¢¢(x) = 0 (S) f ¢ is decreasing in (e, e2)
x®¥

52. Which of the following options is the only correct combination?


(A) (I) (i) (P) (B) (II) (ii) (Q) (C) (III) (iii) (R) (D) (IV) (iv) (S)
53. Which of the following options is the only correct combination?
(A) (I) (ii) (R) (B) (II) (iii) (S) (C) (III) (iv) (P) (D) (IV) (i) (S)
54. Which of the following options is the only incorrect combination?
(A) (I) (iii) (P) (B) (II) (iv) (Q) (C) (III) (i) (R) (D) (II) (iii) (P)
EBD_7809
2017-10 JEE Advanced 2017 Solved Paper

PAPER - 2
1. The question paper consists of three parts (Physics, Chemistry and Mathematics). Each part consists of three
sections.
2. Section I contains 7 questions. Each question has four options (a), (b), (c) and (d). ONLY ONE of these four options
is correct.
3. Section II contains 7 multiple choice questions. Each question has four choice (a), (b), (c) and (d) out of which ONE
OR MORE THAN ONE is/are correct.
4. Section III contains 2 paragraphs each describing theory, experiment and data etc. 4 questions relate to two
paragraphs with two questions on each paragraph. Each question pertaining to a particular passage should have
ONLY ONE correct answer among the four given choices (a), (b), (c) and (d).

PHYSICS æ ö
ç 1 ÷
SECTION - I æpö D = hç - 1÷
(A) D = h sin 2 ç ÷ (B) ç æpö ÷
ènø ç cos ç n ÷ ÷
This section contains 7 questions. Each question has 4 options è è ø ø
(A), (B), (C) and (D). ONLY ONE of these four options is correct.
1. Consider an expanding sphere of instantaneous radius R æ 2 pö 2æ p ö
(C) D = hsin ç ÷ (D) D = h tan ç ÷
whose total mass remains constant. The expansion is such è nø è 2n ø
that the instantaneous density r remains uniform throughout 3. A photoelectric material having work–function f0 is
æ 1 drö æ hc ö
the volume. The rate of fractional change in density ç
è r dt ÷ø illuminated with light of wavelength l ç l < f ÷ . The fastest
è 0ø
is constant. The velocity v of any point on the surface of the photoelectron has a de–Broglie wavelength ld. A change in
expanding sphere is proportional to wavelength of the incident light by Dl result in a change Dld
(A) R (B) R3 in ld. Then the ratio Dld/Dl is proportional to
1 (A) ld/l (B) ld2 / l2
(C) (D) R2/3
R
2. Consider regular polygons with number of sides n = 3, 4, 5.... (C) l3d / l (D) l3d / l2
as shown in the figure. The center of mass of all the polygons 4. A symmetric star shaped conducting wire loop is carrying a
is at height h from the ground. They roll on a horizontal steady state current I as shown in the figure. The distance
surface about the leading vertex without slipping and sliding between the diametrically opposite vertices of the star is 4a.
as depicted. The maximum increase in height of the locus of The magnitude of the magnetic field at the center of the loop
the center of mass for each polygon is D. Then D depends is
on n and h as

4a
h h

m0 l m0 l
(A) 6 [ 3 - 1] (B) 6 [ 3 + 1]
4 pa 4 pa
h
m0 l m0l
(C) 3 [ 3 - 1] (D) 3 [2 - 3]
4 pa 4pa
JEE Advanced 2017 Solved Paper 2017-11
r r r
5. Three vectors P, Q and R are shown in the figure. Let S be
r
any point on the vector R . The distance between the points Region 1 y Region 2 Region 3
r r r × × ×
P and S is b | R |. The general relation among vectors P, Q
r × × B ×
and S is
× × ×
× × ×
Y P b |R|
O × × ×
× × × P2 x
P R = Q- P +Q P1
S × × ×
S (y = –R) ×
Q Q × ×
× × ×
O X 3 R/2
r r r
(A) S= (1 - b)P + bQ
r r r 2 p
(B) S = (b - 1)P + bQ (A) For B > , the particle will re–enter region 1
3 QR
r r r
(C) S = (1 - b2 )P + bQ
8 p
r r r (B) For B = , the particle will enter region 3 through
(D) S = (1 - b)P + b 2 Q 13 QR
6. A rocket is launched normal to the surface of the Earth, the point P2 on x–axis
away from the Sun, along the line joining the Sun and the (C) When the particle re–enters region 1 through the
Earth. The Sun is 3 × 105 times heavier than the Earth and is longest possible path in region 2, the magnitude of the
at a distance 2.5 × 104 times larger than the radius of the change in its linear momentum between point P1 and
Earth. The escape velocity from Earth's gravitational field is
ve = 11.2 km s–1. The minimum initial velocity (vs) required the farthest point from y–axis is p/ 2
for the rocket to be able to leave the Sun–Earth system is (D) For a fixed B, particles of same charge Q and same
closest to (Ignore the rotation and revolution of the Earth velocity v, the distance between the point P1 and the
and the presence of any other planet) point of re–entry into region 1 is inversely proportional
(A) vs= 22 km s–1 (B) vs= 42 km s–1 to the mass of the particle
(C) vs= 62 km s –1 (D) vs= 72 km s–1 9. The instantaneous voltages at three terminals marked X, Y
7. A person measures the depth of a well by measuring the and Z are given by
time interval between dropping a stone and receiving the Vx = V0 sin wt,
sound of impact with the bottom of the well. The error in his
measurement of time is dT = 0.01 seconds and he measures æ 2p ö
VY = V0 sin ç wt + ÷ and
the depth of the well to be L = 20 meters. Take the acceleration è 3 ø
due to gravity g = 10 ms–2 and the velocity of sound is 300
ms–1. Then the fractional error in the measurement, dL/L, is æ 4p ö
closest to VZ = V0 sin ç wt + ÷
è 3 ø
(A) 0.2% (B) 1%
An ideal voltmeter is configured to read rms value of the
(C) 3% (D) 5%
potential difference between its terminals. It is connected
SECTION - II between points X and Y and then between Y and Z. The
reading(s) of the voltmeter will be
This section contains 7 questions. Each question has 4 options
(A), (B), (C) and (D). ONE or MORE THAN ONE of these four rms 3
(A) VXY = V0
options is (are) correct. 2

8. A uniform magnetic field B exists in the region between x = 0 rms 1


(B) VYZ = V0
3R 2
and x = (region 2 in the figure) pointing normally into
2 rms
(C) VXY = V0
the plane of the paper. A particle with charge +Q and
momentum p directed along x–axis enters region 2 from region (D) Independent of the choice of the two terminals
1 at point P1 (y = –R). Which of the following option(s)
is/are correct?
EBD_7809
2017-12 JEE Advanced 2017 Solved Paper
10. A point charge +Q is placed just outside an imaginary
hemispherical surface of radius R as shown in the figure. S
Which of the following statements is/are correct?
R
Q + V
– L1 L2

R
(A) After a long time, the current through L1 will be
V L2
(A) The electric flux passing through the curved surface R L1 + L 2

æ Q 1 ö (B) After a long time, the current through L2 will be


of the hemisphere is - ç1 - ÷
è 2e0 2ø V L1
(B) Total flux through the curved and the flat surfaces is R L1 + L 2
Q (C) The ratio of the currents through L1 and L2 is fixed at
e0 all times (t > 0)
(C) The component of the electric field normal to the flat V
surface is constant over the surface (D) At t = 0, the current through the resistance R is
R
(D) The circumference of the flat surface is an equipotential
11. Two coherent monochromatic point sources S1 and S2 of 13. A rigid uniform bar AB of length L is slipping from its vertical
wavelength l = 600 nm are placed symmetrically on either position on a frictionless floor (as shown in the figure).
side of the centre of the circle as shown. The sources are At some instant of time, the angle made by the bar with the
separated by a distance d = 1.8 mm. This arrangement vertical is q. Which of the following statements about its
produces interference fringes visible as alternate bright and motion is/are correct?
dark spots on the circumference of the circle. The angular
separation between two consecutive bright spots is Dq.
Which of the following options is/are correct?
A
P1 q
L

Dq
B
P2
S1 S2 O
d
(A) The midpoint of the bar will fall vertically downward
(B) The trajectory of the point A is a parabola
(C) Instantaneous torque about the point in contact with
(A) A dark spot will be formed at the point P2 the floor is proportional to sinq
(B) At P2 the order of the fringe will be maximum (D) When the bar makes an angle q with the vertical, the
(C) The total number of fringes produced between P1 and displacement of its midpoint from the initial position is
P2 in the first quadrant is close to 3000 proportional to (1 – cosq)
(D) The angular separation between two consecutive 14. A wheel of radius R and mass M is placed at the bottom of a
bright spots decreases as we move from P1 to P2 along fixed step of height R as shown in the figure. A constant
the first quadrant
force is continuously applied on the surface of the wheel so
12. A source of constant voltage V is connected to a resistance
that it just climbs the step without slipping. Consider the
R and two ideal inductors L1 and L2 through a switch S as
shown. There is no mutual inductance between the two torque t about an axis normal to the plane of the paper passing
inductors. The switch S is initially open. At t = 0, the switch through the point Q. Which of the following options is/are
is closed and current begins to flow. Which of the following correct?
options is/are correct?
JEE Advanced 2017 Solved Paper 2017-13

S V
V0 Process 1

Q 2V0 /3
P Process 2
R V0 /3 T > > RC
X

T 2T t

Figure 2
(A) If the force is applied at point P tangentially then 15. In Process 1, the energy stored in the capacitor E C and heat
decreases continuously as the wheel climbs dissipated across resistance ED are related by :
(B) If the force is applied normal to the circumference at (A) EC = ED (B) EC = ED In 2
point X then t is constant 1
(C) If the force is applied normal to the circumference at (C) EC = ED (D) EC = 2ED
2
point P then t is zero
(D) If the force is applied tangentially at point S then t ¹ 0 16. In Process 2, total energy dissipated across the resistance
but the wheel never climbs the step ED is :
1 æ1 2ö
SECTION - III (A) ED = CV02 (B) ED = 3 ç CV0 ÷
2 è2 ø
This section contains 2 paragraphs, each describing theory,
experiments, data etc. four questions related to the two paragraphs 1æ 1 2ö
with two questions on each paragraph. Each question has only (C) ED = ç CV0 ÷ (D) ED = 3CV02
one correct answer among the four given options (A), (B), (C) and 3è 2 ø
(D). PARAGRAPH 2
PARAGRAPH 1 One twirls a circular ring (of mass M and radius R) near the tip of
Consider a simple RC circuit as shown in Figure 1. one's finger as shown in Figure 1. In the process the finger never
Process 1: In the circuit the switch S is closed at t = 0 and the loses contact with the inner rim of the ring. The finger traces out
capacitor is fully charged to voltage V0 (i.e., charging continues the surface of a cone, shown by the dotted line. The radius of the
for time T >> RC). In the process some dissipation (ED) occurs path traced out by the point where the ring and the finger is in
across the resistance R. The amount of energy finally stored in contact is r. The finger rotates with an angular velocity w0. The
the fully charged capacitor is EC. rotating ring rolls without slipping on the outside of a smaller
V0 circle described by the point where the ring and the finger is in
Process 2: In a different process the voltage is first set to and contact (Figure 2). The coefficient of friction between the ring and
3
the finger is m and the acceleration due to gravity is g.
maintained for a charging time T >> RC. Then the voltage is raised
2V0
to without discharging the capacitor and again maintained
3
for a time T >> RC. The process is repeated one more time by R
raising the voltage to V0 and the capacitor is charged to the same r
final voltage V0 as in Process 1. R
These two processes are depicted in Figure 2.
Figure 1
S
Figure 2
R
17. The total kinetic energy of the ring is
V + C
– 1
(A) Mw02 R 2 (B) Mw20 (R - r)2
2
3
Figure 1 (C) Mw20 (R - r) 2 (D) Mw20 (R - r)2
2
EBD_7809
2017-14 JEE Advanced 2017 Solved Paper
18. The minimum value of w0 below which the ring will drop
down is
1 Wat
g 2g er
(A) (B)
m(R - r) m(R - r) Ice

V.P./bar
(D)
3g g
(C) (D) Water + E
2m(R - r) 2m(R - r) than ol
271 273
T/K
CHEMISTRY
20. For the following cell,
SECTION - I Zn(s) | ZnSO4(aq) || CuSO4(aq) | Cu(s)
This section contains 7 questions. Each question has 4 options when the concentration of Zn2+ is 10 times the concentration
(A), (B), (C) and (D). ONLY ONE of these four options is correct. of Cu2+, the expression for DG (in J mol–1) is [F is Faraday
constant; R is gas constant; T is temperature; E° (cell) = 1.1 V]
19. Pure water freezes at 273 K and 1 bar. The addition of 34.5 g
(A) 1.1F (B) 2.303RT – 2.2F
of ethanol to 500 g of water changes the freezing point of the
solution. Use the freezing point depression constant of water (C) 2.303RT + 1.1F (D) – 2.2F
as 2 K kg mol–1. The figures shown below represent plots of 21. The standard state Gibbs free energies of formation of
vapour pressure (V.P.) versus temperature (T). [molecular C(graphite) an d C(diamond) at T = 298 K ar e
weight of ethanol is 46 g mol–1] Among the following, the DƒG° [C(graphite)] = 0 kJ mol–1
option representing change in the freezing point is DƒG° [C(diamond)] = 2.9 kJ mol–1
The standard state means that the pressure should be 1 bar,
1 and substance should be pure at a given temperature. The
Water conversion of graphite [C(graph ite)] to diamond
Ice [C(diamond)] reduces its volume by 2 × 10–6 m3 mol–1. If
V.P./bar

C(graphite) is converted to C(diamond) isothermally at


(A) T = 298 K, the pressure at which C(graphite) is in equilibrium
Water + Ethanol
with C(diamond), is
270 273 [Useful information : 1 J = 1 kg m2s–2; 1 Pa = 1 kg m–1 s–2; 1
T/K bar = 105 Pa]
(A) 14501 bar (B) 58001 bar
(C) 1450 bar (D) 29001 bar
ter 22. Which of the following combination will produce H2 gas?
Wa (A) Fe Metal and conc. HNO3
V.P/bar

l
(B) 1 ano (B) Cu metal and conc. HNO3
ice Eth
er + (C) Zn metal and NaOH(aq)
Wat (D) Au metal and NaCN(aq) in the presence of air
23. The order of the oxidation state of the phosphorus atom in
271 273
H3PO2, H3PO4, H3PO3 and H4P2O6 is
T/K
(A) H3PO3 > H3PO2 > H3PO4 > H4P2O6
(B) H3PO4 > H3PO2 > H3PO3 > H4P2O6
(C) H3PO4 > H4P2O6 > H3PO3 > H3PO2
ter (D) H3PO2 > H3PO3 > H4P2O6 > H3PO4
Wa
(C) 24. The major product of the following reaction is
V.P/bar

1 ol
Ethan
ice er + OH
Wat
i) NaNO ,HCl,0°C
270 273 ¾¾¾¾¾¾¾
2 ¾
®
ii) aq.NaOH
T/K
NH2
JEE Advanced 2017 Solved Paper 2017-15

(B) With increase in temperature, the value of K for


OH
endothermic reaction increases because unfavourable
change in entropy of the surroundings decreases
(A) (C) With increase in temperature, the value of K for
endothermic reaction increases because the entropy
Cl change of the system is negative
(D) With increase in temperature, the value of K for
–+
ONa exothermic reaction decreases because favourable
change in entropy of the surroundings decreases
(B) 28. In a bimolecular reaction, the steric factor P was experimentally
determined to be 4.5. The correct option(s) among the
N2Cl following is(are)
(A) The activation energy of the reaction is unaffected by
the value of the steric factor
OH
(B) Experimentally determined value of frequency factor
is higher than that predicted by Arrhenius equation
(C) (C) Since P = 4.5, the reaction will not proceed unless an
effective catalyst is used
N=N (D) The value of frequency factor predicted by Arrhenius
equation is higher than that determined experimentally
N=N
OH 29. For the following compounds, the correct statement(s) with
respect to nucleophilic substitution reaction is(are)
(D)
Br Br

25. The order of basicity among the following compounds is


NH NH2 (I) (II)

N NH HN N CH3
H3C NH2 H2N NH CH3
I II III IV H3C—C — Br Br
(A) II > I > IV > III (B) IV > II > III > I
(C) IV > I > II > III (D) I > IV > III > II CH3

(III) (IV)
SECTION - II
(A) I and III follow SN1 mechanism
This section contains 7 questions. Each question has 4 options
(B) I and II follow SN2 mechanism
(A), (B), (C) and (D). ONE or MORE THAN ONE of these four
options is (are) correct. (C) Compound IV undergoes inversion of configuration
(D) The order of reactivity for I, III and IV is : IV > I > III
26. The correct statement(s) about surface properties is(are) 30. Among the following, the correct statement(s) is(are)
(A) Adsorption is accompanied by decrease in enthalpy (A) Al(CH3)3 has the three–centre two–electron bonds in
and decrease in entropy of the system its dimeric structure
(B) The critical temperatures of ethane and nitrogen are
(B) BH3 has the three–centre two–electron bonds in its
563 K and 126 K, respectively. The adsorption of
ethane will be more than that of nitrogen on same dimeric structure
amount of activated charcoal at a given temperature (C) AlCl3 has the three–centre two–electron bonds in its
(C) Cloud is an emulsion type of colloid in which liquid is dimeric structure
dispersed phase and gas is dispersion medium (D) The Lewis acidity of BCl3 is greater than that of AlCl3
(D) Brownian motion of colloidal particles does not depend 31. The option(s) with only amphoteric oxides is(are)
on the size of the particles but depends on viscosity (A) Cr2O3, BeO, SnO, SnO2
of the solution (B) Cr2O3, CrO, SnO, PbO
27. For a reaction taking place in a container in equilibrium with (C) NO, B2O3, PbO, SnO2
its surroundings, the effect of temperature on its equilibrium
(D) ZnO, Al2O3, PbO, PbO2
constant K in terms of change in entropy is described by
(A) With increase in temperature, the value of K for 32. Compounds P and R upon ozonolysis produce Q and S,
exothermic reaction decreases because the entropy respectively. The molecular formula of Q and S is C8H8O. Q
change of the system is positive undergoes Cannizzaro reaction but not haloform reaction,
whereas S undergoes haloform reaction but not Cannizzaro
reaction
EBD_7809
2017-16 JEE Advanced 2017 Solved Paper
i) O /CH Cl COCH3
(i) P ¾¾¾¾¾¾
3 2 2
ii) Zn/H O
® Q
2 (C8 H8O)
(H3C)3C CH3
i) O / CH Cl
(ii) R ¾¾¾¾¾¾
3 2 2® S (B)
ii) Zn / H 2O (C8H8O)
The option(s) with suitable combination of P and R,
respectively, is(are) H3COC

(H3C)3C H3C CH3


(A) H3C and
CH 3 (C)
CH3 H3C
CH3
and HO3S
(B)
H3C CH3 O CH3
(H3C)3C
H3C (D)
CH3 CH3
CH3
(C) and
CH3 CH3 COCH3
36. The reactions, Q to R and R to S, are
H3C (A) Dehydration and Friedel–Crafts acylation
(D) H3C and
(B) Aromatic sulfonation and Friedel–Crafts acylation
CH3 (C) Friedel–Crafts alkylation, dehydration and Friedel–
SECTION - III Crafts acylation
(D) Friedel–Crafts alkylation and Friedel–Crafts acylation
This section contains 2 paragraphs, each describing theory,
experiments, data etc. four questions related to the two paragraphs
with two questions on each paragraph. Each question has only MATHEMATICS
one correct answer among the four given options (A), (B), (C)
and (D). SECTION - I
This section contains 7 questions. Each question has 4 options
PARAGRAPH 1 (A), (B), (C) and (D). ONLY ONE of these four options is correct.
Upon heating KClO3 in the presence of catalytic amount of MnO2,
a gas W is formed. Excess amount of W reacts with white phosphorus 37. The equation of the plane passing through the point (1, 1, 1)
to give X. The reaction of X with pure HNO3 gives Y and Z. and perpendicular to the planes 2x + y – 2z = 5 and 3x – 6y –
33. W and X are, respectively 2z = 7, is
(A) O3 and P4O6 (B) O2 and P4O6 (A) 14x + 2y – 15z = 1
(C) O2 and P4O10 (D) O3 and P4O10 (B) 14x – 2y + 15z = 27
34. Y and Z are, respectively (C) 14x + 2y + 15z = 31
(A) N2O3 and H3PO4 (B) N2O5 and HPO3 (D) –14x + 2y + 15z = 3
(C) N2O4 and HPO3 (D) N2O4 and H3PO3 38. Let O be the origin and let PQR be an arbitrary triangle. The
point S is such that
uuur uuur uuur uuur uuur uuur uuur uuur uuur uuur uuur uuur
PARAGRAPH 2 OP.OQ + OR.OS = OR.OP + OQ.OS = OQ.OR + OP.OS
The reaction of compound P with CH3MgBr (excess) in (C2H5)2O Then the triangle PQR has S as its
followed by addition of H2O gives Q. The compound Q on treatment (A) Centroid (B) Circumcentre
with H2SO4 at 0ºC gives R. The reaction of R with CH3COCl in the (C) Incentre (D) Orthocenter
presence of anhydrous AlCl3 in CH2Cl2 followed by treatment with 39. If y = y(x) satisfies th e differential equation
H2O produces compound S. [Et in compound P is ethyl group]
( )
-1

(H3C)3C CO2Et Q R S 8 x ( 9+ )
x dy = 4+ 9+ x dx, x > 0 and
y (0) = 7 , then y (256) =
(A) 3 (B) 9
P (C) 16 (D) 80
35. The product S is 40. If f : R ® R is a twice differentiable function such that
H3C æ1ö 1
f ¢¢ (x) > 0 for all x Î R, and f f ç ÷ = , f(1) = 1, then
CH3 è2ø 2
(H3C)3C
1
(A) f ¢ (1) £ 0 (B) 0 < f ¢(1) £
(A) 2
1
(C) < f ¢(1) £ 1 (D) f ¢ (1) > 1
COCH3 2
JEE Advanced 2017 Solved Paper 2017-17

41. How many 3 × 3 matrices M with entries from {0, 1, 2} are


cos(2x) cos(2x) sin(2x)
there, for which the sum of the diagonal entries of MT M is 5?
(A) 126 (B) 198 48. If f(x) = - cos x cos x - sin x , then
(C) 162 (D) 135 sin x sin x cos x
42. Let S = {1, 2, 3, ..., 9}. For k = 1, 2, ..., 5, let Nk be the number
of subsets of S, each containing five elements out of which (A) f ¢(x) = 0 at exactly three points in (–p, p)
exactly k are odd. Then N1 + N2 + N3 + N4 + N5 = (B) f ¢(x) = 0 at more than three points in (–p, p)
(A) 210 (B) 252 (C) f(x) attains its maximum at x = 0
(C) 125 (D) 126 (D) f(x) attains its minimum at x = 0
43. Three randomly chosen non–negative integers x, y and z are 49. If the line sx = a divides the area of region
found to satisfy the equation x + y + z = 10. Then the
probability that z is even, is R = {(x, y) Î R 2 : x3 £ y £ x, 0 £ x £ 1} into two equal
36 6 parts, then
(A) (B) 1
55 11 0<a£
(A)
1 5 2
(C) (D) 1
2 11 (B) < a <1
2
SECTION - II
This section contains 7 questions. Each question has 4 options
(C) 2a 4 - 4 a 2 + 1 = 0
(A), (B), (C) and (D). ONE or MORE THAN ONE of these four (D) a 4 + 4a 2 - 1 = 0
options is (are) correct.
k +1 k +1
å k =1 òk
98
50. If I = dx , then
sin(2x) x(x + 1)
44. If g(x) = ò sin -1 (t)dt, then
sin x (A) l > loge 99 (B) l < loge 99
æpö æ pö 49 49
(A) g¢ ç ÷ = -2p (B) g¢ ç - ÷ = 2p (C) 1< (D) l >
è2ø è 2ø 50 50
æpö æ pö SECTION - III
(C) g¢ ç ÷ = 2p (D) g¢ ç - ÷ = -2p
2
è ø è 2ø This section contains 2 paragraphs, Based on each paragraph,
45. Let a and b be non–zero real numbers such that there are 2 questions. Each question has four options (A), (B), (C)
2(cosb – cosa) + cosa cosb = 1. Then which of the following and (D) ONLY ONE of these four options is correct.
is/are true?
PARAGRAPH 1
æaö æbö uuur uuur uuur
(A) tan ç ÷ + 3 tan ç ÷ = 0 Let O be the origin, and OX, OY, OZ be three unit vectors in the
è2ø è2ø uuur uuur uuur
directions of the sides QR, RP, PQ respectively, of a triangle
æaö æbö
(B) 3 tan ç ÷ + tan ç ÷ = 0 PQR.
2
è ø è2ø uuur uuur
51. | OX ´ OY | =
æaö æbö
(C) tan ç ÷ - 3 tan ç ÷ = 0 (A) sin (P + Q) (B) sin 2R
è2ø è2ø (C) sin (P + R) (D) sin (Q + R)
æaö æbö 52. If the triangle PQR varies, then the minimum value of
(D) 3 tan ç ÷ - tan ç ÷ = 0 cos(P + Q) + cos (Q +R) + cos (R + P) is
è2ø è2ø
46. If f : R ® R is a differentiable function such that f ¢(x) > 2f(x) 5 3
(A) - (B) -
for all x Î R, and f(0) = 1, then 3 2
(A) f(x) is increasing in (0,¥) 3 5
(B) f(x) is decreasing in (0, ¥) (C) (D)
(C) f(x) > e2x in (0, ¥) 2 3
(D) f ¢(x) < e2x in (0, ¥) PARAGRAPH–2
Let p, q be integers and let a, b be the roots of the equation, x2 –
1 - x(1+ | 1 - x |) æ 1 ö x – 1 = 0, where a ¹ b. For n = 0, 1, 2, ...., let an = pan + q bn.
47. Let f(x) = cos ç ÷ for x ¹ 1. Then
| 1- x | è 1- x ø FACT : If a and b are rational numbers and a +b 5 = 0, then
(A) lim - f (x) = 0 a = 0 = b.s
x ®1
53. a12 =
(B) lim f (x) does not exist (A) a11 – a10 (B) a11 + a10
x ®1-
(C) 2a11 + a10 (D) a11 + 2a10
(C) lim f (x) = 0
x ®1+ 54. If a4= 28, then p + 2q =
lim f (x) does not exist (A) 21 (B) 14
(D) x ®1+ (C) 7 (D) 12
EBD_7809
2017-18 JEE Advanced 2017 Solved Paper

SOLUTIONS
Paper - 1
PHYSICS 1 2 1
1. (A, B, D) mgR = mv + MV 2
2 2

v Flat plate Flat plate 2 gR


+ – v On solving we get, v = m
1+
M

u u v1 v2 m 2 gR
\V=
M m
1+
M
Before collision Just after collision \ (B) is the correct option.
v 1 T
v1 - v v + v2 3. (A, D) We know that l = =
1= 1= f f m
v +u u–v
Where T = tension of string.
\ v1 = u + 2v \ v2 = u - 2v Here To > TA \ lo > lA so option (C) is wrong.
\ Dv1 = 2u + 2v and Dv2 = 2u - 2v Velocity being a vector quantity has direction. The
velocities of the two pulses cannot be same at
dp1 midpoint.
Now F1 = = rA(u + v )(2u + 2v )
dt TAo = T oA because speed (or velocity) of wave
depends on mediums (and not on the wavelength or
dp2 frequency of wave)
and F2 = = rA(u - v)(2 u - 2 v)
dt (A), (D) are the correct options
4. (C)
\ F1 = 2rA(u + v) 2 and F2 = 2rA(u - v )2
Energy radiated = sA(T 4 - T0 4 ) t
[For a black body e = 1]
F2 DF
P= = 8ruv = sA[(T0 + 10)4 - T0 4 ]t
F1 DF = F1 - F2 = 8rAuv \ A
é 4 ù
4 æ 10 ö
The net force Fnet = F - DF = ma = sAT ê
0 ç 1 + ÷ - 1ú t
êëè T0 ø úû
\ F - 8rAuv = ma
2. (B, C) Let the block be displaced by x . If initially the centre 4 é 40 ù 40
of mass of the system is at origin then = sAT0 ê T ú ´ t = 460 ´1´ 300 ´1 = 61.33J
ë 0û
M ´ x + m( x + R )
O= Energy radiated
M +m P= = sAT 4 - sAT0 4
time
- mR
O = Mx + mx + mR \ x= dp 3
m+M \ dT = sA(4T0 ) \ dp = sA(4T03 )dT0
0
\ 'C' is the correct option
If v is the velocity of mass 'm' as it leaves the block
\ DP = 4sAT03
and V is the velocity of block at that instant then
according to conservation of linear momentum A, B are not correct options as human body is not a
mv = MV black body.
By energy conservation Energy radiated µ A where A is the surface area of
the body
\ 'C' is the correct option
JEE Advanced 2017 Solved Paper 2017-19

5. (B, D) For smaller loop f = BA cos wt


sin 90° 1 -1 æ 1 ö
The rate of change of flux m= = Þ r2 = sin ç ÷
sin r2 sin r2 èmø
df
= – BAw sin wt
dt A
df
For to be maximum, sin wt should be maximum
dt i1 P Q
and this will happen when wt = 90° i.e., the plane of r2
loop is perpendicular to the plane of paper.
Option (D) is correct. B C
The emf produced will oppose each other. The net Emergent ray tangential
emf will also be proportional to sin wt. to the surface
enet = B(2A)wsin wt – BAw sin wt = BAw sin wt
\ option (B) is also correct. But r1 + r2 = A
6. (B, C) The angular frequency at which the current and \ r1 = A – r2
voltage will be at same phase is
-1 æ 1 ö
\ r1 = A – sin ç ÷
wr =
1
=
1
= 106 rad s –1 èmø
LC (10-6 ´10-6 )1/2 Applying Snell's law at 'P' we get
This value is independent of 'R' So (B) is correct sin i1 -1 é -1 1 ù
option. m= \ i1 = sin êm sin( A - sin )
sin r1 ë m úû
V (D) is correct option.
At w » 0 , the current i =
1 2 For minimum deviation PQ || BC
R 2 + (w L - )
wC \ (A) is also a correct option.
i = 0 (The circuit behaves as d.c circuit) 8. (6) DU = S[k ´ 4pr 2 - 4pR 2 ]
\ C is a correct option
é 4 3 4 3ù
If w >> w0, circuit behaves as an inductor. êë where 3 pR = k ´ 3 pr úû
7. (A, C, D) 1
For minimum deviation (when i1 = A) \ R=K 3r

i1 = e
é R2 ù
\ DU = 4ps ê k ´ 2/3 - R ú = 4pSR ëék - 1ûù
2 2 1/3
A
r1 = r2 = r (say) = ë k û
2

A \ DU = 4pSR 2 éë10a /3 - 1ùû


0.1
\ 10–3 = 4p ´ ´ (10-2 ) 2 ëé10a /3 - 1ûù
Q 4p
i1 P e
r1 r2 \ 10 2 = 10 a /3 - 1
a
B C Neglecting 1 we get 102 = 10a/3 \ =2 \ a=6
3
dm = 2i1 - A ,
n 2 n
Here d m = A \ i1 = A \ e=A 9. (5) Here U i = f = 6.25 \ f = 2.5
Uf 2 ni
ni
i1 If ni = 2 then n f = 5
\ r1 =
2 10. (8) Here n × sin 30° = [n – m × 0.1] sin 90°
option (C) is correct \ 1.8 × sin 30° = 1.8 – m × 0.1 \ m = 8
sin i1 sin A é 332 ù
m= = 11. (6) Frequency perceived by reflector = f1 = 492 ê
sin r1 sin A / 2 ë 330 úû
Frequency perceived by the source f2
2sin A / 2 cos A / 2
= = 2 cos A / 2
sin A / 2 é 332 ù é 330 ù
= 492 ê ´ = 498Hz
Applying Snell's law at Q ë 330 úû êë 328 úû
\ Beat frequency = 498 – 492 = 6Hz
EBD_7809
2017-20 JEE Advanced 2017 Solved Paper
12. (5) A = A0 e–lt \ A0 = Aelt = 115 (1 + lt)

é ln 2 ù 15. (B) x
\ A0 = 115 ê1 + ´ 11.5ú
ë t1/ 2 û
E = - E0 ˆj
é 0.7 ù
= 115 ê1 + ´ 11.5ú vel = 0
ë 8 ´ 24 û

A0 » 120 Bq Z
120 Bq activity level is in 2.5 ml
\ 2.4 × 10 5 Bq activity level will be in B = B0 ˆj

Y
2.5 ´ 2.4 ´105
ml
120
The electric field will apply a force on –Y axis thereby
2.5 ´ 2.4 ´ 105 accelerating the charge along –Y axis.
l = 5l
120 ´1000 FB = qv B sin q
13. (D) For the particle to move in straight line , electric force Here q = 180° therefore , FB = 0
should be equal and opposite to the magnetic force. 16. (D) DU = DQ – PDV
(D) is the correct option. show that work done = PDV
r r which is the formula for isobaric process.
FE = -eE = -e(- E0 xˆ ) = eE0 xˆ
17. (B) Work done in isochoric process is zero for which we
r r get a vertical line in P-V graph.
éE ù
FB = q(v ´ B) = -e ê 0 yˆ ´ B0 zˆ ú 18. (D) Laplace's correction of the speed of sound in ideal
B
ë 0 û gas is related to adiabatic process.
r CHEMISTRY
FB = -eE0 xˆ
19. (A, B, C)
(A) During adiabatic expansion, the final temperature is
x less than the initial temperature. Therefore the final
14. (A)
volume in adiabatic expansion will also be less than
2E0 the final volume in isothermal expansion. This can be
xˆ FB
B0 graphically shown as

B P
Z
E
reversible isothermal
reversible adiabatic
Y

The force due to magnetic field FB will provide the


necessary centripetal force for circular motion which The magnitude of work done by the gas is equal to the
will be in X-Y plane. The force due to electric field area under the curve. As seen from the figure the area
will accelerate proton in Z-direction. Thus the path under curve in reversible isothermal is more. Hence,
will be helical with increasing pitch. the magnitude of work done is lesser in adiabatic
reversible expansion as compared to the corresponding
work in isothermal expansion.
JEE Advanced 2017 Solved Paper 2017-21

(B) In free expansion, Pext = 0 \ W = 0 (C) Cl2 + H2O ® HCl + HOCl


If carried out isothermally (DU = 0) Þ q = 0 (Adiabatic) ;
(D) HClO4 + H 2 O ® ClO4- + H3O +
From I law Acid 1 Base 1 Base 1 acid 1
If carried out adiabatically (q = 0) Þ DU = 0 Since H2O is accepting H+ from HClO4 so H2O is
(Isothermal) ; From I law stronger base compared to ClO-4 .
(C) P 22. (C, D)
(P1, V1) The colour of X2 molecules of halogens is due to
absorption of light in the visible region. The energy
acquired in this manner excites the valence electron
(P2, V2)
from the highest occupied molecular orbital (HOMO)
to the lowest unoccupied molecular orbital (LUMO),
V i.e., transition from p* to s* molecular orbital.
During irreversible compression, maximum work is done
X : s1s , s*1s , s2s , s*2s , s2pz , p2px
2 2 2 2 2 2
on the gas (corresponding to shaded area) 2
(D) When T1 = T2 Þ DU = nCVDT = 0
In reversible adiabatic expansion, T2 < T1. p * 2 p 2x = p * 2 p 2y
= p2py2, , s * 2 pz0
\ DT = –ve \ DU = –ve ( HOMO )
20. (B, D)
a ( LUMO )
PL
23. (B, C, D)
Z
Excess NH (aq.)
[Co(H O) ] Cl 2 6 2 ¾ ¾NH ¾ ¾® [Co(NH 3 )6 ] Cl3
¾4Cl¾+ 3Air
(X) (Y)
PL (ideal) Pink Solution 1 : 3 electrolyte
2 3
1 0 HCl (excess) (d sp hybridisation of central
XM
0 1 room temperature metal atom as NH3 is a strong
XL
The graph representing +ve deviation from Raoult’s field ligand)

law therefore M – L < M – M or L – L [CoCl4] 3 AgNO3 (aq)

PL ³ P°L XL (Z)
Blue colour (tetrahedral: 3 AgCl (3 eq of AgCl)
but when XL = 1, mixture has almost pure liquid L so PL
3
= P°L sp as Cl¯ is a weak field ligand)
21. (A, B, D) [CoCl4] + 6H2O
2– 2+
Co(H 2O)6 + 4Cl¯
(A) In both the acids central atom is sp3 hybridized. (Blue) (Pink)
O X and Z in equilibrium at 0°C Þ then equilibrium is
|| sp 3 H–O–Cl shifted towards X, making colour of solution pink.
Cl
HO || sp 3
||

(B) ClO-4 is resonance stabilized anion


HClO4 ¾® ClO-4 + H+
HClO ¾® ClO¯ + H+
Hence HClO4 is more acidic than HClO.

24. (A, B)
IUPAC name : 1-chloro-4-methylbenzence
Cl CH3
1 1
2 2
(A) (B) 4-chlorotoluene
3 3

4 4

CH3 Cl
EBD_7809
2017-22 JEE Advanced 2017 Solved Paper
25. (B, D)
(B) Bromination proceeds through trans-addition in both the reactions.
M and N are identical, hence, M and O and N and P are two set of diastereomers.

(D)
Br
CH 3
H
CH 3 H Br
H Br
CH 3 H H Br
a
Br—Br CH 3 H CH 3 CH 3
Br
b (N)
CH3 CH3
H
a

H Br
b
trans-2-Butene trans-Bromonium ion H CH 3 CH 3
Br Br H
Br
Br H
CH 3 H CH3
(M)
M and N are same
(meso-2, 3-Dibromobutane)

Br
CH 3
H CH 3 H Br
H Br H
CH 3 Br H
c
Br—Br CH3 H CH 3 CH 3
Br
(O)
d
H H
CH 3
c
CH3 Br
d H CH 3
cis-2-Butene cis -Bromonium ion CH3

Br Br H
Br H Br
H CH 3 CH 3
(P)
II and III are enantiomers
rac-2, 3-Dibromobutane
JEE Advanced 2017 Solved Paper 2017-23

Z´M N2 :
2
s1s2, s*1s2, s2s2, s*2s2, p2p x = p 2p2y , s 2p2z
26. (2) Density (d) = (d = density)
a 3 ´ NA (Diamagnetic)
For FCC, Z = 4
Given a = 4 × 10–8 cm O2- : s1s2, s*1s2, s2s2, s*2s2, p 2p2z , p2p2x
4´ M = p2p2y , p * 2p x2 = p *2p1y
8=
( 4 ´ 10 ) ´ 6 ´ 10
-10 3 23
(Paramagnetic)
2 2
s1s2, s*1s2, s2s2, s*2s2, s2pz , p2p x
8 ´ ( 4 ´ 10 ) ´ 6 ´ 10
-8 3 23
F2 :
M= = p2p2y , p * 2p x2 = p *2p2y
4
(Diamagnetic)
8 ´ 6 ´ 1023 ´ 64 ´ 10 -24
=
4
30. (5)
wt
No. of atoms = ´ NA Non-aromatic Anti-aromatic Non-aromatic Anti-aromatic
Molar mass
256 ´ 10 ´ 6 ´ 10 23
= 2 × 1024
8 ´ 6 ´ 16
\ Value of N = 2
a Aromatic Aromatic Aromatic
27. (6) The formula for conductance is G = k ´
l
1
5 ´ 10 -7 = k ´
120
k = 6 × 10–5 s cm–1
k ´ 1000 6 ´ 10 -5 ´ 1000
L cm = = = 40
M 0.0015 Aromatic Aromatic
Q pH = 4 31. (C) 1s wave function for He+ is given by
\ [H+] = 10–4 = ca = 0.0015 a
10-4 Z -
æ Zr ö
a= Y(1s) = Yn, l, ml = exp çè a 0 ÷ø
0.0015 pa 0 3/2
-4
L cm 10
a= Þ = 40 3/ 2 é Zr ù
L om 0.0015 æZö -ê ú
or µ ç ÷ exp ë a0 û
è a0 ø
L om = 6 ´ 102 s cm 2 mole -1
i.e., z = 6 i.e., it is independent of cos q.
The probability of finding an electron at zero distance
28. (6) Species Number of lone pairs from the nucleus is zero. The probability increases
on central atom gradually as the distance increases, goes to maximum
(i) [TeBr6]2– 1 and then begins to decrease.
(ii) [BrF2]+ 2 32. (A) For a given orbital with principal quantum number, n
(iii) SNF3 0 and azimuthal quantum numbr, l.
(iv) [XeF3]– 3
Number of radial nodes = (n – l – 1)
Total number of lone pairs = 1 + 2 + 0 + 3 = 6
29. (6) (H2, Cl2, Be2, C2, N2, F2)
H2 : s1s2 (Diamagnetic)
He +2 : s1s2, s*1s1 (Paramagnetic)
Y n,l,m (r)

Li2 : s1s , s*1s , s2s2 (Diamagnetic)


2 2

Be2 : s1s2, s*1s2, s2s2, s*2s2 (Diamagnetic)


1
B2 : s1s2, s*1s2, s2s2, s*2s2, p2p x = p2p1y r

(Paramagnetic) a0
2
C2 : s1s2, s*1s2, s2s2, s*2s2, p2p x = p2p2y
(Diamagnetic)
EBD_7809
2017-24 JEE Advanced 2017 Solved Paper
33. (C) Refer ans 31.
Energy needed to excite from n = 2 to n = 4
3
DE2 – 4 = 13.6 Z2 × eV
16
Similarly,
8
DE2 – 6 = 13.6 Z2 × eV
36
on dividing
DE 2 - 4 3 36 27
= ´ =
DE 2 -6 16 8 32
34. (A) Alkylbenzenes when treated with Br2 at high temperature, in the presence of sunlight and absence of halogen carrier
undergo halogenation in the side chain. Thus
CH3 CH2 Br CHBr2 CBr3

Br2 / heat Br2 Br2


¾¾¾¾® ¾¾¾¾® ¾¾¾¾®
light heat, light heat, light
Toluene Benzyl bromide Benzyl trichloride

35. (C)

36. (D) Perkin condensation of benzaldehyde with (CH3CO)2O/CH3COOK yields cis and trans form of cinnamic acid.
(i) CH 3 COONa
C6H5CHO + (CH3CO)2O ¾¾¾¾¾®   C6H5CH = CHCOOH
Benzaldehyde Acetic anhydride (ii) H+ Cinnamic acid
cis and trans

MATHEMATICS Þ yk – 8(x + h) = k2 – 16h


37. (A, B, C) Þ 8x – ky = 8h – k2 …(1)
\ 2x – y + 1 = 0 i.e. y = 2x + 1 is a tangent to hyperbola But given, the equation of chord is
2x + y = p …(2)
x2 y2 \ (1) and (2) are identical lines
- =1
a 2 16 8 -k 8h - k 2
\ c2 = a2m2 – b2 Þ = =
12 = a2 × 22 – 16 2 1 p
Þ k = –4 and 8h – 16 = 4p
17 Þ k = –4 and p = 2h – 4
Þ a2 =
4 which are satisfied by option (C).
39. (A, C, D)
17 Let x = n be any integer not equal to zero.
Þ a=
2 Then
\ a, 4, 1; a, 4, 2; 2a, 8, 1 lim - x cos(p (x + [x]))
x®n
17 17 = n cos(p(n + n – 1))
i.e. , 4, 1; , 4, 2; 17 , 8, 1
2 2 = n cos (2n – 1)p = –n
cannot be the sides of a right triangle.
lim+ x cos(p (x + [x]))
38. (C) If (h, k) is the mid point of chord of parabola y2 = 16x, x®n
then equation of chord will be given by = n cos(p(n + [n]))
T = S1 = n cos(p(n + n))
JEE Advanced 2017 Solved Paper 2017-25
= n cos 2np = n 2
LHL ¹ RHL Þ limit does not exist at any non P(X Ç Y) 15 4
zero integer n. = =
P(Y) = P(X / Y) 1 15
\ f is discontinuous at x = –1, 1, 2
At x = 0, LHL = RHL = 0 = f(0) 2
\ f is continuous at x = 0. P(X ' Ç Y) P(Y) - P(X Ç Y)
40. (A, B) Let us check the given options one by one. P(X'/Y) = =
(A) Let g(x) = x9 – f(x) P(Y) P(Y)
Þ g(0) = – f(0) < 0 \ f(x) Î (0, 1) = 1 – P(X/Y)
Also g(1) = 1 – f(1) > 0 1
\ x9 – f(x) = 0 for some x Î (0, 1) =
2
p
-x 1 4 2 7
(B) Let h(x) = x ò 2 f (t) cos t dt P(X È Y) = + - =
0 3 15 15 15
p \ A and B are the correct options.
h (0) = - ò 2 f (t) cos t dt < 0 44. (2) Centre (–1, –2)
0
Geometrically, circle will have exactly 3 common
p -1 points with axes in the cases
and h(1) = 1 - ò 2 f (t) cos t dt > 0 (i) Passing through origin Þ p = 0
0
(ii) Touching x-axis and intersecting y-axis at two
p -x
\ h(x) = x - ò 2 -f (t) cos t dt = 0 points i.e. f2 > C and g2 = C.
0 i.e. 4 > – p and 1 = –p
at some x Î (0, 1) Þ p > –4 and p = –1
x Þ p = –1
(C) ex - ò - f (t) sin t dt (iii) Touching y-axis and intersecting x-axis at two
0
points i.e. f2 = c and g2 > C
Q x Î (0, 1) Þ ex Î (1, e) Þ 4 = –p and 1 > –p
and 0 < f(t) < 1 and 0 < sin t < " x Î (0, 1) Þ p = –4 and p > –1
x which is not possible.
\ 0< ò0 f (t) sin t dt < 1 \ Only two values of p are possible.
x
\ ex - ò f (t) sin t dt ¹ 0 for any x Î (0, 1) æ pö
45. (2) Given f(0) = 0, f ç ÷ = 3, f'(0) = 1
0 è 2ø
p
p
(D) f (x) + ò0
2 f (t) sin t dt > 0 " x Î (0, 1) g(x) = òx éë f ' ( t ) cosec t - cot t cosec t f ( t )ûù dt
2

41. (B, D) In options (A) and (C) |A2| = 1


p
and in option (B) and (D) |A2| = – 1 d
We know |A2| = |A|2 g(x) = lim òx2 ( f ( t ) cosec t ) dt
x ®0 dt
and |A|2 ¹ –1 Þ matrices given in options B & D
cannot be the squares of any 3 × 3 matrix with real = f æç p ö÷ cosec p – f(x) cosec x
entries. è 2ø 2
42. (A, B) a – b = 1, y ¹ 0
f (x)
æ az + b ö = 3 – f(x) cosec x = 3 –
Im çè ÷=y sin x
z +1 ø
f (x) f (x)
é a ( x + iy) + b ( x + 1) - iy ù lim g(x) = lim 3 – = 3 – lim
x® 0 sin x x ® 0 sin x
Þ Im ê x + 1 + iy ´ x + 1 - iy ú = y
x® 0

ë ( ) ( ) û f '( x )
= 3 – xlim = 3 – f' (0) = 3 – 1 = 2
- ( ax + b ) y + ay ( x + 1) ® 0 cos x
Þ =y 46. (1) For infinite many solutions
( x + 1) 2 + y2
1 a a2
-axy - by + axy + ay
=y a 1 a
Þ
( x + 1) 2 + y 2 2
= 0 Þ (1 – a2)2 = 0 Þ a = ± 1
Þ a – b = (x + 1)2 + y2 a a 1
Þ 1 = (x + 1)2 + y2 For a = 1, the system will have no solution and for
a = –1, all three equations reduce to x – y + z = 1
Þ x = –1 ± 1 - y 2 giving infinite many dependent solutions.
1 1 2 \ 1 + a + a2 = 1 – 1 + 1 = 1
43. (A, B) P(X) = , P(X/Y) = , P(Y/X) = 10!
3 2 5 47. (5) x = 10! and y = 10C1 × = 50 × 9!
2 1 2 2!
P(X Ç Y) = P (Y/X) P(X) = ´ = y 50 ´ 9!
5 3 15 \ = =5
9x 10!
EBD_7809
2017-26 JEE Advanced 2017 Solved Paper
48. (6) Let the sides be a – d, a, a + d where d is positive.
Using Pythagoras theorem, a2 2 1
Þ =1 and =
(a + d)2 = (a – d)2 + a2 3a + 42 2
3a + 4 2
Þ a = 4d
\ Sides are 3d, 4d, 5d Þ a4 – 3a2 – 4 = 0 and 3a2 = 12
Þ a2 = 4
1 Also for a2 = 4 equation of ellipse
Area = 24 Þ ´ 3d ´ 4d = 24
2 æ 1ö
Þ d=2 x2 + a2y2 = a2 is satisfied for the point ç 3, ÷
è 2ø
\ Sides are 6, 8, 10.
\ Smallest side = 6. \ II, (iv), R is the correct combination.
49. (B) For a = 2 and point of contact (–1, 1). (For questions 52-54) : We observe the following, in the given
Equation of circle is satisfied table.
x2 + y2 = 2 f(x) = x + logex – xlogex, x Î (0, ¥)
then eqn. of tangent is
–x + y = 2 Þ m = 1 1 1+ x
Þ f ' (x) = - log e x and f ''(x) = – 2
and point of contact x x
æ - ma ö æ- 2 f(1) = 1 > 0 and f (e2) = e2 + 2 – 2e2 = 2 – e2 < 0
a 2ö \ f(x) = 0 for some x Î (1, e2)
ç , ÷=ç , ÷ = (–1, 1)
è m +1 m +1ø è 2
2 2 2ø \ (I) is true.
\ (I) (ii), (Q) is the correct combination. 1
50. (C) Tangent y = x + 8 Þ m = 1 f ' (1) = 1 > 0 and f ' (e) = –1 <0
e
point (8, 16) \ f ' (x) = 0 for some x Î (1, e)
\ both the coordinates as well as m, are positive, \ (II) is true.
æ a 2a ö 1
the only possibility of point is çè 2 , ÷ø = (8, 16) If x Î (0, 1), > 0 and loge x < 0
m m x
Þ a= 8 1
Also it satisfies the equation of curve. \ f ' (x) = – logex > 0 Þ f is increasing on (0, 1)
x
y2 = 4ax for the point (8, 16)
\ f ' (x) ¹ 0 for some x Î (0, 1)
And equation of tangent my = m2x + a is satisfied by
\ (III) is false.
m = 1 and a = 8
If x Î (1, e), f '' (x) < 0 Þ f ' is decreasing on (i, e)
\ (III), (i), (P) is the correct combination.
\ f '' (x) ¹ 0 for some x Î (1, e)
æ 1ö \ (IV) is false.
51. (D) Point of contact çè 3, ÷ø and tangent 3x + 2y =
2 Also lim f(x) = lim x + (1 – x) logex = – ¥
x ®¥ x ®¥
4.
\ (i) is false and (ii) is true.
3 1
\ m= - lim f ' (x) = lim
2 – logex = –¥
x ®¥ x®¥ x
\ Both the coordinates are positive and m is \ (iii) is true
negative the possibilities for points. are
1 1
æ ma a ö lim f '' (x) = lim - 2 - = 0
Qç - , ÷ OR
x ®¥ x ®¥ x x
è m2 + 1 m2 + 1 ø \ (iv) is true.
f is increasing on (0, 1) already discussed
æ a2m 1 ö \ (P) is true.
Rç- , ÷ If x Î (e, e2) then
è a 2 m2 + 1 a 2 m2 + 1 ø
1
f ' (x) = – logex < 0
æ 3a 2a ö æ 1ö x
For point Q ç , ÷ = ç 3, ÷ø Þ f is decreasing in (e, e2)
è 7 7ø è 2
\ (Q) is true.
7 For x Î (0, 1), f "(x) < 0
We get a = 7 and a = Þ f' is decreasing in (0, 1)
4 \ R is false.
which is not possible. For x Î (e, e2) f " (x) < 0
æ a2 3 2 ö æ Þ f' decreasing in (e, e2)
1ö \ (S) is true.
For point R ç , ÷ = ç 3, ÷
2 2
è 3a + 4 3a + 4 ø è 2ø 52. (B) The only correct combination is (II), (ii), (Q)
53. (B) The only correct combination is (II), (iii), (S)
54. (C) The only incorrect combination is (III), (i), (R).
JEE Advanced 2017 Solved Paper 2017-27

Paper - 2
PHYSICS OC
In D OAC cos 60° =
1 dr OA
1. (A) = constant
r dt 1
\ OC = 2a ´ =a
4pR d é m ù 3 2
\ 3m dt ê 4 ú = constant The magnetic field at 'O' due to
ê pR3 ú
ë3 û m0 I
AB = [ sin 60° - sin 30°]
4p a
3 d -3
\ R ( R ) = constant
dt m0 I é 3 1 ù m0 I 1
= ê - ú= ´ ( 3 - 1)
3 -4 dR 4 p a ë 2 2 û 4pa 2
\ R ( -3R ) = constant The total magnetic field due to all the straight segments
dt
of the star is
dR
\ µR é m0 I 1 ù m I
dt =ê ´ ( 3 - 1) ú ´12 = 0 ´ 6( 3 - 1)
ë 4p a 2 û 4p a
O O r r
r r r r S -P
h 5. (A) Here P + bR = S \ R =
p/n p/n b
2. (B) h A r r r
Also R = Q - P
r r
A B B S-P r r r r r r
\ = Q - P \ S - P = bQ - bP
b
p OA h r r r
In DOAB cos = \ OB = p \ S = bQ + (1 - b) P
n OB cos
n
1 GM e m GM e m ´ 3 ´105
h é 1 ù 6. (B) mVe2 - - =0
D= -h = hê -1 2 Re 2.5 ´104 Re
p p ú
cos ê cos ú
n ë n û Ve2 GM e é 3 ´105 ù
= ê1 + 4ú
hC p2 h2 2 Re ë 2.5 ´10 û
3. (D) - f0 = K .E = =
l 2me 2me l d 2
æ 2GM e ö
Differentiating on both sides Ve = 13 ç ÷ = 13 ´11.2 » 42
è Re ø
-hC h2 æ -2 ö
dl = ç 3 ´ d ld ÷
l 2 2me 2L L
è ld ø 7. (B) T = +
g v
d ld l 3
\ µ d2 with error limits
dl l
2( L + dL) L + dL
T + dT = +
A A g v
4. (A)
2 L æ dL ö L æ dL ö
B I B
30° \ T + dT = ç1 + ÷ + ç1 + ÷
2a g è L ø vè L ø
60° 30°
30°
C 60° 2 L æ dL ö L æ dL ö
4a \ T + dT = ´ ç1 + ÷ + ç1 + ÷
O g è 2L ø v è L ø
O

2L 2 L dL L L dL
\ T + dT = + + +
g g 2L v v L
EBD_7809
2017-28 JEE Advanced 2017 Solved Paper

2 L dL L dL Also CP2 = CO 2 + OP2 2


T + dT = T + +
g 2L v L
2 2
æ 5 R ö æ 3R ö
dL é 1 2L L ù = ç ÷ +ç ÷
è 8 ø è 2 ø
dT = L ê 2 g + v ú
ëê úû
13R
Substituting dT = 0.015, L = 20 m, g = 10 ms–2, CP2 =
8
v = 300 ms–1
We get
dL 15
=
L 1600
dL 15 15 p
´ 100 = ´ 100 = % » 1%
\ L 1600 16
8. (A, B) For the charge +Q to return region 1, the radius of the
3R
circular path taken by charge should by
2
mv 2 2p p
= QvB \ = QB
(3R / 2) 3R

2p
\ B = 3QR
Thus the particle will enter region 3 through the point P1 on
2p X-axis
i.e., B should be equal or greater than 'B' is the correct option.
2QR
Change in momentum = 2p
'A' is the correct option.
Thus 'C' is incorrect
8p
When B = mv
13QR mv 2
Further = qvB \ r = \ rµm
r qB
mv 2 æ 8p ö 13 R \ 'D' is incorrect.
= Qv ç ÷ \ r=
r è 13QR ø 8
9. (A, D) The potential difference between X and Y is
Thus 'C' is the of the centre of circular path of radius
VXY = V X - VY
13R
8 VXY = (VXY )0 sin(wt + q1 )

2 2 2 2p
Y where (V XY ) 0 = V0 + V0 - 2V0 cos = 3V0
3

(VXY ) 0 3
and (V XY )rms = = V0
2 2
(A) is the correct option
C Now the potential difference between Y and Z is
5R
VYZ = VY - VZ
8
O X VYZ = (VYZ )0 sin(wt + q2 )
R P2
3R / 2 2p
2 2 2
Where (VYZ )0 = V0 + V0 - 2V0 cos = 3V0
3
P1
+Q (VYZ ) 03
and (VYZ )rms = =
V0
2 2
Thus (D) is the correct option.
JEE Advanced 2017 Solved Paper 2017-29

Now path difference


10. (A, D) P +Q p = dcos q = nl (for bright fringe)
nl
45° q \ cos q =
d
l
\ - sin q Dq = (Dn)
d
Dnl
or Dq = -
d sin q
E
Ecos q As we move from p1 to p2, q decreases and therefore
The circumference of the flat surface is an Dq increases. Therefore (D) is incorrect.
equipotential because the distance of each point on 12. (A, B, C)
the circumference is equal from + Q After a long time the current through the resistor is
(D) is the correct option. constant I will divide into two parts L1 and L2 which
The component of electric field normal to the flat are in parallel
surface is Ecosq. \ I1L1 = I2L2
Here E as well as q changes for different point on the V é L2 ù
flat surface. Therefore (C) is incorrect. Further I1 = R ê L + L ú
The total flux through the curved and flat surface ë 1 û
Q V é L1 ù
should be less than e . Therefore (B) is incorrect. I2 =
0
and R êë L1 + L2 úû
The solid angle subtended by the flat surface at Also the ratio of currents through L1 and L2 is fixed
æ 1 ö at all times At t = 0, I » 0
P = 2p ç 1 - ÷ 13. (A, C, D)
è 2ø
As Fx = 0, ax = 0. Therefore the force acting in vertical
\ Flux passing through curved surface direction will move the mid point of the bar fall
æ 1 ö vertically downwards. (A) is correct option.
2p ç 1 - ÷
Q' è 2ø Q æ 1 ö.
=- =- 1-
e0 4p 2e0 çè ÷
2ø Y
A (0, L)
(A) is the correct option.
11. (B, C)
Path difference at P2 is
p = S1P2 – S2P2 = d = 1.8 mm = 1.8 × 10–3m
= 3000 × 600 × 10–9 m
p = 3000 l. (x, y)
As the path difference is an integral multiple of l, P2 90 – q
B
should be a bright fringe with 300th maxima. (A) is
incorrect.
q
Further at P1, path difference = 0. Therefore a bright
fringe will be present at P1 also. Therefore total
number of fringes between P1 and P2 is 3000. (C) is a
correct option. L/2 L/2
Obviously at P2 the order of the fringe will be q L
mg cos q
maximum. Thus (B) is a correct option. 2
P1
X
P L
Rdq sin q
2

q
os When the bar makes an angle q with the vertical, the
dc
q P2 displacement of its mid point from the initial position
S1 S2
d L L
is - cos q
2 2
EBD_7809
2017-30 JEE Advanced 2017 Solved Paper
(D) is a correct option.
Instantaneous torque about the point of contact P is æ R - rö
17. (C) Here w0 (R - r ) = wR \ w = wo çè ÷
R ø
L
t = mg ´ sin q Now total kinetic energy of the ring (Kinetic rotational
2 + kinetic translational)
(C) is a correct option.
1
L K .Etotal = (2MR 2 )w2 = M w0 2 ( R - r ) 2
Now x = sin q , y = L sin(90 - q) = L cos q 2
2
2 2
18. (A) mM w2min ( R - r ) = Mg
æ 2x ö æ y ö 4x2 y2
\ ç ÷ + ç ÷ = 1 or 2 + 2 = 1 g
è L ø è Lø L L \ wmin =
This is equation of ellipse. Therefore B is incorrect m( R - r )
14. (C) If the force is applied at P tangential than the I remains
constant and is equal to F × 2R where F is the applied
CHEMISTRY
force.
If force is applied normal to X, then as the wheels 19. (C) As T increase, V.P. increases
climbs, then the perpendicular distance of force from DTf = Kf × m
Q will go on changing initially the perpendicular is 34.5 ´ 1000
QM, later it becomes QM'. 273 – Tf¢ = 2 ´
46 ´ 500
\ T¢f = 270 K
F 20. (B) Zn ( s ) + Cu 2aq
+ +
® Zn 2aq + Cu ( s )
( ) ( )

x F
M
é Zn 2+ ù
ë û
Q DG = DG° + 2.303 RT log10Q ; Q =
éCu 2+ ù
ë û

[DG° = –nFE°] = –2 × F × 1.1


x
Given [Zn2+] = 10[Cu2+]
If the force is applied normal to the circumference at \ DG = –2F (1.1) + 2.303 RT log1010
point P then I is zero. So (C) is correct. = 2.303 RT – 2.2F
If the force is applied tangentially at point S then 21. (A) At eq. DG = 0
t = F ´ R and the wheel will climb. DG° = dp(DV)
15. (A) Work done by battery = q × V 2.9 kJ mol–1 = (P2 – 1) × 2 × 10–6 m3 mol–1
\ W = CVo × Vo = CVo2 or
1
Energy stored in the battery =
2
CV0 2
2.9 ´ 103
kg m2
s 2 mol -1
(
( P2 - 1) 2 ´ 10-6
=
m3
mol -1
)
\ Energy dissipated
1 2 1 2 2.9 ´ 103 ´ 106 kg
ED = W – EC = CV02 - 2 CV0 = 2 CV0 (P2 – 1) =
2 ms2
\ EC = ED
16. (C) Let Vi and Vf be the initial and final voltage in each kg
process. Then = 1.45 × 109 = 1.45 × 109 Pa
ms 2
Energy dissipated = Wbattery – DU \ P2 = 1.45 × 109 + 1Pa
1 1 = 14500 × 105 + 1 = 14501 × 105 Pa = 14501 bar
= C(Vf –Vi )Vf – C(Vf – Vi ) 2 = C(Vf – Vi ) 2
2 2 22. (C) Fe + conc. HNO3 ® Fe2O3
\ Total heat dissipated Cu + conc. HNO3 ® CuNO3 + NO2 + H2O
4Au + 8NaCN + O2 + 2H2O ® 4Na[Au (CN)2] + 4NaOH
1 éæ Vo 2V ö ù
2 2 2
ö æ 2V V ö æ Zn + NaOH ® Na2ZnO2 + H2
ED = C êç - 0÷ + ç o - o ÷ + çVo - o ÷ ú
2 ëè 3 ø è 3 3ø è 3 ø û 23. (C) Correct order :
H3PO 4 > H 4 P2 O6 > H3 PO3 > H3PO2
1
= CVo 2 ( +5) ( +4) ( +3) ( +1)
6
JEE Advanced 2017 Solved Paper 2017-31

24. (C) Lone pair is involved in aromatic sextet. Hence not


Step 1 : available.
OH OH Hence the correct order of basic strength is
IV > I > II > III
(i) NaNO 2, HCl, 0°C 26. (A, B)
(A) As adsorption is spontaneous, DG for the process is –
+H ve. Adsorption is accompained by decrease in
NH2 Nº N randomness. Therefore DS and TDS for the process is
|
Cl – also negative. As DS for the process is –ve and the
Step 2 : process is spontaneous, DH for the process has to be
–ve i.e, enthalpy of the system decreases.
OH OH (B) Under a given set of conditions of temperature and
pressure the easily liquefiable gases e.g. C2H6, NH3
(ii) aqNaOH and HCl are adsorbed more than the gases like N2, H2
and CO. The ease with which a gas can be liquefied is
+
NºN determined by its critical temperature.
N=N
| H Critical temperature is the minimum temperature above
Cl Diazocoupling which a gas can be liquified. This implies that gases
with high critical temperature values can be easily
liquified as compared to gases with low critical
temperature value.
25. (C)
+ 27. (B, D)
(IV) 28. (A, B) The Arrhenius equation is
The conjugate acid is stabilized by resonance with k = Ae–Ea/RT
two different –NH2 group. Hence electron density where A = Pre-exponential factor
increases on N of = NH A is not directly related with temperature and activation
energy.

+ Where A = Z ´ P
( Frequency factor ) (Steric factor )
(I) Hence we can say the Ea is independent of steric factor
The conjugate acid is stabilized by resonance with
one –NH2 group. Hence as compared to IV lesser K actual
P=
increase of electron density on N of = NH K theoretical

So, Aactual > Atheoretical


+ 29. (A, B, C)

(II) (A) I is Br (1° benzylic halide) and


Lone pair is not involvd in aromaticity. Hence more
available
C
|
N: C – C – Br (3° alkylhalide). Follow SN1.
|
N C
H (B) I and II follow SN2 also, as both are 1° halide.
(III) (C) Compound (IV) undergoes inversion of configuration
due to presence of chiral carbon atom.
EBD_7809
2017-32 JEE Advanced 2017 Solved Paper
30. (A, B, D)
(A) Structure of Al2(CH3)6

H
H H
C
H3C CH3
Al Al
H3C CH3
C
H H
H
3C-2e– Bond
(B) Structure of B2H6

H
H H
B B
H H
H
3C-2e– Bond

(C) Structure of Al2Cl6


Cl Cl Cl
Al Al
Cl Cl Cl
(D) BCl3 is stronger lewis acid as the bond formed with the base will involve 2p orbital overlap which is stronger than 3p orbital
overlap in the case of AlCl3.
31. (A, D) NO Þ Neutral
B2O3 Þ Acidic
CrO Þ Basic
All other oxides are amphoteric
32. (A, B)

O O
(i) O3, CH2Cl 2
(C) CH3 Q (C8H8O), i.e., CH3 C + H – C– H
(ii) Zn, H2O
H
(P)
(Q) (+ve cannizaro reaction)

O
C O
(i) O 3, CH 2Cl 2
CH3
(ii) Zn, H2O
+ H – C– H

(R) CH3 (S) (+ve haloform reaction)


CH3 O O
O3, CH2Cl2
(D) C + CH3 – C– H
Zn, H2O
H
CH3
CH3
(Q) (+ve cannizaro reaction)
(P)
JEE Advanced 2017 Solved Paper 2017-33

CH3 O
C C O
CH3 O3, CH2Cl 2
CH3 + CH3 – C– CH3
C
Zn, H2O
CH3
(R) CH3
(+ve haloform reaction)
(S)
33. (C) & 34. (B)
D
KClO3 ¾¾¾¾
® KCl + O 2
MnO2
( W)
P4 + O2 ® P4 O10
( white) ( excess) (X)
P4O10 + HNO3 ® N 2O5 + HPO3
( Y) ( Z)
35. (A) & 36. (D)
– +
O OMgBr

(CH3)3C C OEt (CH3)3C CH3 C CH3


CH 3 MgBr(excess)/(C2 H 5 )2 O
¾¾¾¾¾¾¾¾¾¾¾®

(P)
H2O
H + H
O O H

(CH3)3C C(CH3)2 (CH3)3C CH3 C CH3


H SO
¬¾¾¾
2 4¾
0°C

(Q)
–H 2O
CH3
+ CH3
(CH3)3C C(CH3)2 (CH3)3C
(Friedel Craft
alkylation)

(R)

CH3 C Cl/AlCl3
(Friedel Craft Acylation)

CH3
CH3
(CH3)3C

COCH3
(S)
EBD_7809
2017-34 JEE Advanced 2017 Solved Paper
MATHEMATICS
æ 1ö
37. (C) The required equation of plane is given by f (1) - f ç ÷
è 2ø æ1 ö
x -1 y - 1 z -1 f ' (a) = , a Î ç , 1÷
1 è2 ø
1-
2 1 -2 =0 2
3 -6 -2
Þ (x – 1) (–14) – (y – 1) (2) + (z – 1) (–15) = 0 æ1 ö
Þ f ' (a) = 1 for some a Î ç , 1÷
Þ 14x – 14 + 2y – 2 + 15z – 15 = 0 è2 ø
Þ 14 x + 2y + 15z = 31 Þ f ' (1) > 1
uuuur uuur uuuur uuur uuuur uuur uuur uuur
38. (D) OP · OQ + O R· OS = OR · OP + OQ· OS é a1 a2 a3 ù
uuuur uuur uuur uuur uuur uuur ê ú
( ) (
Þ OQ - OR · OP - OQ - OR · OS = 0 ) 41. (B) Let M = ê a 4
êë a 7
a5 a 6 ú where ai Î {0, 1, 2}
a8 a 9 úû
uuuur uuur uuur uuur
Þ ( OQ - OR ) · ( OP - OS) = 0
uuur uur é a1 a4 a 7 ù éa1 a2 a3 ù
Þ RQ ·SP = 0 ê úê ú
Then MTM = ê a 2 a5 a 8 ú êa 4 a5 a6 ú
Þ RQ ^ SP …(1) êë a 3 a6 a 9 úû êëa 7 a8 a 9 úû
uuur uuur uuur uuur uuur uuur uuur uuur
Also OR · OP + OQ · OS = OQ · OR + OP · OS T
Sum of the diagonal entries in M M = 5
uuur uuur uuur uuur uuur uuur
( )
Þ OR · OP - OQ - OS · OP - OQ = 0 ( ) Þ (a12 + a42 + a72) + (a22 + a52 + a82)
+ (a32 + a62 + a92) = 5
uuur uuur uuur uuur
( )(
Þ OP - OQ · OR - OS = 0 ) It is possible when
Case I: 5 ais are 1 and 4 ai's are zero
uuur uuur Which can be done in
Þ QP ·SR = 0
Þ QP ^ SR …(2) 9´8´7´6
9
C4 ways = = 126
From (1) and (2) S should be the orthocentre of DPQR. 4 ´ 3 ´ 2 ´1
39. (A) Given DE can be written as Case II: 1 ai is 1 and 1ai is 2 and rest.
7ai's are zero
1 It can be done in 9C1 × 8C1 = 9 × 8 = 72 ways
ò dy = ò dx
( 4+ 9+ x )( )
9+ x 8 x
42. (D)
\ Total no. of ways = 126 + 72 = 198.
N1 = 5C1 × 4C4 = 5
N2 = 5C2 × 4C3 = 40
Putting 4+ 9+ x =t N3 = 5C3 × 4C2 = 60
We get N4 = 5C4 × 4C1 = 20
N5 = 5C5 = 1
1
dx = dt \ N1 + N2 + N3 + N4 + N5 = 126
2 4+ 9 + x ·2 9+ x ·2 x 43. (B) Total number of non negative solutions of x + y + z
= 10 are = 12C2 = 66 (using n+r–1Cr–1)
\ ò dy = ò dt Þ y= t + c If z is even then there can be following cases:
z = 0 Þ No. of ways of solving x + y = 10 Þ 11C1
or y= z = 2 Þ No. of ways of solving x + y = 8 Þ 9C1
4+ 9+ x +C
z = 4 Þ No. of ways of solving x + y = 6 Þ 7C1
y(0) = 7 Þ C=0 z = 6 Þ No. of ways of solving x + y = 4 Þ 5C1
z = 8 Þ No. of ways of solving x + y = 2 Þ 3C1
\ y= 4+ 9+ x z = 10 Þ No. of ways of solving x + y = 0 Þ 1
\ Total ways when z is even = 11 + 9 + 7 + 5 + 3 + 1 = 36
\ y(256) = 3
40. (D) f " (x) > 0, " x Î R 36 6
\ Required probability = =
æ 1ö 1 66 11
f ç ÷ = , f (1) = 1
è 2ø 2 sin 2x
44. g(x) = òsin x sin -1 ( t ) dt
\ f ' is an increasing function on R.
By Lagrange's Mean Value theorem. Þ g'(x) = sin –1(sin 2x) · 2 cos 2x – sin–1(sin x) · cos x
æ pö æ pö p
g ' ç ÷ = sin –1(sin p) · 2 cos p – sin–1 çè sin ÷ø cos
è 2ø 2 2
JEE Advanced 2017 Solved Paper 2017-35

=0 = –2 × (Some value oscillating between –1 and 1)


æ pö \ does not exist.
g ' ç - ÷ = sin–1(sin(–p)) · 2cos(–p) –
è 2ø
cos 2x cos 2x sin 2x
æ æ -p ö ö æ -p ö - cos x cos x - sinx
sin–1 çè sin çè 2 ÷ø ÷ø · cos çè 2 ÷ø = 0 48. (B, C) f(x) =
sin x sin x cos x
\ None of the options are matching here.
45. (A, C) If we consider tan a/2 = x and tan b/2 = y, then Operating C1 ® C1 – C2
2(cos b – cos a) + cos a cos b = 1

é1 - y2 1 - x 2 ù
Þ 2ê - ú = 1 –
1 - x 2 1 - y2 ( )( ) 0 cos 2x
Þ f (x) = -2 cos x cos x
sin 2x
- sin x
2 2
êë1 + y 1 + x úû 1 + x 2 1 + y2 ( )( ) 0 sin x cos x
2 2 2 2
Þ 2[(1 + x )(1 – y ) – (1 – x ) (1 + y )]
= (1 + x2) (1 + y2) – (1 – x2) (1 – y2) Þ f(x) = 2 cos 3x cos x
Þ 4(x – y ) = 2(x2 + y2)
2 2
Þ f(x) = cos 4x + cos 2x
Þ x2 = 3y2 f max = 2 at x = 0
Þ x= ± 3 y f ' (x) = – 4 sin 4x – 2 sin 2x
a b = – 2sin 2x [4 cos 2x + 1]
Þ tan ± 3 tan = 0
2 2 -1
46. (A, C) f ' (x) – 2 f (x) > 0 f ' (x) = 0 Þ sin 2x = 0 or cos 2x =
4
df ( x )
Þ e–2x - 2e -2x f ( x ) > 0 p p
dx Þ x= - , 0, which is true for some x Î (–p, p)
2 2
Þ
dx
(
d -2x
e f (x) > 0 ) \ f ' (x) = 0 at more than three points in (–p, p)
Þ e–2x f(x) is an increasing function. α 1 1
ò0 (x - x )dx = 2 ò0 (x – x
3 3
49. (B, C) )dx
\ for x > 0
f (x) > f (0)
Þ e–2xf(x) > 1 Y

Þ f(x) > e2x in (0, ¥) 2


Also f '(x) > 2e2x > 0
\ f is an increasing function in (0, ¥) 1
1 - (1 - h ) [1 + h ] æ 1ö
47. (A, D) lim- f(x) = hlim
®0
cos ç ÷
è hø X' X
x ®1 h O
-2 -1 1 2

1 -1 + h2 æ 1ö -1
= lim cos ç ÷
h ®0 h è hø
x=a
-2
æ 1ö
= lim h cos çè ÷ø = 0 Y'
h ®0 h
a 1
æ x2 x4 ö 1 æ x2 x4 ö
lim f(x) = lim 1 - (1 + h )(1 + h ) cos æ 1 ö Þ ç - ÷ = ç - ÷
x ®1+
çè ÷ø è 2 4ø 2è 2 4ø
h®0 h h 0 0

a 2 a4 1 æ 1 1 ö
-2h - h 2 æ 1ö Þ - = ç - ÷
= lim cos ç ÷ 2 4 2 è 2 4ø
h ®0 h è hø
2a 2 - a 4 1
Þ = or 4a2 – 2a4 = 1
æ 1ö 4 8
= lim ( -2 - h ) cos çè ÷ø Þ 4 2
2a – 4a + 1 = 0
h ®0 h
EBD_7809
2017-36 JEE Advanced 2017 Solved Paper
P
4 ± 16 - 8 1
Þ a2 = =1±
4 2
1
\ 0 < a < 1 Þ a2 = 1 –
2 P+Q
1
Þ a = 1 - 0.71 = 0.29 > 0.25 = 2 also a < 1 Q
R
uuur uuur uuur
1 QR uuur RP uuur PQ
Þ <a<1 uuur uuur , OY = uuur , OZ = uuur
2 \ OX = QR RP PQ
98
k +1 k +1
50. (B, D) I = å òk x ( x + 1)
dx uuur uuur
uuur uuur
OR ´ RP
k =1
\ OX ´ OY = uuur uuur
Let x – k = t Þ dx = dt QR RP
98
k +1 uuur uuur
QR RP sin ( P + Q)
1
\ I= å ò0 ( t + k )( t + k + 1) dt uuur uuur
k =1 =
QR RP
98
1 k +1 = sin (P + Q)
Þ I> å ò0 dt
k =1 ( t + k + 1) 2 52. (B) cos (P + Q) + cos (Q + R) + cos (R + P)
= cos (180 – R) + cos (180 – P) + cos (180 – Q)
98 1 = –[cos P + cos Q + cos R]
æ -1 ö
Þ I> å ( k + 1) çè t + k + 1÷ø 3
k =1 0 In any DPQR, cos P + cos Q + cos R <
2
98
æ 1 1 ö
Þ I> å ( k + 1) çè k + 1 - k + 2 ÷ø Þ – (cos P + cos Q + cos R) > –
3
k =1 2
98
1 1 1 1 3
\ Required minimum value = –
Þ I> å k + 2 = 3 + 4 + ¼ + 100 2
k =1
53. (B) Q a, b are roots of x2 – x – 1 = 0
1 1 1 98 49 \ a2 – a – 1 = 0, b2 – b – 1 = 0
Þ I> + + ¼+ = = Þ a2 = a + 1 and b2 = b + 1
100 100 100 100 50
Also a n = pan + qbn
49 Þ a0 =p+q
Þ I>
50 a1 = pa + pb
k +1 k +1 a2 = pa2 + qb2 = p(a + 1) + q(b + 1)
Also x x + 1 < x k + 1 [Q least value of x + 1 is k + 1] = (pa + qb) + (p + q) = a1 + a0
( ) ( ) a 3 = pa3 + qb3 = pa (a + 1) + qb(b + 1)
k +1 1 = (pa2 + qb2) + (pa + qb)
Þ <
x ( x + 1) x = a2 + a1
Proceeding in the same manner, we get
98 98 a12 = a11 + a10
k +1 k +1 k +1 1
\ I= å òk x ( x + 1)
dx < å òk x
dx 54. (D) a 4 = a3 + a2 = a2 + a1 + a2 = 2a2 + a1
k =1 k =1
= 2a1 + 2a0 + a1 = 3a1 + 2a0
98 = 3(pa + qb) + 2(p + q)
æ k + 1ö
Þ I< å log e çè k ø
÷ é æ1+ 5 ö æ1- 5 ö ù
k =1 = 3 êp ç + qç
÷ ÷ ú + 2(p + q)
êë è 2 ø è 2 ø úû
æ2 3 4 99 ö
Þ I < loge çè ´ ´ ´¼´ ÷ø
1 2 3 98 7 3
Þ I < loge99
( p + q ) + ( p - q ) 5 = 28
=
2 2
uuur uuur uuur Þ p=q=4 \ p + 2q = 12
51. (A) OX, OY, OZ are unit vectors in the directions of
uuur uuur uuur
sides OR, RP and PQ respectively,,
JEE ADVANCED 2016
1. The question paper consists of three parts (Physics, Chemistry and Mathematics). Each part consists of three
sections.
2. Section 1 contains 5 questions. Each question has four choices (a), (b), (c) and (d). ONLY ONE of these four
options is correct.
4. Section 3 contains 5 questions. The answer to each of the questions is a single-digit integer ranging from 0 to 9
(both inclusive).
3. Section 2 contains 8 multiple choice questions. Each question has four choices (a), (b), (c) and (d) out of which
ONE OR MORE THAN ONE are correct.

PAPER - 1

PHYSICS 3. A water cooler of storage capacity 120 litres can cool water
at a constant rate of P watts. In a closed circulation system
SECTION - I (as shown schematically in the figure), the water from the
cooler is used to cool an external device that generates
This section contains 5 multiple choice questions. Each question
constantly 3 kW of heat (thermal load). The temperature of
has four choices (a), (b), (c) and (d) out of which ONLY ONE
water fed into the device cannot exceed 30°C and the entire
option is correct.
stored 120 litres of water is initially cooled to 10°C. The
1. In a historical experiment to determine Planck's constant, a entire system is thermally insulated. The minimum value of
metal surface was irradiated with light of different P (in watts) for which the device can be operated for 3 hours
wavelengths. The emitted photoelectron energies were is
measured by applying a stopping potential. The relevant
data for the wavelength (l) of incident light and the
corresponding stopping potential (V0) are given below :
l (mm) V0 (Volt)
0.3 2.0
0.4 1.0 (Specific heat of water is 4.2 kJ kg–1K–1 and the density of
0.5 0.4 water is 1000 kg m–3)
(a) 1600 (b) 2067
Given that c = 3 × 108m s–1 and e = 1.6 × 10–19 C , Planck's (c) 2533 (d) 3933
constant (in units of J s) found from such an experiment is 4. A parallel beam of light is incident from air at an angle a on
(a) 6.0 × 10–34 (b) 6.4 × 10–34 the side PQ of a right angled triangular prism of refractive
(c) 6.6 × 10 –34 (d) 6.8 × 10–34
index n = 2 . Light undergoes total internal reflection in
2. A uniform wooden stick of mass 1.6 kg and length l rests in
an inclined manner on a smooth, vertical wall of height h(< the prism at the face PR when a has a minimum value of 45°.
l) such that a small portion of the stick extends beyond the The angle q of the prism is
wall. The reaction force of the wall on the stick is
perpendicular to the stick. The stick makes an angle of 30° P
with the wall and the bottom of the stick is on a rough floor.
The reaction of the wall on the stick is equal in magnitude to q
the reaction of the floor on the stick. The ratio h/l and the a
frictional force f at the bottom of the stick are
(g = 10 m s–2)
n= 2
h 3 16 3 h 3 16 3
(a) = ,f = N (b) = ,f = N Q R
l 16 3 l 16 3

h 3 3 8 3 h 3 3 16 3 (a) 15° (b) 22.5°


(c) = ,f = N (d) = ,f = N (c) 30° (d) 45°
l 16 3 l 16 3
EBD_7809
2016-2 JEE Advanced 2016 Solved Paper
5. An infinite line charge of uniform electric charge density l (c) The plot below represents schematically the variation
lies along the axis of an electrically conducting infinite of beat frequency with time
cylindrical shell of radius R. At time t = 0, the space inside
the cylinder is filled with a material of permittivity e and v(t)
electrical conductivity s. The electrical conduction in the
material follows Ohm's law. Which one of the following P
graphs best describes the subsequent variation of the Q
magnitude of current density j(t) at any point in the material? vQ

j(t)
j(t) R
t
(a) (b)
(d) The plot below represents schematically the variation
of beat frequency with time
t
t (0, 0)
(0, 0) v(t)

j(t) j(t)
P

vQ
(c) (d) Q

t t
R
(0, 0) (0, 0)
t
SECTION - II 8. An incandescent bulb has a thin filament of tungsten that is
This section contains 8 multiple choice questions. Each question heated to high temperature by passing an electric current.
has 4 choices (a), (b), (c) and (d) out of which ONE or MORE The hot filament emits black–body radiation. The filament
is observed to break up at random locations after a
THAN ONE are correct.
sufficiently long time of operation due to non–uniform
6. Highly excited states for hydrogen–like atoms (also called evaporation of tungsten from the filament. If the bulb is
Rydberg states) with nuclear charge Ze are defined by their powered at constant voltage, which of the following
principal quantum number n, where n>>1. Which of the statement(s) is(are) true?
following statement(s) is(are) true? (a) The temperature distribution over the filament is
(a) Relative change in the radii of two consecutive orbitals uniform
does not depend on Z (b) The resistance over small sections of the filament
(b) Relative change in the radii of two consecutive orbitals decreases with time
varies as 1/n (c) The filament emits more light at higher band of
(c) Relative change in the energy of two consecutive frequencies before it breaks up
orbitals varies as 1/ n 3 (d) The filament consumes less electrical power towards
(d) Relative change in the angular momenta of two the end of the life of the bulb
9. A plano–convex lens is made of a material of refractive index
consecutive orbitals varies as 1/n
n. When a small object is placed 30 cm away in front of the
7. Two loudspeakers M and N are located 20 m apart and emit
curved surface of the lens, an image of double the size of
sound at frequencies 118 Hz and 121 Hz, respectively. A car the object is produced. Due to reflection from the convex
is initially at a point P , 1800 m away from the midpoint Q of surface of the lens, another faint image is observed at a
the line MN and moves towards Q constantly at 60 km/hr distance of 10 cm away from the lens. Which of the following
along the perpendicular bisector of MN. It crosses Q and statement(s) is(are) true?
eventually reaches a point R, 1800 m away from Q. Let v(t) (a) The refractive index of the lens is 2.5
represent the beat frequency measured by a person sitting (b) The radius of curvature of the convex surface is 45 cm
in the car at time t. Let vP , vQ and vR be the beat frequencies (c) The faint image is erect and real
measured at locations P, Q and R, respectively. The speed (d) The focal length of the lens is 20 cm
of sound in air is 330 ms–1 . Which of the following 10. A length–scale (l) depends on the permittivity (e) of a
statement(s) is(are) true regarding the sound heard by the dielectric material. Boltzmann constant (kB), the absolute
person? temperature (T), the number per unit volume (n) of certain
(a) vP + vR = 2 vQ charged particles, and the charge (q) carried by each of the
(b) The rate of change in beat frequency is maximum when particles. Which of the following expression(s) for I is(are)
dimensionally correct?
the car passes through Q
JEE Advanced 2016 Solved Paper 2016-3

æ nq 2 ö æ ek T ö
(a) l= ç ÷ (b) l = ç B2 ÷
ç ek BT ÷ ç nq ÷
è ø è ø

æ q2 ö æ q2 ö
(c) l= ç ÷ (d) l= ç ÷
ç en 2/3 k T ÷ ç en1/3 k T ÷
è B ø è B ø

11. A conducting loop in the shape of a right angled isosceles


triangle of height 10 cm is kept such that the 90° vertex is
very close to an infinitely long conducting wire (see the Which of the following statement(s) is(are) true?
figure). The wire is electrically insulated from the loop. The (a) nl sinqi = n2 sinqf
hypotenuse of the triangle is parallel to the wire. The current (b) n1 sin qi = (n 2 – n 1) sinqf
in the triangular loop is in counterclockwise direction and (c) l is independent of n2
increased at a constant rate of 10 A s–1. Which of the (d) l is dependent on n(z)
following statement(s) is(are) true?
SECTION - III

10 cm This section contains 5 questions. Each question, when worked


90°
out will result in one integer from 0 to 9 (both inclusive).

æ m0 ö 14. A metal is heated in a furnace where a sensor is kept above


(a) The magnitude of induced emf in the wire is çè ÷ø the metal surface to read the power radiated (P) by the metal.
p
The sensor has a scale that displays log 2, (P/P0 ), where P0
volt
is a constant. When the metal surface is at a temperature of
(b) If the loop is rotated at a constant angular speed about
487°C, the sensor shows a value 1. Assume that the
æ m0 ö emissivity of the metallic surface remains constant. What is
the wire, an additional emf of çè ÷ø volt is induced in the value displayed by the sensor when the temperature of
p
the wire the metal surface is raised to 2767°C?
(c) The induced current in the wire is in opposite direction 15. The isotope 12 having a mass 12.014 u undergoes
5 B
to the current along the hypotenuse
(d) There is a repulsive force between the wire and the b–decay to 12 12 has an excited state of the nucleus
6 C. 6 C
loop
12. The position vector rr of a particle of mass m is given by the
following equation
( 126 C*) at 4.041 MeV above its ground state. If 125 E decays
r 12
r (t) = at 3ˆi + bt 2ˆj , to 6 C * , the maximum kinetic energy of the b–particle in
where a = 10/3 ms–3, b = 5 ms–2 and m = 0.1 kg. At t = 1 s, units of MeV is (1 u = 931.5 MeV/c2, where c is the speed of
which of the following statement(s) is(are) true about the light in vacuum).
particle? 16. A hydrogen atom in its ground state is irradiated by light of
(a) The velocity vr is given by vr = (10iˆ + 10ˆj) ms–1 wavelength 970 Å. Taking hc/e = 1.237 × 10–6 eV m and the
r ground state energy of hydrogen atom as –13.6 eV, the
(b) The angular momentum L with respect to the origin is number of lines present in the emission spectrum is
r 17. Consider two solid spheres P and Q each of density 8 gm
given by L = – 5/3) k̂ N m s
r r cm–3 and diameters 1 cm and 0.5 cm, respectively. Sphere P
(c) The force F is given by F = (iˆ + 2ˆj) N is dropped into a liquid of density 0.8 gm cm–3 and viscosity
r h = 3 poiseulles. Sphere Q is dropped into a liquid of density
(d) The torque t with respect to the origin is given by
r 1.6 gm cm–3 and viscosity h = 2 poiseulles. The ratio of the
t = – (20/3) k̂ N m terminal velocities of P and Q is
13. A transparent slab of thickness d has a refractive index n(z) 18. Two inductors L1 (inductance 1 mH, internal resistance 3
that increases with z. Here z is the vertical distance inside
W) and L2 (inductance 2 mH, internal resistance 4 W), and a
the slab, measured from the top. The slab is placed between
resistor R (resistance 12 W) are all connected in parallel
two media with uniform refractive indices n1 and n 2 (> n1),
across a 5 V battery. The circuit is switched on at time t = 0.
as shown in the figure. A ray of light is incident with angle
qi, from medium 1 and emerges in medium 2 with refraction The ratio of the maximum to the minimum current
angle qt with a lateral displacement l. (Imax / I min) drawn from the battery is
EBD_7809
2016-4 JEE Advanced 2016 Solved Paper
CHEMISTRY (c) higher the magnitude of activation energy, stronger is
the temperature dependence of the rate constant.
SECTION - I (d) the pre–exponential factor is a measure of the rate at
which collisions occur, irrespective of their energy.
This section contains 5 multiple choice questions. Each question
25. A plot of the number of neutrons (N ) against the number of
has four choices (a), (b), (c) and (d) out of which ONLY ONE
protons (P) of stable nuclei exhibits upward deviation from
option is correct.
linearity for atomic number, Z > 20. For an unstable nucleus
19. P is the probability of finding the 1s electron of hydrogen having N/P ratio less than 1, the possible mode(s) of decay
atom in a spherical shell of infinitesimal thickness, dr, at a is(are)
distance r from the nucleus. The volume of this shell is (a) b– –decay (b emission)
4p r2dr. The qualitative sketch of the dependence of P on r is (b) orbital or K–electron capture
(c) neutron emission
P P (d) b-–decay (positron emission)
26. The crystalline form of borax has
(a) tetranuclear [B4O5(OH)4]2– unit
(b) all boron atoms in the same plane
(a) (b) (c) equal number of sp2 and sp3 hybridized boron atoms
(d) one terminal hydroxide per boron atom
27. The compound(s) with TWO lone pairs of electrons on the
0 r 0 r
central atom is(are)
(a) BrF5 (b) ClF3
P P (c) XeF4 (d) SF4
28. The reagent(s) that can selectively precipitate S2– from a
mixture of S2– and SO42– in aqueous solution is(are)
(a) CuCl2 (b) BaCl2
(c) (d) (c) Pb(OOCCH3)2 (d) Na2[Fe(CN)5NO]
29. Positive Tollen’s test is observed for
0 r 0 r H O
20. One mole of an ideal gas at 300 K in thermal contact with CHO
surroundings expands isothermally from 1.0 L to 2.0 L against (a) H (b)
a constant pressure of 3.0 atm. In this process, the change H
in entropy of surroundings (DSsurr) in JK–1 is H
(1 L atm = 101.3 J)
OH
(a) 5.763 (b) 1.013 O
(c) –1.013 (d) –5.763 Ph
(c) Ph (d)
21. The increasing order of atomic radii of the following Group
13 elements is O Ph Ph
(a) Al < Ga < In < Tl (b) Ga < Al < In < Tl 30. The product(s) of the following reaction sequence is(are)
(c) Al < In < Ga < Tl (d) Al < Ga < Tl < In
NH2
22. Among [Ni(CO) 4 ], [NiCl 4 ] 2– , [Co(NH 3 ) 4 Cl 2 ]Cl,
(i) Acetic anhydride/pyridine
Na 3[CoF 6 ], Na 2 O 2 and CsO 2, the total number of (ii) KBrO 3/HBr
paramagnetic compounds is
(iii) H 3O +, heat
(a) 2 (b) 3
(iv) NaNO 2/ HCl, 273-278 K
(c) 4 (d) 5
(v) Cu/HBr
23. On complete hydrogenation, natural rubber produces
(a) ethylene–propylene copolymer Br
(b) vulcanised rubber Br
(c) polypropylene
(d) polybutylene
(a) (b)
SECTION - II
Br Br
This section contains 8 multiple choice questions. Each question
has 4 choices (a), (b), (c) and (d) out of which ONE or MORE
THAN ONE are correct. Br
24. According to the Arrhenius equation, Br Br Br
(a) a high activation energy usually implies a fast reaction. Br Br
(b) rate constant increases with increase in temperature. (c) (d)
This is due to a greater number of collisions whose
energy exceeds the activation energy. Br
JEE Advanced 2016 Solved Paper 2016-5

31. The correct statement(s) about the following reaction the equation x2 + 2x tan q – 1 = 0. If a1 > b1 and a2 > b2, then
sequence is(are) a1 + b2 equals
(i) O CHCl / NaOH (a) 2 (sec q – tan q) (b) 2 sec q
Cumene(C9 H12 ) ¾¾¾¾
2 ¾
+
® P ¾¾¾¾¾¾
3 ® (c) –2 tanq (d) 0
(ii)H3O
38. A debate club consists of 6 girls and 4 boys. A team of 4
Q (major) + R (minor) members is to be selected from this club including the
NaOH selection of a captain (from among these 4 members) for the
Q ¾¾¾¾¾
PhCH Br
®S
team. If the team has to include at most one boy, then the
2
(a) R is steam Volatile number of ways of selecting the team is
(b) Q gives dark violet coloration with 1% aqueous FeCl3 (a) 380 (b) 320
solution (c) 260 (d) 95
(c) S gives yellow precipitate with 2, 4–dinitro- ì pü
phenylhydrazine 39. Let S = íx Î( -p, p) : x ¹ 0, ± ý . The sum of all distinct
î 2þ
(d) S gives dark violet coloration with 1% aqueous FeCl3
solutions of the equation 3 sec x + cosec x + 2(tan x – cot
solution x) = 0 in the set S is equal to
SECTION - III
7p 2p
This section contains 5 questions. Each question, when worked (a) – (b) -
9 9
out will result in one integer from 0 to 9 (both inclusive).
5p
(c) 0 (d)
32. The mole fraction of a solute in a solution is 0.1. At 298 K, 9
molarity of this solution is the same as its molality. Density 40. A computer producing factory has only two plants T1 and
of this solution at 298 K is 2.0 g cm–3 . The ratio of the T2. Plant T1 produces 20% and plant T2 produces 80% of
the total computers produced. 7% of computers produced
æ MWsolute ö in the factory turn out to be defective. It is known that
molecular weights of the solute and solvent, ç MW ÷, P (computer turns out to be defective given that it is produced
è solvent ø
in plant T1)
is = 10P
33. The diffusion coefficient of an ideal gas is proportional to (computer turns out to be defective given that it is produced
its mean free path and mean speed. The absolute temperature in plant T2),
of an ideal gas is increased 4 times and its pressure is where P(E) denotes the probability of an event E. A computer
increased 2 times. As a result, the diffusion coefficient of produced in the factory is randomly selected and it does
this gas increases x times. The value of x is not turn out to be defective. Then the probability that it is
34. In neutral or faintly alkaline solution, 8 moles of permanganate produced in plant T2 is
anion quantitatively oxidize thiosulphate anions to produce
X moles of a sulphur containing product. The magnitude of 36 47
(a) (b)
X is 73 79
35. The number of geometric isomers possible for the complex 78 75
[CoL2Cl2]– (L = H2NCH2CH2O–) is (c) (d)
93 83
36. In the following monobromination reaction, the number of
possible chiral products is 1
41. The least value of a Î ¡ for which 4ax2 + ³ 1 , for all x > 0,
x
CH2CH2CH3
is
H Br Br2 (1.0 mole)
300°C 1 1
CH3 (a) (b)
64 32
(1.0 mole)
(enantiomerically pure) 1 1
(c) (d)
27 25
MATHEMATICS
SECTION - II
SECTION - I
This section contains 8 multiple choice questions. Each question
This section contains 5 multiple choice questions. Each question has 4 choices (a), (b), (c) and (d) out of which ONE or MORE
has four choices (a), (b), (c) and (d) out of which ONLY ONE THAN ONE are correct.
option is correct.
42. Consider a pyramid OPQRS located in the first octant (x ³ 0,
p p y ³ 0, z ³ 0) with O as origin, and OP and OR along the x–axis
37. Let - < q < - . Suppose a1 and B1are the roots of the and the y–axis, respectively. The base OPQR of the pyramid
6 12
is a square with OP = 3. The point S is directly above the
equation x2 – 2x sec q + 1 = 0 and a2 and b2 are the roots of mid-point T of diagonal OQ such that TS = 3. Then
EBD_7809
2016-6 JEE Advanced 2016 Solved Paper
46. A solution curve of the differential equation (x2 +xy + 4x +2y
p
(a) the acute angle between OQ and OS is
3 dy
+ 4) – y2 = 0, x > 0, passes through the point (1, 3). Then
(b) the equation of the plane containing the triangle OQS dx
is x – y = 0 the solution curve
(c) the length of the perpendicular from P to the plane (a) intersects y = x + 2 exactly at one point
(b) intersects y = x + 2 exactly at two points
3
containing the triangle OQS is (c) intersects y = (x + 2)2
2 (d) does NOT intersect y = (x +3)2
(d) the perpendicular distance from O to the straight line 47. Let f : ¡ ® ¡ , g : ¡ ® ¡ and h : ¡ ® ¡ be
differentiable functions such that
15
containing RS is
2 f(x) = x3 + 3x + 2, g(f(x)) = x and h (g(g(x))) = x for all x Î ¡ .
Then
43. Let f : (0, ¥) ® ¡ be a differentiable function such that
1
f (x) (a) g¢(2) = (b) h¢(1) = 666
f '(x) = 2 - for all x Î (0, ¥) and f(1) ¹ 1. Then 15
x
(c) h(0) = 16 (d) h(g(3)) = 36
48. 2 2
The circle C1 : x + y = 3, with centre at O, intersects the
æ 1ö æ1ö
(a) lim f ¢ ç ÷ = 1 (b) lim xf ç ÷ = 2 parabola x2 = 2y at the point P in the first quadrant. Let the
x ®0 + è x ø x ®0 + èxø
tangent to the circle C1 at P touches other two circles C2
and C3 at R2 and R3, respectively. Suppose C2 and C3 have
(c) lim x 2 f ¢ (x) = 0 (d) |f(x)| £ 2 for all x Î (0, 2)
x ®0 + equal radii 2 3 and centres Q2 and Q3, respectively. If Q2
and Q3 lie on the y–axis, then
é3 -1 -2ù
ê2 0 a ú (a) Q2Q3 = 12
44. Let P = ê ú , where a Î ¡ . Suppose Q = [qij] is a (b) R2R3 = 4 6
êë3 -5 0 úû
(c) area of the triangle OR2R3 is 6 2
matrix such that PQ = kI, where k Î ¡ , k ¹ 0 and I is the
(d) area of the triangle PQ2Q3 is 4 2
k k2 49. Let RS be the diameter of the circle x2 + y2 = 1, where S is the
identity matrix of order 3. If q23 = – and det(Q) = ,
8 2 point (1, 0). Let P be a variable point (other than R and S) on
then the circle and tangents to the circle at S and P meet at the
(a) a = 0, k = 8 (b) 4a – k + 8 = 0 point Q. The normal to the circle at P intersects a line drawn
(c) det (P adj (Q)) = 2 9 (d) det (Q adj (P)) = 213 through Q parallel to RS at point E. Then the locus of E
45. In a triangle XYZ, let x, y, z be the lengths of sides opposite passes through the point(s)
to the angles X,Y, Z, respectively, and 2s = x + y + z.
æ1 1 ö æ 1 1ö
s-x s-y s-z (a) çè 3 , ÷ø (b) çè , ÷ø
3 4 2
If = = and area of incircle of the triangle
4 3 2
æ1 1 ö æ 1 1ö
8p (c) çè 3 , - ÷ø (d) çè , - ÷ø
XYZ is , then 3 4 2
3
SECTION - III
(a) area of the triangle XYZ is 6 6
This section contains 5 questions. Each question, when worked
35 out will result in one integer from 0 to 9 (both inclusive).
(b) the radius of circumcircle of the triangle XYZ is 6
6 50. The total number of distinct x Î ¡ for which

X Y Z 4
(c) sin sin sin = x x2 1 + x3
2 2 2 35
2x 4x 2 1 + 8x 3
= 10 is
æ X + Yö 3 3x 9x 2 1 + 27x 3
sin 2 ç =
(d) è 2 ÷ø 5
JEE Advanced 2016 Solved Paper 2016-7

51. Let m be the smallest positive integer such that the


coefficient of x2 in the expansion of (1 + x)2 + (1 + x)3 + ... + x 2 sin(bx)
53. Let a, b Î ¡ be such that lim = 1. Then
(1 + x)49 + (1 + mx)50 is (3n + 1) 51C3 for some positive x ® 0 ax - sin x
integer n. Then the value of n is 6 (a + b) equals.
52. The total number of distinct x Î [0, 1] for which
-1 + 3i
x 2 54. Let z = , where i = -1 , and r, s Î {1, 2, 3}. Let
t 2
ò1+ t 4
dt = 2x – 1 is
0 é( - z)r z 2s ù
P = ê 2s ú and I be the identity matrix of order 2.
êë z z r úû
Then the total number of ordered pairs (r, s) for which
P2 = –I is
EBD_7809
2016-8 JEE Advanced 2016 Solved Paper

PAPER - 2
1. The question paper consists of three parts (Physics, Chemistry and Mathematics). Each part consists of three
sections.
2. Section 1 contains 6 questions. Each question has four options (a), (b), (c) and (d). ONLY ONE of these four
options is correct.
3. Section 2 contains 8 multiple choice questions. Each question has four choice (a), (b), (c) and (d) out of which
ONE OR MORE THAN ONE are correct.
4. Section 3 contains 2 paragraphs each describing theory, experiment and data etc. four questions relate to two
paragraphs with two questions on each paragraph. Each question pertaining to a particular passage should have
ONLY ONE correct answer among the four given choices (a), (b), (c) and (d).

PHYSICS process at volume Vf . The amount of heat supplied to the


system in the two-step process is approximately
SECTION - I (a) 112 J (b) 294 J
This section contains 6 multiple choice questions. Each question (c) 588 J (d) 813 J
has four choices (a), (b), (c) and (d) out of which ONLY ONE 4. The ends Q and R of two thin wires, PQ and RS, are soldered
option is correct. (joined) together. Initially each of the wires has a length of 1
m at 10°C. Now the end P is maintained at 10°C, while the
1. The electrostatic energy of Z protons uniformly distributed end S is heated and maintained at 400 °C. The system is
throughout a spherical nucleus of radius R is given by thermally insulated from its surroundings. If the thermal
2 conductivity of wire PQ is twice that of the wire RS and the
3 Z (Z - 1) e coefficient of linear thermal expansion of PQ is 1.2 × 10–5
E=
5 4πε 0R K–1, the change in length of the wire PQ is
(a) 0.78 mm (b) 0.90 mm
The measured masses of the neutron 11 H, 15 15
7 N and 8 O are (c) 1.56 mm (d) 2.34 mm
1.008665 u, 1.007825 u, 15.000109 u and 15.003065 u, 5. A small object is placed 50 cm to the left of a thin convex
15 15 lens of focal length 30 cm. A convex spherical mirror of radius
respectively. Given that the radii of both the 7 N and 8 O of curvature 100 cm is placed to the right of the lens at a
nuclei are same, 1 u = 931.5 Me V/c2 (c is the speed of light) distance of 50 cm.The mirror is tilted such that the axis of
and e2/(4 pe0) = 1.44 MeV fm. Assuming that the difference the mirror is at an angle q =30° to the axis of the lens, as
between the binding energies of 15 15 shown in the figure.
7 N and 8 O is purely due
to the electrostatic energy, the radius of either of the nuclei f = 30 cm
is
(1 fm = 10–15 m)
(a) 2.85 fm (b) 3.03 fm q x
(–50,0) (0, 0) R
(c) 3.42 fm (d) 3.80 fm =
2. An accident in a nuclear laboratory resulted in deposition 10
0
50 cm cm
of a certain amount of radioactive material of half-life 18
days inside the laboratory. Tests revealed that the radiation
was 64 times more than the permissible level required for
safe operation of the laboratory. What is the minimum number (50 + 50 3, -50)
of days after which the laboratory can be considered safe
for use? If the origin of the coordinate system is taken to be at the
(a) 64 (b) 90 centre of the lens, the coordinates (in cm) of the point (x, y)
(c) 108 (d) 120 at which the image is formed are
3. A gas is enclosed in a cylinder with a movable frictionless
piston. Its initial thermodynamic state at pressure Pi = 105 (a) (0, 0) (b) (50 - 25 3, 25)
Pa and volume Vi = 10–3 m3 changes to a final state at Pf = (1/
32) × 105 Pa and Vf = 8 × 10–3 m3 in an adiabatic quasi-static (c) (25, 25 3) (d) (125 / 3,25 3)
process, such that P3V5 = constant. Consider another 6. There are two Vernier calipers both of which have 1 cm
thermodynamic process that brings the system from the same divided into 10 equal divisions on the main scale. The Vernier
initial state to the same final state in two steps: an isobaric scale of one of the calipers (C1) has 10 equal divisions that
expansion at Pi followed by an isochoric (isovolumetric) correspond to 9 main scale divisions. The Vernier scale of
JEE Advanced 2016 Solved Paper 2016-9

the other caliper (C2) has 10 equal divisions that correspond


to 11 main scale divisions. The readings of the two calipers
are shown in the figure. The measured values (in cm) by
calipers C1 and C2, respectively, are Light

2 3 4
C1
Electrons
0 5 10

2 3 4
V
C2 – +
0 5 10

(a) 2.85 and 2.82 (b) 2.87 and 2.83


(a) le decreases with increase in f and lph
(c) 2.87 and 2.86 (d) 2.87 and 2.87
(b) le is approximately halved, if d is doubled
SECTION - II (c) For large potential difference (V >> f/e), le is
approximately halved if V is made four times
This section contains 8 multiple choice questions. Each question
has 4 choices (a), (b), (c) and (d) out of which ONE or MORE (d) le increases at the same rate as lph for lph < hc/f
THAN ONE are correct. 9. In an experiment to determine the acceleration due to gravity
7. Two thin circular discs of mass m and 4m, having radii of a g, the formula used for the time period of a periodic motion
and 2a, respectively, are rigidly fixed by a massless, rigid
7 (R - r)
rod of length l = 24a through their centres. This assembly is T = 2p . The values of R and r are measured to
5g
is laid on a firm and flat surface, and set rolling without
slipping on the surface so that the angular speed about the be (60 ± 1) mm and (10 ± 1) mm, respectively. In five
axis of the rod is w. The angular momentum of the entire successive measurements, the time period is found to be
ur
assembly about the point ‘O’ is L (see the figure). Which 0.52s, 0.56s, 0.57s, 0.54s and 0.59s. The least count of the
of the following statement(s) is (are) true? watch used for the measurement of time period is 0.01s.
Which of the following statement(s) is (are) true?
4m
(a) The error in the measurement of r is 10%
(b) The error in the measurement of T is 3.75%
m (c) The error in the measurement of T is 2%
z w
l (d) The error in the determined value of g is 11%
2a
l a 10. Consider two identical galvanometers and two identical
O resistors with resistance R. If the internal resistance of the
galvanometers RC < R/2, which of the following statement(s)
about any one of the galvanometers is (are) true?
(a) The centre of mass of the assembly rotates about the
(a) The maximum voltage range is obtained when all the
z-axis with an angular speed of w/5
(b) The magnitude of angular momentum of center of mass components are connected in series
of the assembly about the point O is 81 ma2w (b) The maximum voltage range is obtained when the two
(c) The magnitude of angular momentum of the assembly resistors and one galvanometer are connected in series,
about its center of mass is 17 ma2w/2. and the second galvanometer is connected in parallel
ur
(d) The magnitude of the z-component of L is 55 ma2w. to the first galvanometer
8. Light of wavelength lph falls on a cathode plate inside a (c) The maximum current range is obtained when all the
vacuum tube as shown in the figure. The work function of components are connected in parallel
the cathode surface is f and the anode is a wire mesh of
(d) The maximum current range is obtained when the two
conducting material kept at a distance d from the cathode. A
potential difference V is maintained between the electrodes. galvanometers are connected in series and the
If the minimum de Broglie wavelength of the electrons combination is connected in parallel with both the
passing through the anode is le, which of the following resistors
statement(s) is (are) true?
EBD_7809
2016-10 JEE Advanced 2016 Solved Paper
11. In the circuit shown below, the key is pressed at time t = 0.
Which of the following statement(s) is(are) true?

Screen
z
40 mF 25 kW
O
y

v S1 S2
+ d x
D
A 50 W 20 mF (a) Straight bright and dark bands parallel to the x-axis
(b) The region very close to the point O will be dark
(c) Hyperbolic bright and dark bands with foci
+ – symmetrically placed about O in the x-direction
Key (d) Semi circular bright and dark bands centered at point.
5V
14. A rigid wire loop of square shape having side of length L
and resistance R is moving along the x-axis with a constant
(a) The voltmeter displays –5V as soon as the key is velocity v0 in the plane of the paper. At t = 0, the right edge
pressed, and displays +5V after a long time of the loop enters a region of length 3L where there is a
(b) The voltmeter will display 0V at time t = ln 2 seconds uniform magnetic field B0 into the plane of the paper, as
(c) The current in the ammeter becomes 1/e of the initial shown in the figure. For sufficiently large v0. the loop
value after 1 second eventually crosses the region. Let x be the location of the
(d) The current in the ammeter becomes zero after a long right edge of the loop. Let v(x), I(x) and F(x) represent the
time. velocity of the loop, current in the loop, and force on the
12. A block with mass M is connected by a massless spring loop, respectively, as a function of x. Counter-clockwise
current is taken as positive.
with stiffness constant k to a rigid wall and moves without
friction on a horizontal surface. The block oscillates with
small amplitude A about an equilibrium position x0. Consider
two cases: (i) when the block is at x0; and (ii) when the block
is at x = x0 + A. In both the cases, a particle with mass m(<M) R
is softly placed on the block after which they stick to each L
other. Which of the following statement(s) is (are) true about v0
the motion after the mass m is placed on the mass M?
x
(a) The amplitude of oscillation in the first case changes 0 2L 4L
L 3L
M Which of the following schematic plot(s) is (are) correct?
by a factor of , whereas in the second case it
m+M (Ignore gravity)
remains unchanged.
v(x)
(b) The final time period of oscillation in both the cases is
same. v0
(c) The total energy decreases in both the cases.
(d) The instantaneous speed at x0 of the combined masses (a)
decreases in both the cases
13. While conducting the Young’s double slit experiment, a 0 L 2L 3L 4L x
student replaced the two slits with a large opaque plate in
the x-y plane containing two small holes that act as two I(x)
coherent point sources (S1, S2) emitting light of wavelength
600 nm. The student mistakenly placed the screen parallel
to the x-z plane (for z > 0) at a distance D = 3 m from the mid-
point of S1S2, as shown schematically in the figure. The
3L 4L
distance between the sources d = 0.6003 mm. The origin O is
(b) x
at the intersection of the screen and the line joining S1S2. 0 L 2L
Which of the following is(are) true of the intensity pattern
on the screen?
JEE Advanced 2016 Solved Paper 2016-11

I(x) w

R
(c)
m
0 x
L 2L 3L 4L R/2

F(x)
15. The distance r of the block at time t is
R wt -wt R
(a)
4 (
e +e
) (b)
2
cos wt

R 2wt R
+ e -2wt
4( )
(c) e cos 2wt (d)
x 2
0 L 2L 3L 4L
16. The net reaction of the disc on the block is
(d)
1
mw2 R e2wt - e -2wt ˆj + mgkˆ
(
(a)
2 )
1
mw R (e2 wt
- e-wt ˆj + mgkˆ
(b)
2 )
(c) - mw2 R coswt ˆj - mgkˆ
SECTION - II (d) mw2 R sinswt ˆj - mgkˆ
This section contains 2 paragraphs, each describing theory, PARAGRAPH 2
experiments, data etc. four questions related to the two paragraphs Consider an evacuated cylindrical chamber of height h having
with two questions on each paragraph. Each question has only rigid conducting plates at the ends and an insulating curved
one correct answer among the four given options (a), (b), (c) and surface as shown in the figure. A number of spherical balls made
(d). of a light weight and soft material and coated with a conducting
PARAGRAPH 1 material are placed on the bottom plate. The balls have a radius r
A frame of reference that is accelerated with respect to an inertial <<h. Now a high voltage source (HV) is connected across the
frame of reference is called a non-inertial frame of reference. A conducting plates such that the bottom plate is at +V0 and the
coordinate system fixed on a circular disc rotating about a fixed top plate at –V0 . Due to their conducting surface, the balls will
axis with a constant angular velocity w is an example of non- get charged, will become equipotential with the plate and are
inertial frame of reference. The relationship between the force repelled by it. The balls will eventually collide with the top plate,
r where the coefficient of restitution can be taken to be zero due to
Frot experienced by a particle of mass m moving on the rotating the soft nature of the material of the balls. The electric field in the
r chamber can be considered to be that of a parallel plate capacitor.
disc and the force Fin experienced by the particle in an inertial
Assume that there are no collisions between the balls and the
frame of reference is
r r r ur ur r ur interaction between them is negligible. (Ignore gravity)
( ) ( )
Frot = Fin + 2m v rot ´w + m w´ r ´w. A
r
where v rot is the velocity of the particle in the rotating frame of
r –
reference and r is the position vector of the particle with respect HV
to the centre of the disc.
+
Now consider a smooth slot along a diameter of a disc of radius
R rotating counter-clockwise with a constant angular speed w
about its vertical axis through its center. We assign a coordinate 17. Which one of the following statements is correct?
system with the origin at the center of the disc, the x-axis along (a) The balls will stick to the top plate and remain there
the slot, the y-axis perpendicular to the slot and the z-axis along (b) The balls will bounce back to the bottom plate carrying
ur
the same charge they went up with
( )
the rotation axis w = wk̂ . A small block of mass m is gently
(c) The balls will bounce back to the bottom plate carrying
r
placed in the slot at r (R / 2) ˆi at t = 0 and is constrained to move the opposite charge they went up with
(d) The balls will execute simple harmonic motion between
only along the slot.
the two plates
EBD_7809
2016-12 JEE Advanced 2016 Solved Paper
18. The average current in the steady state registered by the 22. The geometries of the ammonia complexes of Ni2+ , Pt2+ and
ammeter in the circuit will be Zn2+ respectively, are
(a) zero (a) octahedral, square planar and tetrahedral
(b) proportional to the potential V0 (b) square planar, octahedral and tetrahedral
(c) tetrahedral, square planar and octahedral
(c) proportional to V01/2
(d) octahedral, tetrahedral and square planar
(d) proportional to V02 23. The correct order of acidity for the following compounds is
CO2H CO2H
CO2H CO2H
CHEMISTRY HO OH OH
SECTION - I
OH
This section contains 6 multiple choice questions. Each question OH
I II III IV
has four choices (a), (b), (c) and (d) out of which ONLY ONE
(a) I > II > III > IV (b) III> I > II > IV
option is correct.
(c) III > IV > II > I (d) I > III > IV > II
19. For the following electrochemical cell at 298 K, 24. The major product of the following reaction sequence is
Pt(s)|H2(g, 1 bar)| H+ (aq, 1 M) || M4+ (aq), M2+ (aq)|Pt(s) O
éM 2+ aq ù (i) HCHO (excess)/NaOH, heat
ê
ë ( )ûú x +
Ecell = 0.092 V when é 4+ = 10 . (ii) HCHO/H (catalytic amount)
êM (aq)ùú
ë û O O O O OH
RT (a) (b)
Given : E°M 4- /M 2+ = 0.151 V; 2.303 = 0.059V
F
The value of x is O OH
(a) –2 (b) –1 HO O O
(c) 1 (d) 2
20. The qualitative sketches I, II and III given below show the (c) (d)
variation of surface tension with molar concentration of
OH
three different aqueous solutions of KCl, CH3OH and
SECTION - II
CH3(CH2)11OSO3– Na+ at room temperature. The correct
assignment of the sketches is This section contains 8 multiple choice questions. Each question
has 4 choices (a), (b), (c) and (d) out of which ONE or MORE
I II III THAN ONE are correct.
Surface tension

Surface tension
Surface tension

25. According to Molecular Orbital Theory,


(a) C22 - is expected to be diamagnetic
(b) O22+ is expected to have a longer bond length than O2
Concentration Concentration Concentration
(c) N+2 and N -2 have the same bond order
(a) I : KCl II : CH3OH III : CH3(CH2)11OSO3– Na+
(b) I : CH3(CH2)11OSO3– Na+ II : CH3OH III : KCl (d) He+2 has the same energy as two isolated He atoms
– +
(c) I : KCl II : CH3(CH2)11OSO3 Na III : CH3OH 26. Mixture(s) showing positive deviation from Raoult’s law at
35 °C is (are)
(d) I : CH3OH II : KCl III : CH3(CH2)11OSO3– Na+ (a) carbon tetrachloride + methanol
21. In the following reaction sequence in aqueous solution, the (b) carbon disulphide + acetone
species X, Y and Z, respectively, are (c) benzene + toluene
(d) phenol + aniline
Ag+ Ag + 27. The CORRECT statement(s) for cubic close packed (ccp)
S2 O32 - ¾¾¾¾¾¾¾® X ¾¾¾¾¾¾¾® Y
clear white three dimensional structure is (are)
solution precipitate
(a) The number of the nearest neighbours of an atom
with time
¾¾¾¾¾¾¾¾¾¾¾® Z present in the topmost layer is 12
black precipitate (b) The efficiency of atom packing is 74%
(a) [Ag(S2O3)2]3–, Ag2S2O3, Ag2S (c) The number of octahedral and tetrahedral voids per
(b) [Ag(S2O3)3]5–, Ag2SO3, Ag2S atom are 1 and 2, respectively
(c) [Ag(SO3)2]3–, Ag2S2O3, Ag (d) The unit cell edge length is 2 2 times the radius of
(d) [Ag(SO3)3]3–, Ag2SO4, Ag the atom
JEE Advanced 2016 Solved Paper 2016-13

28. Extraction of copper from copper pyrite (CuFeS2) involves The standard reaction Gibbs energy, DrG°, of this reaction is
(a) crushing followed by concentration of the ore by froth- positive. At the start of the reaction, there is one mole of X 2 and
flotation no X. As the reaction proceeds, the number of moles of X formed
(b) removal of iron as slag is given by b. Thus, bequilibrium is the number of moles of X formed
(c) self-reduction step to produce ‘blister copper’ at equilibrium. The reaction is carried out at a constant total
following evolution of SO2 pressure of 2 bar. Consider the gases to behave ideally.
(d) refining of ‘blister copper’ by carbon reduction (Given R = 0.083 L bar K–1 mol–1)
29. The nitrogen containing compound produced in the reaction 33. The equilibrium constant Kp for this reaction at 298 K, in
of HNO3 with P4O10 terms of bequilibrium, is
(a) can also be prepared by reaction of P4 and HNO3
(b) is diamagnetic 8b2
equilibrium 8b2
equilibrium
(c) contains one N-N bond (a) ¾¾¾¾¾¾¾¾¾¾¾¾¾¾
2-bequilibrium
® (b) ¾¾¾¾¾¾¾¾¾¾¾¾¾¾
2
®
4 -bequilibrium
(d) reacts with Na metal producing a brown gas
30. For ‘invert sugar’, the correct statement(s) is(are) 4b2 4b2
equilibrium
equilibrium
(Given : specific rotations of (+) -sucrose, (+)-maltose, (c) ¾¾¾¾¾¾¾¾¾¾¾¾¾¾®
2 -bequilibrium
(d) ¾¾¾¾¾¾¾¾¾¾¾¾¾¾®
2
4 -bequilibrium
L-(-)-glucose and L-(+) fructose in aqueous solution are
+ 66°, +140°, –52° and +92°, respectively) 34. The INCORRECT statement among the following, for this
(a) ‘invert sugar’ is prepared by acid catalyzed hydrolysis reaction, is
of maltose (a) Decrease in the total pressure will result in formation
(b) ‘invert sugar’ is an equimolar mixture of D-(+)-glucose of more moles of gaseous X
and D-(-)-fructose (b) At the start of the reaction, dissociation of gaseous
(c) specific rotation of ‘invert sugar’ is –20° X2 takes place spontaneously
(d) on reaction with Br 2 water, ‘invert sugar’ forms (c) bequilibrium = 0.7
saccharic acid as one of the products
(d) KC < 1
31. Reagent(s) which can be used to bring about the following
PARAGRAPH 2
transformation is (are)
Treatment of compound O with KMnO4/H+ gave P, which on
heating with ammonia gave Q. The compound Q on treatment
O O O with Br2/NaOH produced R. On strong heating, Q gave S, which
O O C
C on further treatment with ethyl 2-bromopropanoate in the
presence of KOH followed by acidification, gave a compound T.
H OH
O O
COOH COOH
(a) LiAlH4 in (C2H5)2O (b) BH3 in THF
(c) NaBH4 in C2H5OH (d) Raney Ni/H2 in THF
32. Among the following, reaction(s) which gives(give)
tert-butyl benzene as the major product is(are)
Br Cl
(O)
(a) (b)
NaOC2H5 AlCl3
35. The compound R is
OH O
(c) (d) NH2
H2SO4 BF3OEt2 Br
(a) (b) Br
NH2
SECTION - III O
This section contains 2 paragraphs, each describing theory,
experiments, data etc. four questions related to the two paragraphs O O
with two questions on each paragraph. Each question has one or NHBr
NBr
more than one correct answer(s) among the four given options (c) NHBr (d)
(a), (b), (c) and (d).
O O
PARAGRAPH 1 36. The compound T is
Thermal decomposition of gaseous X2 to gaseous X at 298 K (a) glycine (b) alanine
takes place according to the following equation:
(c) valine (d) serine
X 2 (g) ƒ 2X (g)
EBD_7809
2016-14 JEE Advanced 2016 Solved Paper
MATHEMATICS SECTION - II
This section contains 8 multiple choice questions. Each question
SECTION - I
has 4 choices (a), (b), (c) and (d) out of which ONE or MORE
This section contains 6 multiple choice questions. Each question THAN ONE are correct.
has four choices (a), (b), (c) and (d) out of which ONLY ONE
option is correct. x
æ ön
ç æ nö æ nö ÷
é 1 0 0ù n n ( x + n ) ç x + ÷ ... ç x + ÷
ç è 2ø è nø ÷
ê ú 43. Let f ( x ) = lim ç , for
37. Let P = ê 4 1 0 ú and I be the identity matrix of order 3. æ 2ö æ 2 ö÷
( )
n ®¥ ç
2 2 2 n 2 n ÷
êë16 4 1 úû ç n! x + n çç x + 4 ÷÷ ... çç x + n 2 ÷÷ ÷
è è ø è øø
q 31 + q32 all x > 0. Then
If Q = [qij] is a matrix such that P50 – Q =I, then
q 21
æ1ö æ1ö æ2ö
equals (a) f ç ÷ ³ f (1) (b) f ç ÷ £ f ç ÷
(a) 52 (b) 103 è2ø è 3ø è3ø
(c) 201 (d) 205
38. Let bi > 1 for i = 1, 2, ..., 101. Suppose loge b1, logeb2, ...., loge f ' ( 3) f '(2)
(c) f ' (2) < 0 (d) ³
b101 are in Arithmetic Progression (A.P.) with the common f ( 3) f ( 2)
difference loge 2. Suppose a1, a2, ...., a101 are in A.P. such that
a1= b1 and a51= b51. If t= b1+b2 + .... + b51 and s= a1+a2+ .... + 44. Let a, b Î ¡ and f : ¡ ® ¡ be defined by
a51, then f (x) = a cos (|x3 –x|) + b |x| sin (|x3 +x|).
(a) s > t and a101> b101 (b) s > t and a101 < b101 Then f is.
(c) s < t and a101 > b101 (d) s < t and a101 < b101 (a) differentiable at x = 0 if a = 0 and b = 1
13
1 (b) differentiable at x = 1 if a = 1 and b = 0
39. The value of å
k =1 æ p ( k - 1) p ö æ p kp ö
is equal (c) NOT differentiable at x = 0 if a = 1, b = 0
sin ç + ÷ sin ç + ÷ (d) NOT differentiable at x = 1 if a = 1 and b = 1
è4 6 ø è4 6 ø 45. Let f : ¡ ® (0, ¥) and g : ¡ ® ¡ be twice differentiable
to functions such that f" and g" are continuous functions on
(a) 3- 3 ( )
(b) 2 3 - 3 ¡ . Suppose f '(2) = g(2) = 0, f"(2) ¹ 0 and g'(2) ¹ 0. If
f (x)g(x)
(c) 2 ( 3 -1 ) (d) 2 (2 - 3 ) lim
x ®2 f '(x)g '(x)
= 1 , then.

p (a) f has a local minimum at x = 2


x 2 cos x
2
(b) f has a local maximum at x = 2
40. The value of ò dx is equal to
p 1+ e
x
(c) f "(2) > f (2)
-
2 (d) f (x) – f "(x) = 0 for at least one x Î R
p2 p2 é 1 ù é 1 ù
(a) -2 (b) +2 46. Let f : ê - , 2ú ® ¡ and g : ê - , 2ú ® ¡ be functions
4 4 ë 2 û ë 2 û
p p
(c) (d) p2 + e 2 defined by f (x) = [x –3] and g(x) = |x| f (x) + |4x–7 | f (x), where
2
p2 - e 2
[y] denotes the greatest integer less than or equal to y for
41. Area of the region {( x, y ) Î ¡ : y ³
2
}
x + 3 , 5y £ x + 9 £ 15 y Î ¡ . Then

is equal to é 1 ù
(a) f is discontinuous exactly at three points in ê - , 2ú
1 4 ë 2 û
(a) (b)
6 3
é 1 ù
3 5 (b) f is discontinuous exactly at four points in ê - , 2ú
ë 2 û
(c) (d)
2 3 (c) g is NOT differentiable exactly at four points in
42. Let P be the image of the point (3,1,7) with respect to the
plane x–y+z = 3. Then the equation of the plane passing æ 1 ö
ç - , 2÷
x y z è 2 ø
through P and containing the straight line = = is.
1 2 1 æ 1 ö
(a) x + y – 3z = 0 (b) 3x + z = 0 (d) g is NOT differentiable exactly at five points in ç - , 2 ÷
è 2 ø
(c) x – 4y + 7z = 0 (d) 2x – y = 0
JEE Advanced 2016 Solved Paper 2016-15

47. Let a, b Î ¡ and a + b ¹ 0. 2 2


SECTION - III
ì 1 ü This section contains 2 paragraphs, each describing theory,
Suppose S = í Z Î C : Z = , + Î ¡, t ¹ 0 ý , where experiments, data etc. four questions related to the two paragraphs
î a + ibt þ
with two questions on each paragraph. Each question has only one
i= -1 . If z = x+ iy and z Î S, then (x, y) lies on correct answer among the four given options (a), (b), (c) and (d).
1 æ 1 ö PARAGRAPH 1
(a) the circle with radius and centre ç , 0 ÷ for a > 0,
2a è 2a ø Football teams T1 and T2 have to play two games against each
b¹ 0 other. It is assumed that the outcomes of the two games are
independent. The probabilities of T1 winning, drawing and losing
1 æ 1 ö
(b) the circle with radius - and centre ç - , 0 ÷ for 1 1 1
2a è 2a ø a game against T2 are , and respectively. Each team gets 3
2 6 3
a < 0, b ¹ 0
(c) the x-axis for a ¹ 0, b = 0 points for a win, 1 point for a draw and 0 point for a loss in a game.
(d) the y-axis for a = 0, b ¹ 0 Let X and Y denote the total points scored by teams T1 and T2
48. Let P be the point on the parabola y2 = 4x which is at the respectively after two games.
shortest distance from the center S of the circle x 2 + y2 – 4x 51. P (X > Y) is
–16y + 64 = 0.
1 5
Let Q be the point on the circle dividing the line segment SP (a) (b)
internally. Then 4 12
(a) SP = 2 5 1 7
(c) (d)
(b) SQ : QP = ( )
5 +1 : 2 2
52. P (X = Y) is
12

(c) the x-intercept of the normal to the parabola at P is 6


11 1
1 (a) (b)
(d) the slope of the tangent to the circle at Q is 36 3
2
49. Let a, l, m,Î ¡ . Consider the system of linear equations 13 1
(c) (d)
ax + 2y = l 36 2
3x – 2y = m
PARAGRAPH 2
Which of the following statement(s) is (are) correct?
(a) If a = –3, then the system has infinitely many solutions Let F1(x1, 0) and F2(x2, 0) for x1 < 0 and x2 > 0, be the foci of the
for all values of l and µ
x2 y 2
(b) If a ¹ –3, then the system has a unique solution for all ellipse + = 1 . Suppose a parabola having vertex at the
9 8
values of l and µ
(c) If l + µ = 0, then the system has infinitely many origin and focus at F2 intersects the ellipse at point M in the first
solutions for a = –3 quadrant and at point N in the fourth quadrant.
(d) If l + µ ¹ 0, then the system has no solution for
a = –3 53. The orthocentre of the triangle F1MN is
Ù Ù Ù
50. Let u = u1i + u 2 j + u 3 k be a unit vector in ¡3 and æ 9 ö æ2 ö
(a) ç - ,0÷ (b) ç , 0 ÷
Ù 1 æÙ Ù Ù ö è 10 ø è3 ø
w= ç i + j + 2k ÷ . Given that there exists a vector v in
®

6è ø æ 9 ö æ2 ö
(c) ç ,0÷ (d) ç , 6 ÷
Ù ®
æ Ù
ö Ù ® è 10 ø è3 ø
¡3 such that u´ v =1 and w . ç u ´ v ÷ =1 . Which of the
è ø 54. If the tangents to the ellipse at M and N meet at R and the
following statement(s) is (are) correct? normal to the parabola at M meets the x-axis at Q, then the
® ratio of area of the triangle MQR to area of the quadrilateral
(a) There is exactly one choice for such v MF1NF2 is
® (a) 3 : 4 (b) 4 : 5
(b) There are infinitely many choices for such v
(c) 5 : 8 (d) 2 : 3
Ù
(c) If u lies in the xy-plane then |u1| = |u2|
Ù
(d) If u lies in the xz-plane then 2 |u1| = |u3|
EBD_7809
2016-16 JEE Advanced 2016 Solved Paper

SOLUTIONS
Paper - 1
PHYSICS h h
But cos 30° = \ x= … (iii)
hc f hc f x cos 30°
1. (b) - = V01 and - = V02
el1 e el 2 e
h 3l h 3 3
From (ii) and (iii) = \ =
hc é 1 1 ù cos 30° 8 l 16
\ e ê l - l ú = V01 - V02
ë 1 2û
mC DT V rC DT
3. (b) Pheater – Pcooler = =
e(V01 - V02 )l1l 2 t t
\ h=
(l 2 – l1 )c 0.12 ´ 1000 ´ 4.2 ´103 ´ 20
From the first two values given in data \ (3000 – Pcooler) =
3 ´ 60 ´ 60
1.6 ´ 10-19 [2 - 1] ´ 0.4 ´ 0.3 ´10 -6 \ Pcooler = 2067W
h=
0.1 ´ 3 ´ 108 4. (a) Applying Snell's law at A

h = 0.64 ´ 10 -33 = 6.4 ´ 10-34 J-s


1 × sin 45° = 2 ´ sin r1
Similarly if we calculate h for the last two values of \ r1 = 30° … (i)
data h = 6.4 × 10–34J-s
2. (d) Considering the normal reaction of the floor and wall
to be N and with reference to the figure.
q

Nsin 30° 90° – q


45° M
N 30° A r1 90°
+q
Ncos 30° c B

30°
n= 2
x
h

l/2 1.6g Applying Snell's law at B


N
2 sin C = 1 × sin 90°
A f B \ C = 45° … (ii)
In D AMB, 90° + q + r1 + (90° – C) = 180° (From fig.)
By vertical equilibrium. \ q = 15°

3.2 g I dV / dR
N + N sin 30° = 1.6 g Þ N = … (i) 5. (c) J= = … (i)
3 2pr l 2 pr l
By horizontal equilibrium
dr 1 dr
dR = r = ´ … (ii)
3 16 3 2pr l s 2 pr l
f = Ncos 30° = N= From (i)
2 3
dV
Taking torque about A we get Now E = -
dr
1.6g × AB = N × x
l 3.2 g 3l l
1.6 g × cos 60° = ´x \ = x … (ii) \ dV = –Edr = – dr … (iii)
2 3 8 2p Î r
JEE Advanced 2016 Solved Paper 2016-17

Relative change in the energy of two consecutive orbitals

En – En -1 E n2 -2 n + 1 - 2
= 1 - n -1 = 1 - 2
= »
En En (n - 1) (n - 1) 2 n
+
+ V
+ Ln - Ln -1 L (n - 1) 1
+ = 1 - n -1 = 1 - =
+ I Ln Ln n n
+ V + dV
l + 7. (a,b, c)
+
+
+ r P
+ vccos q q vc
+ dr
+ A
+ a Vx
+
+ 118Hz c
121Hz
+ 10 Q
+ M N
+

R
From (i), (ii), and (iii)
1 é ldr s 2p r l ù ls é v + v cos q ù
J= ê ´ ú = … (iv) n P = 121 - 118 = ê
2pr l ë 2l Î r dr û 2p Î r ë v úû

dV -l s ´ 2pr l -lsl nq = 121 - 118 = 3


Also I = = dr ´ = …(v)
dR 2 p Î r dr Î
é v - vc cos q ù
Here negative sign signifies that the current is n R = (121 –118) ê úû
decreasing ë v

d ( q ) d ( ll ) dl \ n P + n R = 2 n Q Þ ( A ) is correct option
But I = = =l … (vi)
dt dt dt In general when the car is passing through A
From (v) and (vi) é v + vc cos a ù
n = 3ê úû …(i)
dl lsl d l -s ë v
l =- Þ = dt
dt Î l Îl
dn é v sin a ù dn
On integrating \ = -3 ê c is max when sin a = 1
da ë v úû da
l t
dl s i.e., a =90° (at Q)
ò l
= - ò dt
Î0 Þ (b) is correct option.
l0
d n 3vc da
l st s
- t From (i) = (– sin a) … (ii)
\ loge =- \ l = l0 e Î dt v dt
l0 Î
Substituting this value in (iv) we get 10 2 da 10 dx
Also tan a = \ sec a =- 2
x dt x dt
s
sl0 - Î t
J= e da -10v
2p Î r \ = 2 …(iii)
6. (a, b, d)
dt x sec 2 a
From (ii) & (iii)
2
n2 E = - 13.6 Z nh
We know that r = r0 , n , Ln = dn 3v æ -10v ö 30Vc sin a
z n 2 2p = - c sin a ´ ç 2 ÷=
dt v è x sec 2 a ø x 2 sec 2 a
Relative change in the radii of two consecutive orbitals
rn – rn -1 r (n - 1) 2 dn 30vc sin a
= 1 - n -1 = 1 - \ dt = 2 2
= 0.3vc sin 3 a . At a = 90°
does not depend on Z (10 cot a ) sec a
rn rn n2

2n - 1 2 dn
= » = max
(Q n > > 1) dt
n2 n
\ (c) is the correct option
EBD_7809
2016-18 JEE Advanced 2016 Solved Paper
8. (c, d) With the use of filament and the evaporation involved, A B
m0i
the filament will become thinner thereby decreasing f= = Mi
10p x
the area of cross-section and increasing the resistance.
Therefore the filament will consume less power m0 10 cm x x dx
towards the end of life. \M =
10p
As the evaporation is non-uniform, the area of cross- Induced emf in the wire
section will be different at different cross-section. dI
Therefore temperature distribution will be non- di m 0 m = 10 AS -1
=M = ´ 10 = 0 dt
uniform. The filament will break at the point where the dt 10 p p
temperature is maximum. As the current in the triangular wire is decreasing the
æ 1ö induced current in AB is in the same direction as the
When the filament temperature is higher ç l n µ ÷ , current in the hypotenuse of the triangular wire.
è Tø
Therefore force will be repulsive.
it emits light of lower wavelength or higher band of
r
frequencies. 12. (a, b) r = at 3iˆ + bt 2 ˆj
9. (a, d) For lens
r 10
1 1 1 1 1 1 r = t 3iˆ + 5t 2 ˆjm
- = Þ - = 3
v u f v -30 f r
r dr
v= = 10t 2iˆ + 10tjˆ ms –1
v v dt
Also, m = Þ -2 =
u u r
r dv
On solving we get f = + 20 cm a= = 20tiˆ + 10 ˆjms -2
dt
and v = 60 cm.
For reflection At t = 1s
n r 10
1 1 1 2
+ = = Þ
1
+
1
=
2 rt =1 = iˆ + 5 ˆjm ;
v u f R 10 -30 R 3
r
Þ R = 30 cm vt =1 = 10iˆ + 10 ˆj ms –1
The image formed by convex side is faint erect and r
pt =1 = iˆ + ˆj kgms –1
virtual.
By lens maker formula r
at =1 = 20iˆ + 10 ˆj ms -2
1 æ nl öæ 1 1 ö
= ç - 1÷ç - ÷ iˆ ˆj kˆ
f è ns ø è R1 R2 ø r r r
L = r ´ p = 10 5 0 = kˆ é10 - 5 ù = - 5 kˆ kgms -1
3 ê ú ë3 û 3
1 æ nl öæ 1 ö
\ = ç - 1÷ç ÷ \ n = 2.5 1 1 0
20 è 1 øè 30 ø l
r r
q2 F = ma = (2iˆ + ˆj ) N
10. (b, d) We know that, dimensionally Î= ,
l2 F
RT iˆ ˆj kˆ
k BT = = PV = F ´ l
NA r r r 10 é10 ù -20 ˆ
t = r ´F = 5 0 = kˆ ê - 10 ú = k N–m
3 ë3 û 3
1/2
ek BT é q2 F ´l ù 2 1 0
Now = ê 2 ´ –3 2 ú =l
nq 2 êë l F l q úû 13. (a, c, d)
1/2 n1 sin qi = n2 sin q f [Q 1 and 2 interfaces are parallel]
q2 é l2 F ´ q2 ù
Also = ê ú =l l depends on the refractive index of transparent slab
Î n1/3 k BT ëê q 2 l -1 ´ F ´ l ûú but not on n 2 . In fact qf depends on n2.
11. (a, d) The flux passing through the triangular wire if i current 14. (9) Here P µ T 4 or P = P0T 4
flows through the inifinitely long conducting wire
0.1 P
m0 i \ log 2 P = log 2 P0 + log 2 T 4 \ log 2 = 4log 2 T
= ò 2px
´ 2pdx P0
0
JEE Advanced 2016 Solved Paper 2016-19

P CHEMISTRY
At T = 487°C = 760 K, log 2 = 4 log 2 760 = 1 … (1)
P0 19. (d) Radial probability function curve for 1s is (D).
Here P is 4pr 2R2.
At T = 2767°C = 3040K,
r
loge = 4log 2 3040 = 4 log 2 (760 ´ 4)
r0
P
= 4 éëlog 2 760 + log 2 22 ùû
= 4log 2 760 + 8 = 1 + 8 = 9
20. (c) From 1st law of thermodynamics
15. (9) Maximum kinetic energy of b-particle qsys = DU – w = 0 – [–Pext.DV]
= [mass of 12 12 = 3.0 atm × (2.0 L – 1.0 L) = 3.0 L-atm
5 B – mass of 6 C ] × 931.5 – 4.041
= [12.014 – 12] × 931.5 – 4.041] = 9MeV (q rev )surr qsys
\ DSsurr = =-
hc 1.237 ´ 10-6 T T
16. (6) E= = eV = 12.75eV
l 970 ´ 10 -10 3.0 ´ 101.3J
=- = –1.013 J/K
\ The energy of electron after absorbing this photon 300 K
= –13.6 + 12.75 = – 0.85eV 21. (b) Atomic radii increases on moving down a group.
This corresponds to n = 4 However due to poor shielding effect of d orbit, atomic
radius of Ga is smaller than Al (anomaly). Thus the
n(n - 1) 4(4 - 1) correct order is Ga < Al < In < Tl
Number of spectral line = = =6
2 2 22. (b) Compound/Ion Magnetic nature of compound
1. [Ni(CO)4] Diamagnetic
2r12 (s - r1 ) g 2. [NiCl4]2– Paramagnetic
VP 9h1 r 2 (s - r1 ) h2 3. [Co(NH3)4Cl2]Cl Diamagnetic
17. (3) = 2 = 12 ´ 4. Na3[CoF6] Paramagnetic
VQ 2 r2 (s - r 2 ) g r2 (s - r 2 ) h1
5. Na2O2 Diamagnetic
9h2 6. CsO2 Paramagnetic
So total number of paramagnetic compounds is 3.
12 [8 - 0.8] 2 23. (a)
= ´ =3
(0.5) [8 - 1.6] 3
2
CH3 CH3
| |
Polymerisation
CH 2 = C - CH = CH 2 ¾¾¾¾¾¾® -( CH 2 - C = CH - CH 2 -)
1mH 3W Isoprene Natural rubber

H2(excess)
2mH 4W catalyst

18. (8) CH3 CH3


12W |
Copolymerisation
|
CH 2 = CH 2 + CH 2 = CH ¾¾¾¾¾¾¾
® -( CH 2 - CH - CH2- CH 2-)
Ethylene Propylene Completely hydrogenated
Naturalrubber

24. (b, c, d)
5V (A) High activation energy usually implies a slow reaction.
(B) Rate constant of a reaction increases with increase in
temperature due to increase in number of collisions
5
At t = 0 Imin = whose energy exceeds the activation energy.
12
(C) k < P ´ Z ´ e, Ea RT
5 5 10 (D) So, pre-exponential factor (A) = P × Z and it is
At t = a Imax = R = 3 / 2 = 3 independent of activation energy or energy of
eq
molecules.
é 1 1 1 1 8ù 25. (b, d) When N/P ratio is less than one, then proton changes
ê R = 3 + 4 + 12 = 12 ú into neutron.
ë eq û
Position emission : 11 H ¾¾ ® 01 n + +01b
I max 10 12
\ I = ´ =8 k-electron capture : 11 H + 0
® 01 n + X-rays
-1 e ¾¾
min 3 5
EBD_7809
2016-20 JEE Advanced 2016 Solved Paper
26. (a, c, d) 30. (b)
Structure of borax
O O
HO O
– NH2 NH – C – CH3 NH – C – CH3
sp 3 B
OH Ac2O KBrO 3/HBr
O O
B Pyridine
B O B O sp 2
– Acetanilide
HO HO Br
sp 2 3 sp (Major)
+ –
Correct formula of borax is Na2[B4O5(OH)4].8H2O NH2 N NCl Br
(A) Borax has tetranuclear. [B4O5(OH)4]2– unit H 3O+ NaNO2/HCl Cu
(B) Only two 'B' atom lie in same plane 273 – 278 K HBr
(diazotisation)
(C) two Boron are sp2 and two are sp3 hybridised.
(D) one terminal hydroxide per boran atom. Br Br Br
27. (b, c) Compound Number of lone pairs
on central atom 31. (b, c)
BrF5 ® 1
OH OH OH
ClF3 ® 2 CHO
(i) O2 CHCl3/NaOH
XeF4 ® 2 +
(ii) H 3O+
SF4 ® 1
P Q
28. (a or a, c) (Major) CHO
R (Minor)

OH O – CH2Ph
S2- SO 42- CHO CHO
NaOH
Cu 2+ CuS(ppt) CuSO4 (Soluble) PhCH 2Br

Ba 2+ BaS(Soluble) BaSO 4 (ppt) S

Pb(OAc) 2 PbS(ppt) PbSO 4 (ppt) Q is steam volatile not R.


Na 2 [Fe(CN)5 NO] Na 4 [Fe(CN)5(NOS)] ____ Q and R show positive test with 1% aqueous FeCl3
Colour (not a ppt) solution.
Q, R, S give yellow precipitate with 2, 4-dinitrophenyl
29. (a, b, c) hydrazine.
Aldehydes and a-Hydroxyketones show positive 32. (9) 1 mole solution has 0.1 mole solute and 0.9 mole solvent.
Tollen's test. Let M1 = Molar mass solute
M2 = Molar mass solvent
H O
Tollen's 0.1
H ¾¾¾¾® Silver mirror ¯ Molality, m = ´ 1000 ....... (1)
H
Reagent 0.9M 2
H 0.1
Molarity, M = 0.1M + 0.9M ´ 2 ´ 1000 ....... (2)
CHO 1 2
Tollen's
¾¾¾¾® Silver mirror ¯ Q m= M
Reagent
0.1´ 1000 200
Þ =
OH 0.9M 2 0.1M1 + 0.9M 2
Ph Tollen's
Ph ¾¾¾¾® Silver mirror ¯ M1
Reagent
Þ =9
O M2
O 33. (4) Diffusion coefficient µ lµ
Ph
+ Ph T
Since l µ and m µ T
O P
Diketone
JEE Advanced 2016 Solved Paper 2016-21

MATHEMATICS
T T
\ Diffusion coefficient µ 37. (c) x2 – 2x sec q + 1 = 0 Þ x = sec q ± tan q
P
and x2 + 2x tan q – 1 = 0 Þ x = –tan q ± sec q
T T p p
Di 1 1 Q - <q< -
Thus = P = = 6 12
Df 4T 4T (4 ´ 2) 2 4
p p
2P Þ sec > sec q > sec
6 12
Df 4
or = p tan p
Di 1 and - tan < tan q < -
6 12
34. (6) 8MnO -4 + 3S2O 32 - + H 2 O ¾¾
® p p
also tan < - tan q < tan
12 6
8MnO 2 + 6SO24 - + 2OH -
a1,b1 are roots of x2 – 2x secq + 1 = 0
\ 8 moles of MnO–4 form 6 moles of SO42– and a1> b1
35. (5) [CoL2Cl2]– (L = H2NCH2CH2O–) \ a1 = secq – tanq and b1 = secq + tanq
L is unsymmetrical didentate ligand. a2, b2 are roots of x2 + 2x tan q – 1 = 0 and a2 > b2
So the complete is equivalent to [M(AB)2a2] Possible \ a2 = -tanq + secq, b2 = – tanq – secq
G..I. are \ a1 + b2 = secq – tanq – tanq – secq = – 2tanq
a a 38. (a) Either one boy will be selected or no boy will be
A B A A selected. Also out of four members one captain is to
M M be selected.
B A B B \ Required number of ways = (4C1 × 6C3+ 6C4) × 4C1
a a = (80 + 15) × 4 = 380
39. (c) 3 sec x + cosec x + 2 (tan x – cot x) = 0
a a B
A a B a A a 3 1
Þ sin x + cos x = cos2x – sin2 x
M M M 2 2
B A A B B a
æ pö
B A A Þ cos çè x - ÷ø = cos 2x
3
3 4 5
CH2CH2CH3 CH2CH2CH3 p
Þ x- = 2np ± 2x
Br2(1.0 mole)
3
2
36. (5) H Br H Br
300°C 2np p p
1 Þx= + or x = -2np -
CH
3 CH2 – Br 3 9 3
1
(1.0 mole) (Chiral) p p
(enantiomerically pure) For x Î S, n = 0 Þ x = ,-
9 3
CH2CH2CH3 CH2CH3 CH2CH3 7p
3 3 n=1Þx=
2 H Br Br H 9
+ Br Br + +
H Br H Br -5p
n = –1 Þ x =
CH3 CH3 CH3 9
(Achiral) (Chiral) (Chiral)
p p 7 p 5p
\ Sum of all values of x = - + - =0
5
9 3 9 9
CH3 CH3 CH2 – Br
4 4 20 80 7
H Br Br H H H 40. (c) P(T1) = , P(T2) = , P(D) =
100 100 100
+ H H + H H + H H
H Br H Br H Br æ Dö æ Dö
CH3 CH3 CH3 Let P ç ÷ = x, then P ç ÷ = 10x
è T2 ø è T1 ø
(Achiral) (Chiral) (Chiral)
EBD_7809
2016-22 JEE Advanced 2016 Solved Paper
length of perpendicular from P (3, 0, 0) to plane x – y = 0
æ Dö æDö
Also P(D) = P(T1) P ç ÷ + P(T2 ) P ç ÷ 3-0 3
è T1 ø è T2 ø is = =
2 2
7 20 80
Þ = ´ 10x + ´x x y -3 z x y-3 z
100 100 100 Eqn of RS : = = or = = =l
3 -3 3 1 -1 2
7 1 2 2
Þ = x or x =
280 40 If ON is perpendicular to RS, then N (l, -l + 3, 2l)
æ Dö æ Dö Q ON ^ RS Þ 1 ´ l - 1(-l + 3) + 2 ´ 2l = 0
P ç ÷ = 10 and P ç ÷ = 1
è T1 ø 40 è T2 ø 40 1 æ1 5 ö
Þl = Þ N ç , ,1÷
æ D ö 39 2 è2 2 ø
æ Dö 30
Þ Pç ÷ = and P ç T ÷ = 40
è T1 ø 40 è 2ø 1 25 15
\ ON = + +1 =
4 4 2
æ Dö
P ç ÷ P(T2 )
æT ö è T2 ø f (x)
Pç 2 ÷ = 43. (a) f ¢(x) = 2 -
è Dø æ Dö æ Dö x
P ç ÷ P ( T1 ) + P ç ÷ P(T2 )
è T1 ø è T2 ø 1
Þ f ¢(x) + f (x) = 2
x
80 39 If = elog x = x
´
100 40 156 26
= = = \ f(x).x = ò 2x dx = x2 + C
20 30 80 39 186 31
´ + ´
100 40 100 40
C
or f(x) = x + , C ¹ 0 as f(x) ¹ 1
78 26 x
Also =
93 31
æ 1ö
1 (a) lim f ¢ ç ÷ = lim (1 - Cx 2 ) = 1
2 x ®0 + è x ø x ® 0+
41. (c) Let f(x) = 4ax +
x
For x > 0, fmin = 1 æ 1ö æ1 ö 2
(b) lim xf ç ÷ = lim+ x ç + Cx ÷ lim+ 1 + Cx = 1
x®0 + è x ø x ®0 è x ø x ®0
f ¢(x) = 8ax – 1 = 0 Þ x = 1
x2 2a1/3 (c) lim x 2 f ¢ x
x ® 0+
2
æ 1 ö
fmin = 1 Þ 4a ç + 2a1/3 = 1
1/3 ÷ æ Cö 2
è 2a ø = lim x 2 ç1 - 2 ÷ = lim+ x - C = - C
x ®0 + è x ø x ® 0
1
1 (d) for C ¹ 0, f(x) is unbounded as 0 < x < 2
Þ 3a 3 = 1 or a =
27 C C C C
42. (b, c, d) Þ < <¥Þ <x+ <¥
The coordinates of vertices of pyramid OPQRS will be 2 x 2 x
æ3 3 ö P.Q Q
O(0, 0, 0), P (3, 0, 0), Q (3, 3, 0), R (0, 3, 0), S ç , , 3÷ 44. (b,c) PQ = kI Þ = I Þ P -1 =
è2 2 ø k k
dr's of OQ = 1, 1, 0 Also |P| = 12a + 20
dr's of OS = 1, 1, 2 Comparing the third elements of 2nd low on both sides,
\ acute angle between OQ and OS we get

-1 æ 2 ö æ 1 ö p æ 3a + 4 ö 1 - k
= cos ç = cos -1 ç ¹ -ç ÷= ´
è 2 ´ 6 ÷ø è 3 ÷ø 3 è 12a + 20 ø k 8
Þ 24a + 32 = 12a + 20 Þ a = –1
x y z \ |P| = 8
Eqn of plane OQS =
1 1 0
=0 Also PQ = kI Þ |P| |Q| = k3
1 1 2
k2 k2
Þ 8´ = k3 Þ k = 4 Þ |Q| = =8
Þ 2x – 2y = 0 or x – y = 0 2 2
JEE Advanced 2016 Solved Paper 2016-23

(b) 4a – k + 8 = 4 × (–1) – 4 + 8 = 0 1
(c) Now det (P adj Q) = |P| adj Q| \ Solution is : u × y = ò y ´ y2 dy = log y + C
= |P| |Q|2 = 8 × 82 = 29
(d) |Q adj P| = |Q| |P|2 = 29 -y
Þ = log y + C
s-x s- y s -z s-x +s- y+s-z s x+2
45. (a, c, d) = = = =
4 3 2 4+3+ 2 9 As it passes through (1, 3) Þ C = –1 – log 3
5s 6s 7s -y
\x= ,y= ,z= \ = log y – 1 – log 3
9 9 9 x+2
y y
D2 8p Þ log = 1- ...(1)
Area of incircle = pr2 = p 2
= 3 x+2
s 3
Intersection of (1) and y = x + 2
s(s - x)(s - y)(s - z) 8 y
Þ 2
= log = 0 Þ y= 3 Þ x = 1
s 3 3
\ (1, 3) is the only intersection point.
4 ´ 3 ´ 2s3 8 Intersection of (1) and y = (x + 2)2
Þ = or s = 9
9 ´ 9 ´ 9s 3
(x + 2)2 (x + 2)2
\ x = 5, y = 6, z = 7 log = 1 – (x + 2) or log + (x + 2) = 1
3 3
(a) Area of Dxyz = s(s - x)(s - y)(s - z)
(x + 2)2 4
9´ 4 ´ 3´ 2 = 6 6 Q > > 1, "x > 0
3 3
xyz \ LHS > 2, " x > 0 Þ no solution.
(b) Radius of circumcircle, R =
4D 47. (b, c) f(x) = x3 + 3x + 2 Þ f ¢(x) = 3x2 + 3
Also f(0) = 2, f (1) = 6, f(2) = 16, f(3) = 38, f(6) = 236
5´ 6´ 7 35 6 4 And g(f(x)) = x Þ g(2) = 0, g(6) = 1, g(16) = 2, g (38) = 2,
= = =
4´ 6 6 24 35 g (236) = 6
(a) g (f(x)) = x Þ g¢ (f(x)). f ¢(x) = 1
x y z x y z r For g¢(2), f(x) = 2 Þ x = 0
(c) r = 4 R sin sin sin Þ sin sin sin =
2 2 2 2 2 2 4R \ Putting x = 0, we get g¢(f (0)) f¢(0) = 1

2 2 ´ 24 1
= Þ g¢(2) =
3 ´ 4 ´ 35 6 3
(b) h (g(g(x))) = x Þ h¢(g(g(x))). g¢(g(x)). g¢(x) = 1
x+y
2 z For h¢(1), we need g (g(x)) = 1
(d) sin = cos2
2 2 Þ g (x) = 6 Þ x = 236
\ Putting x = 236, we get
s(s - z) 9 ´ 2 3
= = = 1
xy 5´ 6 5 h¢ [g(g(236))] =
g¢ (g(236)).g¢ (236)
dy
46. (a, d) [(x + 2)2 + y (x + 2)] = y2 1
dx Þ h¢ (g(6)) = g¢(6) g¢(236)
2 dx
Þ y - (x + 2)y = (x + 2) 2 1
dy
Þ h¢ (1) = = f ¢(1) f ¢(6)
g¢(f (1)) g¢(f (6))
1 dx 1 1
Þ - = = 6 × 111 = 666
(x + 2) 2
dy (x + 2)y y2 (c) h [g(g(x))] = x
For h (0), g (g(x)) 0 Þ g(x) = 2 Þ x = 16
-1 1 dx du \ Putting x = 16, we get
Let =uÞ =
x+2 2
(x + 2) dy dy h(g(g(16))) = 16
\ Eqn becomes Þ h(0) = 16
(d) h[g(g(x))] = x
du 1 1 For h(g(3)), we need g(x) = 3 Þ x = 38
+ u=
dy y y2 \ Putting x = 38, we get
If = elog = y = y h [g(g(38))] = 38 Þ h (g(3)) = 38
EBD_7809
2016-24 JEE Advanced 2016 Solved Paper
48. (a, b, c) \ Intersection point E of normal and y = tanq
C1 : x2 + y2 = 3 ..(i)
parabola : x2 = 2y ...(ii) tan q 1 - tan 2 q / 2
= x tan q Þ x =
Intersection point of (i) and (ii) in first quadrant 2 2
y2 + 2y – 3 = 0 Þ y = 1 (Q y ¹ -3) 1 - y2
\ Locus of E : x = or y2 = 1 – 2x
\x= 2 2
P( 2,1) æ1 1 ö æ 1 -1 ö
It is satisfied by the points çè 3 , ÷ø and çè 3 , ÷ø
3 3
Equation of tangent to circle C1 at P is 2x + y - 3 = 0
Let centre of circle C2 be (0, k); r = 2 3 x x2 1 + x3
k -3 50. (2) 2x 4x 2 1 + 8x3 = 10
\ = 2 3 Þ K = 9 or –3
3 3x 9x 2 1 + 27x 3
\ Q2 (0, 9), Q3 (0, –3)
(a) Q2 Q3 = 12
1 1 1 1 1 1
(b) R2R3 = length of transverse common tangent 3 6
Þ x 2 4 1 + x 2 4 8 = 10
= (Q2 Q3 ) 2 - (r1 + r2 ) 2 = (12)2 - (4 3)2 = 4 6 3 9 1 3 9 27
1 Operating C2 – C1, C3 – C1 for both the determinants,
(c) Area (DOR2R3) = × R2R3 × length of ^ from origin we get
2
to tangent 1 0 0 1 0 0
1 3
Þ x 2 2 -1 + x 2 2 6 = 10 6
= ´4 6´ 3 = 6 2
2 3 6 -2 3 6 24
1 Þ x3 (–4 + 6) + x6 (48 – 36) = 10
(d) Area (DPQ2Q3) = × Q2Q3 ^ distance of P from
2 Þ 2x3 + 12x6 = 10
1 Þ 6x6 + x3 – 5 = 0
y–axis = ´ 12 ´ 2 = 6 2
2 1
49. (a, c) Circle : x2 + y2 = 1 æ 5ö 3
Þ (6x3 – 5) (x3 + 1) = 0 Þ x = çè ÷ø , – 1
Equation of tangent at P(cos q, sin q) 6
x cos q + y sin q = 1 ...(1) 51. (5) (1 + x)2 + (1 + x)3 + .... + (1 + x)49 + (1 + mx)50
Equation of normal at P
y = x tan q ...(2) é (1 + x)48 - 1ù
Equation of tangent at S is x = 1 = (1 + x)2 ê (1 + x) - 1 ú + (1 + mx)50
ëê ûú
æ 1 - cos q ö æ tan q ö
\ Q ç 1, ÷ø = Q çè 1, 2 ÷ø 1é
è sin q = (1 + x)50 - (1 + x)2 ù + (1 + mx)50
xë û
2
Coeff. of x in the above expansion
Y
= Coeff. of x3 in (1 + x)50 + Coeff. of x2 in (1 + mx)50
(cos q, sin q) Þ 50C3 + 50C2 m2
P
\ (3n + 1) 51C3 = 50C3 + 50C2 m2
Q (1, tan q/2)
50 50
E C3 C2
Þ (3n + 1) = + m2
51 51
X C3 C3
R O S (1, 0)
16 1 2 m2 - 1
Þ 3n + 1 = + m Þn=
17 17 51
Least positive integer m for which n is an integer is
m = 16 and then n = 5
x t2
\ Equation of line through Q and parallel to RS is 52. (1) Let f(x) = ò0 dt - 2x + 1
1+ t4
tan q
y= x2
2 Þ f ¢(x) = - 2 < 0 " x Î [0,1]
1+ x4
JEE Advanced 2016 Solved Paper 2016-25

\ f is decreasing on [0, 1] For above to be possible, we should have


Also f(0) = 1 1
2
a – 1 = 0 and b =
1 t 3!
and f(1) = ò0 dt - 1
1 + t4 1
Þ a = 1 and b =
6
t2 1
For 0 £ t £ 1 Þ 0 £ 4
< æ 1ö
1+ t 2 \ 6 (a + b) = 6 ç1 + ÷ = 7
è 6ø
1 t2 1
\ ò0 4
dt < -1 + i 3
1+ t 2 54. (1) z= Þ z3 = 1 and 1 + z + z2 = 0
2
Þ f(1) < 0
\ f(x) crosses x-axis exactly once in [0, 1] é (- z) r z 2s ù é( - z)r z 2s ù
\ f(x) = 0 has exactly one root in [0, 1] P2 = ê úê ú
êë z 2s z r úû êë z 2s z r úû
x 2 sin bx
( )
53. (7) lim =1
x ® 0 ax - sin x é z 2r + z 4s z 2s (- z)5 + z r ù
ê ú

Þ lim
x 3b
=1
2s
(
r
êë z (-z) + z
r
) 4s
z +z 2r ú
úû
x ® 0 ax - sin x
For P2 = – I we should have
x 3b z2r + z4s = 1 and z2s ((–z)r + zr) = 0
Þ lim =1 Þ z2r + zs + 1 = 0
x ®0 æ x 3 x5 x 7 ö
ax - ç x - + - + ....¥÷ Þ r is odd and s = r but not a multiple of 3.
è 3! 5! 7! ø Which is possible when s = r = 1
\ only one pair is there.
x 3b
Þ lim =1
x ®0 x 3 x5
(a - 1)x + - + ....¥
3! 5!

Paper - 2
PHYSICS A 1
1. (c) Binding energy of nitrogen atom 2. (c) =
A 0 2n
= [8 × 1.008665 + 7 × 1.007825 – 15.000109 × 931]
n A 0 64
Binding energy of oxygen atom \2 = = = 26 Þ n = 6
= [7 × 1.008665 + 8 × 1.007825 – 15.003065] × 931 A 1
\ Difference = 0.0037960 × 931 MeV ...(I) \ time = 6 ´ t 1 = 6 ´ 18 = 108
2
3 8´ 7 e2 3 56
E
Also O = ´ ´ = ´ ´1.44MeV 5
2 R 4pÎ0 5 R 3. (c) P3V5 = constant Þ PV5/3 = constant Þ g =
3
Þ monoatomic gas
3 7´6 e2 3 42 For adiabatic process
EN = ´ ´ = ´ ´1.44MeV
5 R 4pÎ0 5 R
P PiVi 1
3 14
\ E O - E N = ´ ´1.44MeV ...(II)
5 R
2
From (i) & (ii) adiabatic
3 14
´ ´ 1.44 = 0.0037960 ´ 931 PfVf
5 R
\ R = 3.42 fm
V
EBD_7809
2016-26 JEE Advanced 2016 Solved Paper
Therefore object distance for mirror is 25 cm and object
1
´105 ´ 8 ´10-3 - 105 ´ 10-3 is virtual
Pf Vf - Pi Vi 32
W= = 1 1 1 1 1 1
1- g 5 + = \ + =
1- For minor \ v = – 50 cm
3 v u f v 25 50
The image I would have formed as shown had the mirror
25 - 100 75 ´ 3 have been straight. But here the mirror is tilted by 30°.
\ W = ( 3 - 5 ) / 3 = 2 = 112.5J Therefore the image will be tilted by 60° and will be
From first law of thermodynamics q = Du + w formed at A.
\ Du = – w Here MA = 50 cos 60° = 25 cm
\ Du = – 112.5 J and I¢A = 50 sin 60° = 25 3 cm
Now applying first law of thermodynamics for process 6. (b) For C1
1 & 2 and adding q1 + q2 = DU + Pi(Vf – Vi) L.C. = 1MSD – 1VSD
= – 112.5 + 105 (8 – 1) × 10– 3 = 587.55 = 1mm – 0.9 mm = 0.1 mm = 0.01 cm [10 VS D = 9 mm]
4. (a) The heat flow rate is same Reading = MSR + L.C × Verni scale division coinciding
T Main scale division = 2.8 + (0.01) × 7 = 2.87 cm
M 2K
P Q K For C2
10°C 400°C L.C = 1 mm – 1.1 mm [10 VSD = 11 mm]
R H S L.C = – 0.1 mm = – 0.01 cm
x H
dx Reading = 2.8 + (10 – 7) × 0.01 = 2.83 cm

KA ( 400 - T ) 2KA ( T - 10 ) 7. (a, c) In DOAM, OM = l2 + a 2 = 2ha 2 + a 2 = 5a


\ = The circumference of a circle of radius OM will be 2p(5a)
l l
\ T = 140°C = 10pa.
The temperature gradient access Pd is z
Sm Vc r^
dT 140 - 10
= \ dt = 130 dx w
dx 1
Therefore change temperature at a cross-section M A l
q c
distant ‘x’ from P is l a
DT = 130 x q
Extension in a small elemental length ‘dx’ is O M
dl = dx µ DT = dx µ (130x ) For completing this circle once, the smaller disc will

1 10pa
have to take = 5 rounds.
2pa
\ ò dl = 130 µ ò xdx
0 Therefore the C.M. of the assembly rotates about z-
axis with an angular speed of w/5.
1 The angular momentum about the C.M. of the system
\ Dl = 130 × 1.2 × 10–5 × = 78 × 10 – 5 m
2
é1 2ù
1 1 1 Lc = Icw = ê ma ú w
5. (c) For lens - = ë2 û
v u f
é1 2ù 17ma 2w
1
-
1
=
1 +ê ´ 4m ´ ( 2a ) úû ´ w =
ë2 2
v -50 30
\ v = 75 cm m ´ wa + 4m ´ 2wa 9wa
Now vc = =
5m 5
Principal axis of mirror
I¢ ml + 4m ´ 2l 9l
and r1 = =
5m 5

50 cm 5m ´ 9wa 9l 24
30° L of C.M = ´ = 81mwa 2 ´
5 5 5
30°
I A M 81mwa 2 24
LZ = cos q - Icwsin q
Principal 5
axis of
lens 24 24 17ma 2 w 1134
MIRROR = 81mwa
2
´ - = mwa 2
5 5 10 50
JEE Advanced 2016 Solved Paper 2016-27

8. (c) The wavelength of emitted photoelectron as per de 11. (a, b, c, d)


Broglie is At t=0, Capacitors act as short circuit and voltmeter
display –5v
h h
le = = At t = a , Capacitor acts as open circuit and no current
p 2m ( K.E ) flows through voltmeter (Q very high resistance of
When f increases, K.E. decreases and therefore le voltmeter)
increases so it display +5v. (A) is the correct option

( ) , i = VR e
t
When lph increases, N ph decreases , K.E decreases and -t -
q1 = 2Cv 1 - e 2CR
1
2CR

therefore le increases.

),
le is independent of the distance d.
(
t
-t V - t 2CR
q2 = Cv 1 - e 2CR
i2 = e
hc h2 é h ù 2R
Also l + eV - f = l =
2 ê e ú
ph 2mle ë 2 mk. E û
2C R
hc f h2 i1
\ +V- = ...(1)
el ph e 2mele2
V
i2 2R
f
For V >> , f << eV C
e
hc
Also << V . Then from eq (1).
el Ph 5V
1 21 é -t
2CR ù
le µ \ D V = – i2 × 2R + 2 = V êë1 - 2e =0
V C ûú
Therefore if V is made from times, le is approximately (b) is the correct option
half.
io éQ i = io e - t t ù
9. (a, b, d) At t = 1sec, i =
e ëê ûú
1 \ (c) is the correct option
% error in measurement of ‘r’ = ´ 100 = 10%
10 After a long time no current flows since both capcitor
0.52 + 0.56 + 0.57 + 0.54 + 0.59 and voltmeter does not allow current to flow.
Tmean = = 0.556 » 0.56 S 12. (a, b, d)
6
Case (i) : Applying conservation of linear momentum.
0.04 + 0 + 0.01 + 0.02 + 0.03
DT = = 0.016 » 0.02S
6 MV1 = (M + m) V2 – (1) \ V2 = æç M ö÷ V1
\ % error in the measurement of ‘T’ èM+mø

0.02
= ´ 100 = 3.57% v1 v2
0.56
% error in the value of g
From (1)
DT æ DR + Dr ö M (A1 × w1) = (M+m) (A2×w2)
=2 ´ 100 + ç ÷ ´ 100
T è R -r ø K K
\ MA1 × = (M+m) A2 ×
M M+m
= 2 ( 3.57 ) + æç 1 + 1 ö÷ ´100 » 11%
è 60 - 10 ø M
10. (a, c) The range of voltmeter ‘V’ is given by the expression \ A2 = = A1
M+m
V = Ig [Rc + (Rc + R + R)]
V is max in this case as RHS is maximum. Thus (A) is 1
correct. Also E1 = MV12
2
The range of ammeter I is given by the expression
1 1
Ig R c and E2= (M+m)V22 = (M+m)
∗ Ig 1 1 1 1 2 2
I= Where < ∗ ∗
R eq R eq R c R R
M 2 V12 2
1æ M ö 2
Here Req is minimum and therefore I is maximum. Thus ×
(M + m) 2
= ç ÷ V1
2èM+mø
(c) is the correct option.
EBD_7809
2016-28 JEE Advanced 2016 Solved Paper
Clearly E2 < E1
dv
15. (a) Force on the block along flot = m rw2 = m v
m+M dr
The new time Period T2 = 2
K
v r
R 2 dr
ò vdv = ò w2 rdr Þ v = w r - =
2
m+M \
Case (ii) : The new time Period T2 = 2 o R 4 dt
K 2

Also A2 = A1 r t
dr
Here E2 = E1 \ ò R2
= ò w dt
The instantaneous value of speed at X o of the R
4 r2 -
o

combined masses decreases in both the cases. 4


13. (b, d) Path difference at O = d = 0.6003mm
On solving we get
l 600 ´10 -6
Now, = mm = 300×10–6 mm
2 2 R2 R wt
r + r2 - = e
4 2
l 0.6003
For n = d we get n= =2001
2 300 ´10 -6 R 2 R 2 2wt 2 R wt
or r2 – = e + r - 2r e
As n is a whole number, the condition for minima is 4 4 I
satisfied.
Therefore ‘O’ will be dark. R wt
Also, as the screen is perpendicular to the plane
\ r=
4
( e + e-wt )
containing the slits, therefore fringes obtained will be
semi-circular (Top half of the screen is available) ® ®
æ Ù
ö Ù
æ Ù Ùö Ù
16. (b) Frot = Fin + 2m ç Vrot i ÷ ´ w k + m ç w k´ r i ÷ ´ w k
BLV è ø è ø
14. (a, b) i = ...(i)
R Ù ®
æ Ùö Ù
\ m rw2 i = Fin + 2m Vrot wç - j ÷ + mw2 r i
[Counter clockwise direction while entering, Zero when
è ø
completely inside and clockwise while exiting]
B 2 L2 V ®
Ù
F = iLB = ....(ii) Fin = mrV ...(i)
R rot w j
[Toward left while entering and exiting and R wt
zero when completely inside] But r = ée + e - wt ùû

dv B 2 L2 V
\ - mV = dr R
dx R \ dt = Vr = éëwe - we ùû
wt - wt
...(ii)
4
v 2 2 x
BL
\ ò dV = - mR ò dx
v0 0
From (i) and (ii)
® Rw wt - wt Ù
2 2
Fin = 2m
4
(e - e ) w j
BL
V –V0 = – x
mR
®
mRw2 wt Ù

BLx2 2 \ Fin =
2
( e - e - wt ) j
\ V= V0 – ...(iii)
mR
® mR w2 wt Ù Ù
[V decreases from x=0 to x=L, remains constant \ Freaction =
2
( e - e - wt ) j+ mg K
for x = L to x= 3L again decreases from x=3L to x=4L]
From (i) and (iii) 17. (c) After colliding the top plate, the ball will gain negative
charge and get repelled by the top plate and bounce
BL é B2 L2 x ù
i = R ê V0 - mR ú back to the bottom plate.
ë û
Q
[i decreases from x=0 to x=Li becomes zero from x = L to x = 3L 18. (d) Iav a ...(i)
i changes direction and decreases from x = 3L to x = 4L] t
These characteristics are shown in graph (a) and (b) Here Q µ V0 ...(ii)
only.
JEE Advanced 2016 Solved Paper 2016-29

1 2 So, X, Y and Z are [Ag (S2O3)2]3–, Ag2S2O3 and Ag2S


Also S = ut + at respectively.
2
22. (a) Ni2+ with NH3 shows CN = 6 forming [Ni(NH3)6]2+
1 QE 2 1 æ Q ´ 2V0 ö 2 (Octahedral)
h= t = ç ÷´ t
2 m 2 è mh ø Pt2+ with NH3 shows CN = 4 forming [Pt(NH3)4]2+
(5d series CMA, square planner)
1
\ ta – (iii) [Q Q a V0] Zn2+ with NH3 shows CN = 4 forming [Zn(NH3)4]2+
V0 (3d10 configuration, tetrahedral)
From (i), (ii) and (iii) 23. (a)
V0 –
O O
Iav µ 1 = Iav µ V02 H H
V0 COOH C
HO OH O O
CHEMISTRY
ˆˆ† 2H+ (aq) + 2e–
19. (d) At anode : H2(g) ‡ˆˆ
(I) (stabilizes by more H-bonding)
ˆˆ† M2+ (aq)
At cathode : M4+ (aq) + 2e– ‡ˆˆ

O O
COOH H
ˆˆ† 2H+ (aq) +
Net cell reaction : H2(g) + M4+ (aq) ‡ˆˆ
OH O
M2+ (aq)

Now, Ecell = E (
°
M
4+
/M 2 +
- E° +
H /H 2 ) (II) (stabilizes by H-bonding)

2
é H + ù é M 2+ ù –
0.059 log ë û ë û COOH COO
– . 4+ ù
n é
PH2 . M
ë û

2+
0.059 12 ´ éë M ùû OH OH
. log (III) (stabilizes by –I effect)
or, 0.092 = (0.151 – 0) –
2 1´ é M 4+ ù
ë û

COOH COO
é M 2+ ù
ë û
\ +
= 102 Þ x = 2
éM ù4
ë û
20. (d)
• A solution of CH3OH and water shows positive OH OH
(IV) (destabilizes by +M effect)
deviation from Raoult's law, it means by adding CH3OH
intermolecular force of attraction decreases and hence
surface tension decreases. \ acidity order is I > II > III > IV
• By adding KCl in water, intermolecular force of 24. (a)
attraction bit increases, so surface tension increases OH
by small value. O O
O CH2
• By adding surfactant like CH3(CH2)11OSO3– Na +,
surface tension decreases rapidly and after forming H–C–H / NaOH
H
micelle it slightly increases. [Cross aldol reaction]

+ 3- Ag +
S2 O32 - ¾¾¾ ® éë Ag (S2 O3 )2 ùû
Ag
21. (a) ¾¾¾
® OH
OH
(X)
clear solution O CH2
H–C–H / NaOH –
+ HCOO
on standing [Cross cannizaro
Ag 2S2 O3 ¯ ¾¾¾¾¾® Ag 2S ¯ reaction]
(Y) (Z)
white precipitate black precipitate
EBD_7809
2016-30 JEE Advanced 2016 Solved Paper

OH 4
OH O O 4 ´ pr 3
CH2 O
3 = (0.74)
3
H–C–H / H
+ (B) Packing fraction = æ 4r ö
ç ÷
Acetal formation è 2ø
(C) Each FCC unit has effective no of atoms = 4
25. (a, c) Octahedral void = 4
(A) The molecular orbital energy configuration of C2–
2 is Tetrahedral void = 8
2 *2 2
s1s , s1s , s 2s , s*2 2 2 2
2s , p2p = p2p , s2p (D) 4r = a 2
x y z
In the MO of C2– 28. (a, b, c)
2 there is no unpaired electron hence it
is diamagnetic Copper pyrite [CuFeS2]
(B) Bond order of O2+2 is 3 and O2 is 2 therefore bond length Crushing into fine powder
of O2 is greater than O2+2
(C) The molecular orbital energy configuration of N+2 is
Concentrated by froth floatation process
2 *2 2
s1s , s1s , s2s , s*2 2
2s , p2p x = p22p , s12p
y z

1
Bond order of N+2 = (9 - 4) = 2.5 Roasting take place in reverberatory furnace
2
The molecular orbital energy configuration of N–2 is ì 2CuFeS2 + O2 ¾¾ D
® Cu 2 S + 2FeS + SO 2
2 *2 2 ï (air)
s1s , s1s , s2s , s*2 2 2 2 *1 *
2s , p2p = p2p , s2p , p2p = p2p y í
x y z x ï D
î FeS + O 2 ¾¾® FeO + SO 2
1
Bond order of N–2 = (10 - 5) = 2.5
2
éIron is removed as slag of FeSiO 3 ù
(D) He+2 has less energy in comparison to two isolated He Smelting
êFeO + SiO (flux) ¾¾
ë 2
D
® FeSiO 3 ( l)(slag) úû
atoms because some energy is released during the
formation of He+2 from 2 He atoms.
Copper matte (Cu2S + FeS)
26. (a, b)
(A) H-bonding of methanol breaks when CCl4 is added so
bonds become weaker, resulting positive deviation. éSelf reduction occurs in Bessemer converter ù
(B) Mixing of polar and non-polar liquids will produce a Self reduction ê2Cu S + 3O ¾¾ ® 2Cu 2 S + 2SO 2 ( ­) ú
ê 2 2 ú
solution of weaker interaction, resulting positive êëCu 2 S + 2Cu 2 O ¾¾® 6Cu + SO 2 ( ­) úû
deviation
(C) Ideal solution (Blister copper)
(D) –ve deviation because stronger H-bond is formed.
27. (b, c, d)
CCP is ABC ABC …… type packing
(A) In topmost layer, each atom is in contact with 6 atoms Refining of blister copper is done by poling followed by
in same layer and 3 atoms below this layer electrorefining but not by carbon reduction method.

dehydration of HNO
29. (b, d) P4 O10 + 4HNO3 ¾¾¾¾¾¾¾¾
3 ® 4(HPO ) +2 N O
3 2 5
(required product)

(A) P4 + 20HNO3 ¾¾
® 4 H 3 PO 4 + 20NO 2­ + 4H 2 O
(B) N2O5 is diamagnetic in nature
O O

(C) N2O5 ® N N
O O O
N2O5 contains one N–O–N bond not N–N bond.
(D) Na + N2O5 ® NaNO3 + NO2­
(Brown gas)
JEE Advanced 2016 Solved Paper 2016-31

30. (b,c) Invert sugar is an equimolar mixture of D-(+) glucose and D(–) glucose.
H+ +
C12 H 22 O11 + H 2 O ¾¾¾
® C6 H12 O6 + H 2 O ¾¾
® C 6 H12O6
+ sucrose D(+ ) -glucose D( + ) -glucose
( +52°) ( -92°)
1444444 2444444 3
Invert sugar

-92° + 52°
• Specific rotation of invert sugar = = -20°
2
• D-glucose on oxidation with Br2-water produces gluconic acid and not saccharic acid.
31. (c, d)
LiAlH4/(C2H5)2O : Reduces to esters, carboxylic acid, epoxides and aldehydes and ketones.
BH3 in T.H.F : Reduces to –COOH and aldehydes into alcohols but do not reduce to esters and epoxides.
NaBH4 in C2H5OH : Reduces only aldehydes and ketones into alcohols but not to others.
Raney Ni in T.H.F. : Do not reduce to –COOH, –COOR and epoxide but it can reduce aldehyde into alcohols.
Br

32. (b, c, d) +
NaOEt


AlCl3 + 1, 2 H +
Shift
Cl

+
H +

BF 3 1, 2 H

+ +
O–H Shift
H

X 2 (g) ˆˆ†
‡ˆˆ 2X(g)
33. (b)
Initial mole : 1 0

æ beqbm ö
moles at equilibrium : çç 1 - ÷÷ beqbm
è 2 ø
beqbm
1- beqbm
2 P
Partial pressure : ´P æ beqbm ö
æ beqbm ö ç1 + ÷
ç1 + ÷ è 2 ø
è 2 ø

2
(Px )2 beqbm P
\ Kp = =
Px 2 æ beqbm
2 ö
ç1 - ÷
ç 4 ÷
è ø
2
4beqbm P

(
\ Kp = 4 - b 2
eqbm )
Since P = 2 bar

8b2eqbm
So, K p =
( 4 - beqbm
2
)
EBD_7809
2016-32 JEE Advanced 2016 Solved Paper
34. (c) (A) Correct statement.
As on decrease in pressure reaction move indirection where no. of gaseous molecules increase.
(B) Correct statement
At the start of reaction QP < KP so dissociation of X2 take place spontaneousely.
(C) Incorrect statement as
2
8beq 8 ´ (0.7) 2
KP = = >1
4 - b2eq 4 - (0.7) 2

35. (a)
36. (b)
O

COOH C–NH2 NH2


Å NH3 /D Br2 / NaOH
KMnO /H
¾¾¾¾¾¾
4 ® ¾¾¾¾
® ¾¾¾¾¾
®

COOH C–NH2 NH2


(O) (P) (Q) (R)
O

O O
CH3 Br O

CH3 - CH -C -OC 2H 5
N–CH–C–OC2H5 ¬¾¾¾¾¾¾¾¾ NH
KOH
O
(S)
O O

Å
H 3O
CH3–CH–COOH Alanine

NH2
(T)

MATHEMATICS Þ q32 = 50 × 4 = 200

é 1 0 0ù é 0 0 0ù q31 + q32 20600


ê ú ê ú \ = = 103
37. (b) P = 4 1 0 = I + ê 4 0 0ú = I + A
ê ú q 21 200
ê ú ê ú
ë16 4 1 úû
ê ë16 4 0úû
ê 38. (b) logc b1, logc b2,...., logcb101 are in A.P.
Þ b1, b2,...., b101 are in G.P.
é 0 0 0ù é0 0 0 ù Also a1, a2,...., a101 are in A.P.
ê2 ú 3 ê ú
A = 0 0 0 and A = ê0 0 0 ú
ê ú where a1 = b1 are a51 = b51.
ê ú ê ú
ë16 0 0úû
ê ë0 0 0 úû
ê \ b2, b3,...., b50 are GM’s and a2, a3,...., a50 are AM’s
between b1 and b51.
\ An = O, n ³ 3
Q GM < AM Þ b2 < a2, b3 < a3,...., b50 < a50
Now P50 = (I + A)50 = 50C0 I50 + 50C1 I49 A + 50C2
Q b1 + b2 +....+ b51 < a1 + a2 +....+ a51
+ I48 A2 + O
Þ t<s
= I + 50A + 25 × 49 A2.
Also a1, a51, a101 are in AP
\ Q = P50 – I = 50A + 25 × 49 A2.
b1, b51, b101 are in GP
Þ q21 = 50 × 4 = 200
Q a1 = b1 and a51 = b51
Þ q31 = 50 × 16 + 25 × 49 × 16 = 20400
\ b101 > a101
JEE Advanced 2016 Solved Paper 2016-33

The graph of given region is as follows-


13 1
39. (c) å æp p ö æp kpö
k =1 sin ç + k - 1 ÷sin ç
ç (
è4
) ÷ ç +
6ø è4 6 ø
÷
÷

(1, 2)
é ìp kp æp pöïü ù (– 4, 1)
ê sin ïí + -çç + (k - 1) ÷÷ý ú P
Q
13 1 ê ï 4 6 è 4 6 ø þï ú
î
å ê
= k =1 sin p ê æp pö æp kpö ú
ú
ê sin ç
ç + k - 1 ÷ sin
( ) 6 ÷ø çè 4 6 ÷ø úú
ç + ÷ S T R
6ëê è4 û (– 3, 0)

13 é æp pö æp kpöù
å 2 êêëcot ççè 4 + (k - 1) 6 ÷÷ø - cot ççè 4 + ÷÷ú
6 ø úû
k =1 Required area = Area (trap PQRS) – Area (PST + TQR)
éì æp pöüï æp 2pö 1 é -3 1 ù
ï p æp pö = ´(1 + 2)´5 - êò-4 - x - 3 dx +ò-3 x + 3 dx ú
= 2 êícot - cot çç + ÷÷ý + cot çç + ÷÷ - cot ç
ç + ÷÷ 2 ë û
ê
ëï
î 4 è 4 6 øï
þ è4 6 ø è4 6 ø
éæ 3/2 ö-3 æ 3/2 ö1 ù
ì
ï æp 12pö æp 13pöüïù 15 êç 2 (- x - 3) ÷ ç 2 (x + 3) ÷ ú
= - êç ÷ +ç ÷ úú
+ ...... + ícot çç 4 + 6 ÷÷ - cot çç 4 + 6 ÷÷ýúú 2 êç -3 ÷ ç 3 ÷
ï è
î ø è ø þïû ê
ëè ø -4 è ø -3 úû

é p æp 13pöù é 5pù 15 é2 16 ù 15 3
= 2 êêcot - cot çç + ÷ú = 2 ê1 - cot ú
÷ = -ê + ú = - 6 = sq.units
ë 4 è 4 6 øû
ú ê
ë 12 ûú 2 ë ê3 3 ûú 2 2
42. (c) P, the image of point (3, 1, 7) in the plane x – y + z = 3 is
é ù given by
ê1 - 3 - 1ú = 2 é1 - 2 - 3 ù = 2
2
= ê
ë 3 + 1 ûú ëê úû ( ) ( 3 -1 ) x - 3 y -1 z - 7 -2 (3 - 1 + 7 - 3)
= = =
2 1 -1 1 12 + 12 + 12
p/2 x cos x
40. (a) I =ò dx ....(i) x - 3 y -1 z - 7
-p/2
1 + ex Þ = = =–4
1 -1 1
Applying òa f (x) dx
b Þ x = –1, y = 5, z = 3
\ P(–1, 5, 3)
Now equation of plane through (–1, 5, 3) and containing
b
= òa f (a + b - x ) dx, we get
x y z
the line = = is
x 2 1 2 1
p/2 e x cos x
I =ò dx ....(ii)
-p/2
1 + ex x y z
-1 5 3 = 0 Þ – x + 4y – 7z = 0
Adding (i) and (ii)
1 2 1
p/2
2 p/2 2
2I =ò-p/2 x cos x dx = 2ò0 x cos x dx or x – 4y + 7z = 0
43. (b, c)
2 p
I= é x 2
sin x + 2x cos x - 2 sin x ù2 = p - 2 é ù x /n
ëê ûú0 4 ê n æ nö æ nö ú
ê n (x + n) ççx + ÷÷... ç
çx + ÷ ÷ ú
2 n
41. (c) y³ x + 3 Þ y2 = x + 3 f(x) = lim êê è ø è ø ú
n ®¥ ê æ 2ö æ 2 öú
2 2 n n ú
( x2 + ÷ x2 + 2 ÷
2 ïì- (x + 3) if x < -3
ên! x + n ç
ê
ë
ç
è
÷)... ç
4 ø è ç ÷
n ø úû
ú
Þ y =í ....(i)
ïî (x + 3) if x ³ -3
é ù x/n
ê 2n æ x öæ x 1 ö æ x 1 ö ú
x+9 ê n ç ç n + 1÷ç
÷ç n + 2 ÷÷.... ç
ç n + n ÷÷ ú
Also y £ and x £ 6 ....(ii) ê è ø è ø è ø ú
5 = lim ê
n ®¥ ê æ x 2 öæ x 2 1 ö æ x 2 1 ö úú
2n
Solving (i) and (ii) we get intersection points as (1, 2), ên × n! ç ÷ç ÷ ç ÷
ç n 2 + 1÷ç n 2 + 22 ÷.... ç n 2 + n 2 ÷ úú
(6, 3), (– 4, 1), (–39, –6) ê
ë è ø è ø è øû
EBD_7809
2016-34 JEE Advanced 2016 Solved Paper
44. (a, b) f(x) = a cos (|x –x|) + b |x| sin (|x3 + x|)
3 3

x /n (a) If a = 0, b = 1
é ù
ê æ x öæ x 1 ö æ x 1 ö ú
ê ç + 1÷ç
ç ÷ç + ÷÷.... ç
ç + ÷ ÷ ú Þ f(x) = |x| sin |x3 + x|
= lim ê èn ø èn 2ø èn n ø ú = x sin (x3 + x), x Î R
ê öæ x 2 1 ö æ x 2 1 ö úú
n ®¥ êæ x 2
êç1. + 1÷ç 2. + ÷ .... çn. + ÷ \ f is differentiable every where
ç 2 ÷ç 2 2÷ ç 2 n ÷ú
êè n
ë øè n ø è n ø ûú (b), (c) If a = 1, b = 0 Þ f(x) = cos3 (|x3 – x|)
= cos3(x3 – x)
which is differentiable every where.
x éê n æ x 1 ö n æç rx 2 1 ö÷ùú
Þ ln f(x) = n ®¥ n êå ln çç n + r ÷÷ - å ln ç 2 + r ÷ú
lim (d) when a = 1, b = 1, f(x) = cos(x3 – x) + x sin (x3 + x)
êër =1 è ø r =1 è n ø ûú which is differentiable at x = 1
\ Only a and b are the correct options.
é ì æ rx öüï ìï 1 æ r 2 x 2 öüïù
x ên ï 1 ú
lim êå íln + ln çç + 1÷÷ý - íln + ln çç 2 + 1÷÷ýú f ( x) g ( x ) é0 ù
= n ®¥ n êër =1 îï r è n ø þï ï î r è n ø ï
þúû 45. (a, d) lim =1 ê form ú
x ® 2 f ¢ ( x) g¢ ( x ) ë0
ê úû
én æ æ r 2 x 2 öù
x ê ln ç1 + rx ö ç1 + ÷ú f ¢ ( x ) g ( x ) + f ( x ) g¢ ( x )
= nlim
®¥ n ê å ç ÷
÷ - ln
ç 2 ÷ú Þ xlim =1
êër =1 è n ø è n ø úû ®2 f ¢¢ ( x ) g¢ ( x ) + f ¢ ( x ) g¢¢ ( x )

1 1 g¢ (2) f (2)
= xòln (1 + xy) dy - xòln 1 + x 2 y2 dy
( ) Þ f ¢¢ 2 g¢ 2 =1 Þ f ( 2 ) = f ¢¢ ( 2 )
0 0 () ()
Let xy = t Þ x dy = dt \ f(x) – f ¢¢ ( x ) = 0 for atleast one x Î R.
x x 2
\ ( )
ln f(x) = ò0 ln (1 + t ) dt -ò0 ln 1 + t dt Q Range of f(x) is ( 0, ¥ )
\ f(x) > 0, xÎ R
x æ 1+ t ö
ln f (x ) =ò
0
ln ç
ç ÷ dt
÷ Þ f(2) > 0 Þ f ¢¢ ( 2 ) > 0
è1 + t 2 ø
Þ f has a local minimum at x = 2
f ¢ (x) æ 1+ x ö 46. (b, c) f(x) = [x2 – 3] is discontinuous at all integral points in
Þ = ln çç ÷÷
f (x ) è1 + x 2 ø é 1 ù
ê - 2 , 2ú
ë û
f ¢ (2) æ3 ö
Þ = ln çç ÷÷ < 0
f (2) è5 ø Which happens when x = 1, 2, 3 ,2

Þ f ¢ (2) < 0 \ (c) is correct é 1 ù


\ f is discontinuous exactly at four points in ê - , 2ú
ë 2 û
f ¢ (3)
æ 2 ö f ¢ (2)
= ln çç ÷÷ <
( ) è 5 ø f (2) \ (d) is not correct
and f 3
Also g ( x ) = ( x + 4x - 7 ) f ( x )

æ 1+ x ö æ 1 ö
Also f ¢ (x) = f(x) ln çç ÷÷ > 0, , x Î (0, 1) Here f is not differentiable at x = 1, 2, 3 Î ç - 2 , 2 ÷
è1 + x 2 ø è ø
\ f is an increasing function. 7
and |x| + |4x – 7| is not differentiable at O and
1 æ1ö 4
\ <1 Þ f ç ÷ £ f (1)
ç ÷
2 è2ø But f(x) = 0, xÎ é ù
ê 3 , 2 ûú
ë
\ (a) is not correct
7
1 2 æ1 ö æ 2 ö \ g(x) becomes differentiable at x =
and < Þ f ç
ç ÷÷£ f ç
ç ÷
÷ 4
3 3 è3 ø è3ø
Hence g(x) is non-differentiable at four points i.e.,
\ (b) is correct
0,1, 2, 3
Hence (b) and (c) are the correct options.
JEE Advanced 2016 Solved Paper 2016-35

49. (b, c, d)
1
47. (a, c, d) z = = x + iy ax + 2y = l
a + ibt
3x – 2y = m
a - ibt
Þ x + iy = a 2
a + b2 t 2
2
For unique solution, ¹ Þ a ¹ –3
3 -2
a -bt \ (b) is the correct option.
Þ x= 2 , y= 2
2 2
a +b t a + b2 t 2 For infinite many solutions and a = – 3
1 x -3 2 l l
Þ x 2 + y2 = 2 2 2
= = = Þ =-1 or l + m = 0
a +b t a 3 -2 m m
x \ (c) is the correct option.
Þ x 2 + y2 - =0
a
-3 2 l
Also if l + m ¹ 0, then = ¹
æ 1 ö 3 -2 m
\ Locus of z is a circle with centre ç , 0 ÷ and
è 2a ø Þ system has no solution.
1 \ (d) is the correct option.
radius = 2 a irrespective of ‘a’ +ve or –ve r
50. (b, c) û ´ v = 1 Þ v sin q = 1 ...(i)
Also for b = 0, a ¹ 0, we get, y = 0
r
\ locus is x-axis
and for a = 0, b ¹ 0 we get x = 0
( )
ˆ uˆ ´ v = 1 Þ v sin q cos a = 1
w. ...(ii)

Þ locus is y-axis. where a is the angle between ŵ and a vector ^ lat to


\ a, c, d are the correct options. r r
48. (a, c, d) u&v .
Let point P on parabola y2 = 4x be (t2, 2t) From (i) and (ii) cos a = 1 Þ a = 0°
... PS is shortest distance, therefore PS should be the r r
Þ ŵ is perpendicular to the plane containing u & v
normal to parabola.
r
Þ ŵ is perpendicular to u
r
2 Clearly there can be infinite many choices for v .
P(t , 2t)
Also if û lies in xy plane i.e., û = u1iˆ + u 2 ˆj then
Q r
ŵ.u = 0

S(2, 8) Þ u1 + u2 = 0 Þ u1 = u 2

Also if û lies in xz plane, i.e., û = u1iˆ + u 3kˆ then


r
Equation of normal to y2 = 4x at P (t2, 2t) is ŵ.u = 0 Þ u1 + 2u3 = 0 Þ |u1| =2|u3|
y – 2t = – t(x – t2) Hence (b) and (c) are the correct options.
It passes through S(2, 8)
For (Qs. 51-52)
\ 8 – 2t = – t(2 – t2) Þ t3 = 8 or t = 2
\ P(4, 4) (X, Y) = {(6, 0), (4, 1), (3, 3), (2, 2), (4, 4), (0, 6)}
51. (b) P(X > Y) = P(T1 wins 2 games or T1 win one game other
-1 1
Also slope of tangent to circle at Q = = is a draw)
Slope of PS 2
1 1 æ1 1 1 1 ö 1 1 5
Equation of normal at t = 2 is 2x + y = 12 = ´ +ç
ç ´ + ´ ÷ ÷= + =
Clearly x-intercept = 6 2 2 è 2 6 6 2 ø 4 6 12
SP = 2 5 and SQ = r = 2 52. (c) P(X = Y) = P(T1 wins 1 game loses other game or both
\ Q divides SP in the ratio SQ : PQ the games draw)

= 2: 2 ( ) (
5 - 1 or 5 +1 : 4) æ 1 1 1 1 ö 1 1 1 1 13
= ç ´ + ´ ÷+ ´ = + =
Hence a, c , d are the correct options. è 2 3 3 2 ø 6 6 3 36 36
EBD_7809
2016-36 JEE Advanced 2016 Solved Paper
For (Qs. 53-54) Putting y = 0 in above equation, we get
x 2 y2 8 1 x =-
9
For ellipse + =1, e = 1 - =
9 8 9 3 10
\ F1(– 1, 0) and F2 (1, 0) æ 9 ö
Parabola with vertex at (0, 0) and focus at F2(1, 0) is y2 = 4x. \ Orthocentre ç - , 0 ÷
è 10 ø
æ3 ö 54. (c) Tangents to ellipse at M and N are
Intersection points of ellipse and parabola are M ç
ç , 6÷
÷
è 2 ø
x y 6 x y 6
+ = 1 and - =1
æ3 ö 6 8 6 8
and N ç
ç ,- 6 ÷
÷
è2 ø Their intersection point is R (6, 0)
æ3 ö
æ3 ö Also normal to parabola at M ç , 6 ÷ is
Mç , 6 ÷ è2 ø
è 2 ø
6æ 3ö
y- 6 = - çx - ÷
2 è 2ø
Q
æ7 ö
F1(–1, 0) F2(1, 0) Its intersection with x-axis is Q ç
ç , 0÷
÷
è2 ø

1 5 5 6
æ3 ö Now ar (DMQR) = ´ ´ 6=
Nç ,- 6 ÷ 2 2 4
è2 ø
Also area (MF1NF2) = 2 × Ar (F1MF2)
53. (a) For orthocentre of DF1MN, clearly one altitude is x- 1
= 2´ ´ 2´ 6 = 2 6
axis i.e. y = 0 and altitude from M to F1N is y - 6 2

5 æ 3ö ar (DMQR ) 5 6
\ = =5 : 8
= çx- ÷
ç ÷ ar (MF1NF2 ) 4´2 6
2 6 è 2ø
JEE ADVANCED 2015
1. The question paper consists of three parts (Physics, Chemistry and Mathematics). Each part consists of three
sections.
2. Section 1 contains 8 questions. The answer to each of the questions is a single-digit integer ranging from 0 to 9
(both inclusive).
3. Section 2 contains 10 multiple choice questions. Each question has four choices (a), (b), (c) and (d) out of
which ONE OR MORE THAN ONE are correct.
4. Section 3 contains 2 multiple matching questions. One or more entries in Column - I may match with one or
more entries in Column - II.

PAPER - 1

4. Consider a concave mirror and a convex lens (refractive index


PHYSICS = 1.5) of focal length 10 cm each, separated by a distance of
SECTION - I 50 cm in air (refractive index = 1) as shown in the figure. An
object is placed at a distance of 15 cm from the mirror. Its
This section contains 8 questions. Each question, when worked
erect image formed by this combination has magnification
out will result in one integer from 0 to 9 (both inclusive).
M1. When the set-up is kept in a medium of refractive index
1. Two spherical stars A and B emit blackbody radiation. The
radius of A is 400 times that of B and A emits 104 times the 7 M
, the magnification becomes M2. The magnitude 2 is
6 M1
æ lA ö
power emitted from B. The ratio ç l ÷ of their wavelengths
è Bø
lA and lB at which the peaks occur in their respective
radiation curves is
2. A nuclear power plant supplying electrical power to a village 15cm
uses a radioactive material of half life T years as the fuel. The
amount of fuel at the beginning is such that the total power
requirement of the village is 12.5% of the electrical power 50cm
available from the plant at that time. If the plant is able to 5. An infinitely long uniform line charge distribution of charge
meet the total power needs of the village for a maximum period per unit length l lies parallel to the y-axis in the y – z plane at
of nT years, then the value of n is
3. A Young's double slit interference arrangement with slits S1 3
z= a (see figure). If the magnitude of the flux of the
2
4
and S2 is immersed in water (refractive index = ) as shown electric field through the rectangular surface ABCD lying in
3
in the figure. The positions of maximum on the surface of lL
the x – y plane with its centre at the origin is ne
water are given by x 2 = p2 m2l2 – d2 , where l is the 0
wavelength of light in air (refractive index = 1), 2d is the (e0 = permittivity of free space), then the value of n is
separation between the slits and m is an integer. The value of
p is z

S1 L
D 3
C a
d
Air x 2
x
a y
d O
S2 Water
A
B
EBD_780
2015-2 JEE Advanced 2015 Solved Paper
6. Consider a hydrogen atom with its electron in the nth orbital.
w2
An electromagnetic radiation of wavelength 90 nm is used to (a) E1w1 = E2w2 (b) 2
ionize the atom. If the kinetic energy of the ejected electron is w1 = n
10.4 eV, then the value of n is (hc = 1242 eV nm)
E1 E2
7. A bullet is fired vertically upwards with velocity v from the (c) w1w2 = n2 (d) =
surface of a spherical planet. When it reaches its maximum w1 w2
11. A ring of mass M and radius R is rotating with angular speed
1 th
height, its acceleration due to the planet's gravity is of w about a fixed vertical axis passing through its centre O with
4
its value of the surface of the planet. If the escape velocity M
two point masses each of mass at rest at O. These masses
from the planet is vesc = v N , then the value of N is (ignore 8
energy loss due to atmosphere) can move radially outwards along two massless rods fixed
8. Two identical uniform discs roll without slipping on two on the ring as shown in the figure. At some instant the angular
different surfaces AB and CD (see figure) starting at A and C 8
with linear speeds v1 and v2, respectively, and always remain speed of the system is w and one of the masses is at a
9
in contact with the surfaces. If they reach B and D with the
same linear speed and v1 = 3 m/s then v2 in m/s is (g = 10 m/s2) 3
distance of R from O. At this instant the distance of the
5
v1 = 3m/s
A other mass from O is
30m w
B

v2 O
C
27m
D 2 1
(a) R (b) R
3 3
3 4
SECTION - II (c) R (d) R
5 5
This section contains 10 multiple choice questions. Each question 12. The figures below depict two situations in which two infinitely
has 4 choices (a), (b), (c) and (d) out of which ONE or MORE long static line charges of constant positive line charge
THAN ONE are correct. density l are kept parallel to each other. In their resulting
9. Planck’s constant h, speed of light c and gravitational constant electric field, point charges q and –q are kept in equilibrium
G are used to form a unit of length L and a unit of mass M. between them. The point charges are confined to move in the
Then the correct option(s) is(are) x direction only. If they are given a small displacement about
Mµ their equilibrium positions, then the correct statement(s) is(are)
(a) c (b) M µ G
(c) Lµ h (d) L µ G
l l l l
10. Two independent harmonic oscillators of equal mass are
oscillating about the origin with angular frequencies w1 and
w2 and have total energies E1 and E2, respectively. The x x
+q –q
variations of their momenta p with positions x are shown in
a a
the figures. If = n2 and = n, then the correct equation(s)
b R
is(are) (a) Both charges execute simple harmonic motion
P (b) Both charges will continue moving in the direction of
p
their displacement
Energy = E2 (c) Charge +q executes simple harmonic motion while
Energy = E1 charge –q continues moving in the direction of its
b x displacement
x
a R (d) Charge –q executes simple harmonic motion while
charge +q continues moving in the direction of its
displacement
JEE Advanced 2015 Solved Paper 2015-3

13. Two identical glass rods S1 and S2 (refractive index = 1.5)


have one convex end of radius of curvature 10 cm. They are
placed with the curved surfaces at a distance d as shown in
the figure, with their axes (shown by the dashed line) aligned. Q
When a point source of light P is placed inside rod S1 on its
axis at a distance of 50 cm from the curved face, the light rays
emanating from it are found to be parallel to the axis inside
Al 50mm
S2. The distance d is
Fe
S1 S2
P 2mm P
7mm
50 cm d

2475 1875
(a) 60 cm (b) 70 cm (a) µW (b) µW
64 64
(c) 80 cm (d) 90 cm
14. A conductor (shown in the figure) carrying constant current 1875 2475
(c) µW (d) µW
r 49 132
I is kept in the x-y plane in a uniform magnetic field B . If F is
the magnitude of the total magnetic force acting on the 17. For photo-electric effect with incident photon wavelength l,
conductor, then the correct statement(s) is(are) the stopping potential is V0. Identify the correct variation(s)
of V0 with l and 1/l.
y
R R V0 V0
l p/6 p/4 x
L R R L (a) (b)
r l
(a) If B is along $z , F µ (L + R) l

r
(b) If B is along x̂ , F = 0 V0
(c) (d) V0
r
(c) If B is along ŷ , F µ (L + R)
r 1/l 1/l
(d) If B is along $z , F = 0
15. A container of fixed volume has a mixture of one mole of 18. Consider a Vernier callipers in which each 1 cm on the main
hydrogen and one mole of helium in equilibrium at temperature scale is divided into 8 equal divisions and a screw gauge
T. Assuming the gases are ideal, the correct statement(s) is with 100 divisions on its circular scale. In the Vernier callipers,
(are) 5 divisions of the Vernier scale coincide with 4 divisions on
(a) The average energy per mole of the gas mixture is 2RT the main scale and in the screw gauge, one complete rotation
of the circular scale moves it by two divisions on the linear
(b) The ratio of speed of sound in the gas mixture to that in
scale. Then :
helium gas is 6/5 (a) If the pitch of the screw gauge is twice the least count
(c) The ratio of the rms speed of helium atoms to that of of the Vernier callipers, the least count of the screw
hydrogen molecules is 1/2 gauge is 0.01 mm
(d) The ratio of the rms speed of helium atoms to that of (b) If the pitch of the screw gauge is twice the least count
of the Vernier callipers, the least count of the screw
1
hydrogen molecules is gauge is 0.005 mm
2
(c) If the least count of the linear scale of the screw gauge
16. In an aluminium (Al) bar of square cross section, a square is twice the least count of the Vernier callipers, the
hole is drilled and is filled with iron (Fe) as shown in the least count of the screw gauge is 0.01 mm
figure. The electrical resistivities of Al and Fe are 2.7 × 10–8 (d) If the least count of the linear scale of the screw gauge
W m and 1.0 × 10–7 W m, respectively. The electrical resistance is twice the least count of the Vernier callipers, the
between the two faces P and Q of the composite bar is least count of the screw gauge is 0.005 mm
EBD_780
2015-4 JEE Advanced 2015 Solved Paper
SECTION - III CHEMISTRY
This section contains TWO questions. Each question contains
two columns, Column I and Column II. Column I has four entries SECTION - I
(A), (B), (C) and (D). Column II has five entries (P), (Q), (R), (S)
and (T). Match the entries in Column I with the entries in Column This section contains 8 questions. Each question, when worked
II. One or more entries in Column I may match with one or more out will result in one integer from 0 to 9 (both inclusive).
entries in Column II. 1. Among the triatomic molecules/ions, BeCl2, N 3– , N2O,
19. Match the nuclear processes given in column I with the
appropriate option(s) in column II. NO+2 , O3, SCl2, ICl 2– , I3– and XeF2, the total number of
Column I Column II
linear molecule(s)/ion(s) where the hybridization of the central
(A) Nuclear fusion (P) Absorption of thermal
atom does not have contribution from the d-orbital(s) is
neutrons by 235
92 U [Atomic number : S = 16, Cl = 17, I = 53 and Xe = 54]
(B) Fission in a nuclear (Q) 60 Co nucleus 2. Not considering the electronic spin, the degeneracy of the
27
second excited state (n = 3) of H atom is 9, while the
reactor
degeneracy of the second excited state of H– is
(C) b-decay (R) Energy production in
stars via hydrogen 3. All the energy released from the reaction X ® Y, DrG° = –193
conversion to helium kJ mol–1 is used for oxidizing M+ as M+ ® M3+ + 2e–,
(D) g-ray emission (S) Heavy water E° =–0.25 V
(T) Neutrino emission Under standard conditions, the number of moles of M+
20. A particle of unit mass is moving along the x-axis under the oxidized when one mole of X is converted to Y is
influence of a force and its total energy is conserved. Four
[F = 96500 C mol–1]
possible forms of the potential energy of the particle are given
in column I (a and U0 constants). Match the potential energies 4. If the freezing point of a 0.01 molal aqueous solution of a
in column I to the corresponding statement(s) in column II. cobalt(III) chloride-ammonia complex (which behaves as a
Column I Column II strong electrolyte) is –0.0558°C, the number of chloride(s) in
the coordination sphere of the complex is
2
U0 é æ x ö2 ù [Kf of water = 1.86 K kg mol–1]
(A) U1(x) = ê1 - ç ÷ ú (P) The force acting on the
2 êë è a ø úû 5. The total number of stereoisomers that can exist for M is
particle is zero at x = a
2
H3C CH3
U0 æ x ö
(B) U2(x) = ç ÷ (Q) The force acting on the
2 èaø
particle is zero at x = 0
2
U0 æxö H3C
(C) U3(x) = ç ÷ (R) The force acting on the
2 èaø
O
M
é æ x ö2 ù
exp ê- ç ÷ ú particle is zero at x = – a
êë è a ø úû 6. The number of resonance structures for N is

U0 OH
(D) U4(x) = (S) The particle experiences
2 NaOH
N
éx 1æ x ö 3ù
ê – ç ÷ ú an attractive force towards
êë a 3 è a ø úû 7. The total number of lone pairs of electrons in N2O3 is
x = 0 in the region |x| < a 8. For the octahedral complexes of Fe3+ in SCN– (thiocyanato-S)
(T) The particle with total and in CN– ligand environments, the difference between the
U0 spin-only magnetic moments in Bohr magnetons (when
energy can oscillate approximated to the nearest integer) is
4
about the point x = – a [Atomic number of Fe = 26]
JEE Advanced 2015 Solved Paper 2015-5

SECTION - II 12. The major product of the reaction is


This section contains 10 multiple choice questions. Each question H3C NaNO2, aqueous HCl
has 4 choices (a), (b), (c) and (d) out of which ONE or MORE 0°C
THAN ONE are correct.
9. The major product of the following reaction is
H3C H 3C CO2H
O i. KOH, H 2O
ii. H,+ heat (a) (b)

CH3
O H3C CO2H H3C
CH3 CH3 (c) (d)
O O
(a) (b)
13. The correct statement(s) about Cr2+ and Mn3+ is(are)
[Atomic numbers of Cr = 24 and Mn = 25]
O O CH3 (a) Cr2+ is a reducing agent
CH3 (b) Mn3+ is an oxidizing agent
(c) (d) (c) Both Cr2+ and Mn3+ exhibit d4 electronic configuration
(d) When Cr2+ is used as a reducing agent, the chromium
10. In the following reaction, the major product is ion attains d5 electronic configuration
CH3 14. Copper is purified by electrolytic refining of blister copper.
CH2 The correct statement(s) about this process is(are)
HC 1 equivalent HBr (a) Impure Cu strip is used as cathode
2

(b) Acidified aqueous CuSO4 is used as electrolyte


CH3 (c) Pure Cu deposits at cathode
CH3
(d) Impurities settle as anode-mud
CH3
(a) H2C (b) H C 15. Fe3+ is reduced to Fe2+ by using
3
Br (a) H2O2 in presence of NaOH
Br
(b) Na2O2 in water
CH3 CH3 (c) H2O2 in presence of H2SO4
(c) (d) Br (d) Na2O2 in presence of H2SO4
H2C Br H 3C 16. The %yield of ammonia as a function of time in the reaction
11. The structure of D-(+)-glucose is N2(g) + 3H2(g) 2NH3(g), DH < 0 at (P, T1) is given below
CHO
H OH
% y ie ld

HO H
H OH
H OH
time
CH2OH If this reaction is conducted at (P, T2), with T2 > T1, the
The structure of L-(–)-glucose is %yield of ammonia as a function of time is represented by

T2 T1

T1
% y ie ld

T2
% y i e ld

(a) (b)
(a) (b)

time time

T2
T1
T1
% y ie ld

% y ie ld

(c) (d) T2
(c) (d)

time time
EBD_780
2015-6 JEE Advanced 2015 Solved Paper
17. If the unit cell of a mineral has cubic close packed (ccp) array MATHEMATICS
of oxygen atoms with m fraction of octahedral holes occupied
by aluminium ions and n fraction of tetrahedral holes occupied SECTION - I
by magnesium ions, m and n, respectively, are
This section contains 8 questions. Each question, when worked
1 1 1 out will result in one integer from 0 to 9 (both inclusive).
(a) , (b) 1,
2 8 4 1. If the normals of the parabola y2 = 4x drawn at the end points
1 1 1 1 of its latus rectum are tangents to the circle (x – 3)2 + (y + 2)2
(c) , (d) , = r2, then the value of r2 is
2 2 4 8
18. Compound(s) that on hydrogenation produce(s) optically
inactive compound(s) is (are)
2.
[ x], x £ 2
Let f : R ® R be a function defined by f ( x) = 0, x > 2 {
H Br H Br where [x] is the greatest integer less than or equal to x, if
(a) (b) CH3 2
H 3C CH 3 H 2C xf ( x 2 )
I= ò dx
2 + f ( x + 1) , then the value of (4I – 1) is
H Br Br H
-1

(c) H2C (d) H C 3. A cylindrical container is to be made from certain solid


CH3 CH 3
2 material with the following constraints: It has a fixed inner
CH3 volume of V mm3, has a 2 mm thick solid wall and is open at
the top. The bottom of the container is a solid circular disc
SECTION - III of thickness 2 mm and is of radius equal to the outer radius
This section contains TWO questions. Each question contains of the container.
two columns, Column I and Column II. Column I has four entries If the volume of the material used to make the container is
minimum when the inner radius of the container is 10 mm,
(A), (B), (C) and (D). Column II has five entries (P), (Q), (R), (S)
and (T). Match the entries in Column I with the entries in Column V
then the value of is
II. One or more entries in Column I may match with one or more 250p
entries in Column II.
p
x2 +
19. Match the anionic species given in Column-I that are present 6
in the ore(s) given in Column-II. 4. Let F ( x) = ò 2cos 2 t (dt ) for all xÎR and
Column-I Column-II x
(A) Carbonate (P) Siderite
é 1ù
(B) Sulphide (Q) Malachite f : ê0, ú ® [0, ¥) be a continuous function. For
(C) Hydroxide (R) Bauxite ë 2û
(D) Oxide (S) Calamine é 1ù
(T) Argentite a Î ê0, ú , if F ¢(a) + 2 is the area of the region bounded
ë 2û
20. Match the thermodynamic processes given under Column-I
by x = 0, y = 0, y = f(x) and x = a, then f(0) is
with the expressions given under Column-II. 5. The number of distinct solutions of the equation
Column-I Column-II
5
(A) Freezing of water at (P) q = 0 cos2 2 x + cos 4 x + sin 4 x + cos6 x + sin 6 x = 2
273 K and 1 atm 4
(B) Expansion of 1 mol of (Q) w = 0 in the interval [0, 2p] is
6. Let the curve C be the mirror image of the parabola y2 = 4x
an ideal gas into a vacuum
with respect to the line x + y + 4 = 0. If A and B are the points
under isolated conditions
of intersection of C with the line y = –5, then the distance
(C) Mixing of equal volumes (R) DSsys < 0 between A and B is
of two ideal gases at 7. The minimum number of times a fair coin needs to be tossed,
constant temperature so that the probability of getting at least two heads is at
and pressure in an least 0.96, is
isolated container 8. Let n be the number of ways in which 5 boys and 5 girls can
(D) Reversible heating of (S) DU = 0 stand in a queue in such a way that all the girls stand
H2(g) at 1 atm from 300 K consecutively in the queue. Let m be the number of ways in
to 600 K, followed by which 5 boys and 5 girls can stand in a queue in such a way
that exactly four girls stand consecutively in the queue. Then
reversible cooling to 300 K
at 1 atm m
the value of is
(T) DG = 0 n
JEE Advanced 2015 Solved Paper 2015-7

SECTION - II differential equation Py ¢¢ + Qy ¢ + 1 = 0 , where P, Q are


This section contains 10 multiple choice questions. Each question
has 4 choices (a), (b), (c) and (d) out of which ONE or MORE æ dy d 2 yö
functions of x, y and y¢ ç here y ¢= , y ¢¢ = ÷ , then
THAN ONE are correct. è dx dx 2 ø
9. Which of the following values of a satisfy the equation
which of the following statements is (are) true?
(a) P = y + x (b) P = y – x
(1 + a)2 (1 + 2a) 2 (1 + 3a ) 2
(c) P + Q = 1 – x + y + y¢ + (y') (d) P – Q = x + y – y¢ –(y')2
2
(2 + a)2 (2 + 2a)2 (2 + 3a )2 = –648a ? 15. Let g : R ® R be a differentiable function with g(0) = 0, g'(0)
(3 + a )2 (3 + 2a )2 (3 + 3a)2
ì x
ï g ( x ), x ¹ 0
(a) –4 (b) 9 = 0 and g'(1) ¹ 0. Let f ( x ) = í | x|
ïî0, x =0
(c) –9 (d) 4
3
10. In R , consider the planes P1 : y = 0 and P2 : x + z = 1. Let P3 and h(x) = e|x| for all x Î R . Let (foh)(x) denote f(h(x)) and
be the plane, different from P1 and P2, which passes through (hof)(x) denote h(f(x)). Then which of the following is (are) true?
the intersection of P1 and P2. If the distance of the point
(a) f is differentiable at x = 0
(0, 1, 0) from P3 is 1 and the distance of a point (a, b, g) from
(b) h is differentiable at x = 0
P3 is 2, then which of the following relations is (are) true?
(c) foh is differentiable at x = 0
(a) 2a + b +2g + 2 = 0 (b) 2a – b +2g + 4 = 0
(d) hof is differentiable at x = 0
(c) 2a + b –2g – 10 = 0 (d) 2a – b +2g – 8 = 0
11. In R3, let L be a straight line passing through the origin. æp æp öö p
Suppose that all the points on L are at a constant distance 16. Let f ( x) = sin ç sin ç sin x ÷ ÷ for all x Î R and g(x) =
è6 è2 øø 2
from the two planes P1 : x + 2y – z + 1 = 0 and P2 : 2x – y + z
– 1 = 0. Let M be the locus of the feet of the perpendiculars sin x for all x Î R. Let (fog)(x) denote f(g(x)) and (gof)(x)
drawn from the points on L to the plane P1. Which of the denote g(f(x)). Then which of the following is (are) true?
following points lie (s) on M? é 1 1ù
(a) Range of f is ê - , ú
æ 5 2ö æ 1 1 1ö ë 2 2û
(a) ç 0, – , – ÷ (b) ç – , – , ÷
è 6 3ø è 6 3 6ø
é 1 1ù
(b) Range of fog is ê - , ú
æ 5 1ö æ 1 2ö ë 2 2û
(c) ç – ,0, ÷ (d) ç – ,0, ÷
è 6 6ø è 3 3ø
f ( x) p
12. Let P and Q be distinct points on the parabola y2 = 2x such (c) lim =
x ®0 g ( x) 6
that a circle with PQ as diameter passes through the vertex
O of the parabola. If P lies in the first quadrant and the area (d) There is an x Î R such that ( gof )( x ) = 1
of the triangle DOPQ is 3 2 , then which of the following is r uuur r uuur r uuur
17. Let DPQR be a triangle. Let a = QR, b = RP and c = PQ . If
(are) the coordinates of P?
r r rr
(a) (4, 2 2) (b) (9,3 2) a = 12 , b = 4 3 , b . c = 24 , then which of the following
is (are) true?
æ1 1 ö
(c) ç , ÷ (d) (1, 2) r2 r2
è4 2ø c r c r
(a) - a = 12 (b) + a = 30
2 2
13. Let y(x) be a solution of the differential equation
r r r r r r
(1 + e x ) y ¢ + ye x = 1 . If y(0) = 2, then which of the following (c) a ´ b + c ´ a = 48 3 (d) a. b = –72
statement is (are) true? 18. Let X and Y be two arbitrary, 3 × 3, non-zero, skew-symmetric
(a) y(–4) = 0 (b) y(–2) = 0 matrices and Z be an arbitrary 3 × 3, non zero, symmetric
(c) y(x) has a critical point in the interval (–1, 0) matrix. Then which of the following matrices is (are) skew
(d) y(x) has no critical point in the interval (–1, 0) symmetric?
14. Consider the family of all circles whose centers lie on the (a) Y3Z4 – Z4Y3 (b) X44 + Y44
straight line y = x. If this family of circle is represented by the 4 3
(c) X Z – Z X 3 4 (d) X23 + Y23
EBD_780
2015-8 JEE Advanced 2015 Solved Paper

SECTION - III
This section contains TWO questions. Each question contains two columns, Column I and Column II. Column I has four entries (A),
(B), (C) and (D). Column II has five entries (P), (Q), (R), (S) and (T). Match the entries in Column I with the entries in Column II. One
or more entries in Column I may match with one or more entries in Column II.
19. Match the following
Column I Column II
(A) In R2, if the magnitude of the projection vector (P) 1
of the vector a$i + bˆj on 3iˆ + ˆj is 3 and
if a = 2 + 3b , then possible value of a is/are
(B) Let a and b be real numbers such that the (Q) 2
ì -3ax 2 - 2, x < 1
function f ( x) = í 2
îbx + a , x ³ 1
if differentiable for all x Î R
Then possible value of a is (are)
(C) Let w ¹ 1 be a complex cube root of unity. (R) 3
If (3 - 3w + 2w )2 4n+3 2 4n+3
+ (2 + 3w - 3w )
+(-3 + 2w + 3w2 ) 4 n +3 = 0, then possible
value (s) of n is (are)
(D) Let the harmonic mean of two positive real (S) 4
numbers a and b be 4. If q is a positive
real number such that a, 5, q, b is an arithmetic
progression, then the value(s) of | q – a | is (are)
(T) 5
20. Match the following.
Column I Column II
(A) In a triangle DXYZ, let a, b, and c be the (P) 1
lengths of the sides opposite to the angles
X, Y and Z, respectively. If 2(a2 – b2) = c2
sin( X - Y )
and l = , then possible values of
sin Z
n for which cos(npl) = 0 is (are)
(B) In a triangle DXYZ, let a, b and c be the (Q) 2
lengths of the sides opposite to the angles
X, Y, and Z respectively. If 1 + cos 2X – 2cos 2Y
a
= 2 sin X sin Y, then possible value (s) of is (are)
b
(C) In R2, let 3i + ˆj, iˆ + 3 ˆj and biˆ + (1 -b) ˆj (R) 3
be the position vectors of X , Y, and Z with
respect to the origin O, respectively. If the
distance of Z from the bisector of the
uuur uuur 3
acute angle of OX with OY is ,
2
then possible value(s) of b is (are)
(D) Suppose that F(a) denotes the area of the (S) 5
region bounded by x = 0, x = 2, y2 = 4x and
y =| ax - 1| + | ax - 2 | +ax , where a Î {0, 1} .

Then the value(s) of F (a ) + 8 2 , when


3
a = 0 and a = 1, is (are)
(T) 6
JEE Advanced 2015 Solved Paper 2015-9

PAPER - 2
1. The question paper consists of three parts (Physics, Chemistry and Mathematics). Each part consists of three
sections.
2. Section 1 contains 8 questions. The answer to each of the questions is a single-digit integer ranging from 0 to 9
(both inclusive).
3. Section 2 contains 8 multiple choice questions. Each question has four choice (a), (b), (c) and (d) out of which
ONE OR MORE THAN ONE are correct.
4. Section 3 contains 2 paragraphs each describing theory, experiment and data etc. four questions relate to two
paragraphs with two questions on each paragraph. Each question pertaining to a particular passage should have
one or more correct answer among the four given choices (a), (b), (c) and (d).

PHYSICS 4. An electron is an excited state of Li2+ ion has angular


momentum 3h/2p. The de Broglie wavelength of the electron
SECTION - I in this state is pp a0 (where a0 is the Bohr radius). The value
This section contains 8 questions. Each question, when worked of p is
out will result in one integer from 0 to 9 (both inclusive). 5. A large spherical mass M is fixed at one position and two
identical point masses m are kept on a line passing through
1. For a radioactive material, its activity A and rate of change of the centre of M (see figure). The point masses are connected
dN dA by a rigid massless rod of length l and this assembly is free
its activity R are defined as A = - and R = – , where
dt dt to move along the line connecting them. All three masses
N(t) is the number of nuclei at time t. Two radioactive sources interact only through their mutual gravitational interaction.
P (mean life t) and Q (mean life 2t) have the same activity at When the point mass nearer to M is at a distance r = 3l from
t = 0. Their rates of change of activities at t = 2t are RP and
æ M ö
RP n M, the tension in the rod is zero for m = k ç ÷ . The value
RQ, respectively. If = , then the value of n is è 288 ø
RQ e of k is
2. The monochromatic beam of light is incident at 60° on one
face of an equilateral prism of refractive index n and emerges
from the opposite face making an angle q(n) with the normal
dq
(see the figure). For n = 3 the value of q is 60° and = m. 6. The energy of a system as a function of time t is given as E(t)
dn
= A2 exp(–at,) where a = 0.2 s–1. The measurement of A has
The value of m is
an error of 1.25%. If the error in the measurement of time is
1.50%, the percentage error in the value of E(t) at t = 5 s is
7. The densities of two solid spheres A and B of the same radii
ærö
R vary with radial distance r as rA(r) = k ç ÷ and rB(r) =
60° q èRø
5
ærö
k ç ÷ , respectively, where k is a constant. The moments
è Rø
3. In the following circuit, the current through the resistor of inertia of the individual spheres about axes passing through
R (= 2 W) is I amperes. The value of I is
IB n
(=2W) their centres are IA and IB, respectively. If , I = 10 , the
A
value of n is
8. Four harmonic waves of equal frequencies and equal
p 2p
intensities I0 have phase angles 0, , and p. When they
3 3
are superposed, the intensity of the resulting wave is nI0.
The value of n is
EBD_780
2015-10 JEE Advanced 2015 Solved Paper
SECTION - II 12. A spherical body of radius R consists of a fluid of constant
density and is in equilibrium under its own gravity. If P(r) is
This section contains 8 multiple choice questions. Each question the pressure at r(r < R), then the correct option(s) is (are)
has 4 choices (a), (b), (c) and (d) out of which ONE or MORE
THAN ONE are correct. P (r = 3R /4) 63
(a) P(r = 0) = 0 (b) =
P (r = 2R /3) 80
9. In terms of potential difference V, electric current I, permittivity
e0, permeability m0 and speed of light c, the dimensionally P (r = 3R /5) 16 P (r = R /2) 20
correct equation(s) is(are) (c) = (d) =
P (r = 2R /5) 21 P (r = R /3) 27
(a) m0I2 = e0V2 (b) m0I = m0V
13. A parallel plate capacitor having plates of area S and plate
(c) I = e0cV (d) m0cI = e0V
separation d, has capacitance C1 in air. When two dielectrics
10. Consider a uniform spherical charge distribution of radius of different relative primitivities (e1 = 2 and e2 = 4) are
R1 centred at the origin O. In this distribution, a spherical introduced between the two plates as shown in the figure,
cavity of radius R 2, centred at P with distance OP = a
= R1 – R2 (see figure) is made. If the electric field inside the C2
r uruur the capacitance becomes C2. The ratio C is
cavity at position r is E (r), then the correct statement(s) 1
is(are)

ur
(a) E is uniform, its magnitude is independent of R2 but
r
its
ur direction depends on r
(b) E is uniform, its magnitude depends on R2 and its
r
direction depends on r
ur (a) 6/5 (b) 5/3
(c) E is uniform, its magnitude is independent of a but its
r (c) 7/5 (d) 7/3
direction depends on a 14. An ideal monoatomic gas is confined in a horizontal cylinder
ur
(d) E is uniform and both its magnitude and direction by a spring loaded piston (as shown in the figure). Initially
r the gas is at temperature T1, pressure P1 and volume V1 and
depend on a
the spring is in its relaxed state. The gas is then heated very
11. In plotting stress versus strain curves for two materials P slowly to temperature T2, pressure P2 and volume V2. During
and Q,a student by mistake puts strain on the y-axis and this process the piston moves out by a distance x. Ignoring
stress on the x-axis as shown in the figure. Then the correct the friction between the piston and the cylinder, the correct
statement(s) is (are) statement(s) is (are)

(a) If V2 = 2V1 and T2 = 3T1, then the energy stored in the

(a) P has more tensile strength than Q


1
spring is PV
(b) P is more ductile than Q 4 1 1
(c) P is more brittle than Q (b) If V2 = 2V1 and T2 = 3T1, then the change in internal
(d) The Young's modulus of P is more than that of Q energy is 3P1V1
JEE Advanced 2015 Solved Paper 2015-11

(c) If V2 = 3V1 and T2 = 4T1, then the work done by the gas internal reflections at the interface of the media n1 and n2 as shown
7 in the figure. All rays with the angle of incidence i less than a
is PV particular value im are confined in the medium of refractive index
3 1 1
n1. The numerical aperture (NA) of the structure is defined as
(d) If V2 = 3V1 and T2 = 4T1, then the heat supplied to the
sin im.
17
gas is PV
6 1 1
236
15. A fission reaction is given by 92 U ® 140
54 Xe +
94
38 Sr + x + y,
236
where x and y are two particles. Considering 92 U to be at
rest, the kinetic energies of the products are denoted by KXe,
KSr, Kx(2 MeV) and Ky(2 MeV), respectively. Let the binding
236 140 94
energies per nucleon of 92 U, 54 Xe and 38 Sr be 7.5 MeV,
8.5 MeV and 8.5 MeV, respectively. Considering different 17. For two structure namely S1 with n1 = 45 / 4 and n2 = 3/2,
conservation laws, the correct option(s) is(are) and S2 with n1 = 8/5 and n2 = 7/5 and taking the refractive
(a) x = n, y = n, KSr = 129 MeV, KXe = 86 MeV index of water to be 4/3 and that of air to be 1, the correct
(b) x = p, y = e–, KSr = 129 MeV, KXe = 86 MeV option(s) is(are)
(c) x = p, y = n, KSr = 129 MeV, KXe = 86 MeV (a) NA of S1 immersed in water is the same as that of S2
(d) x = n, y = n, KSr = 86 MeV, KXe = 129 MeV
16
16. Two spheres P and Q of equal radii have densities r1 and r2, immersed in a liquid of refractive index
respectively. The spheres are connected by a massless string 3 15
and placed in liquids L1 and L2 of densities s1 and s2 and 6
viscosities h1 and h2, respectively. They float in equilibrium (b) NA of S1 immersed in liquid of refractive index is
with the sphere P in L1 and sphere Q in L2 and the string 15
being taut (see figure). If sphere P alone in L2 has terminal the same as that of S2 immersed in water
ur ur (c) NA of S1 placed in air is the same as that of S2 immersed
velocity VP and Q alone in L1 has terminal velocity V Q ,
4
then in liquid of refractive index
15
(d) NA of S1 placed in air is the same as that of S2 placed in
L1 water
18. If two structure of same cross-sectional area, but different
numerical apertures NA1 and NA2(NA2 < NA1) are joined
L2 longitudinally, the numerical aperture of the combined
structure is

NA1 NA2
(a) (b) NA1 + NA2
NA1 + NA2
ur ur
VP h VP h2
ur = 1 (c) NA 1 (d) NA 2
(a) h (b) ur = h
VQ 2 VQ 1 PARAGRAPH 2
ur ur ur ur In a thin rectangular metallic strip a constant current I flows along
(c) V P .V Q > 0 (d) V P .V Q < 0
the positive x-direction, as shown in the figure. The length, width
SECTION - III and thickness of the strip are l, w and d, respectively.
su
This section contains 2 paragraphs, each describing theory, A uniform magnetic field B is applied on the strip along the
experiments, data etc. four questions related to the two paragraphs positive y-direction. Due to this, the charge carriers experience a
with two questions on each paragraph. Each question has one or net deflection along the z-direction. This results in accumulation
more than one correct answer(s) among the four given options of charge carriers on the surface PQRS and appearance of equal
(a), (b), (c) and (d). and opposite charges on the face opposite to PQRS. A potential
PARAGRAPH 1 difference along the z-direction is thus developed. Charge
Light guidance in an optical fibre can be understood by considering accumulation continues until the magnetic force is balanced by
a structure comprising of thin solid glass cylinder of refractive the electric force. The current is assumed to be uniformly
index n1 surrounded by a medium of lower refractive index n2. The distributed on the cross-section of the strip and carried by
light guidance in the structure takes place due to successive total electrons.
EBD_780
2015-12 JEE Advanced 2015 Solved Paper
4. In dilute aqueous H2 SO 4 , the complex diaquodio-
xalatoferrate(II) is oxidized by MnO4 –. For this reaction, the
ratio of the rate of change of [H+] to the rate of change of
[MnO4–] is
5. The number of hydroxyl group(s) in Q is

H+ aqueous dilute KMnO (excess)


¾¾¾ ® P ¾¾¾¾¾¾¾¾¾¾¾
4
®Q
19. Consider two different metallic strips (1 and 2) of the same heat 0° C

material. Their lengths are the same, widths are w1 and w2


and thicknesses are d1 and d2 respectively. Two points K
and M are symmetrically located on the opposite faces parallel 6. Among the following, the number of reaction(s) that
to the x-y plane (see figure). V1 and V2 are the potential produce(s) benzaldehyde is
differences between K and M in strips 1 and 2, respectively.
Then, for a given current I flowing through them in a given
magnetic field strength B, the correct statement(s) is(are) CO, HCl
I. ¾¾¾¾¾¾¾¾
Anhydrous AlCl / CuCl
®
3
(a) If w1 = w2 and d1 = 2d2, then V2 = 2V1
(b) If w1 = w2 and d1 = 2d2, then V2 = V1
(c) If w1 = 2w2 and d1 = d2, then V2 = 2V1
H O
(d) If w1 = 2w2 and d1 = d2, then V2 = V1 II. ¾¾¾2
100°C
®
20. Consider two different metallic strips (1 and 2) of same
dimensions (length l, width w and thickness d) with carrier
densities n1 and n2, respectively. Strip 1 is placed in magnetic
H2
field B1 and strip 2 is placed in magnetic field B2, both along III. ¾¾¾¾¾
Pd - BaSO
®
4
positive y-directions. Then V1 and V2 are the potential
differences developed between K and M in strips 1 and 2,
respectively. Assuming that the current I is the same for
both the strips, the correct option(s) is(are) DIBAL –H
¾¾¾¾¾¾ ®
IV. Toluene,–78°C
(a) If B1 = B2 and n1 = 2n2, then V2 = 2V1 H 2O

(b) If B1 = B2 and n1 = 2n2, then V2 = V1


7. In the complex acetylbromidodicarbonylbis
(c) If B1 = 2B2 and n1 = n2, then V2 = 0.5V1
(triethylphosphine) iron (II), the number of Fe–C bond(s) is
(d) If B1 = 2B2 and n1 = n2, then V2 = V1 8. Among the complex ions, [Co(NH2–CH2–CH2–NH2)2Cl2]+,
[CrCl2(C2O4)2]3–, [Fe(H2O)4(OH)2]+, [Fe(NH3)2(CN)4]–,
CHEMISTRY [Co(NH2–CH2–CH2–NH2)2(NH3)Cl]2+ and
[Co(NH3)4(H2O)Cl]2+, the number of complex ion(s) that
SECTION - I show(s) cis-trans isomerism is
This section contains 8 questions. Each question, when worked SECTION - II
out will result in one integer from 0 to 9 (both inclusive). This section contains 8 multiple choice questions. Each question
1. Three moles of B2H6 are completely reacted with methanol. has 4 choices (a), (b), (c) and (d) out of which ONE or MORE
The number of moles of boron containing product formed is THAN ONE are correct.
2. The molar conductivity of a solution of a weak acid HX (0.01
9. The major product U in the following reactions is
M) is 10 times smaller than the molar conductivity of a solution
0 0
of a weak acid HY (0.10 M). If l x – » l y – the difference in CH =CH–CH , H+ radical, initiator, O
2
¾¾¾¾¾¾¾¾ 3 2®
® T ¾¾¾¾¾¾¾¾
high pressure, heat
their pKa values, pKa(HX) – pKa (HY), is (consider degree of
ionization of both acids to be <<1)
238
3. A closed vessel with rigid walls contains 1 mol of 92 U and
238
1 mol of air at 298 K. Considering complete decay of 92 U to
206 3
82 Pb, the ratio of the final pressure to the initial pressure of
the system at 298 K is (a)
JEE Advanced 2015 Solved Paper 2015-13

12. The pair(s) of ions where BOTH the ions are precipitated
upon passing H2S gas in presence of dilute HCl, is(are)
(b) (a) Ba2+, Zn2+ (b) Bi3+, Fe3+
(c) Cu2+, Pb2+ (d) Hg2+, Bi3+
13. Under hydrolytic conditions, the compounds used for
preparation of linear polymer and for chain termination,
respectively, are
(a) CH3SiCl3 and Si(CH3)4
(b) (CH3)2SiCl2 and (CH3)3SiCl
(c) (c) (CH3)2SiCl2 and CH3SiCl3
(d) SiCl4 and (CH3)3SiCl
14. When O2 is adsorbed on a metallic surface, electron transfer
occurs from the metal to O2. The true statement(s) regarding
this adsorption is(are)
(d)
(a) O2 is physisorbed
(b) Heat is released
10. In the following reactions, the major product W is
*
OH (c) Occupancy of p2r of O2 is increased
, NaOH (d) Bond length of O2 is increased
NaNO2 , HCl
0°C
15. One mole of a monoatomic real gas satisfies the equation
p(V – b) = RT where b is a constant. The relationship of
interatomic potential V(r) and interatomic distance r for the
gas is given by
(a)
N=N OH

(a)
OH
(b)
N=N

OH
(b)
N=N
(c)

(d) N=N
(c)

11. The correct statement(s) regarding, (i) HClO, (ii) HClO2,


(iii) HClO3 and (iv) HClO4, is(are)
(a) The number of Cl=O bonds in (ii) and (iii) together is
two
(b) The number of lone pairs of electrons on Cl in (ii) and
(iii) together is three (d)
(c) The hybridization of Cl in (iv) is sp3
(d) Amongst (i) to (iv), the strongest acid is (i)
EBD_780
2015-14 JEE Advanced 2015 Solved Paper
16. In the following reactions, the product S is PARAGRAPH 2
In the following reactions
Pd - BaSO 4
C8 H6 C8 H8 ¾¾¾¾¾¾¾ (i) B2 H6
(ii)H 2 O 2 , NaOH, H 2 O
®X
H2

H2O
HgSO 4 ,H 2SO 4

(a) C8 H 8 O ¾ (i)
¾(ii)¾ ¾ ¾® Y
EtMgBr, H2 O
H + , heat

19. Compound X is

(b)
(a) (b)

(c) (d)
(c)
20. The major compound Y is

(a)

(d)

(b)
SECTION - III
This section contains 2 paragraphs, each describing theory,
experiments, data etc. four questions related to the two paragraphs (c)
with two questions on each paragraph. Each question has one or
more than one correct answer(s) among the four given options
(a), (b), (c) and (d).
PARAGRAPH 1
When 100 mL of 1.0 M HCl was mixed with 100 mL of 1.0 M NaOH (d)
in an insulated beaker at constant pressure, a temperature increase
of 5.7 °C was measured for the beaker and its contents (Expt.1).
Because the enthalpy of neutralization of a strong acid with a
strong base is a constant (–57.0 kJ mol–1), this experiment could MATHEMATICS
be used to measure the calorimeter constant. In a second
experiment (Expt. 2), 100 mL of 2.0 M acetic acid (Ka = 2.0 × 10–5) SECTION - I
was mixed with 100 mL of 1.0 M NaOH (under identical conditions This section contains 8 questions. Each question, when worked
to Expt. 1) where a temperature rise of 5.6 °C was measured. out will result in one integer from 0 to 9 (both inclusive).
1. Let m and n be two positive integers greater than 1. If
(Consider heat capacity of all solutions as 4.2 J g–1 K–1 and density
of all solutions as 1.0 g mL–1) æ cos(a n ) ö
e –e
lim ç ÷ = – æç e ö÷ then the value of m is
17. Enthalpy of dissociation (in kJ mol–1) of acetic acid obtained a ® 0 çè am ÷ø è 2ø n
from the Expt.2 is
(a) 1.0 (b) 10.0 1
9 x +3 tan –1 x æ 12 + 9 x 2 ö
(c) 24.5 (d) 51.4 2. If a = ò (e ) ç
è 1 + x2 ø
–1
÷ dx where tan x takes
0
18. The pH of the solution after Expt. 2 is
(a) 2.8 (b) 4.7 æ 3p ö
only principal values, then the value of çè log e |1 + a | – ÷ø
(c) 5.0 (d) 7.0 4
is
JEE Advanced 2015 Solved Paper 2015-15

3. Let f : ¡ ® ¡ be a continuous odd function, which 10. Let E1 and E2 be two ellipses whose centers are at the origin.
1 The major axes of E1 and E2 lie along the x-axis and the
vanishes exactly at one point and f (1) = . Suppose that y-axis, respectively. Let S be the circle x2 + (y – 1)2 = 2. The
2
x
straight line x + y = 3 touches the curves S, E1 and E2 at P, Q
x
F(x) = ò f (t )dt for all x Î [–1, 2] and G(x) = ò t | f ( f (t)) | dt for 2 2
–1 –1 and R respectively. Suppose that PQ = PR = . If e1 and
3
F ( x) 1 æ 1ö
all xÎ[–1, 2]. If lim = , then the value of f çè ÷ø is e2 are the eccentricities of E1 and E2, respectively, then the
x ®1 G ( x ) 14 2 correct expression(s) is (are)
r r
4. Suppose that p, q and rr are three non-coplanar vectors in
r r r r 43 7
¡3 . Let the components of a vector s along p, q and r (a) e12 + e22 = (b) e1e2 =
be 4, 3 and 5, respectively. If the components of this vector 40 2 10
r r r r r r r r r r
s along (– p + q + r ),( p – q + r ) and (– p – q + r ) are x, y
and z, respectively, then the value of 2x + y + z is 5 3
(c) e12 – e22 = (d) e1e2 =
æ kpö æ kpö 8 4
5. For any integer k, let ak = cos çè ÷ø + i sin çè ÷ø , where
7 7 11. Consider the hyperbola H : x2 – y2 = 1 and a circle S with
12 center N(x2, 0). Suppose that H and S touch each other at a
å | ak +1 – ak | point P(x1, y1) with x1 > 1 and y1 > 0. The common tangent to
i= –1 . The value of the expression 3 k =1 is H and S at P intersects the x-axis at point M. If (l, m) is the
centroid of the triangle PMN, then the correct expression(s)
å | a4k –1 – a4k –2 | is(are)
k =1
6. Suppose that all the terms of an arithmetic progression (A.P.) dl 1
are natural numbers. If the ratio of the sum of the first seven (a) dx1 = 1 – 3x12 for x1 > 1
terms to the sum of the first eleven terms is 6 : 11 and the
seventh term lies in between 130 and 140, then the common
difference of this A.P. is dm x1
=
( )
(b) for x1 > 1
7. The coefficient of x9 in the expansion of (1 + x) (1 + x2) (1 + x3) dx1 3 x12 – 1
... (1 + x100) is

x2 y 2 dl 1
8. Suppose that the foci of the ellipse + = 1 are (f1, 0) (c)
9 5 dx1 = 1 + 3x 2 for x1 > 1
1
and (f2, 0) where f1 > 0 and f2 < 0. Let P1 and P2 be two
parabolas with a common vertex at (0, 0) and with foci at (f1, dm 1
(d) =
0) and (2f2, 0), respectively. Let T1 be a tangent to P1 which dy1 3 for y1 > 0
passes through (2f2, 0) and T2 be a tangent to P2 which
passes through (f1, 0). If m1 is the slope of T1 and m2 is the 12. The option(s) with the values of a and L that satisfy the
following equation is(are)
æ 1 2
ö
slope of T2, then the value of ç 2 + m2 ÷ is
è m1 ø 4p

SECTION - II
òe
t
(sin 6
at + cos 4 at dt)
0
p = L?
(sin )
This section contains 8 multiple choice questions. Each question
òe
t 6 4
has 4 choices (a), (b), (c) and (d) out of which ONE or MORE at + cos at dt
0
THAN ONE are correct.

æ 6ö æ 4ö
9. If a = 3sin–1 çè ÷ø and b = 3cos–1 çè ÷ø , where the inverse e4p - 1 e4p + 1
11 9 (a) a = 2, L = (b) a = 2, L =
p
e -1 ep + 1
trigonometric functions take only the principal values, then
the correct option(s) is (are)
(a) cosb > 0 (b) sinb < 0 e4p - 1 e4p + 1
(c) a = 4, L = (d) a = 4, L =
(c) cos(a + b) > 0 (d) cosa < 0 ep - 1 ep + 1
EBD_780
2015-16 JEE Advanced 2015 Solved Paper
13. Let f, g: [–1, 2] ® ¡ be continuous functions which are SECTION - III
twice differentiable on the interval (–1, 2). Let the values of
This section contains 2 paragraphs, each describing theory,
f and g at the points –1, 0 and 2 be as given in the following
experiments, data etc. four questions related to the two paragraphs
table:
with two questions on each paragraph. Each question has one or
more than one correct answer(s) among the four given options
x = –1 x = 0 x = 2
(a), (b), (c) and (d).
f ( x) 3 6 0
PARAGRAPH 1
g ( x) 0 1 –1
Let F : ¡ ® ¡ be a thrice differentiable function. Suppose
In each of the intervals (–1, 0) and (0, 2) the function (f – 3g)"
æ1 ö
never vanishes. Then the correct statement(s) is(are) that F(1) = 0, F(3) = –4 and F(x) < 0 for all x Î çè ,3÷ø . Let f(x)
2
(a) f ¢ ( x) – 3g ¢( x) = 0 has exactly three solutions in (–1, 0)
= xF(x) for all x Ρ .
È (0, 2)
17. The correct statement(s) is(are)
(b) f ¢ ( x) – 3g ¢( x) = 0 has exactly one solution in (–1, 0)
(a) f ¢(1) < 0
(c) f ¢ ( x) – 3g ¢( x) = 0 has exactly one solution in (0, 2) (b) f(2) < 0
(d) f ¢ ( x) – 3g ¢( x) = 0 has exactly two solutions in (–1, 0) (c) f ¢ ( x) ¹ 0 for any x Î (1, 3)
and exactly two solutions in (0, 2) (d) f ¢ ( x) = 0 for some x Î (1, 3)
æ p pö
14. Let f(x) = 7tan8x + 7tan6x – 3tan4x – 3tan2x for all x Î çè – , ÷ø . 3 2 3 3
2 2 18. If ò1 x F¢ (x)dx = –12 and ò1 x F¢¢(x)dx = 40 , then the
Then the correct expression(s) is(are) correct expression(s) is (are)
p/4 p/4 3
1
(a) ò xf ( x )dx =
12 (b) ò f ( x )dx = 0 (a) 9 f ¢ (3) + f ¢(1) – 32 = 0 (b) ò1 f ( x) dx = 12
0 0
3
p/4
1
p/4 (c) 9 f ¢ (3) – f ¢ (1) + 32 = 0 (d) ò1 f ( x ) dx = –12
(c) ò xf ( x)dx =
6 (d) ò f ( x)dx = 1
PARAGRAPH 2
0 0
Let n1 and n2 be the number of red and black balls, respectively, in
box I. Let n 3 and n 4 be the number of red and black balls,
192 x3 æ 1ö
15. Let f ¢( x ) = 4 for all x Ρ with f çè ÷ø = 0. respectively, in box II.
2 + sin px 2
19. One of the two boxes, box I and box II, was selected at
random and a ball was drawn randomly out of this box. The
1
ball was found to be red. If the probability that this red ball
If m < ò f ( x) dx £ M , then the possible values of m and M
1/ 2 1
was drawn from box II is , then the correct option(s) with
are 3
the possible values of n1, n2, n3 and n4 is(are)
1 1 (a) n1 = 3, n2 = 3, n3 = 5, n4 = 15
(a) m = 13, M = 24 (b) m = ,M=
4 2 (b) n1 = 3, n2 = 6, n3 = 10, n4 = 50
(c) m = –11, M = 0 (d) m = 1, M = 12 (c) n1 = 8, n2 = 6, n3 = 5, n4 = 20
16. Let S be the set of all non-zero real numbers a such that the (d) n1 = 6, n2 = 12, n3 = 5, n4 = 20
quadratic equation ax2 – x + a = 0 has two distinct real roots 20. A ball is drawn at random from box I and transferred to box
x1 and x2 satisfying the inequality |x1 – x2| < 1. Which of the II. If the probability of drawing a red ball from box I, after this
following intervals is(are) a subset(s) of S? 1
transfer, is , then the correct option(s) with the possible
3
æ 1 1 ö æ 1 ö
(a) çè – 2 , – ÷ (b) çè – , 0÷ values of n1 and n2 is(are)
5ø 5 ø
(a) n1 = 4 and n2 = 6
(b) n1 = 2 and n2 = 3
æ 1 ö æ 1 1ö
(c) çè 0, ÷ (d) çè , ÷ (c) n1 = 10 and n2 = 20
5ø 5 2ø
(d) n1 = 3 and n2 = 6
JEE Advanced 2015 Solved Paper 2015-17

SOLUTIONS
Paper - 1
PHYSICS fm = –10cm fl = +10cm

PA AA TA4 AA l 4B
1. (2) = = ´
PB AB TB4 AB l 4A
1 1 1 15cm
l éA P ù 4 é R 2 P ù 4 é 400 ´ 400 ù 4
\ A ê A ´ B ú =ê A ´ B ú =ê ú
l B ë AB PA û 2
êë RB PA úû ë 10 4 û
lA
\ 30cm
lB = 2 50cm
For convex lens u = |2fl|
2. (3) T No T No T No Therefore image will have a magnification of 1.
No ¾¾
® ¾¾
® ¾¾
®
2 4 8 When the set – up is kept in a medium
100% 50% 25% 12.5% The focal length of the lens will change
Three half life are required. Therefore n = 3
1 æ nl ö é1.5 ù
3. (3) For maxima ç – 1÷
Path defference = ml fl è n s ø f ¢ ê 1 –1ûú
ë
= Þ l =
\ S2A – S1A = ml 1 æ nl ö 10 é 1.5 ù
S1 ç – 1 ÷ ê – 1ú
f l¢ ç n¢ ÷ ë7 / 6 û
è s ø

d x 2 + d2 Þ fl¢ = 17.5 cm.


1 1 1
x A Applying lens formula – =
v u f l¢
1 1 1
\ – = Þ v = 140 cm.
v –20 17.5
S2 v 140
M l¢ = Magnification by lens = = = –7
é (n - 1) d 2 + x 2 + d 2 + x 2 ù – d 2 - x 2 u –20
\ êë úû = ml
M2 Mmirror ´ M l¢
\ (n – 1) (d 2 + x 2 ) = ml Now M = Mmirror ´ M = 7
1 l
æ4 ö 2 2
\ ç 3 –1÷ d + x = ml 5. (6)
cylinder
è ø
\ d 2 + x 2 = 3ml
\ d2 + x2 = 9m2l2
\ x2 = 9m2l2 – d2 line charge
\ p2 = 9 Þ p=3 +
4. (7) Applying mirror formula
q q
1 1 1
+ = 3a
v u f 2
1 1 1 1 1 a/2 a/2
= – = +
v f u –10 15
1 –15 + 10 –5 –1 a/2 1
\ = = = tanq = =
v 150 150 30 3a / 2 3
\ v = –30cm \ q = 30°
EBD_780
2015-18 JEE Advanced 2015 Solved Paper
The flux through the dotted cylinder by Gauss’s law is Given VB = VD. Therefore from (i) and (ii)
q in lL 3 3
f cylinder = = m (3)2 + mg × 30 = m (v2)2 + mg × 27
e 0 e0 4 4
\ V2 = 7
lL 9. (a, c, d)
\ For 360° angle the flux is e L µ hx cy Gz
0
Dimensionally
lL
\ For 60° angle the flux will be 6e [M 0 L1 T 0 ] = [ML2T –1 ]x [LT –1 ]y [M –1L3T –2 ]z
0
Therefore n = 6 M 0 L1 T 0 = Mx – z L2x + y + 3z T–x – y – 2z
hc 13.6 \ x–z=0 Þ x=z
6. (2) = 2 + 10.2 \ 2x + y + 3z = 1 and –x – y – 2z = 0
l n On solving we get
1242 13.6 1 3 1
\ = 2 + 10.2 x= ,y= – ,z=
90 n 2 2 2
\ n2 = 4 \ Lµ h
\ n=2
Lµ G
7. (2) Let h be the height to which the bullet rises
C, D are correct options
–2 M µ hxcyGz
æ hö
then, g1 = g ç 1 + ÷ 2 –1 x –1 y –1 3 –2 Z
è R ø M ¢L °T ° µ [ML T ] [LT ] [M L T ]
\ x – z 2x + y + 3z T – y –2z
– x
M ¢L °T ° µ M L
–2
g æ hö \ x–z=1
Þ = g ç1 + ÷ 2x + y + 3z = 0
4 è R ø
–x – y – 2z = 0
Þ h=R
On solving we get
2GM 1 1 1
We know that ve = = v N (given) ...(i) x= ,y= ,z=–
R 2 2 2
Now applying conservation of energy for the throw \ Mµ C
Loss of kinetic energy = Gain in gravitaional potential energy
A is the correct option.
1 2 GMm æ GMm ö 10. (b, d)
\ –ç–
R ÷ø
mv = – Maximum linear momentum in case 1 is (p1)max = mnmax
2 2R è
b = m [aw1] ...(i)
GM Maximum linear momentum in case 2 is (p2)max = mnmax
\ v= ...(ii) R = m [Rw2]
R
Comparing (i) & (ii) N=2 \ 1 = mw2 ...(ii)

1 2 1 2 b aw1
Dividing (i) & (ii) =
8. (7) Total kinetic energy of a rolling disc = mv + Iw 1 w2
2 2
w1 b 1
1 æ 1 mR 2 ö æç v ö
2
1 ÷ \ w =a= 2 \ B is a correct option.
= mv2 + ç ÷ç 2 ÷ 2 n
2 2 è2 øè R ø
1
3 Also E1 = m w12 a 2
K.E = mv2 2
4
1
For surface AB E2 = m w22 R 2
k.Ei + loss in gravitational potential energy = K.Ef 2
2 2 2 2
3 3 E1 w1 a w1 1 w1 w2 w1
m (3)2 + mg(30) = m VB2 ...(i) \ E = 2´ 2 = 2´ = 2´ =
4 4 2 w2 R w2 n w2 w1 w2
For surface CD
E1 E 2
3 3 \ w = w D is the correct option
m (v2)2 + mg (27) = m VD2 ...(ii) 1 2
4 4
JEE Advanced 2015 Solved Paper 2015-19

11. (d) Applying conservation of angular mumentum about 1 1


the axis \ =
d - 50 20
8w M 9R2 8w M 2 8w \ d = 70 cm.
MR2 × w = MR2 × + × × + r ×
9 8 25 9 8 9 B is the correct option.
14. (a, b, c)
4R ur uur ur
Þ r=
5 ( )
F = I éë ò dl ´ B ùû
D is the correct option ur r ur
12. (c) Force on change q when it is given a small displacement If B is along z then F = I éë(2 L + 2R )$i ´ B x$ ùû
x is Fnet = F1 – F2 option [A] is correct
l l
ur r ur
1 l 1 l If B is along x then F = I éë(2L + 2R )i$ ´ Bi$ùû = 0
Fnet = 2pe d – x – 2pe d + x
0 0 ur ur
ur
F1 +q F2 If B is along y then F = I éë(2L + 2R)$i ´ ˆj ùû
l éd + x – d + x ù x
\ Fnet = 2
pe0 êë d 2 - x 2 úû x d–x
Option (b) and (c) are also correct
15. (a, b, d)
l 2x d
3 5
\ Fnet = 2pe 2 2 Total energy = RT + RT = 4RT
0 d –x 2 2
2d
line charge 1 line charge 2
When x << d then 4RT
\ Average energy per mole = = 2RT
l 2
Fnet = pe x and is directed towards the mean position
0 gRT
therefore the charge +q will execute SHM. We know that Vsound =
M
l l
n1 + n2 n n
= 1 + 2
g mi x –1 g1 - 1 g 2 –1

2 1 1
–q Þ = +
F2 g mix - 1 5 7
F1 -1 -1
3 5
2 3 5
= +
g mix - 1 2 2 =4

1 3
\ g mix - 1 = \ g mix =
line charge 1 line charge 2 2 2
In case of charge (–q) (Vs )mix g mix M He
F2 > F1 therefore the charge –q continues to move in the = ´
(Vs ) He M mix g He
direction of its displacement.
[C] is the correct option.
3
13. (b) For refraction in S1 ´4
2 é 1´ 2 + 1´ 4 ù
= êQ M mix = = 3ú
n n n – n1 5 ë 2 û
– 1+ 2 = 2 3´
u v R 3
1.5 1 1 - 1.5 6
– + = =
–50 V –10 5
Þ v = 50 cm.
For refraction in S2 3RT
We know that Vrms =
M
n1 n2 n2 – n1
– + =
u v R (Vrms ) He M H2 2 1
\ (V ) = M HE
=
4
=
2
1 1.5 1.5 –1 rms H 2
– + =
–(d – 50) µ 10 \ options [A], [B] and [C] are correct.
EBD_780
2015-20 JEE Advanced 2015 Solved Paper
Now if the least count of the linear scale of the screw gauge
rFe ´ lFe 10–7 ´ 50 ´10 –3 25
16. (b) RFe = = = ´10–4 is twice the least count of venier callipers then.
AFe 4 ´ 10 –6 2
L.C of linear scale of screw gauge = 2 × 0.025 = 0.05cm.
Then pitch = 2 × 0.05 = 0.1cm.
r Al ´ l Al 2.7 ´ 10 –8 ´ 50 ´ 10 –3 2.7 ´ 50
RAl = = = × 10–5
AAl (49 – 4) ´ 10 –6 45 0.1
Then L.C of screw gauge = cm = 0.001cm = 0.01mm.
100
= 0.3 × 10–4
(c) is a correct option.
–4 –4 19. A ® R, T ; B ® P, S ; C ® P, Q, R, T; D ® P, Q, R, T
RFe ´ RAl 12.5 ´10 ´ 0.3 ´ 10
Rtotal = = » 29µW
RFe + RAl 12.8 ´10 –4 Based on facts
20. A ® P, Q, R, T; B ® Q, S; C ® P, Q, R, S; D ® P, R, T
(B) is the correct option.
For A
17. (a, c)
2
We know that
hC
– W = eV0 Þ
hc W
– = V0 dU – d éU 0 æ æ x ö2 ù
l el e Fx = – dx = dx ê 2 ç1 - ç ÷ ú
ç èaø ú
êë è û
1
For V0 versus we should get a straight line with negative
l -2U 0
= ( x - a) x ( x + a)
slope and positive intercept. a3
For V0 vesus l, we will get a hyperbola. As l decreases V0
increases.
(a) and (c) are the correct options U U
18. (b, c) U0
Vernier callipers 2

1cm
1 MSD = = 0.125cm
8
–a +a x x
5 VSD = 4MSD U U
1
\ 5VSD = 4 × cm = 0.5cm
8
\ 1 VSD = 0.1cm
L.C = 1MSD – 1VSD
+a x –a +a x
= 0.125cm – 0.1cm
= 0.025cm
Screw gauge
One complete revolution = 2M.S.D
If the pitch of screw gauge is twice the L.C of vernier callipers
then pitch = 2 × 0.025 = 0.05cm. – dU æ xö
For B Fx = = -U 0 ç ÷
L.C of screw Gauge dx èaø
pitch
e- x / a
2 2
= Total no. of divisions of circular scale – dU
For C Fx = = U0 x( x - a)( x + a)
dx a3
0.05
= cm = 0.0005cm = 0.005 mm. dU U
100 For D Fx = - = - 03 [( x - a)( x + a)]
dx 2a
(b) is a correct option
JEE Advanced 2015 Solved Paper 2015-21

CHEMISTRY 6. (9)

1. (4) Cl – Be – Cl N 3–
Hybridization sp Hybridisation sp
Structure linear Structure linear

+
O= N®O
Hybridisation sp Hybridisation sp 2
Structure Linear Structure Trigonal planar

S
II III IV
Cl Cl
Hybridisation sp 3 Hybridisation sp 3 d
Structure Angular Structure linear

I I F Xe F
Hybridisation sp 3 d Hybridisation sp 3 d
Structure Linear Structure Linear
– +
N= N=O
Hybridisation sp
Structure Linear

Only BeCl 2 , N 3– , N 2 O and NO 2 are linear with


sp-hybridisation.
2. (3) Ground state configuration:

7. (8)
1s 2s 2p
in second excited state, electron will jump from 1s to 2p, so
degeneracy of second excited state of H– is 3.
Number of lone pairs = 8
3. (4) X ¾¾
® Y; DG° = – 193 kJ mol–1
8. (4) Fe(26) ¾¾ 6 2
® [Ar]18 3d 4s
M+ ¾¾ ® M3+ + 2e– E° = – 0.25V
Hence DG° for oxidation will be Fe3+ ¾¾ ® [Ar]18 3d 4s
5 0
DG° = – nFE°
= –2 × 96500 × (–0.25) SCN– is weak field ligand hence pairing will not occur.
= 48250 J = 48.25 kJ \ Fe3+ ¾¾
®
48.25 kJ energy oxidises one mole M+
Unpaired electrons = 5
193
\ 193 kJ energy oxidises mole M+ = 4 mole M+
48.25 Magnetic moment = 5(5 + 2) B.M.
4. (1) Given DTf = 0.0558°C
= 35 B.M. = 5.92 B.M.
as we know, DTf = i × Kf × m
Þ 0.0558 = i × 1.86 × 0.01 CN– is strong field ligand hence pairing will take place.

i =3 \ Fe3+ ¾¾
®
Therefore the complex will be [Co(NH3)5Cl]Cl2 Unpaired electrons = 1
Hence number of chloride in co-ordination sphere is 1.
5. (2) The molecule cannot show geometrical isomerism, so Magnetic moment = 1(1 + 2)B.M. = 3 B.M. = 1.732
only its mirror image will be the other stereoisomer. Difference = 5.92 – 1.732 = 4.188
Hence answer is (4).
EBD_780
2015-22 JEE Advanced 2015 Solved Paper

9. (a)

KOH, H2O

(Minor) (Major)

10. (d)

Å Å

(Major)

11. (a) H OH

D-(+)- glucose L-(–)- glucose

12. (c)

–+
ClN 2

(Retention Product)
(Major)

13. (a, b, c) 16. (b) Initially on increasing temperature rate of reaction will
Cr2+ is a reducing agent and Mn3+ is an oxidizing agent and increase, so % yield will also increase with time. But at
both have electronic configuration d4. equilibrium % yield at high temperature (T2) would be
less than at T1 as reaction is exothermic so the graph is
E° = –0.41V E° = 1.51V
Cr3+ / Cr 2 + Mn 3+ / Mn 2 +
Above E° values explains reducing nature of Cr 2+ and T1
oxidizing behaviour of Mn3+.
14. (b, c, d) T2
%yield

In electrolytic refining of blister Cu, acidified CuSO4 is used


as electrolyte, pure Cu deposits at cathode and impurities
settle as anode mud.
time
15. (c, d)
Fe3+ is reduced to Fe2+ by H2O2 and Na2O2 in acidic medium.
JEE Advanced 2015 Solved Paper 2015-23

17. (a) In ccp, O2– ions are 4. Hence, w = 0


Hence total negative charge = –8 and qp = CpdT (Q dT = 0)
Let Al3+ ions be x, and Mg2+ ions be y. Þq=0
Total positive charge = 3x + 2y DU = CvdT (Q dT = 0)
Þ 3x + 2y = 8 DU = 0
This relation is satisfied only by x = 2 and y = 1. (C) ® P, Q, S
Hence number of Al3+ = 2. Mixing of two ideal gases at constant temperature
and number of Mg2+ = 1. Hence, DT = 0
Þ n = fraction of octahedral holes occupied by Al 3+ \q=0
2 1 DU = 0
= = also w = 0 (DU = q + w)
4 2
(D) ® P, Q, S, T
and m = fraction of tetrahedral holes occupied by Mg2+
Reversible heating and cooling of gas follows same path
also initial and final position is same.
1 q=0ü
=
8 Hence, ý Path same
w = 0þ
18. (b, d)
DU = 0 ü
ý State function
H2/Ni D G = 0þ
(A)
*
MATHEMATICS
H Br H Br
H2/Ni 1. (2) End points of latus rectum of y2 = 4x are (1, +2)
(B) Equation of normal to y2 = 4x at (1, 2) is
(Optically inactive)
y – 2 = –1(x – 1) or x + y –3 = 0
As it is tangent to circle (x – 3)2 + (y + 2)2 = r2
(C) H2C H2/Ni H3C
CH3 * CH3 3 + (–2) – 3
\ = r Þ r2 = 2
CH3 CH3 2

2
xf ( x 2 )
H 2C CH3
H2/Ni
CH3 2. (0) I = ò 2 + f ( x + 1)
dx
(D) H 3C –1

–1 < x < 2 Þ 0 < x2 < 4


19. A-(P, Q, S), B-(T), C-(Q, R), D-(R)
Also 0 < x2 < 1 Þ f(x2) = [x2] = 0
ì Siderite ® FeCO 3 1 < x2 < 2 Þ f(x2) = [x2] = 1
ï Malachite 2 < x2 < 3 Þ f(x2) = 0 (using definition of f)
® CuCO3 . Cu(OH) 2
ï 3 < x2 < 4 Þ f(x2) = 0 (using definition of f)
ï
ï Also 1 < x2 < 2 Þ 1 < x < 2
í Bauxite ® Al2 O3 .xH2 O or
ï AlOx (OH)3 - 2x 0 < x < 1 Þ 2<x+1< 2 +1
ï
ï Calamine ® ZnCO3 Þ f(x + 1) = 0
ï ®
î Argentite Ag 2S
2 2
x ´1 é x2 ù 2 1 1
20. A- (R, T), B -(P, Q, S), C -(P, Q, S), D- (P, Q, S, T) \ I= ò 2+0
dx = ê ú = – =
(A) ® R, T 1 ëê 4 ûú 1
4 4 4

ˆˆ† H O(s) Þ 4I = 1 or 4I – 1 = 0
H2O( l ) ‡ˆˆ 2 3. (4) Let r be the internal radius and R be the external radius.
It is at equilibrium at 273 K and 1 atm
Let h be the internal height of the cylinder.
So DSsys is negative
As it is equilibrium process so DG = 0 V
(B) ® P, Q, S \ pr2h = V Þ h =
pr 2
Expansion of 1 mole of an ideal gas in vacuum under isolated
Also Vol. of material = M = p[(r + 2)2 – r2]h + p(r + 2)2 × 2
condition
EBD_780
2015-24 JEE Advanced 2015 Solved Paper
6. (4) Let (t2, 2t) be any point on y2 = 4x. Let (h, k) be the
V
or M = 4p(r + 1). + 2p (r + 2)2 image of (t2, 2t) in the line x + y + 4 = 0. Then
pr 2
h - t 2 k - 2t –2(t 2 + 2t + 4)
= =
é1 1 ù 1 1 2
Þ M = 4V ê r + 2 ú + 2p(r + 2)2
ë r û Þ h = –(2t + 4) and k = –(t2 + 4)
For its intersection with, y = –5, we have
dM é –1 2 ù –(t2 + 4) = –5 Þ t = +1
= 4V ê 2 – 3 ú + 4p(r + 2)
dr ër r û \ A(–6, –5) and B(–2, –5)
\ AB = 4.
dM 7. (8) P(x > 2) > 0.96
For min. value of M, =0
dr Þ 1 – P(x = 0) – P(x = 1) > 0.96
Þ P(x = 0) + P(x = 1) < 0.04
–4V
Þ (r + 2) + 4p(r + 2) = 0 n n
r3 æ 1ö æ 1ö
Þ çè ÷ø + n çè ÷ø < 0.04
2 2
4V V
Þ 3 = 4p or r3 = = 1000 n +1 1
r p Þ £
n 25
\ V = 1000p 2

V 2n
\ =4 Þ > 25
250p n +1
Þ minimum value of n is 8.
x 2 +p / 6
4. (3) F(x) = ò 2 cos2 t dt 8. (5) n = 5! × 6!
x
For second arrangement,
æ 2 pö 5 boys can be made to stand in a row in 5! ways, creating 6
F ¢ (a ) = 2cos2 çè a + ÷ø .2a – 2cos2a alternate space for girls. A group of 4 girls can be selected in
6 5C ways. A group of 4 and single girl can be arranged at 2
4
a places out of 6 in 6 P2 ways. Also 4 girls can arrange
F ¢ (a ) + 2 = ò0 f ( x )dx themselves in 4! ways.
\ m = 5! × 6P2 × 5C4 × 4!
Þ F ¢¢(a) = f(a)
m 5!´ 6 ´ 5 ´ 5 ´ 4!
= =5
æ 2 pö é æ 2 pö ù n 5!´ 6!
\ f(a) = 4a.2cos çè a + 6 ÷ø . ê – sin çè a + 6 ÷ø ú .2a 9. (b, c) R2 – R1 , R3 – R2
ë û

æ pö (1 + a )2 (1 + 2a ) 2 (1 + 3a ) 2
2
+4 cos2 ç aè
+ ÷ – 4cosa (–sina) 2a + 3 4a + 3 6a + 3 = –648a

2a + 5 4a + 5 6a + 5
p 3
\ f(0) = 4cos2 =4× =3 R3 – R2
6 4

5 (1 + a )2 (1 + 2a )2 (1 + 3a )2
5. (8) cos22x + cos4x + sin4x + cos6x + sin6x = 2 2 2a + 3 4a + 3 6a + 3 = –648a
4
1 1 1
5 1 3
Þ cos22x + 1 – sin 22x + 1 – sin 22x = 2 C2 – C1, C3 – C2
4 2 4

5 (1 + a)2 a (3a + 2) a (5a + 2)


Þ (cos22x – sin22x) = 0 Þ cos4x = 0 2a + 3 2a 2a = –324a
4
1 0 0
p p
Þ 4x = (2n + 1) or x = (2n + 1) Þ 2a2(–2a) = –324a
2 8
Þ a3 – 81a = 0
For xÎ[0, 2p], n can take values 0 to 7
Þ a = 0, 9, –9
\ 8 solutions.
JEE Advanced 2015 Solved Paper 2015-25

10. (b, d) P3 : x + ly + z – 1 = 0
æ a2 ö
l -1 ç ,a ÷
ç 2 ÷P
Also = 1 Þ l2 – 2l + 1 = l2 + 2 è ø
2 + l2

–1
Þ l=
2
O
1
a + lb + g - 1 a – b + g –1
And =2 Þ 2 =+2 Q
2 + l2 3 æ b2 ö
ç ,b÷
2 ç 2 ÷
è ø

1 Q PQ is the diameter of circle through P, O, Q


a – b + g –1 = ±3
2
a b
Þ 2a – b + 2g – 2 = +6 \ ÐPOQ = 90° Þ ´ = –1 Þ ab = –4
2 2
Þ 2a – b + 2g – 8 = 0 or 2a – b + 2g + 4 = 0 a /2 b /2
x y z Þ b is negative.
11. (a, b) L : = = =l
a b c Also ar. DPOQ = 3 2

Where a + 2b – c = 0 ì As L is parallel 0 0 1
í
2a - b + c = 0 îto both P1 and P2 . 2
1 a
Þ a 1 =3 2
a b c 2 2
Þ = =
1 –3 –5 b2
b 1
\ Any point on line L is (l, –3l, –5l) 2
Equation of line perpendicular to P1 drawn from any point
on L is 1
Þ ab (a – b) = + 3 2
x - l y + 3l z + 5l 4
= = =µ
1 2 –1 Þ a–b=+ 3 2 (using ab = –4)
\ M(µ + l, 2µ – 3l, –µ – 5l)
But M lies on P1, As a is positive and b is negative, we have a – b = 3 2
\ µ + l + 4µ – 6l + µ + 5l + 1 = 0
4
a+ = (using ab = –4)
Þ µ=
–1 a 3 2
6
Þ a2 – 3 2 a + 4 = 0
æ 1 –1 1ö
\ M çè l – , –3l , –5l + ÷ø Þ a2 – 2 2 a – 2a+4=0
6 3 6
For locus of M, Þ (a – 2 2 ) (a – 2)=0
1 1 1 Þ a = 2 2, 2
x=l– , y = –3l – , z = 5l +
6 3 6
æ
( ö
)
æ
( 2) ö
2 2
x + 1/ 6 y + 1/ 3 z - 1/ 6 2 2
= = ç ,2 2 ÷ ç , 2÷
Þ =l \ Point P can be ç 2 ÷ or ç ÷
1 –3 –5 2
è ø è ø
æ –5 –2 ö
On checking the given point, we find çè 0, , ÷ø and i.e. (4, 2 2 ) or (1,
6 3 2)

æ –1 –1 1 ö dy ex 1
çè , , ÷ø satisfy the above eqn. 13. (a, c) + y=
6 3 6 dx 1 + e x 1 + ex
12. (a, d) Let point P in first quadrant, lying on parabola y2 = 2x I.F. = 1 + ex
æ a2 ö æ b2 ö \ Soln : y(1 + ex) = x + c
ç
be 2 , a ÷ . Let Q be the point ç 2 , b÷ . Clearly a > 0. y(0) = 2 Þ c = 4
è ø è ø
EBD_780
2015-26 JEE Advanced 2015 Solved Paper

x+4 ì – g ¢( x ) , x < 0
\ y= x ï
e +1 0 , x=0
f ' (x) = í
\ y(–4) = 0 ï g ¢ ( x) , x > 0
î
dy (e x + 1) – e x ( x + 4) \ Lf ' (0) = –g'(0) = 0
Also =
dx (e x + 1) 2 Rf ' (0) = g'(0) = 0
\ f is differentiable at x = 0
dy
For critical point =0
ïìe
–x
dx , x<0
h(x) = e|x| = í x
Þ e (x + 3) = 1
x
ïî e , x³0
Þ x + 3 = e–x
Its solution will be intersection point of y = x + 3 and y = e–x
ïì – e
–x
, x<0
h'(x) = í x Þ Lh'(0) = –1, Rh'(0) = 1
ïî e , x³0
3 \ h is not differentiable at x = 0
x+
y= f oh(x) = f(h(x)) = g(e|x|) as e|x| > 0

y = e–x ì g (e – x ) if x<0
ï
g (1) if x=0
(0, 1) = íï
x
î g (e ) if x>0
–1
ì – g ¢(e – x ).e – x , x<0
ï
f ' [h(x)] = í 0 , x=0
Clearly there is a critical point in (–1, 0).
ï x x
14. (b, c) Let the equation of circle be î g ¢(e )e , x>0
x2 + y2 + 2gx + 2gy + c = 0
Þ 2x + 2yy' + 2g + 2gy' = 0 Lf ' (h(0)) = –g ' (1)
Þ x + yy' + g + gy' = 0 ...(i) Rf ' (h(0)) = g ' (1)
Differentiating again, Q g ' (1) ¹ 0, \ Lf ' (h(0)) ¹ Rf ' (h(0))
1 + yy" + (y')2 + gy" = 0 \ foh is not differentiable at x = 0.

é1 + ( y ¢)2 + yy ¢¢ ù ìïe f ( x) , x¹0


Þ g=–ê ú hof(x) = í
y ¢¢ ïî 1 , x =0
ëê ûú
Substituting value of g in eqn. (i) h( f (0)) – h( f (0 – k ))
Lh'(f(0)) = lim
k ®0 k
1 + ( y ¢ )2 + yy ¢¢ æ 1 + ( y ¢ )2 + yy ¢¢ ö
x + yy' – –ç ÷ y' = 0
y ¢¢ è y ¢¢ ø g (– k ) 1- e
g (– k )
g (– k )
1- e ´
= lim = lim
Þ xy" + yy'y" – 1– (y')2 – yy" – y' – (y')3 – yy¢y¢¢ = 0 k ®0 k k ®0 g (– k ) k
Þ (x – y)y" – y'(1 + y' + (y')2) = 1
æ g (–k ) g (k ) ö
or (y – x)y" + [1 + y' + (y')2] y' + 1 = 0 = 1 × 0 = 0 çQ g ¢(0) = 0 Þ lim = lim = 0÷
è k ®0 k k ®0 k ø
Py" + Qy' + 1 = 0
\ P = y – x, Q = 1 + y' + (y')2 h( f (0 + k )) – h ( f (0))
Also P + Q = 1 – x + y + y' + (y')2 Rh'(f(0)) = lim
k ®0 k
ìx
ï g ( x ), x ¹ 0 e
g (k )
–1 e
g (k )
–1 g (k )
15. (a, d) f(x) = í x = lim = lim ´ =0
ï0 k ®0 k k ®0 g (k ) k
î ,x = 0
\ hof is differentiable at x = 0.
ì – g ( x) , x < 0 16. (a, b, c)
ï
0 , x=0 æp æp öö
= í f(x) = sin ç sin ç sin x ÷ ÷
ï g ( x) , x > 0
î è6 è2 øø
JEE Advanced 2015 Solved Paper 2015-27

p p p r2 r2 rr r2
–1< sin x <1 Þ – £ sin x £ Þ b + c + 2b.c = a
2 2 2
r2 r2 r
æp ö Þ 48 + c + 48 = 144 Þ c = 48 Þ c = 4 3
Þ –1 < sin ç sin x ÷ < 1
è2 ø
r2
–p p æp ö p c r 48
Þ < sin ç sin x ÷ < \ –a = –12 = 12
6 6 è 2 ø 6 2 2

ép æp öù 1 r2
–1 c r
Þ < sin ê sin ç sin x ÷ ú £
2 ë 6 è 2 øû 2 + a = 24 ¹ 30
2
é –1 1 ù r r
\ Range of f = ê , ú Also b = c Þ ÐQ = ÐR
ë 2 2û
r r
ép æ p æ p b.c 1
ö öù and cos(180 – P) = r r =
fog(x) = sin ê 6 sin ç 2 sin ç 2 sin x ÷ ÷ ú b c 2
ë è è ø øû
Þ ÐP = 120° \ ÐQ = ÐR = 30°
é –1 1 ù r r r r r r r r r r r r
Range of fog = ê , ú Again a + b + c = 0 Þ a ´ (a + b + c) = 0 Þ a ´ b = c ´ a
ë 2 2û
r r r r r r
\ a ´ b + c ´ a = 2 a ´ b = 2 × 12 × 4 3 × sin150 = 48 3
æp æp öö
sin ç sin ç sin x ÷ ÷ rr
è 6 è 2 øø And a.b = 12 ´ 4 3 ´ cos150 = –72
lim
x ®0 p 18. (c, d)
sin x
2 X ' = –X, Y ' = –Y, Z ' = Z
(Y 3Z 4 – Z 4Y 3)' = (Z 4)'(Y 3)' – (Y 3)'(Z 4)'
æp æp öö p æ p ö = (Z ')4(Y ')3 – (Y ')3(Z ')4
sin ç sin ç sin x ÷ ÷ sin ç sin x ÷
è 6 è 2 øø 6 è2 ø = –Z 4Y 3 + Y 3Z 4 = Y 3Z 4 – Z 4Y 3
= lim ´ \ Symmetric matrix.
x ®0 p æp ö p
sin ç sin x ÷ sin x Similarly X 44 + Y 44 is symmetric matrix and X 4Z 3 – Z 3X 4
6 è2 ø 2
and X 23 + Y 23 are skew symmetric matrices.
= p/6 19. [A ® Q; B ® P, Q; C ® P, Q, S, T; D ® Q, T]

p æ æp æp ööö 3a + b 2 3 -b
gof(x) = sin ç sin ç 6 sin ç 2 sin x ÷ ÷ ÷ (A) = 3 Þa=
2 è è è øøø 2 3

p æ1ö p æ1ö 2 3 -b
– sin ç ÷ £ g ( f ( x)) £ sin ç ÷ \ = 2 + 3b Þ b = 0 Þ a = 2
2 è2ø 2 è2ø 3
–0.73 < g(f (x)) < 0.73 (B) Lf '(1) = –6a and Rf '(1) = b
\ gof(x) ¹ 1 for any x Î R. –6a = b ...(i)
17. (a, c, d) Also f is continuous at x = 1,
r r r r \ –3a – 2 = b + a2
a +b+c =o Þ a2 – 3a + 2 = 0 (using (i))
Þ a = 1, 2
P (C) (3 – 3w + 2w2 ) 4n + 3 + (2 + 3w – 3w2 ) 4n + 3

+ (–3 + 2w + 3w2)4n + 3 = 0
c b 4n +3
æ 2w2 + 3 - 3w ö
Þ (3 – 3w + 2w2 ) 4n + 3 + ç ÷
ç w2 ÷
è ø
Q R
a 4n + 3
æ –3w + 2w 2 + 3 ö
+ç ÷ =0
r r2 r2 è w ø
Þ b + c = –a
Þ (3 – 3w + 2w2)4n + 3 [1 + w4n + 3 + (w2)4n + 3] = 0
EBD_780
2015-28 JEE Advanced 2015 Solved Paper
Þ 4n + 3 should be an integer other than multiple of 3. (D) For a = 0, y = 3
\ n = 1, 2, 4, 5 For a = 1, y = |x – 1| + |x – 2| + x
Case I
2ab
(D) = 4 Þ ab = 2a + 2b ...(i) F(a) is the area bounded by x = 0, x = 2, y2 = 4x and y = 3
a+b
Also a + q = 10 or a = 10 – q 2

and b + 5 = 2q or b = 2q – 5
\ F(a) = ò0 (3 – 2 x ) dx

Putting values of a and b in eqn(i)


y
15 5
q = 4 or Þ a = 6 or y=3
2 2
\ |q – a| = 2 or 5. x
20. [A®P, R, S; B ® P; C ® P, Q; D®S, T]
(A) 2(a2 – b2) = c2
Þ 2(sin 2x – sin2y) = sin2z
Þ 2sin(x + y) sin(x – y) = sin2z x=2
Þ 2sin(x – y) = sin z (Q sin(x + y) = sin z)
2
sin( x – y ) 1 4x x 8 2
Þ = =l = 3x – =6–
sin z 2 3 3
0
np
\ cos(npl) = 0 Þ cos = 0 Þ n = 1, 3, 5 8
2 \ F(a) + 2 =6
3
(B) 1 + cos2X – 2cos2Y = 2sinXsinY
Þ 2cos2X – 2cos2Y = 2sinXsinY Case II
Þ 1 – sin2X – 1 + 2sin 2Y = sinXsinY F(a) is the area bounded by x = 0, x = 2, y2 = 4x and
Þ sin2X + sinXsinY – 2sin2Y = 0 y = |x – 1| + |x – 2| + x
Þ (sinX – sinY) (sinX + 2sinY) = 0 ì3 – x, 0 £ x < 1

sin X î x + 1,1 £ x £ 2
Þ = 1 or –2
sin Y
1 2
\ F(a) = ò (3 - x - 2 x )dx + ò ( x + 1 – 2 x )dx
a 0 1
\ = 1.
b
y
(C) X( 3 , 1), Y(1, 3 ), Z(b, 1 – b)

y
Y(1, 3) x
1
x
y=
X( 3,1)
x=2
60°
1 2
30° æ x2 4x ö æ x2 4 ö
= ç 3x – – x÷ +ç +x– x x÷
O x ç 2 3 ÷ ç ÷
è ø0 è 2 3 ø1
By symmetry, acute angle bisector of ÐXOY is y = x.
1 4 8 2 1 4 8 2
\ Distance of Z from bisector = 3- - + 2+ 2 – – –1 + = 5 –
2 3 3 2 3 3
b -1+ b 3
= = Þ 2b – 1 = + 3 or b = 2 or –1 8 2
2 2 F(a) + =5
3
\ b = 1, 2
JEE Advanced 2015 Solved Paper 2015-29

Paper - 2
PHYSICS 3´ 6
Req = = 2W
3+ 6

1. (2) R = –
dA
=– – = =
2
d é dN ù d 2 N d N o e
–l t
( )
dt dt êë dt úû dt 2 dt 2
2 –lt –lt
\ R = No l e = (Nol) l e = Aole –lt 2W 1W
Balanced
[Q Ao = Nol] 2W
I wheatstone
8W
2t 6W 2W bridge
– l Pt lQ t
RP l P e lP e 2t 2e 2t 6.5V 4W
\ R = – lQ t
=
l Q
´
l Pt
=
t 2t
=
e
Q lQ e e 10W
et
12W 4W
\ n=2
2. (2) Here ÐMPQ + ÐMQP = 60°. If ÐMPQ = r then ÐMQP
= 60 – r
Applying Snell’s law at P The equivalent resistance of balanced wheat stone
sin60° = n sin r ...(i) bridge is
Differentiating w.r.t ‘n’ we get 6 ´ 18 9
Req= = W
dr 24 2
O = sin r + n cos r × ...(ii)
dn 6.5
\ I= = 1A
2 + 4.5

60° 2W Balanced
wheat stone
P Q I bridge
60° q 6W 2W
r 60°–rr 6.5V 10W
M
12W 4W

Applying Snell’s law at Q


sin q = n sin (60° – r) ...(iii)

Differentiating the above equation w.r.t ‘n’ we get 3h


4. (2) Given mvr = Þn=3
dq é dr ù 2p
cos q
dn
= sin (60° – r) + n cos (60° – r) êë – dn úû
hr 3h é h ù
\
l
=
2p êQ l = mv ú
dq é tan r ù ë û
\cos q = sin (60° – r) – n cos (60° – r) ê –
dn ë n úû 2pr 2 é n2 ù é n2 ù
[from (ii)] \ l = 3 = 3 p ê a0 z ú êQ r = a0 ú
z úû
êë úû êë
dq 1
\ = [sin (60° – r) + cos (60° – r) tan r] ...(iv) 2 é3 ´ 3ù
dn cos q p a0 ê
\
3
l= ú = 2pa0
ë 3 û
From eq. (i), substituting n = 3 we get r = 30° \ p=2
From eq (iii), substituting n = 3 , r = 30° we get q = 5. (7) For the tension in the rod to be zero, the force on both
the masses m and m should be equal in magnitude and
60° direction. Therefore
On substituting the values of r and q in eq (iv) we get
M
dq 1
= m m
dn cos 60° [sin 30° + cos 30° tan 30°] = 2
3. (1) The equivalent resistance of balanced wheatstone
bridge is 3l l
EBD_780
2015-30 JEE Advanced 2015 Solved Paper

GMm Gmm GMm Gmm IB 6


+ = – \ = Þn=6
(4l)2 l2 (3l) 2 l2 I A 10

é1 1 ù é p 2p ù
\ 2m = M ê – ú 8. (3) y = I 0 êsin O + sin + sin + sin pú
ë 9 16 û ë 3 3 û

7M é 3 3ù
\ m= y = I0 ê + ú = 3 I0
288 ë 2 2 û
K=7 \ Ir = y2 = 3I0 Þ n=3
6. (4) E = A2 e–0.2t 9. (a, c)We know that
\ loge E = 2 loge A –0.2t
On differentiating we get 1 m0
C= and R =
m0e 0 e0
dE dA dt
=2 – 0.2 ´ t Now, m0 I2 = e0 V2
E A t
As errors always add up therefore m0 V 2
\ = = R2 Þ Option A is correct
dE æ dA ö æ dt ö e0 I 2
´ 100 = 2 ç ´ 100 ÷ + 0.2t ç ´ 100 ÷
E è A ø è t ø Now, e0 I = m0 V
m0 I 1
dE \ = = Þ Option B is incorrect
\ × 100 = 2 × 1.25% + 0.2 × 5 × 1.5% e0 V R
E
Now, I = e0C V
dE
\ × 100 = 4% 1 V
E \ = =R
e0 C I
R
7. (6) I = ò (dm)r 2 1
\ =R
0 1
e0
m0 e0
R
m0
\ =R Þ Option C is correct
e0
r dr
Now, mo C I = e0 V
m0 V R m0 1
\ = = = ´ = µ0
e0 I C C e0 1
m 0 e0
R
Þ Option (d) is incorrect
\ I = ò r ´ 4 pr 2 dr ´ r 2
10. (d) Assume the cavity to contain similar charge distribution
0
of positive and negative charge as the rest of sphere.
R Electric field at M due to uniformly distributed charge
\ I = 4 p ò r r 4 dr of the whole sphere of radius R1
0
R R M
r 4 pK
\ I A = 4p ò k ´ r 4 dr = òr
5
dr P
r
R R
0 0
a
4pK æ R6 ö R5 O
= ç ÷ = 4pK
R ç 6 ÷ 6
è ø
R 5
ærö
I B = 4p ò K ç ÷ r 4 dr
0
èRø
ur r r
4pK R10 R5 E= r
= ´ = 4pK 3e
R5 10 10
JEE Advanced 2015 Solved Paper 2015-31

Electric field at M due the negative charge distribution P r


in the cavity GM r
\ – ò dp = ò rdr
ur r uuur 0 R3 R
E2 = MP
3e GM r é 2 2 ù
\ The total electric field at M is \ P= R -r
ur ur ur 2 R3 ë û
r r r uuur
E = E1 + E 2 = r+ MP
3e 3e é 2 9R 2 ù 2
êR – ú 7R
ur r r r r r r uuur r P (r = 3R/ 4) ë ê 16 ú
\ E= r+
3e 3e
( ë )
a – r éQ r + MP = a ù
û \ = û = 16 = 63
P (r = 2 R/ 3) é 2 4 R 2 ù 5R 2 80
ur r r êR – ú
êë 9 úû 9
\ E= a
3e
(d) is the correct option é 2 9 R2 ù
11. (a, b) The maximum stress that P can withstand before êR – ú
breaking is greater than Q. Therefore (A) is a correct P (r = 3R/ 5) êë 25 úû 16
and = =
option. P (r = 2 R/ 5) é 2 4R 2 ù 21
êR – ú
Strain êë 25 úû
B and C are correct options.
Max 13. (d)
(strain P)
d/2

Max
(strain Q)
C1 C2
e1 = 2 e2 = 4
s/2

Max + C3
stress e1 = 2
Stress of P
Maximum s/2
stress of Q

The strain of P is more than Q therefore P is more ductile.


Therefore (B) is a correct option.
2 e0 s/2 4 e0 s/2
stress C1 = C2 =
Y= d/2 d/2
strain
For a given strain, stress is more for Q. Therefore
YQ > YP.
12. (b, c) Let us consider an elemental mass dm shown in the
shaded portion.
2 e 0 s/2 e0s
C3 = =
d d
dr 2e 0 s 4 e 0 s
r ´
C1 ´ C2 d d + e0 s
p Ceq = + C3 =
p + dp C1 + C2 6e 0 s d
d
4 e 0 s e0 s
= +
3 d d
GMr 7 e0 s 7 é e0 s ù
Here P 4pr2 – (P + dP) 4pr2 = r (4pr2) dr \ Ceq = = C1 êQ C1 = d ú
R 3 3 d 3 ë û
EBD_780
2015-32 JEE Advanced 2015 Solved Paper
14. (b) Applying combined gas law
é PV
1 1 P2V2 4 P1 ù
PV
1 1 PV ê Here on applying T = T we get P2 = 3 ú
= 2 2 ê 1 2 ú
T1 T2 ê ú
2V1
If V2 = 2 V1 and T2 = 3T1 then ê and V2 = V1 + Ax Þ x = [QV2 = 3V1 ] ú
ë A û
PV
1 1 P ´ 2V1 2
= 2 Þ P1 = P2 1 P1 A 2V1 7
T1 3T1 3 \ W = 2P1V1 + ´ ´ = PV
1 1
2 3 A 3
Now change in internal energy C is correct option
f f Heat supplied
DU = [nR (T2 – T1)] = [P2V2 – P1V1] Q = W + DU
2 2
For monoatomic gas f = 3 7 PV
1 1 3
= + ( P2V2 – PV
1 1)
3 2
3 é3 ù
DU = ê P1 ´ 2V1 – PV
1 1 ú = 3P1 V1 7 PV 3 é4 ù 41
2 ë2 û =
1 1
+ ê P1 3V1 – PV PV
1 1ú = 1 1
\ (b) is the correct option. 3 2 ë3 û 6
Now assuming that the pressure on the piston on the 236
® 140 94
15. (a) 92 U 54 Xe + 38 Sr + x + y
right hand side (not considering the affect of spring) The number of proton in reactants is equal to the
remains the same throughout the motion of the piston products (leaving x and y) and mass number of product
then, (leaving x and y) is two less than reactants
kx kx \ x = p, y = e– is ruled out [B] is incorrect
Pressure of gas = P1 + Þ P2 = P1 + and x = p, y = n is ruled out [C] is incorrect
A A
Total energy loss = (236 × 7.5) – [140 × 8.5 + 94 × 8.5]
where k is spring constant and A = area of piston
= 219 MeV
1 2 The energies of kx and ky together is 4MeV
Energy stored = kx The energy remain is distributed by Sr and Xe which is
2
equal to 219 – 4 = 215 MeV
kx \ A is the correct option
P2 = P1 +
A Also momentum is conserved

3 kx 1
P1 = P1 + \ K .E. µ . Therefore K.Esr > K.Exe
2 A m
16. (a, d) From the figure it is clear that
P1 kx (a) s2 > s1
=
2 A (b) r2 > s2 [As the string is taut]
(c) r1 < s1 [As the string is taut]
P1 A \ r1 < s1 < s2 < r2
\ kx =
2 When P alone is in L2
Also,
V2 = V1 + Ax 2pr 2 (r1 – s 2 ) g
VP = is negative as r1 < s2
V1 = Ax 9h2
Where r is radius of sphere.
V1
\ x= When Q alone is in L1
A
2pr 2 (r2 – s1 ) g
1 P1 A V1 1 VQ = is positive as r2 > s1
\ Energy = ´ = PV
1 1 9h1
2 2 A 4
ur ur
\ A is correct Therefore V P . V Q < O option (d) is correct
Now
æ kx ö kx
W = ò PdV = ò ç P1 + ÷ dV = ò P1dV + ò dV
è A ø A s1

kx T
\ W = ò P1dV + ò ´ (dx) A T
A
s2
kx 2
\ W = P1 (V2 – V1 ) +
2
JEE Advanced 2015 Solved Paper 2015-33

VP r1 – s 2 h1 15 45 9 3 15
Also = ´ ...(i) NA = - =
VQ r2 – s1 h2 6 16 4 24
For equilibrium of Q For S2 (in water)
4 3 4 3 64 49 3 15
T+ pr s 2 g = pr 3 r 2 g ...(ii) NA = – =
3 3 4 25 25 4 5
For equilibrium of P For S2 (in air)
4 3 4 64 49 15
T+ pr r1 g = pr 3 s1 g ...(iii) NA = – =
3 3 25 25 5
(iii) – (ii) gives
4
r1 – s2 = s1 – r2 ...(iv) For S2 (in ns = )
From (i) and (iv) 15

VP h VP h1 15 64 49 3
=– 1 = NA = – =
VQ h2 \ VQ h2 4 25 25 4
\ A is also a correct option æ 16 ö
17. (a, c) For S2 ç in ns = ÷
è 3 15 ø
Applying Snell’s law at P; ns sin im = n1 sin (90° – C)
ns = Refractive index of surrounding 3 15 64 49 9
NA = – =
Q n2 16 25 25 16

ns n1 (a), (c) are correct options


C
P 90° – C 1
18. (d) NA = n12 – n22
im ns
Here
NA2 < NA1
\ the NA of combined structure is equal to the
n2 smaller value of the two numerical apertures.
Also sin C = (d) is the correct option.
n1
19. (a, d) When megnetic force balances electric force
FB = FE
q vd B = q E
n1
V
n2 \ vd B = [Q V = E × w]
w
C é I ù
\ V = wvdB = w ê ú´B
Now ë newd û
é I I ù
n1 n1 n22 êvd = neA = newd ú
NA = sin im = n cos C = n 1 – ë û
s s n12
I
\ V = ´B
ned
n12 – n22
\ NA = 1
ns \ Vµ Þ V1d1 = V2d2
d
For S1 (in air)
when d1 = 2d2, V2 = 2V1
45 9 3 and when d1 = d2, V2 = V1
NA = – = (a), (d) are correct options
16 4 4
For S1 (in water) 20. (a, d) Here
B Vn V n
3 45 9 9 Vµ Þ 11= 2 2
NA = – = n B1 B2
4 16 4 16
If B1 = B2 and n1 = 2n2 Þ V2 = 2V1
æ 6 ö and of B1 = 2B2 and n1 = n2 Þ V2 = 0.5V1
For s1 ç in ns = ÷ A and C are the correct options.
è 15 ø
EBD_780
2015-34 JEE Advanced 2015 Solved Paper
CHEMISTRY 3. (9) Number of moles in gas phase, at start (ni) = 1
238 206 4 0
1. (6) 3B2H6 + 18CH3OH ® 6B(OCH3)3 + 18H2 92 U ®82 Pb + 82 He + 6 – b
2. (3) 1 ® HX 2 ® HY Now number of moles in gas phase, after decomposition (n F)
= 1 + 8 = 9 mole
( l m ) HX ( l m )HY at constant temperature and pressure
a1 = a2 =
l°m l °m
PF n 9
= F = =9
K a1 = C1a12 K a 2 = C2a 22 Pin n in 1

( l m )2HX ( lm ) 2 HY
4. (8) 8H+ + 5[Fe(H2O)2(OX)2]2– + MnO4– ® Mn2+ +
= 0.01
(l )
° 2
m
= 0.1
(l°m )
2 + 5 [Fe(H2O)2(OX)2]– + 4H2O

1 d[H + ] d[MnO 4– ]
2 Rate = =–
K a1 ( ) 2
0.01 l m HX æ ( l m ) HX ö 8 dt dt
\ = = 0.1ç ÷
( ) è ( l m ) HY ø
Ka 2 2
0.1 l m HY
rate of [H + ]decay
Hence, =8
æ 1ö
2 rate of [MnO 4– ] decay
= 0.1 ç ÷ = 10–3
è 10 ø
K a1
pKa(HX) – pKa(HY) = –log K = –log10–3 = 3
a2

+
H 1, 2– Methyl Shift D
+
5. (4) H – H2O + + –H

OH CH CH
3 3
aq. dilute 0°C
KMnO4excess

OH

OH
OH
OH
CHO

6. (4)
CO, HCl
(Gatterman Koch Reaction)
Anhyd AlCl3 /CuCl

OH
CHCl 2 CH CHO
H2O OH
100°C – H 2O

COCl CHO
H2
(Rosenmund Reduction)
Pd–BaSO 4

COOMe CHO
DIBAL–H
Toluene, –78°C H2O
JEE Advanced 2015 Solved Paper 2015-35

Et3P CO
O
+2 –
7. (3) CH3– C – Fe Br

Et3P CO
8. (6) All the complexes given show cis-trans isomerism
[Co (NH2 — CH2 — CH2 — NH2) Cl2] + [CrCl2(C2O4)2]3–
en Cl ox
Cl
en Co en en Co ox Cr ox ox Cr
Cl Cl
Cl Cl Cl

[Fe(H2O)4(OH)2]+ [Fe(NH3)2(CN)4]–
OH OH CN NH3
HO OH2 HO OH
2 2 HN
3
CN NC CN
Fe Fe Fe Fe
HO
2 OH2 HO
2 OH2 HN CN NC CN
3
OH OH2 CN NH3

[Co(NH2–CH2–CH2–NH2)2(NH3)Cl]2+ (Co(NH3)4(H2O)Cl]2+
en NH3 OH2
NH3 HO NH3 H3N
2 NH3
en Co Co Co
en Co en
NH3 Cl NH3 H3N NH3
Cl NH3 Cl
Cl

+
,H Radical, initiator, O 2
9. (b)
High Pressure, heat
Cumene, T Cumene hydroperoxide, U

NH 2
10. (a)

Cl Cl
11. (b, c) Cl–O–H Cl
O O O O OH
OH O OH
(i) (ii) (iii) (iv)
Number of Cl = O bonds in (ii) and (iii) together is 3
Number of lone pairs on Cl in (ii) and (iii) together is 3
Hybridisation of Cl in all the four is sp3
Strongest acid is HClO4 (iv)
EBD_780
2015-36 JEE Advanced 2015 Solved Paper
12. (c, d) Only group II cations precipitate as sulphide with H2S 15. (c) P(V–b) = RT
in acidic medium that is (Cu2+, Pb2+) and (Hg2+, Bi3+) Þ PV – Pb = RT
13. (b) (CH3)2SiCl2 form linear polymer on hydrolysis and PV Pb
(CH3)3SiCl is a chain terminator. Þ = +1
RT RT
14. (b, c, d) Reaction on metal surface
Pb
+ – Þ Z =1+
M ¾¾
®M + e RT
Hence Z > 1 at all pressures.
O2 + e – ¾¾
® O2–
This means , repulsive tendencies will be dominant
This is an example of chemisorption. when interatomic distance are small.
This means, interatomic potential is never negative
but becomes positive at small interatomic distances.

16. (a)
(i) O3 NH 3
(ii) Zn, H 2O

–2H2O

17. (a) Let the heat capacity of insulated beaker be C.


Mass of aqueous content in expt. 1 = (100 + 100) × 1 19. (c) OH
= 200 g Pd-BaSO4 i. B 2H6
Þ ± Total heat capacity = (C + 200 × 4.2) J/K C8 H 6 H2 ii. H2 O2 , NaOH, H2O
Moles of acid, base neutralised in expt.
1 = 0.1 × 1 = 0.1
Þ Heat released in expt. 1 = 0.1 × 57 = 5.7 KJ
= 5.7 × 1000 J
Þ 5.7 × 1000 = (C + 200 × 4.2) × D´T. O
5.7 × 1000 = (C + 200 + 4.2) × 5.7
Þ (C + 200 × 4.2) = 1000 H2O
In second experiment, 20. (d) C8H6
HgSO4 ,
nCH COOH = 0.2, nNaOH = – 0.1 H2SO4
3
i. Et MgBr, H2O
Total mass of aqueous content = 200 g
+
Þ Total heat capacity = (C + 200 × 4.2) = 1000 ii. H , Heat
Þ Heat released = 1000 × 5.6 = 5600 J.
Overall, only 0.1 mol of CH3COOH undergo
neutralization.
–5600
Þ DHneutralization of CH3COOH =
0.1
= – 56000 J/mol
= – 56 KJ/mol. Y
Þ DHneutralization of CH3COOH = 57 – 56 = 1 KJ/mol
18. (b) Final solution contain 0.1 mole of CH3COOH and
CH3COONa each.
Hence it is a buffer solution.
[CH3COO - ]
pH = pKa log
[CH3 COOH]
0.1
= 5–log 2 + log = 4.7
0.1
JEE Advanced 2015 Solved Paper 2015-37
r
MATHEMATICS 4. (9)
r r r
s = 4 p + 3q + 5r
r r r r r r r r r r
n
ecos a – e –e s = x(– p + q + r ) + y( p - q + r ) + z(– p - q + r )
1. (2) lim =
a® 0 a m 2 Þ –x+y–z=4
x–y–z=3
é n ù x+y+z=5
e êe cos a – 1 – 1ú n
lim ë û ´ cos a – 1 = –e
Þ 9 -7
a®0 cos a n - 1 am 2 Solving above equations x = 4, y = ,z=
2 2
2 \ 2x + y + z = 9
æ an ö
n
a ç ÷
–2 sin 2 ç 2 ÷ ipk
2 è ø = –e kp kp
Þ e lim ´ 5. (4) ak = cos + i sin =e 7
2 m 2 7 7
a®0 æ n ö
a a
çç ÷÷
è 2 ø ip( k +1) ipk ipk
ak + 1 – ak = e 7 – e 7 = e 7 (eip/7 - 1)
– e 2n – m – e
Þ a = or a 2 n – m = 1
2 2 a k +1 - a k = eip / 7 - 1

m
Þ 2n – m = 0 Þ = 2 12
n
Þ å ak +1 – a k = 12 eip / 7 - 1
k =1
1 (9 x +3tan –1 x ) æ 12 + 9 x 2 ö
2. (9) a = ò 0
e ç
è 1 + x2 ø
÷ dx 3
Similarly å a 4k -1 – a 4k -2 = 3 ei p / 7 - 1
k =1
12 + 9 x 2
Let 9x + 3tan–1x = t Þ dx = dt
1 + x2 12

3p
å a k +1 – a k
3p k =1
9+ 9+ \ =4
\ a= 4 e t dt
ò =e 4 -1 3
0
å a 4k -1 – a 4k -2
k =1
3p
9+ 3p
\ loge 1 + e 4 –1 – =9
4 7
[2a + 6d ]
2 6
= Þ a = 9d
6. (9) 11 11
x [2a + 10d ]
lim
F ( x)
=
1
Þ lim x
ò–1 f (t )dt 2
3. (7) x ®1 G ( x) 14 x®1
ò t f ( f (t )) dt
–1
a7 = a + 6d = 15d
Q 130 < 15d < 140 Þ d = 9
1 1 (Q All terms are natural numbers \ d Î N )
Q ò-1 f (t )dt = 0 and ò-1t f ( f (t )) dt = 0
7. (8) In expansion of (1 + x) (1 + x2) (1 + x3) .... (1 + x100)
f(t) being odd function x9 can be found in the following ways
\ Using L Hospital’s rule, we get
x9, x1 + 8, x2 + 7, x3 + 6, x4 + 5, x1 + 2 + 6, x1 + 3 + 5, x2 + 3 + 4
f ( x) 1 The coefficient of x9 in each of the above 8 cases is 1.
lim =
x ®1 x f ( f ( x )) 14 \ Required coefficient = 8.

f (1) 1 1/ 2 1 x2 y2
Þ = Þ = 8. (4) Ellipse: + =1
f ( f (1)) 14 æ 1 ö 14 9 5
fç ÷
è 2ø
2
Þ a = 3, b = 5 and e = 3
æ 1ö æ 1ö
Þ f ç ÷ =7 Þ fç ÷ =7 \ f1 = 2 and f2 = –2
è 2ø è 2ø
EBD_780
2015-38 JEE Advanced 2015 Solved Paper
P1 : y2 = 8x and P2 : y2 = –16x eqn of tangent to E1 at Q is
2 5x 4y x y
T1 : y = m1x + m 2
+ 2
= 1 which is identical to + =1
1 3a 3b 3 3
It passes through (–4, 0), 4 1
Þ a2 = 5 and b2 = 4 Þ e12 = 1 – =
2 1 5 5
0 = –4m1 + Þ m12 =
m1 2 eqn of tangent to E2 at R is

4 x 8y x y
T2 : y = m2x – + = 1 identical to + =1
m2 3c 2 3d 2 3 3

It passes through (2, 0) 1 7


Þ c2 = 1, d2 = 8 Þ e22 = 1 – =
4 8 8
0 = 2m2 – Þ m22 = 2
m2 43 7 27
\ e12 + e22 = , e1e2 =
2 2
, e1 - e 2 =
40 2 10 40
1
\ + m22 = 4 11. (a, b, d) H : x2 – y2 = 1 S : Circle with centre N(x2, 0)
m12
Common tangent to H and S at P(x1, y1) is
9. (b, c, d) xx1 – yy1 = 1
6 1 p x1
a = 3sin–1 > 3sin–1 or a > Þ m1 =
11 2 2 y1
\ cosa < 0
Also radius of circle S with centre N(x2, 0) through point of
4 1 contact (x1, y1) is perpendicular to tangent
b = 3cos–1 > 3cos–1 or b > p
9 2 x1 0 - y1
\ cosb < 0 and sinb < 0 \ m1m2 = –1 Þ ´ = –1
y1 x2 - x1
3p Þ x1 = x2 – x1 or x2 = 2x1
Also a + b > \ cos(a + b) > 0.
2 M is the point of intersection of tangent at P and x-axis
10. (a, b)
æ1 ö
\ M ç , 0÷
x2 y2 è x1 ø
Let E1 : + = 1 where a > b
a2 b2
Q Centroid of DPMN is (l, m)
x2 y 2 1
and E2 : 2 + 2 = 1 where c < d \ x1 + + x2 = 3l and y1 = 3m
c d x1
Also S : x2 + (y – 1)2 = 2 Using x2 = 2x1,
Tangent at P(x1, y1) to S is x + y = 3
1æ 1ö
To find point of contact put x = 3 – y in S. We get P(1, 2)
Þ ç 3 x1 + ÷ = l and y1 = m
Writing eqn of tangent in parametric form 3è x1 ø 3

x -1 y - 2 2 2 dl 1 dm 1
= =± \ =1- 2 , =
–1 1 3 dx1 3 x1 dy1 3
2 2
Also (x1, y1) lies on H, \ x12 - y12 = 1
-2 2 2 -2
x= + 1 or + 1 and y = + 2 or +2
3 3 3 3 or y1 = x12 - 1
1 5 8 4 1 2
Þ x= or and y = or \ m= x1 - 1
3 3 3 3 3
æ 5 4ö æ 1 8ö dm x1
\ Q çè , ÷ø and R çè , ÷ø \
3 3 3 3 dx1 =
3 x12 - 1
JEE Advanced 2015 Solved Paper 2015-39

12. (a, c) Let F(t) = et (sin6at + cos6at)


Then F(kp + t) = ekp + t [sin6(kp + t)a + cos6(kp + t)a] 192 x3
15. (d) f ¢(x) =
= ekp.et [sin6at + cos6at] for even values of a. 2 + sin 4 px
\ F(kp + t) = ekp F(t) ...(i)
192 x 3 192 x 3
4p p 2p 3p 4p Þ £ f ¢ ( x) £
Now ò0 F (t)dt = ò0 F (t)dt + òp F (t )dt +ò2p F (t )dt +ò3p F (t )dt 3 2
Þ 64x3 < f ' (x) £ 96x3
2p p
Also òp F (t )dt = ò F (p + x )dx (putting t = p + x) x 3 x x
96 x3 dx
0 Þ ò1/2 64 x dx £ ò
1/2
f ¢( x)dx £ ò
1/2
p p
= ò0 e F ( x )dx using eqn(i)
64 x 4 64 1 x 96 x 4 96
Þ – ´ £ ò f ¢( x )dx £ -
4 4 16 1/2 4 4 ´ 16
p
= ep ò F (t )dt
0 x 3
3p p
Þ 16x4 – 1 < ò1/ 2 f ¢( x) < 24x4 – 2
Similarly ò F (t )dt = e2p F (t )dt
ò
2p 0
3
Þ 16x4 – 1 < f(x) < 24x4 –
4p p 2
ò3p F (t )dt = e3p F (t )dt
ò
0
1 1 1 æ 3ö
ò1/ 2 (16 x - 1)dx £ ò f ( x )dx £ ò
4
24 x 4 – ÷ dx
4p p Þ 1/ 2 çè 2ø
ò0 F (t )dt = (1 + ep + e2p + e3p) ò F (t )dt 1/ 2
\
0
1 1
4p æ 16 x 5 ö 1 é 24 x 5 3 ù
ò0 F (t )dt = e –1 4p
Þ ç
è 5
- x÷ £ ò f ( x )dx £ ê - xú
Þ p , where ‘a’ can take any even ø 1 1/ 2 ëê 5 2 ûú 1
ep –1
ò0 F (t )dt 2 2

1
value. Þ 2.6 < ò1/ 2 f ( x)dx < 3.9
13. (b, c) Let h(x) = f(x) – 3g(x)
h(–1) = h(0) = h(2) = 3 \ Only (d) is the correct option.
\ By Rolle’s theorem h'(x) = 0 has atleast one solution in 16. (a, d) ax2 – x + a = 0 has distinct real roots.
(–1, 0) and atleast one solution in (0, 2) But h¢¢(x) never \ D > 0 Þ 1 – 4a2 > 0
vanishes in (–1, 0) and (0, 2) therefore h'(x) = 0 should
æ 1 1ö
have exactly one solution in each interval. Þ aÎ çè – , ÷ø ...(i)
2 2
14. (a, b) f(x) = 7 tan8x + 7tan6x – 3tan4x – 3tan2x
= (7tan4x – 3) (tan4x + tan2x) Also |x1 – x2| < 1
= (7tan6x – 3tan2x) sec2x Þ (x1 – x2)2 < 1
Þ (x1 + x2)2 – 4x1x2 < 1
p/4 p/4
ò0 f ( x)dx = é tan 7 x - tan 3 x ù
ë û0 1
Þ –4<1
=1–1=0 a2

( )
p/4 p/4 1 1
\ ò0 xf ( x) dx = é x tan 7 x - tan 3 x ù Þ < 5 or a2 >
ë û0 a 2
5

–ò
0
p/4
( tan 7
)
x - tan 3 x dx æ 1 ö æ 1
Þ aÎ çè – ¥, – ÷ø È çè
5
ö
, ¥÷
5 ø
...(ii)

p/4 Combining (i) and (ii)


p/4 é tan 4 x tan 6 x ù
3 2 -
= ò0 tan x (1 - tan x ) sec2x dx = ê
êë 4 6 úû
ú
æ 1 –1 ö æ 1 1 ö
0
S = çè – , ÷ø È çè , ÷
2 5 5 2ø
1
=
12 æ 1 –1 ö æ 1 1ö
\ Subsets of S can be çè – 2 , ÷ø and çè , ÷.
5 5 2ø
EBD_780
2015-40 JEE Advanced 2015 Solved Paper
17. (a, b, c) f(x) = xF(x) Þ f ' (x) = F(x) + xF'(x)
ì Using xF ¢ ( x) = f ¢( x) – F ( x )
ï
æ æ 1 öö í 3 2
\ f '(1) = F(1)+ F '(1) = F ' (1) < 0 çQ F ¢( x) < 0, x Î ç ,3 ÷ ÷ ïîand ò1 x F ¢( x )dx = –12
è è 2 øø
f(2) = 2F(2) < 0, Þ 9(f ' (3) – F(3)) – (f ' (1) – F(1)) = 4
Þ 9f ' (3) – 9 × (–4) – f ' (1) + 0 = 4
(Q F '(x) < 0 Þ F is decreasing on æ 1 ,3ö and F '(1) = 0, Þ 9f ' (3) – f ' (1) + 32 = 0
çè ÷
2 ø
19. (a,b) Let E1 º box I is selected
F(3) = –4) E2 º box II is selected
f '(x) = F(x) + x F '(x) E º ball drawn is red
For the same reason given above and F '(x) < 0 given.
F(x) < 0 " x Î(1, 3) n3 1
´
\ f '(x) ¹ 0, xÎ(1, 3). n3 + n4 2 1
P(E2/E) = n =
1 1 n3 1 3
3 2 ´ + ´
18. (c,d) ò1 x F ¢( x)dx = –12 n1 + n2 2 n3 + n4 2

3 3
é x 2 F ( x) ù – 2 x F ( x) dx = -12
û1 ò1
Þ n3
ë
n3 + n4 1
or =
3 n1 n3 3
Þ 9 F(3) – F(1) – 2 ò xF ( x) dx = -12 +
1 n1 + n2 n3 + n4
3 On checking the options we find A and B are the correct
3
Þ ò1 xF ( x)dx = -12 Þ ò f ( x)dx = -12 ...(i) options.
1 20. (c, d) E1 º Red ball is selected from box I
3 3 E2 º Black ball is selected from box I
Also ò1 x F ¢¢ ( x ) dx = 40 E º Second ball drawn from box I is red
\ P(E) = P(E1) P(E/E1) + P(E2) P(E/E2)
3
é x3 F ¢( x) ù - 3 3 x 2 F ¢( x)dx = 40
Þ ë û1 ò1 n1 n1 - 1 n2 n1
= ´ + ´
n1 + n2 n1 + n2 - 1 n1 + n2 n1 + n2 - 1
3
Þ é x 2 ( f ¢( x) - F ( x) ) ù – 3 × (–12) = 40 On checking the options, we find C and D have the correct
ë û1
values.
JEE ADVANCED 2014
1. The question paper consists of three parts (Physics, Chemistry and Mathematics). Each part consists of two
sections.
2. Section 1 contains 10 multiple choice questions. Each question has four choices (a), (b), (c) and (d) out of
which ONE OR MORE THAN ONE are correct.
3. Section 2 contains 10 questions. The answer to each of the questions is a single-digit integer ranging from 0 to
9 (both inclusive).

PAPER - 1

PHYSICS n1

SECTION - I
This section contains 10 multiple choice questions. Each question Air n2
has 4 choices (a), (b), (c) and (d) out of which ONE or MORE
THAN ONE are correct.
1. Let E1 (r), E2(r) and E3(r) be the respective electric field at a
distance r from a point charge Q, an infinitely long wire with (a) f1 3R (b) f1 2.8R
constant linear charge density , and an infinite plane with (c) f2 2 R (d) f2 1.4R
uniform surface charge density . If E1(r0) = E2(r0) = E3(r0)
4. A student is performing an experiment using a resonance
at a given distance r0, then column and a tuning fork of frequency 244 s–1. He is told that
(a) Q 4 r02 the air in the tube has been replaced by another gas (assume
that the column remains filled with the gas). If the minimum
r0 height at which resonance occurs is 0.350 0.005 m, the
(b)
2 gas in the tube is
(c) E1 r0 / 2 2 E2 r0 / 2 (Useful information: 167 RT 640J1/ 2 mole 1/ 2 ;

(d) E2 r0 / 2 4 E3 r0 / 2 140 RT 590J1/ 2 mole 1/ 2 . The molar masses M in grams

2. Heater of an electric kettle is made of a wire of length L and 10


diameter d. It takes 4 minutes to raise the temperature of are given in the options. Take the values of for each
M
0.5 kg water by 40 K. This heater is replaced by a new heater gas as given there.)
having two wires of the same material, each of length L and
diameter 2d. The way these wires are connected is given in 10 7
(a) Neon M 20,
the options. How much time in minutes will it take to raise the 20 10
temperature of the same amount of water by 40 K?
(a) 4 if wires are in parallel (b) 2 if wires are in series 10 3
(c) 1 if wires are in series (d) 0.5 if wires are in parallel (b) Nitrogen M 28,
28 5
3. A transparent thin film of uniform thickness and refractive
index n1 = 1.4 is coated on the convex spherical surface of 10 9
radius R at one end of a long solid glass cylinder of refractive (c) Oxygen M 32,
32 16
index n2 = 1.5, as shown in the figure. Rays of light parallel to
the axis of the cylinder traversing through the film from air to
10 17
glass get focused at distance f1 from the film, while rays of (d) Argon M 36,
36 32
light traversing from glass to air get focused at distance f2
from the film, Then
EBD_780
2-2014 JEE Advanced 2014 Solved Paper
5. In the figure, a ladder of mass m is shown leaning against a 8. At time t = 0, terminal A in the circuit shown in the figure is
wall. It is in static equilibrium making an angle with the connected to B by a key and an alternating current
horizontal floor. The coefficient of friction between the wall I(t) = I0cos ( t), with I0 = 1 A and = 500 rad s–1 starts
and the ladder is 1 and that between the floor and the ladder flowing in it with the initial direction shown in the figure. At
is 2. The normal reaction of the wall on the ladder is N1 and
that of the floor is N2. If the ladder is about to slip, then 7
t , the key is switched from B to D. Now onwards only
6
1
A and D are connected. A total charge Q flows from the
battery to charge the capacitor fully. If C = 20 F, R = 10
and the battery is ideal with emf of 50 V, identify the correct
statement(s).

B D
A

2 50 V
C = 20 F
mg
(a) 1 0, 2 0 and N2 tan =
2
mg R = 10
(b) 1 0, 2 0 and N1 tan
2 (a) Magnitude of the maximum charge on the capacitor
mg 7
(c) 1 0, 2 0 and N 2 before t is 1 × 10–3 C
1 1 2 6
(b) The current in the left part of the circuit just before
mg
(d) 1 0, 2 0 and N1 tan 7
2 t is clockwise
6. Two ideal batteries of emf V1 and V2 and three resistances 6
R1, R2 and R3 are connected as shown in the figure. The (c) Immediately after A is connected to D, the current in R
current in resistance R2 would be zero if is 10 A
(d) Q = 2 × 10–3 C
9. One end of a taut string of length 3 m along the x-axis is fixed
at x = 0. The speed of the waves in the string is 100 ms –1. The
V1 R1 other end of the string is vibrating in the y-direction so that
R2 stationary waves are set up in the string. The possible
waveform (s) of these stationary waves is(are)
x 50 t
V2 (a) y t A sin cos
6 3
R3
x 100 t
(b) y t A sin cos
3 3
(a) V1 = V2 and R1 = R2 = R3
(b) V1 = V2 and R1 = 2R2 = R3 5 x 250 t
(c) y t A sin cos
(c) V1 = 2V2 and 2R1 = 2R2 = R3 6 3
(d) 2V1 = V2 and 2R1 = R2 = R3
5 x
7. A light source, which emits two wavelength 1 = 400 nm and (d) y t A sin cos 250 t
2
2 = 600 nm, is used in a Young’s double slit experiment. If
recorded fringe widths for 1 and 2 are 1 and 2 and the 10. A parallel plate capacitor has a dielectric slab of dielectric
number of fringes for them within a distance y on one side of constant K between its plates that covers 1/3 of the area of
the central maximum are m1 and m2 respectively, then its plates, as shown in the figure. The total capacitance of the
capacitor is C while that of the portion with dielectric in
(a) 2> 1 between is C1. When the capacitor is charged, the plate area
(b) m1 > m2 covered by the dielectric gets charge Q1 and the rest of the
(c) Form the central maximum, 3rd maximum of 2 overlaps area gets charge Q2. The electric field in the dielectric is E1
with 5th minimum of 1 and that in the other portion is E2. Choose the correct option/
(d) The angular separation of fringes for 1 is greater options, ignoring edge effects.
than 2.
JEE Advanced 2014 Solved Paper 2014-3

Q1 E1

15. A uniform circular disc of mass 1.5 kg and radius 0.5 m is


initially at rest on a horizontal frictionless surface. Three forces
E2 of equal magnitude F = 0.5 N are applied simultaneously
Q2
along the three sides of an equilateral triangle XYZ with its
vertices on the perimeter of the disc (see figure). One second
after applying the forces, the angular speed of the disc in rad
E1 E1 1 s–1 is
(a) 1 (b) F
E2 E2 K X
Q1 3 C 2 K
(c) Q2 K (d) C1 K
O
SECTION - II
This section contains 10 questions. Each question, when worked Y F
out will result in one integer from 0 to 9 (both inclusive). Z
F
11. A galvanometer gives full scale deflection with 0.006 A 16. Consider an elliptical shaped rail PQ in the vertical plane with
current. By connecting it to a 4990 resistance, it can be OP = 3 m and OQ = 4 m. A block of mass 1 kg is pulled along
converted into a voltmeter of range 0 – 30 V. If connected to the rail from P to Q with a force of 18 N, which is always
parallel to line PQ (see the figure given). Assuming no
2n
a resistance, it becomes an ammeter of range 0 – 1.5A. frictionless losses, the kinetic energy of the block when it
249 reaches Q is (n × 10) joules. The value of n is (take acceleration
The value of n is due to gravity = 10 ms–2)
12. To find the distance d over which a signal can be seen clearly Q
in foggy conditions, a railways-engineer uses dimensions
and assumes that the distance depends on the mass density
of the fog, intensity (power/area) S of the light from the
4m
signal and its frequency f. The engineer finds that d is
proportional to S1/n. The value of n is
13. During Searle’s experiment, zero of the Vernier scale lies 90°
between 3.20 × 10–2 m and 3.25 × 10–2 m of the main scale.
O 3m P
The 20th division of the Vernier scale exactly coincides with
one of the main scale divisions. When an additional load of 17. A thermodynamic system is taken from an initial state i with
2 kg is applied to the wire, the zero of the Vernier scale still internal energy Ui = 100 J to the final state f along two different
paths iaf and ibf, as schematically shown in the figure. The
lies between 3.20 × 10–2 m and 3.25 × 10–2 m of the main scale
work done by the system along the paths af, ib and bf are
but now the 45th division of Vernier scale coincides with one
Waf = 200 J, Wib = 50 J and Wbf =100 J respectively. The heat
of the main scale divisions. The length of the thin metallic
supplied to the system along the path iaf, ib and bf are Qiaf,
wire is 2 m and its cross-sectional area is 8 × 10–7 m2. The
Qib and Qbf respectively. If the internal energy of the system
least count of the Vernier scale is 1.0 × 10–5 m. The maximum
percentage error in the Young’s modulus of the wire is Qbf
in the state b is Ub = 200 J and Qiaf = 500 J, The ratio is
14. A horizontal circular platform of radius 0.5 m and mass Qib
0.45 kg is free to rotate about its axis. Two massless spring
toy-guns, each carrying a steel ball of mass 0.05 kg are a f
attached to the platform at a distance 0.25 m from the centre P
on its either sides along its diameter (see figure). Each gun
simultaneously fires the balls horizontally and perpendicular i
to the diameter in opposite directions. After leaving the b
platform, the balls have horizontal speed of 9 ms–1 with V
respect to the ground. The rotational speed of the platform in
rad s–1 after the balls leave the platform is 18. Two parallel wires in the plane of the paper are distance X0
apart. A point charge is moving with speed u between the
EBD_780
4-2014 JEE Advanced 2014 Solved Paper
wires in the same plane at a distance X1 from one of the wires. 3. Hydrogen bonding plays a central role in the following
When the wires carry current of magnitude I in the same phenomena
direction, the radius of curvature of the path of the point (a) Ice floats in water
charge is R1. In contrast, if the currents I in the two wires (b) Higher Lewis basicity of primary amines than tertiary
have directions opposite to each other, the radius of curvature amines in aqueous solutions
(c) Formic acid is more acidic than acetic acid
X0 R1
of the path is R2. If 3 , the value of is (d) Dimerisation of acetic acid in benzene
X1 R2 4. The correct combination of names for isomeric alcohols with
19. Airplanes A and B are flying with constant velocity in the molecular formula C4H10O is/are
same vertical plane at angles 30° and 60° with respect to the (a) Tert-butanol and 2-methylpropan-2-ol
horizontal respectively as shown in figure. The speed of A is (b) Tert-butanol and 1, 1-dimethylethan-1-ol
(c) n-butanol and butan-1-ol
100 3 m/s. At time t = 0 s, an observer in A finds B at a (d) Isobutyl alcohol and 2-methylpropan-1-ol
distance of 500 m. The observer sees B moving with a 5. The reactivity of compound Z with different halogens under
constant velocity perpendicular to the line of motion of A. If appropriate conditions is given below:
at t = t0, A just escapes being hit by B, t0 in seconds is
OH mono halo substituted
A derivative when X2 = I2
X2 di halo substituted
derivative when X2 = Br2
B Z C(CH3)3
tri halo substituted
30° 60° derivative when X2 = Cl2

20. A rocket is moving in a gravity free space with a constant The observed pattern of electrophilic substitution can be
acceleration of 2 m/s2 along +x direction (see figure). The explained by
length of a chamber inside the rocket is 4 m. A ball is thrown (a) The steric effect of the halogen
from the left end of the chamber in +x direction with a speed (b) The steric effect of the tert-butyl group
of 0.3 m/s relative to the rocket. At the same time, another ball (c) The electronic effect of the phenolic group
is thrown in –x direction with a speed of 0.2 m/s from its right (d) The electronic effect of the tert-butyl group
end relative to the rocket. The time in seconds when the two 6. The pair(s) of reagents that yield paramagnetic species is/are
balls hit each other is (a) Na and excess of NH3
(b) K and excess of O2
0.3 m/s 0.2 m/s a = 2 m/s
2 (c) Cu and dilute HNO3
x (d) O2 and 2-ethylanthraquinol
7. For the reaction
4m
I ClO3 H 2SO 4 Cl HSO4 I2
The correct statement(s) in the balanced equation is/are
CHEMISTRY
(a) Stoichiometric coefficient of HSO4 is 6
SECTION - I (b) Iodide is oxidized
This section contains 10 multiple choice questions. Each question (c) Sulphur is reduced
has 4 choices (a), (b), (c) and (d) out of which ONLY ONE or (d) H2O is one of the products
MORE THAN ONE are correct. 8. In the reaction shown below, the major product(s) formed is/
are
1. Upon heating with Cu2S, the reagent(s) that give copper NH2
metal is/are
(a) CuFeS2 (b) CuO acetic anhydride
Product(s)
(c) Cu2O (d) CuSO4 NH2 CH2Cl2
2. In a galvanic cell, the salt bridge Z O
(a) Does not participate chemically in the cell reaction H
(b) Stops the diffusion of ions from one electrode to N CH3
another O
(c) Is necessary for the occurrence of the cell reaction + CH3COOH
(a)
(d) Ensures mixing of the two electrolytic solutions NH2
O
JEE Advanced 2014 Solved Paper 2014-5

diagram. The final internal pressure, volume and absolute


NH2
temperature of the gas are P2, V2 and T2, respectively. For
this expansion,
H + CH3COOH
(b) N CH3
O O Pext = 0

H Pext = 0 Irreversible
N CH3
P2, V2, T2
O P1, V1, T1
(c) H + H2O
N CH3
O Thermal insulation
O
(a) q = 0 (b) T2 = T1
+ – (c) P2V2 = P1V1 (d) P2V2 = P1V1
NH3CH3COO
10. The correct statement(s) for orthoboric acid is/are
(a) It behaves as a weak acid in water due to self ionization.
(d) H
N CH3 (b) Acidity of its aqueous solution increases upon addition
of ethylene glycol
O O
(c) It has a three dimensional structure due to hydrogen
9. An ideal gas in a thermally insulated vessel at internal pressure bonding
= P1, volume = V1 and absolute temperature = T1 expands (d) It is a weak electrolyte in water
irreversibly against zero external pressure, as shown in the

SECTION - II
This section contains 10 questions. Each question, when worked out will result in one integer from 0 to 9 (both inclusive).
11. The total number of distinct naturally occurring amino acids obtained by complete acidic hydrolysis of the peptide shown
below is
O

O O O H
O H H N
N N N
N N N
N O
CH2 O H O H CH2
H
O

12. MX2 dissociates into M2+ and X– ions in an aqueous solution, 16. Consider the following list of reagents:
with a degree of dissociation ( ) of 0.5. The ratio of the Acidified K2Cr2O7, alkaline KMnO4, CuSO4, H2O2, Cl2, O3,
observed depression of freezing point of the aqueous solution FeCl3, HNO3 and Na2S2O3.
to the value of the depression of freezing point in the absence The total number of reagents that can oxidise aqueous iodide
of ionic dissociation is to iodine is
13. If the value of Avogadro number is 6.023 × 1023 mol–1 and 17. A list of species having the formula XZ4 is given below.
the value of Boltzmann constant is 1.380 × 10–23 J K–1, then
the number of significant digits in the calculated value of the XeF4, SF4, SiF4, BF4–, BrF4–, [Cu(NH3)4]2+, [FeCl4]2– ,
universal gas constant is [CoCl4]2– and [PtCl4]2–.
Defining shape on the basis of the location of X and Z atoms,
14. A compound H2X with molar weight of 80g is dissolved in a
the total number of species having a square planar shape is
solvent having density of 0.4 g ml–1. Assuming no change in
18. Consider all possible isomeric ketones, including
volume upon dissolution, the molality of a 3.2 molar solution
stereoisomers of MW = 100. All these isomers are
is
independently reacted with NaBH4 (NOTE: stereoisomers
15. In an atom, the total number of electrons having quantum
are also reacted separately). The total number of ketones
1 that give a racemic product(s) is/are
numbers n = 4, | ml | = 1 and ms = is
2
EBD_780
6-2014 JEE Advanced 2014 Solved Paper
19. The total number(s) of stable conformers with non-zero (b) The second row of M is the transpose of the first column
dipole moment for the following compound is (are) of M
Cl (c) M is a diagonal matrix with non-zero entries in the main
Br CH3 diagonal
(d) The product of entries in the main diagonal of M is not
Br Cl
the square of an integer
CH3
20. Among PbS, CuS, HgS, MnS, Ag2S, NiS, CoS, Bi2S3 and 4. Let x , y and z be three vectors each of magnitude 2
SnS2, the total number of BLACK coloured sulphides is
and the angle between each pair of them is . If a is a
3
MATHEMATICS
SECTION - I non-zero vector perpendicular to x and y z and b is a
This section contains 10 multiple choice questions. Each question
non-zero vector perpendicular to y and z x , then
has 4 choices (a), (b), (c) and (d) out of which ONLY ONE or
MORE THAN ONE are correct.
(a) b b.z z x
1. Let f : [a, b] 1, be a continuous function and let
g: R R be defined as
(b) a a.y y z

(c) a.b a.y b.z


0, if x a,
x
g x f t dt , if a x b; then (d) a a. y z y
a
b 5. From a point P( , , ), perpendicular PQ and PR are drawn
f t dt , if x b. respectively on the lines y = x, z = 1 and y = – x, z = – 1. If P is
a such that QPR is a right angle, then the possible value(s)
of is/(are)
(a) 2 (b) 1
(a) g(x) is continuous but not differentiable at a
(c) – 1 (d) 2
(b) g(x) is differentiable on R
6. Let M and N be two 3 × 3 matrices such that MN = NM.
(c) g(x) is continuous but not differentiable at b
Further, if M N 2 and M 2 = N 4, then
(d) g(x) is continuous and differentiable at either (a) or (b) (a) determinant of (M 2 + MN 2) is 0
but not both (b) there is 3 × 3 non-zero matrix U such that (M 2 + MN 2)U
2. For every pair of continuous functions f, g : [0, 1] R such is the zero matrix
(c) determinant of (M 2 + MN 2) 1
that max f x :x 0,1 = max g x :x 0,1 , the (d) for a 3 × 3 matrix U, if (M 2 + MN 2)U equals the zero
correct statement(s) is (are): matrix then U is the zero matrix

(a) (f (c))2 + 3f (c) = (g(c))2 + 3g(c) for some c [0, 1] x t


1 dt
7. Let f : 0, R be given by f x t t . Then
(b) (f (c))2 + f (c) = (g(c))2 + 3g(c) for some c [0, 1] e
1
(c) (f (c))2 + 3f (c) = (g(c))2 + g(c) for some c [0, 1]
x
(d) (f (c))2 = (g(c))2 for some c [0, 1]
3. Let M be a 2 × 2 symmetric matrix with integer entries. Then M (a) f (x) is monotonically increasing on 1,
is invertible if (b) f (x) is monotonically decreasing on (0, 1)
(a) The first column of M is the transpose of the second 1
(c) f x f 0, for all x 0,
row of M x
(d) f (2x) is an odd function of x on R
JEE Advanced 2014 Solved Paper 2014-7

15. For a point P in the plane, let d1(P) and d2(P) be the distance
8. Let f : , R be given by f (x) = (log(sec x + tan x))3. of the point P form the lines x – y = 0 and x + y = 0 respectively.
2 2
Then The area of the region R consisting of all points P lying
(a) f (x) is an odd function in the first quadrant of the plane and satisfying
(b) f (x) is non-one function
(c) f (x) is an onto function 2 d1 P d2 P 4 , is
(d) f (x) is an even function 16. Let n1 < n2 < n3 < n4 < n5 be positive integers such that
9. A circle S passes through the point (0, 1) and is orthogonal n1 + n2 + n3 + n4 + n5 = 20. Then the number of such distinct
to the circles (x – 1)2 + y2 = 16 and x2 + y2 = 1. Then arrangements (n1, n2, n3, n4, n5) is
(a) radius of S is 8
1
(b) radius of S is 7 d2 5
17. The value of 4 x3 2
1 x2 dx is
(c) centre of S is (– 7, 1) 0 dx
(d) centre of S is (– 8, 1)
10. Let a R and let f : R R be given by 18. Let a , b and c be three non-coplanar unit vectors such
f (x) = x5 – 5x + a. Then
(a) f (x) has three real roots if a > 4 that the angle between every pair of them is . If
(b) f (x) has only real root if a > 4 3
(c) f (x) has three real roots if a < – 4
(d) f (x) has three real roots if – 4 < a < 4 a b b c pa qb r c , where p, q and r are scalars,

SECTION - II
p2 2q 2 r2
This section contains 10 questions. Each question, when worked then the value of is
q2
out will result in one integer from 0 to 9 (both inclusive).
11. The slope of the tangent to the curve (y – x5)2 = x(1 + x2)2 at b
19. Let a, b, c be positive integers such that is an integer. If a,
the point (1, 3) is a
12. Let f : [0, 4 ] [0, ] be defined by f (x) = cos–1(cos x). The b, c are in geometric progression and the arithmetic mean of
number of points x 0, 4 satisfying the equation a2 a 14
a, b, c is b + 2, then the value of is
10 x a 1
f x is
10 20. Let n 2 be an integer. Take n distinct points on a circle and
join each pair of points by a line segment. Colour the line
13. The largest value of non-negative integer a for which
segment joining every pair of adjacent points by blue and
1 x the rest by red. If the number of red and blue line segments
ax sin x 1 a 1 x 1 are equal, then the value of n is
lim is
x 1 x sin x 1 1 4

14. Let f : R R and g : R R be respectively given by f (x)


= | x | + 1 and g(x) = x2 + 1. Define h : R R by

max f x ,g x if x 0,
h x
min f x ,g x if x 0.

The number of points at which h(x) is not differentiable is


EBD_780
8-2014 JEE Advanced 2014 Solved Paper

PAPER - 2
1. The question paper consists of three parts (Physics, Chemistry and Mathematics). Each part consists of three
sections.
2. Section 1 contains 10 multiple choice questions. Each questions has four choices (a), (b), (c) and (d) out of
which ONE is correct.
3. Section 2 contains 3 paragraphs each describing theory, experiment and data etc. Six questions relate to three
paragraph with two questions on each paragraph. Each question pertaining to a particular passage should have
only one correct answer among the four given choices (a), (b), (c) and (d).
4. Section 3 contains 4 multiple choice questions. Each questions has two lists (List-1: P, Q, R and S; List-2: 1,
2, 3 and 4). The options for the correct match are provided as (a), (b), (c) and (d) out of which ONLY ONE is
correct.

PHYSICS (a) 60 0.15 (b) 135 0.56


(c) 60 0.25 (d) 135 0.23
SECTION - I
5. A wire, which passes through the hole in a small bead, is
This section contains 10 multiple choice questions. Each question bent in the form of quarter of a circle. The wire is fixed vertically
has four choices (a), (b), (c) and (d) out of which ONLY ONE on ground as shown in the figure. The bead is released from
option is correct. near the top of the wire and it slides along the wire without
friction. As the bead moves from A to B, the force it applies
1. If Cu is the wavelength of K X-ray line of copper (atomic
on the wire is
number 29) and Mo is the wavelength of the K X-ray line of
molybdenum (atomic number 42), then the ratio Cu/ Mo is A
close to
(a) 1.99 (b) 2.14
(c) 0.50 (d) 0.48
2. A metal surface is illuminated by light of two different
wavelengths 248 nm and 310 nm. The maximum speeds of the
photoelectrons corresponding to these wavelengths are 90°
B
u1 and u2, respectively. If the ratio u1 : u2 = 2 : 1 and hc = 1240
eV nm, the work function of the metal is nearly (a) always radially outwards
(a) 3.7 eV (b) 3.2 eV (b) always radially inwards
(c) 2.8 eV (d) 2.5 eV (c) radially outwards initially and radially inwards later
3. Parallel rays of light of intensity I = 912 Wm–2 are incident on (d) radially inwards initially and radially outwards later
a spherical black body kept in surroundings of temperature 1
300 K. Take Stefan-Boltzmann constant = 5.7 × 10–8 6. A planet of radius R radius of Earth has the same
10
Wm–2K–4 and assume that the energy exchange with the
R
surroundings is only through radiation. The final steady state mass density as Earth. Scientists dig a well of depth on it
temperature of the black body is close to 5
(a) 330 K (b) 660 K and lower a wire of the same length and a linear mass density
(c) 990 K (d) 1550 K 10–3 kg m–1 into it. If the wire is not touching anywhere, the
4. During an experiment with a metre bridge, the galvanometer force applied at the top of the wire by a person holding it in
shows a null point when the jockey is pressed at 40.0 cm place is (take the radius of Earth = 6 × 106 m and the
using a standard resistance of 90 , as shown in the figure. acceleration due to gravity on Earth is 10 ms–2)
The least count of the scale used in the metre bridge is 1mm. (a) 96 N (b) 108 N
The unknown resistance is (c) 120 N (d) 150 N
7. A glass capillary tube is of the shape of a truncated cone
with an apex angle so that its two ends have cross sections
of different radii. When dipped in water vertically, water rises
R 90 in it to a height h, where the radius of its cross section is b. If
the surface tension of water is S, its density is , and its
contact angle with glass is , the value of h will be (g is the
acceleration due to gravity)
40.0 cm
JEE Advanced 2014 Solved Paper 2014-9

(a) 1. 21 (b) 1. 30
(c) 1. 36 (d) 1. 42
10. A tennis ball is dropped on a horizontal smooth surface. It
bounces back to its original position after hitting the surface.
The force on the ball during the collision is proportional to
h
the length of compression of the ball. Which one of the
following sketches describes the variation of its kinetic energy
K with time t most appropriately? The figure are only
illustrative and not to the scale.
K

2S 2S (a)
(a) cos (b) cos
b g b g
2S 2S t
(c) cos /2 (d) cos /2 K
b g b g
8. Charges Q, 2Q and 4Q are uniformly distributed in three
dielectric solid spheres 1, 2 and 3 of radii R/2, R and 2R
respectively, as shown in figure. If magnitude of the electric
fields at point P at a distance R from the centre of sphere 1, 2 (b)
and 3 are E1, E2 and E3 respectively, then
P t
P K
R R
2Q
Q
(c)
R/2
Sphere 1 t
Sphere 2 K

R (d)
4Q
t

SECTION - II
2R
This section contains 3 paragraphs, each describing theory,
experiments, data etc. Six questions related to the three paragraphs
Sphere 3 with two questions on each paragraph. Each question has only
(a) E1 > E2 > E3 (b) E3 > E1 > E2 one correct answer among the four given options (a), (b), (c) and
(c) E2 > E1 > E3 (d) E3 > E2 > E1 (d).
9. A point source S is placed at the bottom of a transparent
block of height 10 mm and refractive index 2.72. It is immersed Paragraph for Questions 11 and 12
in a lower refractive index liquid as shown in the figure. It is The figure shows a circular loop of radius a with two long parallel
found that the light emerging from the block to the liquid wires (numbered 1 and 2) all in the plane of the paper. The distance
forms a circular bright spot of diameter 11.54 mm on the top of each wire from the centre of the loop is d. The loop and the wire
of the block. The refractive index of the liquid is are carrying the same current I.The current in the loop is in the
counterclockwise direction if seen from above.
Liquid Q S
d d

Block Wire 1 a Wire 2

S P R
EBD_780
10-2014 JEE Advanced 2014 Solved Paper
11. When d a but wires are not touching the loop, it is found Paragraph for Questions 15 and 16
that the net magnetic field on the axis of the loop is zero at a A spray gun is shown in the figure where a piston pushes air out
height h above the loop. In that case of a nozzle. A thin tube of uniform cross section is connected to
(a) current in wire 1 and wire 2 in the direction PQ and RS, the nozzle. The other end of the tube is in a small liquid container.
As the piston pushes air through the nozzle, the liquid from the
respectively and h a
container rises into the nozzle and is sprayed out. For the spray
(b) current in wire 1 and wire 2 in the direction PQ and SR,
gun shown, the radii of the piston and the nozzle are 20 mm and
respectively and h a 1 mm respectively. The upper end of the container is open to the
(c) current in wire 1 and wire 2 in the direction PQ and SR, atmosphere.
respectively and h 1.2a
(d) current in wire 1 and wire 2 in the direction PQ and RS,
respectively and h 1.2a
12. Consider d >> a, and the loop is rotated about its diameter
parallel to the wires by 30° from the position shown in the 15. If the piston is pushed at a speed of 5 mms–1, the air comes
figure. If the currents in the wires are in the opposite out of the nozzle with a speed of
directions, the torque on the loop at its new position will be (a) 0.1 ms–1 (b) 1 ms–1
(assume that the net field due to the wires is constant over (c) 2 ms –1 (d) 8 ms–1
the loop). 16. If the density of air is a, and that of the liquid l, then for a
2 2 2 2 given piston speed the rate (volume per unit time) at which
0I a 0I a
the liquid is sprayed will be proportional to
(a) (b)
d 2d
2 2 2 2 a
3 0I a 3 0I a (a) (b)
(c) (d) a l
d 2d l
Paragraph for Questions 13 and 14
In the figure, a container is shown to have a movable (without l
(c) (d) l
friction) piston on top. The container and the piston are all made a
of perfectly insulated material allowing no heat transfer between
outside and inside the container. The container is divided into SECTION - III
two compartments by a rigid partition made of a thermally This section contains 4 questions, each having two matching
conducting material that allows slow transfer of heat. The lower lists. Choices for the correct combination of elements from List-I
compartment of the container is filled with 2 moles of an ideal and List-II are given as options (a), (b), (c) and (d), out of which
monatomic gas at 700 K and the upper compartment is filled with one is correct.
2 moles of an ideal diatomic gas at 400 K. The heat capacities per
17. A person in lift is holding a water jar, which has a small hole
3 5 at the lower end of its side. When the lift is at rest, the water
mole of an ideal monatomic gas are CV R , CP R , and
2 2 jet coming out of the hole hits the floor of the lift at a distance
5 7
those for an ideal diatomic gas are CV R , CP R. d of 1.2 m from the person. In the following, state of the lift’s
2 2
motion is given in List-I and the distance where the water jet
hits the floor of the lift is given in List-II. Match the statements
from List-I with those in List-IIand select the correct answer
using the code given below the lists.
List - I List - II
P. Lift is accelerating 1. d = 1.2 m
vertically up
Q. Lift is accelerating 2. d > 1.2 m
13. Consider the partition to be rigidly fixed so that it does not vertically down with
move. When equilibrium is achieved, the final temperature of an acceleration less
the gases will be than the gravitational
(a) 550 K (b) 525 K acceleration
(c) 513 K (d) 490 K R. Lift is moving vertically 3. d < 1.2 m
14. Now consider the partition to be free to move without friction up with constant speed
so that the pressure of gases in both compartments is the S. Lift is falling freely 4. No water leaks out of the
same. The total work done by the gases till the time they jar
achieve equilibrium will be Code:
(a) 250 R (b) 200 R (a) P-2, Q-3, R-2, S-4 (b) P-2, Q-3, R-1, S-4
(c) 100 R (d) – 100 R (c) P-1, Q-1, R-1, S-4 (d) P-2, Q-3, R-1, S-1
JEE Advanced 2014 Solved Paper 2014-11

18. Four charges Q1, Q2, Q3 and Q4 of same magnitude are fixed 20. A block of mass m1 = 1 kg another mass m2 = 2 kg, are placed
along the x axis at x = – 2a, – a, + a and + 2a, respectively. A together (see figure) on an inclined plane with angle of
positive charge q is placed on the positive y axis at a distance inclination . Various values of are given in List-I. The
b > 0. Four options of the signs of these charges are given in coefficient of friction between the block m1 and plane is
List-I. The direction of the forces on the charge q is given in always zero. The coefficient of static and dynamic friction
List-II. Match List-I with List-II and select the correct answer
between the block m2 and the plane are equal to = 0.3. In
using the code given below the lists.
List-II expressions for the friction on block m2 are given.
q(0, b)
Match the correct expression of the friction in List-II with the
angles given in List-I, and choose the correct option. The
acceleration due to gravity is denoted by g.
[Useful information: tan (5.5°) 0.1; tan (11.5°) 0.2;
Q1 Q2 Q3 Q4 tan (16.5°) 0.3]
(– 2a, 0) (– a, 0) (+ a, 0) (+ 2a, 0)
List - I List - II
P. Q1, Q2, Q3, Q4 all 1. + x m1
positive m2
Q. Q1, Q2 positive; 2. – x
Q3, Q4 negative
R. Q1, Q4 positive; 3. + y
Q2, Q3 negative
S. Q1, Q3 positive; 4. – y List-I List-II
Q2, Q4 negative P. = 5° 1. m2g sin
Code: Q. = 10° 2. (m1 + m2)g sin
(a) P-3, Q-1, R-4, S-2 (b) P-4, Q-2, R-3, S-1 R. = 15° 3. m2g cos
(c) P-3, Q-1, R-2, S-4 (d) P-4, Q-2, R-1, S-3 S. = 20° 4. (m1 + m2)g cos
19. Four combinations of two thin lenses are given in List-I. The Code:
radius of curvature of all curved surfaces is r and the refractive
index of all the lenses is 1.5. Match lens combinations in (a) P-1, Q-1, R-1, S-3 (b) P-2, Q-2, R-2, S-3
List-I with their focal length in List-II and select the correct (c) P-2, Q-2, R-2, S-4 (d) P-2, Q-2, R-3, S-3
answer using the code given below the lists.
List - I List - II CHEMISTRY
SECTION - I
P. 1. 2r This section contains 10 multiple choice questions. Each question
has four choices (a), (b), (c) and (d) out of which ONLY ONE
option is correct.

1. For the identification of -naphthol using dye test, it is


r necessary to use
Q. 2.
2 (a) Dichloromethane solution of -naphthol
(b) Acidic solution of -naphthol
(c) Neutral solution of -naphthol
(d) Alkaline solution of -naphthol
R. 3. –r 2. Assuming 2s-2p mixing is NOT operative, the paramagnetic
species among the following is
(a) Be2 (b) B2
(c) C2 (d) N2
3. For the elementary reaction M N, the rate of disappearance
S. 4. r of M increases by a factor of 8 upon doubling the
concentration of M. The order of the reaction with respect to
M is
Code: (a) 4 (b) 3
(a) P-1, Q-2, R-3, S-4 (b) P-2, Q-4, R-3, S-1 (c) 2 (d) 1
(c) P-4, Q-1, R-2, S-3 (d) P-2, Q-1, R-3, S-4
EBD_780
12-2014 JEE Advanced 2014 Solved Paper
4. Isomers of hexane, based on their branching, can be divided 7. Hydrogen peroxide in its reaction with KIO4 and NH2OH
into three distinct classes as shown in the figure. respectively, is acting as a
(a) Reducing agent, oxidising agent
and (b) Reducing agent, reducing agent
and
(c) Oxidising agent, oxidising agent
(d) Oxidising agent, reducing agent
(I) (II) 8. The product formed in the reaction of SOCl2 with white
phosphorous is
(a) PCl3 (b) SO2Cl2
(c) SCl2 (d) POCl3
9. Under ambient conditions, the total number of gases released
(III) as products in the final step of the reaction scheme shown
below is
The correct order of their boiling point is
Complete
(a) I > II > III (b) III > II > I Hydrolysis
(c) II > III > I (d) III > I > II XeF6 P + Other product
5. The acidic hydrolysis of ether (X) shown below is fastest
when –
OH /H2O

Slow disproportionation
Acid in OH–/H2O
OR OH + ROH

Products
(a) 0 (b) 1
(c) 2 (d) 3
(a) One phenyl group is replaced by a methyl group 10. For the process
(b) One phenyl group is replaced by a para-methoxyphenyl H2O(l) H2O(g)
group at T = 100°C and 1 atmosphere pressure, the correct choice is
(c) Two phenyl groups are replaced by two para- (a) Ssystem > 0 and Ssurroundings > 0
methoxyphenyl groups (b) Ssystem > 0 and Ssurroundings < 0
(d) No structural change is made to X (c) Ssystem < 0 and Ssurroundings > 0
6. The major product in the following reaction is (d) Ssystem < 0 and Ssurroundings < 0

O 1. CH3MgBr, dry ether, 0°C


SECTION - II
Cl 2. aq. acid This section contains 3 paragraphs, each describing theory,
CH3 experiments, data etc. 6 questions related to the three paragraphs
with two questions on each paragraph. Each question has only
O one correct answer among the four given options (a), (b), (c) and
(d).
(a) H3C
CH3 Paragraph for Questions 11 and 12
X and Y are two volatile liquids with molar weights of 10 g mol–1
OH and 40 g mol–1 respectively. Two cotton plugs, one soaked in X
and the other soaked in Y, are simultaneously placed at the ends
of a tube of length L = 24 cm, as shown in the figure. The tube is
(b) H2C CH3 filled with an inert gas at 1 atmosphere pressure and a temperature
CH3 of 300 K. Vapours of X and Y react to form a product which is first
observed at a distance d cm from the plug soaked in X. Take X and
Y to have equal molecular diameters and assume ideal behaviour
for the inert gas and the two vapours.
(c) L = 24 cm
CH2
O

CH3 Cotton wool Cotton wool


soaked in X d Initial formation soaked in Y
(d) of the product
O CH3
JEE Advanced 2014 Solved Paper 2014-13

11. The value of d in cm (shown in the figure), as estimated from (d) It gives a positive iodoform test and is a geometrical
Graham’s law, is isomer of X
(a) 8 (b) 12
Paragraph For Questions 15 and 16
(c) 16 (d) 20
An aqueous solution of metal ion M1 reacts separately with
12. The experimental value of d is found to be smaller than the
estimate obtained using Graham’s law. This is due to reagents Q and R in excess to give tetrahedral and square planar
(a) Larger mean free path for X as compared to that of Y complexes, respectively. An aqueous solution of another metal
ion M2 always forms tetrahedral complexes with these reagents.
(b) Larger mean free path for Y as compared to that of X
Aqueous solution of M2 on reaction with reagent S gives white
(c) Increased collision frequency of Y with the inert gas as
compared to that of X with the inert gas precipitate which dissolves in excess of S. The reactions are
(d) Increased collision frequency of X with the inert gas as summarized in the scheme given below:
compared to that of Y with the inert gas Scheme:
Q R
Tetrahedral M1 Square planar
Paragraph For Questions 13 and 14 Excess Excess
Schemes 1 and 2 describe sequential transformation of alkynes M
Q R
and N. Consider only the major products formed in each step for Tetrahedral M2 Tetrahedral
both the schemes. Excess Excess

1. NaNH 2 (excess)
S, stoichiometric amount
2. CH3CH 2I (1 equivalent)
H X Scheme-1
3. CH3I (1 equivalent) R
HO M 4. H2, Lindlar’s catalyst White precipitate Precipitate
Excess
dissolves
1. NaNH 2 (2 equivalent) 15. M1, Q and R, respectively are
OH (a) Zn2+, KCN and HCl (b) Ni2+, HCl and KCN
2. Br (c) Cd2+, KCN and HCl (d) Co2+, HCl and KCN
Y Scheme-2 16. Reagent S is
H +
3. H 3O, (mild) (a) K4[Fe(CN)6] (b) Na2HPO4
N 4. H2, Pd/C (c) K2CrO4 (d) KOH
5. CrO3
SECTION - III
13. The product X is
H3CO This section contains 4 questions, each having two matching
lists. Choice for the correct combination of elements from List-I
and List-II are given as options (a), (b), (c) and (d), out of which
(a) one is correct.
H H 17. Match each coordination compound in List-I with an
H appropriate pair of characteristics from List- II and select the
correct answer using the code given below the lists.
H {en = H2NCH2CH2NH2; atomic numbers : Ti = 22; Cr = 24;
(b)
Co = 27; Pt = 78}
H3CO
List-IList-II
CH3CH2O P. [Cr(NH3)4Cl2]Cl 1. Paramagnetic and exhibits
ionisation isomerism
(c) Q. [Ti(H2O)5Cl](NO3)2 2. Diamagnetic and exhibits
cis-trans isomerism
H H R. [Pt(en)(NH3)Cl]NO3 3. Paramagnetic and exhibits
H cis-trans isomerism
S. [Co(NH3)4(NO3)2]NO3 4. Diamagnetic and exhibits
(d) H ionisation isomerism
Code:
CH3CH2O
P Q R S
14. The correct statement with respect to product Y is
(a) It gives a positive Tollens test and is a functional isomer
of X (a) 4 2 3 1
(b) It gives a positive Tollens test and is a geometrical (b) 3 1 4 2
isomer of X (c) 2 1 3 4
(c) It gives a positive iodoform test and is a functional (d) 1 3 4 2
isomer of X
EBD_780
14-2014 JEE Advanced 2014 Solved Paper
18. Match the orbital overlap figures shown in List-I with the description given in List-II and select the correct answer using the code
given below the lists.
List-I List-II

P. 1. p–d antibonding

Q. 2. d–d bonding

R. 3. p–d bonding

S. 4. d–d antibonding

Code:
P Q R S
(a) 2 1 3 4
(b) 4 3 1 2
(c) 2 3 1 4
(d) 4 1 3 2
19. Different possible thermal decomposition pathways for peroxyesters are shown below. Match each pathway from List-I with an
appropriate structure from List-II and select the correct answer using the code given below the lists.
P
R +R
– CO 2
O
O Q
R +R R + X + carbonyl compound
R O R
– CO 2
(Peroxyester)
R R + X + carbonyl compound
RCO2 + R O
– CO 2

S
RCO2 + R O R +RO
– CO 2

List-I List-II
O
O
P. Pathway 1. C 6H 5CH 2 O CH3

O
O
Q. Pathway 2. C6H5 O CH3

O
O CH3
R. Pathway 3. C6H5CH2 O CH3
CH2C6H5

O
O CH3
S. Pathway 4. C6H5 O CH3
C6H5
JEE Advanced 2014 Solved Paper 2014-15

Code:
P Q R S
(a) 1 3 4 2
(b) 2 4 3 1
(c) 4 1 2 3
(d) 3 2 1 4
20. Match the four starting materials (P, Q, R, S) given in List-I with the corresponding reaction schemes (I, II, III, IV) provided in
List-II and select the correct answer using the code given below the lists.
List - I List - II
P. H H 1. Scheme I

(i) KMnO4, HO , heat (ii) H , H2O
OH (iii) SOCl2 (iv) NH3
? ————————————— C7H6N2O3

Q. 2. Scheme II
OH (ii) Sn/HCl (ii) CH3COCl (iii) conc. H2SO4

(iv) HNO3 (v) dil. H2SO4, heat (vi) HO
NO2
? ————————————— C6H6N2O2

R. 3. Scheme III
(i) red hot iron, 873 K (ii) fuming HNO3, H2SO4, heat
(iii) H2S.NH3 (iv) NaNO2, H2SO4 (v) hydrolysis
NO2
? ————————————————— C6H5NO3

S. 4. Scheme IV
(i) conc. H2SO4, 60°C
(ii) conc. HNO3, conc. H2SO4 (iii) dil. H2SO4, heat
CH3
? ————————————————— C6H5NO4

Code:
P Q R S
(a) 1 4 2 3
(b) 3 1 4 2
(c) 3 4 2 1
(d) 4 1 3 2

MATHEMATICS 2. In a triangle the sum of two sides is x and the product of the
same sides is y. If x2 – c2 = y, where c is the third side of the
SECTION - I triangle, then the ratio of the in radius to the circum-radius of
the triangle is
This section contains 10 multiple choice questions. Each question
has four choices (a), (b), (c) and (d) out of which ONLY ONE 3y 3y
(a) (b)
option is correct. 2x x c 2c x c

1. Six cards and six envelopes are numbered 1, 2, 3, 4, 5, 6 and 3y


3y
cards are to be placed in envelopes so that each envelope (c) (d)
4x x c 4c x c
contains exactly one card and no card is placed in the
envelope bearing the same number and moreover the card 3. The common tangents to the circle x2 + y2 = 2 and the parabola
numbered 1 is always placed in envelope numbered 2. Then y2 = 8x touch the circle at the points P, Q and the parabola at
the number of ways it can be done is the points R, S. Then the area of the quadrilateral PQRS is
(a) 264 (b) 265 (a) 3 (b) 6
(c) 53 (d) 67 (c) 9 (d) 15
EBD_780
16-2014 JEE Advanced 2014 Solved Paper
4. Three boys and two girls stand in a queue. The probability, 10. The function y = f (x) is the solution of the differential equation
that the number of boys ahead of every girl is at least one
more than the number of girls ahead of her, is dy xy x4 2x
2
in (– 1, 1) satisfying f (0) = 0. Then
dx x 1 1 x2
1 1
(a) (b)
2 3
3
2 3 2
(c) (d)
3 4 f x d x is
5. The quadratic equation p(x) = 0 with real coefficients has 3
purely imaginary roots. Then the equation p(p(x)) = 0 has 2
(a) one purely imaginary roots
(b) all real roots 3 3
(a) (b)
(c) two real and two purely imaginary roots 3 2 3 4
(d) neither real nor purely imaginary roots 3 3
(c) (d)
6. For x 0, , the equation sinx + 2sin 2x – sin 3x = 3 has 6 4 6 2
(a) infinitely many solutions SECTION - II
(b) three solutions
This section contains 3 paragraphs, each describing theory,
(c) one solution
experiments, data etc. 6 questions related to the three paragraphs
(d) no solution with two questions on each paragraph. Each question has only one
correct answer among the four given options (a), (b), (c) and (d).
2
17 Paragraph for Questions 11 and 12
7. The following integral 2cosec x dx is equal to
Box 1 contains three cards bearing numbers 1, 2, 3; box 2 contains
five cards bearing numbers 1, 2, 3, 4, 5; and box 3 contains seven
4
cards bearing numbers 1, 2, 3, 4, 5, 6, 7. A card is drawn from each
log 1 2
of the boxes. Let xi be number on the card drawn from the ith box,
16 i = 1, 2, 3.
(a) 2 eu e u du
0 11. The probability that x1 + x2 + x3 is odd, is
29 53
log 1 2 (a) (b)
17
105 105
(b) eu e u du
0 57 1
(c) (d)
105 2
log 1 2 12. The probability that x1, x2, x3 are in an arithmetic progression,
17
(c) eu e u du is
0
9 10
(a) (b)
105 105
log 1 2
16
(d) 2 eu e u du 11 7
0 (c) (d)
105 105
8. Coefficient of x11 in the expansion of (1 + x2)4(1 + x3)7 (1 +
Paragraph For Questions 13 and 14
x4)12 is
Let a, r, s, t be nonzero real numbers. Let P (at 2, 2at), Q,
(a) 1051 (b) 1106
R (ar2, 2ar) and S (as2, 2as) be distinct points on the parabola y2
(c) 1113 (d) 1120
= 4ax. Suppose that PQ is the focal chord and lines QR and PK are
9. Let f : [0, 2] R be a function which is continuous on [0, 2]
parallel, where K is the point (2a, 0)
and is differentiable on (0, 2) with f (0) = 1. Let F(x) =
13. The value of r is
x2
f t dt for x 0, 2 . If F (x) = f (x) for all x 0, 2 , 1 t2 1
(a) (b)
0 t t
then F(2) equals
1 t2 1
(a) e2 – 1 (b) e4 – 1 (c) (d)
t t
(c) e – 1 (d) e 4
JEE Advanced 2014 Solved Paper 2014-17

14. If st = 1, then the tangent at P and the normal at S to the P Q R S


parabola meet at a point whose ordinate is (a) 3 2 4 1
2 2 (b) 2 3 4 1
t2 1 a t2 1
(a) (b) (c) 3 2 1 4
2t 3 2t 3 (d) 2 3 1 4
2 2 18. List - I List - II
a t2 1 a t2 2
(c) (d) 1
t3 t3 P. Let y x cos 3cos x , 1. 1
Paragraph For Questions 15 and 16
3
1 h x 1,1 , x . Then
a a 1 2
Given that for each a (0, 1), lim t (1 t ) dt exists. Let
h 0 h
this limit be g(a). In addition, it is given that the function g(a) is 1 d2y x dy x
differentiable on (0, 1). x2 1 2
x
y x dx dx
1
15. The value of g is
2 equals
(a) (b) 2 Q. Let A1, A2, ..., An (n > 2) 2. 2
be the vertices of a regular
(c) (d)
2 4 polygon of n sides with its
1
16. The value of g is
2 centre at the origin. Let ak

(a) (b) be the position vector of the


2 point Ak, k = 1, 2, ....., n.
(c) (d) 0
2 n 1
SECTION - III If ak ak 1
k 1
This section contains 4 questions, each having two matching
lists. Choice for the correct combination of elements from List-I
and List-II are given as options (a), (b), (c) and (d), out of which n 1
= ak . ak 1 ,
one is correct. k 1

17. List - I List - II


then the minimum value of n is
P. The number of polynomials 1. 8
f (x) with non-negative R. If the normal from the 3. 8
integer coefficients of point P(h, 1) on the ellipse
degree 2, satisfying f (0) = 0
x2 y2
1 1 is perpendicular
and f x dx 1, is 6 3
0
Q. The number of points 2. 2 to the line x + y = 8, then
the value of h is
in the interval 13, 13
S. Number of positive 4. 9
at which f (x) = sin(x2) + cos(x2) solutions satisfying the
attains its maximum value, is
2 1 1
3x2 equation tan
R. dx equals 3. 4 2x 1
2 1 ex
1 1 1 1 2
+ tan = tan is
2
1 x 4x 1 x2
cos 2 x log dx
1
1 x P Q R S
2 (a) 4 3 2 1
S. 4. 0
1 (b) 2 4 3 1
2
1 x (c) 4 3 1 2
cos 2 x log dx
1 x (d) 2 4 1 3
0
EBD_780
18-2014 JEE Advanced 2014 Solved Paper
19. Let f1 : R R, f 2 : 0, R, f3 : R R and 2k 2k
20. Let zk cos i sin ; k = 1, 2, ...., 9.
10 10
x if x 0,
f4 : R 0, be defined by f1 x List-I List-II
x
e if x 0; P. For each zk there exists 1. True
as zj such that zk. zj = 1
sin x if x 0,
f2(x) = x2; f3 x Q. There exists a 2. False
x if x 0;
k 1, 2,...,9 such
f 2 f1 x if x 0, that z1.z = zk has no
and f 4 x solution z in the set
f 2 f1 x 1 if x 0.
of complex numbers
List-I List-II
1 z1 1 z2 .... 1 z9
P. f4 is 1. Onto but not one-one R. 3. 1
Q. f3 is 2. Neither continuous nor 10
one-one equals
R. f2of1 is 3. Differentiable but not 9
one-one 2k
S. 1 cos equals 4. 2
S. f2 is 4. Continuous and one- k 1
10
one P Q R S
P Q R S (a) 1 2 4 3
(a) 3 1 4 2 (b) 2 1 3 4
(b) 1 3 4 2 (c) 1 2 3 4
(c) 3 1 2 4 (d) 2 1 4 3
(d) 1 3 2 4
JEE Advanced 2014 Solved Paper 2014-19

SOLUTIONS
Paper - 1
PHYSICS V2
2. (b, d) H 4 ...(i)
R
r0
1. (c) 1 Q 4 l
+Q E1 = where R
4 0 r0
2
d2
r0 When resistances are connected in series
+ 1 2
+ E2 = 4 l R R
+ 4 r0
0 Total resistance = R1 + R2 = 2 2
=2
4 d 4 2
r0
++ E3 = V2
++ H t2 ...(ii)
++
0
R/2
From (i) and (ii) t2 = 2 min. Therefore (b) is correct.
When resistance are connected in parallel
E1 = E2 (Given)
1 Q 1 2 R1R 2 R12 R/4 R
Total resistance =
4 2 4 r0 R1 R 2 2R1 2 8
0 r0 0

Q = 2 r0 ...(1) V2
H t2 ...(iii)
E2 = E3 (Given) R /8
From (i) and (iii) t2 = 0.5 min
1 2
r0 (d) is correct
4 0 r0 2 0 3. (a, c)
(b) is incorrect There will be no effect of the transparent thin film of
uniform thickness and refractive index n 1 = 1.4
E1 = E3 (Given)
n1 n2 n2 n1
1 Q Therefore, [For case (i)]
Q 2 r02 u v R
4 2 2
0 r0 0
1 1.5 1.5 1
(a) is incorrect f1 = 3R
f1 R
1 4Q (a) is a correct option
Now E1 (r0/2) =
4 0 r02 n1 n2 n2 n1
Again applying [For case (ii)]
u v R
1 4 2 r0 1 8
1.5 1 1 1.5
4 0 r02 4 0 r0 f2 2R
f2 R
(c) is a correct option.
1 4 1 8
and 2E2 (r0/2) = 2
4 0 r0 4 0 r0 v RT 1
4. (d) Here, v = × 4l
(c) is correct
4lM 10 3 4l
v = 336.7 m/s to 346.5 m/s
1 2 1 4 For monatomic gas =1.67
E2 (r0/2) =
4 0 0 /2
r 4 0 r0 0 r0 RT 10
v 100 RT
3 M
4 2 2 M 10
4E3 (r0/2) =
2 0 0 0 r0
10 10
= 167RT 640
(d) is incorrect. M M
EBD_780
20-2014 JEE Advanced 2014 Solved Paper
O P
10 7 6. (a, b, d) i
For Neon M = 20 and
20 10
7 V1 R1
v 640 = 448 ms–1
10 R2
(a) is incorrect N Q
10 17
For Argon M = 36, V2
36 32
R3 i
17
v = 640 = 340 ms–1 M A
32
Applying KVL in MNOPQAM
(d) is the correct option.
For diatomic gas = 1.4 V1 iR1 V2 iR 3 0

10 10 V1 V2
v 140RT 590 i ... (i)
M M R1 R 3
Applying KVL in NOPQN
10 9
For Oxygen V
32 16 v1 iR1 0 i ...(ii)
R1
9
v 590 = 331.87 ms–1
16 V1
V1 V2
(c) is incorrect From (i) & (ii)
R1
R1 R 3
10 3 V1R1 + V1R3 = V1R1 + V2R1
For Nitrogen V1R3 = V2R1
28 5
If V1 = V2 then R1 = R3 = R2 (a) is correct option.
3 If V1 = V2 then R1 = R3 = 2R2 (b) is correct option.
v 590 = 354 ms–1
5 (R2 can have any value as there is no current flowing
(b) is incorrect through d)
5. (c, d) When 1 0 and 2 0 If V1 = 2V2 then 2R3 = R1 (c) is incorrect option.
If 2V1 = V2 then R3 = 2R1 = R2 (d) is correct option.
7. (a, b, c)
N1 f1 = 1N1 D
We know that
d
1 As 2 > 1 2> 1 (a) is correct option.
Therefore m1 > m2 (b) is correct option.
N2
mg 2D 2 5 1 1 D
2 As 3
d 2 d
P f2 = 2N2 3 × 600 = 4.5 × 400 (c) is correct option.
N1 = 2N2 [... horizontal equilibrium]
The angular width = (d) is incorrect option.
mg = N2 + N1 [... vertical equilibrium] d
Solving the above equation we get 8. (c, d) I = cos 500 t
mg I
N2
1 1 2
(c) is the correct option. 1
When 1 = 0 7
Taking torque about P we get 6
t
l
mg cos N1 l sin 2
2
mg –1
N1 tan
2
(d) is correct
JEE Advanced 2014 Solved Paper 2014-21

B D
10. (a, d) C1
A
K
50 V A/3 Q1 E1
C = 20 F

R = 10 E2
2A/3 Q2
7
Till t , the charge will be maximum at C2 = C – C1
6 2
d
/2 /2
sin 500t
Q cos 500t dt = This is a combination of two capacitors in parallel.
500 0 Therefore
0
C = C1 + C2 C2 = C – C1
1 1
= sin 500 C kA 2A
500 2 500 500 where C1 and C C1
3 0d 3 0d
(a) is incorrect
C C1 2
7
From the graph it is clear that just before t , the C1 k
6
current is in anticlockwise direction. C 2
1
(b) is incorrect C1 k
7
At t , the charge on the upper plate of capacitor C 2
6 1
C1 k
is

7
C 2
1
6 C1 k
1 7
cos 500t dt sin 500
500 6 500 (d) is a correct option.
0
kA
1 1 Now, Q1 = C1V = ×V
10 3 C 3 0d
500 2
2A
Now applying KVL (when A is just connected to D) and Q2 = (C – C1)V = V
3 0d
3
10
50 i 10 0 i = 10 A Q1 k
6
20 10 (c) is incorrect
Q2 2
(c) is the correct option.
The maximum charge on C is Q = CV = 20 × 10–6 × 50 Also V = E × d
=10–3C V
Therefore, the total charge flown = 2 × 10 –3 C E E1 E2 (a) is a correct option
d
(d) is the correct option.
9. (a, c, d) I Ig V
11. (5) S R
Clearly in the given situation a displacement node is Ig Ig
present at x = 0 and a displacement antinode is present
at x = 3 m. Therefore, y = 0 at x = 0 and y A at 1.5 0.006 2n 30
4990
x = 3 m. 0.006 249 0.006
n 5
The velocity v = 100 ms–1. x y
k 12. (3) d S fz
a, c and d are the correct options which satisfy the
above conditions. M 0 L1T 0 M x L 3x M yT yT z
EBD_780
22-2014 JEE Advanced 2014 Solved Paper

M 0 L1T 0 M x y L 3xT y z µ0 1 1
x + y = 0, – 3x = 1 2I
R1 4 X1 X0 X1
1 1 R2 µ0 1 1
x and y = 2I
3 3 4 X1 X0 X1
n=3
X0 X1 X1 X0
FL = X
13. (4) Y 0 X1 X1 X0 2 X1
a l
Here F, a and L are accurately known. X0
R1 X1 3
Y L 1.0 10 5 3
100 100 = 100 = 4% R2 X0 3 2
Y l 25 10 5 2
X1
14. (4) By conservation of angular momentum
2 (mvr) = I 19. (5) vA
1
2 × 0.05 × 9 × 0.25 = × 0.45 × (0.5)2 ×
2 vB cos 30°
vB
= 4 rad s–1 vB sin 30°
30°
1 30° 60°
15. (2) 3 F r I
2
Here
1 1 vA = vB cos 30°
3 0.5 0.5 = 1.5 0.5 0.5
2 2
3
= 2 rad s–1 100 3 vB
2
= 0+ t = 0 + 2 × 1 = 2 rad s–1
vB = 200 ms–1
16. (5) Work done = Increase in potential energy
+ gain in kinetic energy displacement
Time
F × d = mgh + gain in K.E. velocity
18 × 5 = 1 × 10 × 4 + gain in K.E.
500 500
Gain in K.E. = 50 J = 10n t0 5 sec
vB sin 30 200 sin 30
n=5
17. (2) Applying first law of thermodynamics to path iaf 20. (8)
Qiaf = Uiaf + Wiaf 0.3 ms –1 0.2 ms –1 a = 2 ms –2
A x
500 = Uiaf + 200 Uiaf = 300 J B
Now,
Qibf = Uibf + Wib + Wbf 4m
= 300 + 50 + 100 For ball A
Qib + Qbf = 450 J ...(1) u1 0.3 ms 1 , a1 2ms 2 , s1 = x, t1 = t
Also Qib U ib Wib
1 2
Qib = 100 + 50 = 150 J ...(2) s1 u1t1 a1t1
2
Qbf 300 x = 0.3t – t2 ...(1)
From (1) & (2) 2
Qib 150 For ball B
u2 = 0.2 ms–1, a2 = 2ms–2, s2 = 4 – x, t2 = t
mv 1
18. (3) R a 2 t 22
qB s2 = u2t2 +
2
4 – x = 0.2 t + t2 ...(2)
R1 B2
R2 B1 [... m, q, v are the same] From (1) and (2) t = 8 sec
JEE Advanced 2014 Solved Paper 2014-23

CHEMISTRY OH
1. (b, c, d) Cu2S + 2Cu2O 6Cu + SO2 c
a
Cu2S + 2CuO 4Cu + SO2
Cu2S + CuSO4 3Cu + 2SO2 C(CH3)3
b
2. (a, b) Salt bridge is introduced to keep the solutions of two
electrodes separate, so that the ions in electrodes do 6. (a, b, c) Na + NH3 (excess) Dilute solution of Na in liq.
not mix freely with each other. Salt bridge stops the NH3 Paramagnetic
diffusion of ions from one electrode to another. It is K + O2 (excess) KO2 (O2– is paramagnetic)
not necessary for occurrence of cell reaction, as we Cu + HNO3 (dil.) Cu(NO3)2 + NO
know that there is no salt bridge in lead accumulator, (NO is paramagnetic)
but still reactions take place. 2-Ethylanthraquinol + O2
3. (a, b, d) In ice, water molecules are excessively H-bonded 2-Ethylanthraquinone + H2O2
giving a cage-like structure which is lighter than water.
(H2O2 is diamagnetic)
Primary amines are more basic than tertiary amine,
because the protonated 1° amines are extensively H- 7. (a, b, d) Balanced chemical equation is
bonded and hence more stable than the corresponding 1 0
protonated 3° amines. 6I ClO3 6H 2SO 4 Cl 6HSO4 3I 2 3H 2O

H H 8. (a)
R — NH 2 R — NH3 ; R 3 N R 3 NH — NH 2 group is acetylated by acetic anhydride in
methylene chloride (solvent). Note that —CONH2
H OH 2 group does not undergo acetylation because here lone
+
+ pair of electrons is delocalised.
R N H OH 2 > R3NH OH2
H OH 2 COCH3
CH 2Cl2
More stable Less stable CH2 NH2 + (CH3CO)2O CH2 NH + CH3COOH
Acetic acid undergoes dimerisation in benzene.
O O–
O H O +
H3C C C CH3 C NH2 C NH2
O H O
4. (a, c, d) Isomeric alcohols with molecular formula C4H10O 9. (a, b, c) Since the vessel is thermally insulated, q = 0
are Further since, Pext = 0, so w = 0, hence U = 0
Since T = 0, T2 = T1, and P2V2 = P1V1
However, the process is adiabatic irreversible, so we
OH
can’t apply P2V2 = P1V1 .
n-Butanol OH 10. (b, d)H3BO3 does not undergo self ionization. However, it
(Buta-1-ol)
Butan-2-ol acts as a weak acid in water (hence it is a weak
electrolyte in water).
OH
OH H3 BO3 H 2O B(OH)4 H
lsobutyl alcohol tert-Butanol
(2-methylpropan-1-ol) (2-methylpropan-2-ol)
Addition of cis-diols (e.g., ethylene glycol) to aqueous
solution of orthoboric acid leads to complex formation,
thus acidity of aqueous solution of orthoboric acid is
5. (a, b, c) —OH group is strongly activating and o, p-directing
increased.
due to +M effect. Thus positions a, b and c are the

sites for attack by an electrophile. However, sites b CH2 — OH CH2 — O O — CH2

and c are not preferred by bulky electrophile due to B(OH)4 + 2 B + 4H2O
CH2 — OH CH2 — O O — CH2
steric crowding. Thus more bulky electrophile (like I2)
can attack only site a, which is least sterically hindered, It arranges to planner sheets due to H-bonding.
a bit smaller electrophile (Br2) can attack at sites a and 11. (1) On hydrolysis, the given peptide gives only one
also b (relatively less sterically hindered site) and the naturally occurring amino acid (glycine).
smallest electrophile (Cl2) can attack all the three sites,
viz., a, b and c (most sterically hindered site).
12. (2) MX2 M2 2X
1 2
EBD_780
24-2014 JEE Advanced 2014 Solved Paper

obser. depression in f.pt of aq. solution –


i=
depression of f.p.t. in absence of ionic dissociation
F F
BrF4– = Br Square planar (sp3d2)
1 2 F F
= 1 2 = 1 + 2 × 0.5 = 2
1
13. (4) R = NA × k
= 6.023 × 1023 × 1.380 × 10–23 H3N NH3
2+

= 8.312 which has 4 significant figures [Cu(NH3)4]2+ = Cu


14. (8) H3N NH3
wt. of solute in 1 L of solution 1000 Square planar (dsp2)
Molality = wt. of solvent in 1 L of solution mol. wt. of solute 2–
Cl
Calculation of wt. of solvent Cl
1 mL of solvent = 0.4 g [FeCl4]2– = Fe Tetrahedral (sp3)
1000 mL of solvent = 400 g Cl Cl
Calculation of wt. of solute 2–
Cl
1000 mL of solution contain = 3.2 × 80 g solute = 256 g
Cl
256 1000 [CoCl4]2– = Co Tetrahedral (sp3)
Molality = 8 Cl
400 80 Cl

ml 1 means ml can be +1 and –1. 2–


15. (6) Cl Cl
So, for n = 4, six orbitals are possible and each has 1 [PtCl4]2– = Pt Square planar (dsp2)
Cl Cl
1
. So total number of electrons = 6.
electron with s = 18. (5) General molecular formula for ketones is CnH2nO
2
CnH2nO = 100 or 12n + 2n + 16 = 100, n = 6
16. (7) K2Cr2O7 + KI + H2SO4 K2SO4 + Cr2(SO4)3 Possible isomeric ketones with 6 carbon atoms are
+ I2 + H2O O O
2CuSO4 + KI 2CuI + I2 + 2K2SO4
H2O2 + 2KI 2KOH + I2 CH3CH2CH2CH2—C—CH3 CH3CH2CH2—C—CH2CH3
Cl2 + 2KI 2KCl + I2 (I) (II)
O3 + H2O + 2KI 2KOH + O2 + I2 O
FeCl3 + 2KI 2KCl + FeCl2 + I2
HNO3 + KI KNO3 + I2 + NO CH3 O CH3—CH—CH2—C—CH3
*
CH3CH2CH—C—CH3 CH3
(III) (IV)
F F
17. (4) XeF4 = Xe Square planar (sp3d2) CH3 O
F F CH3 O CH3—C — C—CH3
CH3—CH—C—CH2CH3 CH3
F (V) (VI)
F
Note that only isomer III has a chiral carbon so on
SF4 = S See-saw (sp3d)
F reduction with NaBH4 it will give diastereomeric
F alcohols, while all other five isomers will give racemic
mixture.
F
F 19. (3) Cl CH3
SiF4 = Si Tetrahedral (sp3)
Br CH3 Cl Br
F F
Br Cl Br Cl

F
F CH3 CH3
BF4– = B Tetrahedral (sp3)
Following three conformers (with 0) are possible
F F
JEE Advanced 2014 Solved Paper 2014-25

2. (a, d) Let f and g be maximum at c1 and c2 respectively,


CH3 CH3 CH3
Br c1 , c2 0,1
Cl CH3 Br Cl CH3
Then, f (c1) = g(c2)
Br Cl Let h(x) = f (x) – g(x)
Br Cl Br Cl
Then, h(c1) = f (c1) – g(c1) > 0
CH3 Cl Br and h(c2) = f (c2) – g(c2) < 0
h(x) = 0 has atleast one root in (c1, c2)
20. (7) All except MnS (buff coloured) and SnS2 (yellow) are
C (c1, c2)
black in colour.
i.e. for
f (c) = g(c)
MATHEMATICS which gives A and D are correct options.
1. (a,c) g(x) may be discontinuous at x = a or x = b. a b
Let us check the continuity of g(x) at x = a and x = b 3. (c,d) Let M where a, b, c are integers.
b c
lim g x 0
x a a b
M is invertible if 0 ac b2
x a
b c
lim g x lim f t dt f t dt 0
x a x a a a b
a
Then b c
a=b=c ac = b2.
a
(a) is not correct.
g a f t dt = 0
a If b c a b b=a=c
g (x) is continuous at x = a ac = b2
x b (b) is not correct.
Also lim g x lim f t dt f t dt a 0
x b– x b– a a If M = , then M ac 0
0 c
b M is invertible.
lim g x f t dt (c) is correct
x b a
2
As ac integer ac b2
b
g b f t dt (d) is correct.
a
g(x) is continuous at x = b 4. (a, b, c) x y z 2
Hence g(x) is continuous x R

0, x a Angle between each pair is


3
Now g (x) = f x , a x b
0, x b
a x y z

g a 0 and g a f a

= x z y x y z
g b– f b and g b 0

As f (a), f (b) 1, f (a), f(b) 0


= 2. 2 cos y 2. 2 cos z
3 3
Hence g a g a and g b g b

g is not differentiable at a and b. = y z


EBD_780
26-2014 JEE Advanced 2014 Solved Paper
–( + )+ – =0
b y z x =0
dr’s of PQ are 0, 0, – 1
and dr’s of PR are , , + 1
= y.x z y. z x ( – l) ( + 1) = 0 = 1 or – 1
QPR 90
But for = 1, we get point Q itself
2. 2.cos z 2. 2.cos x we take = – 1
= 3 3 6. (a,b) Given MN = NM, M N2 and M 2 = N 4.
Then M 2 = N 4 (M + N 2) (M – N 2) = 0
= z x (i) M + N 2 = 0 and M – N 2 0
(ii) |M + N 2| = 0 and |M – N 2| = 0
In each case |M + N 2| = 0
Now b . z z.z x. z
|M 2 + MN 2| = |M | |M + N 2| = 0
= (2 – 1) = (a) is correct and (c) is not correct.
Also we know if |A| = 0, then there can be many matrices
b b.z z x is correct U, such that AU = 0
(M 2 + MN 2)U = 0 will be true for many values of U.
Hence (b) is correct.
Also a . y y. y z.y = (2 – 1) = Again if AX = 0 and |A| = 0, then X can be non-zero.
(d) is not correct.
a a.y y z is also correct x 1
t dt
t
7. (a, c, d) f x e
t
1/ x
a.b y. z y.x z.z z.x
1
x 1 1
x x 2 x
e x x e x
= (1 – 1 – 2 + 1) = – =– a. y b.z f x e =
x
x x2
(c) is correct.
For x 1, , f x 0
a.y z y z y a
f is monotonically increasing on 1,
(d) is not correct. (a) is correct.
5. (c) Lines are x = y, z = 1
For x 0,1 , f ( x ) 0
x 0 y 0 z 1
or ...(1)
1 1 0 (b) is not correct
and y = – x, z = – 1
x 1 dt 1/ x 1 dt
1 t t
x 0 y 0 z 1 t t t t
or ...(2) f x f e e =0
1 1 0 x
1/ x x
Let Q ( , , 1) and R (– , , – 1)
Direction ratios of PQ are – , – , – 1 (c) is not correct.
and direction ratios of PR are + , – , + 1
... PQ is perpendicular to line (1) 1
Replacing x by 2x in f (x) + f =0
x
P
x
We get f (2x) + f 2 =0
Q R
or f (2x) = – f (2–x)
f (2x) is an odd function.
(d) is correct.
1 2
JEE Advanced 2014 Solved Paper 2014-27

Required circle is
8. (a, b, c) f : , R x + y 2 + 14x – 2y + 1 = 0
2
2 2
With centre (– 7, 1) and radius = 7
f (x) = [log(sec x + tan x)]3 (b) and (c) are correct options.
f (– x) = [log(sec x – tan x)]3
10. (b,d) f x x5 5 x a
3
sec x tan x sec x tan x
= log f x 0 x5 5 x a 0
sec x tan x
a = 5x – x5 = g(x)
3
1 3 g(x) = 0 when x = 0, 51/ 4 51/ 4
= log log sec x tan x
sec x tan x
and g x 0 x = 1, – 1
3 Also g (– 1) = – 4 and g(1) = 4
= – log sec x tan x f x
graph of g(x) will be as shown below.
f is an odd function. From graph
(a) is correct and (d) is not correct.
Also

2 sec x tan x sec 2 x 4


f x 3 log sec x tan x .
sec x tan x

2
= 3sec x log sec x tan x 0 x ,
2 2
1/4
1/4 –1 O 5
–5 1
f is increasing on ,
2 2
We know that strictly increasing function is one one.
–4
f is one one
(b) is correct.
3
Also lim log sec x tan x if a 4, 4
x
2
then g(x) = a or
f (x) = 0 has 3 real roots
3 If a > 4 or a < – 4
and lim log sec x tan x
then f(x) = 0 has only one real root.
x
2 (b) and (d) are the correct options.
Range of f = (– , ) = R 2 2
f is an onto function. 11. (8) y x5 x 1 x2
(c) is correct.
9. (b, c) Let the equation of circle be dy 2
2 y x5 5 x4 1 x2 2 x 1 x 2 .2 x
x2 + y2 + 2gx + 2 f y + c = 0 dx
It passes through (0, 1) At point (1, 3)
1 + 2f + c = 0 ...(i)
This circle is orthogonal to (x – 1) 2 + y 2 = 16 dy 2
2 3 1 5 1 1 2 1 1 .2
i.e. x2 + y2 – 2x – 15 = 0 dx
and x2 + y2 – 1 = 0
dy
We should have 8
dx
2g (– 1) + 2f (0) = c – 15
or 2g + c – 15 = 0 ...(ii) 12. (3) We have f : 0, 4 0,
and 2g(0) + 2f (0) = c – 1
1
or c = 1 ...(iii) f x cos cos x
Solving (i), (ii) and (iii), we get
c = 1, g = 7, f = – 1 10 x x
and g x 1
10 10
EBD_780
28-2014 JEE Advanced 2014 Solved Paper
15. (6) Let the point P be (x, y)
x y x y
Then d1 P and d2 P
1 2 2
For P lying in first quadrant x > 0, y > 0.
Also 2 d1(P) + d2(P) 4

x y x y
2 3 10 4 2 4
2 2
The graph of y = f (x) and y = g (x) are as follows.
Clearly f(x) = g(x) has 3 solutions. x y x y
If x > y, then 2 4
2
1 x
ax sin x 1 a 1 x 1 or 2 x 2 2
13. (2) lim
x 1 x sin x 1 1 4 If x < y, then
y x x y
1 x 2 4
a 1 x sin x 1 2
lim
x 1 x 1 sin x 1
or 2 y 2 2
The required region is the shaded region in the figure
1 x given below.
sin( x 1)
a
lim x 1
x 1 sin x 1 y
1
x 1 y=x

2
a 1 1 y=2 2
2 4
a = 0 or 2
y= 2
Largest value of a is 2.
x 1, x 0
14. (3) f x x 1
x 1, x 0
g(x) = x2 + 1 x
O
x= 2 x=2 2
y = g(x)
2 2
Required area = 2 2 2 = 8 – 2 = 6 sq units.
16. (7) ...n1, n2, n3, n4 and n5 are positive integers such that
y = f(x) n1 < n2 < n3 < n4 < n5
Then for n1 + n2 + n3 + n4 + n5 = 20
If n1, n2 , n3 , n4 take minimum values 1, 2, 3, 4
respectively then n5 will be maximum 10.
Corresponding to n5 = 10, there is only one solution
n1 = 1, n2 = 2, n3 = 3, n4 = 4.
Corresponding to n5 = 9, we can have, only solution
n1 = 1, n2 = 2, n3 = 3, n4 = 5 i.e., one solution
Corresponding to n5 = 8, we can have, only solution
n1 = 1, n2 = 2, n3 = 3, n4 = 6
From graph there are 3 points at which h(x) is not
or n1 = 1, n2 = 2, n3 = 4, n4 = 4
differentiable.
i.e., 2 solution
JEE Advanced 2014 Solved Paper 2014-29

For n5 = 7, we can have 19. (4) ... a, b, c are in G.P

n1 = 1, n2 = 1, n3 = 4, n4 = 6 b = ar and c = ar2
or n1 = 1, n2 = 3, n3 = 4, n4 = 5 i.e. 2 solutions
b
For n5 = 6, we can have Also is an integer
a
n1 = 2, n2 = 3, n3 = 4, n4 = 5
r is an integer
i.e., one solution ... A.M. of a, b, c is b + 2
Thus there can be 7 solutions.
a b c
1
d2 b 2
3 2 5 3
17. (2) 4x 1 x dx
2
0 dx
a ar ar 2 3ar 6
1 1
3 d
1 x2
5 d
1 x2
5
.12 x 2 dx
a r 2 2r 1 6
= 4x
dx 0
dx
0
2
a r 1 6
2 2 51 1
2 5 ... a and
r are integers
= 12 x 1 x 1 x .24 xdx
0 0
The only possible values of a and r can be 6 and 2
respectively.
1
= 12 1 x
2 5
. 2 x dx a2 a 14 36 6 14 28
Then = =4
0 a 1 6 1 7
20. (5) Number of adjacent lines = n
6 1 Number of non adjacent lines = nC2 – n
1 x2 1
= 12 12 0 2
nC – n = n
n n 1
6 6 2 – 2n = 0
2
0
n 2 5n 0 n = 0 or 5
1
18. (4) a.b b.c c.a cos But n 2 n=5
3 2
S(4) : 1, Z1, Z2, ... Z9 are 10th roots of unity.
Z10 – 1 = 0
Given p a q b r c a b b c From equation 1 + Z1 + Z2 + .... + Z9 = 0
Re (1) + Re (Z1) + Re (Z2) + .... + Re(Z9) = 0
Taking its dot product with a , b , c , we get
Re (Z1) + Re (Z2) + ..... Re(Z9) = – 1

1 1 9
2k
p q r a b c ...(1) cos 1
2 2 K 1 10

1 1 9
2k
p q r 0 ...(2) 1 cos 2
2 2 10
K 1
1 1 Hence (c) is the correct option.
p q r a b c ...(3)
2 2
From (1) and (3), p = r Using (2) q = – p

p2 2q 2 r2 p2 2 p2 p2
4
2 2
q p
EBD_780
30-2014 JEE Advanced 2014 Solved Paper

Paper - 2
PHYSICS R l 100 l

2
R l 100 l
2 2
Cu Z Mo 1 42 1 41
1. (b) 2.14 R 0.1 0.1
2 29 1 28
Mo ZCu 1 60 40 60
R 0.25
Z b here b 1 Therefore, R 60 0.25
2
Z 1 5. (d) h N
1 2
Z 1 v
mg sin
hC 1
2. (a) W mu12 mg cos mg
1 2

hC 1
and W mu22
2 2
Dividing the above two equations, we get As the bead is moving in the circular path
hC
W mv 2
mg cos N
1 u12 R
hC u22
W mv 2
2 N mg cos ...(1)
R
1240 1 2
W By energy conservation, mv mg R R cos
248 4 2
1240 1
W v2
310 2 g 1 cos ..(2)
R
1240 4 1240 From (1) and (2)
W 4W
248 310 N mg cos m 2 g 2 g cos
W = 3.7 eV
3. (a) In steady state N mg cos 2mg 2mg cos
Energy lost = Energy gained N 3mg cos 2mg
4
T T04 4 R 2
I R 2 N mg 3cos 2
Clearly N is positive (acts radially outwards) when
4
5.7 10 8 T 4 300 4 912 2
cos
T = 330 K 3
4. (c) In case of a meter bridge 2
Similarly, N acts radially inwards if cos
R X 3
Here X = 90 , l = 40.0 cm
l 100 l
6. (b) Rp
Xl Rp
R 10
100 l M 5
For finding the value of R
Rp
90 40
R 60
60
For finding the value of R
JEE Advanced 2014 Solved Paper 2014-31

Re 1 2Q 1 Q/2
Rp 6 105 m E2 E· ·
2 ; 3
10 4 0 R 4 0 R2
The mass of the wire = 10–3 × 1.2 × 105 = 120 kg Clearly E2 > E1 > E3
Let gpM be the acceleration due to gravity at point M where Q/2 is the charge enclosed in a sphere of radius
which is the mid point of the wire and is at a depth of R concentric with the given sphere.
Rp
.
10 4Q Q'
Let gp be the acceleration due to gravity at the surface 4 3 4 3
2R R
of the planet. 3 3
4 9. (c)
gp GRP n2
3
11.54 mm
4 2
ge = GRE
3 P
gp Rp 1 10 mm
ge RE 10 n1

10
gp 1 ms –2 Applying Snell’s law at point P
10
n1 sin = n2sin 90°
R p /10 11.54 / 2
and g pM gp 1 1 1 0.1 0.9 ms 2 n2 2.72
Rp 2
2 11.54
10
Force = mass of wire × gpM = 120 × 0.9 = 108 N 2
n2 = 1.36
1 2 1 2 1 2
7. (d) 10. (b) K.E. = mv = m u at = m 0 gt
C 2 2 2
R
2 1
K .E mgt 2 K .E t 2 ...(1)
+ 2
M
b 2 P First the kinetic energy will increase as per eq (1). As
the balls touches the ground it starts deforming and
loses its K.E. (K.E. converting into elastic potential
energy). When the deformation is maximum, K.E. = 0.
h The ball then again regain its shape when its elastic
potential energy changes into K.E. As the ball moves
up it loses K.E. and gain gravitational potential energy.
These characteristics are according to graph B.

11. (c) B
b
In PCM cos ...(1) B2 sin
2 R B1 sin

2S B2 B1
Also P0 h g P0
R (90º – )

2S 2S B1cos + B2 cos
h cos 2
R g b g a2 h2
h
1 Q
8. (c) E1 · ;
4 0 R2 Wire 1 a a Wire 2
EBD_780
32-2014 JEE Advanced 2014 Solved Paper
Magnetic field due to current carrying loop = 15. (c) From principle of continuity,
Magnetic field due to straight wires a1 v 1 = a 2 v 2
B = B1cos + B2cos = 2 B1cos
r12 v1 r22 v2
2
0 Ia 0I a 2 2
= 2 20 5 1 v2
2 2 3/ 2 2 a2
h 2
a 2
h 2
2 a h
v2 = 2000 mms–1 = 2 ms–1
h 1.2a
16. (a) A B
The current is from P to Q in wire 1 and from R to S in
wire 2. C
12. (b) We know that torque

M B
= MB sin 1 2
PA PB a va
2
0I
= I a2 2 sin 30
2 d 1 2
PC PB l vl
2
2 2
a 0I But PC = PA
=
2d
13. (d) Heat lost by monatomic gas at constant volume 1 2 1 2 a
l vl a va vl va
= Heat gained by diatomic gas at constant pressure 2 2 l

nCv1 700 T nC p2 T 400


a
Volume flow rate
3 7 l
R 700 T R T 400
2 2
17. (c)
2100 3T 7T 2800
10T = 4900
T = 490 K h2
14. (d) In this case both the gases are at constant pressure.
v = 2gh2
nC p1 700 T nC p2 T 400

5 7 h1
R 700 T R T 400
2 2
3500 – 5T = 7T – 2800
2gh2 × 2h1
12T = 6300 g
T = 525 K
= 2 h1h2
Applying first law of thermodynamics
W1 U1 Q1 If geff > g
geff = g
and W2 U2 Q2 geff < g
As the gas two system is thermally insulated, therefore
In all the three cases d 2 h1h2 1.2 m
Q1 Q2 0
If geff = 0, then no water leaks out
W1 W2 U1 U2 18. (a) If Q1, Q2, Q3 and Q4 are all positive, then the force will
be along + y-direction.
= nCv1 525 700 n2 Cv2 525 400
If Q1, Q2 are positive and Q3, Q4 are negative the force
3R 5R will act along + x-direction.
= 2 175 2 125 If Q1, Q4 are positive and Q2, Q3 are negative then
2 2
attractive force will dominate repulsive force and the
= –525R + 625R = – 100 R
force will be along – y direction.
Therefore, total work done = –100 R
JEE Advanced 2014 Solved Paper 2014-33

At = 11.5° the bodies are on the verge of moving,


1 1 1 f = 0.6 g
19. (b) For (P) 1
f R1 R2 At > 11.5° the bodies start moving and f = 0.6 g
The above relationship is true for (d).
2 1
1.5 1 f r
r r
CHEMISTRY
For the combination
1. (d) In dye test, phenolic — OH group is converted to
1 1 1 1 1 2
— O– which activates the ring towards electrophilic
F f1 f2 r r r aromatic substitution
2. (c) Be2 = 1s2 * 1s2 2s2 *2s2
r
F B2 = 1s2 * 1s2 2s2 * 2s2 2pz2
2
C2 = 1s2 *1s2 2s2 * 2s2 2pz2 2px1 2py1
1 1 1 N2 = 1s2 *1s2 2s2 *2s2 2pz2 2px2 2py2
For (Q) 1
f R1 R2 Thus only C2 will be paramagnetic
3. (b) M N
1 1 0.5 1 r = k [M]x
1.5 1
r r 2r when M = 2M; r = 8r, thus
f = 2r 8r = k[2M]x
For the combination 8 = (2)x
x= 3
1 1 1 1 1 2 1
4. (b) Greater the extent of branching, lesser is the boiling
F f1 f2 2r 2r 2r r
point of the hydrocarbon, so order of b.p is III > II > I.
F=r 5. (c) The given reaction proceeds through SN1 mechanism
Similarly, we can either find or do not find the remaining which involves carbocation as intermediate.
options (b) is the correct option.
20. (d) If (m1 + m2) sin < N2 the bodies will be at rest H
Ph 3C — O — R Ph3C — O — R Ph3C ROH
i.e., (m1 + m2)g sin < m2 g cos H
Thus, higher the stability of the carbocation, greater
will be reactivity. Presence of electron releasing group
f = N2 (e.g., — OCH3) in p-position of the phenyl group will
N2 disperse the positive charge of the carbocation by +M
effect, hence stabilizes the carbocation.
6. (d)
(m1 + m2)g cos (m1 + m2) sin (i) CH 3 MgBr
(m1 + m2)g (ii) aq. acid
O
Cl O Cl O
7. (a) KIO4 + H2O2 KIO3 + H2O + O2
m2 g Thus H2O2 is acting as a reducing agent
tan
m1 m2 g 2NH2OH + H2O2 N2 + 4H2O
Here H2O2 is acting as an oxidising agent
m2
tan 8. (a) P4 8SOCl 2 4PCl3 4SO 2 2S2 Cl 2
m1 m2
white phosphorus

0.3 2 9. (c)
tan
1 2
Complete
tan < 0.2 XeF6 Hydrolysis
XeO3 + H2F2
(P)
i.e., If the angle < 11.5° the frictional force is less than
OH –/H2O
N2 = m2g = 0.3 × 2 × g = 0.6 g OH –/H2O
and is equal to (m1 + m2)g sin HXeO4– XeO64– + Xe(g)
(Q) (Slow disproportionat
ion) + H2O + O2(g)
EBD_780
34-2014 JEE Advanced 2014 Solved Paper
10. (b) Given conditions are boiling conditions for water due
d 40
to which 2
24 d 10
Stotal = 0
Ssystem + Ssurroundings = 0 d = 48 – 2d
Ssystem = – Ssurroundings 3d = 48
d = 16 cm
For process, Ssystem > 0
12. (d) The general formula of mean free path ( ) is
Ssurroundings < 0
11. (c) According to Graham’s law of diffusion, if all conditions RT
=
are identical, 2 d2 N A p
1 (d = diameter of molecule, p = pressure inside the vessel)
r=
M Since d and p are same for both gases, ideally their
As in this question, all conditions are identical for X are same. Hence it must be the higher drift speed of X
and Y, then due to which it is facing more collisions per second
with the inert gas in comparison to gas Y. Hence X
rx My faces more resistance from inert gas than Y and hence
covers lesser distance than that predicted by Graham’s
ry Mx
law.
CH 3CH2 I
NaNH2 + – C– Na+
H Na O (1 eq.)
13. (a) HO

CH3 I C H2
+– C H3CO H3CO
(1 eq.) Lindlar
Na O
catalyst
H H
(X), C7H14O
OH
NaNH2 Br
14. (c) H C–Na+

O–Na+ (i) H + OH CrO3


C O
(ii) H2 , pd/C

(Y), C7H17O
2 2
15. (b) Ni 4HCl [NiCl 4 ]
(M1) (Q) Tetrahedral
2
Ni 4KCN [Ni(CN) 4 ]2
(M1) (R) Square planar

[Note: Co2 6CN [Co(CN)6 ]4 ]


Octahedral

OH
16. (d) Zn 2 2OH Zn(OH)2 [Zn(OH) 4 ]2
(M2) (S) excess
white ppt. so lub le

17. (b) Complex Magnetic character Isomerism


3+ 3
P, [Cr(NH 3 ) 4 Cl2 ]Cl Cr is d , hence paramagnetic cis-trans
Q, [Ti(H 2O)5Cl](NO 3 ) 2 Ti3+ is d 1, hence paramagnetic. ionization
2+ 8
R, [Pt(en)(NH3 )Cl]NO3 Pt is d , complex is square planar, ionization
all electrons are paired, hence diamagnetic
3+ 6
S, [Co(NH3 ) 4 (NO3 ) 2 ]NO3 Co is d , all electrons are paired cis-trans
due to strong ligands, hence diamagnetic
JEE Advanced 2014 Solved Paper 2014-35

18. (c) P. d – d ( bonding)

Q. p – d( bonding)

R. p–d antibonding)

S. d–d antibonding)

19. (a)
O O
(1) C6H5CH2—C—O CH3 C 6H 5CH 2 + CO 2 + CH 3 – O
O O CH3 CH3
(3) C6H5CH2—C—O C— CH3 CO2 + C6H5CH2 + CH3—C—O
CH2 – C6H5 CH2—C6H5
O
C O CH3 O CH3
(4) C6H5 O C—CH3 C6H5— C — O + O — C — CH3
CH3 CH3
O O
C O C
(2) C 6H5 O CH3 C 6H 5 O + CH 3 — O
NO2

red hot Nitration NH4HS


20. (c) Scheme (III) : H H Fe, 873 K
Acetylene, P
NO2
NO2 NO2 NO2

NaNO2 H 2O
+ N2 + H +
H+ + boil
NH2 N2 OH
(C6H5NO3)
OH OH OH OH

Conc. H2SO4 Conc. HNO3


O2N O2N
dil. H2SO4
Scheme (IV) :
60°C Conc. H2 SO4 heat
OH OH OH OH
Q (C6 H5NO4 )
SO3H SO3H
EBD_780
36-2014 JEE Advanced 2014 Solved Paper
NO2 NH2 NHCOCH3 NHCOCH3

Sn/HCl CH 3COCl Conc.


Scheme (II) :
H2SO4

R
SO3H
NHCOCH3 NHCOCH3 NH2
NO2 NO2 NO2
Conc. HNO3 dil. OH –
H2SO4 , heat

(C6H6N2O2)
SO3H
NO2 NO2 NO2 NO2

(i) KMnO4, OH , heat SOCl2 NH3
Scheme (I) : +
(ii) H , H2O

CH3 COOH COCl CONH2


S (C7H6N2O3)

3
2ab
MATHEMATICS 2a2 b 2 sin 2 120 4 3y
1. (c) Card numbered 1 is always placed in envelope a b c abc x c c 2c x c
numbered 2, we can consider two cases. 3. (d)
Case I: Card numbered 2 is placed in envelope numbered
1.
Y
Then it is dearrangement of 4 objects, which can be S
done in
P
1 1 1 1
4! 1 = 9 ways
1! 2! 3! 4!
Case II: Card numbered 2 is not placed in envelope X
numbered 1. (2, 0)
(– 2, 0)
Then it is dearrangement of 5 objects, which can be
done in Q
1 1 1 1 1 R
5! 1 = 44 ways
1! 2! 3! 4! 5!
Total ways = 44 + 9 = 53
Let the tangent to
2. (b) Let two sides of be a and b.
y2 = 8x be
Then a + b = x and ab = y
Also given x2 – c2 = y, where c is the third side of . 2
y mx
2
m
a b c2 ab a2 b2 c2 ab If it is common tangent to parabola and circle, then

a 2 b2 c 2 1 1 2
cos c c = 120° y mx is a tangent to x2 + y2 = 2
2ab 2 2 m

r 4 2
where = area of triangle m 4
R s abc 2 2
m 2
1 m 2 (1 m2 )
2
1
8 ab sin c
r 4 2
2 m4 + m2 – 2 = 0
R a b c a b c abc (m2 + 2) (m2 – 1) = 0
abc
2
JEE Advanced 2014 Solved Paper 2014-37

m = 1 or – 1 9
Required tangents are y = x + 2 and y = – x – 2 LHS and RHS 3
4
Their common point is (– 2, 0) Equation has no solution.
Tangents are drawn from (– 2, 0)
Chord of contact PQ to circle is 2
17
x. (– 2) + y.0 = 2 or x = – 1 7. (a) Let I = 2cosec x dx
and Chord of contact RS to parabola is
4
y. 0 = 4 (x – 2) or x = 2
Hence coordinates of P and Q are 2
16
(– 1, 1) and (– 1, – 1) = cosec x cot x cosec x cot x 2cosec x dx
Also coordinates of R and S are
(2, – 4) and (2, 4) 4

1 2 16
Area of trapezium PQRS is 2 8 3 15 1
2 I= 2 cosec x cot x . cosec x dx
cosec x cot x
4. (a) According to given condition, we can have the
4
following cases Let cosec x + cot x = eu
(I) G G B B B (II) B G G B B
(III) G B G B B (IV) B G B G B cosec x cot x cosec 2 x dx eu du
(V) G B B G B – cosec x dx = du
i.e., the two girls can occupy two of the first three places
(case I, II, III) or second and fourth (case IV) or first Also at x ,u ln 2 1
4
and fourth (case V) places.
Thus favourable cases are at x , u = ln 1 = 0
2
= 3 × 2! × 3! + 2 × 2! × 3! = 60 0
16
Total ways in which 5 persons can be seated = 5! = 120 I 2 eu e u du
60 1 ln 2 1
Required probability =
120 2 ln 2 1
16
5. (d) Quadratic equation with real coefficients and purely = 2 eu e u du
imaginary roots can be considered as 0
p(x) = x2 + a = 0 where a > 0 and a R 4 7 12
The p[p(x)] = 0 8. (c) Coeff. of x11 in exp. of 1 x2 1 x3 1 x4
(x2 + a)2 + a = 0 = (Coeff. of xa ) × (Coeff. of xb ) × (Coeff. of xc )
x4 + 2ax2 + (a2 + a) = 0 Such that a + b + c = 11
Here a = 2m, b = 3n, c = 4p
2a 4a 2 4a 2 4a 2m + 3n + 4p = 11
x2
2 Case I : m = 0, n = 1, p = 2
Case II : m = 1, n = 3, p = 0
x2 a ai Case III : m = 2, n = 1, p = 1
Case IV : m = 4, n = 1, p = 0
x a ai i where , 0
Required coeff.
p[p(x)] = 0 has complex roots which are neither purely
real nor purely imaginary. = 4 C0 7
C1 12 C2 4 C1 7 C3 12 C0
6. (d) sin x + 2sin 2x – sin 3x = 3 + 4 C2 7 C1 12
C1 4 C4 7
C1 12 C0
sin x + 4sin x cos x – 3sin x + 4sin3 x = 3 = 462 + 140 + 504 + 7 = 1113
sin x (–2 + 2cos x + 4sin2 x) = 3
sin x (– 2 + 2cos x + 4 – 4cos2 x) = 3 x2
9. (b) F x f t dt for x 0, 2
3
2 + 2 cos x – 4cos2x = 0
sin x
F x f x .2 x
1 1 1 3
2 – 4cos 2 x 2 . 2cos x. + =
2 4 4 sin x Now F x f x x 0, 2
2 f x .2 x f x
9 1 3
2 cos x
4 2 sin x f x
2x
f x
EBD_780
38-2014 JEE Advanced 2014 Solved Paper
ln f (x) = x2 + c 1 2 4 1 3 3 2 2 3 2 3 4
2
x2 =
f (x) = e x c
e .e c 3 5 7 3 5 7 3 5 7 3 5 7
As f (0) = 1 1 = ec 8 9 12 24 53
=
2 105 105
f x ex
12. (c) If x1, x2, x3 are in AP then 2x2 = x1 + x3
x2 2
LHS is even, x1 & x3 can be both even or both odd.
So F x e x dx ex 1 x1 and x3 both can be even in 1 × 3 = 3 ways
0 x1 and x3 both can be odd in 2 × 4 = 8 ways
F 2 e4 1 Total favourable ways = 3 + 8 = 11
10. (b) Given D.E. can be written as Also one number from each box can be drawn in 3 × 5
× 7 ways
dy x x4 2x
y Total ways = 105
dx 1 x 2 1 x2 11
x Hence required probability =
If dx 1
log 1 x 2 105
2
e 1 x e 2 1 x2 a 2a
Solution is given by 13. (d) PQ is a focal chord, Q ,
t2 t
x4 2x Also QR || PK mQR = mPK
y 1 x2 1 x2 . dx
1 x 2 2a
2ar
t 0 2at
5
x a
y 1 x2 x2 c ar 2 2a at 2
5 t 2
AB f (0) = 0 At x = 0, y = 0
1
c=0 2a r
t 2at
x5 =
y 1 x2 x2 a
1
r
1
r a 2 t2
5 t t
x5 1
x2 2 – t2 = t r
or y = f (x) = 5 t
1 x2 r
1
otherwise Q will coincide with R
t
3 x5
2 x2 t2 1
I 5 dx 2 t2 1 tr r
t
3 1 x2 14. (b) Tangent at P is
2
ty x at 2 ....(i)
3 Normal at S
2
x2 x5 sx + y = 2as + as3 ....(ii)
= 2 dx is odd
1
0 1 x2 1 x2 But given st = 1 s
t
put x = sin dx = cos d
x 2a a
y
3 3 t t t3
I = 2 sin 2 d 1 cos 2 d 2 3
xt yt 2at 2 a
0 0
Putting value of x from equation (i) in above equation
we get
sin 2 3 3
= t 2 ty at 2 yt 3 2at 2 a
2 0 3 4
11. (b) x1 + x2 + x3 will be odd t 3 t 3 y at 4 2at 2 a
If two are even and one is odd
or all three are odd. 2t 3 y a t4 2t 2 1
Required probability 2
= P (EEO) + P(EOE) + P(OEE) + P(OQO) a t4 2t 2 1 a t2 1
y
2t 3 2t 3
JEE Advanced 2014 Solved Paper 2014-39

1 h or a = 3 and b = 0
a 1 There are only 2 solutions.
15. (a) g a lim t a 1 t dt
h 0 h
Q(3) f (x) = sinx2 + cosx2
1 h 9
1 1/ 2 1/ 2 f (x) is max. 2 at x2 = or
g lim t 1 t dt 4 4
2 h 0 h
3
1 h x or 13, 13
1 h 2 2
1 1
= lim dt = lim dt There are four points.
t 1 t h 0 2 2
h 0 h h 1 1
t 2 2
3x2
2 2 3x2
R(1) I dx = x
dx
1 h 21 ex 21 e
1
t 1 h
= lim sin 1 2 lim sin 1 2t 1 b b
h 0 1 h 0 Using f x dx f a + b – x dx
h
2 h a a
2
1 1 3 x2 e x
= lim sin 1 2h sin 2h 1 = dx
h 0
21 ex

=
2 3 x2 1 e x 2
2 2 2 2I dx = 3x 2 dx
x
2 1 e 2
1 h
a 1
16. (d) g a lim t a
1 t dt 3 2
2I x 8 8 16
h 0 h 2
I=8
1 h
a a 1 1/ 2
g a lim 1 t t dt 1 x
h 0 cos 2 x log dx
h 1 x
1/ 2
b b S(4) 1/ 2
=0
using f x dx f a b x dx 1 x
cos 2 x log dx
a a 1 x
0
... Numerator
= 0, function being odd.
1 h
a
Also g 1 a lim ta 1 1 t dt Hence option (d) is correct sequence.
h 0 h 1
18. (a) P(4) y cos 3cos x
Thus g a g 1 a
y cos cos 1 4 x 3 3 x
g a g 1 a g a g 1 a 0
y = 4x3 – 3x
1 1 1
Putting a we get g g 0 dy d2y
2 2 2 12 x 2 3 and 24 x
dx dx 2
1
or g 0 1 d2y dy
2 x2 1 x
17. (d) P(2) Let f (x) = ax2 + bx + c y dx dx
where a, b, c 0 and a, b, c are integers.
1
f (0) = 0 c = 0 = 3
x 2 1 24 x x 12 x 2 3
f (x) = ax2 + bx 4x 3x
1 3x
= 8 x2 8 4 x2 1
Also f x dx 1 4x 3
3x
0
3x 12 x 2 9 9 4 x3 3 x
2 1 9
ax3 bx a b = =
1; 1 4 x3 3x 4 x3 3x
3 2 3 2 Q(3) ... A1, A2, A3, ... An are the vertices of a regular
0
2a + 3b = 6
a and b are integers polygon of n sides with its centre at origin and a1, a2 ,
a = 0 and b = 2
.... an are their position vectors.
EBD_780
40-2014 JEE Advanced 2014 Solved Paper
Range of f4 = 0,
a1 a2 .... an
f4 is onto
2 From graph f4 is not one one.
2
Then ak ak 1 sin
n sin x, x 0
Q(3) : f3 x
2 x, x 0
2
and ak ak 1 cos From graph f is differentiable
n
Hence given equation reduces to but not one one.

n 1 2 sin
2
n 1 2 cos x2 , x 0
n n R(2): f 2 of1 x
2x
e , x 0
2 2
tan 1 n 8
n n 4
x2 y2
R.(2) Normal from P(h, 1) on 1 is
6 3
x h y 1
h/6 1/ 3
2 x h h y 1 ; 2x – hy – h = 0
It is perpendicular to x + y = 8
From graph f2of1 is neither
2
1 1 h=2 continuous nor one one.
h
1 1 2 S(4): f 2 x x 2 , x 0,
S.(1) tan 1 tan 1 tan 1 It is continuous and one one.
2x 1 4x 1 x2
1 1
2k 2k
1 2x 1 4x 1 2 20. (c) P(1): zk cos i sin , k = 1 to 9
tan tan 1 10 10
1 1 x2
1 . 2k
2x 1 4 x 1 i
zk e 10
1 4x 1 2 x 1 1 2
tan tan 2k
2
8x 6x 1 1 x2 1 i
10
Now zk.zj = 1 zj e zk
6x 2 2 zk
tan 1 tan 1
8x2 6 x x2 We know if zk is 10th root of unity so will be zk.
1 3x 1 1 2 For every zk, there exist zi = zk
tan tan
2 2
4x 3x x Such that zk . z j zk . z k 1
3x 1 2 Hence the statement is true.
4 x 2 3x x 2 zk
Q(2): z1 z k z for z1 0
3x2 – 7x – 6 = 0 (for x > 0) z1
2 We can always find a solution to z1.z zk
x = 3 or (rejected as x > 0)
3 Hence the statement is false.
Only one +ve solution is there
Hence (a) is the correct option. R(3): We know z10 1 z 1 z z1 .... z z9

x2 , x 0 z10 1
19. (d) P(1): f 4 x z z1 z z2 ..... z z9
z 1
e2 x 1, x 0 = 1 + z + z2 + ... z9
For z = 1 we get
1 z1 1 z2 ..... 1 z9 10
1 z1 1 z2 ..... 1 z9
1
10
JEE Advanced 2013 Solved Paper 2013-1

JEE ADVANCED 2013


1. The question paper consists of 3 parts (Physics, Chemistry & Mathematics). Each part consists of 3 sections.
2. For each question in Section I, you will be awarded 2 marks if you darken the bubble corresponding to the correct
answer and zero mark if no bubbles are darkened. No negative marks will be awarded for incorrect answers in this
section.
3. For each question in Section II, you will be awarded 4 marks if you darken all the bubble(s) corresponding to only
the correct answer(s) and zero mark if no bubbles are darkened. In all other cases, minus one (–1) mark will be
awarded.
4. For each question in Section III, you will be awarded 4 marks if you darken all the bubble(s) corresponding to only
the correct answer(s) and zero mark if no bubbles are darkened. In all other cases, minus one (–1) mark will be
awarded.

PAPER - 1
5. Two non-reactive monoatomic ideal gases have their atomic
PHYSICS masses in the ratio 2 : 3. The ratio of their partial pressures,
SECTION - I when enclosed in a vessel kept at a constant temperature, is
4 : 3. The ratio of their densities is
DIRECTIONS : This section contains 10 multiple choice (a) 1 : 4 (b) 1 : 2
questions. Each question has four choices (a), (b), (c) and (d) out (c) 6 : 9 (d) 8 : 9
of which ONLY ONE is correct. 6. Two rectangular blocks, having identical dimensions, can be
1. The image of an object, formed by a plano-convex lens at a arranged either in configuration-I or in configuration-II
distance of 8 m behind the lens, is real is one-third the size of as shown in the figure. One of the blocks has thermal
conductivity k and the other 2k. The temperature difference
2
the object. The wavelength of light inside the lens is between the ends along the x-axis is the same in both the
3 configurations. It takes 9 s to transport a certain amount of
times the wavelength in free space. The radius of the curved heat from the hot end to the cold end in the configuration-I.
surface of the lens is The time to transport the same amount of heat in the
(a) 1 m (b) 2 m configuration-II is
(c) 3 m (d) 6 m Configuration-II
2. The diameter of a cylinder is measured using a Vernier callipers Configuration-I
with no zero error. It is found that the zero of the Vernier scale
lies between 5.10 cm and 5.15 cm of the main scale. The 2k
Vernier scale has 50 divisions equivalent to 2.45 cm. The 24th
division of the Vernier scale exactly coincides with one of k 2k k
the main scale divisions. The diameter of the cylinder is x
(a) 5.112 cm (b) 5.124 cm (a) 2.0 s (b) 3.0 s
(c) 5.136 cm (d) 5.148 cm (c) 4.5 s (d) 6.0 s
7. A pulse of light of duration 100 ns is absorbed completely by
1 a small object initially at rest. Power of the pulse is 30 mW
3. A ray of light travelling in the direction (iˆ 3 ˆj ) is incident
2 and the speed of light is 3×108 ms–1. The final momentum of
on a plane mirror. After reflection, it travels along the direction the object is
(a) 0.3 × 10–17 kg ms–1 (b) 1.0 × 10–17 kg ms–1
1 ˆ
(i 3 ˆj ) . The angle of incidence is (c) 3.0 × 10–17 kg ms–1 (d) 9.0 × 10–17 kg ms–1
2 8. A particle of mass m is projected from the ground with an
(a) 30° (b) 45° initial speed u0 at an angle with the horizontal. At the
(c) 60° (d) 75° highest point of its trajectory, it makes a completely inelastic
4. In the Young’s double slit experiment using a monochromatic collision with another identical particle, which was thrown
light of wavelength , the path difference (in terms of an vertically upward from the ground with the same initial speed
integer n) corresponding to any point having half the peak u0. The angle that the composite system makes with the
intensity is horizontal immediately after the collision is

(a) (2n 1) (b) (2n 1) (a) (b)


2 4 4 4
(c) (2 n 1) (d) (2 n 1) (c) (d)
8 16 2 2
EBD_780
2-2013 JEE Advanced 2013 Solved Paper
9. The work done on a particle of mass m by a force, (a) Thedirection of themagnetic field is –z direction
(b) The direction of the magnetic field is +z direction
x y
K iˆ ˆj 50 M
2 2 32 2
(x y ) (x y 2 )3 2 (c) The magnitude of the magnetic field units
3Q
(K being a constant of appropriate dimensions), when the
particle is taken from the point (a, 0) to the point (0, a) along 100 M
a circular path of radius a about the origin in the x – y plane (d) The magnitude of the magnetic field is units
3Q
is
14. Two non-conducting solid spheres of radii R and 2R, having
2K K uniform volume charge densities 1 and 2 respectively, touch
(a) (b)
a a each other. The net electric field at a distance 2R from the
centre of the smaller sphere, along the line joining the centres
K
(c) (d) 0
2a of the spheres, is zero. The ratio 1
can be
10. One end of a horizontal thick copper wire of length 2L and 2
radius 2R is welded to an end of another horizontal thin 32
copper wire of length L and radius R. When the arrangement (a) –4 (b)
25
is stretched by applying forces at two ends, the ratio of the
elongation in the thin wire to that in the thick wire is 32
(a) 0.25 (b) 0.50 (c) (d) 4
25
(c) 2.00 (d) 4.00 15. In the circuit shown in the figure, there are two parallel plate
SECTION - II capacitors each of capacitance C. The switch S1 is pressed
This section contains 5 multiple choice questions. Each question first to fully charge the capacitor C1 and then released. The
has four choices (a), (b), (c) and (d) out of which ONE or MORE switch S2 is then pressed to charge the capacitor C2. After
are correct. some time, S2 is released and then S3 is pressed. After some
time
11. A horizontal stretched string, fixed at two ends, is vibrating
in its fifth harmonic according to the equation, S1 S2 S3
y(x, t) = (0.01 m) sin [(62.8 m–1)x] cos[(628 s–1)t]. Assuming
= 3.14, the correct statement(s) is (are)
(a) The number of nodes is 5
(b) The length of the string is 0.25 m C1 C2 C3
2V0
(c) The maximum displacement of the midpoint of the string,
from its equilibrium position is 0.01 m
(d) The fundamental frequency is 100 Hz
12. A solid sphere of radius R and density is attached to one
end of a mass-less spring of force constant k. The other end
of the spring is connected to another solid sphere of radius (a) he charge on the upper plate of C1 is 2CV0
R and density 3 . The complete arrangement is placed in a (b) The charge on the upper plate of C1 is CV0
liquid of density 2 and is allowed to reach equilibrium. The (c) The charge on the upper plate of C2 is 0
correct statement(s) is (are) (d) The charge on the upper plate of C2 is –CV0
4 R3 g SECTION - III
(a) The net elongation of the spring is This section contains 5 questions. The answer to each question
3k
is a single digit integer, ranging from 0 to 9 (both inclusive).
8 R3 g
(b) The net elongation of the spring is 16. The work functions of Silver and Sodium are 4.6 and 2.3 eV,
3k respectively. The ratio of the slope of the stopping potential
(c) The light sphere is partially submerged versus frequency plot for Silver to that of Sodium is
(d) The light sphere is completely submerged 17. A freshly prepared sample of a radioisotope of half-life
13. A particle of mass M and positive charge Q, moving with a 1386 s has activity 103 disintegrations per second. Given
constant velocity u1 4iˆ ms 1 , enters a region of uniform that ln 2 = 0.693, the fraction of the initial number of nuclei
(expressed in nearest integer percentage) that will decay in
static magnetic field, normal to the x-y plane. The region of
the first 80 s after preparation of the sample is
the magnetic field extends from x = 0 to x = L for all values of
18. A particle of mass 0.2 kg is moving in one dimension under a
y. After passing through this region, the particle emerges on
force that delivers a constant power 0.5 W to the particle. If
the other side after 10 milliseconds with a velocity
the initial speed (in ms–1) of the particle is zero, the speed
1 . The correct statement(s) is (are) (in ms–1) after 5 s is
u2 2( 3iˆ ˆj ) ms
JEE Advanced 2013 Solved Paper 2013-3

19. A uniform circular disc of mass 50 kg and radius 0.4 m is 5. In the reaction,
rotating with an angular velocity of 10 rad s–1 about its own P+Q ?R+S
axis, which is vertical. Two uniform circular rings, each of The time taken for 75% reaction of P is twice the time taken
mass 6.25 kg and radius 0.2 m, are gently placed symmetrically for 50% reaction of P. The concentration of Q varies with
on the disc in such a manner that they are touching each reaction time as shown in the figure. The overall order of the
other along the axis of the disc and are horizontal. Assume reaction is
that the friction is large enough such that the rings are at rest
relative to the disc and the system rotates about the original
[Q]0
axis. The new angular velocity (in rad s–1) of the system is
20. A bob of mass m, suspended by a string of length l1, is given
a minimum velocity required to complete a full circle in the [Q]
vertical plane. At the highest point, it collides elastically
Time
with another bob of mass m suspended by a string of length
l2, which is initially at rest. Both the strings are mass-less (a) 2 (b) 3
and inextensible. If the second bob, after collision acquires (c) 0 (d) 1
the minimum speed required to complete a full circle in the 6. Concentrated nitric acid, upon long standing, turns yellow
brown due to the formation of
l1 (a) NO (b) NO2
vertical plane, the ratio is
l2 (c) N2O (d) N2O4
7. The arrangement of X– ions around A+ ion in solid AX is
CHEMISTRY given in the figure (not drawn to scale). If the radius of X– is
250 pm, the radius of A+ is
SECTION - I
This section contains 10 multiple choice questions. Each question –
has 4 choices (a), (b), (c) and (d) out of which ONLY ONE is X
correct. A+
1. The standard enthalpies of formation of CO2(g), H2O(l) and
glucose(s) at 25°C are –400 kJ/mol, –300 kJ/mol and –1300
kJ/mol, respectively. The standard enthalpy of combustion
(a) 104 pm (c) 183 pm
per gram of glucose at 25°C is
(b) 125 pm (d) 57 pm
(a) +2900 kJ (b) –2900 kJ
8. Upon treatment with ammoniacal H2S, the metal ion that
(c) –16.11 kJ (d) +16.11 kJ precipitates as a sulfide is
2. KI in acetone, undergoes SN2 reaction with each of P, Q, R (a) Fe(III) (c) Mg(II)
and S. The rates of the reaction vary as (b) Al(III) (d) Zn(II)
9. Methylene blue, from its aqueous solution, is adsorbed on
O
activated charcoal at 25°C. For this process, the correct
Cl statement is
H3C – Cl Cl Cl
(a) The adsorption requires activation at 25°C
(b) The adsorption is accompanied by a decrease in
P Q R S enthalpy
(c) The adsorption increases with increase of temperature
(a) P > Q > R > S (b) S > P > R > Q (d) The adsorption is irreversible
(c) P > R > Q > S (d) R > P > S > Q 10. Sulfide ores are common for the metals
3. The compound that does NOT liberate CO2, on treatment (a) Ag, Cu and Pb (c) Ag, Mg and Pb
with aqueous sodium bicarbonate solution, is (b) Ag, Cu and Sn (d) Al, Cu and Pb
(a) Benzoic acid (b) Benzenesulphonic acid SECTION - II
(c) Salicylic acid (d) Carbolic acid (Phenol)
This section contains 5 multiple choice questions. Each question
4. Consider the following complex ions, P, Q and R.
has four choices (a), (b), (c) and (d) out of which ONE OR MORE
P = [FeF6]3–, Q = [V(H2O)6]2+ and R = [Fe(H2O)6]2+ are correct.
The correct order of the complex ions, according to their
spin-only magnetic moment values (in B.M.) is 11. Benzene and naphthalene form an ideal solution at room
(a) R < Q < P (b) Q < R < P temperature. For this process, the true statement(s) is(are)
(c) R < P < Q (d) Q < P < R (a) G is positive (b) Ssystem is positive
(c) Ssurroundings = 0 (d) H= 0
EBD_780
4-2013 JEE Advanced 2013 Solved Paper
12. The pair(s) of coordination complexes/ions exhibiting the 20. A tetrapeptide has —COOH group on alanine. This produces
same kind of isomerism is(are) glycine (Gly), valine (Val), phenyl alanine (Phe) and alanine
(a) [Cr(NH3)5Cl]Cl2 and [Cr(NH3)4Cl2]Cl (Ala), on complete hydrolysis. For this tetrapeptide, the
(b) [Co(NH3)4Cl2]+ and [Pt(NH3)2(H2O)Cl]+ number of possible sequences (primary structures) with —
(c) [CoBr2Cl2]2– and [PtBr2Cl2]2– NH2 group attached to a chiral center is
(d) [Pt(NH3)3](NO3)Cl and [Pt(NH3)3Cl]Br
13. The initial rate of hydrolysis of methyl acetate (1M) by a MATHEMATICS
weak acid (HA, 1M) is 1/100th of that of a strong acid (HX,
1M), at 25°C. The Ka of HA is SECTION - I
(a) 1 × 10–4 (b) 1 × 10–5 DIRECTIONS : This section contains 10 multiple choice
(c) 1 × 10–6 (d) 1 × 10–3 questions. Each question has four choices (a), (b), (c) and (d) out
14. The hyperconjugative stabilities of tert-butyl cation and 2- of which ONLY ONE is correct.
butene, respectively, are due to
(a) p (empty) and * electron delocalisations 23 n
(b) * and electron delocalisations 1
1. The value of cot cot 1 2k is
(c) p (filled) and electron delocalisations n 1 k 1
(d) p (filled) * and * electron delocalisations
15. Among P, Q, R and S, the aromatic compound(s) is/are 23 25
(a) (b)
25 23
Cl 23 24
(c) (d)
24 23
AlCl3
P 2. Let PR 3iˆ ˆj 2 kˆ and SQ iˆ 3jˆ 4 kˆ determine
diagonals of a parallelogram PQRS and PT ˆi 2ˆj 3kˆ be
NaH
Q another vector. Then the volume of the parallelpiped
(NH4)2CO 3 determined by the vectors PT, PQ and PS is
R (a) 5 b) 20
100–115 °C
(c) 10 (d) 30
O O
O 1
HCl 3. Let complex numbers and lie on circles (x – x0)2 +
S
(y – y0)2 = r2 and (x – x0)2 + (y – y0)2 = 4r 2.
respectively. If z0 = x0 + iy0 satisfies the equation
2
(a) P (c) R 2 z0 r 2 2, then
(b) Q (d) S
SECTION - III 1 1
(a) (b)
This section contains 5 questions. The answer to each question 2 2
is a single digit integer, ranging from 0 to 9 (both inclusive).
1 1
16. The total number of lone-pairs of electrons in melamine is (c) (d)
7 3
17. The total number of carboxylic acid groups in the product
P is 4. For a > b > c > 0, the distance between (1, 1) and the point of
intersection of the lines ax + by + c = 0 and bx + ay + c = 0 is
O O less than 2 2 . Then
+
1.H 3O . (a) a + b – c > 0 (b) a – b + c < 0
O 2.O3 P (c) a – b + c > 0 (d) a + b – c < 0
3.H2 O2 5. Perpendiculars are drawn from points on the line
O O
x 2 y 1 z
to the plane x + y + z = 3. The feet of
2 1 3
18. The atomic masses of ‘He’ and ‘Ne’ are 4 and 20 a.m.u.,
respectively. The value of the de Broglie wavelength of ‘He’ perpendiculars lie on the line
gas at —73°C is “M” times that of the de Broglie wavelength x y 1 z 2 x y 1 z 2
of ‘Ne’ at 727°C ‘M’ is (a) (b)
5 8 13 2 3 5
19. EDTA4– is ethylenediaminetetraacetate ion. The total
number of N—Co—O bond angles in [Co(EDTA)]1–complex x y 1 z 2 x y 1 z 2
(c) (d)
ion is 4 3 7 2 7 5
JEE Advanced 2013 Solved Paper 2013-5

6. Four persons independently solve a certain problem correctly SECTION - II


1 3 1 1 DIRECTIONS : This section contains 5 multiple choice questions.
with probabilities , , , . Then the probability that the
2 4 4 8 Each question has four choices (a), (b), (c) and (d) out of which
problem is solved correctly by at least one of them is ONE or MORE are correct
11. A rectangular sheet of fixed perimeter with sides having
235 21 their lengths in the ratio 8 : 15 is converted into an open
(a) (b)
256 256 rectangular box by folding after removing squares of equal
area from all four corners. If the total area of removed squares
3 253 is 100, the resulting box has maximum volume. Then the
(c) (d)
256 256 lengths of the sides of the rectangular sheet are
7. The area enclosed by the curves y = sin x + cos x and (a) 24 (b) 32
(c) 45 (d) 60
y = cos x sin x over the interval 0, is k (k 1)
2 4n 2
12. Let Sn ( 1) k 2 . Then Sn can take value(s)
(a) 4( 2 1) (b) 2 2 ( 2 1) k 1

(c) 2( 2 1) (d) 2 2( 2 1) (a) 1056 (b) 1088


(c) 1120 (d) 1332
13. A line l passing through the origin is perpendicular to the
8. A curve passes through the point 1, . Let the slope of lines
6
l1 :(3 t)iˆ (–1 2t) ˆj (4 2t)k,
ˆ – t
y y
the curve at each point (x, y) be sec , x 0.
x x l2 :(3 2s)iˆ (3 2s) ˆj (2 s)k,
ˆ – s
Then the equation of the curve is Then, the coordinate(s) of the point(s) on l2 at a distance of

y 1 y 17 from the point of intersection of l and l1 is (are)


(a) sin log x (b) cos ec log x 2
x 2 x 7 7 5
(a) , , (b) (–1, –1, 0)
3 3 3
2y 2y 1
(c) s ec log x 2 (d) cos log x
x x 2 7 7 8
(c) (1, 1, 1) (d) , ,
9 9 9
14. Let f(x) = x sin x, x > 0. Then for all natural numbers n, f (x)
9. Let f : ,1 R (the set of all real number) be a positive, vanishes at
2
non-constant and differentiable function such that 1
(a) A unique point in the interval n, n
2
1
1
f (x) < 2f(x) and f 1. Then the value of f (x) dx lies 1
2
1/ 2 (b) A unique point in the interval n ,n 1
2
in the interval
(c) A unique point in the interval (n, n + 1)
(a) (2e – 1, 2e) (b) (e – 1, 2e – 1)
(d) Two points in the interval (n, n + 1)
e 1 e 1 15. For 3 × 3 matrices M and N, which of the following
(c) ,e 1 (d) 0,
2 2 statement(s) is (are) NOT correct?
(a) N'MN is symmetric or skew symmetric, according as
10. The number of points in (– ), for which
M is symmetric or skew symmetric
2
x – x sin x – cos x = 0, is
(b) MN – NM is skew symmetric for all symmetric matrices
(a) 6 (b) 4 M and N
(c) 2 (d) 0 (c) MN is symmetric for all symmetric matrices M and N
(d) (adj M) (adj N) = adj (MN) for all invertible matrices M
and N
EBD_780
6-2013 JEE Advanced 2013 Solved Paper
SECTION - III 18. Consider the set of eight vectors
This section contains 5 questions The answer to each question is V aiˆ bjˆ ckˆ :a, b, c {–1,1} . Three non-coplanar
a single digit integer, ranging from 0 to 9 (both inclusive).
vectors can be chosen from V in 2p ways. Then p is
16. A vertical line passing through the point (h, 0) intersects the 19. Of the three independent events E1, E2 and E3, the probability
x2 y2 that only E1 occurs is , only E2 occurs is and only
ellipse 1 at the points P and Q. Let the tangents
4 3 E3 occurs is . Let the probability p that none of events
to the ellipse at P and Q meet at the point R. If (h) = area of E1, E2 or E3 occurs satisfy the equations ( 2 p = and
( – 3 )p = 2 . All the given probabilities are assumed to
the triangle PQR, max (h) and min (h),
1 1/ 2 h 1 2 1/ 2 h 1 lie in the interval (0, 1).
8 Pr obability of occurrence of E1
then 1 8 2 Then
5 Pr obability of occurrence of E 3
17. The coefficients of three consecutive terms of (1 + x)n+5 are 20. A pack contains n cards numbered from 1 to n. Two
in the ratio 5 : 10 : 14. Then n = consecutive numbered cards are removed from the pack and
the sum of the numbers on the remaining cards is 1224. If the
smaller of the numbers on the removed cards is k, then k – 20 =

PAPER - 2

1. The question paper consists of 3 parts (Physics, Chemistry & Mathematics). Each part consists of 3 sections.
2. For each question in Section I, you will be awarded 3 marks if you darken all the bubble(s) corresponding to only
the correct answer(s) and zero mark if no bubbles are darkened. In all other cases, minus one (–1) mark will be
awarded.
3. For each question in Section II and III, you will be awarded 3 marks if you darken the bubble(s) corresponding to
only the correct answer(s) and zero mark if no bubbles are darkened. In all other cases, minus one (–1) mark will be
awarded.

PHYSICS
SECTION - I
DIRECTIONS : This section contains 8 multiple choice questions.
Each question has four choices (a), (b), (c) and (d) out of which
ONE OR MORE arecorrect. C
1. The radius of the orbit of an electron in a Hydrogen-like
atom is 4.5 a0, where a0 is the Bohr radius. Its orbital angular
3h
momentum is . It is given that h is Planck constant and R
2
100 200 300 400 500
is Rydberg constant. The possible wavelength(s), when the
atom de-excites, is (are) (a) The rate at which heat is absorbed in the range 0 -100 K
varies linearly with temperature T.
9 9 (b) Heat absorbed in increasing the temperature from 0-
(a) (b) 100 K is less than the heat required for increasing the
32R 16R
temperature from 400-500 K.
9 4 (c) There is no change in the rate of heat absorption in the
(c) (d)
5R 3R range 400-500 K.
2. The figure below shows the variation of specific heat (d) The rate of heat absorption increases in the range 200-
capacity (C) of a solid as a function of temperature (T). The 300 K.
temperature is increased continuously from 0 to 500 K at a 3. Two non-conducting spheres of radii R1 and R2 and carrying
constant rate. Ignoring any volume change, the following uniform volume charge densities + and – , respectively,
statement(s) is (are) correct to a reasonable approximation. are placed such that they partially overlap, as shown in the
figure. At all points in the overlapping region
JEE Advanced 2013 Solved Paper 2013-7

(a) The speed of the particle when it returns to its


equilibrium position is u0
– (b) The time at which the particle passes through the
m
equilibrium position for the first time is t
R1 R2 k
(c) The time at which the maximum compression of the
4 m
spring occurs is t
3 k
(a) The electrostatic field is zero (d) The time at which the particle passes through the
(b) The electrostatic potential is constant 5 m
(c) The electrostatic field is constant in magnitude equilibrium position for the second time is t
3 k
(d) The electrostatic field has same direction 8. Two bodies, each of mass M, are kept fixed with a separation
4. Using the expression 2d sin = , one calculates the values 2L. A particle of mass m is projected from the midpoint of the
of d by measuring the corresponding angles in the range 0 line joining their centres, perpendicular to the line. The
to 90°. The wavelength is exactly known and the error in gravitational constant is G. The correct statement(s) is (are)
is constant for all values of . As increases from 0° (a) The minimum initial velocity of the mass m to escape
(a) The absolute error in d remains constant GM
the gravitational field of the two bodies is 4
(b) The absolute error in d increases L
(c) The fractional error in d remains constant (b) The minimum initial velocity of the mass m to escape
(d) The fractional error in d decreases
GM
5. Two vehicles, each moving with speed u on the same the gravitational field of the two bodies is 2
L
horizontal straight road, are approaching each other. Wind
blows along the road with velocity w. One of these vehicles (c) The minimum initial velocity of the mass m to escape
blows a whistle of frequency f1. An observer in the other 2GM
vehicle hears the frequency of the whistle to be f2. The the gravitational field of the two bodies is
L
speed of sound in still air is V. The correct statement(s) is (d) The energy of the mass m remains constant
(are)
(a) If the wind blows from the observer to the source, SECTION - II
f2 > f1
(b) If the wind blows from the source to the observer, This section contains 4 Paragraphs each describing theory,
f2 > f1 experiment, data etc. Eight questions relate to four paragraphs
with two questions on each paragraph. Each question of a
(c) If the wind blows from observer to the source,
paragraph has only one correct answer among the four choices
f2 < f1 (a), (b), (c) and (d).
(d) If the wind blows from the source to the observer,
Paragraph for Questions 9 and 10
f2 < f1
6. A steady current I flows along an infinitely long hollow The mass of a nucleus A Z X is less than the sum of the masses of
cylindrical conductor of radius R. This cylinder is placed (A-Z) number of neutrons and Z number of protons in the nucleus.
coaxially inside an infinite solenoid of radius 2R. The solenoid The energy equivalent to the corresponding mass difference is
has n turns per unit length and carries a steady current I. known as the binding energy of the nucleus. A heavy nucleus of
Consider a point P at a distance r from the common axis. The mass M can break into two light nuclei of masses m1 and m2 only
correct statement(s) is (are) if (m1 + m2) < M. Also two light nuclei of masses m3 and m4 can
(a) In the region 0 < r < R, the magnetic field is non-zero undergo complete fusion and form a heavy nucleus of mass M'
only if (m3 + m4) > M'. The masses of some neutral atoms are
(b) In the region R < r < 2R, the magnetic field is along the given in the table below:
common axis
1 1.007825 u 2 2.014102 u
(c) In the region R < r < 2R, the magnetic field is tangential 1H 1H
to the circle of radius r, centered on the axis 3 3.016050 u 4 4.002603 u
1H 2 He
(d) In the region r > 2R, the magnetic field is non-zero
6 6.015123 u 7 7.016004 u
7. A particle of mass m is attached to one end of a mass-less 3 Li 3 Li
spring of force constant k, lying on a frictionless horizontal 70 82
30 Zn 69.925325 u 34 Se 81.916709 u
plane. The other end of the spring is fixed. The particle starts
moving horizontally from its equilibrium position at time 152 151.919803 u 206 205.974455 u
64 Gd 82 Pb
t = 0 with an initial velocity u0. When the speed of the particle
209 208.980388 u 210 209.982876 u
is 0.5 u0, it collides elastically with a rigid wall. After this 83 Bi 84 Po
collision (1u = 932 MeV/c2)
EBD_780
8-2013 JEE Advanced 2013 Solved Paper
9. The kinetic energy (in keV) of the alpha particle, when the 13. In the method using the transformers, assume that the ratio
210
nucleus 84 Po at rest undergoes alpha decay, is of the number of turns in the primary to that in the secondary
(a) 5319 (b) 5422 in the step-up transformer is 1 : 10. If the power to the
(c) 5707 (d) 5818 consumers has to be supplied at 200 V, the ratio of the
10. The correct statement is number of turns in the primary to that in the secondary in
the step-down transformer is
(a) The nucleus 63 Li can emit an alpha particle (a) 200 : 1 (b) 150 : 1
210 (c) 100 : 1 (d) 50 : 1
(b) The nucleus 84 Po can emit a proton
14. If the direct transmission method with a cable of resistance
(c) Deuteron and alpha particle can undergo complete 0.4 km–1 is used, the power dissipation| (in %) during
fusion transmission is
(d) The nuclei 70 and 82 can undergo complete (a) 20 (b) 30
30 Zn 34 Se
(c) 40 (d) 50
fusion
Paragraphs for Questions 15 and 16
Paragraph for Questions 11 and 12
A small block of mass 1 kg is released from rest at the top of a
A point charge Q is moving in a circular orbit of radius R in rough track. The track is a circular arc of radius 40 m. The block
the x-y plane with an angular velocity . This can be considered slides along the track without toppling and a frictional force acts
Q on it in the direction opposite to the instantaneous velocity. The
as equivalent to a loop carrying a steady current . A uniform work done in overcoming the friction up to the point Q, as shown
2
magnetic field along the positive z-axis is now switched on, which in the figure below, is 150 J.
increases at a constant rate from 0 to B in one second. Assume (Take the acceleration due to gravity, g = 10 ms-2)
that the radius of the orbit remains constant. The application of y
the magnetic field induces an emf in the orbit. The induced emf is
defined as the work done by an induced electric field in moving a
R P
unit positive charge around a closed loop. It is known that, for an
30°
orbiting charge, the magnetic dipole moment is proportional to
the angular momentum with a proportionality constant .
11. The change in the magnetic dipole moment associated with Q R
the orbit, at the end of the time interval of the magnetic field
change, is x
2 O
BQR
(a) – BQR2 (b) 15. The magnitude of the normal reaction that acts on the block
2 at the point Q is
(a) 7.5 N (b) 8.6 N
BQR 2
(c) (d) BQR 2 (c) 11.5 N (d) 22.5 N
2 16. The speed of the block when it reaches the point Q is
12. The magnitude of the induced electric field in the orbit at (a) 5 ms–1 (b) 10 ms–1
any instant of time during the time interval of the magnetic
(c) 10 3 ms 1 (d) 20 ms–1
field change is
BR BR SECTION - III
(a) (b)
4 2
This section contains 4 multiple choice questions. Each question
(c) BR (d) 2BR has matching lists. The codes for the lists have choices (a), (b), (c)
Paragraph for Questions 13 and 14 and (d) out of which ONLY ONE is correct.
A thermal power plant produces electric power of 600 kW at 17. One mole of a monatomic ideal gas is taken along two cyclic
4000 V, which is to be transported to a place 20 km away from the processes E F G E and E F H E as shown in
power plant for consumers' usage. It can be transported either the PV diagram. The processes involved are purely isochoric,
directly with a cable of large current carrying capacity or by using isobaric, isothermal or adiabatic.
a combination of step-up and step-down transformers at the two
P
ends. The drawback of the direct transmission is the large energy F
32 P0
dissipation. In the method using transformers, the dissipation is
much smaller. In this method , a step-up transformer is used at the
plant side so that the current is reduced to a smaller value. At the
consumers' end, a step-down transformer is used to supply power
to the consumers at the specified lower voltage. It is reasonable to
assume that the power cable is purely resistive and the transformers
are ideal with power factor unity. All the currents and voltages P0 G
E H
mentioned are rms values.
V
V0
JEE Advanced 2013 Solved Paper 2013-9
Match the paths in List I with the magnitudes of the work Codes:
done in List II and select the correct answer using the codes P Q R S
given below the lists. (a) 2 3 1 4
List I List II
(b) 1 2 4 3
P. G E 1. 160 P0V0 ln2
(c) 4 1 2 3
Q. G H 2. 36 P0V0
(d) 2 3 4 1
R. F H 3. 24 P0V0
20. Match List I with List II and select the correct answer using
S. F G 4. 31 P0V0 the codes given below the lists:
Codes: List I List II
P Q R S P. Boltzmann constant 1. [ML2T-1]
(a) 4 3 2 1 Q. Coefficient of viscosity 2. [ML–1T–1]
(b) 4 3 1 2 R. Planck constant 3. [MLT–3K–1]
(c) 3 1 2 4 S. Thermal conductivity 4. [ML2T–2K–1]
Codes:
(d) 1 3 2 4
P Q R S
18. Match List I of the nuclear processes with List II containing (a) 3 1 2 4
parent nucleus and one of the end products of each process (b) 3 2 1 4
and then select the correct answer using the codes given (c) 4 2 1 3
below the lists: (d) 4 1 2 3
List I List II
P. Alpha decay 1. 15
8 O
15
7 O ... CHEMISTRY
Q. + decay 2. 138 234 SECTION - I
92 U 90 Th ...
R. Fission 3. 185 184 This section contains 8 multiple choice questions. Each question
83 Bi 82 Pb ...
has four choices (a), (b), (c) and (d) out of which ONE or MORE
S. Proton emission 4. 239 140
94 Pu 57 La ... are correct
Codes: 1. The Ksp of Ag2CrO4 is 1.1 × 10–12 at 298 K. The solubility
P Q R S (in mol/L) of Ag2CrO4 in a 0.1 M AgNO3 solution is
(A) 4 2 1 3 (a) 1.1 × 10–11 (b) 1.1 × 10–10
(B) 1 3 2 4 (c) 1.1 × 10 –12 (d) 1.1 × 10–9
(C) 2 1 4 3 2. In the following reaction, the product(s) formed is(are)
(D) 4 3 2 1
19. A right angled prism of refractive index 1 is placed in a
rectangular block of refractive index 2, which is surrounded
by a medium of refractive index 3, as shown in the figure.
A ray of light 'e' enters the rectangular block at normal
incidence. Depending upon the relationships between 1, 2
and 3, it takes one of the four possible paths 'ef', 'eg', 'eh' or 'ei'.

e 45° g

1 h
i

3
2

Match the paths in List I with conditions of refractive indices in


List II and select the correct answer using the codes given below
the lists:
List I List II
P. e f 1. 1> 2 2
Q. e g 2. 2> 1 and 2> 3
R. e h 3. 1= 2 (a) P (major) (b) Q (minor)
S. e i 4. (c) R (minor) (d) S (major)
2< 1< 2 2 and 2 > 3
EBD_780
10-2013 JEE Advanced 2013 Solved Paper
3. The major product(s) of the following reaction is(are) 5. The correct statement(s) about O3 is(are)
(a) O—O bond lengths are equal
(b) Thermal decomposition of O3 is endothermic
(c) O3 is diamagnetic in nature
(d) O3 has a bent structure
6. In the nuclear transmutation
9 8
4 Be +X 4 Be + Y
(X, Y) is(are)
(a) ( , n) (b) (p, D)
(d) (n, D) (d) ( , p)
7. The carbon-based reduction method is NOT used for the
extraction of
(a) Tin from SnO2
(b) Iron from Fe2O3
(c) Aluminium from Al2O3
(d) Magnesium from MgCO3.CaCO3
8. The thermal dissociation equilibrium of CaCO3(s) is studied
under different conditions
CaCO3(s) CaO(s) + CO2(g). For this equilibrium, the
correct statement(s) is(are)
(a) H is dependent on T
(b) K is independent of the initial amount of CaCO3
(c) K is dependent on the pressure of CO2 at a given T
(d) H is independent of catalyst, if any
SECTION - II
This section contains 4 Paragraphs each describing theory,
(a) P (b) Q experiment, data etc. Eight questions relate to four paragraphs
(c) R (d) S with two questions on each paragraph. Each question of a
4. After completion of the reactions (I and II), the organic paragraph has only one correct answer among the four choices
compound(s) in the reaction mixtures is(are) (a), (b), (c) and (d).
Paragraph for Questions 9 and 10
An aqueous solution of a mixture of two inorganic salts, when
treated with dilute HCl, gave a precipitate (P) and a filtrate (Q). The
Reaction I :
precipitate P was found to dissolve in hot water. The filtrate (Q)
remained unchanged, when treated with H2S in a dilute mineral
acid medium. However, it gave a precipitate (R) with H2S in an
ammoniacal medium. The precipitate R gave a coloured solution
(S), when treated with H2O2 in an aqueous NaOH medium.
Reaction II : 9. The precipitate P contains
(a) Pb2+ (b) Hg2 2+
(c) Ag + (d) Hg2+
10. The coloured solution S contains
(a) Fe2(SO4)3 (b) CuSO4
(c) ZnSO4 (d) Na2CrO4
Paragraph for Questions 11 and 12
P and Q are isomers of dicarboxylic acid C4H4O4. Both decolorize
Br2/H2O. On heating, P forms the cyclic anhydride. Upon treat-
ment with dilute alkaline KMnO4, P as well as Q could produce
one or more than one from S, T and U.

(a) Reaction I : P and Reaction II : P


(b) Reaction I : U, acetone and Reaction II : Q, acetone
(c) Reaction I : T, U, acetone and Reaction II : P
(d) Reaction I : R, acetone and Reaction II : S, acetone
JEE Advanced 2013 Solved Paper 2013-11

11. Compounds formed from P and Q are, respectively 14. The pair of isochoric processes among the transformation
(a) Optically active S and optically active pair of states is
(T, U) (a) K to L and L to M (b) L to M and N to K
(b) Optically inactive S and optically inactive pair (c) L to M and M to N (d) M to N and N to K
(T, U) Paragraph for Questions 15 and 16
The reactions of Cl2 gas with cold-dilute and hot-concentrated
(c) Optically active pair (T, U) and optically active S
NaOH in water give sodium salts of two (different) oxoacids of
(d) Optically inactive pair (T, U)) and optically inactive S chlorine, P and Q, respectively. The Cl2 gas reacts with SO2 gas,
12. In the following reaction sequences V and W are respectively in presence of charcoal, to give a product R. R reacts with white
phosphorus to give a compound S. On hydrolysis, S gives an
oxoacid of phosphorus, T.
15. P and Q, respectively, are the sodium salts of
(a) Hypochlorus and chloric acids
(b) Hypochlorus and chlorus acids
(c) Chloric and perchloric acids
(d) Chloric and hypochlorus acids
16. R, S and T respectively, are
(a) SO2Cl2, PCl5 and H3PO4
(a) (b) SO2Cl2, PCl3 and H3PO3
(c) SOCl2, PCl3 and H3PO2
(d) SOCl2, PCl5 and H3PO4
SECTION - III
This section contains 4 multiple choice questions. Each question
(b) has matching lists. The codes for the lists have choices (a), (b), (c)
and (d) out of which ONLY ONE is correct.
17. The unbalanced chemical reactions given in List I show miss-
ing reagent or condition (?) which are provided in List II.
Match List I with List II and select the correct answer using
the code given below the lists :
(c)
List I List II
P. PbO2 + H2SO4 PbSO4 + O2 + other product 1. NO
Q. Na2S2O3 + H2O NaHSO4 + other product 2. I2
R. N2H4 N2 + other product 3. Warm
(d) S. XeF2 Xe + other product 4. Cl2
Codes :
Paragraph for Questions 13 and 14 P Q R S
A fixed mass 'm' of a gas is subjected to transformation of states (a) 4 2 3 1
from K to L to M to N and back to K as shown in the figure (b) 3 2 1 4
(c) 1 4 2 3
(d) 3 4 2 1
18. Match the chemical conversions in List I with the appropri-
ate reagents in List II and select the correct answer using
the code given below the lists :
List I List II

P 1. (i) Hg(OAc)2; (ii) NaBH4

Q. 2. NaOEt
13. The succeeding operations that enable this transformation
of states are R. 3. Et-Br
(a) Heating, cooling, heating, cooling
(b) Cooling, heating, cooling, heating
(c) Heating, cooling, cooling, heating S. 4. (i) BH3; (ii) H2O2/NaOH
(d) Cooling, heating, heating, cooling
EBD_780
12-2013 JEE Advanced 2013 Solved Paper
Codes : MATHEMATICS
P Q R S
SECTION - I
(a) 2 3 1 4
(b) 3 2 1 4 DIRECTIONS : This section contains 8 multiple choice questions.
(c) 2 3 4 1 Each question has four choices (a), (b), (c) and (d) out of which
ONE or MORE are correct
(d) 3 2 4 1
19. An aqueous solution of X is added slowly to an aqueous 1. For a (the set of all real numbers), a –1,
solution of Y as shown in List I. The variation in conductiv-
(1a 2a ... n a) ) 1
ity of these reactions is given in List II. Match list I with List lim .
a 1
II and select the correct answer using the code given below n (n 1) [(na 1) (na 2) ... (na n)] 60
the lists : Then a =
List I List II (a) 5 (b) 7
P. (C2H5)3N + CH3COOH 1. Conductivity decreases and 15 17
X Y then increases (c) (d)
2 2
Q. KI (0.1M) + AgNO3(0.01M) 2. Conductivity decreases and 2. Circle(s) touching x-axis at a distance 3 from the origin and
X Y then does not change much
having an intercept of length 2 7 on y-axis is (are)
R. CH3COOH + KOH 3. Conductivity increases and
(a) x2 + y2 – 6x + 8y + 9 = 0
X Y then does not change much
(b) x2 + y2 – 6x + 7y + 9 = 0
S. NaOH + HI 4. Conductivity does not change
(c) x2 + y2 – 6x – 8y + 9 = 0
X Y much and then increases (d) x2 + y2 – 6x – 7y + 9 = 0
Codes :
P Q R S y z y z
3. Two lines L1 : x = 5, and L2 : x = ,
(a) 3 4 2 1 3 2 1 2
(b) 4 3 2 1 are coplanar. Then can take value(s)
(c) 2 3 4 1 (a) 1 (b) 2
(d) 1 4 3 2 (c) 3 (d) 4
20. The standard reduction potential data at 25°C is given below. 1
4. In a triangle PQR, P is the largest angle and cos P = .
E°(Fe3+, Fe2+) = + 0.77 V; 3
E°(Fe2+, Fe) = – 0.44 V Further the incircle of the triangle touches the sides PQ, QR
E°(Cu2+, Cu) = + 0.34 V; and RP at N, L and M respectively, such that the lengths of
PN, QL and RM are consecutive even integers. Then possible
E°(Cu+, Cu) = + 0.52 V
length(s) of the side(s) of the triangle is (are)
E°[O2(g) + 4H+ + 4e– 2H2O] = + 1.23 V; (a) 16 (b) 18
E°[O2(g) + 2H2O + 4e– 4OH–] = + 0.40 V (c) 24 (d) 22
E°(Cr3+, Cr) = – 0.74 V;
3 i
E°(Cr2+, Cr) = – 0.91 V 5. Let w and P = {wn : n = 1, 2, 3, ...}. Further H1 =
2
Match E° of the redox pair in List I with the values given in
List II and select the correct answer using the code given 1 1
z : Re z and H 2 z : Re z , where c is the
below the lists : 2 2
List I List II
set of all complex numbers. If z1 P H1, z2 P H 2 and
P. E°(Fe3+, Fe) 1. – 0.18 V
O represents the origin, then z1Oz2 =
Q. E°(4H2O 4H+ + 4OH–) 2. – 0.4 V
R. E°(Cu2+ + Cu 2Cu+) 3. – 0.04 V (a) (b)
2 6
S. E°(Cr3+, Cr2+) 4. – 0.83 V
Codes : 2 5
(c) (d)
P Q R S 3 6
(a) 4 1 2 3 6. If 3x = 4x–1, then x =
(b) 2 3 4 1 2log3 2 2
(c) 1 2 3 4 (a) 2log3 2 1 (b) 2 log 2 3
(d) 3 4 1 2
1 2log 2 3
(c) 1 log 4 3 (d) 2log 2 3 1
JEE Advanced 2013 Solved Paper 2013-13

7. Let be a complex cube root of unity with 1 and P = [pij] 12. If chord PQ subtends an angle at the vertex of y2 = 4ax,
be a n × n matrix with pij = i+j. Then p2 0, when n = then tan =
(a) 57 (b) 55 2 2
(c) 58 (d) 56 (a) 7 (b) 7
3 3
8. The function f(x) = 2|x| + |x + 2| – | |x + 2| – 2 |x| | has a local
minimum or a local maximum at x = 2 2
(c) 5 (d) 5
3 3
2
(a) – 2 (b) Paragraphs for Questions 13 and 14
3
Let S = S1 S2 S3, where
2
(c) 2 (d) z –1 3i
3 S1 {z : | z | 4}, S2 z : Im 0
1– 3i
SECTION - II
and S3 {z : Re z 0}.
This section contains 4 Paragraphs each describing theory,
experiment, data etc. Eight questions relate to four paragraphs 13. Area of S =
with two questions on each paragraph. Each question of a 10 20
paragraph has only one correct answer among the four choices (a) (b)
3 3
(a), (b), (c) and (d).
Paragraphs for Questions 9 and 10 16 32
(c) (d)
Let f : [0, 1] (the set of all real numbers) be a function. Suppose 3 3
the function f is twice differentiable, f(0) = f(1) = 0 and satisfies min |1 3i z |
f (x) – 2f (x) + f (x) . ex, x [0, 1]. 14. z S
9. Which of the following is true for 0 < x < 1?
2 3 2 3
1 1 (a) (b)
(a) 0 f ( x) (b) f ( x) 2 2
2 2
3 3 3 3
1 (c) (d)
(c) f ( x) 1 (d) f ( x) 0 2 2
4
Paragraphs for Questions 15 and 16
10. If the function e–x f(x) assumes its minimum in the interval
1 A box B1 contains 1 white ball, 3 red balls and 2 black balls. Another
[0, 1] at x , which of the following is true? box B2 contains 2 white balls, 3 red balls and 4 black balls. A third
4
box B3 contains 3 white balls, 4 red balls and 5 black balls.
1 3
(a) f'( x) f ( x), x 15. If 1 ball is drawn from each of the boxes B1, B2 and B3, the
4 4 probability that all 3 drawn balls are of the same colour is
1 82 90
(b) f '( x) f ( x),0 x (a) (b)
4 648 648
1
(c) f '( x ) f ( x ),0 x 558 566
4 (c) (d)
648 648
3 16. If 2 balls are drawn (without replacement) from a randomly
(d) f'( x) f ( x), x 1
4 selected box and one of the balls is white and the other ball
Paragraphs for Questions 11 and 12 is red, the probability that these 2 balls are drawn from box
Let PQ be a focal chord of the parabola y2 = 4ax. The tangents to B2 is
the parabola at P and Q meet at a point lying on the line y = 2x + a, 116 126
a > 0. (a) (b)
181 181
11. Length of chord PQ is
(a) 7a (b) 5a (c) 2a (d) 3a 65 55
(c) (d)
181 181
EBD_780
14-2013 JEE Advanced 2013 Solved Paper
SECTION - III
This section contains 4 multiple choice questions. Each question has matching lists. The codes for the lists have choices (a), (b), (c)
and (d) out of which ONLY ONE is correct.
17. Match List I with List II and select the correct answer using the code given below the lists :
List I List II
1/2
1 1
1 cos(tan y ) y sin(tan y)2 4 1 5
P. y takes value 1.
y2 cot(sin 1
y ) tan(sin 1
y) 2 3

Q. If cosx + cosy + cosz = 0 = sinx + siny + sinz then 2. 2


x y
possible value of cos is
2

1
R. If cos x cos 2x + sinx sin 2 secx = cosx sin2x secx + 3.
4 2

cos x cos 2x then possible value of secx is


4

1
S. If cot sin 1 x2 sin tan 1
x 6 ,x 0, 4. 1

then possible value of x is


Codes:
P Q R S
(a) 4 3 1 2
(b) 4 3 2 1
(c) 3 4 2 1
(d) 3 4 1 2
18. A line L : y = mx + 3 meets y – axis at E(0, 3) and the arc of the parabola y2 = 16x, 0 y 6 at the point F(x0, y0). The tangent to the
parabola at F(x0, y0) intersects the y-axis at G(0, y1). The slope m of the line L is chosen such that the area of the triangle EFG has
a local maximum.
Match List I with List II and select the correct answer using the code given below the lists :
List I List II
1
P. m= 1.
2
Q. Maximum area of EFG is 2. 4
R. y0 = 3. 2
S. y1 = 4. 1
Codes:
P Q R S
(a) 4 1 2 3
(b) 3 4 1 2
(c) 1 3 2 4
(d) 1 3 4 2
19. Match List I with List II and select the correct answer using the code given below the lists :
List I List II
P. Volume of parallelepiped determined by vectors a , b and c is 2. 1. 100
Then the volume of the parallelepiped determined by vectors
2( a b ), 3(b c ) and 2(c a ) is
JEE Advanced 2013 Solved Paper 2013-15

Q. Volume of parallelepiped determined by vectors a , b and c is 5. 2. 30


Then the volume of the parallelepiped determined by vectors
3( a b ),3(b c ) and 2(c a ) is
R. Area of a triangle with adjacent sides determined by vectors a and 3. 24

b is 20. Then the area of the triangle with adjacent sides determined
by vectors (2a 3b ) and (a b ) is
S. Area of a parallelogram with adjacent sides determined by vectors 4. 60
a and b is 30. Then the area of the parallelogram with adjacent

sides determined by vectors (a b ) and a is


Codes:
P Q R S
(a) 4 2 3 1
(b) 2 3 1 4
(c) 3 4 1 2
(d) 1 4 3 2
20. Consider the lines
x 1 y z 3 x 4 y 3 z 3
L1 : , L2 and the planes P1 : 7x + y + 2z = 3, P2 =3x + 5y – 6z = 4.
2 1 1 1 1 2
Let ax + by + cz = d be the equation of the plane passing through the point of intersection of lines L1 and L2, and perpendicular
to planes P1 and P2.
Match List I with List II and select the correct answer using the code given below the lists :
List I List II
P. a = 1. 13
Q. b = 2. – 3
R. c = 3. 1
S. d = 4. – 2
Codes:
P Q R S
(a) 3 2 4 1
(b) 1 3 4 2
(c) 3 2 1 4
(d) 2 4 1 3
EBD_780
16-2013 JEE Advanced 2013 Solved Paper

SOLUTIONS
Paper - 1
PHYSICS For a phase difference of 2 the path difference is

SECTION - I For a phase difference of (2n+1) the path difference


2
1. (c) For a plano convex lens
is (2n 1) . option (b) is correct.
1 (µ 1) 1 1 4
...(i)
f R v u 5. (d) P1M1 = d1RT
P2M2 = d2RT
a a 3
Here µ 1.5 P1 M1 d1
m 2 2
a P2 M2 d2
3
Where a= wavelength of light in air 4 2 d1
m = wavelength of light in water 3 3 d2

R d1 8
d2 9 option (d) is correct.

6. (a) Given HI = HII


A T (A A) T
k1 t1 k2 t2
v = 8m
v 1 k1 t1
Also m u 24 cm. t2 ...(i)
u 3 k2 4
1.5 1 1 1 1 1 1
From (i) Where k1 and k2 are the equivalent conductivities in
R 8 24 8 24 6 configuration I and II respectively.
R = 3m option (c) is correct For configuration I :
2. (b) Reading = M.S.R + No of division of V.S matching the
2 3
main scale division (1MSD – 1VSD)
k1 k 2k k1 2k
2.45
= 5.10 24 0.05
50 4k
k1 ...(ii)
= 5.124 cm Option (b) is correct. 3
For configuration II :
ˆi 3 ˆ ˆi 3ˆ k2(A+A) = kA + 2kA
j . j
2 2 2 2 3k
3. (a) cos(180 2 ) k2 ...(iii)
2 2 2 2
1 3 1 3
2 2 2 2 4k
3 9
From (i) , (ii) and (iii) t 2 2sec
1 3k 4
cos(180 2 ) 180°-2
2 2
180° – 2 = 120° option (a) is correct
= 30°
3 9
option (a) is correct E P t 30 10 100 10 17 1
7. (b) p 8
10 kg ms
c c 3 10
4. (b) The intensity I is given as option (b) is correct
I Io cos 2
where Io is the peak intensity 8. (a) Activity B to M for particle thrown upwards
2
u 02 sin 2
Here I
Io Io 2 v12 u 02 2( g)
, I o cos (2n 1) 2g
2 2 2 2
JEE Advanced 2013 Solved Paper 2013-17

SECTION - II
v12 u 02 (1 sin 2 ) u 02 cos 2
11. (b, c)
v1 = u0 cos ...(i)
y = [0.01 sin (62.8x)] cos (628 t).
2mv sin 2mv M
y
1
x 2mv cos Length of string = 5 5 0.25 m
2 20
u 20 sin 2
H
u0 2g 2
u0 62.8
m muo cos m
The midpoint M is an antinode and has the maximum
Applying conservation of linear momentum in displacement = 0.01 m
Y-direction /k
The fundamental frequency = 20Hz
2mv sin = mv1 = mu0cos ...(ii) [from (i)] 2l 2l
Applying conservation of linear momentum in 12. (a, d)
X-direction Consider the equilibrium of the system of both spheres
2mv cos = mu0 cos ...(iii) and the spring.
on dividing (ii) and (iii) we get

tan = 1 =
4
option (a) is correct 2
9. (d) Let us consider a point on the circle
The equation of circle is x2 + y2 = a2
The force is

xiˆ yjˆ 3
F K
(x 2 y2 )3/ 2 (x 2 y2 )3/ 2

y
4 3 4 3
The weight of system = R (3 )g R g
3 3
(0, a) ds
4 3
P(x, y) 4 R g
3
F This is to be balanced by the buoyant force.
yjˆ This can be possible only when the light sphere is
x completely submerged. In this way the buoyant force
xiˆ (a,0)
Fs B

xiˆ yjˆ
F K
2 3/ 2 2 3/ 2
(a ) (a )
W
K
F xiˆ yjˆ 4 4
a3 B R3 2 (2 ) g 4 R3 g
3 3
The force acts radially outwards as shown in the figure
and the displacement is tangential to the circular path. Now considering the equilibrium of the heavy sphere
Therefore the angle between the force and displacement Fs + B = W
is 90° and W = 0 Fs = W – B
option (d) is correct. 4 3 4 3
Kx R (3 )g R (2 )g
3 3
F / (2R) 2 F / R2
10. (c) y
1 / 2L 2 /L 4 R3 g
x
3 K
2
2 (a) and (d) are correct options.
1 13. (a, c)
option (c) is correct The magnetic field should be in the –z direction
(Fleming’s left hand rule)
EBD_780
18-2013 JEE Advanced 2013 Solved Paper
vy 15. (b,d)
2
tan Step 1 : When S1 is pressed. The capacitor C1 gets
vx 2 3 charged such that its upper plate acquires a positive
charge + 2 CV0 and lower plate – 2 CV0.
Step 2 : When S2 is pressed (S1 open). As C1 = C2 the
6 charge gets distributed equal. The upper plates of
y C1 and C2 now take charge + CV0 each and lower plate
– CV0 each.
O z x (b) and (d) are correct option.
u2 = v
× × × × ×
vy = 2
× SECTION - III
× × × × × × 16. (1) For photoelectric effect
× × × × vx ×2 3 × Vo
× × × × × × h o
Vo
e e
Q,M×+ × ×
ui u 4iˆ
× × ×
× × × × × × The slope is
× × × × × × h
tan constant
e
arc speed time
angle The ratio will be 1.
radius radius
17. (4) For a radioactive decay
4 10 10 3 Mv
N N0 e t
radius
6 M 4 / QB QB
N t
50 M e
B N0
3Q
(a) and (c) are the correct options N t
1 1 e
14. (d) Electric field E1 due to smaller sphere at P is N0

4 0.693
1 R3 N0 N
t
1 3 t1 2
E1 1 e
4 0 (2R)2 N

1 e 0.04 1 (1 0.04)
+ + + + + 0.04 4% [ e x 1 x x <<1]
+ + + + + + +
+ +
+ + + + E2 P E1 + + + + 18. (5) Here K.E. = W = P × t
+ + + + + + + +
+ + + 1
2R R + + + + mv 2 P t
+ + 2
+ + + + + +
1
+ + + + + 2Pt 2 0.5 5
v 5ms 1
m 0.2
2
19. (8) Applying conservation of angular momentum
I 1 1 = I2 2
1 1R 1 R
E1 1
4 0 4 0 3 3 MR 2 1
Electric field E2 due to bigger sphere at P is 1 1 2
2
I2 1
MR 2 2[2mr 2 ]
E2 2R 2
3 0
1
50 0.4 0.4 10
As E1 1R 2R 2
E2
4 3 3 1
0 0 50 0.4 0.4 2[2 6.25 0.2 0.2]
2
1 40
4 8 rad / s
2 4 1
option (d) is correct.
JEE Advanced 2013 Solved Paper 2013-19

20. (5) Velocity at the highest point of bob tied to string 1 is 6. (b) The slow decomposition of HNO3 is represented by
acquired by the bob tied to string 2 due to elastic the eqn.
head-on collision of equal masses 4HNO3 4NO2 + 2H2O + O2
(yellow-brown)
Therefore g 1 5g 2 7. (a) The arrangement given shows octahedral void
arrangement-limiting radius ratio for octahedral void is
1
5 r
2 A
= 0.414
r
CHEMISTRY X–
rA+ = 0.414 × rX– = 0.414 × 250 = 103.5 104 pm.
SECTION - I 8. (d) The group reagent of fourth group is ammoniacal H2S
1. (c) The standard enthalpy of the combustion of glucose by which Zn2+ ion will be precipitated as ZnS, whereas
can be calculated by the eqn. Fe3+ ion and Al3+ ions will be precipitated as hydroxides.
C6H12O6(s) + 6O2(g) 6CO2(g) + 6H2O(l) 9. (b) The adsorption of methylene blue on activated charcoal
HC = 6 × Hf(CO2) + 6 × Hf (H2O) – Hf [C6H12O6] is an example of physiosorption which is exothermic,
multilayer and does not have energy barrier.
H° = 6 (–400) + 6(–300) – (–1300)
10. (a) Silver, copper and lead are commonly found in earth's
H° = –2900 kJ/mol
crust as Ag2S (silver glance), CuFeS2 (copper pyrites)
For one gram of glucose, enthalpy of combustion and PbS (galena)
2900 SECTION - II
H° = – 16.11kJ / gm
180
11. (b,c,d)
2. (b) For ideal solution, Ssystem > 0
O Ssurrounding = 0
Cl Hmixing = 0
Compounds : CH3 – Cl : Cl : Cl
12. (b, d) The pair of complex ions [Co(NH3 ) 4 Cl 2 ] + and
S P R Q [Pt(NH3)2(H2O)Cl]+ show geometrical isomerism.
Relative reactivities 1,00,000 : 200 : 79 : 0.02 The pair of complexes [Pt(NH3 ) 3 (NO 3 )]Cl and
towards SN 2 reaction [Pt(NH3)3Cl]Br show ionisation isomerism. The other
pairs given do not have same type of isomerism.
3. (d) Carbolic acid (Phenol) is weaker acid than carbonic acid 13. (a) As ester hydrolysis is first order with respect [H+].
and hence does not liberate CO2 on treatment with aq. RHA = K[H+]HA [ester]
NaHCO3 solution. RHX = K[H+]HX [ester]
4. (b) The electronic configuration of central metal ion in
complex ions P, Q and R are R HA [H ]HA
R HX [H ]HX
P = [FeF6]3–; Fe3+ :
3d 1
[H ]HA = 0.01
100
Q = [V(H2O)6]2+; V2+ H+ +
3d HA A–
1– 0.01 0.01 0.01
1
R = [Fe(H2O)6]2+; Fe2+ 0.01 0.01
3d 4
Ka 10
Higher the no. of unpaired electron(s), higher will be 1
magnetic moment. 14. (a)
Thus the correct order of spin only magnetic moment
is H H
Q<R<P |
CH2 CH2
5. (d) For P, if t50% = x
| ||
then t75% = 2x CH3– C CH3– C
This is true only for first order reaction.
| |
So, order with respect to P is 1. CH3 CH3
Further the graph shows that concentration of Q
decreases with time. So rate, with respect to Q, remains In tert butyl cation, carbon bearing positive charge has
constant. Hence, it is zero order wrt Q. one vacant p-orbital hence it is –p (empty) conjugation
So, overall order is 1+ 0 = 1 or hyperconjugation.
EBD_780
20-2013 JEE Advanced 2013 Solved Paper

H
H CH2
CH2 –
CH–CH–CH3
CH=CH–CH3
In 2-butene, hyperconjugation is between bond.
15. (a, b, c, d)
Cl

AlCl3 –
NaH
AlCl4 ; Na H2

Aromatic
Aromatic
(P)
(Q)

NH3 100º 115ºC


–2H 2 O
from (NH4)2CO3
O O O NH OH HO OH
:

2 N N
| |
H H
Aromatic
(R)
(NH4)2CO3 2NH3 + CO2 + H2O

O OH
|| |
HCl

Cl

Aromatic
(S)
Note : P has 2 electrons, while Q, R & S have 6 electrons each. Hence all the 4 are aromatic.
SECTION - III
16. (6) Structure of melamine is as follows :
.. .. ..
H2N N NH2

N N
..

..

NH
.. 2
Total no. of lone pairs of electron is ‘6’.
O O O O O
COOH
17. (2) 1.H3 O O3 / H 2 O 2 HOOC
O –2CO 2 HOOC
COOH
O O O O O

No. of –COOH group is '2'


JEE Advanced 2013 Solved Paper 2013-21

18. (5) Since, ˆj 2kˆ and a – b


a b 3iˆ iˆ 3 ˆj 4kˆ
h h
(sin ce K.E. T) a 2iˆ ˆj 3kˆ, b iˆ 2 ˆj kˆ
mV 2M K.E
Let c PT iˆ 2 ˆj 3kˆ
1
Then the volume of parallelepipe determined by vectors
MT
For two gases, 2 1 3
c
a , b , and is = 1 2 1 = 10
He M Ne TNe 20 1000
1 2 3
Ne M He THe 4 200 = 5
3. (c) As lies on the circle (x – x0)2 + (y – y0)2 = r2
– | – z0|2 = r2
19. (8)
(I) ( z0 )( z0 ) = r2
O O –C=O
z0 z0 z0 z0 = r2
C CH2 CH |
2
2 2
(II) O N z0 z0 z0 = r2 (i)
(I) CH2
| 1
Co Also lies on the circle (x – x0)2 + (y – y0)2 = 4r2
(II) CH2
(III) O N 1
2

C CH2 CH z0 = 4r2
2
O O–C=O
(IV) 1 1
z0 z0 4r 2

Total no. of N – Co – O bonds are 8. 1 z0 z0


z0 z0 4r 2
20. (4) According to question C – Terminal must be alanine
and N – Terminal do have chiral carbon means it should
1 z0 z0
not be glycine. So possible sequence is : | z0 |2 4r 2
2 2
Val Phe Gly Ala | | | | | |2
Val Gly Phe Ala 1 | |2 | z0 |2 z0 z0 4r 2 | |2 (ii)
Phe Val Gly Ala n
Subtracting eq (i) from (ii) we get
Phe Gly Val Ala
1–| |2 | z0 |2 (| |2 1) r 2 (4 | |2 1)

MATHEMATICS or (| |2 1)(| z0 |2 1) r 2 (4 | |2 1)

SECTION - I r2 2
Using | z0 |2 we get
2
r2
1
n (| |2 1) r 2 (4 | |2 1)
1. (b) cot 1 2k = cot–1 [1 + n(n+1)] 2
k 1
| |2 1 8 | |2 2
1 ( n 1) n 1
= tan = tan–1 (n + 1) – tan–1n | |
1 (n 1) n 7
23 c c
23
[tan 1 (n 1) tan 1
n] tan 1
24 tan 1 1 tan 1 4. (a) The intersection point of two lines is ,
n 1 25 a b a b
c c
23
1
n
1 23 25 Distance between (1, 1) and , 2 2
cot cot 1 2k cot tan a b a b
n 1 k 1 25 23
2
c
2 1 8
2. (c) Let a and b represent the adjacent sides PQ and a b
PS of the parallelogram PQRS then c
1 2 a+b–c>0
a b
EBD_780
22-2013 JEE Advanced 2013 Solved Paper
5. (d) For some value of , point A on line PA is dy y y
(2 – 2, – – 1, 3 ) 8. (a) sec
dx x x
dy dv

= z
P(–2, –1, 0) Putting y = vx and v x , we get

3
–1 1
dx dx

+
= y dv
2 2
x = sec v
x+

dx
dx
or cos v dv =
A N x
sin v = log x + c ( x 0)
x+y+z=3
y
sin = log x + c
As it lies on x + y + z = 3 x
2 – 2 – –1 + 3 =3
1
3 5 9 It passes through 1, C
= A 1, , 6 2
2 2 2
y 1
x 2 y 1 z sin= log x
Also equation of PN is x 2
1 1 1
9. (d) We have f (x) – 2 f(x) < 0
For some value of , N ( – 2, – 1, )
2x 2x
N lies on x + y + z = 3 = 2. e f '( x ) 2e f ( x) 0
N(0, 1, 2)
d 2x
7 5 (e f ( x )) 0
dr’s of AN are 1, , or 2, –7, 5 dx
2 2
Equation of AN is e 2 x f ( x) is strictly decreasing function on
x y 1 z 2 1
= ,1
2 7 5 2
6. (a) P (atleast one of them solves the problem)
2x 1 1
= 1 – P (none of them solves it) e f ( x) e f
2
1 1 3 7 21 235
=1 1
2 4 4 8 256 256 or f ( x) e2 x 1

7. (b) The rough graph of y = sin x + cos x and Also given that f(x) is positive function so f(x) > 0
y = |cos x – sin x| suggest the required area is
0 f ( x) e2 x 1
2
= [(sin x cos x ) | cos x sin x |] dx 1 1
0
0 f ( x)dx e2 x 1dx
12 12
y
1
1 e2 x 1
y = sin x + cos x 0 f ( x)dx
12 2
12

1 1 e 1
y = |cos x – sin x| f ( x )dx 0,
12 2
x
0 /4 /2 10. (c) Let f(x) = x2 – x sin x – cos x
f (x) = 2x – x cos x = x(2 – cos x)
4 2
= 2sin x dx 2 cos x dx f is increasing on (0, ) and decreasing on (– , 0)
0 4
Also lim f(x) = , lim f(x) = and f(o) = –1
x x
= 2 ( cos x )0 4 (sin x) 24
y = f(x) meets x-axis twice.
= 2 2( 2 1) i.e., f(x) = 0 has two points in (– , ).
JEE Advanced 2013 Solved Paper 2013-23

SECTION - II is (2 , –3 , 2 )
11. (a, c) Let L = 8x, B = 15x and y be the length of square cut off For Intersection point P of and 1
from each corner. Then volume of box t + 3 = 2 , 2t – 1 = –3 , 2t + 4 = 2
= (8x – 2y) (15x – 2y)y t = –1, = 1 P( 2, –3, 2)
V = 120x2y – 46xy2 + 4y3 Any point Q on 2 is (2s + 3, 2s + 3, s + 2)
dV As per question PQ =
= 120x2 – 92xy + 12y2 17
dy
(2s + 1)2 + (2s + 6)2 + s2 = 17
dV 9s2 + 28s + 20 = 0
Now 0 at y = 5 for maximum value of V.
dy
10
[30x2 – 23xy + 3y2]y = 5 = 0 s = –2,
9
6x2 – 23x + 15 = 0
7 7 8
5 Point Q can be (–1, –1, 0) and , ,
x = 3, 9 9 9
6
For x = 3, sides are 45 and 24. 14. (b, c) We have f(x) = x sin x, x > 0
12. (a,d) Sn = –12 – 22 + 32 + 42 –52 – 62 + .... f (x) = sin x + x cos x
= (32 + 72 + 112 + ...) + (42 + 82 + 122 + ...) f (x) = 0 tan x = – x
–(12 + 52 + 92 + ...) – (22 + 62 + 102 + ...) y
n n n n
= (4 r 1)2 (4 r ) 2 (4r 3) 2 (4 r 2) 2
r 1 r 1 r 1 r 1

n n x 0
= (4r 1)2 – (4r 3) 2 4 (2 r ) 2 (2 r 1) 2 1/2 1 3/2
r 1 r 1

n n
= 8 (2r 1) 4 (4r 1)
r 1 r 1 y
We observe, from graph of y = tan x and y = – x that
n(n 1) n(n 1)
= 8 2 n 4 4 n they intersect at unique point in the intervals
2 2
= 8n2 + 8n2 + 4n = 16n2 + 4n 1
(n, n + 1) and n ,n 1
For n = 8, 16n2 + 4n = 1056 2
and for n = 9, 16n2 + 4n = 1332 15. (c, d)
13. (b, d) The given lines are (a) (N M N) = (M N) N = N M N = N M N or –N M N
x 3 y 1 z 4 According as M is symm. or skew symm. correct
1: t
1 2 2 (b) (MN – NM) = (MN) – (NM) = N M – M N
x 3 y 3 z 2 = NM – MN = –(MN – NM)
2 : s
2 2 1 It is skew symm. Statement B is also correct.
Let direction ratios of be a, b, c then as 1 and 2 (c) (MN) = N M = NM MN
a + 2b + 2c = 0 Statement C is incorrect
2a + 2b + c = 0
(d) (adj M ) (adj N) = adj (MN) is incorrect.
a b c
= SECTION - III
2 3 2

x y z x2 y2
: = 16. (9) Vertical line x = h, meets the ellipse 1 at
2 3 2 4 3
Any point on 1 is (t + 3, 2t – 1, 2t + 4) and any point on 3 3
P h, 4 h2 and Q h, 4 h2
2 2
EBD_780
24-2013 JEE Advanced 2013 Solved Paper
By symmetry, tangents at P and Q will meet each other 17. (6) Let the coefficients of three consecutive terms of
at x-axis. (1 + x)n + 5 be
y n + 5C , n + 5C , n + 5C , then we have
r–1 r r+1
n + 5C n + 5C : n + 5C
r–1 : r r+1 = 5 : 10 : 14
n 5
P Cr 1 5 r 1
n 5 10 n 6 r 2
Cr
x or n – 3r + 6 = 0 ...(1)
0 (h, 0) R
n 1
Q Cr 10 r 1 5
Also n 5
Cr 1 14 n r 5 7
or 5n – 12r + 18 = 0 ...(2)
xh y 3 Solving (1) and (2) we get n = 6.
Tangent at P is 4 h2 1 18. (5) Given 8 vectors are
4 6
(1, 1, 1), (–1, –1, –1); (–1, 1, 1), (1, –1, –1); (1, –1, 1),
4 (–1, 1, –1); (1, 1, –1), (–1, –1, 1)
which meets x-axis at R ,0
h These are 4 diagonals of a cube and their opposites.
For 3 non coplanar vectors first we select 3 groups of
1 4 diagonals and its opposite in 4C3 ways. Then one
Area of PQR = 3 4 h2 h
2 h vector from each group can be selected in 2 × 2 × 2
ways.
3 (4 h 2 )3 2 Total ways = 4C3 × 2 × 2× 2 = 32 = 25
i.e., (h) = p=5
2 h
19. (6) Let P(E1) = x; P(E2) = y, P(E3) = z
d 4 h 2 (h 2 2) P(only E1) = x(1 – y) (1 – z) =
= 3 0 P(only E2) = (1 – x) y (1 – z) =
dh h 2
P(only E3) = (1 – x) ( 1 – y) z =
P(none) = (1 – x) (1 – y) (1 – z) = p.
(h) is a decreasing function.
Now given ( – 2 ) p =
1 1 x = 2y
h 1 max and min = (1)
2 2 and ( – 3r)p = 2 r
y = 3z
1
32 x = 6z
4
3 4 45 P ( E1 ) x
1 = 5 Hence = 6
2 1 8 P ( E3 ) z
2 20. (5) Let k, k + 1 be removed from pack.
(1 + 2 + 3 + ... + n) – (k + k + 1) = 1224
33 3 9
2 = n(n 1)
2 1 2 2k = 1225
2
8
1 8 2 = 45 – 36 = 9 n(n 1) 2450
5 k=
4
for n = 50, k = 25
k – 20 = 5

Paper - 2
PHYSICS n2 9
4.5 4.5 z 2
SECTION - I z z
we know that
nh 3h
1. (a, c)Angular momentum . Therefore n = 3.
2 2 1 1 1
R z2
ao n2 n12 n 22
Also rn 4.5a o
z
JEE Advanced 2013 Solved Paper 2013-25

4. (d) Given 2d sin =


1 1 1
4R
n12 n 22 d= cosec ... (i)
2
9 9 Differentiating the above equation w.r.t ' ' we get
For n2 = 3, n1 = 1 we get
8 4R 32R
d(d)
36 9 cosec cot
For n2 = 3, n1 = 2 we get d 2
5 4R 5R

4 1 d (d) = – cosec cot d ... (ii)


For n2 = 2, n1 = 1 we get 2
3 4R 3R on dividing (i) and (ii) we get
(a), (c) are correct options
2. (a, b, c, d) We know that dQ = m C dT in the range 0 to 100K d(d)
= cot d
From the graph, C increases linearly with temperature d
therefore the rate at which heat is absorbed varies
As increases from 0° to 90°, cot decreases and
linearly with temperature. Option (a) is correct
As the value of C is greater in the temperature range d(d)
400-500K, the heat absorbed in increasing the therefore decreases option (d) is correct
d
temperature from 0 - 100K is less than the heat required
for increasing the temperature from 400 - 500K option cos
(b) is correct. From (ii) |d (d)| = 2
From the graph it is clear that the value of C does not 2 sin
change in the temperature range 400-500K, therefore
there is no change in the rate of heat absorption in cos
This value of decreases as increases from
this range. Option (c) is correct. sin 2
As the value of C increases from 200-300K, the rate of
heat absorption increases in the range 200-300K. Option 0° to 90°
(d) is also correct. Source Observer
3. (c, d) Let us consider a point P on the overlapping region. 5. (a,b) S O
u u
The electric field intensity at P due to positively f1 f2
r1
charged sphere = Wind blows from observer tosource
3 0 case1:
(V w) u f 2 f1
f 2 f1
The electric field intensity at P due to negatively (V w) u

r2 Wind blows fromsource to observer


charged sphere = . The total electric field, case2:
3 0
(V w) u f 2 f1
f 2 f1
r1 r2 (V w) u
E r1 r 2
3 0 3 0 3 0 (a) and (b) are correct options
6. (a, d) In the region O < r < R, the magnetic field is present due
E r to current in solenoid.
3 0
Therefore the electric field is same in magnitude and
direction option (c) and (d) are correct.

P
r1 r2
+ – In the region r > 2 R, the magnetic field is present due to
r the current in the cylinder.
For the region R < r < 2R, the magnetic field is neither
along the common axis, nor tangential to the circle of
radius r. (a) and (d) are correct options.
EBD_780
26-2013 JEE Advanced 2013 Solved Paper
7. (a, d) The particle collides elastically with rigid wall. Here 210 206 4
9. (a) 84 Po 82 Pb 2 He
V
e 1 V 0.5u 0 Here m = [209.982876 –(205.974455 + 4.002603)]× 932 MeV
0.5u 0
= 5.422 MeV = 5422 keV
u0 By conservation of linear momentum
Linear momentum of -particle = linear momentum of
lead
B p = plead
A 0.5u0 2m K.E = 2m lead K.E lead
C

Equilibrium position m lead K.E lead 206 K.E lead


K.E ...(1)
m 4
i.e. the particle rebounds with the same speed. Therefore
the particle will return to its equilibrium position with Also K.E + K.Elead = 5422 keV ...(2)
speed u0. option (a) is correct. On solving the above two equations we get
The velocity of the particle becomes 0.5u0 after time t.
Then using the equation V = Vmax cos wt we get K.E = 5319 keV
0.5u0 = u0 cos wt (a) is the correct option.
2 T 10. (c) Only in case of c we have m3 + m4 > M
T t (c) is the correct option.
3 t 6
In other cases of fission m1 + m2 M and in the other
m m case of fusion m3 + m4 M
The time period T 2 . Therefore t =
k 3 k 11. (b) Given M = L
The time taken by the particle to pass through the M = m R2
2 m M = m ( ) R2 ...(1)
equilibrium for the first time = 2t = . Therefore
3 k
Q×B
option (b) is incorrect But = ...(2)
The time taken for the maximum compression 2m
= tAB + tBA + tAC
QB BQR 2
m m m m 1 1 1 From (1) and (2) M = m R2
= + + 2m 2
3 k 3 k 3 k k 3 3 2
The negative sign shows that change is opposite to
7 m the direction of B.
= . Therefore option c is incorrect.
6 k (b) is the correct option.
The time taken for particle to pass through the
d d dB
equilibrium position second time 12. (b) E .dl = (B R 2 ) R2
dt dt dt
m m m 2 5 m
= 2 + = +1 =
3 k k k 3 3 k R 2B
option (d) is correct. E × 2 R = – R2B
1 GMm GM BR
8. (b) mv 2 = 2 v 2 E=
2 L L 2
The potential energy is a combined property of the three (b) is the correct option.
mass system. The kinetic energy of mass m is only its energy
which decreases as it moves. Ns Vs 10 Vs
13. (a) For step up transformer
(b) is the correct option. Np Vp 1 4000

Vs = 40,000 V

Np Vp 40,000 200
For step down transformer =
Ns Vs 200 1

(a) is the correct option.


JEE Advanced 2013 Solved Paper 2013-27

14. (b) We know that P = V × I (a) is the correct option


P 600 1000 18. (c) 15 15 0
I= 8 O 7 N 1
V 4000 particle
I = 150 A
238 234 4
Total resistance = 0.4 × 20 = 8 92 U 90 Th 2 He
particle
Power dissipated as heat = I2R = (150)2 × 8 = 180,000W
= 180 kW 185 184 1
83 Bi 82 Pb 1H
180 proton
% loss = 100 30%
600 239 140 99
(b) is the correct option. 94 Pu 57 La 37 X
(c) is the correct option.
15. (a) 19. (d) e f. For the ray to bend towards the normal at the
30° prism surface 2 > 1. The ray then moves away
40 sin 30° from the normal when it emerges out of the
40m rectangular block. Therefore 2 > 3.
e g. As there is no deviation of the ray as it emerges
N out of the prism, 2 = 1.
e h. As the ray emerges out of prism, it moves away
mg sin 60° from the normal. Therefore 2 < 1 . As the ray
v moves away from the normal as it emerges out
60°
of the rectangular block, therefore 2 > 3.
mg cos 60° mg
e i. At the prism surface, total internal reflection has
h2
mv2 taken place. For this sin 45° >
N – mg cos 60° = h1
r
1> 2 2.
mv2 (d) is the correct option.
N = mg cos 60° + ...(1)
r
Loss in P.E. = mg × 40 sin 30° = 200 J R PV ML 1T 2 L3
20. (c) Boltzmannn constant =
Work done in over coming friction = 150 J N nTN K
K.E. possessed by the particle = 50 J
ML2 T 2 K 1
1
mv2 50J F MLT 2
2 Coefficient of viscosity =
6 rv 1
mv2 = 100 J ...(2) L LT
1 100 ML 1T 1
From (1) and (2), N = 1 × 10 × = 5 + 2.5 = 7.5 N
2 40
(a) is the correct option. E ML2T 2
Planck constant = 1
ML 2T 1
16. (b) From (2), mv2 = 100 v T
v = 10 ms–1
(b) is the correct option. H ML2 T 2
L
Thermal conductivity = 2
17. (a) WGE = P0 (V0 – 32 V0) = – 31 P0V0 tA T T L K
WGH = P0 (8V0 – 32V0) = – 24 P0V0
= MLT–3K–1
P0 (8V0 ) 32P0 (V0 ) (c) is the correct option.
(WFH)adiabatic = 36P0 V0
5
1
3 CHEMISTRY
32V0 SECTION - I
(WFG) isothermal = 1(32 P0V0) loge
V0
1. (b) Ag 2 CrO 4 2Ag CrO24
= 32 P0V0 loge 25
= 160 P0V0 loge2 Ksp = 1.1 × 10–12 = [Ag+]2 [CrO 4 2 ]
EBD_780
28-2013 JEE Advanced 2013 Solved Paper
1.1 × 10–12 = [0.1]2 [s]
s = 1.1 × 10–10
OH O OH
CHO
2. (b, d) CHCl3 +
OH

H3C CHCl2 (S)


(Minor)
CH3 CH3 (major)
(Q)
CHCl3 OH :CCl2 H 2 O Cl


OH O

OH +H2O

CH3 CH3

O O OH OH

CCl2 CHCl2 CHO
+: CCl2 H OH

CH3 CH3 CH3 CH3 (major)



O O O

H 2O


: CCl2 H3C CCl2 H3C CHCl2 (minor)
CH3

OH OH OH
3. (b)
Br Br Br Br
Aq. Br2 (3.0 equivalents)
SO 3
O
Br S
SO3H O H Br
O
O O
Br2 (1.0 mol)
4. (c) Reaction I : CH3COONa + CHBr3 +
CH3 CH3 Na OH
(T) (U) CH3 CH3
1.0 mol (unreacted)
O O
Br2 (1.0 mol)
Reaction II : CH3 CH 3COOH CH2Br
CH3 CH3
1.0 mol (P)
JEE Advanced 2013 Solved Paper 2013-29

5. (a, c, d)

O O
O O– –O O
Ozone is diamagnetic in nature (due to presence of paired electron) and both the O – O bond length are equal. It has a bent
structure.
6. (a, b) The reaction can occur by following two ways.
9 0 8 1
4 Be 0 4 Be 0n

9 1 2 8
4 Be 1H 1H 4 Be
7. (c, d) Al from Al2O3 and Mg from MgCO3.CaCO3 are separately extracted by electrolytic reduction.
8. (a, b, d)
(A) H = CP(rxn) T
Hence enthalpy depends on temperature.
(B) CaCO3(s) CaO(s) + CO2(g) p= PCO2
(C) Keq depends only on temperature and not on Pressure.
(D) Enthalpy of reaction is independent of the catalyst. Catalyst generaly changes activation energy.
9. (a) Lead salts give white precipitate of PbCl2 with dil. HCl which is soluble in hot water.
Pb++ + 2Cl– ——— PbCl2 (White ppt) soluble in hot water
10. (d) The filtrate on treatment with ammoniacal H2S gives a precipitate which dissolves in aqueous NaOH containing H2O2
giving a coloured solution. It contains Cr3+ ion.
3 H 2S in ammoniacal
Cr 3 NH 4OH Cr(OH)3
medium
(Green)

2Cr(OH)3 3H 2O 2 4NaOH 2Na 2CrO 4 8H 2


(yellow colour)
11. & 12.
COOH
11. (b)
HOOC COOH Cold alk. H OH
C=C KMnO 4
H H
'P' (Cis) (Syn addition) H OH

COOH
Meso (S), optically inactive

COOH COOH
H COOH Cold alk. H OH OH H
C=C KMnO 4 +
HOOC H
'Q' (trans) (Syn addition) OH H H OH

COOH COOH
T U
(Racemic Mixture), optically inactive

H 2 / Ni
12. (a) HOOC–CH=CH–COOH O

Q O
V
EBD_780
30-2013 JEE Advanced 2013 Solved Paper

O O
OH
OH
Anhydrous AlCl3 Zn Hg
+ O
O HCl
O
O
V

H3PO 4

K L

13. (c) Pressure


N M

Volume
K L
V increasing at constant P
Hence T increases (Heating)
L M P decreasing at constant V
Hence T decreases (Cooling)
M N V decreasing at constant P
Hence T decreases (Cooling)
N K P increasing at constant V
Hence T increases (Heating)
14. (b) L to M and N to K, both are having constant volume therefore these processes are isochoric.
cold
Cl 2 2NaOH(dil.) NaOCl NaCl H 2O
15. (a) (P)

hot
3Cl2 6NaOH(conc.) NaClO3 5NaCl 3H 2O
(Q)
(P) and (Q) are salts of HOCl and HClO3 respectively.
Charcoal
16. (a) SO 2 Cl 2 SO2Cl 2
Catalyst
(R)

10SO 2 Cl 2 P4 4PCl 5 10SO 2


(R ) (S)

PCl5 4H 2 O H 3 PO 4 5HCl
(S) (T)

Warm
17. (d) (P) 2PbO2 2H 2SO4 2PbSO4 2H 2 O O 2

(Q) Na 2S2 O3 5H 2O 4Cl 2 2NaHSO 4 8HCl

(R) N 2 H 4 2I 2 N 2 4HI

(S) XeF2 2NO Xe 2NOF


JEE Advanced 2013 Solved Paper 2013-31

NaOEt
18. (a) (P) E2

EtBr
(Q) SN 2

(i) Hg(OAc) 2
(R) (ii) NaBH 4 OH (Markovnikov addition)

(i) BH 3
(S) (ii) H 2 O2 / NaOH
(Antimarkovnikov addition)

OH
19. (a) (p) C 2 H5 3 N CH 3COOH C2 H 5 3 NH CH 3COO
X Y
Initially conductivity increases because on neutralisation ions are created. After that it becomes practically constant
because X alone can not form ions.
(Q) KI 0.1M AgNO3 0.01M AgI KNO3
x y
Number of ions in the solution remains constant as only AgNO3 precipitated as AgI. Thereafter conductance in-
creases due to increase in number of ions.
(R) Initially conductance decreases due to the decrease in the number of OH ions as OH– is getting replaced by CH3COO–
which has poorer conductivity thereafter it slowly increases due to the increase in number of H+ ions.
(S) Initially it decreases due to decrease in H+ ions and then increases due to the increase in OH– ions.

0.77V 0.44V
20. (d) (P) Fe3 n 1
Fe2 n 2
Fe

xV n=3
Go 3 Go 3 Go
Fe / Fe Fe / Fe 2 Fe 2 / Fe

3 FE o 1 FE o 2 FE o
(Fe 3 / Fe) (Fe 3 / Fe 2 ) (Fe 2 / Fe)
3 × x = 1 × 0.77 + 2 × (–0.44)
0.11
x V 0.04 V.
3

(Q) 2H 2 O O 2 4H 4e E 1.23V
4e O 2 2H 2 O 4OH E 0.40 V
4H 2 O 4H 4OH E – 0.83V

(R) Cu 2 2e Cu E 0.34 V
2Cu 2Cu 2e E 0.52 V
Cu 2 Cu 2Cu E 0.18 V

x 0.91V
(S) Cr3 n 1
Cr 2 n 2
Cr

– 0.74V, n=3
x × 1 + 2 × (– 0.91) = 3 × (– 0.74)
x – 1.82 = – 2.22 x = – 0.4V
EBD_780
32-2013 JEE Advanced 2013 Solved Paper
MATHEMATICS or (x – 3)2 + (y + 4)2 = 42
i.e. x2 + y2 – 6x – 8y + 9 = 0
SECTION - I or x2 + y2 – 6x + 8y + 9 = 0
3. (a, d)
1. (b, d)
x 5 y z
1a 2a na 1 L1 :
lim 0 3 2
n (n 1) a 1[(na 1) (na 2) (na n)] 60
x y z
L2 :
a a a 0 1 2
1 2 n
As L1, L2 are coplanar, therefore
na n n n 1
lim
n (n 1) a 1 2 n(n 1) 60 5 0 0
n a
2 0 3 2 0
a
1 n r 0 1 2
a 1
n nr 1 n 1
lim a 1 2
n (n 1) a 1 1 1 60 (5 )[6 5 2] 0
2 n (5 )( 1)( 4) 0
n a 1, 4,5.
a 1 1 r
1 nr 1 n 1 4. (b, d) Let PN = x, QL = x + 2, RM = x + 4
lim where x is an even integer.
n 1 1 1 60
1 a 1
n 2 n P
1 x x
a 1
x dx M
1 xa 1 1
N
0 0
1 60 1 60
2

x+
a (a 1) a
x+

2 2

4
Q x+2 L x+4 R
1 1
Then PM = PN = x, QN = QL = x + 2
1 60
(a 1) a and RL = RM = x + 4
2
So that PQ = 2x + 2, QR = 2x + 6, PR = 2x + 4
2a 2 3a 119 0 1
Now cos P
17 3
a 7 or
2 PQ2 PR 2 QR 2 1
2. (a, c)There can be two possibilites for the given circle as 2PQ.PR 3
shown in the figure
y (2x 2) 2 (2x 4) 2 (2x 6) 2 1
2.(2x 2).(2x 4) 3

(3, 4) 3[(x 1)2 (x 2) 2 (x 3) 2 ] 2(x 1)(x 2)


3
3 (x2 – 4) = 2(x + 1)(x + 2)
7
4 3x – 6 = 2x + 2
4
x=8
x PQ = 18, QR = 22, PR = 20

3 i
5. (c, d) w cos isin
2 6 6
(3, – 4)
n n
and w n cos
i sin
6 6
P contains all those points which lie on unit circle
The equations of circles can be
2 3
(x – 3)2 + (y – 4)2 = 42 and have arguments , , and so on.
6 6 6
JEE Advanced 2013 Solved Paper 2013-33

As z1 P H1 and z2 P H2, therefore z1 and z2 can Then we can write


have possible positions as shown in the figure.
2 x 4, x 2
1 1
Re z Re z 2
2 2 2 x 4, 2 x
3
f ( x) 2
4 x, x 0
3
z2 z1 4 x, 0 x 2
2 x 4, x 2
/6 /6
The graph of y = f(x) is as follows
/6 O /6

z2 z1

2 5
z1Oz2 can be or .
3 6
6. (a, b, c) –3 –2 2 0 2 3
3x = 4x–1 xlog 3 = 2(x – 1)log2 3
2 log 2
x From graph f(x) has local minimum at –2 and 0 and local
2 log 2 log 3
2
maximum at
2 log3 2 2 3
x
2log3 2 –1 2 log 2 3 9. (d) We have f (x) – 2f (x) +f(x) ex
f (x) – f (x) – f (x) – f (x) ex
1 1
Also x
1 1 log 4 3
1 log 2 3 e –x f (x) – e –x f (x) – e –x f (x) – e –x f (x) 1
2
7. (b, c, d) d –x d –x
e f (x) – e f (x) 1
2 3 4 dx dx

3 4 5 d –x
For n = 3, P e f (x) – e – x f (x) 1
4 5 6
dx

d d –x
e f (x) 1
0 0 0 dx dx
and P 2 0 0 0 Let g(x) = e–xf(x)
0 0 0 Then we have g (x) 1 0
It shows P2 = 0 if n is a multiple of 3. So g is concave upward.
So for P2 0, n should not be a multiple of 3 i.e. n can Also g(0) = g(1) = 0
take values 55, 58, 56 g(x) < 0, x (0, 1)
8. (a, b) For f (x) = 2 x x 2 | x 2| 2| x| e–x f(x) < 0
f(x) < 0, x (0, 1)
the critical points can be obtained by solving x 0, 10. (c) g(x) = e–xf(x)
g (x) = e–xf (x) – e–xf(x)
x 2 0 and | x 2 | 2 | x | 0 –x
= e (f (x) – f(x))
2 1
we get x = 0, –2, 2, As x is point of local minima in [0, 1]
3 4
EBD_780
34-2013 JEE Advanced 2013 Solved Paper
1 13. (b) Area of shaded region
g (x) < 0 for x 0,
4
42 60 8 20
1 42 4
and g (x) > 0 for x ,1 4 360 3 3
4
14. (c) S1 : x2 + y2 < 16
1
In 0, , g (x) < 0 (x 1) i(y 3)
4 S2 : Im 0
1 i 3
e–x (f (x) – f (x)) < 0
f (x) < f(x) 3(x 1) (y 3) 0 y 3x 0
2 2 S3 : x > 0
a 2a
11. (b) PQ = at 2 – 2at Then S : S1 S2 S3 is as shown in the figure given
t2 t below.
2 2 2
1 1 1
a t t– 4 t y
t t t
2
1 1
a t t– 4 (0, 4)
t t
x2+y2=16
2
1
a t
t
= 5a x
12. (d) As PQ is the focal chord of y2 = 4ax 60° (4, 0)
Coordinates of P and Q can be taken as
(1, – 3) (2, - 2 3)

2
R P (at , 2at)
y +x 3 = 0
O
s (a, 0)

Q
min 1 3i z = min distance between z and (1, –3)
z s
a 2a
, Clearly (from figure) minimum distance between z s
2 t
t
and (1, –3) from line y x 3 0
2 2a a
P(at , 2at) and Q 2 , 3 3 3 3
t t
i.e. 2
3 1
Tangents at P and Q are
15. (a) Probability that all balls are of same colour
x a
y at and y –xt – = P (all red) + P (all white) + P (all black)
t t
1 3 3 4 1 2 3 2 4 5
which intersect each other at R –a, a t – 6 9 12 6 9 12 6 9 12
t
As R lies on the y = 2x + a, a > 0
82
1 1 648
a t– –2a a t– –1
t t
1W 2W 3W
1 16. (d) B1 3R B2 3R B3 4R
t 5
t 2B 4B 5B
2
Now, mOP and mOQ –2t Let E1, E2, E3 be the events that bag B1, B2 and B3 is
t 1
2 selected respectively.
2t 2 t
t t 2 5 Let E be the event that one white and one red ball is
tan
1– 4 –3 –3 selected.
JEE Advanced 2013 Solved Paper 2013-35

Then by baye’s theorem,


2sin sin x cos 2x 2sin x(cos x – sin x)
P(E \ E 2 )P(E 2 ) 4
P(E2 \ E)
P(E \ E1 )P(E1 ) P(E \ E 2 ) P(E 2 ) P(E \ E3 )P(E 3 )
1
2sin x (cos 2 x – sin 2 x) – (cos x – sin x) 0
2 3 2
9C 2 55
cos x sin x
1 3 2 3 3 4 181 2sin x cos x – sin x –1 0
6 9 12 2
C2 C2 C2
17. (b) sinx = 0 or tanx = 1 or cos x – 1
4
1
2
1 cos (tan 1
y) y sin (tan 1
y)
2 x
(P) y4 x = 0,
2 1 1 4
y cot (sin y) tan (sin y)
secx = 1 or 2
1 (R) (2)
2 2

cos c os 1 1
y sin sin 1 y (S) cos sin –1 1 – x 2 sin tan –1 x 6
2 2
1 1 y 1 y
2
y4
= y
1 1 y 2
1 y x x 6 5
cot cot tan tan x
y 1 y2 1 – x2 1 6x 2 2 3
(S) (1)
Hence (P) (4), (Q) (3), (R) (2), (S) (1)
1
2 2 18. (a) Equation of tangent to y2 = 16x at F (x0, y0)
1 y2 yy0 = 8(x + x0)
1 1 y4 8x 0
= y2 y ( 1 y 2 ) G 0,
y0

1
Area of EFG = (3 y1) x0
1 2
= 1 y4 y4 2 1
1 8x
(P) (4) A x0 3 – 0
2 y0
(Q) we have cos x + cos y = – cos z
sin x + sin y = – sin z
1 y20 y 1 y30
Squaring and adding we get A 3 0 3y20
(cos x + cos y)2 + (sin x + sin y)2 = cos2 z + sin2 z 2 16 2 32 2
2 + 2 cos (x – y) = 1
dA 1 3y02
x y x y 1 6y0
4 cos 2 1 or cos dy0 32 2
2 2 2
Q (3) dA
(R) We have 0 y0 4 x0 1
dy0

cos – x cos2x + sinx sin2x secx 8 1


4 y1= 2
4
= cosx sin2x secx + cos x cos2x Also y0 = mx0 + 3
4 4 = m + 3 or m = 1
maximum area of EFG
cos 2x cos – x – cos x
4 4 1 43 1 1
3 42 48 32
= sin2x secx (cosx – sinx) 32 2 32 2
EBD_780
36-2013 JEE Advanced 2013 Solved Paper

y 1
2a 3b a–b
2
E (0, 3)
1
–2a b 3b a
G (0, y1) F (x1, y0) 2
x 1
5a b
2
5
40 100 (R) (1)
2

(P) (4), (Q) (1), (R) (2), (S) (3) (S) a b 30

19. (c) (P) a b c 2 a b a b a 30

2 a b 3 b c c a (S) (2)
20. (a) Any point on L1 is (2 + 1, – , – 3)
6 a b b c c a and that on L2 is ( + 4, – 3, 2 –3)
For point of intersection of L1 and L2
2 2 + 1 = + 4, – = – 3, – 3 = 2 – 3
6 a b c = 6 × 4 = 24
= 2, = 1
(P) (3) Intersection point of L1 and L2 is (5, –2, – 1)
ax + by + cz = d is perpendicular to p 1 & p2
(Q) a b c 5
7a + b + 2c = 0 and 3a + 5b – 6c = 0
3 a b b c 2 c a a b c
16 48 32
6 a b b c c a
a b c
6 2 a b c = 6 × 2 × 5 = 60 1 3 2
Equation of plane is x – 3y – 2z = d
(Q) (4)
As it passes through (5, –2, –1)
1 5 + 6 + 2 = d = 13
(R) a b 20
2 a = 1, b = –3, c = –2, d = 13
or (P) (3)
a b 40 (Q) (2)
(R) (4)
(S) (1)
IIT-JEE 2012 : PAPER - 1
No. of Questions : 60 Max. Marks : 210
1. The question paper consist of 3 parts (physics, Chemistry and Mathematics). Each part consist of three sections.
2. For each question in Section I, you will be awarded 3 marks if you darken the bubble corresponding to the correct
answer ONLY and zero marks if no bubbles are darkened. In all other cases, minus one (–1) mark will be awarded in
this section.
3. For each question in Section II, you will be awarded 4 marks if you darken. All the bubble(s) corresponding to the
correct answer(s) ONLY. In all other cases zero(0) marks will be awarded. No negative marks will be awarded for
incorrect answer in this section.
4. For each question in Section III, you will be awarded 4 marks if you darken the bubble corresponding to the correct
answer ONLY. In all other cases zero (0) marks will be awarded. No negative marks will be awarded for incorrect.

r r
PHYSICS (A) LO and LP do not vary with time
SECTION-I r r
(B) LO varies with time while LP remains constant
DIRECTIONS : This section contains 10 multiple choice r r
questions. Each question has four choices (A), (B), (C) and (C) LO remains constant while LP varies with time
(D) out of which ONLY ONE is correct. r r
(D) LO and LP both vary with time
æ 4MLg ö
1. In the determination of Young’s modulus ç Y = ÷ 3. A bi-convex lens is formed with two thin plano-convex
è pld 2 ø lenses as shown in the figure. Refractive index n of the
by using Searle’s method, a wire of length L = 2 m and first lens is 1.5 and that of the second lens is 1.2. Both
diameter d = 0.5 mm is used. For a load M = 2.5 kg, an the curved surface are of the same radius of curvature
extension l = 0.25 mm in the length of the wire is R = 14 cm. For this bi-convex lens, for an object distance
observed. Quantities d and l are measured using a screw of 40 cm, the image distance will be
gauge and a micrometer, respectively. They have the
same pitch of 0.5 mm. The number of divisions on their
circular scale is 100. The contributions to the maximum
probable error of the Y measurement
(A) due to the errors in the measurements of d
and l are the same.
(B) due to the error in the measurement of d is twice
that due to the error in the measurement of l .
(C) due to the error in the measurement of l is twice (A) -280.0 cm (B) 40.0 cm
that due to the error in the measurement of d. (C) 21.5 cm (D) 13.3 cm
(D) due to the error in the measurement of d is four 4. A thin uniform rod, pivoted at O, is rotating in the
times that due to the error in the measurement of l. horizontal plane with constant angular speed w, as
2. A small mass m is attached to a massless string whose shown in the figure. At time t = 0, a small insect starts
other end is fixed at P as shown in the figure. The mass from O and moves with constant speed v, with respect
is undergoing circular motion in the x-y plane with to the rod towards the other end. It reaches the end of
centre at O and constant angular speed w. If the angular the rod at t = T and stops. The angular speed of the
momentum of the system, calculated about O and P are system remains w throughout. The magnitude of the
r r
denoted by LO and LP respectively, then torque ( rt ) about O, as a function of time is best
represented by which plot?
z
z
P
w

O m v®

w O
EBD_780
2012-2 IIT-JEE 2012 SOLVED PAPER
(A) 50 m/s (B) 51 m/s
r r
|t| |t |
(C) 52 m/s (D) 53 m/s
(A) (B)
O t O t 9. Young’s double slit experiment is carried out by using
T T green, red and blue light, one color at a time. The fringe
widths recorded are bG, bR and bB, respectively. Then,
r r (A) bG > bB > bR (B) bB > bG > bR
|t| |t | (C) bR > bB > bG (D) bR > bG > bB
(C) (D) 10. Consider a thin spherical shell of radius R with centre at
O t O t the origin, carrying uniform positive surface charge
T T
density. The variation of the magnitude of the electric
5. A mixture of 2 moles of helium gas (atomic mass = 4 r
amu) and 1 mole of argon gas (atomic mass = 40 amu) field E(r ) and the electric potential V(r) with the distance
is kept at 300 K in a container. The ratio of the rms r from the centre, is best represented by which graph?
æ v (helium) ö ®
speeds ç rms is | E(r)| V(r)
è vrms (argon) ÷ø
(A) 0.32 (B) 0.45
(C) 2.24 (D) 3.16
6. Two large vertical and parallel metal plates having a (A)
separation of 1 cm are connected to a DC voltage source
of potential difference X. A proton is released at rest O R r
midway between the two plates. It is found to move at
45° to the vertical JUST after release. Then X is nearly ®
(A) 1 ´ 10–5 V (B) 1 ´ 10–7 V |E(r)| V(r)
(C) 1 ´ 10–9 V (D) 1 ´ 10–10 V
7. Three very large plates of same area are kept parallel
and close to each other. They are considered as ideal
black surfaces and have very high thermal (B)
conductivity. The first and third plates are maintained
O R r
at temperatures 2T and 3T respectively. The temperature
of the middle (i.e. second) plate under steady state
condition is ®
| E(r)| V(r)
1/4 1/4
æ 65 ö T æ 97 ö T
(A) ç ÷ (B) çè ÷ø
è 2ø 4
1/4 (C)
æ 97 ö
(C) çè ÷ø T (D) (97)1/4 T
2
O R r
8. A small block is connected to one end of a massless
spring of un-stretched length 4.9 m. The other end of ®
the spring (see the figure) is fixed. The system lies on a | E(r)| V(r)
horizontal frictionless surface. The block is stretched
by 0.2 m and released from rest at t = 0. It then executes
simple harmonic motion with angular frequency
w = p/3 rad/s. Simultaneously at t = 0, a small pebble is (D)
projected with speed v form point P at an angle of 45°
as shown in the figure. Point P is at a horizontal O R r
distance of 10 m from O. If the pebble hits the block at
t = 1 s, the value of v is (take g = 10 m/s2) SECTION-II
z DIRECTIONS : This section contains 5 multiple choice
questions. Each question has four choices (A), (B), (C) and
(D) out of which ONE OR MORE are correct.
11. Consider the motion of a positive point charge in a
v
region where there are simultaneous uniform electric
r r
°
45 and magnetic fields E = E0 ˆj and B = B0 ˆj . At time t = 0,

O P
x this charge has velocity vr in the in the x-y plane, making
an angle q with the x-axis. Which of the following
10 m
option(s) is (are) correct for time t > 0?
IIT-JEE 2012 SOLVED PAPER 2012-3

(A) If q = 0°, the charge moves in a circular path in the (C) q > 45° and a frictional force acts on the block
x-z plane. towards Q.
(B) If q = 0°, the charge undergoes helical motion with (D) q < 45° and a frictional force acts on the block
constant pitch along the y-axis. towards Q.
(C) If q = 10°, the charge undergoes helical motion with 15. For the resistance network shown in the figure, choose
its pitch increasing with time, along the y-axis. the correct option(s)
(D) If q = 90°, the charge undergoes linear but
accelerated motion along the y-axis.
12. A cubical region of side a has its centre at the origin.
It encloses three fixed point charges, -q at (0, -a/4, 0),
+3q at (0, 0, 0) and -q at (0, +a/4, 0). Choose the correct
options(s)
z
a

–q
(A) The current through PQ is zero.
–q 3q (B) I1 = 3A
(C) The potential at S is less than that at Q.
x
(D) I2 = 2A
(A) The net electric flux crossing the plane x = +a/2 SECTION-III
is equal to the net electric flux crossing the plane
DIRECTIONS : This section contains 5 questions. The
x = -a/2
answer to each question is single digit integer, ranging from
(B) The net electric flux crossing the plane y = +a/2 is
0 to 9 (both inclusive).
more than the net electric flux crossing the plane
y = -a/2. 16. A circular wireloop of radius R z
(C) The net electric flux crossing the entire region is is placed in the x-y plane
45°
q centered at the origin O. A
e0 square loop of side a(a<<R) a
having two turns is placed with 3R
(D) The net electric flux crossing the plane z = +a/2 is its centre at
equal to the net electric flux crossing the plane R y
x = +a/2. z = 3R along the axis of the O
13. A person blows into open-end of a long pipe. As a result, circular wire loop, as shown in x
a high pressure pulse of air travels down the pipe.
figure. The plane of the square loop makes an angle of 45°
When this pulse reaches the other end of the pipe,
with respect to the z-axis. If the mutual inductance between
(A) a high-pressure pulse starts travelling up the pipe,
if the other end of the pipe is open. m0 a 2
(B) a low-pressure pulse starts travelling up the pipe, the loops is given by p/2 , then the value of p is
if the other end of the pipe is open. 2 R
(C) a low-pressure pulse starts travelling up the pipe, 17. An infinitely long solid cylinder of radius R has a
if the other end of the pipe is closed. uniform volume charge density r. It has a spherical
(D) a high-pressure pulse starts travelling up the pipe, cavity of radius R/2 with its centre on the axis of the
if the other end of the pipe is closed. cylinder, as shown in the figure. The magnitude of the
14. A small block of mass of 0.1 kg lies on a fixed inclined electric field at the point P, which is at a distance 2R
plane PQ which makes an angle q with the horizontal. from the axis of the cylinder, is given by the expression
A horizontal force of 1 N acts on the block through its
centre of mass as shown in the figure. 23rR
. The value of k is
The block remains stationary if (take g = 10 m/s2) 16K e 0

(A) q = 45°
(B) q > 45° and a frictional force acts on the block
towards P.
EBD_780
2012-4 IIT-JEE 2012 SOLVED PAPER
18. A proton is fired from very far away towards a nucleus
with charge Q = 120 e, where e is the electronic charge. h2 h2
(A) (B)
It makes a closest approach of 10 fm to the nucleus. The 4p2 ma02 16p 2 ma02
de Broglie wavelength (in units of fm) of the proton at
its start is: (take the proton mass, mp= (5/3) ´ 10 – 27 kg; h2 h2
h/e = 4.2 ´ 10–15 J.s / C; (C) (D)
32p 2 ma02 64p 2 ma02
1 3. The number of aldol reaction(s) that occurs in the given
= 9 × 109 m/F; 1 fm = 10–15 m
4pe 0 transformation is :
19. A lamina is made by removing a small disc of diameter OH
OH
2R from a bigger disc of uniform mass density and conc. aq. NaOH
CH3CHO + 4HCHO ¾¾¾¾¾¾¾
®
radius 2R, as shown in the figure. The moment of inertia
HO
of this lamina about axes passing though O and P is OH
IO and IP respectively. Both these axes are perpendicular (A) 1 (B) 2
to the plane of the lamina. The ratio IP / IO to the nearest (C) 3 (D) 4
integer is 4. For one mole of a van der Waal’s gas when b = 0 and
T = 300 K, the PV vs, 1/V plot is shown below. The value
of the van der Waal’s constant a (atm. liter2 mol–2) is :

24.6
23.1
PV(liter atm mol )
–1

21.6
20. A cylindrical cavity of diameter a exists inside a
cylinder of diameter 2a as shown in the figure. Both the 20.1
cylinder and the cavity are infinity long. A uniform
current density J flows along the length. If the
magnitude of the magnetic field at the point P is given
N
by m 0 aJ , then the value of N is
12
0 2.0 3.0
–1
1/V(mol liter )
(A) 1.0 (B) 4.5
(C) 1.5 (D) 3.0
5. In allene (C3H4), the type(s) of hybridisation of the
carbon atoms is (are) :
(A) sp and sp3 (B) sp and sp2
(C) only sp3 (D) sp2 and sp3
6. A compound M pXq has cubic close packing (ccp)
arrangement of X. Its unit cell structure is shown below.
CHEMISTRY The empirical formula of the compound is
SECTION-I
DIRECTIONS : This section contains 10 multiple choice
questions. Each question has four choices (A), (B), (C) and
(D) out of which ONLY ONE is correct. M

1. Which ordering of compounds is according to the X


decreasing order of the oxidation state of nitrogen?
(A) HNO3, NO, NH4Cl, N2
(B) HNO3, NO, N2, NH4 Cl
(C) HNO3, NH4Cl, NO, N2
(D) NO, HNO3, NH4Cl, N2
2. The kinetic energy of an electron in the second Bohr (A) MX (B) MX2
orbit of a hydrogen atom is [a0 is Bohr radius] : (C) M2X (D) M5 X14
IIT-JEE 2012 SOLVED PAPER 2012-5

7. The number of optically active products obtained from 13. Identify the binary mixture(s) that can be separated into
the complete ozonolysis of the given compound is : individual compounds, by differential extraction as
shown in the given scheme.
CH3 H NaOH (aq)
Compound 1 + Compound 2
CH3 CH CH C CH CH C CH CH CH3 Binary mixture
containing
Compound 1 and
H CH3 Compound 2
NaHCO3 (aq)
(A) 0 (B) 1 Compound 1 + Compound 2
(C) 2 (D) 4 (A) C6H5OH and C6H5COOH
8. As per IUPAC nomenclature, the name of the complex (B) C6H5COOH and C6H5CH2OH
[Co(H2O)4(NH3)2]Cl3 is : (C) C6H5CH2OH and C6H5OH
(A) Tetraaquadiaminecobalt (III) chloride (D) C6H5CH2OH and C6H5CH2COOH
(B) Tetraaquadiamminecobalt (III) chloride 14. Choose the correct reason(s) for the stability of the
(C) Diaminetetraaquacoblat (II) chloride lyophobic colloidal particles.
(D) Diamminetetraaquacobalt (III) chloride (A) Preferential adsorption of ions on their surface
9. The carboxyl functional group (– COOH) is present in from the solution.
(A) picric acid (B) barbituric acid (B) Preferential adsorption of solvent on their surface
(C) ascorbic acid (D) aspirin
from the solution.
10. The colour of light absorbed by an aqueous solution of
(C) Attraction between different particles having
CuSO4 is:
opposite charges on their surface.
(A) orange-red (B) blue-green
(D) Potential difference between the fixed layer and
(C) yellow (D) violet
the diffused layer of opposite charges around the
SECTION-II colloidal particles.
DIRECTIONS : This section contains 5 multiple choice 15. Which of the following halides react(s) with AgNO3(aq)
questions. Each question has four choices (A), (B), (C) and to give a precipitate that dissolves in Na2S2O3(aq)?
(D) out of which ONE OR MORE may be correct. (A) HCl (B) HF
11. For an ideal gas, consider only P–V work in going from (C) HBr (D) HI
an initial state X to the final state Z. The final state Z SECTION-III
can be reached by either of the two paths shown in the
figure. Which of the following choice(s) is (are) correct DIRECTIONS : This section contains 5 questions. The
? answer to each question is a single-digit integer, ranging
[Take DS as change in entropy and w as work done]. from 0 to 9 (both inclusive).
16. An organic compound undergoes first-order
P(atmosphere)

X Y decomposition. The time taken for its decomposition to


1/8 and 1/10 of its initial concentration are t1/8 and
é t1/ 8 ù
Z
t1/10 respectively. What is the value of ê t ú × 10 ?
ë 1/10 û
V(litre) (log102 = 0.3)
(A) DSx®z = DSx®y + DSy®z 17. When the following aldohexose exists in its
(B) wx®z = wx®y + wy®z D-configuration, the total number of stereoisomers in
(C) wx®y®z = wx®y its pyranose form is :
(D) DSx®y®z = DSx®y
12. Which of the following molecules, in pure form, is (are) CHO — CH2 — CHOH — CHOH — CHOH — CH2OH
unstable at room temperature ? 18. The substituents R1 and R2 for nine peptides are listed
in the table given below. How many of these peptides
(A) (B) are positively charged at pH = 7.0 ?

O + –
O H 3N–CH–CO–NH–CH–CO–NH–CH–CO–NH–CH–COO

(C) (D) H R1 R2 H
EBD_780
2012-6 IIT-JEE 2012 SOLVED PAPER
Peptide R1 R2
sec2 x
I H H 5. The integral ò 9
dx equals (for some
II H CH 3 ( sec x + tan x) 2
III CH 2 COOH H arbitrary constant K)
IV
V
CH 2 CONH 2
CH 2 CONH 2
(CH 2 )4 NH 2
CH 2 CONH 2
(A) -
1
11
( sec x + tan x ) 2
{ 1 1
}
- ( sec x + tan x )2 + K
11 7
VI (CH 2 )4 NH 2 (CH 2 ) 4 NH 2
VII
VIII
CH 2 COOH
CH 2 OH
CH 2 CONH 2
(CH 2 )4 NH 2
(B)
1
11
( sec x + tan x ) 2
{ 1 1
}
- ( sec x + tan x )2 + K
11 7
IX (CH 2 )4 NH 2 CH 3
19. The periodic table consists of 18 groups. An isotope of
copper, on bombardment with protons, undergoes a
(C) -
1
11
( sec x + tan x ) 2
{ 1 1
11 7 }
+ ( sec x + tan x )2 + K

nuclear reaction yielding element X as shown below.


To which group, element X belongs in the periodic
table? (D)
1
11
( sec x + tan x ) 2
{ 1 1
}
+ ( sec x + tan x )2 + K
11 7
63
29
Cu + 11H ® 610 n + 24a + 211 H + X 6. The point P is the intersection of the straight line
joining the points Q(2, 3, 5) and R(1, –1, 4) with the
20. 29.2% (w/w) HCl stock solution has a density of
plane 5x – 4y – z = 1. If S is the foot of the perpendicular
1.25 g mL–1. The molecular weight of HCl is 36.5 g mol–1.
drawn from the point T(2, 1, 4) to QR, then the length of
The volume (mL) of stock solution required to prepare the line segment PS is
a 200 mL solution of 0.4 M HCl is :
1
(A) (B) 2 (C) 2 (D) 2 2
MATHEMATICS 2
SECTION-I ì 2 p
ï x cos , x ¹ 0
DIRECTIONS : This section contains 10 multiple choice 7. Let f ( x ) = í x , x Î R then f is
questions. Each question has four choices (A), (B), (C) and ï 0, x = 0
î
(D) out of which ONLY ONE is correct.
(A) differentiable both at x = 0 and at x = 2
(B) differentiable at x = 0 but not differentiable at x = 2
æ x2 + x + 1 ö (C) not differentiable at x = 0 but differentiable at x = 2
1. If lim ç - ax - b ÷ = 4, then
ç
x ®¥ è x +1 ÷ (D) differentiable neither at x = 0 nor at x = 2
ø
8. Let z be a complex number such that the imaginary
(A) a = 1, b = 4 (B) a = 1, b = – 4 part of z is non-zero and a = z2 + z + 1 is real. Then a
(C) a = 2, b = –3 (D) a = 2, b = 3 cannot take the value
2. Let P = [aij] be a 3 ´ 3 matrix and let Q = [bij], where
1 1 3
bij = 2i + j aij for 1 £ i , j £ 3. If the determinant of P is 2, (A) –1 (B) (C) (D)
then the determinant of the matrix Q is 3 2 4
(A) 2 10 (B) 2 11 x2 y2
(C) 2 12 (D) 2 13 9. The ellipse E1 : + = 1 is inscribed in a rectangle
9 4
3. The locus of the mid-point of the chord of contact of
tangents drawn from points lying on the straight line R whose sides are parallel to the coordinate axes.
Another ellipse E2 passing through the point (0, 4)
4x – 5y = 20 to the circle x2 + y2 = 9 is
circumscribes the rectangle R. The eccentricity of the
(A) 20 (x2 + y2) – 36x + 45 y = 0
ellipse E2 is
(B) 20 (x2 + y2) + 36x – 45 y = 0
(C) 36 (x2 + y2) – 20x + 45 y = 0 2 3 1 3
(A) (B) (C) (D)
(D) 36 (x2 + y2) + 20x – 45 y = 0 2 2 2 4
4. The total number of ways in which 5 balls of different 10. The function f : [0, 3] ® [1, 29], defined by
colours can be distributed among 3 persons so that f(x) = 2x3 – 15x2 + 36x + 1, is
each person gets at least one ball is (A) one–one and onto
(A) 75 (B) 150 (B) onto but not one–one
(C) 210 (D) 243 (C) one–one but not onto
(D) neither one–one nor onto
IIT-JEE 2012 SOLVED PAPER 2012-7

SECTION-II 3
(A) P éë X1 X ùû =
c
DIRECTIONS : This section contains 5 multiple choice 16
questions. Each question has four choices (A), (B), (C) and (B) P [Exactly two engines of the ship are functioning
(D) out of which ONE OR MORE may be correct.
7
X] =
x2 y2 8
11. Tangents are drawn to the hyperbola - = 1,
9 4
5
parallel to the straight line 2x – y = 1. The points of (C) P éëX X2 ùû =
16
contact of the tangents on the hyperbola are
7
æ 9 1 ö æ 9 1 ö (D) P ëéX X1 ûù =
(A) ç , ÷ (B) ç - ,- ÷ 16
è2 2 2ø è 2 2 2ø
2
15. Let S be the area of the region enclosed by y = e - x ,
(C) (3 3,- 2 2 ) (D) ( -3 3, 2 2 )
y = 0, x = 0 and x = 1; then
12. Let q, j Î [0, 2p] be such that 2 cosq (1 – sin j) = sin2q
1 1
æ q qö (A) S ³ (B) S ³ 1 -
ç tan + cot ÷ cos j - 1, tan ( 2p - q ) > 0 and e e
è 2 2ø
1æ 1 ö 1 1 æ 1 ö
(C) S £ 1+ (D) S £ + ç1-
4 çè ÷ ÷
3
-1 < sin q < - , then j cannot satisfy eø 2 eè 2ø
2
SECTION-III
p p 4p
(A) 0 < j < (B) <j< DIRECTIONS : This section contains 5 questions. The
2 2 3
answer to each question is single digit integer, ranging from
4p 3p 3p 0 to 9 (both inclusive).
(C) <j< (D) < j < 2p
3 2 2 r r r
16. If a , b and c are unit vectors satisfying
13. If y(x) satisfies the differential equation y¢ – ytanx r r
r r r r r r r
= 2x secx and y(0) = 0, then | a - b |2 +|b - c |2 +|c - a |2 = 9 , then |2 a + 5b + 5c | is
2 2
æ pö p æ pö p 2
(A) y ç ÷ = (B) y ' ç ÷ = 17. let f : IR ® IR be defined as f (x) = x + x - 1 . The total
è4ø 8 2 è 4 ø 18
number of points at which f attains either a local
2 2
æpö p æ p ö 4p 2p maximum or a local minimum is
(C) y ç ÷ = (D) y ' ç 3 ÷ = 3 + 3 3 18. Let S be the focus of the parabola y2 = 8x and let PQ be
è3ø 9 è ø
the common chord of the circle x2 + y2 – 2x – 4y = 0
14. A ship is fitted with three engines E1 , E2 and E3 . The
and the given parabola. The area of the triangle PQS is
engines function independently of each other with
19. Let p(x) be a real polynomial of least degree which has
1 1 1 a local maximum at x = 1 and a local minimum at x = 3.
respective probabilities , and . For the ship to
2 4 4 If p(1) = 6 and p(3) = 2, then p’(0) is
be operational at least two of its engines must 20. The value of
function. Let X denote the event that the ship is
operational and let X1 , X2 and X3 denote respectively æ ö
1 1 1 1
the events that the engines E 1 , E 2 and E 3 are 6 + log 3 ç 4- 4- 4- ... ÷ is
ç3 2 3 2 3 2 3 2 ÷
functioning. Which of the following is(are) true ? 2è ø
EBD_780
2012-8 IIT-JEE 2012 SOLVED PAPER
IIT-JEE 2012 : PAPER - 2
No. of Questions : 60 Max. Marks : 198

1. The question paper consist of 3 parts(physics, Chemistry and Mathematics). Each part consist of three sections.
2. For each question in Section I and Section II, you will be awarded 3 marks if you darken the bubble corresponding to
the correct answer ONLY and zero (0) marks if no bubbles are darkened. In all other cases, minus one (–1) mark will
be awarded in these sections.
3. For each question in Section III, you will be awarded 4 marks if you darken. All the bubble(s) corresponding to the
correct answer(s) ONLY. In all other cases zero (0) marks will be awarded. No negative marks will be awarded for
incorrect.

PHYSICS
SECTION-I

DIRECTIONS : This section contains 8 multiple choice


questions. Each question has four choices (A), (B), (C) and
(D) out of which ONLY ONE is correct.
2 ˆ æp ö 2
1. Two identical discs of same radius R are rotating about (A) a I k (B) çè + 1÷ø a I kˆ
their axes in opposite directions with the same constant 2
angular speed w. The discs are in the same horizontal æp ö 2
plane. At time t = 0, the points P and Q are facing each (C) - çè + 1÷ø a I kˆ (D) (2p + 1)a2 I kˆ
2
other as shown in the figure. The relative speed between
the two points P and Q is vr. In one time period (T) of 3. An infinitely long hollow conducting cylinder with
rotation of the discs, vr as a function of time is best inner radius R/2 and outer radius R carries a uniform
represented by current density along its length. The magnitude of the
r
magnetic field, B as a function of the radial distance
r from the axis is best represented by
(A)
r
B

(A)
(B) r
B

(B)

(C) r
B
(C)

(D) r
B
(D)

2. A loop carrying current I lies in the x-y plane as shown 4. A thin uniform cylindrical shell, closed at both ends, is
partially filled with water. It is floating vertically in
in the figure. The unit vector k̂ is coming out of the water in half-submerged state. If rc is the relative
plane of the paper. The magnetic moment of the current density of the material of the shell with respect to water,
loop is then the correct statement is that the shell is
IIT-JEE 2012 SOLVED PAPER 2012-9

(A) more than half-filled if rc is less than 0.5. SECTION-II


(B) more than half-filled if rc is more than 1.0.
(C) half-filled if rc is more than 0.5. DIRECTIONS : This section contains 6 multiple choice
(D) less than half-filled if rc is less than 0.5. questions relating to three paragraphs with two questions
5. In the given circuit, a charge of +80 mC is given to the on each paragraph. Each question has four choices (A), (B),
upper plate of the 4 mF capacitor. Then in the steady state, (C) and (D), out of which ONLY ONE is correct.
the charge on the upper plate of the 3 mF capacitor is
Paragraph for Questions 9 and 10
The general motion of a rigid body can be considered to be a
combination of (i) a motion of its centre of mass about an
axis, and (ii) its motion about an instantaneous axis passing
through the centre of mass.
These axes need not be stationary. Consider, for example, a
thin uniform disc welded (rigidly fixed) horizontally at its
rim to a massless, stick, as shown in the figure. When the
disc-stick system is rotated about the origin on a horizontal
frictionless plane with angular speed w, the motion at any
(A) + 32 mC (B) + 40 mC instant can be taken as a combination of (i) a rotation of the
(C) + 48 mC (D) + 80 mC
centre of mass of the disc about the z-axis and (ii) a rotation
6. Two moles of ideal helium gas are in a rubber balloon
of the disc through an instantaneous vertical axis passing
at 30°C. The balloon is fully expandable and can be
through its centre of mass (as is seen from the changed
assumed to require no energy in its expansion. The
temperature of the gas in the balloon is slowly changed orientation of points P and Q). Both these motions have the
to 35°C. The amount of heat required in raising the same angular speed w in this case
temperature is nearly (take R = 8.31 J/mol.K)
(A) 62 J (B) 104 J
(C) 124 J (D) 208 J
7. Consider a disc rotating in the horizontal plane with a
constant angular speed w about its centre O. The disc
has a shaded region on one side of the diameter and an
unshaded region on the other side as shown in the
figure. When the disc is in the orientation as shown,
two pebbles P and Q are simultaneously projected at
an angle towards R. The velocity of projection is in the
y-z plane and is same for both pebbles with respect to Now consider two similar systems as shown in the figure:
the disc. Assume that (i) they land back on the disc Case (a) the disc with its face vertical and parallel to x-z
before the disc has completed 1/8 rotation, (ii) their plane; Case (b) the disc with its face making an angle of
range is less than half the disc radius, and (iii) w remains 45° with x-y plane and its horizontal diameter parallel to
constant throughout. Then x-axis. In both the cases, the disc is welded at point P, and
the systems are rotated with constant angular speed w about
the z-axis.

z Q
w

(A) P lands in the shaded region and Q in the


unshaded region. y
(B) P lands in the unshaded region and Q in the P
shaded region.
(C) Both P and Q land in the unshaded region. x Case (a)
(D) Both P and Q land in the shaded region.
8. A student is performing the experiment of resonance
column. The diameter of the column tube is 4 cm. The z
frequency of the tuning fork is 512 Hz. The air w Q
temperature is 38°C in which the speed of sound is
336 m/s. The zero of the meter scale coincides with the
top end of the resonance column tube. When the first 45° y
resonance occurs, the reading of the water level in the P
column is
(A) 14.0 cm (B) 15.2 cm x Case (b)
(C) 16.4 cm (D) 17.6 cm
EBD_780
2012-10 IIT-JEE 2012 SOLVED PAPER
9. Which of the following statements about the to electromagnetism, the refractive index of the medium is
instantaneous axis (passing through the centre of mass) given by the relation, n = c/ v = ± er mr , where c is the speed
is correct?
(A) It is vertical for both the cases (a) and (b) of electromagnetic waves in vacuum, v its speed in the
(B) It is vertical for case (a); and is at 45° to the x-z medium, e r and mr are the relative permittivity and
plane and lies in the plane of the disc for case (b). permeability of the medium respectively.
(C) It is horizontal for case (a); and is at 45° to the x-z In normal materials, both e r and mr , are positive, implying
plane and is normal to the plane of the disc for
case (b). positive n for the medium. When both e r and mr are
(D) It is vertical for case (a); and is 45° to the x-z plane negative, one must choose the negative root of n. Such
and is normal to the plane of the disc for case (b). negative refractive index materials can now be artificially
10. Which of the following statements regarding the prepared and are called meta-materials. They exhibit
angular speed about the instantaneous axis (passing significantly different optical behavior, without violating
through the centre of mass) is correct? any physical laws. Since n is negative, it results in a change
(A) It is 2w for both the cases in the direction of propagation of the refracted light. However,
similar to normal materials, the frequency of light remains
(B) It is w for case (a); and w/ 2 for case (b) unchanged upon refraction even in meta-materials.
(C) It is w for case (a); and 2w for case (b) 13. For light incident from air on a meta-material, the
(D) It is w for both the cases. appropriate ray diagram is
Paragraph for Questions 11 and 12
The b -decay process, discovered around 1900, is basically
the decay of a neutron (n). In the laboratory, a proton (p) and
an electron (e–) are observed as the decay products of the q1
neutron. Therefore, considering the decay of a neutron as a
two-body decay process, it was predicted theoretically that Air
the kinetic energy of the electron should be a constant. But (A) Meta-material
experimentally, it was observed that the electron kinetic
energy has continuous spectrum. Considering a three-body q2
decay process, i.e. n ® p + e - + n e , around 1930, Pauli
explained the observed electron energy spectrum. Assuming
the anti-neutrino (ne ) to be massless and possessing
negligible energy, and the neutron to be at rest, momentum
and energy conservation principles are applied. From this
calculation, the maximum kinetic energy of the electron is
0.8 × 106 eV. The kinetic energy carried by the proton is only (B)
the recoil energy.
11. If the anti-neutrino had a mass of 3 eV/c2 (where c is
the speed of light) instead of zero mass, what should
be the range of the kinetic energy, K, of the electron?
(A) 0 £ K £ 0.8 ´ 106 eV
6
(B) 3.0 eV £ K £ 0.8 ´ 10 eV q1
6
(C) 3.0 eV £ K < 0.8 ´ 10 eV
Air
(D) 0 £ K < 0.8 ´ 106 eV
(C) Meta-material
12. What is the maximum energy of the anti-neutrino?
(A) Zero q2
(B) Much less than 0.8 ´ 106 eV .
(C) Nearly 0.8 ´ 106 eV
(D) Much larger than 0.8 ´ 106 eV
Paragraph for Questions 13 and 14
Most materials have the refractive index, n > 1. So, when a q1
light ray from air enters a naturally occurring material, then
Air
sin q1 n2 (D) Meta-material
by Snell’s law, = , it is understood that the
sin q2 n1 q2
refracted ray bends towards the normal. But it never emerges
on the same side of the normal as the incident ray. According
IIT-JEE 2012 SOLVED PAPER 2012-11

14. Choose the correct statement. 17. Two spherical planets P and Q have the same uniform
(A) The speed of light in the meta-material is v = c|n| density r, masses MP and MQ and surface areas A and
c 4A respectively. A spherical planet R also has uniform
(B) The speed of light in the meta-material is v = density r and its mass is (MP + M Q). The escape
n velocities from the planets P, Q and R are VP, VQ and
(C) The speed of light in the meta-material is v = c. VR, respectively. Then
(D) The wavelength of the light in the meta-material (A) VQ > VR > VP (B) VR > VQ > VP
(lm) is given by lm = lair |n|, where lair is 1
wavelength of the light in air. (C) VR/ VP = 3 (D) VP / VQ =
2
SECTION-III 18. The figure shows a system consisting of (i) a ring of
DIRECTIONS : This section contains 6 multiple choice outer radius 3R rolling clockwise without slipping on
questions. Each question has four choices (A), (B), (C) and a horizontal surface with angular speed w and (ii) an
(D) out of which ONE or MORE are correct. inner disc of radius 2R rotating anti-clockwise with
angular speed w/2. The ring and disc are separated by
15. In the given circuit, the AC source has w = 100 rad/s. frictionless ball bearings. The point P on the inner disc
Considering the inductor and capacitor to be ideal, the is at a distance R from the origin, where OP makes an
correct choice(s) is (are) angle of 30° with the horizontal. Then with respect to
the horizontal surface,
100mF 100W z
w

0.5 H 50W w/ 2
3R
x
O 2R

20 V
~
(A) the point O has linear velocity 3 Rw î
(A) The current through the circuit, I is 0.3 A.
11 3
(B) The current through the circuit, I is 0.3 2A (B) the point P has linear velocity Rwiˆ + Rwkˆ .
4 4
(C) The voltage across 100 W resistor = 10 2V
13 3
(D) The voltage across 50 W resistor = 10 V (C) the point P has linear velocity Rwiˆ - Rwkˆ
16. Six point charges are kept at the vertices of a regular 4 4
hexagon of side L and centre O, as shown in the figure. (D) the point P has linear velocity

1 q æ 3ö ˆ 1 ˆ
Given that K = , which of the following çè 3 - ÷ø Rwi + Rwk
4pe 0 L2 4 4
19. Two solid cylinders P and Q of same mass and same
statement(s) is (are) correct?
radius start rolling down a fixed inclined plane from
L E–q the same height at the same time. Cylinder P has most
+q F of its mass concentrated near its surface, while Q has
P most of its mass concentrated near the axis. Which
statement(s) is(are) correct?
(A) Both cylinders P and Q reach the ground at the
same time.
(B) Cylinders P has larger linear acceleration than
A S T D
O cylinder Q.
+2q –2q (C) Both cylinders reach the ground with same
translational kinetic energy.
(D) Cylinder Q reaches the ground with larger angular
speed.
R
20. A current carrying infinitely long wire is kept along
B+ q – qC the diameter of a circular wire loop, without touching
it, the correct statement(s) is(are)
(A) The electric field at O is 6K along OD (A) The emf induced in the loop is zero if the current is
(B) The potential at O is zero constant.
(C) The potential at all points on the line PR is same (B) The emf induced in the loop is finite if the current
(D) The potential at all points on the line ST is same is constant.
EBD_780
2012-12 IIT-JEE 2012 SOLVED PAPER
(C) The emf induced in the loop is zero if the current 6. For a dilute solution containing 2.5 g of a non-volatile
decreases at a steady rate. non-electrolyte solute in 100 g of water, the elevation
(D) The emf induced in the loop is infinite if the current in boiling point at 1 atm pressure is 2°C. Assuming
decreases at a steady rate. concentration of solute is much lower than the
concentration of solvent, the vapour pressure
CHEMISTRY (mm of Hg) of the solution is (take Kb = 0.76 K kg mol–
1)
SECTION-I
(A) 724 (B) 740
DIRECTIONS : This section contains 8 multiple choice
(C) 736 (D) 718
questions. Each question has four choices (A), (B), (C) and
7. The compound that undergoes decarboxylation most
(D) out of which ONLY ONE is correct.
readily under mild condition is
1. The major product H of the given reaction sequence is
COOH COOH
e

CN 95% H 2 SO 4 CH2COOH O
CH3 — CH2 — CO — CH3 ¾¾¾
®G ¾¾¾¾¾¾
Heat
®H
(A) (B)
(A) CH3 CH C COOH

CH3 COOH CH2COOH


COOH O
(B) CH3 CH C CN (C) (D)

CH3
8. Using the data provided, calculate the multiple bond
OH energy (kJ mol–1) of a C º C bond in C2H2. That energy
is (take the bond energy of a C – H bond as 350 kJ mol–1)
(C) CH3 CH2 C COOH 2C(s) + H2(g) ¾¾ ® HC = CH(g); DH = 225 kJ mol–1
2C(s) ¾¾ ® 2C(g) ; DH = 1410 kJ mol–1
CH3 H2(g) ¾¾ ® 2H(g) ; DH = 330 kJ mol–1
(A) 1165 (B) 837
(D) CH3 CH C CO NH2 (C) 865 (D) 815
CH3 SECTION-II
2. NiCl2 {P(C2H5)2(C6H5)}2 exhibits temperature depend- DIRECTION : This section contains 6 multiple choice
ent magnetic behaviour (paramagnetic/diamagnetic). questions relating to three paragraphs with two questions
The coordination geometries of Ni 2+ in the on each paragraph. Each question has four choices (A), (B),
paramagnetic and diamagnetic states are respectively (C) and (D), out of which ONLY ONE is correct.
(A) tetrahedral and tetrahedral Paragraph for Questions 9 and 10
(B) square planar and square planar In the following reaction sequence, the compound J is an
(C) tetrahedral and square planar intermediate.
(D) square planar and tetrahedral
3. In the cyanide extraction process of silver from argentite 3 (CH CO) O
2 2 (i) H , Pd/C
I ¾¾¾¾¾¾® J ¾¾¾¾¾¾¾® K
CH 3 COONa (ii) SOCl 2
ore, the oxidising and reducing agents used are (iii) anhyd. AlCl 3
(A) O2 and CO respectively
(B) O2 and Zn dust respectively J (C9H8O2) gives effervescence on treatment with NaHCO3
(C) HNO3 and Zn dust respectively and a positive Baeyer’s test.
(D) HNO3 and CO respectively 9. The compound I is
4. The reaction of white phosphorus with aqueous NaOH O H OH
gives phosphine along with another phosphorus
containing compound. The reaction type; the oxidation
states of phosphorus in phosphine and the other (A) (B)
product are respectively
(A) redox reaction; – 3 and – 5 H
(B) redox reaction; + 3 and + 5 O CH3
(C) disproportionation reaction; – 3 and + 5 H
(D) disproportionation reaction; – 3 and + 3 H
5. The shape of XeO2F2 molecule is (C) (D)
(A) trigonal bipyramidal (B) square planar
(C) tetrahedral (D) see-saw
IIT-JEE 2012 SOLVED PAPER 2012-13

10. The compound K is


(P1, V1, T1)
(a) (c)
isothermal
O O
P (P2, V2, T2)
adiabatic (P3, V2, T3)
(b) (d) O

O V
Paragraph for Questions 11 and 12
(A) T1 = T2
The electrochemical cell shown below is a concentration cell.
(B) T3 > T1
M | M2+ (saturated solution of a sparingly soluble salt,
(C) wisothermal > wadiabatic
MX2) || M2+ (0.001 mol dm–3) | M.
(D) DUisothermal > DUadiabatic
The emf of the cell depends on the difference in concentrations
16. For the given aqueous reactions, which of the statement (s)
of M2+ ions at the two electrodes. The emf of the cell at 298 K is
is (are) true?
0.059 V.
11. The value of DG (kJ mol–1) for the given cell is (take 1F 2
excess KI + K3[Fe(CN)6] ¾¾¾¾¾¾4 dilute H SO
® brownish-
= 96500 C mol–1)
yellow
(A) –5.7 (B) 5.7
solution
(C) 11.4 (D) –11.4
12. The solubility product (Ksp; mol3 dm–9) of MX2 at 298 ZnSO4
K based on the information available for the given
concentration cell is (take 2.303 × R × 298/F = 0.059 V) white precipitate + brownish-yellow filtrate
(A) 1 × 10–15 (B) 4 × 10–15 14444 4244444 3
(C) 1 × 10 –12 (D) 4 × 10–12 Na2S2O3
Paragraph for Questions 13 and 14
Bleaching powder and bleach solution are produced on a colourless solution
large scale and used in several household products. The (A) The first reaction is a redox reaction.
effectiveness of bleach solution is often measured by (B) White precipitate is Zn3[Fe(CN)6]2.
iodometry. (C) Addition of filtrate to starch solution gives blue
13. Bleaching powder contains a salt of an oxoacid as one colour.
of its components. The anhydride of that oxoacid is (D) White precipitate is soluble in NaOH solution.
(A) Cl2O (B) Cl2O7 17. With reference to the scheme given below, which of the
(C) ClO2 (D) Cl2O6 given statement(s) about T, U, V and W is (are) correct ?
14. 25 mL of household solution was mixed with 30 mL of
0.50 M KI and 10 mL of 4N acetic acid. In the titration of O
the liberated iodine, 48 mL of 0.25 N Na2S2O3 was
used to reach the end point. The molarity of the O
household bleach solution is
H3C T
(A) 0.48 M (B) 0.96 M
(C) 0.24 M (D) 0.024 M LiAlH4
SECTION-III excess
Å
CrO3/ H (CH3CO)2O
The section contains 6 multiple choice questions. Each V U W
question has four choices (A), (B), (C) and (D) out of which
ONE or MORE are correct. (A) T is soluble in hot aqueous NaOH
15. The reversible expansion of an ideal gas under (B) U is optically active
adiabatic and isothermal conditions is shown in the (C) Molecular formula of W is C10H18O4
figure. Which of the following statement(s) is (are) (D) V gives effervescence on treatment with aqueous
correct ? NaHCO3.
EBD_780
2012-14 IIT-JEE 2012 SOLVED PAPER
18. Which of the given statement(s) about N, O, P and Q
with respect to M is (are) correct ? III
200 K

Amount of gas
Cl

adsorbed
HO HO 250 K
H H
H OH
HO Cl H CH3 HO
H OH H
CH 3 Cl CH 3 P
M N O
IV
CH 3 CH 3

Potential energy
H OH HO H
E
0
Distance of molecule from the surface
HO H HO H
–1
DH = 150 kJ mol
Cl Cl
P Q
(A) M and N are non-mirror image stereoisomers (A) I is physisorption and II is chemisorption
(B) M and O are identical (B) I is physisorption and III is chemisorption
(C) M and P are enantiomers (C) IV is chemisorption and II is chemisorption
(D) M and Q are identical (D) IV is chemisorption and III is chemisorption
19. With respect to graphite and diamond, which of the
statement(s) given below is (are) correct ? MATHEMATICS
(A) Graphite is harder than diamond. SECTION-I
(B) Graphite has higher electrical conductivity than
diamond. DIRECTIONS : This section contains 8 multiple choice
(C) Graphite has higher thermal conductivity than questions. Each question has four choices (A), (B), (C) and
diamond. (D) out of which ONLY ONE is correct.
(D) Graphite has higher C–C bond order than 1. Let a1, a2, a3, ..... be in harmonic progression with a1 = 5
diamond.
and a20 = 25. The least positive integer n for which
20. The given graphs/data I, II, III and IV represent general
an < 0 is
trends observed for different physisorption and
(A) 22 (B) 23 (C) 24 (D) 25
chemisorption processes under mild conditions of
temperature and presure. Which of the following 2. The equation of a plane passing through the line of
choice(s) about I, II, III and IV is (are) correct ? intersection of the planes x + 2y + 3z = 2 and x – y + z
2
= 3 and at a distance from the point (3, 1 ,–1) is
P constant 3
I
Amount of gas
adsorbed

(A) 5x – 11y + z = 17 (B) 2x + y = 3 2 - 1

(C) x + y + z = 3 (D) x - 2 y = 1 - 2
7
3. Let PQR be a triangle of area D with a = 2, b = and
T 2
5
II P constant c= ; where a, b, and c are the lengths of the sides of
2
Amount of gas

the triangle opposite to the angles at P,Q and R


adsorbed

2 sin P - sin 2 P
respectively. Then equals.
2 sin P + sin 2 P
2 2
3 45 æ 3 ö æ 45 ö
(A) (B) (C) ç ÷ (D) ç ÷
T 4D 4D è 4D ø è 4D ø
IIT-JEE 2012 SOLVED PAPER 2012-15

r r ® ® 9. A possible equation of L is
4. If a and b are vectors such that a + b = 29 and
(A) x - 3 y = 1 (B) x + 3 y = 1
r r
( ) ( )
a ´ 2iˆ + 3 jˆ + 4kˆ = 2iˆ + 3 jˆ + 4kˆ ´ b , then a possible (C) x - 3 y = -1 (D) x + 3y = 5
r r 10. A common tangent of the two circles is
value of ( a + b ) . ( -7 iˆ + 2 ˆj + 3kˆ ) is (A) x = 4 (B) y = 2
(A) 0 (B) 3 (C) 4 (D) 8 (C) x + 3 y = 4 (D) x + 2 2 y = 6
5. If P is a 3 ´ 3 matrix such that PT = 2P + I, where PT is
Paragraph for Questions 11 and 12
the transpose of P and I is the 3 ´ 3 identity matrix, then
Let f (x) = (1 – x)2 sin 2 x+ x2 for all x Î IR and let
é x ù é0 ù x
æ 2 ( t - 1) ö
there exists a column matrix X = êê y úú ¹ êê0 úú such that
êë z úû êë0 úû
g( x ) = ò çè t +1
- ln t ÷ f (t) dt for all x Î (1, ¥ ).
ø
1
11. Consider the statements:
é0 ù P : There exists some x Î R such that f (x) + 2x
(A) PX = ê0 ú (B) PX = X = 2(1 + x2)
ê ú
êë0 úû Q : There exists some x Î R such that 2 f (x) + 1 = 2x
(1 + x)
(C) PX = 2X (D) PX = –X Then
6. Let a(a) and b(a) be the roots of the equation (A) both P and Q are true
( 3 1 + a - 1) x 2 + ( ) ( 6 1 + a - 1) = 0
1+ a -1 x + where (B) P is true and Q is false
(C) P is false and Q is true
a > –1. Then lim+ a ( a ) and lim+ b ( a ) are (D) both P and Q are false
a ®0 x ®0 12. Which of the following is true?
5 1 (A) g is increasing on (1, ¥)
(A) - and 1 (B) - and – 1
2 2 (B) g is decreasing on (1, ¥)
(C) g is increasing on (1, 2) and decreasing on (2, ¥)
7 9
(C) - and 2 (D) - and 3 (D) g is decreasing on (1, 2) and increasing on (2, ¥)
2 2 Paragraph for Questions 13 and 14
7. Four fair dice D1, D2, D3 and D4 ; each having six Let an denote the number of all n-digit positive integers
faces numbered 1, 2, 3, 4, 5 and 6 are rolled formed by the digits 0, 1 or both such that no consecutive
simultaneously. The probability that D 4 shows a digits in them are 0. Let bn = the number of such n-digit
number appearing on one of D1, D2 and D3 is integers ending with digit 1 and cn = the number of such n–
91 108 125 127 digit integers ending with digit 0.
(A) (B) (C) (D) 13. The value of b6 is
216 216 216 216
(A) 7 (B) 8
p/ 2
æ 2 p+ x ö (C) 9 (D) 11
8. The value of the integral
ò ç x + ln
è
÷ cos xdx is
p- x ø 14. Which of the following is correct?
-p/ 2 (A) a17 = a16 + a15 (B) c17 ¹ c16 + c15
(C) b17 ¹ b16 + c16 (D) a17 = c17 + b16
p2
(A) 0 (B) -4 SECTION-III
2
DIRECTIONS : This section contains 6 multiple choice
p2 p2 questions. Each question has four choices (A), (B), (C) and
(C) +4 (D)
2 2 (D) out of which ONE or MORE are correct.
SECTION-II
15. For every integer n, let an and bn be real numbers. Let
DIRECTIONS : This section contains 6 multiple choice function f : IR ® IR be given by
questions relating to three paragraphs with two questions
on each paragraph. Each question has four choices (A), (B), ì an + sin px , for x Î [ 2n, 2n + 1]
f (x) = í
(C) and (D), out of which ONLY ONE is correct. îbn + cos px , for x Î ( 2n - 1, 2n )
Paragraph for Questions 9 and 10 for all integers n. If f is continuous, then which of the
A tangent PT is drawn to the circle x2 + y2 = 4 at the point following hold(s) for all n ?
P ( )
3 ,1 . A straight line L, perpendicular to PT is a tangent (A) an–1 – bn–1 = 0 (B) an – bn = 1
to the circle (x – 3)2 + y2 = 1. (C) an – bn+1 = 1 (D) an–1 – bn = –1
EBD_780
2012-16 IIT-JEE 2012 SOLVED PAPER
x -1 y +1 z 1
16. If the straight lines = = and 19. Let X and Y be two events such that P ( X|Y ) = ,
2 k 2 2
x +1 y +1 z 1 1
= = are coplanar, then the plane (s) P (Y /X ) = and P ( X Ç Y ) = . Which of the following
5 2 k 3 6
containing these two lines is (are) is (are) correct ?
(A) y + 2z = –1 (B) y + z = –1 2
(C) y – z = –1 (D) y – 2z = –1 (A) P ( X È Y ) =
3
é1 4 4ù (B) X and Y are independent
ê2 1 7 ú (C) X and Y are not independent
17. If the adjoint of a 3 ´ 3 matrix P is ê ú , then the
êë 1 1 3 úû
(
(D) P X c Ç Y = ) 1
3
possible value(s) of the determinant of P is (are)
(A) –2 (B) –1 x
t2
(C) 1 (D) 2 20. If f ( x ) = ò0 e ( t - 2 )( t - 3 ) dt for all x Î (0,¥), then
2 (A) f has a local maximum at x = 2
18. Let f : (–1, 1) ® IR be such that f ( cos 4q ) = for
2 - sec 2 q (B) f is decreasing on (2, 3)
(C) there exists some c Î (0, ¥), such that f”(c) = 0
æ pö æp pö
q Î ç 0, ÷ È ç , ÷ . Then the value (s) of f æç 1 ö÷ is (are) (D) f has a local minimum at x = 3
è 4ø è4 2ø è3ø
3 3
(A) 1 - (B) 1 +
2 2

2 2
(C) 1 - (D) 1 +
3 3
IIT-JEE 2012 SOLVED PAPER 2012-17

SOLUTIONS
Paper - 1
PHYSICS é 1 1ù 1
= (1.5 - 1) ê - ú =
SECTION - I ë 14 ¥ û 28
The focal length (f2) of the lens with n = 1.2 is
1. (A) The maximum possible error in Y due to l and d
given by
are
DY Dl 2 Dd 1 é 1 1 ù
= + = (n2 - 1) ê - ú
Y l d f2 ë R1 R2 û
The least count
é1 1 ù 1
Pitch = (1.2 - 1) ê - =
= ë ¥ -14 úû 70
Number of divisions on circular scale The focal length F of the combination is
0.5 1 1 1 1
= mm = 0.005 mm = + =
100 F f1 f 2 20
Dl 0.005 mm 1 Applying lens formula for the combination of lens
Error contribution of l = = =
l 0.25 mm 50 1 1 1 1 1 1
- = Þ - =
Error contribution of d V U F V -40 20
2Dd 2 × 0.005 mm 1 Þ V = 40 cm
= = = r
d 0.5 mm 50 r dL
4. (B) We know that |t|=
2. (C) The angular momentum of the mass m about O is dt
mr2w and is directed toward +z direction for all where L = Iw
locations of m.
d dI
LP \ t = (I w) = w ...(i)
dt dt
From the situation it is clear that the moment of
P inertia for (rod + insect) system is increasing.
z
w

l
v

LO O
v x
Let at any instant of time 't', the insect is at a
r m distance x from O. At this instant, the moment of
O
inertia of the system is
1
The angular momentum of mass m about P is mvl I= ML2 + mx2 ...(ii)
3
and is directed for the given location of m as
shown in the figure. where M = mass of the rod
r L = length of the rod
The direction of LP remains changing for different m = mass of the insect
locations of m. From (i) & (ii)
3. (B) The focal length (f1) of the lens with n = 1.5 is given d é1 ù d
by   t =w ê ML2 + mx 2 ú = w m ( x 2 )
dt ë 3 û dt
1 é 1 1 ù
= (n1 - 1) ê - ú dx
f1 R R = 2wmx = 2wmxv
ë 1 2û dt
EBD_780
2012-18 IIT-JEE 2012 SOLVED PAPER
= 2wmv2t [Q x = vt] where q = charge on sphere
\ tµt (till t = T) R = Radius of sphere
ur (ii) Outside the shell at any point at any distance
When the insect stops moving, L does not change
and therefore t becomes constant. 1 1
r from the centre E µ and V µ
2 r
vrms (helium) Margon 40 r
5. (D) = = = 10 » 3.16 11. (C, D) When q = 0º, the charged particle is projected
vrms (argon) Mhelium 4
ur
6. (C) The two forces acting on the proton just after the along x-axis, due to B the charged particle
release are shown in the figure. In this situation will tend to move in a circular path in y-z plane
but due to force of electric field, the particle
will move in a helical path with increasing
qE pitch. Therefore options (A) and (B) are
+
q incorrect.

y
v
mg
E B

qE = mg [\ q = 45º]
æV ö
\ q ç ÷ = mg x
èdø

mgd 1.67 ´ 10 -27 ´ 10 ´ 10 -2


\ V= = = 10 -9 V
q 1.6 ´ 10 -19
When q = 10º, we can resolve velocity into two
7. (C) Under steady conditions, the heat gained per
rectangular components. One along x-axis
second by a plate is equal to the heat released per
second by the plate. (v cos 10º) and one along y-axis (v sin 10º). Due to
v cos 10º, the particle will move in circular path
Heat gained Heat gained and due to v sin 10º plus the force due to electric
[by (2) from (1)] +
Second Second field, the particle will undergo helical motion with
Heat gained its pitch increasing.
[(by ((2) from (3)] = (by 2) If q = 90º, the charge is moving along the magnetic
Second
field. Therefore the force due to magnetic field is
\ sA(2T)4 + sA(3T)4 2T T' 3T
zero. But the force due to electric field will
= s(2A) (T¢)4
accelerate the particle along y-axis.
1/ 4 12. (A, C, D)
é 97 ù
\ T¢ = ê ú T
ë 2 û a a
The electric flux passing through x = + , x = - ,
2 2
2V sin q
8. (A) T = A A a
g z= + is same due to symmetry.
2
2V sin 45º The net electric flux through the cubical region is
\ 1=
g 1 2 3
-q + 3q - q q .
=
\ v = 50 ms -1 e0 e0
lD 13. (B, D) When sound pulse is reflected through a rigid
9. (D) We know that b =
d boundary (closed end of a pipe), no phase
Now, lR > lG > lB change occurs between the incident and
\ bR > bG > bB reflected pulse i.e., a high pressure pulse is
10. (D) For a thin uniformly positive charged spherical reflected as a high pressure pulse.
shell When a sound pulse is reflected from open
(i) Inside the shell at any point end of a pipe, a phase change of a radian
1 q occurs between the incident and the reflected
E = O and V = = constt. pulse. A high pressure pulse is reflected as a
4 p Î0 R
low pressure pulse.
IIT-JEE 2012 SOLVED PAPER 2012-19

14. (A, C) The forces are resolved as shown in the figure. 17. (6) We suppose that the cavity is filled up by a positive
When q = 45º, sinq = cosq as w ell as negative volume charge of r. So
the electric field now produced at P is the
Q
superposition of two electric fields.
(a) The electric field created due to the infinitely
cos q long solid cylinder is
R
q rR
1N E1 = directed towards the +Y direction
4e 0
sin q q (b) The electric field created due to the spherical
sin q negative charge density
cos q
1N rR
E2 =
96e 0 directed towards the –Y direction.
FIXED q \ The net electric field is
P
The block will remain stationary and the 1 é 23rR ù
frictional force is zero. E = E1 - E2 = ê ú
When q > 45º, sinq > cosq 6 ë 16e 0 û
Therefore a frictional force acts towards Q. 18. (7) Loss in K.E. of proton = Gain in potential energy
15. (A, B, C, D) of the proton – nucleus system
The given circuit is an extension of wheatstone
1 1 q1 q2
bridge, therefore points P and Q are at the same mv2 =
potential and point S and T are also at the same 2 4 p e0 r
potential. Therefore no current passes through PQ
and ST and the circuit reduces to as shown p2 1 q1 q2
\ =
6W 2m 4p Î0 r
I2
1 æ h2 ö 1 q1 q2
\ ç ÷=
I1 – I2 2m è l 2 ø 4p Î0 r
12 W
I1 4p Î0 r . h2
\ l= = 7 fm
q1 q2 (2 m)
12V 19. (3) Let s be the surface mass density. Then
12 é 6 ´ 12 ù 1
I1 =
4 êëQ Req =
6 + 12 úû IO = s[p(2 R)2 ] ´ (2R)2 -
2
= 3A
é1 2 2 2 2ù
\ I2 = 3 ê
é 12 ù êë 2 (s p R ) + s (p R ) ´ R úû
= 2A
ë 6 + 12 úû
13
As P and Q are equipotential and potential at S is = psR 4
less than the potential at P (potential drops across 2
a resistance as current passes through it), therefore IP = 8 p s R4 + sp(2R)2 × (2R)2
VS < VQ.
é1
( (2R) + R ) ùúûú
2
16. (7) The magnetic field due to current carrying wire at 2 2 2 2 2
ê s (pR )R + s(pR )
the location of square loop is ëê 2
µ0 2piR2 µ i = 24 p s R4 – 5.5 s p R4 = 18.5 p s R4
B= = 0
4p (R + 3R2 )3/2 16R
2
IP 18.5 psR4 37
The mutual induction \ = = »3
IO 13 13
psR 4
N f 2 é µ0 i 2 ù 2
M= = ê ´ a cos 45º ú
i i ë 16R û current I I'
20. (5) Current density J = = =
\ M=
µ0 a2 area p(2a) 2
( )
pa 2
7
22 R I
Þ I' =
4
EBD_780
2012-20 IIT-JEE 2012 SOLVED PAPER
Let us consider the cavity to have current I' flowing CH2OH
in both the directions. Q
The magnetic field at P due to the current flowing OH /HCHO
¾¾¾¾¾¾® HOCH2 C CHO
3rd aldol
through the cylinder condensation
µ0 2 I CH2OH
B1 =
4p a
The magnetic field at P due to the current (I’) CH2OH

flowing in opposite direction is OH / HCHO
HOCH2 C CH2OH
µ 0 3I ' µ 2( I /4) µ0 I Cannizzaro
B2 = = 0 = reaction
4 p 3 a /2 4 p 3 a /2 4 p 3a CH2OH
\ The net magnetic field is
4. (C)
µ0 I é 1ù µ I 5
B = B1 – B2 = 2- ú = 0 ´ 24.6
4p a êë 3 û 4p a 3

µ 0 J p a2 5 5 Ja
\ B= ´ = µ0
4p a 3 12 PV 21.6
20.1
CHEMISTRY
SECTION - I
3.0
+5 +2 0 -3 2.0 1/V
1. (B) HNO 3 , NO , N 2 , NH 4 Cl 0

2. (C) As per Bohr’s postulate, æ a ö


çè P + 2 ÷ø (V) = RT
nh V
mvr = PV + a/V = RT
2p
PV = RT – a(V)
nh y = RT – a(x)
So, v =
2pmr 21.6 - 20.1 1.5
So, slope = a = = = 1.5
1 3-2 1
KE = mv2
2 sp 2 sp sp 2
5. (B) Allene (C3H4) is H 2 C = C = CH 2
1 æ nh ö 2
So, KE = mç ÷ 1
2 è 2 pmr ø 6. (B) No. of M atoms = ´ 4+1=1+1=2
4
ao ´ n2 1 1
Since, r = No. of X atoms = ×6+ ×8=3+1=4
z 2 8
So, for 2nd Bohr orbit So, formula = M2X4 = MX2
7. (A)
ao ´ 22
r= = 4 ao CH3 H
1

1 æ 2 2 h2 ö H3C CH CH C CH CH C CH CH CH3
KE = mç 2 2
2 è 4p m ´ (4 ao ) ÷ø
2
H CH3
2
h
KE = O3/ Zn/ H2O
32p 2 mao2
a OHQ CH3
3. (C) CH 3 CHO + HCHO ¾¾¾¾¾¾
®
1st aldol
condensation
2 H3C CH O + 2O CH C CH O
a CH2 OH
CH 2 CHO OH– / HCHO a
CH CHO H
2nd aldol
CH 2 OH condensation (achiral) (achiral)
CH2 OH
IIT-JEE 2012 SOLVED PAPER 2012-21

8. (D) [Co(H2O)4(NH3)2]Cl3 SECTION - III


= Diamminetetraaquacobalt (III) chloride.
2.303 log 8 2.303 ´ 3 log 2
16. (9) t1/8 = =
OH O k k
9. (D) O2N NO 2 2.303 2.303
HN NH t1/10 = log 10 =
k k
O O
NO2 æ 2.303 ´ 3 log 2 ö
é t1/8 ù çè ÷ø
k
(Picric acid) (Barbituric acid) êt ú ´ 10 = × 10 = 9
ë 1/10 û æ 2.303 ö
è k ø
HO 17. (8)
O O COOH O
HO 1 1
CHO CHOH
O C CH3 6
CH2OH
2 2
HO OH CH2 CH2
5 O
4 1
(Ascorbic acid) (Aspirin) 3 3 H OH
CHOH CHOH O or HO
4
CHOH
4
CHOH 3 2
10. (A) CuSO4 will be absorbing orange-red colour and
OH
hence will be of blue colour. 5 5
H C OH H C
SECTION - II
CH2OH CH2OH
6 6
11. (A, C) DS x® z = DS x® y + DS y® z [Entropy is a state
D- Aldohexose D- Aldohexopyranose
function, hence additive]
Wx® y®z = Wx® y [Work done in y ® z, is
Thus, total number of stereoisomers in pyranose
zero because it is an isochoric process]. form of D-configuration = 23 = 8
12. (B, C) B and C, being antiaromatic, are unstable at 18. (4) Peptides with isoelectric point (pI) more than 7,
room temperature. would exist as cation in neutral solution (pH = 7)
13. (B, D) (A) Both are soluble in NaOH, hence which means the given polypeptide is of basic
inseparable. nature, so it must contain two or more amino
(B) Only benzoic acid (C6H5COOH) is soluble groups. Hence IV, VI, VIII and IX are correct options.
in NaOH and NaHCO3, while benzyl
A
alcohol (C6H5CH2OH) is not. Hence, 19. (8) 63
29
Cu + 11H ® 6 10 n + 42 He + 211 H + Z X
separable.
Balancing the atomic mass and atomic number
(C) Although NaOH can enable separation
63 + 1 = (6 × 1) + 4 + 2 + A
between benzyl alcohol (C6H5CH2OH)
A = 52
and phenol (C6H5OH) as only the later is
soluble in NaOH. However, in NaHCO3, 29 + 1 = (6 × 0) + 2 + 2 + Z
both are insoluble. Hence, inseparable. Z = 26
(D) a-Phenylacetic acid (C6H5CH2COOH) is A
X = 52 X or 52
Fe
Thus Z 26 26
soluble in NaOH and NaHCO3. While
benzyl alcohol (C 6 H5 CH2OH) is not. Hence, X belongs to group 8 in the periodic table.
Hence, separable. 20. (8) Molarity of stock solution of HCl
14. (A, D)
29.2 ´ 1000 ´ 1.25
15. (A, C, D) =
100 ´ 36.5
HX + AgNO 3 ¾¾
® AgX ¯ + HNO 3 (X = Cl, Br, I) Let the volume of stock solution required = V mL
29.2 ´ 1000 ´ 1.25
Thus, V ´ = 200 × 0.4 = 8 mL
AgX + 2Na 2 S 2 O 3 ¾¾
® Na 3 [Ag(S 2 O 3 )2 ] + NaX 100 ´ 36.5
EBD_780
2012-22 IIT-JEE 2012 SOLVED PAPER
MATHEMATICS 4a - 20 ö
drawn from point P æç a , ÷ is
SECTION - I è 5 ø
1. (B) Given
æ 4a - 20 ö
x. a + y. ç ÷ =9 ....(1)
æ x2 + x + 1 ö è 5 ø
lim ç - ax - b ÷ = 4
x ®¥ èç x +1 ÷
ø Also the equation of chord AB whose mid point is
(h, k) is
x 2 + x + 1 - ax 2 - ax - bx - b hx + ky = h 2 + k 2 ....(2)
Þ lim =4
x ®¥ x +1
Q Equations (1) and (2) represent the same line,
therefore
( 1 - a ) x + (1 - a - b ) x + (1 - b )
2
Þ lim =4
x ®¥ x +1 h k h2 + k 2
= =
For this limit to be finite 1 - a = 0 Þ a = 1 a 4a - 20 9
then given limit reduces to 5

-bx + (1 - b ) -b +
(1 - b ) Þ 5k a = 4ha - 20h and 9h = a h2 + k 2 ( )
lim = 4 Þ lim x =4
x ®¥ x +1 x ®¥ 1 20 h 9h
1+ Þ a= and a =
x 4h - 5k h + k2
2

Þ – b = 4 or b = – 4 20h 9h
Hence a = 1, b = –4. Þ = Þ 20( h2 + k 2 ) = 9 ( 4h - 5k )
4h - 5k h 2 + k 2
2. (D) We have
\ Locus of (h, k) is

( )
2 3 4
2 a11 2 a12 2 a13 20 x 2 + y 2 - 36 x + 45 y = 0
3 4 5
Q = 2 a21 2 a22 2 a23 4. (B) Q Each person gets at least one ball.
2 4 a31 2 5 a32 26 a33 \ 3 Persons can have 5 balls in the following
systems

a11 a12 a13 Person I II III


2a21 2 a22 2 a23 No. of balls 1 1 3
= 2 2.2 3.2 4
22 a31 22 a32 2 2 a33 or

Person I II III
a11 a12 a13
No. of balls 1 2 2
= 29.2.2 2 a21 a22 a23
a31 a32 a33 The number of ways to distribute the balls in first
system
= 212 ´ P = 212 ´ 2 = 213 = 5 C1 ´ 4 C1 ´ 3 C3

æ 4a - 20 ö Also 3, persons having 1, 1 and 3 balls can be


3. (A) Any point P on line 4x – 5y = 20 is ç a , ÷. 3!
è 5 ø
arranged in ways.
Equation of chord of contact AB to the circle 2!
\ No. of ways to distribute 1, 1, 3 balls to the three
x2 + y2 = 9 persons
3!
A = 5 C1 ´ 4 C1 ´ 3C3 ´ = 60
æ 4a - 20 ö 2!
P ç a, ÷
è 5 ø Similarly the total no. of ways to distribute 1, 2, 2
M(h,k) balls to the three persons
C(op) 3!
B 4x – 5y = 20 = 5 C1 ´ 4C2 ´ 2C2 ´ = 90
2!
\ The required number of ways = 60 + 90 = 150
IIT-JEE 2012 SOLVED PAPER 2012-23

Alternative method Now let point S on QR be


Key Concept : The number of ways in which n different
( -m + 2, - 4m + 3, - m + 5 )
things can be distributed into r different groups (when
blank groups are not allowed) is Q S is the foot of perpendicular drawn from
T (2, 1, 4) to QR, where dr’s of ST are µ, 4µ – 2, µ –1
n n r -1 r
r n - r C1 ( r - 1) + r C2 ( r - 2 ) - ............ + ( -1) Cr - 1 and dr’s of QR are –1, – 4, –1
Required number of ways \ – m - 16m + 8 - m + 1 = 0
5 5
= 35 - 3 C1 ( 3 - 1) + 3 C2 ( 3 - 2 ) = 243 – 96 + 3 = 150 1
Þ 18m = 9 Þ m =
2
sec 2 x
5. (C) I = ò ( sec x + tan x )9/2 dx \
æ3 9ö
S = ç , 1, ÷
è 2 2 ø
1 \
Let sec x + tan x = t Þ sec x - tan x = Distance between P and S
t
2 2 2
= æ 4 - 3 ö + æ 1 - 1 ö + æ 13 - 9 ö
1 æ 1ö ç ÷ ç ÷ ç ÷
Þ sec x = ç t + ÷ è3 2ø è3 ø è 3 2ø
2 è tø

Also sec x ( sec x + tan x ) dx = dt 1 4 1 1


= + + =
36 9 36 2
dt
Þ sec x dx = T(2,1,4)
t
Q(2,3,5)
æ t + 1 ö dt
ç ÷
ò (t )
1 è tø = 1
ò
-9/2
\ I= + t -13/2 dt
2 t 9/2 .t 2 S
q
é ù æ 4 1 13 ö
Pç , , ÷
1 ê t -9/ 2 +1 t-13/2 + 1 ú è3 3 3 ø
= ê + ú+K
2 ê - 9 +1 13
- +1 ú
ë 2 2 û

-1 -7/2 1 R(1,–1,4)
= t - t-11/2 + K
7 11 Alternative

1 1 1 æ 1 t2 ö æ 4 1 13 ö
=- - +K=- çç + ÷÷ + K As in previous method P is ç , , ÷
7/2 11/2 11/2 è3 3 3 ø
7t 11t t è 11 7 ø
\ dr's of PT are 2, 2, –1
=
-1
( sec x + tan x )
1 1
11/2 11 7 (
+ { 2
sec x + tan x ) + K } \
Also dr's of QR are 1, 4, 1
If q be the angle between PT and QR, then

6. (A) Equation of st. line joining Q (2, 3, 5) and 2 ´ 1 + 2 ´ 4 - 1´ 1 1


cos q = =
R (1, –1, 4) is 4 + 4 + 1 1 + 16 + 1 2
x-2 y-3 z-5 Þ q = 45°
= = =l
-1 -4 1 1
Þ PS = TS = PT cos 45° = (Q PT = 1 )
Let P ( -l + 2, - 4l + 3, - l + 5 ) 2
As P lies on 5x – 4y – z = 1 f (0 + h ) - f (0 )
+
\ -5l + 10 + 16l - 12 + l - 5 = 1 7. (B) We have f '(0 ) = lim
h ®0 h
2
Þ 12l = 8 Þ l = p
3 h 2 cos
h p
= lim = lim h cos
æ 4 1 13 ö h® 0 h h ®0 h
\ P=ç , , ÷
è3 3 3 ø = 0 × some finite value = 0
EBD_780
2012-24 IIT-JEE 2012 SOLVED PAPER
Also, 8. (D) Q Im (z) ¹ 0 Þ z is non real
p and equation z 2 + z + ( 1 - a ) = 0
h 2 cos
f (0 - h) - f (0) -h
f '(0 - ) = lim = lim will have non real roots, if D < 0
h ®0 -h h ®0 -h
3
Þ 1 – 4(1 – a) < 0 Þ 4a < 3 Þ a <
p 4
= lim - h cos = 0 ´ some finite value
h ®0 h 3
= 0 \ a can not take the value
4
Q f '(0+ ) = f '(0 - ) Þ f is differentiable at x = 0 9. (C) As rectangle ABCD circumscribed the ellipse

f ( 2 + h) - f (2) x2 y2
+ + =1
Now f '(2 ) = lim 9 4
h ®0 h
\ A = (3, 2)
p p Y
( 2 + h )2 cos - 4 cos
2+h 2
= lim (0, 4)
h ®0 h
B A(3, 2)
2 æ p ö
(2 + h) ç cos ÷
è 2+hø
= lim
h ®0 h X¢ X
O
(2 + h) 2 æp p ö
= lim sin ç - ÷ C D
h ®0 h è 2 2 + hø

(2 + h)2 æ ph ö
= lim sin çç ÷÷ Y¢
h ®0 h è 2 (2 + h) ø
Let the ellipse circumscribing the rectangle ABCD
æ ph ö x2 y2
sin ç ÷ is + =1
è 2 ( 2 + h ) ø ´ ph
2
(2 + h) a2 b2
= lim ´ =p
h ®0 h æ ph ö 2 (2 + h) Given that it passes through (a, 4)
ç ÷ \ b2 = 16
è 2 ( 2 + h) ø
Also it passes through A (3, 2)
f (2 - h) - f (2) 9 4
-
Also f '(2 ) = lim \ 2
+ = 1 Þ a2 = 12
h ®0 -h a 16

p ö 12 1 1
( 2 - h )2 cos çæ ÷ -0 \ e = 1- = =
è2-hø 16 4 2
= lim
h® 0 -h 10. (B) We have f(x) = 2x3 – 15x2 + 36x + 1
Þ f ¢(x) = 6x2 – 30x + 36
p ö
- ( 2 - h ) cos æç = 6(x2 – 5x + 6)
2
÷
è 2-hø = 6 (x – 2) (x – 3)
= lim
h ®0 -h Q f ¢(x) > 0 " x Î[0, 2] and f ¢(x) < 0 " x Î [2, 3]
p p ö \ f(x) is increasing on [0, 2] and decreasing
( 2 - h )2 sin æç - ÷ on [2, 3]
2 2-h è ø
= lim \ f(x) is many one on [0, 3]
h® 0 h
Also f(0) = 1, f(2) = 29, f(3) = 28
\ Global min = 1 and Global max = 29
æ -ph ö
sin ç ÷ i.e., Range of f = [1, 29] = codomain
è 2 ( 2 - h ) ø ´ æ -ph ö
2
( 2 - h) \ f is onto.
= lim ´ çç 2 ( 2 - h ) ÷÷ = –p
h®0 h æ -ph ö 11. (A, B)
è ø
ç ÷ Key Concept : If slope of tangent is m, then equations of
è 2 ( 2 - h ) ø
x2 y2
+ -
As f '(2 ) ¹ f '(2 ) Þ f is not differentiable at tangents to hyperbola 2 - 2 = 1 are
a b
x = 2.
IIT-JEE 2012 SOLVED PAPER 2012-25

Þ 2 cos q(1 – sin f) = 2 sin q cos f – 1


y = mx ± a2 m 2 - b 2 with the points of contact Þ 2 cos q + 1 = 2 sin (q + f)
æ ± a2 m ± b2 ö æ 3p 5p ö
Q q Îç , \ 2 cos q + 1 Î (1, 2)
ç
ç 2 2 2
, ÷
÷ è 2 3 ÷ø
è a m -b a2 m2 - b 2 ø
Þ 1 < 2 sin (q + f ) < 2
2 2
x y 1 p 5p
Q Tangent to hyperbola - = 1 is parallel to 2x – y Þ < sin (q + f) < 1 Þ q + f Î æç , ö÷
9 4 2 è 6 6ø
= 1, therefore slope of tangent = 2
æ p 5p ö æ 13 p 17 p ö
or q + f Î ç 2 p + , 2p + ÷ or ç , ÷
æ ± 9´2 ±4 ö è 6 6 ø è 6 6 ø
\ Points of contact are çè , ÷
9´4-4 9´4-4ø
3p 5p ö
But q Î æç , (as q + f Î [0, 4p])
æ 9 1 ö æ -9 -1 ö è 2 3 ÷ø
i.e. ç , and ç ,
è 2 2 2 ÷ø è 2 2 2 ÷ø
13p 17 p 13p 17 p
Alternative : Let P(3 sec q, 2 tan q) be any point on Þ < q+f < Þ -q < f < -q
6 6 6 6
x2 y2 13p 17 p
hyperbola - = 1 at which equation of tangent is Þ - qmax < f < - qmin
9 4 6 6
x sec q y tan q 13p 5p 17 p 3p p 4p
- =1 Þ - <f< - Þ <f<
3 2 6 3 6 2 2 3
As tangent is parallel to 2x – y = 1 13. (A, D)
\ slope of tangent = 2 The given differential equation is
2 sec q 1 dy
Þ = 2 Þ sin q =
3 tan q 3 – y tanx = 2x sec x
dx
3 1
I.F. = e ò
If q Î I quad, sec q = and tan q = - tan xdx
= cos x
2 2 2 2
\ y.cos x = ò 2 x dx = x 2 + c
æ 9 1 ö
\ point of contact is ç ,
è 2 2 2 ÷ø y (0) = 0 Þ c = 0
\ y = x2 sec x
-3 -1
If q Î II quad, sec q = and tan q = p p2 p2
2 2 2 2 Now at x = , y= ´ 2=
4 16 8 2
æ -9 -1 ö
\ point of contact is ç ,
è 2 2 2 ÷ø p p2 2p 2
At x = , y= ´2=
12. (A, C, D) 3 9 9
As tan(2p – q) > 0 Þ – tan q > 0 Þ tan q < 0
\ q Î II or IV quadrant ...(1) p 2p p2 p2 p
At x = , y’= ´ 2+ ´1 = +
4 4 8 2 8 2 2
- 3
And -1 < sin q <
2 p 2p 2p2 2 p2 4p
At x = , y’= ´2+ ´ 3 = +
Þ q Î III or IV quadrant ...(2) 3 3 9 3 3 3
Also q Î [0, 2p] ...(3) 14. (B, D)
Combining above three equations (1), (2) and (3) ;
1 1 1
3p 5p We have P(X1) = , P(X2) = , P(X3) =
q Î IV quadrant and more precisely <q< 2 4 4
2 3
P(X) = P(at least 2 engines are functioning)
3
(Q – 1 < sin q < - ) = P(X1 Ç X2 Ç X3C ) + P(X1 Ç X2C Ç X3 )
2
Now, + P(X1C Ç X2 Ç X3 ) + P(X1 Ç X2 Ç X3 )
2 cos q (1 – sin f) = sin2q (tan q/2 + cot q/2) cos f – 1
1 1 3 1 3 1 1 1 1 1 1 1 1
1 = ´ ´ + ´ ´ + ´ ´ + ´ ´ =
Þ 2 cos q(1 – sin f) = sin2q × .cos f - 1 2 4 4 2 4 4 2 4 4 2 4 4 4
sin q/2 cos q/2
EBD_780
2012-26 IIT-JEE 2012 SOLVED PAPER
Hence its graph is as shown in figure given below
C P(X1C Ç X ) P(X1C Ç X2 Ç X 3 )
(A) P(X1 / X ) = = Y
P(X ) P(X )
A(0, 1) P Q
æ 1 1ö
1 1 1 çè , ÷
´ ´ 1
2 eø
C
= 2 4 4 = N
1 8
4 T
M B æç 1, 1ö÷
è eø
(B) P [Exactly two engines are functioning /X]
P[(Exactly two engines are functioning) Ç X]
= O S R
X
P(X )
Now, S = area exclosed by curve = ABRO
P(X1C Ç X2 Ç X3 ) + P(X1 Ç X2C Ç X3 ) + P(X1 Ç X2 Ç X3C )
= 1
P(X ) and area of rectangle ORBM =
e
1 1 1 1 3 1 1 1 3 1
´ ´ + ´ ´ + ´ ´ Clearly S > \ A is true.
7 e
=2 4 4 2 4 4 2 4 4 =
1 8
Also x 2 < x " x Î[0, 1]
4
2
P(X Ç X2 ) Þ – x2 > – x Þ e - x ³ e - x " x Î[0, 1]
(C) P(X/X2) =
P (X 2 ) 1 1
-x 2
-x 1
Þ ò e dx > ò e dx = 1 - e
0 0
P(X1 Ç X2 Ç X3 ) + P(X1C Ç X2 Ç X3 ) + P(X1 Ç X2 Ç X 3C )
= 1
P(X2 ) Þ S > 1- \ B is true.
e
Now S < area of rectangle APSO + area of rectangle
1 1 1 1 1 1 1 1 3 CSRN
´ ´ + ´ ´ + ´ ´
2 4 4 2 4 4 2 4 4 =5
= 1 1 æ 1 ö 1
8 Þ S< ´ 1+ ç1- ÷
4 2 è 2ø e
1 1 æ 1 ö
P ( X Ç X1 ) \ S< + ç1- ÷
(D) P ( X /X1 )= 2 eè 2ø
P ( X1 )
Q D is true

=
( ) (
P ( X1 Ç X2 Ç X3 ) + P X1 Ç X2C Ç X3 + P X1 Ç X2 Ç X3C ) Also as


ç1+
1 ö

÷ < 1-
1
e
\ C is incorrect.
P ( X1 )
r r r
16. 3 a , b , c are units vectors such that
1 1 1 1 3 1 1 1 3
´ ´ + ´ ´ + ´ ´ r r2 r r2 r r2
7 a -b + b -c + c -a = 9
= 2 4 4 2 4 4 2 4 4 =
r rr rr rr
1
2
16
( )
Þ 2 ar 2 + b 2 + cr 2 - 2( a.b + b .c + c .a ) = 9

15. (A, B, D) r r r r r r -3
Þ a.b + b .c + c .a =
First of all let us draw a rough sketch of y = e–x. 2
At x = 0, y = 1 r r r 2 r 2 r2 r2 rr rr rr
Also a + b + c = a + b + c + 2(a.b + b .c + c .a )
and at x = 1, y = 1/e
3
dy 2 = 1 + 1 + 1 + 2 × æç - ö÷ = 0
Also = -2xe - x < 0 " x Î(0, 1) è 2ø
dx r r r r r r
Þ a + b + c = 0 Þ (b + c ) = - a
2
\ y = e - x is decreasing on (0, 1) r r r r r r
\ 2 a + 5( b + c ) = 2 a - 5 a = -3 a = 3
IIT-JEE 2012 SOLVED PAPER 2012-27

17. 5 We have f(x) = x + x 2 - 1 \ Area of DPQS =


1 1
´ PS ´ QS = ´ 2 ´ 4 = 4 sq. units
2 2

ì - x + x2 - 1, x < -1 Y
ï
2
ï-
ï x - x + 1, - 1 £ x £ 0

2
ï x - x + 1, 0 < x < 1 Q(2, 4)
ï 2
ïî x + x - 1, x ³ 1
(1, 2)

From graph of the function, we observe there are 5


X¢ (0, 0)P S X
points of local maximum or local minimum. (2, 0)


o 19. 9 Q p(x) has a local maximum at x = 1 and a local
–1 – ½
–2
½ 1 2 minimum at x = 3 and p(x) is a real polynomial of
least degree
Alternative \ Let p¢(x) = k (x – 1)(x – 3 ) = k(x2 – 4x + 3)

æ x3 ö
é 2x - 1 , x < -1 Þ p(x) = k ç - 2 x 2 + 3 x÷ + C
è 3 ø
ê
We have f ¢(x) = ê -2 x - 1 , -1 £ x £ 0
Given p(1) = 6 and p(3) = 2
ê-2 x + 1 , 0< x<1
ê 4
ë 2x + 1 , x >1
Þ k + C = 6 and 0 + C = 2 Þ k = 3
3
1 -1 \ p¢(x) = 3(x – 1)(x – 3) Þ p¢(0) = 9
Critical pts are , , -1, 0 and 1
2 2
1 1 1
–ve +ve –ve +ve –ve +ve 20. 4 Let 4- 4- 4- ........ = y
3 2 3 2 3 2
–1 – ½ 0 ½ 1
1 1 1
We observe at five points f ’(x) changes its sign Þ 4- 4- 4- ....... = y 2
3 2 3 2 3 2
\ There are 5 points of local maximum or local
minimum.
y 2 y
18. 4 We observe both parabola y2 = 8x Þ 4- = y2 Þ y + -4=0
3 2 3 2
and circle x2 + y2 – 2x – 4y = 0 pass through origin
\ One end of common chord PQ is origin. Say Þ 3 2 y 2 + y - 12 2 = 0
P(0, 0)
-1 + 1 + 288
Let Q be the point (2t2, 4t), then it will satisfy the Þ y= (rejecting –ve value)
6 2
equation of circle.
\ 4t4 + 16t2 – 4t2 – 16t = 0 -1 + 17 8
Þ y= =
Þ t4 + 3t2 – 4t = 0 Þ t (t3 + 3t – 4) = 0 6 2 3 2
Þ t (t – 1)(t2 + t – 4) = 0 Þ t = 0 or 1 \ The required value is
For t = 0, we get point P, therefore t = 1 gives point æ 1 ´ 8 ö 4
Q as (2, 4). 6 + log 3/2 ç ÷ = 6 + log 3/2
è3 2 3 2ø 9
We also observe here that P(0, 0) and Q(2, 4) are
end points of diameter of the given circle and focus 2 log 2/ 3
=6+ =6-2=4
of the parabola is the point S(2, 0). log 3/2
EBD_780
2012-28 IIT-JEE 2012 SOLVED PAPER

Paper - 2
PHYSICS 4. (A) Let V be the volume of the material of which the
cylinder is made. The cylinder is half immersed in
SECTION - I water. Therefore the volume of water displaced
1. (A) At t = 0, the relative velocity will be zero. V
because of the material of the cylinder is . Let
T 2
At t = , the relative velocity will be maximum in
4 h be the total height of the cylinder. As the cylinder
magnitude. is half submerged therefore buoyant force
T
At t = , the relative velocity will be zero.
2 A
3T
At t = , the relative velocity will be maximum
4 h
in magnitude h
h'
At t = T, the relative velocity again becomes zero. 2
2. (B) The magnetic moment of a current carrying loop is
r r
given by M = NIA
2
æ aö é pù V rw g hArw g
Here N = 1, A = a2 + 2p ç ÷ = a2 ê1 + ú , the B= +
è2ø ë 2û 2 2
direction is towards positive z-axis. where A is the area of cross-section of the cylinder
r p
\ M = Ia 2 é1 + ù kˆ The weight of the cylinder W = Vrc g
êë 2 úû
The weight of the water inside the cylinder
R = h'Arw g
3. (D) For r < , B=0
2 For equilibrium,

V rw g Ahrw g
+ = V rc g + h ' Arw g
2 2
Here rw = 1
R V h
\ + A = V rc + h ' A
r P 2 2
R
2 \ V + hA = 2Vrc + 2h'A
\ V [1 – 2rc] + hA = 2h'A
h V
\ h' = + [1 - 2rc ]
| B| 2 2A

h
If rc < 0.5 then h ' >
2

h
and if rc > 0.5 then h ' <
2
R R r
2 h
if rc = 0, h ' =
2

R µ 0 é R2 ù 5. (C) The total charge on plate A will be – 80 µC. If qB


For £ r < R, B= êr - úJ and qC be the charges on plate B and C then
2 2 ëê 2r ûú
qB + qC = 80 µC ...(1)
µ i
For r > R, B= 0
2 pr
IIT-JEE 2012 SOLVED PAPER 2012-29

8. (B) Considering the end correction [e = 0.3 D], we get


+80 mC l
L+e=
4
4mF
–80 mC A l
Þ L= –e
4
+qB B +qC C
3mF 336 ´ 100 é vù
2mF
–qB –qC =
512 ´ 4
- 0.3 ´ 4 êëQ l = u úû

= 15.2 cm
9. (A) Axis of rotation is parallel to z-axis.
10. (D) Since the body is rigid, w is same for any point of
Also 2µF and 3µF capacitors are in parallel. the body.
Therefore, 11. (D) K should be less than 0.8 × 106 eV as anti-neutrino
qB qC will have some energy.
=
2 3 12. (C) The energy shared between anti-neutrino and
80 - qC qC electron. If the energy of electron is almost zero
\ = then the maximum energy of anti-neutrino is
2 3
nearly 0.8 × 106 eV.
240 13. (C) As n is negative, therefore direction charges
\ 240 – 3qC = 2qC \ qC =
= 48µC
5 n1 sin q1 = n2 sin q2
This charge will obviously be positive. 14. (B) The physical characteristics remain unchanged.
6. (D) The heat is supplied at constant pressure. 15. (A, C) Impedance across AB
Therefore,
Q = n Cp Dt 2
æ 1 ö 2
5 Z1 = Xc2 + R12 = ç ÷ + R1
= 2 éê R ùú ´ Dt è wC ø
ë2 û

æ 5 ö = (100)2 + (100)2 = 100 2


çQ Cp = R for mono-atomic gas ÷
è 2 ø
V 20
\ I1 = = [leads emf by f1]
5 Z1 100 2
=2× × 8.31 × 5 = 208 J
2
R 100 1
where cos f1 = = = Þ q = 45º
w Z1 100 2 2
7. (C) R R
Q Q 100 µF
O O 100 W
45° A B
I1

P P
0.5 H 50 W
p C D
t =0 t=
4w I2

pR ~
p 20V
The x-coordinate of P = vx × t = wR × =
4w 4
This horizontal distance travelled will be greater than Impedance across CD is
any point on the disc between O and P. Therefore
the landing will be in unshaded area. In the same Z2 = X L2 + R22 = (wL)2 + R22
way, the horizontal distance travelled by Q is always
less than that of any point between O and R.
Therefore the landing will be in unshaded area. = (0.5 ´ 100)2 + (50)2 = 50 2 W
EBD_780
2012-30 IIT-JEE 2012 SOLVED PAPER
At any point ST, the electric field will be directed from
V 20
\ I2 = = [lags emf by f2] S to T. The potential decreases along the electric field
Z2 50 2 line.
17. (B, D) Let the mass of P be m.
3/ 2
I1 4 3 4 éAù
Then m = r´ pr = r´ p ê ú
3 3 ë 4p û
3/2
45º 4 é 4A ù
The mass of Q = r ´ p =8m
45º I 20V 3 êë 4p úû
\ The mass of R = 9 m
If the radius of P = r
I2 Then the radius of Q = 2r
é æ 4A ö
3/2
æ Aö
3/2 ù
R 50 1 êQ rQ = ç ÷ = 2 ç ÷ ú
where cos f2 = = = Þ f2 = 45º ëê è 4p ø è 4p ø ûú
Z2 50 2 2
and radius of R = 91/3r
\ The current I from the circuit is
I = I1 + I2 ; 0.3 A éQ MR = MP + MQ ù
ê ú
16. (A, B, C) 3 3 3 3
ëê rR = r + (2r ) = 9r ûú
+q F E–q 2GMP 2Gm
Now, vP = =
Rp r

+ 2q –2q 2GMQ 2G (8m)


A D vQ = = = 2vP
RQ 2r

2G (9 m)
+q –q vR = = 91/3 vP
B C 91/3 r
18. (A, B) For rolling motion, the velocity of the point of
contact with respect to the surface should be zero.
EB +EE For this
r r
3Rw( -iˆ) + v = 0 o \ v = 3Rw iˆ o

R R
60º w w cos30º kˆ
2 2
O 60º EA ED P
R
w sin 30º ( -iˆ ) 3R w iˆ
2
P
EF +EC

uuur uuur 30º


|EA | uur uuur |ED | uuur uuur
Here =|EB |=|EC | = =| EE |=|EF | = K
2 2 A shown in the figure, the point P will have two
\ EO = EA + ED + (EF + EC) cos 60º + (EB + EC) cos 60º velocities
1 1 (i) 3Rw iˆ (due to translational motion)
= 2K + 2K + (K + K) × + (K + K) × = 6K
2 2 R
(ii) w making an angle of 30º with the vertical
The electric potential at O is 2
1 due to rotation
VO = [2q + q + q – q – q – 2q]
4p e 0 L uur é ˆ Rw ù 3 Rw ˆ
\ vP = ê 3 Rw i - iˆ ú + k
=0 ë 4 û 4
PR is perpendicular bisector (the equatorial line) for
11 3
the electric dipoles AB, FE and BC. Therefore the electric = Rw iˆ + Rw kˆ
potential will be zero at any point on PR. 4 4
IIT-JEE 2012 SOLVED PAPER 2012-31

19. (D) The acceleration of the center of mass of cylinder CHEMISTRY


rolling down an inclined plane is
SECTION - I
g sin q
aP = O
I
1+ P 2 –
MR CN
1. (A) CH3 CH2 C CH3

O OH
C
95%
CH3CH2 C CH3 ¾¾¾¾
H SO
® CH3CH2 C CH3
2 4

ac CN COOH
[G]
D
¾¾
® CH3CH C CH3

COOH
[H]
Here IP > IQ because in case of P the mass is 2. (C) In both states (paramagnetic and diamagnetic) of
concentrated away from the axis. the given complex, Ni exists as Ni 2+ whose
electronic configuration is [Ar] 3d84s0.
g sin q
aQ = 3d 4s 4p
IQ
1+ Ni2+ :
MR2
\ aP < aQ sp3
\ Q will reach the ground earlier with larger In the above paramagnetic state the geometry of
angular speed and larger translational kinetic the complex is sp3 giving tetrahedral geometry.
energy. The diamagnetic state is achieved by pairing of
20. (A) Emf will be induced in the circular wire loop when electrons in 3d orbital.
flux through it changes with time.
Df 3d 4s 4p
e =-
Dt
when the current is constant, the flux changing
through it will be zero. dsp2
Thus the geometry of the complex will be dsp2
giving square planar geometry.
I 3. (B) The reactions involved in cyanide extraction
process are :
Ag 2 S + 4NaCN ® 2Na [Ag(CN)2] + Na2S
(argentite)
®
B 4Na2S + 5O 2 + 2H2O ® 2Na2SO4 + 4NaOH + 2S
Oxiding
® agent
B
2Na[Ag(CN)2] + Zn ® Na2 [Zn(CN)4] + 2 Ag ¯
(reducing
agent)
Circular
loop
4. (C) oxidation
0
–3 +1
P4 + 3NaOH + 3H2O PH3 + 3NaH2PO4
When the current is decreasing at a steady rate
D
then the change in the flux (decreasing inwards) reduction
on the right half of the wire is equal to the change +5
in flux (decreasing outwards) on the left half of the Na3 PO 4 + PH3
wire such that Df through the circular loop is zero.
EBD_780
2012-32 IIT-JEE 2012 SOLVED PAPER
5. (D) XeO2F2 has trigonal bipyramidal geometry, due
CHO CH
to presence of lone pair of electrons on equitorial (CH3CO)2O
¾¾¾¾¾® CH
position, its shape is see-saw. CH3COONa
F COOH
O I (J)
Xe
CH2
O (i) H 2 – Pd CH2
F ¾¾¾¾¾ ®
(ii) SOCl2
6. (A) From Raoult law relation, COCl
p0 – p No.of moles of solute
= anh.AlCl
p0 No. of moles of solvent + No. of moles of solute ¾¾¾¾¾

When the concentration of solute is much lower


than the concentration of solvent, O
(K)
p0 – p No.of moles of solute 11. (D) At anode : M(s) + 2X– (aq) ¾¾
® MX2(aq) + 2e

0
=
p No. of moles of solvent
At cathode : M+2
(aq) + 2e– ¾¾® M(s)
Thus, here n = 2
760 – p 2.5/m
= ...(i) DG = – nFEcell
760 100/18 = – 2 × 96500 × 0.059 × 10–3 kJ/mole
From elevation in boiling point, DTb = Kb × m = – 11.4 kJ/ mole
2 = 0.76 × m 12. (B) M|M2+ (aq) || M2+ (aq) |M
0.001 M
2
m= ...(ii) Anode : ® M 2+ (aq) + 2e-
M ¾¾
0.76
From (i) and (ii), p = 724 mm Cathode : M 2 + (aq) + 2e - ¾¾
®M
__________________________
7. (B) b-Ketoacids undergo decarboxylation easily. 2+
M (aq) c ‡ˆˆˆˆ† M 2 + (aq) a
8. (D) (i) 2C(s) + H2(g) ¾¾
® H – C º C – H(g) 0.059 ìï M 2 + (aq)a üï
E cell = 0 - log í ý
DH = 225 kJ mol–1 2 ïî 10 –3 ïþ
(ii) 2C(s) ¾¾
® 2C(g) DH = 1410 kJ mol–1 ìï M 2+ (aq)a üï
0.059
Þ 0.059 = - log í ý
(iii) H2(g) ¾¾
® 2H(g) DH = 330 kJ mol–1 2 ïî 10-3 ïþ
From equation (i) :
ïì M 2 + (aq)a ïü
-2 = log í ý
225 = éë 2 ´ DHC(s) ¾¾® C(g) + 1 ´ BE H – H ùû îï 10 –3 þï
Þ 10 – 2 × 10– 3 = M2+ (aq)a = solubility = s
- [2 ´ BE C – H + 1 ´ BECº C ] Þ Ksp = 4s3 = 4 × (10–5)3 = 4 × 10–15
225 = [1410 + 1 × 330] – [2 × 350 + 1 × BEC º C] 13. (A) Ca(OCl) Cl ¾¾ ® Ca2+ + –OCl + Cl–
–OCl (Hypochlorite ion) is anion of the acid HOCl
225 = [1410 + 330] – [700 + BEC º C]
which on dehydration gives Cl2O.
225 = 1740 – 700 – BEC º C
BEC º C = 815 kJ mol–1 2HOCl ¾¾
® H2O + Cl2O
SECTION - II 14. (C) Number of millimole of hypo = 0.25 × 48
9. (C) 10. (A) = 2 × millimole of Cl2
Reactions of compound J (C9H8O2) indicates that it 0.25 ´ 48
has C = C linkage and – COOH group. Thus, J can be \ Number of millimole of Cl2 = =6
2
written as C6H5CH = CH COOH. Since, J is unsaturated millimole of Cl2 = millimole of CaOCl2
carboxylic acid and it is formed by the reactions of Molarity of bleaching solution
compound I with (CH3 CO)2 O and CH3COONa,
compound I should be an aldehyde (recall Perkin Millimoles of CaOCl2 6
reaction). Thus the whole series of reactions can be = = = 0.24
Vol.(in mL)of CaOCl 2 25
written as below.
IIT-JEE 2012 SOLVED PAPER 2012-33

15. (A, C, D) 18. (A, B, C)


T1 = T2 because process is isothermal. Converting all the structures in the Fischer projection
Work done in adiabatic process is less than in
isothermal process because area covered by isothermal
curve is more than the area covered by the adiabatic
curve.
In adiabatic process expansion occurs by using internal
energy, hence, it decreases while in isothermal process
temperature remains constant that's why no change in
internal energy.
16. (A, C, D)

KI(aq) + K3[Fe (CN)6](aq) ¾¾


®

KI3(aq) + K4[Fe(CN)6](aq)
Brownish-yellow

ZnSO4(aq)
M and N are diastereoisomers
M and O are identical
K2Zn3 [Fe(CN)6]2 or {K2Zn [Fe(CN)6]}¯ + KI3(aq) M and P are enantiomers
white ppt white ppt M and Q are diastereoisomers
Hence, the correct options are A, B, C.
19. (B, D)
Na2S2O3
(A) Diamond is harder than graphite.
(B) Graphite is good conductor of electricity as each

I (aq) + S4O2– carbon is attached to three C-atoms leaving one
6 (aq)
valency free, which is responsible for electrical
Colourless
conduction, while in diamond, all the four
K 2 Zn[Fe(CN)6 ] + NaOH ¾¾
® valencies of carbon are satisfied, hence insulator.
white ppt.
(C) Diamond is better thermal conductor than graphite.
[Zn(OH)4 ]2– (aq) + [Fe(CN)6]4– (aq) Whereas electrical conduction is due to
availability of free electrons; thermal conduction
17 . (A, C, D)
is due to transfer of thermal vibrations from atom
O to atom. A compact and precisely aligned crystal
like diamond thus facilitates fast movement of heat.
O LiAlH4 (D) In graphite, C – C bond acquires double bond
character, hence higher bond order than in
H3C T diamond.
OH OCOCH3 20. (A, C)
excess
OH OCOCH3 Graph (I) and (III) represent physisorption because, in
(CH3CO)2O
physisorption, the amount of adsorption decreases
with the increase of temperature and increases with
(U) (W) the increase of pressure.
(no chiral centre) (C10H18O4) Graph (II) represent chemisorption, because in
chemisorption amount of adsorption increases with
+ the increase of temperature. Graph (IV) is showing the
CrO3 / H
formation of a chemical bond, hence chemisorption.
O MATHEMATICS
O 1. (D) Q a1, a2, a3, ....... are in H.P.
1 1 1
\ , , ....... are in A.P.
a1 a2 a3
(V)
(Effervescence with NaHCO3) 1 1 1 1
\ = and =
a1 5 a20 25
EBD_780
2012-34 IIT-JEE 2012 SOLVED PAPER
r r
1
+ 19d =
1 1
Þ + 19d =
1
Þ d=
-4 4. (C) Given that a ´ (2iˆ + 3 jˆ + 4kˆ ) = (2iˆ + 3 jˆ + 4kˆ ) ´ b
a1 a20 5 25 475 r r r
Þ (a + b ) ´ (2iˆ + 3 ˆj + 4 kˆ ) = 0
1 1
= + (n - 1) æç
-4 ö r r
Now ÷ But neither a + b nor 2iˆ + 3 jˆ + 4kˆ is a null vector
an 5 è 475 ø
r r r r
\ (a + b ) || (2iˆ + 3 ˆj + 4kˆ) Þ a + b = l(2iˆ + 3 ˆj + 4kˆ )
1
Clearly an < 0 if <0 r
an Also given ar + b = 29 Þ l = ± 1
r r
1 4n 4 \ a + b = ±(2iˆ + 3 ˆj + 4kˆ )
Þ - + <0
5 475 475 r r
\ ( a + b ).(-7 iˆ + 2iˆ + 3 kˆ) = ± 4
99 3
Þ – 4n < – 99 or n > = 24 5. (D) We have PT = 2P + I
4 4 Þ P = 2PT + I Þ P = 2(2P + I ) + I
\ n ³ 25 Þ P = 4P + 3I Þ P + I = 0
Hence least value of n is 25. Þ PX + X = 0 Þ PX = – X
2. (A) The plane passing through the intersection line of 6. (B) The given equation is
given planes is
( 3 1 + a - 1) x 2 + ( 1 + a - 1) x + ( 6 1 + a - 1) = 0
(x + 2y + 3z – 2) + l( x - y + z - 3) = 0
Let a + 1 = y, then equation reduces to
or (1 + l )x + (2 - l)y + (3 + l )z + (-2 - 3l ) = 0 (y1/3 – 1)x2 + (y1/2 – 1)x + (y1/6 – 1) = 0
Dividing both sides by y – 1, we get
2
Its distance from the point (3, 1, –1) is æ y1/3 - 1ö 2 æ y1/ 2 - 1 ö æ y1/6 - 1 ö
3
ç ÷ x +ç ÷ x+ç ÷ =0
è y -1 ø è y -1 ø è y -1 ø
3(1 + l) + 1(2 - l ) - 1(3 + l) + (-2 - 3l ) 2
\ 2 2 2
= Taking limit as y ® 1 i.e. a ® 0 on both sides we
(1 + l ) + (2 - l ) + (3 + l ) 3
1 2 1 1
get x + x+ = 0
-2 l 2 3 2 6
Þ =
2
3l + 4l + 14 3 Þ 2x2 + 3x + 1 = 0
1
7 Þ x = –1, - (roots of the equation)
Þ 3 l 2 + 4 l + 14 = 3 l 2 Þ l = - 2
2
1
\ Required equation of plane is Thus lim a( a) = -1, lim b( a) = -
+ 2
a ®0 a ®0 +
7 7. (A) D4 can show a number appearing on one of
(x + 2y + 3z – 2) – (x – y + z – 3) = 0
2 D1, D2 and D3 in the following cases.
or 5x – 11y + z = 17 Case I : D4 shows a number which is shown by
3. (C) We have only one of D1, D2 and D3.
2 sin P - sin 2P 2 sin P - 2sin P cos P 1 - cos P D4 shows a number in 6C1 ways.
= = One out of D1, D2 and D3 can be selected in 3C1
2 sin P + sin 2P 2 sin P + 2sin P cos P 1 - sin P
ways.
P The selected die shows the same number as on D4
2 sin 2 in one way and rest two dice show the different
= 2 = tan 2 P = ( s - b)( s - c)
P 2 s( s - a) number in 5 ways each.
2 cos 2 \ Number of ways to happen case I
2
= 6C1 × 3C1 × 1 × 5 × 5 = 450
a+b+c Case II : D4 shows a number which is shown by
where s = only two of D1, D2 and D3.
2
As discussed in case I, it can happen in the
( s - b)2 ( s - c )2 ( a + c - b)2 ( a + b - c)2 following number of ways
= = = 6C1 × 3C2 × 1 × 1 × 5 = 90
s(s - a)( s - b)( s - c) 16.D 2
Case III : D4 shows a number which is shown by
2 all three dice D1, D2 and D3.
1´ 9 æ 3 ö
= = ç ÷ Number of ways it can be done
16 D 2 è 4 D ø = 6C1 × 3C3 × 1 × 1 × 1 = 6
IIT-JEE 2012 SOLVED PAPER 2012-35

\ Total number of favourable ways Let the equation of tangent through P be


= 450 + 90 + 6 = 546 y = m (x – 6)
Also total ways = 6 × 6 × 6 × 6 As it touches x2 + y2 = 4
546 91 6m
\ Required Probability = = \ = 2 Þ 36 m2 = 4(m2 + 1)
6 ´ 6 ´ 6 ´ 6 216 m2 + 1
p /2
é 2 æ p + x öù 1
8. (B) ò ê x + ln çè p - x ÷ø ú cos x dx
ë û
Þ m= ±
2 2
-p /2
\ Equations of common tangents are
p /2 p /2
p+xö 1
= ò x 2 cos x dx + ò ln æç ÷ cos x dx y=± ( x - 6)
-p /2 -p /2
èp-xø 2 2
Also x = 2 is the common tangent to the two circles.
p /2 11. (C) For the statement P
=2 ò x 2 cos x dx + 0 [as x2 cos x is an even f(x) + 2x = 2 (1 + x2)
0 Þ (1 – x)2 sin2x + x2 + 2x = 2(1+ x2)
Þ (1 – x)2 sin2x = x2 – 2x + 1 + 1
æp+xö Þ (1 – x)2 sin2x = (1 – x)2 + 1
function and ln ç ÷ cos x is an odd function]
èp-xø Þ (1 – x)2 cos2x = –1
Which is not possible for any real value of x.
= 2 [ x 2 sin x + 2 x cos x - 2 sin x ]p /2 \ P is not true.
0
Also let H(x) = 2f(x) + 1 – 2x (1 + x)
æ 2 ö p2 H(0) = 2f(0) + 1 – 0 = 1
= 2 ç p - 2÷ = -4 and H(1) = 2f(1) + 1 – 4 = – 3
è 4 ø 2
Þ H(x) has a solution in (0, 1)
9. (A) Equation of tangent PT to the circle x2 + y2 = 4 \ Q is true.
at the point P( 3, 1) is x 3 + y = 4 x
2(t - 1)
Let the line L, perpendicular to tangent PT be 12. (B) We have g(x) = ò æç ö
- ln t÷ f (t)dt, x Î(1, ¥)
è t +1 ø
1
x-y 3+l =0
2( x - 1)
As it is tangent to the circle (x – 3)2 + y2 = 1 \ g '(x ) = éê ù
- ln x ú f ( x )
\ length of perpendicular from centre of circle to ë x+1 û
the tangent = radius of circle. Þ Here f(x) > 0 , " x Î(1, ¥)
3+l 2( x - 1)
Þ = 1 Þ l = – 1 or – 5 Also let h(x) = – ln x
2 x +1
\ Equation of L can be x - 3 y = 1 or x - 3y = 5 4 1 -(x - 1)2
h¢(x) = - = < 0, x Î(1, ¥)
10. (D) ( x + 1)2 x ( x + 1)2 x
\ h(x) is decreasing function.
T1 \ For x > 1
T2 h(x) < h(1)
2 Þ h(x) < 0 " x > 1
1
X \ g¢(x) < 0 " x Î (1, ¥ )
C1(0, 0) C2(3, 0) P \ g(x) is decreasing on (1, ¥).
13. (B) Q an = number of all n digit +ve integers formed
by the digits 0, 1 or both such that no consecutive
x=2 digits in them are 0.
and bn = number of such n digit integers ending
From the figure it is clear that the intersection point with 1
of two direct common tangents lies on x-axis. cn = number of such n digit integers ending with 0
Also DPT1C1 ~ DPT2C2 Clearly, an = bn + cn (Q an can end with 0 or 1)
Þ PC1 : PC2 = 2 : 1 Also bn = an – 1
or P divides C1C2 in the ratio 2 : 1 externally and cn = an – 2 [Q if last digit is 0, second last has
\ Coordinates of P are (6, 0) to be 1]
EBD_780
2012-36 IIT-JEE 2012 SOLVED PAPER
\ We get an = an – 1 + an – 2, n ³ 3 18. (A, B)
Also a1 = 1, a2 = 2,
2 2 cos 2 q
By this recurring formula Given f (cos 4 q) = 2
=
a3 = a2 + a1 = 3 2 - sec q 2 cos 2 q - 1
a4 = a3 + a2 = 3 + 2 = 5 1 + cos 2q 1
a5 = a4 + a3 = 5 + 3 = 8 = = 1+
cos 2q cos 2q
Also b6 = a5 = 8
14. (A) By recurring formula, a17 = a16 + a15 is correct 1 1
Let cos 4 q = Þ 2 cos 2 2q - 1 =
Also c17 ¹ c16 + c15 3 3

Þ a15 ¹ a14 + a13 (Q cn = an- 2 ) 2


Þ cos 2q = ±
\ Incorrect 3
Similarly, other parts are also incorrect.
15. (B, D) 3
\ f (cos 4q) = 1 ±
2
ì an + sin px , x Î [2n, 2n + 1]
We have f(x) = íb + cos px , x Î (2n - 1, 2 n) 19. (A, B)
î n
P( X Ç Y )
As f is continuous for all n We know P(X/Y) =
P(Y )
\ At x = 2n, LHL = RHL = f (2n)
Þ bn + cos 2pn = an + sin 2pn = an + sin 2pn 1 1/6 1
Þ bn + 1 = an Þ = Þ P(Y ) =
2 P(Y ) 3
Þ an – bn = 1
\ B is correct. P(X Ç Y )
Also at x = 2n + 1, LHL = RHL = f (2n + 1) Similarly, P(Y/X) =
P (X )
Þ lim an + sin p (2n + 1 – h) 1 1/6 1
h®0 Þ = Þ P (X ) =
3 P (X ) 2
= lim bn+1 + cos p(2n + 1 – h) = an + sin (2n + 1)p
h®0 \ P(X È Y) = P(X) + P(Y) – P(X Ç Y)
Þ an = bn + 1 – 1 = an
1 1 1 2
Þ an – bn + 1 = – 1 = + - =
\ C is incorrect 2 3 6 3
Þ an–1 – bn = – 1 Also P(X Ç Y) = P(X)P(Y)
\ D is correct. Þ X and Y are independent events.
16. (B, C) \ XC and Y are also independent events.
For given lines to be coplanar, we should have 1 1 1
\ P(XC Ç Y) = P(XC) × P(Y) = ´ =
2 0 0 2 3 6
2 k 2 =0 20. (A, B, C, D)
5 2 k x
2
We have f ( x ) = ò et (t - 2)(t - 3)dt
Þ k=±2 0
For k = 2, obviously the plane y + 1 = z is common in
both lines. 2
Þ f ¢(x) = e x .(x - 2)( x - 3)
For k = – 2, the plane is given by
f ¢(x) = 0 Þ x = 2, 3
x -1 y +1 z 2 2
2 -2 2 =0 f ¢¢(x) = e x .2 x( x 2 - 5 x + 6) + e x (2 x - 5)
5 2 -2 f ¢¢(2) = – ve and f ¢¢(3) = + ve
\ x = 2 is a point of local maxima
Þy+z+1=0 and x = 3 is a point of local minima
17. (A, D) Also or xÎ(2, 3) f ’(x)< 0
We know for a third order matrix P,
Þ f is decreasing on (2, 3)
|Adj P| = |P|2 Also we observe
Where
f ¢¢(0) < 0 and f ¢¢(1) > 0
|Adj P|= 1 (3 – 7) – 4(6 – 7) + 4 (2 – 1) = 4
\ There exists some C Î(0, 1) such that f¢¢ (C) = 0
\ |P|2 = 4 Þ|P|= 2 or – 2
\ All the options are correct.
IIT-JEE 2011 PHYSICS
PAPER - 1
SECTION-I
DIRECTIONS : This section contains 7 multiple choice questions.
Each question has four choices (a), (b), (c) and (d) out of which
ONLY ONE is correct.
1. A police car with a siren of frequency 8 kHz is moving with
uniform velocity 36 km/hr towards a tall building which reflects (a) 0% (b) 20%
the sound waves. The speed of sound in air is 320 m/s. The (c) 75% (d) 80%
frequency of the siren heard by the car driver is 6. A meter bridge is set up as shown, to determine an unknown
resistance ‘X’ using a standard 10 ohm resistor. The
(a) 8.50 kHz (b) 8.25 kHz
galvanometer shows null point when tapping-key is at 52
(c) 7.75 kHz (d) 7.50 kHZ cm mark. The end-corrections are 1 cm and 2 cm respectively
2. The wavelength of the first spectral line in the Balmer series for the ends A and B. The determined value of ‘X’ is
of hydrogen atom is 6561 A°. The wavelength of the second
spectral line in the Balmer series of singly-ionized helium
atom is
(a) 1215 A° (b) 1640 A°
(c) 2430 A° (d) 4687 A°
(a) 10.2 ohm (b) 10.6 ohm
3. Consider an electric field E E0 xˆ where E0 is a constant.
(c) 10.8 ohm (d) 11.1 ohm
The flux through the shaded area (as shown in the figure) 7. A ball of mass (m) 0.5 kg is attached to the end of a string
due to this field is having length (L) 0.5 m. The ball is rotated on a horizontal
circular path about vertical axis. The maximum tension that
z
the string can bear is 324 N. The maximum possible value of
(a,0,a) (a,a,a) anguar velocity of ball (in radian/s) is

L
y
(0,0,0) (0,a,0)
x

(a) 2E0a2 (b) 2 E0 a 2 m


(a) 9 (b) 18
(c) 27 (d) 36
E0a 2
(c) E0 a2 (d)
2 SECTION-II
4. 5.6 liter of helium gas at STP is adiabatically compressed to DIRECTIONS : This section contains 4 multiple choice questions.
0.7 liter. Taking the initial temperature to be T1, the work Each question has four choices (a), (b), (c) and (d) out of which
done in the process is ONE OR MORE may be correct.
9 3 8. A metal rod of length ‘L’ and mass ‘m’ is pivoted at one end.
(a) RT1 (b) RT1
8 2 A thin disc of mass ‘M’ and radius ‘R’ (<L) is attached at its
center to the free end of the rod. Consider two ways the disc
15 9 is attached: (case A). The disc is not free to rotate about its
(c) RT1 (d) RT1
8 2 centre and (case B) the disc is free to rotate about its centre.
5. A 2 F capacitor is charged as shown in the figure. The The rod disc system performs SHM in vertical plane after
percentage of its stored energy dissipated after the switch being released from the same displaced position. Which of
S is turned to position 2 is the following statement(s) is (are) true?
EBD_780
2-2011 IIT-JEE 2011 SOLVED PAPER
Paragraph for Question Nos. 12 and 13
A dense collection of equal number of electrons and positive ions
is called neutral plasma. Certain solids containing fixed positive
ions surrounded by free electrons can be treated as neutral plasma.
Let ‘N’ be the number density of free electrons, each of mass ‘m’.
When the electrons are subjected to an electric field, they are
(a) Restoring torque in case A = Restoring torque in displaced relatively away from the heavy positive ions. If the
case B electric field becomes zero, the electrons begin to oscillate about
(b) restoring torque in case A < Restoring torque in case B the positive ions with a natural angular frequency ‘ p’ which is
(c) Angular frequency for case A > angular frequency for called the plasma frequency. To sustain the oscillations, a time
case B. varying electric field needs to be applied that has an angular
(d) Angular frequency for case A < Angular frequency for
frequency , where a part of the energy is absorbed and a part of
case B.
it is reflected. As approaches p all the free electrons are set to
9. A spherical metal shell A of radius RA and a solid metal
sphere B of radius RB(<RA) are kept far apart and each is resonance together and all the energy is reflected. This is the
given charge ‘+Q’. Now they are connected by a thin metal explanation of high reflectivity of metals.
wire. Then 12. Taking the electronic charge as ‘e’ and the permittivity as
‘ 0’. Use dimensional analysis to determine the correct
(a) E inside
A 0 (b) QA > QB
expression for p.
RB
(c) A (d) E Aon surface EBon surface Ne m 0
B RA (a) (b)
m 0 Ne
10. A composite block is made of slabs A, B, C, D and E of
different thermal conductivities (given in terms of a constant
K and sizes (given in terms of length, L) as shown in the Ne 2 Ne 2
(c) (d)
figure. All slabs are of same width. Heat ‘Q’ flows only from m 0 m 0
left to right through the blocks. Then in steady state
13. Estimate the wavelength at which plasma reflection will occur
for a metal having the density of electrons
N 4 1027 m–3. Taking 0 = 10–11 and mass m 10–30,
where these quantities are in proper SI units.
(a) 800 nm (b) 600 nm
(c) 300 nm (d) 200 nm

Paragraph for Question Nos. 14 to 16


Phase space diagrams are useful tools in analyzing all kinds of
(a) heat flow through A and E slabs are same. dynamical problems. They are especially useful in studying the
(b) heat flow through slab E is maximum.
changes in motion as initial position and momenum are changed.
(c) temperature difference across slab E is smallest.
Here we consider some simple dynamical systems in one
(d) heat flow through C = heat flow through B + heat flow
through D. dimension. For such systems, phase space is a plane in which
11. An electron and a proton are moving on straight parallel position is plotted along horizontal axis and momentum is plotted
paths with same velocity. They enter a semi infinite region along vertical axis. The phase space diagram is x(t) vs. p(t) curve
of uniform magnetic field perpendicular to the velocity. in this plane. The arrow on the curve indicates the time flow. For
Which of the following statement(s) is / are true? example, the phase space diagram for a particle moving with
(a) They will never come out of the magnetic field region. constant velocity is a straight line as shown in the figure. We use
(b) They will come out travelling along parallel paths. the sign convention in which positon or momentum upwards (or
(c) They will come out at the same time. to right) is positive and downwards (or to left) is negative.
(d) They will come out at different times.

SECTION-III
Momentum

DIRECTIONS : This section contains 2 paragraphs. Based upon


one of paragraphs 2 multiple choice questions and based on the
other paragraph 3 multiple choice questions have to be answered.
Each of these questions has four choices (a), (b), (c) and (d) out of
which ONLY ONE is correct.
IIT-JEE 2011 SOLVED PAPER 2011-3

14. The phase space diagram for a ball thrown vertically up 16. Consider the spring-mass system, with the mass submerged
from ground is in water, as shown in the figure. The phase space diagram
for one cycle of this system is

(a)

(a)

(b)

(b)
(c)

(c)

(d)

15. The phase space diagram for simple harmonic motion is a


circle centered at the origin. In the figure, the two circles
represent the same oscillator but for different initial (d)
conditions, and E1 and E2 are the total mechanical energies
respectively. Then

SECTION-IV

DIRECTIONS : This section contains 7 questions. The answer


to each of the questions is a single digit integer, ranging from 0 to
9. The bubble corresponding to the correct is to be darkened in
the ORS.
17. A boy is pushing a ring of mass 2 kg and radius 0.5 m with a
stick as shown in the figure. The stick applies a force of 2N
on the ring and rolls it without slipping with an acceleration
of 0.3 m/s2. The coefficient of friction between the ground
and the ring is large enough that rolling always occurs and
(a) E1 2 E2 (b) E1 = 2E2 the coefficient of friction between the stick and the ring is
(c) E1 = 4E2 (d) E1 = 16E2 (P/10). The value of P is
EBD_780
4-2011 IIT-JEE 2011 SOLVED PAPER
1/ N
q2
planar film are in equilibrium, and a k , where ‘k’

is a constant. Then N is
21. A block is moving on an inclined plane making an angle 45°
with the horizontal and the coefficient of friction is . The
force required to just push it up the inclined plane is 3 times
the force required to just prevent it from sliding down. If we
define N = 10 , then N is
18. Four solid spheres each of diameter 5 cm and mass 0.5 kg 22. The activity of a freshly prepared radioactive sample is 1010
are placed with their centers at the corners of a square of disintegrations per second, whose mean life is 109 s. The
side 4 cm. The moment of inertia of the system about the mass of an atom of this radioisotope is 10–25 kg. The mass
diagonal of the square is N 10–4 kg– m2, then N is. (in mg) of the radioactive sample is
19. Steel wire of lenght ‘L’ at 40°C is suspended from the ceiling 23. A long circular tube of length 10 m and radius 0.3 m carries
and then a mass ‘m’ is hung from its free end. The wire is a current I along its curved surface as shown. A wire-loop of
cooled down from 40°C to 30°C to regain its original length resistance 0.005 ohm and of radius 0.1 m is placed inside the
‘L’. The coefficient of linear thermal expansion of the steel is tube with its axis coinciding with the axis of the tube. The
10–5 /° C, Young’s modulus of steel is 1011 N/m2 and radius current varies as I = I0cos(300 t) where I0 is constant. If the
of the wire is 1 mm. Assume that L >>diameter of the wire. magnetic moment of the loop is N 0I0sin(300 t), then ‘N’ is
Then the value of ‘m’ in kg is nearly
20. Four point charges, each of +q, are rigidly fixed at the four
corners of a square planar soap film of side ‘a’. The surface
tension of the soap film is . The system of charges and

PAPER - 2
2. A ball of mass 0.2 kg rests on a vertical post of height 5 m. A
SECTION-I
bullet of mass 0.01 kg, traveling with a velocity V m/s in a
horizontal direction, hits the centre of the ball. After the
DIRECTIONS : This section contains 8 multiple choice questions. collision, the ball and bullet travel independently. The ball
Each question has four choices (a), (b), (c) and (d) out of which hits the ground at a distance of 20 m and the bullet at a
ONLY ONE is correct. distance of 100 m from the foot of the post. The velocity V
1. Which of the field patterns given below is valid for electric of the bullet is
field as well as for magnetic field?

(a) (b)

(a) 250 m/s (b) 250 2 m/s


(c) 400 m/s (d) 500 m/s
3. The density of a solid ball is to be determined in an
experiment. The diameter of the ball is measured with a screw
(c) (d)
gauge, whose pitch is 0.5 mm and there are 50 divisions on
the circular scale. The reading on the main scale is
2.5 mm and that on the circular scale is 20 divisions. If the
measured mass of the ball has a relative error of 2 %, the
relative percentage error in the density is
IIT-JEE 2011 SOLVED PAPER 2011-5

(a) 0.9 % (b) 2.4 % 6. A satellite is moving with a constant speed ‘V’ in a circular
(c) 3.1 % (d) 4.2 % orbit about the earth. An object of mass ‘m’ is ejected from
4. A wooden block performs SHM on a frictionless surface the satellite such that it just escapes from the gravitational
with frequency, 0. The block carries a charge +Q on its pull of the earth. At the time of its ejection, the kinetic energy
surface. If now a uniform electric field E is switched-on as of the object is
shown, then the SHM of the block will be 1
(a) mV 2 (b) mV2
2
3
(c) mV 2 (d) 2mV 2
2
7 A long insulated copper wire is closely wound as a spiral of
(a) of the same frequency and with shifted mean position. ‘N’ turns. The spiral has inner radius ‘a’ and outer radius
(b) of the same frequency and with the same mean position ‘b’. The spiral lies in the XY plane and a steady current ‘I’
(c) of changed frequency and with shifted mean position. flows through the wire. The Z-component of the magnetic
(d) of changed frequency and with the same mean position. field at the centre of the spiral is
5. A light ray travelling in glass medium is incident on glass-
air interface at an angle of incidence . The reflected (R) and
transmitted (T) intensities, both as function of , are plotted.
The correct sketch is

(a)
Intensity

0 NI b 0 NI
b a
(b) (a) n (b) n
2(b a) a 2(b a) b a

0 NI b 0 NI b a
Intensity

(c) n (d) n
2b a 2b b a
8. A point mass is subjected to two simultaneous sinusoidal
displacements in x-direction, x1(t) = A sin t and x2(t) =
2
A sin t . Adding a third sinusoidal displacement
3
(c) x3(t) = B sin ( t + ) brings the mass to a complete rest. The
values of B and are
3 4
Intensity

(a) 2 A, (b) A,
4 3
5
(c) 3 A, (d) A,
6 3
SECTION-II
DIRECTIONS : This section contains 4 multiple choice questions.
(d) Each question has four choices (a), (b), (c) and (d), out of which
ONE OR MORE may be correct.
9. Two solid spheres A and B of equal volumes but of different
Intesitya

densities dA and dBare connected by a string. They are fully


immersed in a fluid of density dF. They get arranged into an
equilibrium state as shown in the figure with a tension in the
string. The arrangement is possible only if
EBD_780
6-2011 IIT-JEE 2011 SOLVED PAPER
SECTION-III
DIRECTIONS : This section contains 6 questions. The answer
to each of the questions is a single-digit integer, ranging from 0 to
9. The bubble corresponding to the correct answer is to be
darkened in the ORS.
13. A series R-C combination is connected to an AC voltage of
angular frequency = 500 radian/s. If the impedance of the
R-C circuit is R 1.25 , the time constant (in millisecond) of
the circuit is
(a) dA < dF (b) dB > dF 14. A silver sphere of radius 1 cm and work function 4.7 eV is
(c) dA > dF (d) dA + dB = 2dF suspended from an insulating thread in freespace. It is under
continuous illumination of 200 nm wavelength light. As
10. A thin ring of mass 2 kg and radius 0.5 m is rolling without
photoelectrons are emitted, the sphere gets charged and acquires
on a horizontal plane with velocity 1 m/s. A small ball of a potential. The maximum number of photoelectrons emitted from
mass 0.1 kg, moving with velocity 20 m/s in the opposite the sphere is A 10z (where 1 < A < 10). The value of ‘z’ is
direction hits the ring at a height of 0.75 m and goes vertically 15. A train is moving along a straight line with a constant
up with velocity 10 m/s. Immediately after the collision acceleration ‘a’. A boy standing in the train throws a ball
forward with a speed of 10 m/s, at an angle of 60° to the
horizontal. The boy has to move forward by 1.15 m inside
the train to catch the ball back at the initial height. The
acceleration of the train, in m/s2, is
16. A block of mass 0.18 kg is attached to a spring of force-
constant 2 N/m. The coefficient of friction between the block
and the floor is 0.1. Initially the block is at rest and the
(a) the ring has pure rotation about its stationary spring is un-stretched. An impulse is given to the block as
CM. shown in the figure. The block slides a distance of 0.06 m
and comes to rest for the first time. The initial velocity of the
(b) the ring comes to a complete stop.
block in m/s is V = N/10. Then N is
(c) friction between the ring and the ground is to
the left.
(d) there is no friction between the ring and the
ground. 17. Two batteries of different emfs and different internal
resistances are connected as shown. The voltage across
11. Which of the following statement(s) is/are correct? AB in volts is
(a) If the electric field due to a point charge varies
as r –2.5 instead of r –2 , then the Gauss law will
still be valid.
(b) The Gauss law can be used to calculate the field
distribution around an electric dipole.
(c) If the electric field between two point charges is
zero somewhere, then the sign of the two 4
18. Water (with refractive index = ) in a tank is 18 cm deep. Oil
charges is the same. 3
(d) The work done by the external force in moving a 7
unit positive charge from point A at potential VA of refractive index lies on water making a convex surface
4
to point B at potential VB is (VB VA). of radius of curvature ‘R = 6 cm’ as shown. Consider oil to
12. A series R– C circuit is connected to AC voltage source. act as a thin lens. An object ‘S’ is placed 24 cm above water
Consider two cases; (A) when C is without a dielectric surface. The location of its image is at ‘x’ cm above the
medium and (B) when C is filled with dielectric of constant 4. bottom of the tank. Then ‘x’ is
The current IR through the resistor and voltage VC across
the capacitor are compared in the two cases. Which of the
following is/are true?

(a) I RA I RB (b) I RA I RB

(c) VCA VCB (d) VCA VCB


IIT-JEE 2011 SOLVED PAPER 7
2011-

SECTION-IV
DIRECTIONS : This section contains 2 questions. Each question has four statements (A, B, C and D) given in Column I and five
statements (p, q, r, s and t) in Column II. Any given statement in Column I can have correct matching with ONE or MORE statement(s)
given in Column II. For example, if for a given question, statement B matches with the statements given q and r, then for the particular
question, against statement B, darken the bubbles corresponding to q and r in the ORS.
19. One mole of a monatomic gas is taken through a cycle ABCDA as shown in the P-V diagram. Column II give the characteristics
involved in the cycle. Match them with each of the processes given in Column I.

Column I Column II
(A) Process A B (p) Internal energy decreases
(B) Process B C (q) Internal energy increases
(C) Process C D (r) Heat is lost
(D) Process D A (s) Heat is gained
(t) Work is done on the gas
20. Column I shows four systems, each of the same length L, for producing standing waves. The lowest possible natural frequency
of a system is called its fundamental frequency, whose wavelength is denoted as f. Match each system with statements given
in Column II describing the nature and wavelength of the standing waves.
Column I Column II
(A) Pipe closed at one end (p) Longitudinal waves

(B) Pipe open at both ends (q) Transverse waves

(C) Stretched wire clamped at both ends (r) f


=L

(D) Stretched wire clamped at both ends (s) f


= 2L
and at mid-point

(t) f
= 4L
EBD_780
8-2011 IIT-JEE 2011 SOLVED PAPER

SOLUTIONS
Paper - 1
1. (a) The frequency f of the siren = 8 kHz.
3. (c) Given E Eo xˆ
The frequency of sound incident on the tall building
This shows that the electric field acts along + x direction
v and is a constant. The area vector makes an angle
f1 = f ...(i)
v vs of 45° with the electric field. Therefore the electric
flux through the shaded portion whose area is
Where v = velocity of sound in air and
vs = velocity of police car (source) a 2a 2 a 2 is E. A EA cos E0 ( 2 a 2 ) cos
The frequency of sound reflected by the tall building
2 1
will be same as the incident frequency (f1). 45° = E0 ( 2 a ) = E0a2
The frequency of the reflected sound as perceived by 2
the driver of the police car moving towards the tall z
building is
v vo
f2 = f1
v
(a,0,a) (a,a,a) = 45°
where vo = velocity of police car (observer) a
E
v v vo
f2 f from (i) A
v vs v y
(0,0,0) (0,a,0)
v vo
= f v v
s

Here v = 320 m/s (given) x

5 4. (a) Initially
vo = vs = 36 = 10 m/s V1 = 5.6 , T1 = 273K, P1 = 1 atm,
18
320 10 33 5
f2 8 8 8.5 kHz (For monoatomic gas)
320 10 31 3
The number of moles of gas is
1 1 1
2. (a) We know that RZ 2 5.6 1
n12 n22 n
22.4 4
The wave length of first spectral line in the Finally (after adiabatic compression)
Balmer series of hydrogen atom is 6561Å . Here n2 = 3 V2 = 0.7
and n1 = 2 For adiabatic compression
1 1 1 5R 1 1
R(1) 2 ...(i) T1V1 T2V2
6561 4 9 36
5
For the second spectral line in the Balmer series of 1 1
V 5.6 3
singly ionised helium ion n2 = 4 and n1 = 2 ; Z = 2 T2 T1 1 T1 = T1 (8) 2/ 3 4T1
V2 0.7
1 2 1 1 3R
= R (2) ...(ii) We know that work done in adiabatic process is
4 16 4
Dividing equation (i) and equation (ii) we get nR T 9
W RT1
5R 4 5 1 8
6561 36 3 R 27 5. (d) When S and 1 are connected
1215 Å The 2 F capacitor gets charged. The potential
difference across its plates will be V.
IIT-JEE 2011 SOLVED PAPER 9
2011-

The potential energy stored in 2 F capacitor

1 1
Ui CV 2 2 V2 V2
2 2
L
When S and 2 are connected
The 8 F capacitor also gets charged. During this
charging process current flows in the wire and some T
T cos
amount of energy is dissipated as heat. The energy
loss is

1 C1 C2 T sin
2
U = 2 C C (V1 V2 ) r
1 2

Here, C1 = 2 F, C2 8 F, V1= V, V2 = 0
mg
1 2 8 4 2 From (i) and (ii)
U (V 0) 2 V
2 2 8 5 r 2
T mr
L
U
The percentage of the energy dissipated = 100
Ui T 324 18
36 rad/s
mL 0.5 0.5 0.5
4 2
V 8. (a,d)
5 100 80% Applying I
V2
6. (b) At Null point For case A : mg sin Mg ( sin ) IA A
2

X 10

2 sin
A B
mg
52 cm 48 cm 2

sin
X 10
1 2

Here 1 = 52 + End correction = 52 + 1 = 53 cm Mg

2 = 48 + End correction = 48 + 2 = 50 cm
For case B : mg sin Mg ( sin ) I B B
X 10 2
53 50 The restoring torque in both the cases is same.
Also I A IB A B
53
X 10.6
5 A B
(a) and (d) are correct options.
7. (d) Here, the horizontal component of tension provides
9. (a,b,c,d)
the necessary centripetal force.
Electric field inside a spherical metallic shell with charge
T sin = mr 2 ...(i) on the surface is always zero. Therefore option [a] is
r correct.
Also sin ...(ii) When the shells are connected with a thin metal wire
L
then electric potentials will be equal, say V.
EBD_780
10-2011 IIT-JEE 2011 SOLVED PAPER
Option (c) is the correct answer.
1 QA 1 QB
V Also
4 0 RA 4 0 RB
TC TC
As RA > RB therefore QA > AB. option [b] is also correct. QC 2 KZ ( TC )
RC 1/ 2 KZ
QA
2 TB TC 3KZ ( TC )
A 4 RA RB2 QA RB2 4 0 R AV QB [ TB = TC]
As QB 2 QB R 2 4 0 RBV
RB 4 / 3KZ 4
B RA A
2
4 RB TD TC 5KZ ( Tc )
QD [ TD TC ]
RD 4 / 5KZ 4
A RB
B RA 3KZ ( TC ) 5KZ ( TC )
QB QD
Option (c) is also correct 4 4
8KZ ( TC )
A B 2 KZ ( TC ) QC
Also E A & EB
0 0
4
(d) is the correct option.
EA A RB 11. (b,d)
1
EB B RA Figure shows that the megnetic field B is present on
EA < EB the right hand side of AB. The electron (e) and proton
Option (d) is also correct (p) moving on straight parallel paths with the same
10. (a,c,d) velocity enter the region of uniform magnetic field.
It is given that heat Q flows only from left to right The entry and exit of electron & proton in the magnetic
through the blocks. Therefore heat flow through A and field makes the same angle with AB as shown.
E slabs are the same. Therefore both will come out travelling in parallel paths.
[a] is correct option A

B × × × × ×
A 3K QB RB E
2K 6K
C RC
B
× × × × ×
4K QC
QA QE

× × × × ×
RA D RE
5K QD RD p
e–
QA = QB + QC + QD = QE
× × × × ×
Since heat flow through slabs A and E is same,
[b] is not correct.
× × × × ×
We know that resistance to heat flow is R =
KA
Let the width of slabs be Z. Then B
The time taken by proton
L 1 4L 4 2 Rp
RA ,R distance arc angle × radius
2 K (4L) Z 8KZ B 3K ( LZ ) 3KZ tp
speed speed speed v
4L 1 4L 4
RC ,R 2 mpv 2 mp
4 K (2 LZ ) 2 KZ D 5K ( LZ ) 5 KZ = v eB eB
L 1 The time taken by electron is
RE
6 K (4 LZ ) 24 KZ
(2 2 ) Re (2 2 ) me v (2 2 ) me
Now, T = QR te
As R E is least, TE is also smallest ie since the v v eB eB
resistance to heat flow is least for slab E, the clearly te is not equal to tp as mp >> me
temperature difference across is smallest. (b) , (d) are correct options
IIT-JEE 2011 SOLVED PAPER 2011-11

12. (c) e = [AT] 17. 4


= [T–1] Under the influence of the force of stick (2N), the point of
–3
N = [L ] contact O of the ring with ground tends to slide. But the
= [M–1 L–3 A2 T4] frictional force f2 does not allow this and creates a torque
o
which starts rolling the ring. A friction force f1 also acts
We do not want Ampere [A] in the expression. This is
between the ring & the stick.
only possible when 0 occurs as square. Therefore
Applying Fnet = ma in the horizontal direction. We get
options a and b are incorrect.

Ne 2 L 3 A2T 2
= T 2 T 1
m o M M 1 L 3 A2T 4 C
R 2N
2 R f1
Ne c
13. (b) p 2 2
m o f2
O
2 – f2 = 2 × 0.3 f2 = 1.4 N
m o
2 c Applying = I about C we get
2
Ne
a
(f2 – f1) R = I = I [ For rolling a = R ]
R
22 10 30 10 11
=2 3 108 = 600 nm
7 4 10 27 (1.6 10 19 ) 2 0.3
[1.4 – × 2] × 0.5 = 2 × (0.5)2 × [ I = MR2]
0.5
14. (d) When the ball is thrown upwards, at the point of throw
(O) the linear momentum is in upwards direction (and = 0.4
has a maximum value ) and the position is zero. As the P
time passes, the ball moves upwards and its momentum Given = P=4
10
goes on decreasing and the position becomes positive.
18. 9
The momentum becomes zero at the topmost point (A).
Let the four spheres be A, B, C, & D
Momentum IXY = I1 + IB + IC + ID = 2 IA + 2IB
2 2
=2 MR 2 Ma 2 2 MR 2
5 5

0 A X D
Position
C

As the time increases, the ball starts moving down 4 cm


with an increasing linear momentum in the downward
direction (negative) and reaches back to its original a = 4 22
position.
These characteristics are represented by graph (d).
15. (c) We know that the A B
Y
total mechanical energy (Amplitude)2
E1 (2a)2 2
& E2 a2 4 MR 2 2 Ma 2
5
E1
4 8 2
E2 M R 2(a)2
5
16. (b) As the mass is osicillating in water its amplitude will go
on decreasing and the amplitude will decrease with 2
8 5
time. Options (c) and (d) cannot be true. = 0.5 2 8 10 4
When the position of the mass is at one extreme end in 5 2
the positive side (the topmost point), the momentum is
zero. As the mass moves towards the mean position = 0.5 2 16 10 4
9 10 4
the momentum increases in the negative direction.
These characteristics are depicted in option (b). N=9
EBD_780
12-2011 IIT-JEE 2011 SOLVED PAPER
19. 3 The force required to just prevent it from sliding down
We know that F2 = mg sin – N = mg (sin – cos )
mg / A mg Given , F1 = 3F2
Y ....(1) sin + cos = 3(sin – cos )
/ A
1 + = 3(1 – ) [ sin = cos ]
Also T ...(2) 4 =2 = 0.5
From (1) and (2) N = 10 = 5
mg mg 22. 1
Y
A T A T dN 1
We know that , N N
dt Tmean
YA T 1011 (10 3 )2 10 5
10
m 3 1
g 10 1010 N
20. 3 109
N = 1019
kq 2 kq 2 1 q 2
i.e. 1019 radioactive atoms are present in the freshly prepared
Felectric 2 constant
2a 2 a2 2 a2 sample.
The mass of the sample = 1019 × 10–25 kg = 10–6kg = 1 mg
2 23. 6
kq kq
2
2
Let us consider an amperian loop ABCD which is a rectangle
2a a
2 as shown in the figure.
kq
2 Applying ampere’s circuital law we get
2
a

D C
L

As the system of charges and planar film is in equlibrium,


therefore A B
q2
constant = a constant
a2 B.d o× (current passing through the loop)
1/ 3
2
q I
a k B.d o
L
N =3 I
21. 5 B× = o
L
oI o
B Io cos (300 t)
L L
F1 The magnetic moment of the loop
mg sin mg cos = (current in the loop) × r2
f mg 1 d
= r2
45° R dt
The pushing force F1 = mg sin + f 2 4
1 d r dB
F1 = mg sin + mg cos = mg (sin + cos ) = (B r2 ) r2
R dt R dt
2 4
f= r o
= I o sin(300 t ) 300
R L

F2 Comparing it with the expression given in the question we


mg sin get
300 2 r 4 1 300(3.14)2 (0.1)4
N 6
R L 0.005 10
IIT-JEE 2011 SOLVED PAPER 13
2011-

Paper - 2
1. (c) The pattern of field lines shown in option (c) is correct
M D
because 100 = 100 3 100
(a) a current carrying toroid produces magnetic field M
lines of such pattern 0.01
(b) a changing magnetic field with respect to time in =2+3× 100 = 3.1%
2.70
a region perpendicular to the paper produces
induced electric field lines of such pattern. 4. (a) The frequency of SHM performed by wooden block is
2. (d) For vertical motion of bullet or ball 1 k
u=0 0= 2 m
s = 5m
t=?
a = 10m/s2

M = 0.2 kg E
v
m = 0.01 kg +Q

when electric field is switched on, the value of k and m


5m is not affected and therefore the frequency of SHM
remains the same. But as an external force QE starts
acting on the block towards right, the mean position of
QE
SHM shifts towards right by
20 m k
100 m correct option is (a).
1 2 1 Note : In SHM if a constant additional force is applied
S ut at 5 10 t 2 then it only shift the quilibrium position and does not
2 2
change the frequency fo SHM.
t = 1 sec 5. (c) When the light is incident on glass - an interface at an
For horizontal motion of ball angle less than critical angle a small part of light will be
xball = Vball t 20 = Vball × 1 = Vball reflected and most part will be transmitted.
When the light is incident greater than the critical angle,
For horizontal motion of bullet
it gets completed reflected (total internal reflection)
xbullet = Vbullet × t 100 = Vbullet × 1 = Vbullet These characteristics are depicted in option (c).
Applying conservation of linear momentum during 6. (b) V is the orbital velocity. If VC is the escape velocity
collision, w e get then Ve = 2 V . The kinetic energy at the time of ejection
mV = mVbullet + MVball
1 1
0.01 V = 0.01 × 100 + 0.2 × 20 KE mVe2 m( 2 V )2 mV 2
2 2
5 7. (a) Let us consider a thickness dx of wire. Let it be at a
V= = 500 m/s
0.01 distance x from the centre O.
3. (c) Diameter D = M.S.R. + (C.S.R) × L.C.
0.5
D = 2.5 + 20 ×
50
dx
D = 2.70 mm
The uncertainty in the measurement of diameter x
D = 0.01 mm. a
We know that O
Mass M M b
= = = 3
Volume V 4 D
3 2
EBD_780
14-2011 IIT-JEE 2011 SOLVED PAPER
N For equilibrium of B :
Number of turns per unit length = T + Vdfg = VdBg
b a
T = Vg (dB – df) ...(2)
N For T > 0, dB > df
Number of turns in thickness dx = dx (b) is the correct option
b a
Small amount of magnetic field is produced at O due to From (1) & (2) Vg (df – dA) = Vg (dB – df)
thickness dx of the wire. df – d A = d B – d f
2df = dA + dB
0NI dx
dB = (d) is the correct option.
2 (b a) x 10. (c) The frictional force between the ring and the ball is
On integrating, we get, impulsive. The angular impulse created by this force
b 0 NI dx tends to decrease the angular speed of the ring about
0 NI b dx
B= a a O. After the collision the angular speed decreases but
2 b a x 2 (b a ) x
the ring remains rotating in the anticlock wise direction.
0 NI Therefore the friction between the ring and the ground
[loge x] ba (at the point of contact) is towards left.
2 (b a )

0 NI b dt
B= loge Jx=2N-s
2 (b a) a
8. (b) Two sinusoidal displacements have amplitude A each, o
Jy=1Ns
with a phase difference of 2 . It is given that
3 c
sinusoidal displacement x3(t) brings the mass to a
11. (c,d)
complete rest. This is possible when the amplitude of
(a) is not correct because it is valid only when E r–2
third is A and is having a phase difference of 4 with (b) is not correct
3 (c) is correct as between two point charges we will get a
respect to x1 (t) as shown in the figure. point where the electric field due to the two point
charges cancel out each other.
(d) is correct when the work done is without accelerating
2 /3 the charge.
12. (a,c)
A A
2
2 /3 1
We know that Z = R2
4 /3 WC
A
The capacitance in case B is four times the capacitance in
9. (a,b,d) case A
Let V be the volume of spheres. Impedance in case B is less then that of case A (ZB < ZA)
For equilibrium of A : V
T + VdAg = Vdfg Now I =
Z
T = Vg (df – dA) ...(1)
For T > 0, df > dA or dA < df I RA I RB . option (a) is correct.
(a) is the correct option
VRA VRB .

VCA VCB
VdF g
[ If V is the applied potential difference access
A VdA g
series R-C circuit then V VR2 VC2 ]
T (c) is the correct option.
13. 4
T Time constant = RC
VdF g
B 2
1
VdB g Impedance = R2
C
IIT-JEE 2011 SOLVED PAPER 2011-15

Given impedance = R 1.25 Considering horizontal motion from the perspective of


observer B. Let u be the speed of train at the time of throw.
2 The horizontal distance travelled by the ball = (u + 5)
1 3.
R 1.25 = R2
C The horizonal distance travelled by the boy

2 1
2 1 = u 3 a ( 3) 2 1.15
2
R (1.25) = R 2
C
As the boy catches the ball therefore
2 1
R 3
= (u 5) 3 = u 3 a 1.15
4 C 2

R 1 5 3 = 1.5a + 1.15
=
2 C 7.51 = 1.5a
2 2 a 5 m/s2
RC = 1000 ms 16. 4
500
Let v be the speed of the block just after impulse. At B, the
RC = 4 ms
block comes to rest. Therefore
14. 7
1 hc A
v
Stopping potential = where hc = 1240eV –nm
e

1 1240 1 B
= 4.7 = 6.2 4.7
e 200 e

1 Loss in K.E. of the block = Gain in P.E. of the spring


= 1.5eV = 1.5V
e + Work done against friction
1 q 1 ne 1 2 1 2
But V = = mv = kx mg.x
4 0 r 4 0 r 2 2

Vr (4 0 ) 1.5 10 2 k 2
n= = v= x gx .
e 9 109 1.6 10 19 m
n = 1.04 × 10 7
2
Comparing it with A × 10z we get, z = 7 v= 0.06 0.06 0.1 10 0.06
15. 5 0.18
From the perspective of observer A, considering vertical
4
motion of the ball from the point of throw till it reaches back v=
at the initial height. 10

Uy = + 5 3 m/s N=4
17. 5
Sy = 0
ay = – 10m/s2 Let i be the current flowing in the circuit. Apply Kirchhoff’s
t=? law in the loop we get

5 3 m/s 10m/s 6v
1
A
B
i
u
5m/s A B

1 2 2 i
Applying S = ut + at 3v
2
0 = 5 3t 5t 2 –3 – 2i – i + 6 = 0
3i = 3
t= 3 sec
i = 1 Amp
EBD_780
16-2011 IIT-JEE 2011 SOLVED PAPER
Now let us travel in the circuit from A to B through battery TA = TD
of 6V, we get U= 0
VA – 6 + 1 × 1 = VB Now, Q = U + W
VA – VB = 5 volt. Q = W.
Alternatively, when dissimilar battery are corrected in parallel The energy obtained by the gas by work done on it is lost to
E1 E2 the surroundings as = 0.
r1 r2 (r, t) are correct matching.
then the emf of the combination is E = 1 1 20. A-p,t; B-p,s; C-q,s; D-q,r
r1 r2 (A) Pipe closed at one end
Waves produced are longitudinal (sound waves)
18. 2
For the convex spherical refracting surface of oil we apply L

1 2 2 1
=
u v R
f

7 4
1
1 7/4 4 0
( 24) v 6 f
24 cm L
v = 21 cm 4
For water-oil interface oil f=4L
(p, t) are correct matching
7 4
V (B) Pipe open at both ends
4 3 =0
waves produced are longitudinal (sound waves)
21 V water
I L
V = 16 cm.
This is the image distance from water-oil interface. Therefore
the distance of the image from the bottom of the tank is 2
cm. f

19. A-p,r,t; B-p,r; C-q,s; D-r,t 2


(A) Process A B
This is an isobaric process in which the volume of the gas f
L
decreases. Therefore work is done on the gas. 2
W = P (3V – V) = 2PV
f = 2L
Also temerature at B is less than temperature at A (p, s) are correct matching.
Heat is lost & internal energy decreases.
(c) Stretched wire clamped at both ends
(p, r, t) are correct matching
Waves produced are transverse in nature. (waves on string)
(B) Process B C
This is an isovolumic/isochoric process in which the pressure
decreases
Here temperature at B is less than temperature at C. f
Heat is lost and internal energy decreases. 2
(p, r) are correct matching.
(C) Process C D f
This is isobaric expansion where temperature at D is greater L
2
than temperature at C. Therefore internal energy increases
and heat is gained. f = 2L
(q, s) are correct matching (q, s) are correct matching.
(D) D A (D) Stretched wave clamped at both ends & mid point
This is a process in which volume decreases. Therefore Waves produced are transverse in nature (waves on string)
work is done on the gas.
Applying PV = nRT
9 PV
for D P (9V) = 1 RTD TD = f
R
f=L
9 PV (q, r) are correct matching.
for A 3P (3V) = 1RTA TA =
R
IIT-JEE 2011 CHEMISTRY
PAPER - 1
SECTION-I
DIRECTION : This section contains 7 multiple choice questions.
Each question has four choices (A), (B), (C) and (D) out of which (B)
ONLY ONE is correct.
1. Extra pure N2 can be obtained by heating
(A) NH3 with CuO (B) NH4NO3
(C) (NH4)2Cr2O7 (D) Ba(N3)2
2. Dissolving 120 g of urea (mol. wt. 60) in 1000 g of water gave
a solution of density 1.15 g/mL. The molarity of the solution (C)
is
(A) 1.78 M (B) 2.00 M
(C) 2.05 M (D) 2.22 M
3. Bombardment of aluminium by -particle leads to its artificial
disintegration in two ways, (i) and (ii) as shown. Products X,
Y and Z respectively are,
(D)
27 (ii) 30
13 Al 15 P Y
(i)

30 30 6. Among the following compounds, the most acidic is


14 Si X 14 Si Z
(A) p-nitrophenol (B) p-hydroxybenzoic acid
(A) proton, neutron, positron (C) o-hydroxybenzoic acid (D) p-toluic acid
(B) neutron, positron, proton 7. AgNO 3 (aq.) was added to an aqueous KCl solution
(C) proton, positron, neutron gradually and the conductivity of the solution was measured.
(D) positron, proton, neutron The plot of conductance versus the volume of AgNO3
4. Geometrical shapes of the complexes formed by the reaction is
of Ni2+ with Cl– , CN– and H2O, respectively, are
(A) octahedral, tetrahedral and square planar
(B) tetrahedral, square planar and octahedral
(C) square planar, tetrahedral and octahedral
(D) octahedral, square planar and octahedral
5. The major product of the following reaction is

(i) KOH

(A)

(A) (P) (B) (Q)


(C) (R) (D (S)
EBD_780
18-2011 IIT-JEE 2011 SOLVED PAPER
SECTION-II Paragraph for Question Nos. 12 and 13
An acyclic hydrocarbon P, having molecular formula C6H10, gave
DIRECTION : This section contains 4 multiple choice questions. acetone as the only organic product through the
Each question has four choices (A), (B), (C) and (D) out of which following sequence of reactions, in which Q is an intermediate
ONE OR MORE may be correct. organic compound.
8. Amongst the given options, the compound(s) in which all (i) conc. H 2SO 4
(catalytic amount)
the atoms are in one plane in all the possible conformations (i) dil. H 2SO 4 / HgSO 4 ( H 2O)
(if any), is (are) P Q 2
(C6 H10 ) (ii) NaBH 4 / ethanol (ii) O 3
(iii) dil. acid (iii) Zn / H 2O

(A)

12. The structure of compound P is


(A) CH3CH2CH2CH2C CH
(B) (B) CH3CH2C C CH2CH3

(C) (C)
(D)
9. Extraction of metal from the ore cassiterite involves
(A) carbon reduction of an oxide ore
(B) self-reduction of a sulphide ore (D)
(C) removal of copper impurity
(D) removal of iron impurity 13. The structure of the compound Q is
10 According to kinetic theory of gases
(A) collisions are always elastic
(B) heavier molecules transfer more momentum to the wall
(A)
of the container
(C) only a small number of molecules have very high
velocity
(D) between collisions, the molecules move in straight lines
with constant velocities
11. The correct statement(s) pertaining to the adsorption of a (B)
gas on a solid surface is (are)
(A) Adsorption is always exothermic
(B) Physisorption may transform into chemisorption at high
temperature
(C) Physiosorption increases with increasing temperature (C)
but chemisorption decreases with increasing
temperature
(D) Chemisorption is more exothermic than physiosorption,
however it is very slow due to higher energy of (D)
activation
Paragraph for Question Nos. 14 to 16
SECTION-III When a metal rod M is dipped into an aqueous colourless
concentrated solution of compound N, the solution turns light
DIRECTION : This section contains 2 paragraphs. Based upon blue. Addition of aqueous NaCl to the blue solution gives a white
one of paragraphs 2 multiple choice questions and based on the precipitate O. Addition of aqueous NH3 dissolves O and gives an
other paragraph 3 multiple choice questions have to be answered. intense blue solution.
Each of these questions has four choices (A), (B), (C) and (D) out 14. The metal rod M is
of which ONLY ONE is correct. (A) Fe (B) Cu
(C) Ni (D) CO
IIT-JEE 2011 SOLVED PAPER 2011- 19
15. The compound N is 19. The work function ( ) of some metals is listed below. The
(A) AgNO3 (B) Zn(NO3)2 number of metals which will show photoelectric effect when
(C) Al(NO3)3 (D) Pb(NO3)2 light of 300 nm wavelength falls on the metal is
16. The final solution contains Metal Li Na K Mg Cu Ag Fe Pt W
(eV) 2.4 2.3 2.2 3.7 4.8 4.3 4.7 6.3 4.75
2 2
(A) Pb(NH3 )4 and CoCl4
20. The maximum number of electrons that can have principal
3 2 1
(B) Al(NH3 )4 and Cu(NH3 )4 quantum number, n = 3, and spin quantum ms , is
2
2
(C) Ag(NH3 )2 and Cu(NH3 )4 21. Reaction of Br 2 with Na2CO3in aqueous solution gives
sodium bromide and sodium bromate with evolution of CO2
2 gas. The number of sodium bromide molecules involved in
(D) Ag(NH3 )2 and Ni(NH3 )6
the balanced chemical equation is
SECTION-IV 22. To an evacuated vessel with movable piston under external
pressure of 1 atm, 0.1 mol of He and 1.0 mol of an
DIRECTIONS : This section contains 7 questions. The answer to unknown compound (vapour pressure 0.68 atm. at 0°C) are
each of the questions is a single digit integer, ranging from 0 to 9. introduced. Considering the ideal gas behaviour, the
The bubble corresponding to the correct is to be darkened in the total volume (in litre) of the gases at 0°C is close to
ORS. 23. The total number of alkenes possible by
17. The difference in the oxidation numbers of the two types of dehydrobromination of 3-bromo-3-cyclopentylhexane using
sulphur atoms in Na2S4O6 is alcoholic KOH is
18. A decapeptide (Mol. wt. 796) on complete hydrolysis gives
glycine (Mol. wt. 75), alanine and phenylalanine. Glycine
contributes 47.0% to the total weight of the hydrolysed
products. The number of glycine units present in the
decapeptide is
EBD_780
20-2011 IIT-JEE 2011 SOLVED PAPER

PAPER - 2

SECTION-I 4. The following carbohydrate is

DIRECTIONS : This section contains 8 multiple choice questions.


Each question has four choices (A), (B), (C) and (D) out of which
ONLY ONE is correct.
1. The freezing point (in °C) of a solution containing 0.1 g
of K3[Fe(CN)6] (Mol. wt. 329) in 100 g of water (Kf = 1.86 K
kg mol–1) is (A) a ketohexose (B) an aldohexose
(A) –2.3 × 10–2 (B) –5.7 × 10–2 (C) an -furanose (D) an -pyranose
5 Oxidation states of the metal in the minerals haematite and
(C) –5.7 × 10–3 (D) –1.2 × 10–2
magnetite, respectively, are
2. Amongst the compounds given, the one that would form a (A) II, III in haematite and III in magnetite
brilliant colored dye on treatment with NaNO2 in dil. HCl (B) II, III in haematite and II in magnetite
followed by addition to an alkaline solution of -naphthol is (C) II in haematite and II, III in magnetite
(D) III in haematite and II, III in magnetite
N(CH3)2
6. Among the following complexes (K P)
(A) K3[Fe(CN)6] (K), [Co(NH3)6]Cl3 (L), Na3[Co(oxalate)3] (M), the
[Ni(H2O)6]Cl2 (N), K2[Pt(CN)4] (O) and [Zn(H2O)6](NO3)2 (P)
the diamagnetic complexes are
(A) K, L, M, N (B) K, M, O, P
NHCH3
(C) L, M, O, P (D) L, M, N, O
(B) 7. Passing H2S gas into a mixture of Mn2+, Ni2+, Cu2+ and
Hg2+ ions in an acidified aqueous solution precipitates
(A) CuS and HgS (B) MnS and CuS
NH2 (C) MnS and NiS (D) NiS and HgS
8. Consider the following cell reaction:
(C) 2
H3C 2Fe(s) O 2(g) 4H (aq) 2Fe(aq) 2H 2 O (l ) ; E 1.67V

At [Fe2+] = 10–3 M, P(O2) = 0.1 atm and pH = 3, the cell


potential at 25ºC is
CH2NH2
(A) 1.47 V (B) 1.77 V
(D) (C) 1.87 V (D) 1.57 V
SECTION-II
3. The major product of the following reaction is DIRECTION : This section contains 4 multiple choice questions.
Each question has four choices (A), (B), (C) and (D), out of which
ONE OR MORE may be correct.
9. Reduction of the metal centre in aqueous permanganate ion
RCH 2OH
involves
H (anhydrous)
(A) 3 electrons in neutral medium
(B) 5 electrons in neutral medium
(A) a hemiacetal (B) an acetal (C) 3 electrons in alkaline medium
(D) 5 electrons in acidic medium
(C) an ether (D) an ester
IIT-JEE 2011 SOLVED PAPER 21
2011-

10. The correct functional group X and the reagent/reaction 13. The maximum number of isomers (including stereoisomers)
conditions Y in the following scheme are that are possible on monochlorination of the following
compound is
(i) Y
X – (CH2)4 – X condensation
polymer
(ii) , heat

(A) X = COOCH3, Y = H2/Ni/heat


(B) X = CONH2, Y = H2/Ni/heat
(C) X = CONH2, Y = Br2/NaOH
(D) X = CN, Y = H2/Ni/heat 14. The total number of contributing structures showing
11. For the first order reaction hyperconjugation (involving C H bonds) for the following
carbocation is
2N 2O5 (g) 4NO2 (g) O 2 (g)
(A) the concentration of the reactant decreases
exponentially with time
(B) the half-life of the reaction decreases with increasing
temperature
(C) the half-life of the reaction depends on the initial
concentration of the reactant
(D) the reaction proceeds to 99.6% completion in eight half-
life duration
12. The equilibrium 15. Among the following, the number of compounds than can
react with PCl5 to give POCl3 is
2Cu1 Cu 0 Cu II
O2, CO2, SO2, H2O, H2SO4, P4O10
in aqueous medium at 25°C shifts towards the left in the
16. The volume (in mL) of 0.1 M AgNO3 required for complete
presence of
precipitation of chloride ions present in 30 mL of 0.01 M

(A) NO3 (B) Cl– solution of [Cr(H2O)5Cl]Cl2, as silver chloride is close to
(C) SCN– (D) CN– 17. In 1 L saturated solution of AgCl [Ksp(AgCl) = 1.6 10–10],
0.1 mol of CuCl [Ksp(CuCl) = 1.0 10–6] is added. The
SECTION-III
resultant concentration of Ag+ in the solution is 1.6 10–x.
DIRECTION : This section contains 6 questions. The answer to The value of “x” is
each of the questions is a single-digit integer, ranging from 0 to 9. 18. The number of hexagonal faces that are present in a
The bubble corresponding to the correct answer is to be darkened
truncated octahedron is
in the ORS.

SECTION-IV

DIRECTION : This section contains 2 questions. Each question has four statements (A, B, C and D) given in Column I and five
statements (p, q, r, s and t) in Column II. Any given statement in Column I can have correct matching with ONE or MORE statement(s)
given in Column II. For example, if for a given question, statement B matches with the statements given q and r, then for the particular
question, against statement B, darken the bubbles corresponding to q and r in the ORS.

19. Match the transformations in column I with appropriate options in column II


Column-I Column-II
(A) CO 2 (s) CO 2 (g) (p) phase transition

(B) CaCO3 (s) CaO(s) CO 2 (g) (q) allotropic change


EBD_780
22-2011 IIT-JEE 2011 SOLVED PAPER
(C) 2H H 2 (g) (r) H is positive
(D) P(white, solid) P(red, solid) (s) S is positive
(t) S is negative
20. Match the reactions in column I with appropriate types of steps/reactive intermediate involved in these reactions as given in
column II
Column I Column II

(A) (p) Nucleophilic substitution


aq. NaOH

(B) CH3MgI (q) Electrophilic substitution

(C) H2SO4 (r) Dehydration

H2SO4

(D) (s) Nucleophilic addition

(t) Carbanion
IIT-JEE 2011 SOLVED PAPER 23
2011-

SOLUTIONS
Paper - 1
SECTION - I [Ni(CN)4]2– = 3d8 configuration with nickel in + 2
oxidation state, CN– being strong field ligand compels
1. (D) Ba(N 3 ) 2 Ba 3N 2 for pairing of electrons.
120 So,
2. (C) Number of moles of urea = 2 4s 4p
60
3d
Total mass of solution = 1000 + 120 = 1120 g
[NiCN4]–2
Mass
Total volume of solution (in L) = 2
Density dsp hybridisation
Hence, complex has square planar geometry.
1120 112
= L 2–
1.15 103 115
NC CN
Number of moles
Molarity of the solution =
Volumein L Ni

2×115 NC CN
= 2.05 mol L–1
112
27 4 30 1 Ni 2 6H 2 O [Ni(H 2 O) 6 ]2
3. (A) 13 A 2 14Si 1p 8
(X) [Ni(H2O)6] = 3d configuration with nickel in + 2
oxidation state. As with 3d8 configuration two d-
27 4 orbitals are not available for d2sp3 hybridisation. So,
13 A 2 15
P30 0n
1
hybridisation of Ni (II) is sp3d2 and Ni (II) with six
(Y)
co-ordination will have octahedral geometry.
30
15 P 14
Si30 1
0 H2O 2+
(Z) H2O OH2
2
4. (B) Ni 4Cl [NiCl4 ]2 Ni
sp 3
[NiCl4]2– . = 3d8 configuration with nickel in + 2 H2O OH2
oxidation state, Cl– being weak field ligand does not H2O
compel for pairing of electrons.
So, Note : With water as ligand, Ni (II) forms octahedral
complexes.
4s 4p 5. (A)
3d
2
[NiCl4 ]
O O
3
sp hybridisation Br CH2 Cl
(1) KOH –
NH N
Hence, complex has tetrahedral geometry Acid base SN2 reaction
reaction
Cl 2– O O
O
Ni
Cl Cl
N CH2 Br
Cl

Ni 2
4CN [Ni(CN) 4 ]2 O
EBD_780
24-2011 IIT-JEE 2011 SOLVED PAPER
6. (c) o-Hydroxybenzoic acid is strongest acid and the
decreasing order of acidity is NaCl AgNO 3 AgCl NaNO3
(remaining) (O)
COOH COOH COOH OH
OH 2AgCl 2NH3 (aq) Ag(NH3 ) 2
> > > Soluble
Cu(NO 3 ) 2 4NH 3 ( aq) [Cu(NH 3 ) 4 ](NO3 ) 2
CH3 OH NO2 (remaining) (Deep blue colour)
SECTION - IV
7. (d) AgNO3 ( aq ) KCl( aq ) AgCl(s ) KNO 3 (aq )
Conductivity of the solution is almost compensated 17. 5
due to formation of KNO3(aq). However, after at end
point, conductivity increases more rapidly due to
O O
addition of excess AgNO3 solution.
+ – .. .. –
SECTION - II Na O S .S. .S. S O Na
8. (B, C) O O
9. (A, C, D)
10. (A, B, C, D) +5
11. (A, B, D) 0
(a) H is negative for adsorption Difference in oxidation number = 5 – 0 = 5
(b) Fact based
(d) Chemical bonds are stronger than vander Waal’s forces, 18. 6
so chemical adsorption is more exothermic. Molecular weight of decapeptide = 796 g/mol
SECTION - III Total bonds to be hydrolysed = (10 – 1) = 9 per molecule
Total weight of H2O added = 9 × 18 = 162 g/mol
12. (d) 13. (b) Total weight of hydrolysis products = 796 + 162 = 958 g
Total weight % of glycine (given) = 47%
CH3 CH3 O
diH2SO 4/HgSO4 958 47
CH3 C C CH CH3 C C CH3 Total weight of glycine in product g 450g
100
CH3 CH3 Molecular weight of glycine = 75 g/mol
(P) 450
Number of glycine molecules = 6
75
19. 4
CH3 OH
hc
NaBH 4/dil acid Energy associated with incident photon =
CH3 C CH CH3
34
CH3 6.6 10 3 108
E= J
9
300 10
conc.H 2SO 4 34
(–H2O) 6.6 10 3 108
= eV 4.16 eV
9 19
300 10 1.6 10
CH3 Photoelectric effect can take place only when Ephoton >
CH3
Thus, number of metals showing photoelectric effect will be
O 3/Zn/H2O
2CH3 C O CH3 C C CH3 (major) 4 (i.e. Li, Na, K and Mg).
20. 9
CH3 Maximum number of electrons (n2) when n = 3 = 32 = 9
(Q) Number of orbitals = 9
1
Number of electrons with ms will be 9.
14. (b) 15. (a) 16. (c) 2
21. 5
Cu 2AgNO 3 Cu(NO3 ) 2 2Ag
(M) (N) Blue 3Br2 3Na 2 CO 3 5NaBr NaBrO3 3CO 2
IIT-JEE 2011 SOLVED PAPER 25
2011-

22. 7
PHe = 1 – 0.68 = 0.32 atm, n = 0.1 H3C CH2 CH2 C CH CH3
nRT 0.1 0.0821 273
V= = =7 (E & Z)
P 0.32
23. 5
Total no. of alkenes will be = 5
H3C CH2 CH C CH2 CH3
Br
or
(E & Z)
H3C CH2 CH2 C CH2 CH3

H3C CH2 CH2 C CH2 CH3


alc.KOH

or
(Only 1)

Paper - 2
SECTION - I
1. (A) Tf = i × Kf × m 6. (C) Complex No. of electrons No. of unpaired
Where m = Molality of the solution in outer d electron (s)
(i.e. number of moles of solute per 1000 g of the solvent) orbital
0.1 3– 5 –
Here m = 100 [Fe(CN)6 ] 3d 1 (CN causes
329
pairing of
0.1 100 electrons)
Thus Tf = 4 × 1.86 × 3+ 6 –
329 [Co(NH3 )6 ] 3d
= 2.3 × 10–2 [Co(oxal.)3 ]
3–
3d
6 –
Thus Tf = 0 – 2.3 × 10–2 = – 2.3 × 10–2 ºC 2+ 8 2
2. (C) Only primary aromatic amines undergo diazotisation [Ni(H2O)6] 3d
2– 8 –
followed by coupling. [Pt(CN)4] 5d D (CN causes
+ pairing of
H
3. (B) electrons)
.. +
O
.. O O [Zn(H2 O)6 ]
2+
3d
10 –
+

RCH 2OH –H + Thus L, M, O and P are diamagnetic.


O
+
O CH2R O OCH2R 7. (A) In presence of acid, ionisation of H2S is supressed, so
H less number of S2– are furnished. Hence only those
An acetal
sulphides are precipitated which has low solubility
OH product (Ksp); thus only CuS and HgS are precipitated.
H H
O 8. (D) Here n = 4, and [H+] = 10–pH = 10– 3
OH
Applying Nernst equation
4. (B) HO
HO OH 0.059 [Fe2 ]2
H E = Eº – log
H n [H ]4 pO2
H
It is a -pyranose hence it is an aldohexose.
0.059 (10 3 )2
5. (D) (i) Haematite is Fe2O3 in which Fe is present in III 1.67 log
oxidation state. 4 (10 3 ) 4 0.1
(ii) Magnetite (Fe3 O4) is an equimolar mixture of FeO
0.03
and Fe2O3. 1.67 log107
Oxidation state of Fe in FeO is II. 2
Oxidation state of Fe in Fe2O3 is III. = 1.67 – 0.105 = 1.565 V
EBD_780
26-2011 IIT-JEE 2011 SOLVED PAPER
0.693
SECTION - II Also, t1/2 = . Which is independent of concentration
K
9. (A,D) and t1/2 decreases with the increase of temperature.
In acidic medium 2.303 100
t99.6 log
MnO 4 Mn 2+ K 0.4

Change in oxidation state of Mn = 7 – 2 = 5 2.303 0.693


t99.6 (2.4) 8 8 t1/ 2
Thus electrons lost = 5 K K
In neutral medium 12. (B,C,D)
Cu2+ ions will react with CN– and SCN– forming [Cu(CN)4]3–
MnO 4 MnO 2
and [Cu(SCN)4]3– leading the reaction in the backward
Change in oxidation state of Mn = 7 – 4 = 3 direction.
Electrons lost = 3
10. (A,B,C,D) Cu 2 2CN Cu(CN) 2
Condensation polymers are formed by condensation of a 2Cu(CN)2 2CuCN (CN)2
diol or diamine with a dicarboxylic acid.
CuCN 3CN [Cu(CN) 4 ]3
O O
|| || Cu 2 4SCN [Cu(SCN) 4 ]3
Hence, X may be C OR or C NH 2 or C N Cu2+ also combines with CuCl2 which reacts with Cu to
produce CuCl pushing the reaction in the backward direction.
O O
|| || CuCl 2 Cu 2CuCl
H 2 / Ni
H3CO C (CH 2 ) 4 C OCH3
SECTION - III
OHH 2 C (CH 2 )4 CH 2 OH
13. 8
O O
|| || CH 3
HO C (CH 2 ) 4 C OH
Polyester |
CH3CH 2 C CH 2 CH 2 Cl Enantiomeric pair = 2
|
O O
|| || H
H 2 / Ni
H 2 N C (CH 2 )4 — C — NH 2 CH3
H 2 NCH 2 (CH 2 )4 CH 2 NH 2 CH3CH2 — C — CHCH3 Two enantiomeric pairs = 4
O O
|| || H Cl
HO—C (CH 2 ) 4 —C—OH

Polyamide CH3
Br2/OH ,
Hofmann bromamide CH3CH2 — C — CH2CH3 =1
reaction
Cl

O O CH2Cl
|| ||
HO— C –(CH 2 ) 4 — C —OH CH3CH2 — C — CH2CH3 =1
H 2 N — (CH 2 )4 — NH 2 Polyamide
H
H 2 / Ni Total = 2 + 4 + 1 + 1 = 8
N C (CH 2 ) 4 C N
H 2 NCH 2 (CH 2 )4 CH 2 NH 2 14. 6

O O a b
|| || H3C CH2CH3
HO— C –(CH 2 ) 4 — C —OH +
c
Polyamide
11. (A, B, D)
For first order reaction
a = 3 Hyperconjugative H's
[A] = [A] 0e–kt b = 2 Hyperconjugative H's
Hence concentration of [NO2] decreases exponentially. c = 1 Hyperconjugative H
IIT-JEE 2011 SOLVED PAPER 27
2011-

15. 4 Ksp of AgCl = [Ag+] [Cl–]


1.6 × 10–10 = x (x + y) ....(i)
PCl5 + SO2 POCl3 + SOCl2
Similarly, Ksp of CuCl = [Cu+][Cl–]
PCl5 + H2O POCl3 + 2HCl 1.6 × 10–6 = y(x + y) … (ii)
On solving, (i) and (ii)
PCl5 + H2SO4 POCl3 + SO2Cl2 + 2HCl [Ag+] = 1.6 × 10–7
x=7
6PCl5 + P4O10 10POCl3 18. 8
16. 6
SECTION - IV
m moles of [Cr (H2O)6Cl]Cl2 = 0.01 × 30 = 0.3
m moles of Cl– = 0.3 × 2 = 0.6 19. A – p, r, s ; B – r, s ; C – t ; D – p, q, t
[1 mole of complex gives 2 Cl– ions] (A) CO2 (s) CO2 (g)
m moles of Ag+ = m moles of Cl– It is phase transition. The process is endothermic
0.1 × V = 0.6 (sublimation). Gas is produced, so entropy increases.
V = 6 mL (B) On heating CaCO 3 decomposes. So, process is
17. 7 endothermic.
Let the solubility of AgCl is x mol litre-1 and that of CuCl is The entropy increases as gaseous product is formed.
y mol litre-1 (C) 2H• H2(g)
Entropy decreases as number of gaseous particles
AgCl Ag Cl decreases.
x x (D) It is phase transition.
CuCl Cu Cl White and red P are allotropes.
Red P is more stable than white.
y y
So H is –ve.

20. A – r, s, t ; B – p, s ;C - r, s; D – q, r
O

CH3 O O
O
H2O
C — CH2 — CH2 — C — CH2
aq/NaOH (Nuclophilic addition) OH
(Carbanion)
O

dehydration
OH /

CH3MgI C — CH2 — CH2 — CH2


CH2CH2CH2Cl
(B) Nucleophilic
addition CH3

O
H2O
(Nucleophilic CH3
substitution)
EBD_780
28-2011 IIT-JEE 2011 SOLVED PAPER

O OH

(C) CH2CH2CH218OH CH2CH2CH218OH

18 18
O O
(Nucleophilic addition)
H2SO4
dehydration OH

CH2CH2CH2C(CH3)2 CH2CH2CH2C(CH3)2 Electrophilic


H2SO4
substitution
(D) OH
dehydration
H3C CH3
IIT-JEE 2011 MATHEMATICS
PAPER - 1
SECTION-I 3 1
(a) (b)
DIRECTIONS : This section contains 7 multiple choice questions. 4 2
Each question has four choices (a), (b), (c) and (d) out of which
ONLY ONE is correct. 1 1
(c) (d)
3 4
1. Let (x0, y0) be the solution of the following equations 6. Let and be the roots of x2 – 6x – 2 = 0, with > . If
n2 n3
(2 x) (3 y ) a10 2a8
n n for n 1, then the value of is
an
3 nx
2 ny 2a9
Then x0 is (a) 1 (b) 2
1 1 (c) 3 (d) 4
(a) (b) 7. A straight line L through the point (3, –2) is inclined at an
6 3
angle 60° to the line 3x y 1. If L also intersects the x -
1
(c) (d) 6 axis, then the equation of L is
2
(a) y 3x 2 3 3 0
n3
x sin x 2
2. The value of dx is (b) y 3x 2 3 3 0
n2
sin x 2 sin( n6 x 2 )
(c) 3y x 3 2 3 0
1 3 1 3
(a) n (b) n
4 2 2 2 (d) 3y x 3 2 3 0

3 1 3 SECTION-II
(c) n (d) n
2 6 2
DIRECTIONS : This section contains 4 multiple choice questions.
3. Let a iˆ ˆj kˆ, b iˆ ˆj kˆ and c iˆ ˆj kˆ be three Each question has four choices (a), (b), (c) and (d) out of which
ONE OR MORE may be correct.
vectors. A vector v in the plane of a and b , whose
8. The vector (s) which is/are coplanar with vectors iˆ ˆj 2kˆ
1
projection on c is , is given by
3 and iˆ 2 ˆj kˆ , and perpendicular to the vector iˆ ˆj kˆ

(a) iˆ 3 ˆj 3kˆ (b) 3iˆ 3 ˆj kˆ is/are

ˆj 3kˆ
(a) ĵ kˆ (b) iˆ ˆj
(c) 3iˆ (d) iˆ 3 ˆj 3kˆ
(c) iˆ ˆj (d) ˆj kˆ
4. Let P = { : sin – cos = 2 cos } and
9. Let M and N be two 3 × 3 non-singular skew- symmetric
Q = { : sin + cos = 2 sin } be two sets. Then matrices such that MN = NM. If PT denotes the transpose
(a) P Q and Q – P of P, then M2N2 (MTN)–1 (MN–1)T is equal to
(b) Q P (a) M2 (b) –N2
(c) P Q (c) – M2 (d) MN
(d) P = Q
5. Let the straight line x = b divide the area enclosed by x2 y2
10. Let the eccentricity of the hyperbola 2
1 be
y = (1 – x)2 , y = 0, and x = 0 into two parts R1 (0 x b) and a b2
reciprocal to that of the ellipse x2 + 4y2 = 4. If the hyperbola
1
R2 (b x 1) such that R1 R2 . Then b equals passes through a focus of the ellipse, then
4
EBD_780
30-2011 IIT-JEE 2011 SOLVED PAPER
15. The probability of the drawn ball from U2 being white is
x2 y2
(a) the equation of the hyperbola is 1 13 23
3 2 (a) (b)
30 30
(b) a focus of the hyperbola is (2, 0)
19 11
5 (c) (d)
(c) the eccentricity of the hyperbola is 30 30
3 16. Given that the drawn ball from U2 is white, the probability
(d) the equation of the hyperbola is x2 – 3y2 = 3 that head appeared on the coin is
11. Let f : R R be a function such that f (x + y) = f (x) + f (y), 17 11
x, y R. If f (x) is differentiable at x = 0, then (a) (b)
23 23
(a) f (x) is differentiable only in a finite interval containing
zero 15 12
(c) (d)
(b) f (x) is continuous x R 23 23
(c) f (x) is constant x R SECTION-IV
(d) f (x) is differentiable except at finitely many points.
DIRECTIONS : This section contains 7 questions. The answer
SECTION-III to each of the questions is a single digit integer, ranging from 0 to
9. The bubble corresponding to the correct is to be darkened in
DIRECTIONS : This section contains 2 paragraphs. Based upon the ORS.
one of paragraphs 3 multiple choice questions and based on the 17. Consider the parabola y2 = 8x . Let 1 be the area of the
other paragraph 2 multiple choice questions have to be answered. triangle formed by the end points of its latus rectum and the
Each of these questions has four choices (a), (b), (c) and (d) out of
which ONLY ONE is correct. 1
point P , 2 on the parabola and
2 2 be the area of the
Paragraph for Questions Nos. 12 to 14
Let a, b and c be three real numbers satisfying triangle formed by drawing tangents at P and at the end
1
1 9 7 points of the latus rectum. Then is
2
[a b c ] 8 2 7 [0 00] ...(E) 18. Let a1, a2, a3 .....a100 be an arithmetic progression with a1 = 3
7 3 7
p
12. If the point P(a, b, c), with reference to (E), lies on the plane and S p = ai ,1 p 100 . For any integer n with
2x + y + z = 1, then the value of 7a + b + c is i 1
(a) 0 (b) 12 Sm
(c) 7 (d) 6 1 n 20 , let m 5n. If S does not depend on n, then
n
13. Let be a solution of x3 – 1 = 0 with Im ( ) > 0 , if a = 2 with
a2 is
3 1 3 19. The positive integer value of n > 3 satisfying the equation
b and c satisfying (E), then the value of a b c is

equal to 1 1 1
(a) – 2 (b) 2 2 3 is
(c) 3 (d) – 3 sin sin sin
n n n
14. Let b = 6, with a and c satisfying (E). If and are the roots
of the quadratic equation ax 2 + bx + c = 0, then 20. Let f : [1, ) [2, ) be a differentiable function such that
x
n
1 1 f (1) = 2. If 6 f (t )dt 3xf ( x) x3 for all x 1 , then the
is 1
n 0
value of f (2) is
(a) 6 (b) 7 21. If z is any complex number satisfying |z – 3 – 2i| 2, then the
6 minimum value of |2z – 6 + 5i| is
(c) (d) 22. The minimum value of the sum of real numbers a–5, a–4,
7
3a–3, 1, a8 and a10 where a > 0 is
Paragraph for Question Nos. 15 and 16
Let U1 and U2 be two urns such that U1 contains 3 white and 2 red 1 sin
balls, and U2 contains only 1 white ball. A fair coin is tossed. If 23. Let f ( ) = sin tan , where .
cos 2 4 4
head appears then 1 ball is drawn at random from U1 and put into
U2. However, if tail appears then 2 balls are drawn at random from d
Then the value of ( f ( )) is
U1 and put into U2 . Now 1 ball is drawn at random from U2. d (tan )
IIT-JEE 2011 SOLVED PAPER 31
2011-

PAPER - 2
SECTION-I 6. Let f : [– 1, 2] [0, ) be a continuous function such that
f(x) = f(1 – x) for all x [–1, 2]
DIRECTIONS : This section contains 8 multiple choice questions.
Each question has four choices (a), (b), (c) and (d) out of which 2
ONLY ONE is correct. Let R1 xf ( x )dx , and R 2 be the area of the region
1
x2 y2 bounded by y = f (x), x = –1, x = 2, and the x -axis.
1. Let P(6, 3) be a point on the hyperbola 2 2
1 . If the
a b Then
normal at the point P intersects the x-axis at (9, 0), then the (a) R1 2R2 (b) R1 3R2
eccentricity of the hyperbola is
(c) 2R1 R2 (d) 3R1 R2
5 3
(a) (b) 7. Let f (x) = x2 and g(x) = sin x for all x R. Then the set of all
2 2
x satisfying (f o g o g o f) (x) = (g o g o f) (x), where (f o g) (x)
(c) 2 (d) 3 = f (g(x)), is
2. A value of b for which the equations (a) n , n {0,1, 2,....}
x2 + bx – 1 = 0
x2 + x + b = 0 (b) n , n {1, 2,....}
have one root in common is
(a) 2 (b) i 3 (c) 2n , n {... 2, 1, 0,1, 2....}
2
(c) i 5 (d) 2 (d) 2n , n {... 2, 1, 0,1, 2,....}
3. Let 1 be a cube root of unity and S be the set of all non-
8. Let (x, y) be any point on the parabola y2 = 4x. Let P be the
point that divides the line segment from (0, 0) to (x, y) in the
1 a b
ratio 1 : 3. Then the locus of P is
singular matrices of the form 1 c
(a) x2 = y (b) y2 = 2x
2
1 (c) y2 = x (d) x2 = 2y
where each of a, b and c is either or 2. Then the number SECTION-II
of distinct matrices in the set S is
(a) 2 (b) 6 DIRECTIONS : This section contains 4 multiple choice questions.
(c) 4 (d) 8 Each question has four choices (a), (b), (c) and (d), out of which
4. The circle passing through the point (– 1, 0) and touching ONE OR MORE may be correct.
the y-axis at (0, 2) also passes through the point
3 5 x , x
(a) ,0 (b) ,2 2 2
2 2
9. If f (x) = cos x , x 0, then
3 5 2
(c) , (d) (– 4, 0) x 1 , 0 x 1
2 2
nx , x 1
1/ x
5. If lim 1 x n (1 b2 ) = 2b sin 2 , b 0 and
x 0
(a) f (x) is continuous at x =
( , ], then the value of is 2
(b) f (x) is not differentiable at x = 0
(a) (b) (c) f (x) is differentiable at x = 1
4 3
3
(d) f (x) is differentiable at x =
2
(c) (d)
6 2
EBD_780
32-2011 IIT-JEE 2011 SOLVED PAPER
10. Let E and F be two independent events. The probability 16. Let M be a 3 × 3 matrix satisfying
11 0 1 1 1 1 0
that exactly one of them occurs is and the probability of
25 M 1 2 , M 1 1 1 0
, and M . Then the
0 3 0 1 1 12
2
none of them occurring is . If P(T) denotes the probability
25 sum of the diagonal entries of M is
of occurrence of the event T, then 17. Let a i kˆ, b iˆ ˆj and c iˆ 2 ˆj 3kˆ be three given
4 3 1 2
(a) P(E) = , P(F) = (b) P(E) = , P(F) = vectors. If r is a vector such that r b c b and r . a 0 ,
5 5 5 5
then the value of r . b is
2 1 3 4
(c) P(E) = , P(F) = (d) P(E) = , P(F) = 18. The straight line 2x – 3y = 1 divides the circular region
5 5 5 5
x2 + y2 6 into two parts.
11. Let L be a normal to the parabola y2 = 4x. If L passes through
3 5 3 1 1 1 1
the point (9, 6), then L is given by If S = 2, , , , , , , then the number of
4 2 4 4 4 8 4
(a) y – x + 3 = 0 (b) y + 3x – 33 = 0
(c) y + x – 15 = 0 (d) y – 2x + 12 = 0 points (s) in S lying inside the smaller part is
b x SECTION-IV
12. Let f : (0, 1) R be defined by f (x) = , where b is a
1 bx
constant such that 0 < b < 1. Then DIRECTIONS : This section contains 2 questions. Each question
(a) f is not invertible on (0, 1) has four statements (A, B, C and D) given in Column I and five
statements (p, q, r, s and t) in Column II. Any given statement in
1 Column I can have correct matching with ONE or MORE
(b) f f –1 on (0, 1) and f (b) = f '(0)
statement(s) given in Column II. For example, if for a given
question, statement B matches with the statements given q and r,
1
(c) f = f –1 on (0, 1) and f (b) = f '(0) then for the particular question, against statement B, darken the
bubbles corresponding to q and r in the ORS.
(d) f –1 is differentiable (0, 1)
19. Match the statements given in Column-I with the values
given in Column-II.
SECTION-III
Column-I Column-II
DIRECTIONS : This section contains 6 questions. The answer
to each of the questions is a single-digit integer, ranging from 0 to (A) If a ˆj 3kˆ, b ˆj 3kˆ and (p)
6
9. The bubble corresponding to the correct answer is to be
darkened in the ORS. c 2 3kˆ form a triangle, then
the internal angle of the triangle
i
13. Let e3 , and a, b, c, x, y, z be non-zero complex numbers between a and b is
such that b
2
a+b+c=x (B) If (f (x) – 3 x ) dx = a2 – b2, (q)
3
a+b +c 2=y a

a+b 2+c =z
then the value of f 6 is
| x |2 | y |2 | z |2
Then the value of is
| a |2 | b |2 | c |2
2 5/6
14. The number of distinct real roots of (C) The value of sec( x) dx (r)
n3 3
7/6
x4 – 4x3 + 12x2 + x – 1 = 0 is
15. Let y (x) + y(x) g (x) = g(x), g (x), y(0) = 0, x R, where f (x) is
(D) The maximum value of (s)
denotes df ( x) an d g(x) is a given non-con stant
dx 1
Arg for |z| = 1, z 1 is (t)
differentiable function on R with g(0) = g(2) = 0. Then the 1 z 2
value of y(2) is given by
IIT-JEE 2011 SOLVED PAPER 33
2011-

20. Match the statements given in Column-I with the intervals/union of intervals given in Column-II.
Column-I Column-II
2iz
(A) The set Re : z is a (p) ( , 1) (1, )
1 z2
complex number, |z| = 1.
z ± 1} is
(B) The domain of the funcion (q) ( , 0) (0, )

8(3) x 2
f (x) = sin –1 is
1 32( x 1)

1 tan 1
(C) If f ( ) = tan 1 tan , (r) [2, )
1 tan 1

then the set f ( ):0 is (s) ( , 1] [1, )


2
(D) If f (x) = x3/2 (3x – 10), x 0, (t) ( ,0] [2, )
then f (x) is increasing in
EBD_780
34-2011 IIT-JEE 2011 SOLVED PAPER

SOLUTIONS
Paper - 1
1. (c) We have (2x) n2 = (3y) n3
1
n2. n2x = n3. n3y Projection of v on c is
n2. n2x = n3. ( n3 + ny) ...(1) 3
Also given 3 nx = 2 ny
nx. n3 = ny. n2 v.c 1
|c| 3
nx. n3
ny =
n2 ( ) ( ) ( ) 1
Substituting this value of ny in equation (1), we get 3 3
nx. n3 – = 1 or = + 1
n2. n2x = n3 n3
n2 v = (2 1)iˆ ˆj (2 1) kˆ
( n2)2 n2x = ( n3)2 n2 + ( n3)2 nx
( n2)2 n2x = ( n3)2 ( n2 + nx) For = 1, v = 3iˆ ˆj 3kˆ.
( n2)2 n2x – ( n3)2 n2x = 0
[( n2)2 – ( n3)2] n2x = 0 4. (d) P = { : sin cos 2 cos }
n2x = 0
sin = ( 2 1) cos
1
2x = 1 or x = 2 1
2 tan =

1 n3 2 x sin x 2 Q = { : sin cos 2 sin }


2. (a) I = 2 dx
n2 sin x 2 sin( n6 x 2 ) cos = ( 2 1)sin
Let x2 = t 2x dx = dt
or tan = 2 1
Also, when x = n 2 , t = n2
P=Q
when x = n3 , t = n3
b
n3
b
2 ( x 1)3 (b 1)3 1
1 sin t dt 5. (b) R1 = ( x 1) dx =
I= ...(1) 0 3 3
2 n2 sin t sin( n 6 t ) 0

b b
Using f ( x ) dx f (a b x)dx
a a Y
We get
1 n3 sin( n6 t )
I= dt ...(2) 1
2 n2 sin t sin( n6 t )
Adding values of I in equation (1) and (2)
R1
1 n3 1 1 3
2I= 1 dt ( n3 n 2) n R2
2 n2 2 2 2 O 1
X X
1 3 Y x=b
I= n
4 2
3. (c) As v lies in the plane of a and b 1
1
2 ( x 1)3 (b 1)3
v = a b R2 = ( x 1) dx =–
b 3 3
b
v = ( + ) iˆ ( ) ˆj ( )kˆ
IIT-JEE 2011 SOLVED PAPER 2011-35

1 2(b 1)3 1 1 r = ( ˆj kˆ)


As R1 – R2 =
4 3 3 4
For = 1, we get r ĵ kˆ
1 1 1
or (b – 1)3 = or b – 1 = or b = and for = –1, we get r ĵ kˆ
8 2 2
a and d are the correct options.
6. (c) are the roots of x2 – 6x – 2 = 0
2–6 –2=0
9. (c) [As a skew symmetric matrix of order 3 cannot be non
singular, therefore the data given in the question is
10 – 6 9– 2 8 = 0
inconsistent.]
10 – 2 8 = 6 9 ...(1) We have
Similarly 10 – 2 8 = 6 9 ...(2) M2N2 (MT N)–1 (MN–1)T = M2N2N–1 (MT)–1 (N–1)T
From equation (1) and (2) MT
10 – 10 – 2 ( 8– 8) = 6 ( 9– 9) = M2 N (MT)–1 (N–1)T MT = –M2NM–1 N–1M
a10 – 2a8 = 6a9 ( MT = –M, NT = –N and (N–1)T = (NT)–1
= – M (NM) (NM)–1 M ( MN = NM)
a10 2a8
3 = – MM = –M2
2a9
x2 y2
7. (b) Let the slope of line L be m. 10. (b, d) For x2 + 4y2 = 4 or 1
4 1
m 3
Then = 3 1 3
1 3m e= 1
4 2

Y b 2
As per question, 1
2 3
a
3x y 1
L b 1
(0, 1)
a 3
X O X
60° focus of ellipse is (± 3 , 0)
Y (3, –2)
As hyperbola passes through (± 3 , 0)

m + 3 = ± ( 3 – 3m) 3
1 or a = 3
a2
4m = 0 or 2m = 2 3
b = 1 and focus of hyperbola (± 2, 0)
m = 0 or m = 3
x y2
Equation of hyperbola is 1
L intersects x -axis, m= 3 3 1
or x2 – 3y2 = 3
Equation of L is y + 2 = 3 (x – 3) 11. (b, c) f (x + y) = f (x) + f (y) V x, y R
or 3 x – y – (2 + 3 3 ) = 0 Putting x = y = 0, we get
f (0) = 0
ˆj 2kˆ, b = iˆ 2 ˆj kˆ, c = iˆ ˆj kˆ
8. (a, d) Let a = iˆ f ( x h) f ( x)
Also f (x) = lim
Required vector is coplanar with a and b h 0 h

r a b f ( h)
= lim = f (0) = k (say)
h 0 h
or r iˆ + ) ĵ + ) k̂
f (x) = kx + c
As r c r .c 0 But f (0) = 0 c = 0
+ + +2 +2 + =0 f (x) = kx
+ =0 Which is continuous and differentiable V x R.
=– b and c are the correct options.
EBD_780
36-2011 IIT-JEE 2011 SOLVED PAPER
12. (d) From equation (e), we get 17. 2
a + 8b + 7c = 0
1 3
9a + 2b + 3c = 0 1= Area of PLL = 8 6
2 2
a+ b+ c =0
Y
1 8 7
L(2,4)
Here 9 2 3 0,
1 1 1
A
Therefore system has infinite many solutions. p(½,2)
Solving these, we get
X
b = 6a and c = – 7a C O
X
Now (a, b, c) lies on 2x + y + z = 1 B
2a + 6a – 7a = 1 a = 1
7a + b + c = 7 + 6 – 7 = 6
13. (a) If a = 2 then b = 12, c = –14
L (2,–4)
3 1 3 3 1 3 Y
a b c = 2 12 14
Equation of AB, y = 2x + 1
= 3 +1+3 2=1–3=–2
Equation of AC, y = x + 2
14. (b) If b = 6 then a = 1, c = – 7
Equation of BC, – y = x + 2
Equation becomes x2 + 6x – 7 = 0 or (x + 7) (x – 1) = 0
Solving above equations we get
whose roots are 1 and –7.
Let = 1 and = – 7 A (1, 3), B (–1, –1), C (–2, 0)

n n 1 3 1
1 1 6 1
7 = 1
1 7 6 7 1 1 1 3
n 0 n 0 1 2= 2
7 2 0 1
15. (b) P (white) = P (H white) + P ( T white)
= P (H) P (white/H) + P (T) P (white/T)
1
2
1 3 2 1 1 2
= 1
2 5 5 2 2 18. 9
3 2 3
C2 C2 1 C1. C1 2 5n
× 1 2 3 (5n 1)d
5
C2 5
C2 3 5
C2 3 S S5 n 2
We have m
Sn Sn n
[6 (n 1)d ]
1 8 1 3 1 12 2
=
2 10 2 10 30 30
5[(6 d ) 5nd ]
4 11 23 =
(6 d ) nd ]
=
10 30 30
which will be independent of n if d = 6 or d = 0
1 3 2 1 For a proper A.P. we take d = 6
1
P ( H white) 2 5 5 2 then a2 = 3 + 6 = 9
16. (d) P(H/white) = 19. 7
P(white) 23
30 We have

4 1 1 1
=
10 12 3 2
sin sin sin
= 23 23 n n n
30
IIT-JEE 2011 SOLVED PAPER 2011-37

21. 5
3 Given |z – 3 – 2i| < 2
sin sin
n n 1 which represents a circular region with centre (3, 2)
=
3 2 and radius 2.
sin sin sin
n n n
5
Now |2z – 6 + 5i| = 2 z 3
2
i
2
2cos sin 1 = 2 × distance of z from P (where Z lies in or on the circle)
n n =
2
sin sin sin
n n n Y

2 2 3 (3, 2)
2sin cos = sin
n n n

4 3
sin sin =0 O A X
n n 5/2
7 P 3, –5
2 cos sin =0 2
2n 2n

7 5
cos = 0 or sin =0 Also min distance of z from P =
2n 2n 2
Minimum value of |2z – 6 + 5i| = 5
7 22. 8
= (2k 1) or = 2k where k Z
2n 2 2n a > 0, a–5, a–4, 3a–3, 1, a8, a10 > 0
Using AM > GM for positive real numbers we get
7 1
n= or n = 1 1 1 1 1
2k 1 4k 1 a8 a10
a5 a4 a3 a3 a3
(not possible for any integral value of k) 8
As n > 3, for k = 0, n is integer we get n = 7.
1
20. 6 1 1 1 1 1 8 10 8
. . . . .1.a .a
x a 5 a 4 a 3 a3 a 3
6 f (t )dt 3 xf ( x ) x 3
1 1
1 1 3
Differentiating, we get 6f(x) = 3 f(x) + 3xf (x) – 3x2 1 a8 a10 8(1) 8
a5 a 4 a3
1 Minimum value of the sum = 8
f (x) – f ( x) x 23. 1
x
1 sin
1 f( ) = sin tan
I.F. = cos 2
x

1 1 sin
Solution is f(x). = 1.dx x c = sin sin 2
x sin cos 2
f(x) = x2 + cx
But f (1) = 2 c=1 f (x) = x2 + x 1 x 1 x
tan sin
Hence f (2) = 4 + 2 = 6 y x 2
y2
Note : Putting x = 1 in given integral equation, we get
1 sin
= sin sin = tan
1 cos
f (1) = while given f(1) = 2.
3
df ( )
Data given in the question is inconsistent. 1.
d tan
EBD_780
38-2011 IIT-JEE 2011 SOLVED PAPER

Paper - 2
a 2 and c 2 where is complex cube root of
x2 y2 unity.
1. (b) For hyperbola 1 , we have
a2 b2 As a, b and c are complex cube roots of unity
a and c can take only one value i.e. while b can
2x 2 y dy dy b2 x take 2 values i.e. and 2.
=0
a2 b2 dx dx a2 y Total number of distinct matrices
Slope of normal at P (6, 3) =1×1×2=2
4. (d) Let centre of the circle be (h, 2) then radius = |h|
1 3a 2 Equation of circle becomes (x – h)2 + (y – 2)2 = h2
=
dy 6b 2 As it passes through (–1, 0)
dx (6,3)

Equation of normal is

y 3 3a
x 6 6b (h, 2)
As it intersects x-axis at (9, 0) (0, 2)

0 3 3a 2
a2 2b2 ...(1)
9 6 6b2
Also for hyperbola, b2 = a2 (e2 – 1)
Using a2 = 2b2; we get
b2 = 2b2 (e2 – 1)

1 3 3
= e2 – 1 or e2 = or e = (–1 – h)2 + 4 = h2
2 2 2
2. (b) Let be the common root of given equations, then 5
2+b –1=0 h=
...(1) 2
and 2 + +b=0 ...(2)
5
Subtracting (2) from (1), we get Centre , 2 and r = 5
(b – 1) – (b + 1) = 0 2 2

b 1 5
or = Distance of centre from (–4, 0) is
b 1 2
Substituting this value of in equation (1), we get It lies on the circle.
2 1
b 1 b 1
b 1 =0 5. (d) lim [1 x n(1 b2 )] x = 2b sin2
b 1 b 1 x 0

or b3 + 3b = 0 1
lim n[1 x n (1 b 2 )]
ex 0x
= 2b sin2
b = 0, i 3, i 3
3. (a) For the given matrix to be non singular 2
lim n[1 x n(1 b )] n (1 b 2 )
x 0 2
x n (1 b )
e = 2b sin2
1 a b
1 c 0 n (1 b 2 ) = 2b sin2
e
2
1 1 + b2 = 2b sin2

1 – (a + c) + a c 2 0 1
2 sin2 =b+
(1 – a ) (1 – c ) 0 b
IIT-JEE 2011 SOLVED PAPER 39
2011-

1 Let P (h, k) divides OA in the ratio 1 : 3


We know that 2 sin2 < 2 and b + > 2 for b > 0
b
t 2 2t
Then (h, k) = ,
1 4 4
2 sin2 = b + =2 sin2 = 1
b
t t
As ( , ], h= and k =
4 2

=± h = k2
2 locus of P (h, k) is x = y2.
6. (c) We have 9. (a, b, c, d)
2 2
R1 = xf ( x)dx (1 x ) f (1 x)dx At x =
1 1 2
b b
[Using f ( x )dx = f (a b x )dx ]
a a LHL = im x =0
1 2
x
2 2
R1 = (1 x ) f ( x )dx [As f (x) = f (1–x) on [–1, 2]
1

2 2 RHL = im cos x = 0 and f 0


R1 + R1 = xf ( x)dx (1 x ) f ( x)dx 2
x
1 1 2

2
2R1 = f ( x )dx = R2 LHL = RHL = f
1 2
7. (a) Given that f(x) = x2 and g(x) = sin x, V x R
Then (gof) (x) = sin x2 f(x) is continuous at x =
2
(gogof) (x) = sin (sin x2)
Also at x = 0
(fogogof) (x) = sin 2 (sin x2)
Lf (0) = sin 0 = 0; Rf (0) = 1 – 0 = 1
As given that (fogogof) (x) = (gogof) (x)
Lf (0) Rf (0)
sin 2 (sin x2) = sin (sin x2)
f is not differentiable at x = 0
sin (sin x2) = 0, 1
At x = 1
sin x2 = n or ( (4 n 1) where n Z Lf (1) = R f (1) f is differentiable at x = 1.
2
3
2 At x = , f (x) = – cos x which is differentiable.
sin x2 = 0 sin x [ 1,1] 2
x2 = n All four options are correct.
10. (a,d)
x=± n where n W
E and F are independent events
8. (c) Let A (x, y) = (t2, 2 t) be any point on parabola y2 = 4x.
P( E F ) = P(E). P(F) ...(1)
A(x, y) = (t , 2t)2 Given that
11
P(E F ) P( E F) =
25
3
11
P P(E) P ( F ) + P ( E ) P (F) =
O 25
1
(0, 0) 11
P(E) ( 1 – P (F)) + (1 – P (E)) P (F) =
25

11
P (E) – P (E) P (F) + P (F) – P(E) P (F) =
25
EBD_780
40-2011 IIT-JEE 2011 SOLVED PAPER
x2 (1 – b2) = x1 (1 – b2)
11
P (E) + P (F) – 2 P(E). P(F) = ...(2) x1 = x2 as 1 – b2 0
25
f is one one.
2 b x
and P( E F) = Also =y
25 1 bx
2 b – x = y – bxy
P( E ) P( F ) = (by – 1) x = y – b
25

2 y b
[1 – P(E)] [1 – P(F)] = x=
25 by 1

2 1
1 – P(E) – P(F) + P(E) P(F) = ...(3) For y = , x is not defined
25 b
Adding equation (2) and (3) we get f is not onto.
f is not invertible
13
1 – P(E) P (F) =
25 1(1 bx) ( b)(b x) b2 1
Also f (x) = =
(1 bx) 2 (1 bx )2
12
or P(E) P (F) = ...(4)
25 1
f (b) = 2 and f (0) = b2 – 1
Using the result in equation (2) we get b 1
35 1
P (E) + P(F) = ...(5)
25 f (b) =
f (0)
Solving (4) and (5) we get
Hence a and b are the correct options.
3 4 13. 3
P(E) = and P(F) =
5 5 The expression may not attain integral value for all a, b, c.
If we consider a = b = c then
4 3
or P(E) = and P(F) = x = 3a, y = a (1 + + 2) = a (1 + i
3)
5 5
(a) and (d) are the correct options. Z = a (1 + 2+ ) = a(1 + i 3 )
11. (a, b, d) |x|2 + |y|2 + |z|2 = 9 |a|2 + 4 |a|2 + 4 |a|2 = 17 |a|2
The equation of normal to y2 = 4x is y = mx – 2m – m3
As it passes through (9, 6) | x |2 | y |2 | z |2 17
2 2 2 =
6 = 9m – 2m – m3 |a| |b| |c| 3
m3 – 7m + 6 = 0
(m – 1) (m2 + m – 6) = 0 i (2 )
Note : However if e 3 , then the value of expression
(m – 1) (m + 3) (m – 2) = 0
can be evaluated as follows
m = 1, 2, –3
Normal is y = x – 3 or y = 2x – 12 or y = – 3x + 33 | x |2 | y |2 | z |2 xx yy zz
2 2 2 2 2
a, b, d are the correct option. |a| |b| |c| |a| |b| | c |2
12. (a, b)

b x ( a b c)( a b c ) (a b c 2 )(a b 2 c )
We have f (x) = , 0< b< 1 2
1 bx = (a b c )(a b c )
2 2
Let f (x1) = f(x2) |a| |b| | c |2

b x1 b x2
= 1 bx 3 | a |2 3 | b |2 3 | c |2 (ab ab bc bc ac ac )(1 2
)
1 bx1 2 =
| a |2 | b |2 | c |2
b – b2 x2 – x1 + bx1x2 = b – x2 – b2x1 + bx1 x2 =3 ( 1+ + 2 = 0)
IIT-JEE 2011 SOLVED PAPER 41
2011-

14. 2 16. 9
We have x4 – 4x3 + 12x2 + x – 1 = 0 a1 b1 c1
x4 – 4x3 + 6x2 – 4x + 1 + 6x2 + 5x – 2 = 0 a2 b2 c2
Let M =
(x – 1)4 + 6x2 + 5x – 2 = 0 a3 b3 c3
(x – 1)4 = – 6x2 – 5x + 2
a1 b1 c1 0 1 b1 1
To solve the above polynomial, it is equivalent to find the
intersection points of the curves y = (x – 1)4 and a2 b2 c2 1 2 b2 2
then =
a3 b3 c3 0 3 b3 3
2
5 1 73
y = – 6x2 – 5x + 2 or y = (x – 1)4 and x = y a1 b1 c1 1 1 a1 b1 1
12 6 24
a2 b2 c2 1 1 a2 b2 1
The graph of above two curves as follows. a3 c3 c3 0 1 a3 b3 1
Clearly they have two points of intersection. a1 = 0, a2 = 3, a3 = 2
Hence the given polynomial has two real roots. a1 b1 c1 1 0
a3 b 3 c3 12
a2 b2 c2 1 0
and = c3 7
y a3 b3 c3 1 12
Sum of diagonal elements = a1 + b2 + c3 = 0 + 2 + 7 = 9
17. 9
We have r b c b 0

1 (r c ) b 0

x r c || b
1 2 Let r c = b or r = c + b
r = iˆ 2 ˆj 3kˆ iˆ ˆj

= (1 )i (2 ) ˆj 3kˆ
r .a = 0 –1+ –3=0
=4
15. 0
r = 3iˆ 6 ˆj 3kˆ
The given equation is
r .b
dy 18. 2
g ( x ) y = g(x) g (x)
dx The smaller region of circle is the region given by
x2 + y2 < 6 ...(1)
g ( x ) dx and 2x – 3y > 1 ...(2)
I.F. = e = eg(x)
Solution is Y
g(x)
y.e = e g ( x ) g ( x ). g ( x ) dx
1
put g (x) = t so that g (x) dx = dt y=
2
x +y =6 2 –3
2x
= et t dt = et (t – 1) + c
y.eg(x) = eg(x) [g(x) – 1] + c
As y (0) = 0 and g (0) = 0 X O X
C=1
y.eg(x) = eg(x) [g(x) – 1] + 1
As g (2) = 0, putting x = 2 we get
y(2).eg(2) = eg(2) [g(2) – 1] + 1
y (2) = 0
Y
EBD_780
42-2011 IIT-JEE 2011 SOLVED PAPER

3 1 1
We observe that only two points 2, and , = n3 =
4 4 4 n3
satisfy both the inequations (1) and (2) C s
2 points in S lie inside the smaller part. For |z| = 1 and z 1
19. A q, B p, C s, D t Let z = ei

As a b c Then 1 – z = 1 – cos – i sin = 2sin


2
2i sin cos
The figure is as shown. 2 2 2

a.b 1 or 1 – z = 2sin sin i cos


Clearly cos (180 – ) = = 2 2 2
| a || b | 2

1 1
= 1 i cot
1 z 2 2

c b 1 1
Here real part of is always
1 z 2

1 1
180°– Locus of is x =
1 z 2
a
1
For which max Arg 1 z
1 2
cos = =
2 3
A q is max. value of i.e. .
2
b
( f ( x) 3x) dx = a2 – b2 y
a

b 3
f ( x) dx [ b2 a 2 ] = a2 – b2
a 2

b 1 2 x
f ( x ) dx = (a b 2 ) O
a 2
x=½
d b
f ( x) dx = b
db a
1
f (b) = b Clearly max. Arg approaches to but will not be
1 z 2
attained.
f =
6 6 D t.
B p 20. A (s), B (t), C (r) D (r)

2 5 2 5 (A) As |z| = 1 and z ± 1


6 n | sec x tan x | 67
n3 7 sec( x )dx =
n3
Let z = ei where n
6 6
2iz 2i (cos i sin )
Then Re = Re
5 5 7 7 1 z2 1 cos 2 i sin 2
= n | sec tan n | sec tan |
n3 6 6 6 6
i (cos i sin )
2 1 2 1 = Re
= n n n sin (cos( ) i sin ( )
3 3 3 3 3 2 2
IIT-JEE 2011 SOLVED PAPER 43
2011-

i 1 8t 9 t 2 8t 9 t 2
= Re cos i sin = Re 0 and 0
sin 2 2 sin 9 t2 9 t2
= Re (–cosec )
t 2 8t 9 t 2 8t 9
as + cosec < – 1 or cosec > 1 0 and 0
2
t 9 t2 9
– cosec > 1 or – cosec < –1
Required set is (– , –1] [1, ) (t 9)(t 1) (t 9)(t 1)
0 and 0
A s (t 3)(t 3) (t 3)(t 3)
Alternative method
Also t = 3x can not be – ve
Let z = x + iy where and x2 + y2 = 1 and x ± 1
t (0,3) [9, ) and t (0,1] [3, )
2iz 2i ( x iy )
Then Re = Re x ( ,1) [2, ) and x ( , 0] (1, )
1 z2 1 ( x2 y2 2ixy )
Combining the two, we get

2 y 2ix 2 y 2ix x ( , 0] [2, )


= Re = Re 2 y ( y ix) B t
1 x2 y2 2ixy
1 tan 1
1 1
= Re = tan 1 tan
y y (C) f ( ) = =
1 tan 1
1 1
where –1< y <| 1 of 1 0 0 2
y y
tan 1 tan
R1 + R3
2iz 1 tan 1
Re ( , 1] [1, )
1 z2

= 2 (1 + tan 2 ) = 2sec2 [2, ) for 0


1 8(3) x 2 2
(B) For the domain of f(x) = sin 2( x 1)
1 3 C r
3/ 2
We should have (D) f(x) = x (3x 10), x 0

8(3) x 2 3
1 1 f (x) = x (3x 10) 3 x x
1 3 2( x 1) 2
For f(x) to be increasing f (x) > 0
x
8.3 3 x [3x 10 2 x] 0
1 1
9 32 x
x (5 x 10) 0
8t
Let 3x = t then 1 1 but x 0 x 2
9 t2
f (x) is incresing on [2, )
8t 8t D r.
1 and 1
9 t2 9 t2
EBD_780
IIT-JEE 2010 PHYSICS
PAPER -1
SECTION – I 3. An AC voltage source of variable angular frequency and
(Single Correct Choice Type) fixed amplitude V0 is connected in series with a capacitance
This section contains 8 multiple choice questions. Each question C and an electric bulb of resistance R (inductance zero).
has 4 choices (a), (b), (c) and (d) for its answer, out of which ONLY When is increased
ONE is correct. (a) the bulb glows dimmer
(b) the bulb glows brighter
1. Incandescent bulbs are designed by keeping in mind that (c) total impedance of the circuit is unchanged
the resistance of their filament increases with the increase (d) total impedance of the circuit increases
in temperature. If at room temperature, 100 W, 60 W and 4. A thin flexible wire of length L is connected to two adjacent
40 W bulbs have filament resistances R100, R60 and R40,
fixed points and carries a current I in the clockwise direction,
respectively, the relation between these resistances is
as shown in the figure. When the system is put in a uniform
1 1 1 magnetic field of strength B going into the plane of the
(a) R R40 R60 (b) R100 = R40 + R60 paper, the wire takes the shape of a circle. The tension in
100
the wire is
1 1 1
(c) R100 > R60 > R40 (d) R100 R60 R40
2. To verify Ohm’s law, a student is provided with a test resistor
RT, a high resistance R1, a small resistance R2, two identical
galvanometers G1 and G2, and a variable voltage source V.
The correct circuit to carry out the experiment is

G1 IBL
(a) IBL (b)
R2
IBL IBL
(c) (d)
(a) RT R1 2 4
5. A block of mass m is on an inclined plane of angle . The
V coefficient of friction between the block and the plane is
and tan > . The block is held stationary by applying a
force P parallel to the plane. The direction of force pointing
up the plane is taken to be positive. As P is varied from
P1 = mg(sin – cos ) to P2 = mg(sin + cos ), the
frictional force f versus P graph will look like
(b)
f f

(a) (b)

(c)
f f

(c) (d)

(d)
6. A thin uniform annular disc (see figure) of mass M has
outer radius 4R and inner radius 3R. The work required to
take a unit mass from point P on its axis to infinity is
EBD_780
2 IIT-JEE 2010 SOLVED PAPER
(a) Internal energies at A and B are the same
(b) Work done by the gas in process AB is P0V0 ln 4
P0
(c) Pressure at C is
4
T0
(d) Temperature at C is
4
2GM 2GM
(a) (4 2 5) (b) (4 2 5) 11. A student uses a simple pendulum of exactly 1m length to
7R 7R
determine g, the acceleration due to gravity. He uses a stop
GM 2GM watch with the least count of 1 sec for this and records 40
(c) (d) ( 2 1)
4R 5R seconds for 20 oscillations. For this observation, which of
7. Consider a thin square sheet of side L and thickness t, made the following statement(s) is (are) true?
of a material of resistivity . The resistance between two (a) Error T in measuring T, the time period, is 0.05 seconds
opposite faces, shown by the shaded areas in the figure is (b) Error T in measuring T, the time period, is 1 second
(c) Percentage error in the determination of g is 5%
(d) Percentage error in the determination of g is 2.5%
12. A few electric field lines for a system of two charges Q1 and
Q2 fixed at two different points on the x-axis are shown in
t the figure. These lines suggest that
L

(a) directly proportional to L


(b) directly proportional to t
Q Q
(c) independent of L
(d) independent of t
8. A real gas behaves like an ideal gas if its
(a) pressure and temperature are both high
(b) pressure and temperature are both low (a) |Q1| > |Q2|
(c) pressure is high and temperature is low (b) |Q1| < |Q2|
(d) pressure is low and temperature is high
(c) at a finite distance to the left of Q1 the electric field is
zero
SECTION – II
(Multiple Correct Choice Type) (d) at a finite distance to the right of Q2 the electric field is
zero
This section contains 5 multiple choice questions. Each
13. A ray OP of monochromatic light is incident on the face AB
question has four choices (a), (b), (c) and (d) out of which
of prism ABCD near vertex B at an incident angle of 60° (see
ONE OR MORE may be correct.
figure). If the refractive index of the material of the prism is
9. A point mass of 1 kg collides elastically with a stationary
point mass of 5 kg. After their collision, the 1 kg mass reverses 3 , which of the following is (are) correct?
its direction and moves with a speed of 2 ms–1. Which of the
following statement(s) is (are) correct for the system of these
two masses?
(a) Total momentum of the system is 3 kg ms–1
(b) Momentum of 5 kg mass after collision is 4 kg ms–1
(c) Kinetic energy of the centre of mass is 0.75 J
(d) Total kinetic energy of the system is 4J
10. One mole of an ideal gas in initial state A undergoes a cyclic
process ABCA, as shown in the figure. Its pressure at A is
P0. Choose the correct option(s) from the following
V (a) The ray gets totally internally reflected at face CD
4V0 B (b) The ray comes out through face AD
(c) The angle between the incident ray and the emergent
V0 A ray is 90°
C
(d) The angle between the incident ray and the emergent
T0 T ray is 120°
IIT-JEE 2010 SOLVED PAPER 3
SECTION –III can be seen easily using dimensional analysis. However,
(Paragraph Type) the motion of a particle can be periodic even when its
This section contains 2 paragraphs. Based upon the first potential energy increases on both sides of x = 0 in a way
paragraph 2 multiple choice questions and based upon the different from kx2 and its total energy is such that the particle
second paragraph 3 multiple choice questions have to be does not escape to infinity. Consider a particle of mass m
answered. Each of these questions has four choices (a), (b), moving on the x-axis. Its potential energy is V(x) = x4 ( >
(c) and (d) out of which ONLY ONE is correct. 0) for |x| near the origin and becomes a constant equal to V0
Paragraph for Questions 14 to 15 for |x| > X0 (see figure).
Electrical resistance of certain materials, known as V(x)
superconductors, changes abruptly from a nonzero value
to zero as their temperature is lowered below a critical
temperature T C (0). An in teresting pr operty of V0
superconductors is that their critical temperature becomes
smaller than TC (0) if they are placed in a magnetic field, i.e.,
the critical temperature TC (B) is a function of the x
magnetic field strength B. The dependence of TC (B) on B is X0
shown in the figure. 16. If the total energy of the particle is E, it will perform
periodic motion only if
TC (B)
(a) E < 0 (b) E > 0
(c) V0 > E > 0 (d) E > V0
TC (0) 17. For periodic motion of small amplitude A, the time period T
of this particle is proportional to
m 1 m
(a) A (b)
A
O B
14. In the graphs below, the resistance R of a superconductor is 1
(c) A (d)
shown as a function of its temperature T for two different m A m
magnetic fields B1 (solid line) and B2 (dashed line). If B2 is 18. The acceleration of this particle for |x| > X0 is
larger than B1 which of the following graphs shows the
V0
correct variation of R with T in these fields? (a) proportional to V0 (b) proportional to mX
0

V0
(c) proportional to (d) zero
mX 0
(a) (b)
SECTION –IV
(Integer Type)
This section contains TEN questions. The answer to each
question is a single digit integer ranging from 0 to 9. The
(c) (d) correct digit below the question number in the ORS is to be
bubbled.
6
15. A superconductor has TC (0) = 100 K. When a magnetic 19. Gravitational acceleration on the surface of a planet is g.
11
field of 7.5 Tesla is applied, its TC decreases to 75 K. For this
material one can definitely say that when where g is the gravitational acceleration on the surface of
(a) B = 5 Tesla, TC (B) = 80 K 2
the earth. The average mass density of the planet is
(b) B = 5 Tesla, 75 K < TC (B) < 100 K 3
(c) B = 10 Tesla, 75K < TC (B) < 100 K times that of the earth. If the escape speed on the surface of
(d) B = 10 Tesla, TC (B) = 70K the earth is taken to be 11 kms–1, the escape speed on the
surface of the planet in kms–1 will be
Paragraph for Questions 16 to 18 20. A piece of ice (heat capacity = 2100 J kg–1 °C–1 and latent heat
When a particle of mass m moves on the x-axis in a potential = 3.36 × 105 J kg–1) of mass m grams is at –5°C at atmospheric
of the form V(x) = kx2 it performs simple harmonic motion. pressure. It is given 420 J of heat so that the ice starts melting.
Finally when the ice-water mixture is in equilibrium, it is found
m
The corresponding time period is proportional to ,as that 1 gm of ice has melted. Assuming there is no other heat
k exchange in the process, the value of m is
EBD_780
4 IIT-JEE 2010 SOLVED PAPER
21. A stationary source is emitting sound at a fixed frequency 25. Two spherical bodies A (radius 6 cm ) and B(radius 18 cm )
f0, which is reflected by two cars approaching the source. are at temperature T1 and T2, respectively. The maximum
The difference between the frequencies of sound reflected intensity in the emission spectrum of A is at 500 nm and in
from the cars is 1.2% of f0. What is the difference in the that of B is at 1500 nm. Considering them to be black bodies,
speeds of the cars (in km per hour) to the nearest integer ? what will be the ratio of the rate of total energy radiated by
The cars are moving at constant speeds much smaller than A to that of B?
the speed of sound which is 330 ms–1. 26. When two progressive waves y1 = 4 sin (2x – 6t) and y2 =
22. The focal length of a thin biconvex lens is 20 cm. When an
object is moved from a distance of 25 cm in front of it to 50 3sin 2 x 6t are superimposed, the amplitude of the
cm, the magnification of its image changes from m25 to m50. 2
resultant wave is
m25
The ratio m is 27. A 0.1 kg mass is suspended from a wire of negligible mass.
50 The length of the wire is 1m and its crosssectional area is 4.9
23. An -particle and a proton are accelerated from rest by a × 10–7 m2. If the mass is pulled a little in the vertically
potential difference of 100 V. After this, their de Broglie downward direction and released, it performs simple
harmonic motion of angular frequency 140 rad s–1. If the
p Young’s modulus of the material of the wire is n × 109 Nm–2,
wavelengths are and p respectively. The ratio , to
the value of n is
the nearest integer, is 28. A binary star consists of two stars A (mass 2.2Ms) and B
24. When two identical batteries of internal resistance 1 each (mass 11Ms), where Ms is the mass of the sun. They are
are connected in series across a resistor R, the rate of heat separated by distance d and are rotating about their centre
produced in R is J1. When the same batteries are connected of mass, which is stationary. The ratio of the total angular
in parallel across R, the rate is J2. If J1 = 2.25 J2 then the momentum of the binary star to the angular momentum of
value of R in is star B about the centre of mass is
IIT-JEE 2010 SOLVED PAPER 5

PAPER -2
(a) 0.02 mm (b) 0.05 mm
SECTION-I
(c) 0.1 mm (d) 0.2 mm
Single Correct Choice Type 5. A biconvex lens of focal length 15 cm is in front of a plane
This section contains 6 multiple choice questions. Each mirror. The distance between the lens and the mirror is 10
cm. A small object is kept at a distance of 30 cm from the
question has four choices (a), (b), (c) and (d) out of which lens. The final image is
ONLY ONE is correct. (a) virtual and at a distance of 16 cm from the mirror
1. A block of mass 2 kg is free to move along the x-axis. It is at (b) real and at a distance of 16 cm from the mirror
rest and from t = 0 onwards it is subjected to a time-dependent (c) virtual and at a distance of 20 cm from the mirror
force F(t) in the x direction. The force F(t) varies with t as (d) real and at a distance of 20 cm from the mirror
shown in the figure. The kinetic energy of the block after 4.5
6. A hollow pipe of length 0.8 m is closed at one end. At its
seconds is
open end a 0.5 m long uniform string is vibrating in its
F(t) second harmonic and it resonates with the fundamental
frequency of the pipe. If the tension in the wire is 50 N and
4N the speed of sound is 320 ms–1, the mass of the string is
(a) 5 grams (b) 10 grams
4.5s (c) 20 grams (d) 40 grams
O 3s t
SECTION –II
(Integer Type)
(a) 4.50 J (b) 7.50 J This section contains 5 questions. The answer to each
(c) 5.06 J (d) 14.06 J question is a single-digit integer, ranging from 0 to 9. The
2. A uniformly charged thin spherical shell of radius R carries correct digit below the question no. in the ORS is to be
uniform surface charge density of per unit area. It is made
bubbled.
of two hemispherical shells, held together by pressing them
with force F (see figure). F is proportional to 7. A large glass slab ( 5 / 3) of thickness 8 cm is placed over
a point source of light on a plane surface. It is seen that light
emerges out of the top surface of the slab from a circular
F F
area of radius R cm. What is the value of R?
8. Image of an object approaching a convex mirror of radius of
curvature 20 m along its optical axis is observed to move
1 2 1 25 50
(a) R2 (b)
2
R from m to m in 30 seconds. What is the speed of the
0 0 3 7
2 2 object in km per hour?
1 1
(c) (d) 9. To determine the half life of a radioactive element, a student
0 R 0 R2
3. A tiny spherical oil drop carrying a net charge q is balanced dN (t ) dN (t )
plots a graph of ln versus t. Here is the rate
in still air will a vertical uniform electric field of dt dt
81 of radioactive decay at time t. If the number of radioactive
strength 105 Vm –1 . When the field is switched off,
7 nuclei of this element decreases by a factor of p after 4.16
the drop is observed to fall with terminal years, the value of p is
velocity 2 10 –3 ms –1. Given g 9.8 m s –2 , viscosity of the
air 1.8 10–5 Ns m–2 and the density of oil = 900
kg m –3 , the magnitude of q is
ln|dN(t)/dt|

(a) 1.6 10–19 C (b) 3.2 10 –19 C


(c) 4.8 10–19 C (d) 8.0 10 –19 C
4. A vernier calipers has 1 mm marks on the main scale. It has 20
equal divisions on the Vernier scale which match with 16 main
scale divisions. For this Vernier calipers, the least count is
EBD_780
6 IIT-JEE 2010 SOLVED PAPER
1 Paragraph for question 15 to 17.
10. A diatomic ideal gas is compressed adiabatically to of The key feature of Bohr’s theory of spectrum of hydrogen
32
atom is the quantization of angular momentum when an
its initial volume. If the initial temperature of the gas is Ti (in
electron is revolving around a proton. We will extend this to
Kelvin) and the final temperature is a Ti , the value of a is a general rotational motion to find quantized rotational
11. At time t = 0, a battery of 10 V is connected across points energy of a diatomic molecule assuming it to be rigid. The
A and B in the given circuit. If the capacitors have no rule to be applied is Bohr’s quantization condition.
charge initially, at what time (in sceonds) does the voltage 15. A diatomic molecule has moment of inertia I. By Bohr’s
across them become 4 V? quantization condition its rotational energy in the nth level
(n = 0 is not allowed) is
[Take : ln5 =1.6, ln3 = 1.1]
1 h2 1 h2
2M 2 F (a) (b)
n2 8 2
I n 8 2I

h2 h2
A B (c) n 2 (d) n2 2
8 I 8 I
2M 2 F 16. It is found that the excitation frequency from ground to the
first excited state of rotation for the CO molecule is close
Section III
4
(Paragraph Type) to 1011 Hz. Then the moment of inertia of CO molecule
Directions : This Section contains 2 paragraphs. Based upon
about its center of mass is close to
each of the paragraphs 3 multiple choice questions have to be
answered. Each of these questions has four choices (a), (b), (c) (Take h 2 10 –34 J s)
and (d) out of which ONLY ONE is correect
Paragraph for questions 12 to 14. (a) 2.76 10 –46 kg m2 (b) 1.87 10 –46 kg m2
When liquid medicine of density is to put in the eye, it is done (c) 4.67 10–47 kg m2 (d) 1.17 10 –47 kg m2
with the help of a dropper. As the bulb on the top of the dropper
17. In a CO molecule, the distance between C (mass = 12 a.m.u.)
is pressed, a drop forms at the opening of the dropper. We wish to
estimate the size of the drop. We first assume that the drop formed 5
and O (mass = 16 a.m.u.), where 1 a.m.u. 10 –27 kg , is
at the opening is spherical because that requires a minimum 3
increase in its surface energy. To determine the size, we calculate close to
the net vertical force due to the surface tension T when the radius
of the drop is R. When this force becomes smaller than the weight (a) 2.4 10 –10 m (b) 1.9 10 –10 m
of the drop, the drop gets detached from the dropper.
(c) 1.3 10 –10 m (d) 4.4 10 –11 m
12. If the radius of the opening of the dropper is r, the vertical
force due to the surface tension on the drop of radius R
(assuming r << R) is Section IV
(a) 2 rT (b) 2 RT Directions : This section contains 2 questions. Each question
has four statements (A, B, C and D) given in Column I and five
2 r 2T 2 R 2T statements (p, q, r, s and t) in Column II. Any given statement in
(c) (d)
R r Column I can have correct matching with one or more statement(s)
given in Column II. For example, if for a given question, statement
13. If r 5 10–4 m, 103 kgm –3 , g 10 ms –2 , T 0.11Nm –1, B matches with the statements given in q and r, then for that
the radius of the drop when it detaches from the dropper is particular question, against statement B, darken the bubbles
approximately corresponding to q and r in the ORS.
(a) 1.4 10 –3 m (b) 3.3 10 –3 m 18. Two transparent media of refractive indices and 3 have
a solid lens shaped transparent material of refractive
(c) 2.0 10 –3 m (d) 4.1 10–3 m
index 2 between them as shown in figures in Column II. A
14. After the drop detaches, its surface energy is
ray traversing these media is also shown in the figures. In
(a) 1.4 10–6 J (b) 2.7 10 –6 J Column I different relationships between 1, 2, and 3 are
(c) 5.4 10 J –6 (d) 8.1 10 J –6
given. Match them to the ray diagrams shown in Column II.
IIT-JEE 2010 SOLVED PAPER 7
Column I Column II

(A) 1 2 (p)
3 2 1

(B) 1 2 (q) 3 2 1

(C) 2 3 (r)
3 2 1

(D) 2 3 (s)
3 2 1

(t)
3 2 1

19. You are given many resistances, capacitors and inductors. These are connected to a variable DC voltage source (the first two
circuits) or an AC voltage source of 50 Hz frequency (the next three circuits) in different ways as shown in Column II. When a
current I (steady state for DC or rms for AC) flows through the circuit, the corresponding voltage V1and V2 , (indicated in
circuits) are related as shown in Column I. Match the two
Column I Column II

V1 V2

6mH 3 F
(A) I 0, V1 is proportional to I (p)

V
EBD_780
8 IIT-JEE 2010 SOLVED PAPER
V1 V2

6mH 2
(B) I 0, V2 V1 (q)

V
V1 V2

6mH 2
(D) V1 0, V2 V (r)

V
V1 V2

6mH 3 F
(D) I 0, V2 is proportional to I (s)

V
V1 V2

1k 3 F
(t)

V
SOLUTIONS
Paper -1
V2
1. (d) We know that P T cos d T cos d
R 2 2
For a given potential difference at a particular temperature d d
2
T Tsin d 2
2 Tsin d T
1
P 2
R I d R I
d
It is given that the powers of the bulbs are in the order 2 2
100W > 60 W > 40W

1 1 1
R100 R60 R40
FB = BI (dl) directed upwards as shown
2. (c) The following points should be considered while making
the circuit : arc(dl )
= BI (Rd ) angle (d )
(i) An ammeter is made by connecting a low resistance R2 radius R
in parallel with the galvanometer G2.
L L
BI d 2 R L R
G2 2 2
Let T be the tension in the wire acting along both ends
of the elemental length as shown. On resolving T, we
R2 d
find that the components. T cos cancel out and
(ii) A voltmeter is made by connecting a high resistance R1 2
in series with the galvanometer G1. d
the components. T sin add up to balance FB.
R1 2
G1
d L
(iii) Voltmeter is connected in parallel with the test resistor At equilibrium 2T sin BI d
2 2
RT.
(iv) Ammeter is connected in series with the test resistor d L d
2T BI d small
RT. 2 2 2
(v) A variable voltage source V is connected in series with BIL
the test resistor RT. T
2
5. (a) As tan > , the block has a tendency to move down
Vrms
3. (b) I rms the incline. Therfore a force P is applied upwards along
1 the incline.
R2
C Here, at equilibrium P + f = mg sin
f = mg sin P
C R N
f
Irms
~
mg sin mg cos
As increases, Irms increases. Therefore the bulb glows
brighter.
mg
4. (c) Let us consider an elemental length dl subtending an
angle d at the centre of the circle. Let FB be the
magnetic force acting on this length. Then
EBD_780
10 IIT-JEE 2010 SOLVED PAPER
Now as P increases, f decreases linearly with respect
2GM
to P. V 4 2 5
When P = mg sin , f = 0. 7R
When P is increased further, the block has a tendency WP
to move upwards along the incline. Now V VP VP V 0
1
N 2GM
WP 4 2 5
7R
P l
mg cos 7. (c) We know that R =
mg sin F a
mg

Therefore the functional force acts downwards along


the incline. L
Here, at equilibrium P = f + mg sin
t
f = P – mg sin L
Now as P increases, f increases linearly w.r.t P.
This is represented by graph (a) .
6. (a) Let us consider a circular elemental area of radius x and Where l is the length of the conductor through which
thickness dx. The area of the shaded portion = 2 xdx. the current flows and a is the area of cross-section.
Let dm be the mass of the shaded portion. Here l = L and a = L × t
L
P R
L t t
R is independent of L
4R
8. (d) A real gas behaves as an ideal gas when the average
distance between the gas molecules is large enough
so that (i) the force of attraction between the gas
x dx molecules becomes almost zero (ii) the actual volume
of the gas molecules is negligible as compared to the
occupied volume of the gas.
The above conditions are true for low pressure and
Mass M dm high temperature.
2
area 4R 2
3R 2 xdx 9. (a, c)
For head-on elastic collision
2M
dm xdx u2 = 0
7R2 u1
The gravitational potential of the mass dm at P is
G dm G 2M 1 kg 5 kg
dV xdx
2 2 16 R 2
x 2 7R 2 Before collision
4R x
(m1 m2 )u1 2m2 u2
2GM xdx v1
1 m1 m2 m1 m2
7 R2 16 R 2 x 2
Suppose 16R2 + x2 = t2 (1 5)u1 4
2 0 2 u1
2xdx = 2tdt 1 5 6
xdx = tdt u1= –2m/s
Also for x = 3R, t = 5R v2
and for x = 4R, t = 4 2R
On integrating equation (1), taking the above limits, we 1 kg 2 kg
get
u1 3m / s
4 2R
2GM 2GM 4 2R 2GM The total momentum centre of masss system
V dt t 4 2 R 5R
SR
7R2 7R2 5R
7R2 (before collision) = 1 × 3 + 5 × 0 = 3 kg m/s
IIT-JEE 2010 SOLVED PAPER 11
Option (a) is correct 11. (a, c) As the length of the string of simple pendulum is
The velocity of centre of mass of the system before exactly l m (given), therefore the error in length l = 0.
collision is Further the possibility of error in measuring time is 1s
in 40s.
m1u1 m2 u2 1 3 0 1
vc m/s t T 1
m1 m2 1 5 2
t T 40
Kinetic energy of the center of mass
40
2 The time period T = 2 seconds
1 2 1 1 20
= mc vc (1 5) 0.75J
2 2 2
T 1 T 1
Option (c) is correct T 0.05sec
T 40 2 40
Now,
l l
v2
(m2 m1 )u 2 2 m1u1 (5 1) 0 2 1 3
1m/s We know that T 2 T2 4 2

m1 m2 m1 m2 1 5 1 5 g g
Linear momentum of 5 kg particle is 5 kg m/s. l
2
Option (b) is wrong. g 4
T2
Total kinetic energy of the system (before collision)
1 1 g l T
1 (3)2 5 (0) 2 = 4.5 J 100 100 2 100
2 2 g l T
Option (d) is wrong. g 1
Alternative 100 0 2 100 = 5
g 40
According to law of conservation of linear momentum
1 × u1 + 5 × 0 = 1 (–2) + 5 (v2) 12. (a, d)
The electric field lines are orginating from Q1 and
u1 = – 2 + 5 v2 ...(i)
terminating on Q2. Therefore Q1 is positive and Q2 is
The coefficient of restituition
negative.
v2 v1 v2 ( 2) As the number of lines associated with Q1 is greater
e 1 than that associated with Q2, therefore |Q1| > |Q2|.
u1 u 2 u1 0
Option (a) is correct.
u1 = v2 + 2 ...(ii) At a finite distance on the left of Q1 , the electric field
On solving (i) & (ii) we get desired results. intensity cannot be zero because the electric field
10. (a, b) created by Q1 will be greater than Q2. This is because
Process A to B the magnitude of Q1 is greater and the distance smaller
As the temperature remains the same, this process is
isothermal. Therefore there is no change in the internal Q
E
energy. Option (a) is correct. r2
V At a finite distance to the right of Q2, the electric field
is zero. Here, the electric field created by Q2 at a
4V0 B particular point will cancel out the electric field created
by Q1.
13. (a,b,c)
Applying Snell’s law at P
V0 A n1 sin 60° = n 2 sin
C
sin 60 3 sin
T0 T = 30°
In quadrilateral BCQP,
P0
Also P0 V0 = PB × 4 V0 PB B
4
Work done 60° C
60°
4V0 P 30°= 135°
W nRT0 loge
V0 i
= P0V0 log e 4 [ P0V0 = nRT0 ] 45° 45°
30°
Option (b) is correct 75°
The process BC is not clear. Therefore no judgement D
can be made for point C. A M
EBD_780
12 IIT-JEE 2010 SOLVED PAPER
60° + (90° + 30°) + 135° + PQC = 360°
dV ( x)
PQC 45 18. (d) F
dx
i = 45° As V (x) = constant for x > X0
The critical angle for prism - air pair of media is F=0 for x > X0
Since F = 0, a = 0
1 1 19. 3.
C = sin which is less than 45°.
3
We know that v 2 gR
Therefore total internal reflection takes place at face
CD. vp gp Rp
option (a) is correct. ...(i)
v g R
In QDM, QMD = 180° – (45° + 75°) = 60°
Therefore the angle of incidence of ray QM on AD is gp 6
30°. Given = ...(ii)
ge 11
This angle is less than the critical angle. Therefore the
ray emerges out of face AD. 4
Also g G R
Option (b) is correct. 3
Applying Snell’s law at M, we get gp p Rp
3 g R
1
3 sin 30 1 sin sin 60
2 2
6 2 Rp p
In quadrilateral PQMN, PNM = 360° – [60° + 90° (given)
11 3 R 3
+120°] = 90°
The angle between the incident ray and the emergent Rp 3 6
ray is 90° ...(iii)
R 22
Option (c) is correct.
14. (a) From the given graph it is clear that with increase of vp 6 3 6 3 6 3
the magnitude of magnetic field (B), the critical From (i), (ii) & (iii)
v 11 22 11 22 11
temperature TC (B) decreases.
Given B2 > B1. Therefore for B2, the temperature at 3 3
vp v 11km/s 3 km/s
which the resistance becomes zero should be less. The 11 11
above statement is true for graph (a). 20. 8.
15. (b) We know that as B increases, TC decreases but the Heat supplied = Heat used in converting m grams of
exact dependence is not known. ice from –5°C to 0°C + Heat used in converting 1 gram
Given at B = 0, TC = 100 K of ice at 0°C to water at 0°C
and at B = 7.5T, TC = 75 K 2100 1 3.36 105
At B = 5T, TC should be between 75 K and 100 K. 420 m 5
1000 1000
16. (c) If the energy is zero, the particle will not perform
oscillations. Therefore E should be greater than zero. 420 m 10.5 336
Further if E = V0, the potential energy will become 84
constant as depicted in the graph given. In this case m 8grams
10.5
also the particle will not oscillate. 21. 7
Therefore E should be greater than zero but less than Let v be the speed of sound and vc the speed of car.
V0. The frequency of sound as received by the listener in the
17. (b) We can get the answer of this question with the help of
dimensional analysis. v vc
car is f1 = f0
Given potential energy = x4 v
The frequency of sound reflected by the car is
Potential energy ML2T 2
[ ML 2T 2
] v v vc v
x4 L4 f1 f1 f0
v vc v v vc
1 m 1 M
Now T
A L ML 2T 2 v vc
f1 f0
v vc
Therefore option (b) is correct.
IIT-JEE 2010 SOLVED PAPER 13
Differentiating the above equation w.r.t. vc ,we get
Cells connected in parallel
d d R
(v vc ) (v vc ) (v vc ) (v vc ) R
d f1 dvc dvc
f0
dvc (v vc ) 2
I
E r

d f1 (v vc ).1 (v vc )( 1) E r
f0 E r 2
dvc (v vc )2
2
v vc v vc
f0 E
(v vc )2 J2 I2R R
r ...(2)
R
2
df1 2v 2v
f0 f0 ( vc v) Given J1 = 2.25 J2
dvc (v vc ) v2
(2 E )2 E2
.R 2.25 .R
df1 2 (2r R) 2 r 2
100 dvc (R )
dvc v 2

2 dvc 4 2.25
0.012 dvc = 0.198 m/s 7 km/h 2 2
330 (2r R) r
R
22. 6 2
Given f = + 20cm
4[ R 0.5]2 2.25[2 R]2 [ r 1 ]
f
Also m = 2 (R + 0.5) = 1.5 (2 + R)
f u
R=4
25. 9
20
m25 20 25 30
6 T2
m50 20 5
20 50 T1
23. 3
r1= 6cm r2= 18cm
h
We know that,
2mqV

1 m q 4 2 A
8 3
mpqp 1 1 B
m1 = 500 nm m2 =1500 nm
24. 4.
Rate of total energy radiated by A
Cells connected in series Rate of total energy radiated by B
R R
4 2
T14 (4 r12 ) T1 r1
=
T24 (4 r22 ) T2 r2
I
4 2
m2 r1 T1 m2
= by Wein 's law
m1 r2 T2 m1
r r
E E 2E 2r
4
1500 6
2 =
2E 500 18
J1 I 2R .R ...(1)
2r R =9
EBD_780
14 IIT-JEE 2010 SOLVED PAPER
26. 5 28. 6
Let the center of mass of the binary star system be at
Resultant amplitude, A A12 A22 2 A1 A2 cos the origin. Then
A B
x dx
2.2 M s c.m 11 M s
4 2 32 2 4 3 cos 16 9 0 5 origin
2
27. 4 2.2M s ( x) 11M s (d x)
0
2.2 M s 11M s
K
We know that ...(i) 0 = 2.2 Ms (– x) + 11Ms (d – x)
m
5d
FL YA x
Here Y F l 6
Al L For a binary star system, angular speed about the
Complaring the above equation with F = kl we get centre of mass is same for both the stars.
YA 2 2
K ...(ii) 5d d
L 2.2 M s 11 M s
LTotal 6 6
2
=6
YA LB d
From (i) & (ii), 11 M s
ml 6

m 109 4.9 10 7
140
0.1 1
n=4
IIT-JEE 2010 SOLVED PAPER 15

Paper -2
1. (c) Area under F – t graph gives the impulse or the change 3
in the linear momentum of the body. As the initial 4 9 vt 2 g
velocity (and therefore the initial linear momentum) of From (i) & (ii) g
3 2 g E
the body is zero, the area under F – t graph gives the
final linear momentum of the body. 3
1 4 9 1.8 10 5 2 10 3 2
900 9.8 7
Area of AOB = 3 4 6 N-s
2 3 2 900 9.8 81 105
F(t)
= 7.8 × 10–19 C
A 4. (d)
4N
20 divisions on the vernier scale = 16 divisions of main
scale
O B 4.5 s
3s C t 16
1 division on the vernier scale = divisions of
20
D main scale

OA CD 16
= 1mm
Also 20
OB CB
= 0.8 mm
4 CD We know that least count = 1MSD – 1VSD
f
3 1.5 = 1 mm – 0.8 mm
CD = 2 = 0.2 mm
1 5. (b) Focal length of the biconvex lens is 15 cm. A small
Area of BCD = 1.5 2 1.5 N-s object is placed at a distance of 30cm from the lens i.e.
2
at a distance of 2f. Therefore the image should form at
The final linear momentum = 6 – 1.5 = 4.5 N-s
30cm from the lens at I1.
p2 (4.5)2
Kinetic energy of the block 5.06 J
2m 2 2
2. (a)
The electrostatic pressure at a point on the surface of
2 O I3 I2 I1
a uniformly charged sphere =
2 0

2 10cm
The force on a hemispherical shell = R2
2 0 But since the ray strike the plane mirror before reaching
3. (d) When the electric field is on I1, the image I1 acts as the virtual object for reflection
Force due to electric field = weight on plane mirror kept at a distance of 20 cm from it.
qE = mg It should produce an image I2 but as the ray encounters
the lens, it gets refracted and the final image is formed
4 3 at I3. For the last refraction from the biconvex lens, u =
qE = R g
3 10 cm.
4 R3 g 1 1 1
q ...(i) Applying lens formula
3E v u f
When the electric field is switched off
Weight = viscous drag force 1 1 1
mg = 6 Rvt v 10 15

4 3 1 1 1 25
R g 6 Rvt v 15 10 150
3
v = 6 cm.
9 vt Therefore a real image is formed at a distance of 16 cm
R ...(ii)
2 g from the plane mirror.
EBD_780
16 IIT-JEE 2010 SOLVED PAPER
6. (b) Frequency of 2nd harmonic of string = Fundamental
frequency produced in the pipe 25
u1 ?, v1 cm, f 10cm
3
1 1 1
v1 u1 f
0.5 m
3 1 1
25 u1 10

1 1 3 5
0.8 m u1 = – 50 m
u1 10 25 250
1 T v Using mirror formula for the second position
2
2l1 4l2 1 1 1 7 1 1 1 1 7
v2 u2 f 50 u2 10 u2 10 50
1 50 320 u2 = – 25m
0.5 4 0.8 Change in position of object = 25 m
= 0.02 kg m–1 25 18 1
The mass of the string = l1 Speed of object = 3 kmh
30 5
= 0.02 × 0.5 kg
9. 8
= 10g
t
7. 6 We know that N N 0e
In the figure, C represents the critical angle
dN
= N 0 e – t (– ) = –N0 e – t
dt
P R Taking log on both sides
dN
log e loge ( N0 ) t
C C dt
8 cm Comparing it with the graph line,
C
C 1 1 AC 1
we get yr
2 BC 2
R O
M
6
1 3
sin C 5
5 A
4
3 B
tan C 3 C
4
2
OM R
In POM, tan C 1
PM 8 2 3 4 5 6 7 8
3
R 8 6cm
4 0.693
8. 3. T1/ 2 0.693 2 1.386 years
Using mirror formula for first position
t 4.16
N 1 T1/ 2 1 1 1.386 1
Now
N0 2 p 2 8
10. 4
For an adiabatic process, the temperature-volume
object Image relationship is
1
1 1 V2
T1V1 T2V2 T1 T2
V1
IIT-JEE 2010 SOLVED PAPER 17
2
V1 4 3 2 r T
Here = 1.4 (for diatomic gas). V2 , T1 Ti , T2 aTi R g
32 3 R
1.4 1
1 3 r 2T 3 (5 10 4 ) 2 0.11
Ti aTi R4 4.12 10 12

32 2 g 2 1000 10

0.4
R = 1.42 × 10–3 m
1 aTi U
Ti aTi 14. (b) We know that, T
25 4 a
a=4 U= T × a
11. 2 = T × 4 R2
The equivalent circuit is shown in the figure. 22
R = 1M , C = 4 F = 0.11 4 (1.42 10 3 ) 2 2.7 10 6 J
7
4 F nh
1M 15. (d) According to Bohr’s quantisation principle L
2
2
1 2 1 L
Rotational kinetic energy = I I
2 2 I
[ L I ]
1 L2 1 n2 h 2 h2
= 2
n2 2 ...(i)
10V 2 I 2I 4 8 I
The time constant = RC = 4 sec 16. (b) Energy given = change in kinetic energy
The potential across 4 F capacitor at any time ‘t’ is h2
given as h Kf Ki 2
[22 12 ] [From (i)]
8 I
t
3h 2
V V0 1 e h 2
8 I
34
3h 3 2 10 3 45
t t t I 10
4 10 1 e4 8 2
2 4 11 16
0.4 1 e 4 e 4 0.6 8 10

= 1.87 × 10–46 kg m2
t 17. (c) Centre of mass divides the distance between the point
2.303log10 0.6
4 masses in inverse ratio of their masses.
t = 2 sec m2 d m1d
12. (c) The vertical force due to surface tension r1 and r2
m1 m2 m1 m2
= (Tcos ) × 2 r
Also the moment of inertia of the system is
r 2 r 2T C
T 2 r r1
R R r2
c.m
d
I m1r12 m2 r22
r
2
5
R 16 10 27 d
46 5 27
3
1.87 10 12 10 5
3 28 10 27
3

2
5
13. (a) When the drop is about to detach from the dropper 12 10 27 d
5 2 3
Weight = vertical force due to surface tension 16 10
3 5
28 10 27
2 r 2T 3
mg =
R d = 1.3 × 10–10 m
EBD_780
18 IIT-JEE 2010 SOLVED PAPER
18. A-p, r; B-q,s,t; C-p,r,t, D-q,s 19. A-r,s,t; B-q,r,s,t; C-p,q; D-q,r,s,t
(a) When 1 < 2, the ray of light while entering the lens The following are the important concepts which are applied
will bend towards the normal. Therefore p, r are the in the given situation.
correct options (i) For DC circuit, in steady state, the current I through
(B) When 1> 2, the ray of light while entering the lens the capacitor is zero. In case of L-C circuit, the potential
will bend away from the normal. Therefore q,s,t are the difference across the inductor is zero and that across
correct options. the capacitor is equal to the applied potential difference.
(C) When 2 = 3, the ray of light while coming out from In case of L-R circuit, the potential difference across
the lens does not deviate from its path. Therefore p,r,t inductor is zero across resistor is equal to the applied
are the correct option. voltage.
(D) 2> 3, the ray of light coming out of the lens deviates (ii) For AC circuit in steady state, Irms current flows through
away from the normal. Therefore q,s are the correct the capacitor, inductor and resistor. The potential
options. difference across resistor, inductor and capacitor is
proportional to I.
(iii) For DC circuit, for changing current, the potential
difference across inductor, capacitor or resistor is
proportional to the current.
IIT-JEE 2010 CHEMISTRY
PAPER -1
Useful Data 3. The correct statement about the following disaccharide is
Atomic numbers: Be = 4; C = 6; N = 7; O = 8; Al = 13; Si = 14;
CH2OH HOH2C H
Cr = 24; Fe = 26; Zn = 30; Br = 35. H O
O H
1 amu = 1.66 × 10–27 kg H (A) (B)
H HO
R = 0.082 L atm K–1 mol–1 OH H OCH2CH2O CH2OH
h = 6.626 × 10–34 Js HO
OH H
NA = 6.022 × 1023 H OH
me = 9.1 × 10–31 kg (a) Ring (A) is pyranose with - glycosidic link
e = 1.6 × 10–19 C (b) Ring (A) is furanose with - glycosidic link
c = 3.0 × 108 ms–1 (c) Ring (B) is furanose with - glycosidic link
F = 96500 C mol–1 (d) Ring (B) is pyranose with - glycosidic link
RH = 2.18 × 10–18 J 4. The synthesis of 3-octyne is achieved by adding a
4 0 = 1.11 × 10–10 J–1 C2 m–1 bromoalkane into a mixture of sodium amide and an alkyne.
SECTION – I The bromoalkane and alkyne respectively are
(Single Correct Choice Type) (a) BrCH2CH2CH2CH2CH3 and CH2CH2C CH
This section contains 8 multiple choice questions. Each question (b) BrCH2CH2CH3 and CH3CH2CH2C CH
has 4 choices (a), (b), (c) and (d) for its answer, out of which ONLY (c) BrCH2CH2CH2CH2CH3 and CH3C CH
ONE is correct. (d) BrCH2CH2CH2CH3 and CH3CH2C CH
1. Plots showing the variation of the rate constant (k) with 5. The ionisation isomer of [Cr(H2O)4Cl(NO2)]Cl is
temperature (T) are given below. The plot that follows (a) [Cr(H2O)4(O2N)]Cl2
Arrhenius equation is (b) [Cr(H2O)4Cl2](NO2)
(c) [Cr(H2O)4Cl(ONO)]Cl
(d) [Cr(H2O)4Cl2(NO2)].H2O
6. The correct structure of ethylenediaminetetraacetic acid
(EDTA) is
(a) (b)
HOOC – H2C CH2– COOH
(a) N – CH = CH – N
HOOC – H2C CH2– COOH
HOOC COOH
(b) N – CH2 – CH2 – N
HOOC COOH
(c) (d) HOOC–H2C CH2–COOH
(c) N – CH2 – CH2 – N
HOOC–H2C CH2–COOH

HBr COOH
2. In the reaction OCH3 the products are
CH2
HOOC–H2C H
N–CH–CH–N
(d) H CH2–COOH
(a) OCH3and H2 CH2
HOOC

Br and CH3Br 7. The bond energy (in kcal mol–1) of a C–C single bond is
(b)
approximately
(a) 1 (b) 10
Br and CH3OH (c) 100 (d) 1000
(c)
8. The species which by definition has ZERO standard
molar enthalpy of formation at 298 K is
OH and CH3Br (a) Br2 (g) (b) Cl2 (g)
(d)
(c) H2O (g) (d) CH4 (g)
EBD_780
20 IIT-JEE 2010 SOLVED PAPER
SECTION – II Paragraph for Question 14 to 16
(Multiple Correct Choice Type) Copper is the most noble of the first row transition metals
This section contains 5 multiple choice questions . Each question and occurs in small deposits in several countries. Ores of
has 4 choices (a), (b), (c) and (d) for its answer, out of which ONE copper include chalcanthite (CuSO4.5H2 O), atacamite
OR MORE is/are correct. (Cu2Cl(OH)3), cuprite (Cu2O), copper glance (Cu2S) and
OH malachite (Cu2(OH)2CO3). However, 80% of the world copper
production comes from the ore chalcopyrite (CuFeS2). The
NaOH(aq)/Br2 extraction of copper from chalcopyrite involves partial
9. In the reaction the intermediate
roasting, removal of iron and self-reduction.
(s) is (are) 14. Partial roasting of chalcopyrite produces
- (a) Cu2S and FeO (b) Cu2O and FeO
O O (c) CuS and Fe2O3 (d) Cu2O and Fe2O3
Br
15. Iron is removed from chalcopyrite as
(a) (b) (a) FeO (b) FeS
Br Br (c) Fe2O3 (d) FeSiO3
Br
16. In self-reduction, the reducing species is
- (a) S (b) O2–
O -
O (c) S 2– (d) SO2

(c) (d)
Paragraph for Question 17 to 18
Br Br
The concentration of potassium ions inside a biological cell
10. Among the following, the intensive property is (properties are) is atleast twenty times higher than the outside. The resulting
(a) molar conductivity (b) electromotive force potential difference across the cell is important in several
(c) resistance (d) heat capacity processes such as transmission of nerve impulses and
11. The reagent(s) used for softening the temporary hardness maintaining the ion balance. A simple model for such a
of water is (are) concentration cell involving a metal M is
(a) Ca3 (PO4)2 (b) Ca(OH)2
(c) Na2CO3 (d) NaOCl M(s) | M+(aq; 0.05 molar) || M+(aq; 1 molar) | M(s)
12. Aqueous solutions of HNO 3 , KOH, CH3 COOH and For the above electrolytic cell the magnitude of the cell
CH3COONa of identical concentrations are provided. The potential |Ecell| = 70 mV.
pair(s) of solutions which form a buffer upon mixing is(are) 17. For the above cell
(a) HNO3 and CH3COOH
(b) KOH and CH3COONa (a) Ecell 0; G 0 (b) Ecell 0; G 0
(c) HNO3 and CH3COONa (c) Ecell 0; G 0 (d) Ecell 0; G 0
(d) CH3COOH and CH3COONa
13. In the Newman projection for 2,2-dimethylbutane 18. If the 0.05 molar solution of M+ is replaced by a 0.0025 molar
M+ solution, then the magnitude of the cell potential would
X
be
H3C CH3
(a) 35mV (b) 70mV
H H (c) 140 mV (d) 700 mV

Y
X and Y can respectively be SECTION – IV
(a) H and H (b) H and C2H5 (Integer Type)
(c) C2H5 and H (d) CH3 and CH3
This section contains TEN questions. The answer to each question
SECTION – III is a single digit integer ranging from 0 to 9. The correct digit
(Paragraph Type ) below the question number in the ORS is to be bubbled.
This section contains 2 paragraphs. Based upon the first 19. In the scheme given below, the total number of intramolecular
paragraph 3 multiple choice questions and based upon the aldol condensation products formed from ‘Y’ is
second paragraph 2 multiple choice questions have to be
answered. Each of these questions has four choices (a), (b), 1. O3 1. NaOH (aq)
Y 2. heat
(c) and (d) out of which ONLY ONE is correct. 2. Zn, H2O
IIT-JEE 2010 SOLVED PAPER 21
20. Amongst the following, the total number of compounds 23. The value of n in the molecular formula BenAl2Si6O18 is
soluble in aqueous NaOH is 24. A student performs a titration with different burettes and
H3C CH3 finds titre values of 25.2 mL, 25.25 mL, and 25.0 mL. The
N COOH OCH2CH3 OH number of significant figures in the average titre value is
CH2OH 25. The concentration of R in the reaction R P was measured
as a function of time and the following data is obtained:
[R] (molar) 1.0 0.75 0.40 0.10
t(min.) 0.0 0.05 0.12 0.18

NO2 OH CH2CH3 COOH The order of reaction is


235
CH2CH3 26. The number of neutrons emitted when 92 U undergoes
142 90
controlled nuclear fission to 54 Xe and 38 Sr is
27. The total number of basic groups in the following form of
N
H3C CH3 lysine is

21. Amongst the following, the total number of compounds


whose aqueous solution turns red litmus paper blue is
KCN K2SO4
(NH4)2C2O4 NaClZn(NO3)2
FeCl3 K2CO3
NH4NO3 LiCN 28. The total number of cyclic isomers possible for a
22. Based on VSEPR theory, the number of 90 degree F–Br–F hydrocarbon with the molecular formula C4H6 is 5.
angles in BrF5 is

PAPER -2

SECTION – I
(Single Correct Choice Type)
This section contains 6 multiple choice questions. Each question (c) H3C NH
has four choices (a), (b), (c) and (d) out of which ONLY ONE is C
correct. O
O
1. In the reaction O
(d) H3C C
O (1) NaOH/Br2 NH C
H3C C T
O
NH2 (2) C
Cl
2. Assuming that Hund’s rule is violated, the bond order
the structure of the product T is :
and magnetic nature of the diatomic molecule B2 is :
O (a) 1 and diamagnetic (b) 0 and dimagnetic
(a) H3C C O
O C (c) 1 and paramagnetic (d) 0 and paramagnetic
3. The compounds P, Q and S

O
COOH OCH3 C
O
(b) NH
C CH3 HO H3C
O P Q S
EBD_780
22 IIT-JEE 2010 SOLVED PAPER
were separately subjected to nitration using HNO3/H2SO4 (a) 39.27% (b) 68.02%
mixture. The major product formed in each case (c) 74.05% (d) 78.54%
respectively, is :
(a)
SECTION-II
O
COOH OCH3
(Integer Type)
C
O
This section contains a group of 5 questions. The answer to
HO H3C O2N each of the questions is a single digit integer, ranging from 0 to
NO2 NO2
9. The correct digit below the question no. in the ORS is to be
bubbled.
(b)
O 7. One mole of an ideal gas is taken from a to b along two
COOH OCH3 C paths denoted by the solid and the dashed lines as shown
O in the graphs below. If the work done along the solid line
HO NO2 H3C NO2 path ws and that along the dotted line path is wd, then the
NO2
integer closest to the ratio wd / ws is :

(c)
NO2
O
4.5
COOH OCH3 C
O 4.0
3.5
HO H3C NO2
3.0
NO2
P 25
(atm) 2.0

(d) 1.5
0.5
O b
COOH OCH3 C 0.0
O NO2 0.0 0.5 1.0 2.0 2.5 3.0 3.5 4.0 4.5 5.0 5.5 6.0
HO H3C NO2
NO2
4. The species having pyramidal shape is :
(a) SO3 (b) BrF3 8. Among the following, the number of elements showing only
(c) SiO32– (d) OSF2 one non-zero oxidation state is :

5. The complex showing a spin-only magnetic moment of O, Cl, F, N, P, Sn, Tl, Na, Ti
2.82 B.M. is : 9. Silver (atomic weight = 108 g mol–1) has a density of 10.5 g
(a) Ni(CO)4 (b) [NiCl4 ]2– cm–3. The number of silver atoms on a surface of area 10–12
(c) Ni(PPh3)4 (d) [Ni(CN)4]2– m2 can be expressed in scientific notation as y × 10x. The
6. The packing efficiency of the two-dimensional square unit value of x is :

cell shown below is : 10. Total number of geometrical isomers for the complex
[RhCl(CO) (PPh3) (NH3)] is

11. The total number of diprotic acids among the following is:

H3PO4 H2SO4 H3PO3 H2CO3 H2S2O7

H3BO3 H3PO2 H2CrO4 H2SO3


L
IIT-JEE 2010 SOLVED PAPER 23
SECTION-III 14. The compound S is :
(Paragraph Type) O
This section contains 2 paragraphs. Based upon each of CH3 O
H3C C
CH C C H
the paragraphs 3 multiple choice questions have to be
(a) H3C CH H (b) H3C
answered. Each of these questions has four choices (a), (b), CH2
CH2
(c) and (d) out of which ONLY ONE is correct. CN
CN
Paragraph for question 12 to 14
CH3 CN CN
Two aliphatic aldehydes P and Q react in the presence of
CH H3C CH
aqueous K2CO3 to give compound R, which upon treatment CH
(c) H3C CH OH (d) C OH
with HCN provides compound S. On acidification and H3C CH
CH2 2
heating, S gives the product shown below.
OH OH
H3C OH
H3C

O Paragraph for questions 15 to 17


O
12. The compounds P and Q respectively are : The hydrogen-like species Li2+ is in a spherically symmetric
CH3 state S1 with one radial node. Upon absorbing light the ion
CH H H3C H undergoes transition to a state S2. The state S2 has one
and
(a) H3C C C radial node and its energy is equal to the ground state energy
O O
of the hydrogen atom.
CH3
H
15. The state S1 is :
CH H H
and
(b) H3C C C (a) 1 s (b) 2s
O O (c) 2p (d) 3s
H3C CH2 H H3C H 16. Energy of the state S1 in units of the hydrogen atom ground
CH C and state energy is :
(c) C
CH3 O (a) 0.75 (b) 1.50
O
H3C H (c) 2.25 (d) 4.50
CH2 H H
CH C and 17. The orbital angular momentum quantum number of the state
(d) C
CH3 S2 is :
O O
(a) 0 (b) 1
13. The compound R is :
(c) 2 (d) 3
O O
H3C H3C C SECTION-IV
C
C C H This section contains 2 questions. Each question has four
H
(a) H3C (b) H3C CH statements (A, B, C and D) given in Column I and five statements
CH2
H3C OH (p, q, r, s and t) in Column II. Any given statement in Column I can
OH
have correct matching with one or more statements given in
CH3 O CH3 O
Column II. For example, if for a given question, statement B
CH C CH C matches with the statements given in q and r, then for that
(c) H3C CH H (d) H3C CH H particular question, against statement B, darken the bubbles
CH2 CH corresponding to q and r in the ORS.
OH H3C OH
EBD_780
24 IIT-JEE 2010 SOLVED PAPER
18. Match the reaction in Column I with appropriate options in Column II.
Column-I Column-II
NaOH/H2O
(A) N2Cl + OH N=N OH (p) Racemic mixture
0ºC

O
OH OH C CH3
H2SO4
(B) H3C — C — C — CH3 H3C C (q) Addition reaction
CH3
H3C CH3 CH3

O OH
1. LiAlH4
C CH
(C) 2. H3O+ (r) Substitution reaction
CH3 CH3

Base S
(D) HS Cl (s) Coupling reaction

(t) Carbocation intermediate


19. All the compounds listed in Column I react with water. Match the result of the respective reactions with the appropriate options
listed in Column II.
Column-I Column-II
(A) (CH3)2SiCl2 (p) Hydrogen halide formation
(B) XeF4 (q) Redox reaction
(C) Cl2 (r) Reacts with glass
(D) VCl5 (s) Polymerization
(t) O2 formation
SOLUTIONS
Paper -1
10. (a, b) Mass independent properties (molar conductivity and
1. (a) As per Arrhenius equation (k Ae Ea / RT ) , the rate
electromotive force) are intensive properties. Resistance
constant increases exponentially with temperature. and heat capacity are mass dependent, hence extensive
2. (d) properties.
11. (b,c,d) Temporary hardness is due to bicarbonates of calcium
H+ + Br – and magnesium. Temporary hardness can be removed
OCH3 O–CH3
by Clark’s process, which involves the addition of
H slaked lime, Ca(OH)2 . Washing soda (Na 2 CO 3 )
removes both the temporary and permanent hardness
OH + CH3Br by converting soluble calcium and magnesium
compounds into insoluble carbonates.
3. (a) Ca(HCO3) + Ca (OH)2 2CaCO3 + 2H2O
4. (d) Only (d) can form 3-Octyne Ca(HCO3) + Na2CO3 CaCO3 + 2NaHCO3
NaNH 2
CH 3 CH 2 C CH NH3
CH 3CH 2 C C Na 2OCl– + 2H2O 2HOCl + 2OH–
CH 3CH 2 CH 2CH 2 Br Ca (HCO3)2 + 2OH– CaCO3 + CO32 + 2H2O
(S N 2)
12. (c,d) Any solution of a weak acid and its salt with strong
CH3CH 2C CCH 2 CH 2CH 2 CH3 NaBr base acts as an acidic buffer solution.
5. (b) Ionisation isomer of [Cr(H2 O) 4 Cl(NO 2 )]Cl is If volume of HNO3 solution added is less as compared to
[Cr(H2O)4Cl2]NO2. that of CH3COONa solution, it results in the formation of an
6. (c) The correct structure of EDTA is acidic buffer solution.
CH3COONa + HNO3 CH3COOH + NaNO3
HOOC–H2C CH2–COOH
N – CH2 – CH2 – N Excess limiting
HOOC–H2C CH2–COOH reagent
MV MV – –
348
7. (c) C – C bond energy = 348 kJ/mol = kcal/mol M (V – V ) 0 MV MV (V < V)
4.2 13. (b, d) Structural formula of 2, 2-dimethylbutane is
= 82.85 kcal/mol 100 kcal/mol.
CH3
8. (b) The species in its elemental form has zero standard
1 2 3 4
molar enthalpy of formation at 298 K. At 298K, Cl2 is CH3–C–CH2–CH3
gas while Br2 is liquid.
9. (a, c) CH3
(I) Newman projection using C1–C2 bond
- O
- O
-
OH O
Br X
NaoH Br2 Br 2 2
H3C CH3
–H2O –Br – –Br – 1

Br Br H H
(I) (II) (III) C2H5
(II) Newman projection using C3–C2 bond
O
-
Br CH3
Br 2
Br CH3 2 CH3
–Br – 3

H H
Br
(IV) CH3
Product of reaction of phenol with NaOH/Br2 is sodium 14. (b) 2CuFeS2 + O2 Cu2S + SO2 + 2FeS
salt of 2,4,6-tribromophenol. Hence, species (I), (II), 2FeS + 3O2 2FeO + 2SO2
(III) are formed as intermediate. 2Cu2S + 3O2 2Cu2O + 2SO2
EBD_780
26 IIT-JEE 2010 SOLVED PAPER
15. (d) FeO + SiO2 FeSiO3 22. 0.
16. (b) Cu2S + 2Cu2O 6Cu + SO2 According to VSEPR theory, number of electron pairs
The reducing species is the one which gets oxidised. around central atom (Br) are 6.
So, it is S2– ion getting oxidised to S4+. N 7 5
6. (Five are bond pairs and one is lone
17. (b) M(s) M (aq)1M M (aq).05M M(s) 2 2
pair )
According to Nernst equation,
Its geometry is octahedral but due to lone pair –bond
2.303RT M .05M pair repulsion, the four fluorine atoms at corner are
E cell 0 log forced towards the upper fluorine atom thus reducing
F M1M
F–Br–F angle from 90° to 84.8°.
2.303 RT F
0 log(5 10 2 )
F F F
= + ve Br
Hence, |Ecell| = Ecell = 0.70 V and G < 0 for the
feasibility of the reaction. F F

2.303RT 23. 3.
18. (c) From above equation 0.0538
F Total cationic charge = Total anionic charge
2n + 6 + 24 = 36
°
So, Ecell = Ecell 0.0538 n=3
log 0.0025
1 24. 3.
The least significant figure in titre values is 3.
0.0538
0 log 0.0025
1 25.2 25.25 25 75.4
Average titre value = 25.1
0.13988 V 3 3
140 mV The number of significant figures in average titre value will
19. 1 also be 3.
O 25. 0.
1. O 3 NaOH(aq) The integrated form of a zero-order reaction is
2. Zn, H 2O [A0] – [At] = k0t
1.0 – 0.75 = k0 × 0.05, k0 = 5
O
Again, 1.0 – 0.4 = k0 × 0.12, k0 = 5
O OH 26. 3.
235
heat
92 U Xe142 38 Sr90 y0 n1
54
–H2O
235 = 142 + 90 + y
(–) O O O y = 3.
The number of intra molecular aldol condensation The number of neutrons emitted are 3.
products ( , –unsaturated carbonyl compound 27. 2.
formed from Y is 1. The basic groups in the given form of lysine is NH2
20. 4.
All carboxylic acids and phenols are soluble in aqueous (not N H 3 ) and CO2 .
NaOH. Four compounds are soluble in aqueous NaOH. 28. 5.
21. 3. The number of cyclic isomers for a hydrocarbon with
KCN, K2CO3 and LiCN are the salts of weak acid and molecular formula C4H6 is 5.
strong base. So, their aqueous solutions turns red litmus The structures are
paper blue. CH3 CH2
and
CH3
IIT-JEE 2010 SOLVED PAPER 27

Paper -2
1. (c)

:
O
(1) NaOH/Br2
Me NH2
H3C C Hofmann S (Shape is trigonal pyramidal)
NH2 Bromamide
reaction O F
F
O
C O The shapes of SO3, BrF3 and SiO32 are triangular
Cl
Me NH — C —
– HCl planar respectively.
2. (a) Molecular orbital configuration of B2(10) as per the 5. (b) [NiCl4]2–, O.S. of Ni = +2
condition will be Ni(28) = 3d8 4s2
1s2, * 1s2, 2s2, * 2s2, 2 p 2y 3d 4s 4p
6 4
Bond order of B2 = 1.
2
B2 will be diamagnetic. –
Cl being weak ligand it cannot pair up the two
electrons present in 3d orbital
CO2H CO2H
3d 4s 4p
HNO3/H2SO4
3. (c)

NO2
OH OH 3
(P) sp

No. of unpaired electrons = 2


(OH group is activing)
Magnetic moment, = 2.82 BM.
OCH3 OCH3
NO2 Area occupied by effective circles
6. (d) Packing effieincy =
HNO3/H2SO4 Area of square

2 r2 2 r2
= 100 100 = 100 78.54%
CH3 CH3 L2 2( 2 r )2 4
(Q)
7. 2
(OCH3 group is more activating)
3 2 5 5
wd = 4 ( 1 1) 6 1
O 2 3 2 3
HNO 3/H2SO4
O 26
wd = – L atm
3

O 5.5
ws = – 2.303 RT log 2.303 PV log 11
1/ 2
O NO2 ws = – 4.606 × 1.04 = – 4.8 L atm
26
wd 3
(Benzene ring having –O– is activated)
ws = 4.8
= 1.80 2.0
N 6 2
4. (d) OSF2 : 4. 8. 2
2 2
Fluorine generally shows 0 and –1 oxidation states while
It has 1 lone pair.
sodium shows 0 and + 1 oxidation state.
EBD_780
28 IIT-JEE 2010 SOLVED PAPER
9. 7

mass H3C OH H3C


OH
d= 10.5 g/cc means in 1 cc
V H3C
H3C
– H 2O O
10.5 g of Ag is present. O O
HO OH

H+/H2O
Heat
10.5 Unstable
Number of atoms of Ag in 1 cc NA
108
H3C H3C
O OH
10.5 HCN
In 1 cm, number of atoms of Ag = 3 NA H3C H3C
108
H C N
OH

aq. K2CO3
HO
2/3
10.5 (R) (S)
In 1 cm2, number of atoms of Ag = NA
108
H3C O H
In 10–12 m2 or 10–8 cm2, number of atoms of Ag + H
H3C H O
2/3 2/3
10.5 8 10.5 6.022 1023 8
= NA 10 10 (P & Q)
108 108

= 1. 5× 107 12. (b) 13. (a) 14. (d)


(15-17) The spherically symmetric state S1 of Li2+ with one
Hence x = 7 radial node is 2s. Upon absorbing light, the ion gets
10. 3 excited to state S2, which also has one radial node. The
energy of electron in S2 is same as that of H-atom in its
PPh3 PPh3 CO
ground state.
Cl Cl Cl
Z2
En = E1 where E1 is the energy of H-atom in the
Rh Rh Rh n2

(3)2 E1
CO NH3 NH3 CO NH3 PPh3 ground state = for Li2+
n2
The number of geometrical isomers is 3. En = E1 n=3
State S2 of Li2+ having one radial node is 3p.
11. 6 Orbital angular momentum quantum number of 3p is 1.
Diprotic acids are H2SO4, H3PO3, H2CO3, H2S2O7, (3) 2
Energy of state S1 E1 2.25 E1
H2CrO4 and H2SO3. (2) 2
15. (b) 16. (c) 17. (b)
18. (A) – (r), (s) ; (B) – (t) ; (C) – (p), (q) ; (D) – (r)
(12- 14) 19. (A) – (p), (s)
Let us summarize the given facts of the problem. (CH3)2SiCl2 + 2H2O (CH 3 ) 2 Si(OH) 2
2HCl
aq.K 2CO3 HCN Polymerisation
P Q R S [(CH3)2Si–O]n.
(2aldehydes)
(B) – (p), (q), (r), (t)
XeF4 + H2O Xe + XeO3 + H2F2 + O2
H3C OH
SiO2 + 4HF SiF4 + 2H2O
H / H 2O
H3C glass
Heat
O SiF4 + 2HF H2[SiF6]
O
Soluble hexathorosilicic(IV) acid
Structures of P, Q, R and S can be established on going (C) – (p), (q) Cl2 + H2O HCl + HOCl
(D) – (p), (q)
backward from the known final product. VCl5 + 7H2O [V(H2O)6]3+ + 3Cl– + HCl + HOCl
IIT-JEE 2010 MATHEMATICS
PAPER -1
SECTION – I
(b) ( p3 q) x2 ( p3 2q) x ( p3 q) 0
(Single Correct Choice Type)
This section contains 8 multiple choice questions. Each question (c) ( p3 q) x2 (5 p3 2q) x ( p3 q) 0
has 4 choices (a), (b), (c) and (d) for its answer, out of which ONLY
ONE is correct. (d) ( p3 q) x2 (5 p3 2q) x ( p3 q) 0
6. Let f, g and h be real-valued functions defined on the interval
1. Let be a complex cube root of unity with 1. A fair die
2
is thrown three times. If r1, r2 and r3 are the numbers obtained x2 2
x2
[0, 1] by f ( x) ex e , g ( x) xe x e and
r1 r2 r3
on the die, then the probability that 0 is 2
x2
h( x ) x 2e x e . If a, b and c denote, respectively, the
1 1
(a) (b) absolute maximum of f, g and h on [0, 1], then
18 9
(a) a = b and c b (b) a = c and a b
2 1 (c) a b and c b (d) a = b = c
(c) (d) 7. If the angles A, B and C of a triangle are in an arithmetic
9 36
2. Let P, Q, R and S be the points on the plane with position progression and if a, b and c denote the lengths of the sides
opposite to A, B and C respectively, then the value of the
vectors 2iˆ ˆj , 4iˆ,3iˆ 3 ˆj and 3iˆ 2 ˆj respectively. The
a c
quadrilateral PQRS must be a expression sin 2C sin 2 A is
c a
(a) parallelogram, which is neither a rhombus nor a rectangle
(b) square 1 3
(c) rectangle, but not a square (a) (b)
2 2
(d) rhombus, but not a square
3. The number of 3 × 3 matrices A whose entries are either 0 or (c) 1 (d) 3
8. Equation of the plane containing the straight line
x 1
1 and for which the system A y 0 has exactly two x y z
and perpendicular to the plane containing the
2 3 4
z 0
x y z x y z
distinct solutions, is straight lines and is
(a) 0 (b) 29–1 3 4 2 4 2 3
(c) 168 (d) 2 (a) x + 2y – 2z = 0 (b) 3x + 2y – 2z = 0
(c) x – 2y + z = 0 (d) 5x + 2y – 4z = 0
x
1 t ln (1 t )
4. The value of xlim0 3 dt is SECTION - II
x 0 t4 4
(Multiple Correct Choice Type)
1 This section contains 5 multiple choice questions. Each question
(a) 0 (b) has four choices (a), (b), (c) and (d) out of which ONE OR MORE
12
may be correct.
1 1
(c) (d) 9. Let z1 and z2 be two distinct complex numbers and let z =
24 64 (1 – t) z1 + tz2 for some real number t with 0 < t < 1. If Arg (w)
5. Let p and q be real numbers such that p 0, p3 q and denotes the principal argument of a non-zero complex number
w, then
p3 q . If and are nonzero complex numbers satisfying (a) | z z1 | | z z2 | | z1 z2 |
+ = – p and 3+ 3 = q, then a quadratic equation having
(b) Arg (z – z1) = Arg (z – z2)

and as its roots is z z1 z z1


(c) z2 z1 z2 z1
(a) ( p3 q) x 2 ( p3 2q) x ( p3 q) 0 (d) Arg (z – z1) = Arg (z2 – z1)
EBD_780
30 MATHEMATICS
1
x 4 (1 x) 4 (a) 2 x 5 y 20 0 (b) 2x 5y 4 0
10. The value(s) of dx is (are)
2 (c) 3x 4 y 8 0 (d) 4x – 3y + 4 = 0
0 1 x
15. Equation of the circle with AB as its diameter is
22 2 (a) x2 + y2 – 12x + 24 = 0 (b) x2 + y2 + 12x + 24 = 0
(a) (b)
7 105 (c) x2 + y2 + 24x – 12 = 0 (d) x2 + y2 – 24x – 12 = 0
71 3
(c) 0 (d) Paragraph for Questions 16 to 18
15 2
Let p be an odd prime number and Tp be the following set
of 2 × 2 matrices :
11. Let ABC be a triangle such that ACB
and let a, b and
6
a b
c denote the lengths of the sides opposite to A, B and C TP A : a, b, c {0,1, 2,..., p 1}
respectively. The value(s) of x for which a = x2 + x + 1, c a
b = x2 – 1 and c = 2x + 1 is (are)
16. The number of A in Tp such that A is either symmetric or
(a) (2 3) (b) 1 3 skew-symmetric or both, and det(A)
divisible by p is
(c) 2 3 (d) 4 3
(a) (p – 1)2 (b) 2 (p – 1)
12. Let A and B be two distinct points on the parabola y2 = 4x. If (c) (p – 1)2 + 1 (d) 2p – 1
the axis of the parabola touches a circle of radius r having 17. The number of A in Tp such that the trace of A is not divisible
AB as its diameter, then the slope of the line joining A and B by p but det (A) is divisible by p is
can be [Note: The trace of a matrix is the sum of its diagonal
1 1 entries.]
(a) (b)
r r (a) (p – 1) (p2 – p + 1) (b) p3 – (p – 1)2
(c) (p – 1) 2 (d) (p – 1) (p2 – 2)
2 2
(c) (d) 18. The number of A in Tp such that det (A) is not divisible by p
r r is
13. Let f be a real-valued function defined on the interval (0, ) (a) 2p2 (b) p3 – 5p
3
(c) p – 3p (d) p3 – p2
x
by f ( x ) ln x 1 sin t dt. Then which of the following
SECTION - IV
0
(Integer Type)
statement(s) is (are) true?
This section contains TEN questions. The answer to each
(a) f (x) exists for all x (0, ) question is a single digit integer ranging from 0 to 9. The
(b) f (x) exists for all x (0, ) and f is continuous on correct digit below the question number in the ORS is to be
bubbled.
(0, ) , but not differentiable on (0, )
(c) there exists > 1 such that | f ( x ) | | f ( x) | for all 19. Let Sk, k = 1, 2, ….. , 100, denote the sum of the infinite
x ( , ) k 1
geometric series whose first term is and the common
k!
(d) there exists > 0 such that | f ( x) | | f ( x) | for all
100
x (0, )
1 1002 | (k 2 3k 1) S k |
ratio is . Then the value of +
k 100! k 1
SECTION - III
(Paragraph Type) is
20. The number of all possible values of where 0 < < , for
This section contains 2 paragraphs. Based upon the first which the system of equations
paragraph 2 multiple choice questions and based upon the (y + z) cos 3 = (xyz) sin 3
second paragraph 3 multiple choice questions have to be
2cos 3 2sin 3
answered. Each of these questions has four choices (a), (b), x sin 3
(c) and (d) out of which ONLY ONE is correct. y z
Paragraph for Questions 14 & 15 (xyz) sin 3 = (y + 2z) cos 3 + y sin 3
have a solution (x0, y0, z0) with y0 z0 0, is
x2 y2 21. Let f be a real-valued differentiable function on R (the set of
The circle x2 + y2 – 8x = 0 and hyperbola 1
9 4 all real numbers) such that f(1) = 1. If the y-intercept of the
intersect at the points A and B. tangent at any point P(x, y) on the curve y = f(x) is equal to
14. Equation of a common tangent with positive slope to the the cube of the abscissa of P, then the value of f(–3) is equal
circle as well as to the hyperbola is to
IIT-JEE 2010 SOLVED PAPER 31
27. For any real number x, let [x] denote the largest integer less
22. The number of values of in the interval, , such than or equal to x. Let f be a real valued function defined on
2 2 the interval [–10, 10] by
n
that for n = 0, 1, 2 and tan = cot 5 as well as x [ x] if [ x]is odd,
5 f ( x)
sin 2 = cos 4 is 1 [ x] x if [ x ]is even
23. The maximum value of the expression
2 10
1 Then the value of f ( x) cos x dx is
2 2
is 10
sin 3sin cos 5cos 10

iˆ 2 ˆj 2 2
24. If a and b are vectors in space given by a 28. Let i sin
be the complex number cos . Then the
5 3 3
number of distinct complex n umbers z satisfying
2iˆ ˆj 3kˆ
and b , then the value of 2a b . 2
14 z 1
z 2
1 0 is equal to
a b a 2b is
2
1 z
2 2
x y
25. The line 2x + y = 1 is tangent to the hyperbola 2 1.
a b2
If this line passes through the point of intersection of the
nearest directrix and the x-axis, then the eccentricity of the
hyperbola is
26. If the distance between the plane Ax – 2y + z = d and the
x 1 y 2 z 3
plane containing the lines and
2 3 4
x 2 y 3 z 4
is 6 , then |d| is
3 4 5
EBD_780
32 MATHEMATICS

PAPER -2
SECTION - I 5. Let f be a real-valued function defined on the interval (–1,
(Single Correct Choice Type) x
This section contains 6 multiple choice questions. Each 1) such that e–x f (x)= 2 t 4 1 dt, for all x –1,1) , and
question has four choices (a), (b), (c) and (d) out of which 0
ONLY ONE is correct. let f –1 be the inverse function of f. Then (f –1) (2) is equal to

1. If the distance of the point P (1, –2, 1) from the plane x + 2y 1


(a) 1 (b)
–2z = , where > 0, is 5, then the foot of the perpendicular 3
from P to the plane is 1 1
(c) (d)
8 4 7 4 4 1 2 e
(a) , , (b) , ,
3 3 3 3 3 3 6. Let S={1, 2, 3, 4} . The total number of unordered pairs of
disjoint subsets of S is equal to
1 2 10 2 1 5 (a) 25 (b) 34
(c) , , (d) , ,
3 3 3 3 3 2 (c) 42 (d) 41

4 SECTION - II
2. A signal which can be green or red with probability and
5 (Integer Type)
1 Directions : This section contains 5 questions. The answer to
respectively, is received by station A and then transmitted each question is a single-digit integer, ranging from 0 to 9. The
5
to station B. The probability of each station receiving the correct digit below the question no. in the ORS is to be bubbled.

3 7. Let a1,a2,a3........, a11 be real numbers satisfying


signal correctly is . If the signal received at station B is a1=15, 27–2a2> 0 and ak=2ak–1–ak–2 for k = 3, 4,..........11.
4
green, then the probability that the original signal was green a12 a22 ..... a11 2
if 90 , then the value of
is 11
3 6 a1 a2 ..... a11
(a) (b) is equal to
5 7 11
8. Let f be a function defined on R (the set of all real numbers)
20 9 such that f (x)=2010 (x–2009) (x–2010)2 (x–2011)3 (x–2012)4
(c) (d)
23 20 .for all x R.

3. Two adjacent sides of a parallelogram ABCD are given by If g is a function defined on R with values in the interval
( 0, ) such that
AB 2iˆ 10 ˆj 11kˆ and AD iˆ 2 ˆj 2kˆ
f (x)=ln ( g (x)), for all x R
The side AD is rotated by an acute angle in the plane of then the number of points in R at which g has a local maximum
the parallelogram so that AD becomes AD . If AD makes a is
right angle with the side AB, then the cosine of the angle 9. Let k be a positive real number and let
is given by
2k –1 2 k 2 k 0 2k –1 k
8 17
(a) (b) A 2 k 1 – 2k and B 1– 2k 0 2 k
9 9
–2 k 2k –1 – k –2 k 0
1 4 5
(c) (d) If det (adj A) + det (adj B) = 106. then [k] is equal to
9 9
[Note : adj M denotes the adjoint of square matrix M and [k]
4. For r = 0, 1, …, 10, let Ar, Brand Cr denote, respectively,
denotes the largest integer less than or equal k.
the coefficient of xr in the expansions of (1 + x)10 ,
10. Two parallel chords of a circle of radius 2 are at a distance
10
3 1 apart . If the chords subtend at the center , angles of
(1 + x)20 and (1 + x)30. Then Ar(B10Br – C10Ar) is equal to
r 1 2
and , where k > 0, then the value of [k] is
(a) B10 – C10 (b) A10(B210C10A10) k k
(c) 0 (d) C10–B10 [Note : [k] denotes the largest integer less than or equal to k ]
IIT-JEE 2010 SOLVED PAPER 33
11 Consider a triangle ABC and let a , b and c denote the lengths
of the sides opposite to vertices A, B and C 1 1
(b) decreasing in –t , – and increasing in – , t
respectively. Suppose a=6, b=10 and the area of the triangle 4 4
is 15 3 , if CB is obtuse and if r denotes the radius of (c) increasing in ( –t, t)
the incircle of the triangle, then r2 is equal to (d) decreasing in ( –t, t)
SECTION – III
(Paragraph Type) Paragraph for questions 15 to 12.
Tangents are drawn from the point P(3, 4) to the ellipse
This section contains 2 paragraphs. Based upon each of
the paragraphs 3 multiple choice questions have to be x2 y 2
answered. Each of these questions has four choices (a), (b), 1 touching the ellipse at points A and B.
9 4
(c) and (d) out of which ONLY ONE is correct.
15 The coordinates of A and B are
Paragraph for questions 12 to 14.
Consider the polynomial (a) (3,0 ) and ( 0,2 )
f (x) = 1 + 2x + 3x2 + 4x3.
Let s be the sum of all distinct real roots of f (x) and let t = |s|. 8 2 161 9 8
12 The real numbers lies in the interval (b) – 5 , 15 and – ,
5 5

1 3 8 2 161 9 8
(a) – ,0 (b) 11, (c) – , and(0, 2) (d) (3,0) and – ,
4 4 5 15 5 5
3 1 1 16 The orthocenter of the triangle PAB is
(c) – ,– (d) 0,
4 2 4
8 7 25
13 The area bounded by the curve y = f (x) and the lines x = 0, (a) 5, (b) ,
7 5 8
y = 0 and x = t, lies in the interval
11 8 8 7
(c) , (d) ,
3 21 11 5 5 25 5
(a) ,3 (b) ,
4 64 16 17. The equation of the locus of the point whose distances
21 from the point P and the line AB are equal, is
(c) (9, 10) (d) 0, (a) 9x2 + y2 –6xy –54x –62y + 241 = 0
64
(b) x2 + 9y2 +6xy –54x +62y –241 = 0
14 The function f (x) is
(c) 9x2 +9y2 –6xy –54x –62y–241 = 0
1 1 (d) x2 + y2 – 2xy + 27x + 31y – 120 = 0
(a) increasing in –t , – and decreasing in – , t
4 4
SECTION – IV
This section contains 2 questions. Each question has four statements (A, B, C and D) given in Column I and five statements (p,
q, r, s and t) in Column II. Any given statement in Column I can have correct matching with one or more statement(s) given in
Column II. For example, if for a given question, statement B matches with the statements given in q and r, then for that
particular question, against statement B, darken the bubbles corresponding to q and r in the ORS.
18. Match the statements in Column-1 with those in Column -II
[Note : Here z takes values in the complex plane and Im z and Re z denote , respectively, the imaginary part and the real part of z.]
Column-I Column-II
4
(A) The set of points z satisfying (p) an ellipse with eccentricity
5
|z – i| z | | = |z + i | z || is contained in or equal to (q) the set of points z
satisfying Im z = 0
(B) The set of points z satisfying (r) the set of points z satisfying |Im z | < 1
|z + 4 | + |z – 4 | = 10 is contained in or equal to
(C) If | w | = 2, then the set of points (s) the set of points z satisfying | Re z | < 2
1
z = w + is contained in or equal to
w
(D) If | w | = 1, then the set of points (t) the set of points z satisfying | z | < 3
1
z= w– is contained in or equal to.
w
EBD_780
34 MATHEMATICS
19. Match the statement in Column-1 with the values in Column -II
Column – I Column – II
(A) A line from the origin meets the lines (p) – 4
8
x 2 y 1 z 1 x
and 3 y 3 z 1
1 2 1 2 1 1
at P and Q respectively. If length PQ = d, then d2 is

(B) The values of x satisfying


1 3
tan–1(x + 3) – tan –1(x – 3) = sin are (q) 0
5
(C) Non-zero vectors a, b and c satisfy a.b = 0.

(b a ).(b c ) 0 and 2 | b c | | b a | .
If a b 4c , then the possible values of are (r) 4
(D) Let f be the function on [– , ] given by f (0) = 9
9x x
and f(x) = sin sin for x 0 (s) 5
2 2

2
The value of f ( x )dx is

(t) 6
SOLUTIONS
IIT-JEE 2010 SOLVED PAPER 35

Paper -1
1. (c) If is a complex cube root of unity then, we 3 3
5. (b) Given that –p and q
know that
3m 3n +1
=0 3 p +2 ( )3 – 3 ( ) q
Where m, n, p are integers.
– p3 – 3 (– p) q
r1 , r2 ,r3 should be of the form 3m, 3n + 1 and 3p + 2 taken
in any order. As r1 , r2 , r3 are the numbers obtained on p3 q
die, these can take any value from 1 to 6. 3p
m can take values 1 or 2 Now for required quadratic equation,
n can take values 0 or 1 2 2
p can take values 0 or 1 sum of roots
Number of ways of selecting r1, r2, r3
2
C1 2C1 2C1 3!. p3 q
Also the total number of ways of getting r1, r2, r3 on die p2 – 2
( 2
) –2 3p
=6× 6×6
p3 q
2 2
C1 C1 2 C1 3! 2 3p
Required probability
6 6 6 9
2. (a) We have PQ 6iˆ j , QR – i 3 j , SR 6i j, 3 p3 – 2 p3 – 2q p3 – 2q
p3 q p3 q
PS i 3j

PQ SR; QR PS and PQ. PS 0 and product of roots . 1

Also PQ QR Required equation is


PQRS is a parallelogram but neither a rhombus nor a p3 – 2q
rectangle. x2 – x 1 0
p3 q
a1 b1 c1
3. (a) Let A a2 b2 c2 where ai , bi , ci have or ( p3 q) x2 – ( p3 – 2q) x ( p3 q) 0
a3 b3 c3 values 0 or 1 for i 1, 2, 3. 2 2 2 2
6. (d) f ( x) ex e– x f '( x) 2 x e x – e– x 0,
Then the given system is equivalent to
a1 x b1 y c1 z 1 x 0,1
a2 x b2 y c2 z 0 f (x) is an increasing function on 0,1
a3 x b3 y c3 z 0
1
Which represent three distinct planes. But three planes Hence f max = f(1) = e+ = a
e
can not intersect at two distinct points, therefore no
such system exists. 2 2
g(x) = xe x + e –x
t ln(1 t )
1 x 0
4. (b) lim dt form 2 2
x x 0 t4 4
0 3
0 g '( x ) (2 x 2 1)e x – 2 xe – x 0, x 0,1
Applying L’ Hospital’s rule, we get g (x) is an increasing function on 0,1

xln(1 x) 1
4
g max g (1) e b
e
lim x 2 4 lim
ln (1 x )
. 4
1
x 0 3x x 0 x 3( x 4) 2 2
h( x ) x2ex e– x
1 1 2 2
1. h '( x ) 2 x e x (1 x 2 ) – e – x 0, x 0,1
12 12
EBD_780
36 MATHEMATICS

h (x) is an increasing function on 0,1 z – z1


arg 0
1 z 2 – z1
hmax h (1) e c
e
z – z1 z – z1
Hence a = b = c. Also t t
7. (d) A, B, C are in A.P.. z 2 – z1 z2 – z1
B = 60°
z – z1 z – z1
a c z 2 – z1 z2 – z1
then sin 2C sin 2 A
c a
( z – z1 )( z2 – z1 ) ( z – z1 ) ( z2 – z1 )
Ksin A KsinC
.2sin C cos C .sin A cos A
Ksin C Ksin A z – z1 z – z1
0
z2 – z1 z2 – z1
2(sin A cos C cos A sinC ) 2 sin( A C ) 2 sin B

3
1 x 4 (1 – x ) 4 1 4
10. (a) dx x6 – 4 x5 5 x4 – 4 x2 4– dx
2 3 0 1 x2 0 1 x2
2
1
x y z x7 2 x 6 4 x3
8. (c) Plane containing two lines and – x – 5
4 x – 4 tan –1
x
3 4 2 7 3 3
0
x y z
is given by 1 2 4
4 2 3 – 1– 4 – 4.
7 3 3 4
x y z
22
3 4 2 0 –
7
4 2 3
11. (b) Given that a x2 x 1, b x 2 –1, c 2x 1
8 x – y –10 z 0
Now equation of plane containing the line and C 6

x y z a2 b2 – c 2
and perpendicular to the plane 8x – y – 10 Using cos C
2 3 4 2ab
z = 0 is
(x2 x 1) 2 ( x 2 – 1) 2 – (2 x 1) 2
x y z we get cos
6 2( x 2 x 1)( x 2 – 1)
2 3 4 0
8 –1 –10 3 (x2 3x 2) ( x 2 – x ) ( x 2 1) 2
2
2 2( x x 1)( x 2 – 1)
– 26 x 52 y – 26 z 0 or x – 2 y z 0
9. (a, c, d)
Given that z (1 – t ) z1 t z2 where 0 t 1 3( x 2 x 1) ( x 1)( x – 1) x( x – 1)( x 1)( x 2)

(1 – t ) z1 tz2 ( x 1) 2 ( x – 1) 2
z
(1 t ) t
( x 1)( x – 1) 3( x 2 x 1) – x ( x 2) – ( x 1)( x – 1) 0
z divides the join of z1and z2internally in the ratio
t : (1– t). ( x 1)( x –1) 3 – 2 x2 3–2 x 3 1 0
t 1–t
z1 , z and z 2 are collinear
z1 z z2 x –1,1, 3 1 ,– 3 2

z – z1 z – z2 z1 – z2 Now for x = –1and 1, b = 0 which is not possible


Also z (1 – t ) z1 t z2 Also for x – 2 3 ,c – 4 – 2 3 1 0 which is

z – z1 not possible
t purely real number
z2 – z1 x 3 1
IIT-JEE 2010 SOLVED PAPER 37
12. (c, d) 14. (b) We known that tangent to hyperbola
Given parabola y 2 4x x2 y2
2
– 2
1 is y mx a 2 m 2 – b2
Let A(t12 , 2t1 )
and B(t22 , 2t2 ) a b
Then centre of circle drawn with AB as diameter is x2 y2
The tangent to hyperbola – 1 is
t12 t 22 9 4
, t1 t 2
2
y mx 9m 2 – 4 where m > 0
As circle touches x-axis As it is also the tangent to the circle
r t1 t2 t1 t2 r x2 y2 – 8x 0
2(t 2 – t1 ) 2 2 With centre (4 ,0) and radius 4
Also slope of AB
t 22 – t12 t2 t1 r
4m 9m 2 – 4
13. (b, c) We have 4
m2 1
x
f ( x) lnx 1 sin t dt
0
16m 2 9m 2 – 4 8m 9m 2 – 4 16m 2 16
1
f ( x) 1 sin x which exists x (0, )
x 9m 2 – 20 8m 9 m 2 – 4
and f '(x) has finite value x (0, ), so f '( x) is 81m 4 – 360m 2 400 64m 2 (9m 2 – 4)
continuous
495m 4 104m 2 – 400 0
1 cos x
Also f ''( x) –
x2 2 1 sin x –104 (104) 2 4 495 400
m2
Which does not exist at the points where 2 495
3 7
sin x –1 like x , ,... –104 802816 –104 896 792 4
2 2
990 990 990 5
f '( x) is not differentiable.
(a) is false but (b) is true 2
m as m 0
x
5
Now 1 sin t 0 1 sin t dt 0 x (0, )
0 2 4
the tan gent is y m
And lnx 0 x (1, ) 5 5
f ( x) 0 x (1, ) or 2 x – 5 y 4 0
For x e3 Alternative :-
x x2 y 2 x sec y tan
f ( x) ln x 1 sin t dt 3 Any tangent to – 1 is – 1
0 9 4 3 2
It touches circle with center (4,0) and radius = 4
1 1
f '( x ) 1 sin x 2, x 0
x x 4sec – 3
3 4
Now for x e3
sec 2 tan 2
1 1 9 4
0 f '( x ) 2 2 3 x (e 3 , )
x e3
f '( x) f ( x) sec 2 tan 2
16 sec 2 – 24sec 9 144
(c) is true. 9 4
Also lim f ( x)
x 12 sec 2 8sec–15 0
f ( x) f '( x) is not bounded. 5 –3
sec or
(d) is wrong. 6 2
EBD_780
38 MATHEMATICS
But this possibility is already included when A is
5
but sec 1is not possible symmetric and (a, b) = (0, 0).
6 Again if A is both symmetric and skew symmetric, then
5 clearly A is null matrix which case is already included.
sec –3/ 2 tan Hence total number of ways = p + (p – 1) = 2p – 1
2 17. (c) Trace A = a + a = 2a is not divisible by p
2 sec 2 –3 2 a is not divisible by p
slope of tangent a 0
3 tan 3 – 5 2 But |A| is divisible by p
(for +ve value of m) a2 – bc is divisible by p
l
It will be so if on dividing a2 by p suppose we get m p
5
l
–x y 5 then on dividing bc by p we should get n p for some
Equation of tangent is 1
2 4
integeral values of m, n and l.
or 2 x – 5 y 4 0 i.e. the remainder should be same in each case, so that
15. (a) The intersection points of given circle
a 2 bc l l
x 2 2
y – 8x 0...(1) ...(1) m n = (m – n) = an integer
p p p
and hyperbola 4 x2 – 9 y 2 – 36 0 ...(2) For this to happen a can take any value from 1 to p–1,
can be obtained by solving these equation s also if b takes any value from 1 to p–1 then c should
Substituting value of y2 from eqn (1) in eqn (2), take only that value corresponding to which the
we get remainder is same.
No. of ways = (p – 1) × (p – 1) × 1 = (p – 1)2.
4 x 2 – 9(8 x – x 2 ) 36 18. (d) Total number of matrices
2 = total number of ways a, b, c can be selected
13 x – 72 x – 36 0 = p × p × p = p3 .
–6 Number of ways when det (A) is divisible by p and
x 6, trace (A) 0 are equal to number of ways det (A) is
13
divisible by p and trace (A) is not divisible by
y2 12,
–48 –36
( not possible)
p = (p – 1)2
13 169 Also number of ways when det (A) is divisible by p
and trace A = 0 are the ways when bc is multiple of p
6, 2 3 and 6, –2 3 are points of intersection. b = 0 or c = 0
So eqn of required circle is for b = 0, c can take values 0, 1, 2, ......., p – 1
For c = 0, b can take values 0, 1, 2, ............, p – 1
( x – 6)( x – 6) y–2 3 y 2 3 0 Here (b, c) = (0, 0) is coming twice.
Total ways of selecting b and c = p + p – 1
x 2 36 –12 x y 2 –12 0 = 2p – 1
Total number of ways when det (A) is divisible
x 2 y 2 –12 x 24 0 by p = (p – 1)2 + 2p – 1 = p2
16. (d) If A is symmetrie then b =c Hence the number of ways when det (A) is not divisible
by p = p3 – p2.
A a 2 – b2 (a b)(a – b)
Which is divisible by p if (a + b) is divisible by p or a
19. (3) Using S , we get
(a – b) is divisible by p. 1 r
Now (a + b) is divisible by p if (a, b) can take values
(1, p – 1), (2, p – 2),(3, p – 3),...( p–1, 1) k 1
k! , k 1
( p –1)ways. 1
Also (a – b) is divisible by p only when a – b = 0 1
Sk k!
i.e. a = b, then (a, b) can take values (0,0) (1,1), (2,2).....
0, k 1
(pÝ, p–2)
p ways. 1
, k 2
If A is skew symmetric, then a = 0 and b = – c or b + c = 0 (k 1)!
which gives |A| = 0 when b2 b = 0, c = 0
IIT-JEE 2010 SOLVED PAPER 39
100 100 Y–y dy dy
1
Now | (k 2 3k 1) Sk | | (k 2 3k 1) | or X – x – Y–y 0
(k 1)! X –x dx dx
k 1 k 2

100 dy dy
( k 2 –1) 1 – 3( k – 1) – 2 X –Y x –y
–1 as k 2 – 3k 1 0 k 3 dx dx
k 3
( k – 1)!
dy
100
Its y-intercept y–x x3
1 1 dx
1 –
(k – 3)! (k – 1)! dy y
k 3
– – x2
dx x
1 1 1 1 1
1 1– – – .... 1
2! 1! 3! 2! 4! – dx
x 1
I.F. e
x
1 1 1 1
.. – – 1 1 – x2
96! 98! 97! 99! y. – x2 dx C
x x 2
1 1 9900 100
3– – 3– – – x3
98! 99! 100! 100! y Cx
2
2
10000 100
3– 3– As f (1) 1 At x 1, y 1
100! 100!
–1
100 1 C C 32
1002 2
3 k 2 – 3k 1 S k 3.
100! k 1 x3 3x
20. 3 y –
2 2
The given equations are
27 9
xyz sin 3 ( y z ) cos 3 — (1) At x –3, y – 9
2 2
xyz sin 3 2 z cos 3 2 y sin 3 — (2)
f (–3) 9.
xyz sin 3 y 2 z cos 3 y sin 3 — (3) 22. 3
Operating (1) – (2) and (3) – (1), we get n
tan cot 5 ,
cos3 – 2sin y – cos 3 z 0 5
cos cos 5 – sin 5 sin 0
and sin 3 y cos3 z 0
cos 6 0
which is homogeneous system of linear equation. But
y 0, z 0 –5 –3 – 3 5
6 , , , , ,
2 2 2 2 2 2
cos3 – 2sin 3 cos 3

sin 3 cos 3 –5 – – 5
, , , , ,
12 4 12 12 4 12
cos3 sin 3
Again sin 2 cos 4 1 – 2 sin 2 2
tan 3 1 3 n
4 2sin 2 2 sin 2 –1 0
1
4n 1 ,n Z sin 2 –1,
12 2
For 0, – 5
2 , ,
2 6 6
5 3
, , – 5
12 12 4 , ,
4 12 12
Three such solutions are possible.
21. 9 – 5
So common solutions are , and
The equation of tangent to the curve 4 12 12
y f (x) at the point P (x,y) is Number of solutions = 3.
EBD_780
40 MATHEMATICS
23. 2
4a 2 – a 2 e2 –1 1
1
Let f ( )
g( ) e2 e2 2
4 – e –1 1
4 4
Where g ( ) sin 2 3sin cos 5cos2
Clearly f is maximum when g is minimum 4e2 – e4 e2 4

Now g ( )
1– cos 2 3
sin 2
5
1 cos 2 e4 – 5e4 4 0
2 2 2
e2 4 as e 1 for hyperbola.
3 9 e 2
3 2cos 2 sin 2 3 – 4
2 4 26. 6
The equation of plane containing the lines
5 1
g min 3– x –1 y–2 z–3 x–2 y–3 z–4
2 2 and is
2 3 4 3 4 5
f max 2.
24. 5 x –1 y–2 z –3
2 3 4 0
iˆ – 2 j 2iˆ j 3k
We have a ,b 3 4 5
5 14
x – 2y z 0
Clearly a 1 b 1 and a.b 0
Distance between x – 2 y z 0 and Ax – 2 y z d
2a b . a b a – 2b = Perpendicular distance between parallel planes
A 1
– 2a b . a – 2b a b
d
6 d 6.
– 2a b . a – 2b .b a – a – 2b .a b 6
27. 4
2 2
– 2a b . a.b – 2 b a – a – 2b.a b x – [ x] if [ x] is odd
Given function is f ( x)
1 [ x] – x if [ x] is even
– 2a b . –2a – b The graph of this function is as below

2 2 T=2
2a b . 2a b 4a b a.b 0

4 1 5.
25. 3 –3 –2 –1 1 2 3 4
a Clearly f(x) is periodic with period 2
Intersection point of nearest directrix x and x-axis Also cos x is periodic with period 2
e
f ( x)cos x is periodic with period 2
a
is ,0 2
e 10
I f ( x) cos x dx
10 –10
a
As 2 x y 1 passes through ,0 2
e 2
10 f ( x) cos x dx
10 0
2a e
1 a
e 2 2
1 2
(1 – x ) cos x dx ( x – 1) cos x dx
0 1
x2 y2
Also y –2 x 1 is a tangent to – 1
a2 b2 sin x
1 1 sin x
2
(1 – x ) dx
2 2 2 2 2 0
1 a –2 –b 4a – b 1 0
IIT-JEE 2010 SOLVED PAPER 41

2 2 2
sin x 2 sin x z 1
( x – 1) dx
2 2
1 1 z 1 z 1 0
2
z 1 1 z
1 2
2 1 1
– 2
cos x – 2
cos x
2
0 1
2
– cos cos 0 – – cos 2 cos 1 z 1 0
1 1 z
2 2 4
28. 1
z 1 z2 z z 2 3
–1 – z –1
2 2 –1 i 3
We have cos i sin 2 2
3 3 2 1– z – 0
2 3
1 0 and 1
z z2 z z 2
–z – 2 2
–z 2
– 4
0
2
z 1
z z2 0 z3 0 z 0
2
z 1 0
Then z 0 is the only solution.
2
1 z

C1 C1 C2 C3
Paper -2
1. (a) As perpendicular distance of x 2 y – 2 z from the 2. (c) Let G oreginal signal is green P(G) 45
point (1, – 2, 1) is 5
E1 A receives the signal correctly P(E1 ) 34
1– 4 – 2 –
5 E2 B receives the signal correctly P(E 2 ) 34
3
P(1, –2, 1) E Signal received by B is green.
Then E can happen in the following ways
Original Signal Received at A Received at B
Re d Re d Green

Re d Green Green
N
Green Green Green
x + 2y – 2z = Green
Green Re d
–5 –
5 or – 5 P(E) ) G E1 E2 G E1 E2
3
–20 or 10
P G E1 E2 P G E1 E2
As 0 10
Plane becomes x 2 y – 2 z –10 0 1 3 1 1 3 1 4 3 3 4 1 1
x –1 y 2 z –1 5 4 4 5 4 4 5 4 4 5 4 4
Equation of PN is
1 2 –2 3 3 36 4 46 23
For some value of , N 1, 2 – 2, –2 1 80 80 40
It lies on x 2 y – 2 z –10 0 P(G E) P(G E1 E 2 ) P(G E1 E2 )
P(G / E)
1 4 – 4 4 – 2 10 P(E) P(E)

9 15 53 4 3 3 4 1 1
5 4 4 5 4 4 40 80 20
2 4 –7
N , , 23 40 23 40 23
3 3 3
EBD_780
42 MATHEMATICS

D theorem and comparing the coefficients of x10 in their


product on both sides, we get
D C 2 2 2 2
10 10 10 10
C0 C1 C2 ... C10
3. (b)
– coeff of x10 in (1 x) 20 20
C10
A B
2 2 2
10 10 10 20
C1 C2 ... C10 C10 –1
AB AD
sin 90 – Substituting these values in equation (1), we get
AB AD 20 30 30 20
C10 C10 – 1 – C10 C10 – 1
Where
30 20
C10 – C10 C10 – B10
l j k x
AB AD 2 10 11 5. (b) e – x f ( x) 2 1 t 4 dt x (–1,1)
0
–1 2 2 At x = 0, f (0) = 2
Now on differentiating, we get
– 2l –15 j 14 k
– e – x f ( x) e – x f '( x) 0 1 x4
AB AD 4 225 196 425
– f (0) f '(0) 1 f '(0) 3
AB 4 100 121 225 15 –1
Now f f ( x) x

AD 1 4 4 3 '
f –1 f ( x) f '( x ) 1

sin 90 –
425 17 f –1 ' f (0) f ' 0 1
15 3 9
–1 '(2) 1
17 f
cos 3
9 6. (d) S = {1, 2, 3, 4}
4. (d) Clearly Ar 10
Cr , Br 20
Cr , Cr 30
Cr Let P and Q be disjoint subsets of S
Now for any element a s, following cases are
10
10 20 20
possible
Now Cr C10 Cr – 30 C1010 Cr
r 1 a P and a Q
10 10
a P and a Q
20 10 20
C10 Cr Cr – 30 C10 10
Cr 10
Cr a P and a Q
r 1 r 1 For every element there are three option
20
C10 10
C1 20 C1 10
C2 20 C2 ... 10
C10 20 C10 Total options = 34 = 81
Here P Q except when P Q
30 10 10 10 10 10
C10 C1 C1 C2 C2 ... C10 10 C10 ....(1)
80 ordered pairs (P, Q) are there for which P Q.
10 20 Hence total number of unordered pairs of disjoint
Now expanding 1 x and 1 x by binomial theorem
80
and comparing the coefficients of x 20 in their product, on subsets 1 41
2
both sides, we get 7. 0
10 20 10
C0 C0 C120C1 10
C2 20
C2 ... 10 20
C10 C10 Given that ak 2ak –1 – ak – 2
30
= coeff of x 20 in 1 x ak –2 ak
ak –1 , 3 k 11
30 30
2
C20 C10
a1 , a2 , a3 ,..., a11 are in AP..
10
C120C1 10
C220C2 ... 10
C10 20
C10 30
C10 –1 If a is the first term and D the commen difference then
Again expending 1 x
10
and x 1
10
by binomial a12 a22 ... a11
2
990
IIT-JEE 2010 SOLVED PAPER 43

11a 2 d 2 12 22 ... 102 2ad 1 2 ... 10


10. 3
990
From figure, we get
10 11 21 2 10 11
11a 2 d 2ad 990
6 2
A B
k
a2 35d 2 150d 90 2 3 1
2 O
Using a = 15, we get C 2k
D
2 2
35d 150d 135 0 or 7d 30d 27 0

d 3 7d 9 0 d –3 or – 9 7

9 96 27 2 cos 2 cos 3 1
then a2 15 – 3 12 or 15 – k 2k
7 7 2

d –9 7 2 2 cos 2 2 cos –2 3 1
2k 2k
11
2 15 10 –3
Hence a1 a2 ... a11 2 4 cos 2 2 cos – 3 3 0
0 2k 2k
11 11
8. 1 –2 4 16 3 3 –1 13 4 3
We have , cos
2k 8 4
f (x) 2010 ( x – 2009)( x – 2010)2 ( x – 2011)3 ( x – 2012)4
–1 2 3 1 3 3 1
As f ( x) ln g ( x) or –
4 2 2
g ( x) e f ( x ) g '( x) e f ( x) . f '( x)

For max/min, g '( x) 0 f '( x) 0 3


As is an acute angle, cos cos
2k 2k 2 6
Out of two points one should be a point of maxima and
other that of minima. k 3
There is only one point of local maxima. 11. 3
9. 4
A

2k – 1 2 k 2 k 2k – 1 0 2 k c
A 2 k 1 – 2k 2 k 1 2k – 2k b = 10
–2 k 2k –1 –2 k 1 2k –1
B C
a=6
1
2k – 1 0 2 k We know that area of ab sin C
2
4 k 0 1 – 2k (1 2k ) (8k – 4k 4 k 2 1)
1
–2 k 1 2k –1 6 10 sin C 15 3
2

(2k 1)3 sin C


3
2
Also B 0 as B is skew symmetric of odd order..
C 120 as C is obtuse angle.
2 2 6
Adj A Adj B A B 10 a2 b2 – c 2
Now using cos C ,
2ab
(2k 1)6 106 2k 1 10 k 4.5
36 100 – c 2
k 4 we get cos120
120
EBD_780
44 MATHEMATICS

c2 196 or c 14 14. (b) f '( x) 2(6 x2 3 x 1)


a b c f ''( x) 6 4x 1
s 15
2
Critical point x –1 4
15 3 1 – +
There radius of incircle, r 3
s 15 decreasing in – t , –
4 t 1 t
4
2
r 3
1
12. (c) f ( x) 4 x 3 3x 2 2x 1 and increasing in – , t
4
f ( x) is a cubic polynomial 15. (d)
It has at least one real root. x2 y2
Tangent to 1 at the point (3cos , 2sin ) is
Also f '( x ) 12 x
2
6x 2 2 6 x 2 3x 1 0 x R 32 22
x cos y sin
f ( x) is strictly increasing function 1
3 2
There is only one real root of f ( x) 0
P
3 1
Also f –1 2 1–1 – 0
2) (3, 4)
4 2 (0,
3 27 27 B
and f –3 4 1– – 0
2 16 16
3 1
Real root lies between – and – and hence A (3, 0)
4 2
3 1
s – ,–
4 2
13. (a) y f ( x ), x 0, y 0 and x t bounds the area as
As it passes throught (3,4), we get
shown in the figure
cos 2 sin 1
Y
y f ( x) 4sin 2
1 cos2 – 2cos

5cos 2 – 2cos – 3 0

3
cos 1,
X 5
34 12 12 34
4
sin 0,
x t 5
Required area is given by 9 8
t Required points are A (3, 0) and B – ,
A dx 5 5
0
Alternative
t
4 x3 3x 2 2 x 1 dx x2 y2
0 Let the eqn of tangent to 1 with slope m be
9 4
4 3 2 2
t t t t t t 1 t 1
y mx 9m 2 4
1 3 3 7 5 25 As it passes through (3,4)
New t ; t 1 ; t2 1
2 4 2 4 4 16
4 3m 9 m2 4
1 3 5 3 7 25
A
2 2 4 4 4 16 9m2 16 – 24m 9m 2 4 or 0m2 – 24m 12 0
15 525 3 1
or A , ,3 m and from the other value of m tangent is vertical.
16 256 4 2
IIT-JEE 2010 SOLVED PAPER 45
Clearly for vertical tangent point of contact is A(3, 0) and 2
2 2 x 3y – 3
1 x–3 y–4
for m equation of tangent becomes 10
2
2 2
or 9 x y – 6 xy – 54 x – 62 y 241 0
x 5 –x 2y
y or 1
2 2 5 5 18. (A) q z –i z z –i z
But if ( x1 , y1 ) is the point of contact tangent is
z is equidistant from two points 0, z and
xx1 yy1
1 0, – z which lie on imaginary axis.
9 4
On comparing the two, we get z must lie on real axis Im( z ) 0
x1 / 9 y1 4 –9 8 (B) p
1 x1 and y1 Sum of distances of z from two fined points (–4, 0) and
–1 5 25 5 5
(4, 0) is 10 which is greater than 8.
Point B –9 5,8 5 z traces an ellipse with 2a = 10 and 2ae 8
16. (c) e 4
Let H be the orthocentre of PAB, then as BH AP, BH 5
is a horizontal line through B. (C) p,t
Let 2(cos i sin )
P (3, 4)
– 95 , 85 1 1
H then z – 2(cos i sin ) (cos – i sin )
B 2
3 5
x iy cos i sin
A (3, 0) 2 2

9 25 34
Here z 3
4 4
3 5
y- cordinate of B = 8/5 Also x cos , y sin
2 2
Let H has coordinater ( ,8 5)
4x2 4 y2
1
8 9 25
–4
5 –12
Then slope of PH 9 4
–3 5 –3 Which is an ellipse with e 1–
25 5
8 (D) q,t
–0
5 8 –1
and slope of AB Let cos i sin
–9 –24 3
–3 then z 2 cos Im z 0
5
Also z 3
–12 –1
But PH AB –1 19. ( A) t
5 –3 3
x y z
4 –5 15 or 11 5 Let the line through origin be — (1)
a b c
Hence H 115 , 8 5 then as it intersects
x–2 y –1 z 1
17. (a) Clearly the moving point traces a parabola with focus —(2)
1 –2 1
at P(3, 4) and directrix as
x – 8 3 y 3 z –1
y – 0 –1 and —(3)
AB : or x 3 y – 3 0 2 –1 1
x–3 3 at P and Q, shortest distance of (1) with (2) and (3) should
Equation of parabola is be zero.
EBD_780
46 MATHEMATICS

x2 x1 y2 y1 z2 z1 a– b
b –a . b 0
Using a1 b1 c1 0 4
a2 b2 c2
4– a
b –a . b 0
4 4
2 1 –1
we get a b c 0 a 3b 5c 0 —(4) 4– a2
b2 – 0
1 –2 1 4 4
4– b2 – a 2 0 —(1)
8 3 –3 1
Also
and a b c 0 3a b – 5c 0 —(5)
2
2 –1 1 4– a 2
22 b b –a
4 4
Solving (4) and (5), we get
a b c a b c (4 – )2 b 2 a2 4b2 4a 2
or
–15 – 5 15 5 1– 9 5 –5 2
(4 – )2 – 4 b 2 3a 2 —(2)
x y z From (1) and (2), we get
Hence equation (1) becomes
5 –5 2
2
4– –4 3
For some value of , P(5 , –5 , 2 )
4– 1
which lies on (2) also
2
5 –2 –5 – 1 2 1 –8 12 12 – 3
1
1 –2 1 2
–5 0
P (5, –5, 2) 0 or 5
Also for some value of , Q(5 , –5 , 2 ) (D) – r
which lies on (3) also
2 sin(9 x 2) 2 sin 9 x 2
5 –8 3 –5 3 2 –1 I dx 2 dx
23 – sin( x 2) 0 sin x 2
3 –1 1
x
10 –10 4 Let 0 dx 2d
Q , , 2
3 3 3
Also at x 0, 0 and at x x, 2
2 25 25 4 8 2 sin 9
Hence d PQ 2 6 I d
9 9 9 0 sin
(B) p,r 8 2 sin 9 – sin 7 (sin 7 – sin 5 )
I
3
0 sin sin
tan –1 x 3 – tan –1 x – 3 tan –1
4 (sin 5 – sin 3 ) (sin 3 – sin ) sin
d
–1 x 3– x 3 –1 3 sin sin sin
tan tan , x2 – 9 –1
1 x2 – 9 4 16 2 8 2
(cos8 cos 6 cos 4 cos 2 1)d d
0 0
6 3
x2 16 or x 4, –4 /2
x2 – 8 4 16 sin 8 sin 6 sin 4 sin 2
8 6 4 2
(C) q,s 0

2 8
a b ( )0 / 2
We have c 0
4
8
0 –0 4
Then b – a . b c 0 2
IIT-JOINT ENTRANCE EXAMINATION (IIT-JEE)
SOLVED PAPER - 2009

PAPER -1
CHEMISTRY
5. The term that corrects for the attractive forces present in a
SECTION – I
real gas in the van der Waals equation is
Single Correct Choice Type
This section contains 8 multiple choice questions. Each question an2
has 4 choices (a), (b), (c) and (d) for its answer, out of which ONLY (a) nb (b)
V2
ONE is correct.
1. Among the electrolytes Na2SO4, CaCl2, Al2(SO4)3 and an2
(c) – (d) – nb
NH4Cl, the most effective coagulating agent for Sb2S3 sol V2
is
6. Among cellulose, poly (vinyl chloride), nylon and natural
(a) Na2SO4 (b) CaCl2 rubber, the polymer in which the intermolecular force of
(c) Al2(SO4)3 (d) NH4Cl attraction is weakest is
2. Given that the abundances of isotopes 54Fe, 56Fe and 57 Fe (a) Nylon (b) Poly (vinyl chloride)
are 5%, 90% and 5%, respectively, the atomic mass of Fe is (c) Cellulose (d) Natural Rubber
(a) 55.85 (b) 55.95 7. The reaction of P4 with X leads selectively to P4O6. The X
(c) 55.75 (d) 56.05 is
(a) Dry O2
3. The correct acidity order of the following is
(b) A mixture of O2 and N2
OH OH COOH COOH (c) Moist O2
(d) O2 in the presence of aqueous NaOH
8. The Henry’s law constant for the solubility of N2 gas in
water at 298 K is 1.0 × 105 atm. The mole fraction of N2 in air
is 0.8. The number of moles of N2 from air dissolved in
Cl CH3 10 moles of water at 298 K and 5 atm pressure is
(a) 4.0× 10– 4 (b) 4.0 × 10–5
(I) (II) (III) (IV)
(c) 5.0 × 10– 4 (d) 4.0 × 10–6
(a) (III) > (IV) > (II) > (I) (b) (IV) > (III) > (I) > (II)
(c) (III) >(II) >(I) > (IV) (d) (II) > (III) > (IV) > (I)
SEC TION – II
4. The IUPAC name of the following compound is
Multiple Correct Choice Type
OH This section contains 4 multiple choice questions . Each question
has 4 choices (a), (b), (c) and (d) for its answer, out of which ONE
OR MORE is/are correct.
9. The compound(s) that exhibit(s) geometrical isomerism is
CN
(are)
Cl
(a) [Pt (en) Cl2] (b) [Pt (en)2] Cl2
(a) 4-Bromo-3-cyanophenol (c) [Pt (en)2 Cl2] Cl2 (d) [Pt (NH3)2 Cl2]
10. The compound(s) formed upon combustion of sodium metal
(b) 2-Bromo-5-hydroxybenzonitrile
in excess air is (are)
(c) 2- Cyano-4-hydroxybromobenzene (a) Na2O2 (b) Na2O
(d) 6-Bromo-3-hydroxybenzonitrile (c) NaO2 (d) NaOH
EBD_780
2009- 2 Target IIT-JEE
11. The correct statement(s) regarding defects in solids is (are) 16. The structure of the carbonyl compound P is
(a) Frenkel defect is usually favoured by a very small
difference in the sizes of cation and anion Me
(b) Frenkel defect is a dislocation defect (a) (b)
(c) Trapping of an electron in the lattice leads to the O Me O Me
formation of F-centre
(d) Schottky defects have no effect on the physical O
properties of solids
12. The correct statement(s) about the compound
H3C(HO)HC–CH= CH – CH(OH)CH3 (X) is(are) (c) (d)
(a) The total number of stereoisomers possible for X is 6 O Et Me
(b) The total number of diastereomers possible for X is 3
17. The structures of the products Q and R, respectively, are
(c) If the stereochemistry about the double bond in X is
O
trans, the number of enantiomers possible for X is 4
(d) If the stereochemistry about the double bond in X is (a) H
Me ,
cis, the number of enantiomers possible for X is 2
COMe
SECTION – III Me Me Me Me
Comprehension Type O
This section contains 2 groups of questions. Each group has 3
(b) , H
multiple choice questions based on a paragraph. Each question
has 4 choices (a), (b), (c) and (d) for its answer, out of which ONLY Me CHO
Me
ONE is correct Me Me
Paragraph for Question Nos. 13 to 15 O
p-Amino-N, N-dimethylaniline is added to a strongly acidic
H
solution of X. The resulting solution is treated with a few drops
(c) , CHO
of aqueous solution of Y to yield blue coloration due to the
formation of methylene blue. Treatment of the aqueous solution Me Et Me Et
of Y with the reagent potassium hexacyanoferrate(II) leads to the Me O
formation of an intense blue precipitate. The precipitate dissolves
(d) , CH3
on excess addition of the reagent. Similarly, treatment of the
CHO
solution of Y with the solution of potassium hexacyanoferrate (III)
Me Me Et
leads to a brown coloration due to the formation of Z.
18. The structure of the product S is
13. The compound X is
O
(a) NaNO3 (b) NaCl
(c) Na2SO4 (d) Na2S (a) (b)
14. The compound Y is O
(a) MgCl2 (b) FeCl2 Me Me
Me
(c) FeCl3 (d) ZnCl2
O O
15. The compound Z is
Me
(a) Mg2[Fe(CN)6] (b) Fe[Fe(CN)6]
(c) (d)
(c) Fe4[Fe(CN)6]3 (d) K2Zn3[Fe(CN)6]2
Paragraph for Question Nos. 16 to 18 Me Me
A carbonyl compound P, which gives positive iodoform test,
SECTION – IV
undergoes reaction with MeMgBr followed by dehydration to
give an olefin Q. Ozonolysis of Q leads to a dicarbonyl compound Matrix - Match Type
R, which undergoes intramolecular aldol reaction to give This section contains 2 questions. Each question contains
statements given in two columns, which have to be matched. The
predominantly S. statements in Column I are labelled A, B, C and D, while the
statements in Column-II are labelled p, q, r, s and t. Any given
1. MeMgBr 1. O 1. OH

statement in Column -I can have correct matching with ONE OR
P + Q 3
R S MORE statement(s) in Column-II. The appropriate bubbles
2.H , H O 2. Zn,H O 2.
2
2
3.H 2SO 4 ,
corresponding to the answers to these questions have to be
darkened as illustrated in the following example.
2009 IIT-JEE Solved Paper 2009-3
If the correct matches are A–p, s and t; B–q and r; C–p and q; and 22. Let P (3, 2, 6) be a point in space and Q be a point on the line
D–s and t; then the correct darkening of bubbles will look like
the following. r (iˆ – ˆj 2kˆ) (–3iˆ ˆj 5kˆ)
p q r s t Then the value of for which the vector PQ is parallel to
A p q r s t the plane x – 4y + 3z =1 is
B p q r s t 1 1
(a) (b) –
C p q r s t 4 4
D p q r s t 1 1
(c) (d) –
8 8
19. Match each of the diatomic molecules in Column I with its 23. If a , b , c and d are unit vectors such that
property/properties in Column II.
Column I Column II a b c d 1
(A) B2 (p) Paramagnetic 1
(B) N2 (q) Undergoes oxidation and a c ,
2
(C) O–2 (r) Undergoes reduction then
(s) Bond order 2 (a) a , b , c are non-coplanar
(D) O2
(t) Mixing of 's' and 'p' orbital (b) b , c , d are non-coplanar
20. Match each of the compounds in Column I with its
(c) b , d are non-parallel
characteristic reaction(s) in Column II.
Column I Column II (d) a , d are parallel and b , c are parallel
(A) CH3CH2 CH2CN (p) Reduction with Pd–C/H2
(B) CH3 CH2 OCOCH3 (q) Reduction with SnCl2/HCl 24. The number of seven digit integers, with sum of the digits
(C) CH3–CH= CH–CH2OH (r) Development of foul smell equal to 10 and formed by using the digits 1, 2 and 3 only, is
on treatment with chloroform (a) 55 (b) 66
and alcoholic KOH (c) 77 (d) 88
(D) CH3CH2CH2CH2NH2 (s) Reduction with 25. Tangents drawn from the point P(1, 8) to the circle
diisobutylaluminium x2 + y2 –6x– 4y –11 = 0
hydride(DIBAL-H) touch the circle at the points A and B. The equation of the
(t) Alkaline hydrolysis. circumcircle of the triangle PAB is
(a) x2 + y2 + 4x– 6y +19 = 0
(b) x2 + y2 – 4x – 10y + 19 = 0
MATHEMATICS (c) x2 + y2 – 2x + 6y – 29 = 0
(d) x2 + y2 – 6x – 4y +19 = 0
SECTION – I 26. Let z = cos + i sin . Then the value of
Single Correct Choice Type 15
This section contains 8 multiple choice questions. Each question Im( z 2 m –1 )
has 4 choices (a), (b), (c) and (d) for its answer, out of which ONLY
m 1
ONE is correct.
at = 2° is
21. Let f be a non-negative function defined on the interval
x x 1 1
2 (a) (b)
[0, 1]. If 1 – ( f '(t )) dt f (t ) dt , 0 x 1, sin 2 3sin 2
0 0 1 1
and f (0) = 0, then (c) (d)
2 sin 2 4 sin 2
1 1 1 1 27. The line passing through the extremity A of the major axis
(a) f and f
2 2 3 3 and extremity B of the minor axis of the ellipse
x2 + 9y2 = 9
1 1 1 1
(b) f and f meets its auxiliary circle at the point M. Then the area of the
2 2 3 3 triangle with vertices at A, M and the origin O is
1 1 1 1 31 29
(c) f and f (a) (b)
2 2 3 3 10 10
1 1 1 1 21 27
(d) f and f (c) (d)
2 2 3 3 10 10
EBD_780
2009- 4 Target IIT-JEE
28. Let z = x + iy be a complex number where x and y are integers.
SECTION – III
Then the area of the rectangle whose vertices are the roots
of the equation Comprehension Type
This section contains 2 groups of questions. Each group has 3
z z–3 z– z3 350 multiple choice questions based on a paragraph. Each question
is has 4 choices (a), (b), (c) and (d) for its answer, out of which ONLY
(a) 48 (b) 32 ONE is correct.
Paragraph for Question Nos. 33 to 35
(c) 40 (d) 80
Let A be the set of all 3× 3 symmetric matrices all of whose
SECTION – II entries are either 0 or 1. Five of these entries are 1 and four of them
Multiple Correct Choice Type are 0.
This section contains 4 multiple choice quesitons. Each question 33. The number of matrices in A is
has 4 choices (a), (b), (c) and (d) for its answer, out of which ONE
(a) 12 (b) 6
OR MORE is/are correct. (c) 9 (d) 3
29. Let
34. The number of matrices A in A for which the system of
x2
2 2 linear equations
a– a –x –
L = lim 4 ,a 0. x 1
x 0 x4
A y 0
If L is finite, then
z 0
(a) a = 2 (b) a=1
has a unique solution, is
1 1
(c) L= (d) L = (a) less than 4 (b) at least 4 but less than 7
64 32 (c) at least 7 but less than 10(d) at least 10
30. In a triangle ABC with fixed base BC, the vertex A moves
such that
35. The number of matrices A in A for which the system of
linear equations
A.
cos B cos C 4sin 2 x 1
2
If a, b and c denote the lengths of the sides of the triangle A y 0
opposite to the angles A, B and C, respectively, then z 0
(a) b + c = 4a is inconsistent, is
(b) b + c = 2a (a) 0 (b) more than 2
(c) locus of point A is an ellipse (c) 2 (d) 1
(d) locus of point A is a pair of straight lines Paragraph for Question Nos. 36 to 38
31. Area of the region bounded by the curve y = ex and lines A fair die is tossed repeatedly until a six is obtained. Let X denote
x = 0 and y = e is the number of tosses required.
e 36. The probability that X = 3 equals
(a) e –1 (b) ln (e 1– y ) dy 25 25
1 (a) (b)
216 36
1 e
5 125
(c) x
e – e dx (d) ln y dy (c) (d)
1 36 216
0
37. The probability that X 3 equals
32. If
125 25
4 4 (a) (b)
sin x cos x 1 216 36
,
2 3 5 5 25
then (c) (d)
36 216
38. The conditional probability that X 6 given X > 3 equals
2 sin8 x cos8 x 1
(a) tan 2 x (b) 125 25
3 8 27 125 (a) (b)
216 216
1 sin8 x cos8 x 2
(c) tan 2 x (d) 5 25
3 8 27 125 (c) (d)
36 36
2009 IIT-JEE Solved Paper 5
2009-

SECTION – IV PHYSICS
Matrix-Match Type SECTION – I
This section contains 2 questions. Each question contains
statements given in two columns, which have to be matched. The Single Correct Choice Type
statements in Column-I are labelled A, B, C and D, while the This section contains 8 multiple choice questions. Each question
statements in Column-II are labelled p, q, r, s and t. Any given has 4 choices (a), (b), (c) and (d) for its answer, out of which ONLY
statement in Column -I can have correct matching with ONE OR ONE is correct.
MORE statement(s) in Column-II. The appropriate bubbles
corresponding to the answers to these questions have to be 41. A ball is dropped from a height of 20 m above the surface of
darkened as illustrated in the following example: water in a lake. The refractive index of water is 4/3. A fish
If the correct matches are A–p, s and t; p q r s t inside the lake, in the line of fall of the ball, is looking at the
B–q and r; C–p and q; and D–s and t; A p q r s t ball. At an instant, when the ball is 12.8 m above the water
then the correct darkening of bubbles B p q r s t surface, the fish sees the speed of ball as [Take g = 10 m/s2.]
will look like the following. (a) 9 m/s (b) 12 m/s
C p q r s t
(c) 16 m/s (d) 21.33 m/s
D p q r s t 42. Three concentric metallic spherical shells of radii R, 2R, 3R,
39. Match the statements/expressions in Column I with the open are given charges Q1, Q2, Q3, respectively. It is found that
intervals in Column II. the surface charge densities on the outer surfaces of the
shells are equal. Then, the ratio of the charges given to the
Column I Column II shells, Q1 : Q2 : Q3, is
(a) 1 : 2 : 3 (b) 1 : 3 : 5
(A) Interval contained in the domain of (p) – , (c) 1 : 4 : 9 (d) 1 : 8 : 18
2 2
43. The figure shows certain wire segments joined together to
definition of non-zero solutions of the form a coplanar loop. The loop is placed in a perpendicular
differential equation ( x – 3)2 y + y = 0 magnetic field in the direction going into the plane of the
(B) Interval containing the value of the (q) 0, figure. The magnitude of the field increases with time. I1 and
2 I2 are the currents in the segments ab and cd. Then,
integral
5 c d
( x – 1)( x – 2)( x – 3)( x – 4)( x – 5)dx a b
1
5
(C) Interval in which at least one of the (r) ,
8 4
points of local maximum of
cos2x + sin x lies
(D) Interval in which tan–1 (sin x + cos x) (s) 0,
8 (a) I1 > I2
is increasing (b) I1 < I2
(c) I1 is in the direction ba and I2 is in the direction cd
(t) – ,
(d) I1 is in the direction ab and I2 is in the direction dc
40. Match the conics in Column I with the statements/ 44. The x-t graph of a particle undergoing simple harmonic
expressions in Column II. motion is shown below. The acceleration of the particle at
t = 4 / 3 s is
Column I Column II
(A) Circle (p) The locus of the point (h,k) for which 1
the line hx + ky =1 touches the circle
x2 + y2 =4
x (cm)

(B) Parabola (q) Points z in the complex plane satisfying


0 t(s)
| z + 2| – | z – 2 |= ±3 4 8 12
(C) Ellipse (r) Points of the conic have parametric
representation
1– t2 2t –1
x 3 , y
2 2
1 t 1 t 3 – 2
2
(D) Hyperbola (s) The eccentricity of the conic lies in (a) cm/s 2 (b) cm/s 2
32 32
the interval 1 x
(t) Points z in the complex plane satisfying 2
3 2
2 2 (c) cm/s 2 (d) – cm/s2
Re ( z 1) |z| 1 32 32
EBD_780
2009- 6 Target IIT-JEE
45. A block of base 10 cm × 10 cm and height 15 cm is kept on an 48. Look at the drawing given in the figure which has been
inclined plane. The coefficient of friction between them is drawn with ink of uniform line-thickness. The mass of ink
used to draw each of the two inner circles, and each of the
3. The inclination of this inclined plane from the
two line segments is m. The mass of the ink used to draw the
horizontal plane is gradually increased from 0°. Then outer circle is 6m. The coordinates of the centres of the
(a) at = 30°, the block will start sliding down the plane different parts are: outer circle (0, 0), left inner circle (– a, a),
(b) the block will remain at rest on the plane up to certain right inner circle (a, a), vertical line (0, 0) and horizontal line
and then it will topple (0, – a). The y-coordinate of the centre of mass of the ink in
(c) at = 60°, the block will start sliding down the plane this drawing is
and continue to do so at higher angles y
(d) at = 60°, the block will start sliding down the plane
and on further increasing , it will topple at certain
46 A disk of radius a / 4 having a uniformly distributed charge
6 C is placed in the x - y plane with its centre at (–a / 2, 0, 0).
A rod of length a carrying a uniformly distributed charge x
8 C is placed on the x - axis from x = a /4 to x = 5a / 4. Two
point charges – 7 C and 3 C are placed at (a / 4, – a /4, 0) and
(– 3a /4, 3a / 4, 0), respectively. Consider a cubical surface
formed by six surfaces x = a / 2, y = a / 2, z = a / 2. The a a
electric flux through this cubical surface is (a) (b)
10 8

y a a
(c) (d)
12 3
SECTION – II
Multiple Correct Choice Type
x
This section contains 4 multiple choice questions. Each question
has 4 choices (a), (b), (c) and d for its answer, out of which ONE
OR MORE is/are correct.
49. A student performed the experiment of determination of focal
length of a concave mirror by u-v method using an optical
2C bench of length 1.5 meter. The focal length of the mirror
–2 C
(a) (b) used is 24 cm. The maximum error in the location of the
0
0 image can be 0.2 cm. The 5 sets of (u, v ) values recorded by
10 C 12 C the student (in cm) are : (42, 56), (48, 48), (60, 40), (66, 33),
(c) (d) (78, 39). The data set(s) that cannot come from experiment
0 0 and is (are) incorrectly recorded, is (are)
47. Two small particles of equal masses start moving in opposite (a) (42, 56) (b) (48, 48)
directions from a point A in a horizontal circular orbit. Their (c) (66, 33) (d) (78, 39)
tangential velocities are v and 2v, respectively, as shown in 50. For the circuit shown in the figure
the figure. Between collisions, the particles move with
constant speeds. After making how many elastic collisions,
other than that at A, these two particles will again reach the
2k R1
point A?
I
A
v 2v

24 V 6k R2 RL 1.5 k

(a) the current I through the battery is 7.5 mA


(b) the potential difference across RL is 18 V
(c) ratio of powers dissipated in R1 and R2 is 3
(a) 4 (b) 3 (d) if R1 and R2 are interchanged, magnitude of the power
(c) 2 (d) 1 dissipated in RL will decrease by a factor of 9
2009 IIT-JEE Solved Paper 2009-7
51. If the resultant of all the external forces acting on a system 54. Assume that two deuteron nuclei in the core of fusion reactor
of particles is zero, then from an inertial frame, one can surely at temperature T are moving towards each other, each with
say that kinetic energy 1.5 kT, when the separation between them is
(a) linear momentum of the system does not change in large enough to neglect Coulomb potential energy. Also
time neglect any interaction from other particles in the core. The
(b) kinetic energy of the system does not change in time minimum temperature T required for them to reach a
(c) angular momentum of the system does not change in
separation of 4 × 10–15 m is in the range
time
(d) potential energy of the system does not change in time (a) 1.0 × 109 K < T < 2.0 × 109 K
52. Cv and Cp denote the molar specific heat capacities of a gas (b) 2.0 × 109 K < T < 3.0 × 109 K
at constant volume and constant pressure, respectively. (c) 3.0 × 109 K < T < 4.0 × 109 K
Then (d) 4.0 × 109 K < T < 5.0 × 109 K
(a) Cp – Cv is larger for a diatomic ideal gas than for a 55. Results of calculations for four different designs of a fusion
monoatomic ideal gas
reactor using D-D reaction are given below. Which of these
(b) Cp + Cv is larger for a diatomic ideal gas than for a
is most promising based on Lawson criterion?
monatomic ideal gas
(c) Cp / Cv is larger for a diatomic ideal gas than for a (a) deuteron density = 2.0 × 1012 cm–3, confinement time
monoatomic ideal gas = 5.0 × 10–3 s
(d) Cp . Cv is larger for a diatomic ideal gas than for a (b) deuteron density = 8.0 × 1014 cm–3, confinement time
monoatomic ideal gas = 9.0 × 10–1 s
(c) deuteron density = 4.0 × 1023 cm–3, confinement time
SECTION – III = 1.0 × 10–11 s
Comprehension Type (d) deuteron density = 1.0 × 1024 cm–3, confinement time
This section contains 2 groups of questions. Each group has 3 = 4.0 × 10–12 s
multiple choice questions based on a paragraph. Each question Paragraph for Question Nos. 56 to 58
has 4 choices (a), (b), (c) and (d) for its answer, out of which ONLY When a particle is restricted to move along x - axis between x = 0
ONE is correct. and x = a, where a is of nanometer dimension, its energy can take
Paragraph for Question Nos. 53 to 55 only certain specific values. The allowed energies of the particle
Scientists are working hard to develop nuclear fusion reactor. moving in such a restricted region, correspond to the formation of
standing waves with nodes at its ends x = 0 and x = a . The
Nuclei of heavy hydrogen, 12 H, known as deuteron and denoted
wavelength of this standing wave is related to the linear momentum
by D, can be thought of as a candidate for fusion reactor. The p of the particle according to the de Broglie relation. The energy
D-D reaction is 2 2 3
He n energy. In the core of of the particle of mass m is related to its linear momentum as
1H 1H 2
fusion reactor, a gas of heavy hydrogen is fully ionized into p2
E . Thus, the energy of the particle can be denoted by a
deuteron nuclei and electrons. This collection of 12 H nuclei and 2m
electrons is known as plasma. The nuclei move randomly in the quantum number ‘n’ taking values1, 2, 3, ... (n = 1, called the ground
reactor core and occasionally come close enough for nuclear state) corresponding to the number of loops in the standing wave.
fusion to take place. Usually, the temperatures in the reactor core Use the model described above to answer the following three
are too high and no material wall can be used to confine the plasma. questions for a particle moving in the line x = 0 to x = a. Take
Special techniques areused which confine theplasma for a time t0 h = 6.6 × 10–34 Js and e = 1.6 × 10–19 C.
before the particles fly away from the core. If n is the density
56. The allowed energy for the particle for a particular value of
(number/volume) of deuterons, the product nt0 is called Lawson
number. In one of the criteria, a reactor is termed successful if n is proportional to
Lawson number is greater than 5 × 1014 s/cm3. (a) a–2 (b) a–3/2
It may be helpful to use the following: Boltzmann constant (c) a –1 (d) a2
57. If the mass of the particle is m = 1.0 × 10–30 kg and a = 6.6 nm,
e2
k = 8.6 × 10 – 5 eV/K; = 1.44 × 10–9 eVm. the energy of the particle in its ground state is closest to
4 0
(a) 0.8 meV (b) 8 meV
53. In the core of nuclear fusion reactor, the gas becomes plasma (c) 80 meV (d) 800 meV
because of
58. The speed of the particle, that can take discrete values, is
(a) strong nuclear force acting between the deuterons
(b) coulomb force acting between the deuterons proportional to
(c) coulomb force acting between deuteron-electron pairs (a) n–3/2 (b) n–1
(d) the high temperature maintained inside the reactor core (c) n1/2 (d) n
EBD_780
2009- 8 Target IIT-JEE

SECTION – IV
Matrix–Match Type
This section contains 2 questions. Each question contains statements given in two columns, which have to be matched. The statements
in Column-I are labelled A, B, C and D, while the statements in Column II are labelled p, q, r, s and t. Any given statement in
Column-I can have correct matching with ONE OR MORE statement(s) in Column-II. The appropriate bubbles corresponding to the
answers to these questions have to be darkened as illustrated in the following example :
If the correct matches are A—p, s and t; B— q and r; C—p and q; and D—s and t; then the correct darkeneing of bubbles will look like
the following.

p q r s t
A p q r s t
B p q r s t
C p q r s t
D p q r s t

59. Six point charges, each of the same magnitude q, are arranged in different manners as shown in Column-II. In each case, a point
M and line PQ passing through M are shown. Let E be the electric field and V be the electric potential at M (potential at infinity
is zero) due to the given charge distribution when it is at rest. Now, the whole system is set into rotation with a constant angular
velocity about the line PQ. Let B be the magnetic field at M and be the magnetic moment of the system in this condition.
Assume each rotating charge to the equivalent to a steady current.

Column-I Column-II
(A) E = 0 + – Q
(p) Charges are at the corners of a regular hexagon. M is at the centre
(B) V 0 – + of the hexagon. PQ is perpendicular to the plane of the hexagon.
M
(C) B=0 P+ –

(D) P
Charges are on a line perpendicular to PQ at equal intervals. M is
– + – + – + the mid-point between the two innermost charges.
(q)
M
Q

Charges are placed on two coplanar insulating rings at equal


Q intervals. M is the common centre of the rings. PQ is perpendicular
+ – +
to the plane of the rings.
(r) – M –

P +
Charges are placed at the corners of a rectangle of sides a and 2a
– + –
and at the mid points of the longer sides. M is at the centre of the
M rectangle. PQ is parallel to the longer sides.
(s) P Q
– + –

P
+ –
Charges are placed on two coplanar, identical insulating rings at
(t + + M– – equal intervals. M is the mid-point between the centres of the
rings. PQ is perpendicular to the line joining the centres and
Q coplanar to the rings.
2009 IIT-JEE Solved Paper 2009-9
60. Column-II shows five systems in which two objects are labelled as X and Y. Also in each case a point P is shown. Column-I gives
some statements about X and/or Y. Match these statements to the appropriate system(s) from Column-II.
Column-I Column-II
(A) The force exerted by X on (p)
Y has a magnitude Mg.
(B) The gravitational Y Block Y of mass M left on a fixed inclined
potential energy of X is X plane X, slides on it with a constant velocity.
continuously increasing.
(C) Mechanical energy of the
system X +Y is P
continuously decreasing.
(D) The torque of the weight P Two ring magnets Y and Z, each of mass M,
of Y about point P is zero. are kept in frictionless vertical plastic stand
so that they repel each other. Y rests on the
Z base X and Z hangs in air in equilibrium. P is
(q) the topmost point of the stand on the common
Y axis of the two rings. The whole system is in
a lift that is going up with a constant velocity.
X

A pulley Y of mass m0 is fixed to a table


P through a clamp X. A block of mass M hangs
Y from a string that goes over the pulley and is
(r) fixed at point P of the table. The whole system
X is kept in a lift that is going down with a
constant velocity.

A sphere Y of mass M is put in a non-viscous


liquid X kept in a container at rest. The sphere
Y is released and it moves down in the liquid.
(s)

X
P

A sphere Y of mass M is falling with its


terminal velocity in a viscous liquid X kept in
Y a container.
(t)

X
P
EBD_780
2009- 10 Target IIT-JEE

PAPER -2
CHEMISTRY
5. In the reaction 2X + B2H6 [BH2 (X)2] + [BH4]–
SECTION – I the amine(s) X is (are)
Single Correct Choice Type (a) NH3 (b) CH3 NH2
(c) (CH3)2 NH (d) (CH3)3 N
This section contains 4 multiple choice questions. Each question
6. The nitrogen oxide(s) that contain(s) N-N bond(s) is(are)
has 4 choices (a), (b), (c) and (d) for its answer, out of which ONLY (a) N2O (b) N2O3
ONE is correct. (c) N2O4 (d) N2O5
1. The spin only magnetic moment value (in Bohr magneton
7. For the reduction of NO3– ion in an aqueous solution, E 0 is
units) of Cr(CO)6 is
+ 0.96V. Values of E0 for some metal ions are given below
(a) 0 (b) 2.84
V2+ (aq) + 2e– V E0 = – 1.19 V
(c) 4.90 (d) 5.92 Fe3+ (aq) + 3e– Fe E0 = – 0.04 V
2. In the following carbocation, H/CH3 that is most likely to Au3+ (aq) + 3e– Au E0 = + 1.40 V
migrate to the positively charged carbon is Hg2+ (aq) + 2e– Hg E0 = + 0.86 V
H H The pair(s) of metals that is(are) oxidized by NO3– in
1 2 + 4 5 aqueous solution is(are)
H3C — C —3C — C — CH3 (a) V and Hg (b) Hg and Fe
(c) Fe and Au (d) Fe and V
HO H CH3 8. The correct statement(s) about the following sugars X and
Y is(are)
(a) CH3 at C-4 (b) H at C-4
(c) CH3 at C-2 (d) H at C-2 CH2OH
3. For a first order reaction A P, the temperature (T) dependent O HOH2C O
H H H H
rate constant (k) was found to follow the equation
OH H O H HO CH2OH
HO
1
log k = – (2000) 6.0 . The pre-exponential factor A and
T H OH OH H
the activation energy Ea, respectively, are X
(a) 1.0 × 106 s–1 and 9.2 kJ mol–1 CH2OH
(b) 6.0 s–1 and 16.6 kJ mol–1 H O H
CH2OH H
(c) 1.0 × 106 s–1 and 16.6 kJ mol–1 H HO OH
(d) 1.0 × 106 s–1 and 38.3 kJ mol–1 H H O O
H H H H
4. The correct stability order of the following resonance HO OH
structures is H OH
+ – + – (a) X is a reducing sugar and Y is a non-reducing sugar
H2C = N = N H2C – N = N (b) X is a non-reducing sugar and Y is a reducing sugar
(I) (II) (c) The glucosidic linkages in X and Y are and ,
respectively
– + – + (d) The glucosidic linkages in X and Y are and ,
H 2C – N = N H2C – N = N
(III) (IV) respectively
9.. Among the following the state function(s) is (are)
(a) (I) > (II) > (IV) > (III) (a) Internal energy
(b) (I) > (III) > (II) > (IV) (b) Irreversible expansion work
(c) (II) > (I) > (III) > (IV) (c) Reversible expansion work
(d) (III) > (I) > (IV) > (II) (d) Molar enthalpy
SECTION – III
SECTION – II Matrix - Match Type
Multiple Correct Choice Type This section contains 2 questions. Each question contains
This section contains 5 multiple choice questions. Each question statements given in two columns, which have to be matched. The
has 4 choices (a), (b), (c), and (d) for its answer, out of which ONE statements in Column I are labelled , B, C, and D, while the
OR MORE is/are correct. statements in Column II are labelled p, q, r, s and t. Any given
statement in Column I can have correct matching with ONE OR
2009 IIT-JEE Solved Paper 11
2009-

MORE statement(s) in Column II. The appropriate bubbles 12. The number of water molecule(s) directly bonded to the
corresponding to the answers to these questions have to be metal centre in CuSO4.5H2O is
darkened as illustrated in the following example: 13. The dissociation constant of a substituted benzoic acid at
If the correct matches are A — p, s p q r s t
25°C is 1.0 × 10–4. The pH of a 0.01 M solution of its sodium
and t; B—q and r; C—p and q; and A p q r s t
D— s and t; then the correct salt is
darkening of bubbles will look like B p q r s t 14. The oxidation number of Mn in the product of alkaline
as shown : C p q r s t oxidative fusion of MnO2 is
D p q r s t 15. At 400 K, the root mean square (rms) speed of a gas X
(molecular weight = 40) is equal to the most probable speed
10. Match each of the reactions given in Column I with the of gas Y at 60 K. The molecular weight of the gas Y is
corresponding product(s) given in Column II. 16. The total number of cyclic structural as well as stereo
Column I Column II isomers possible for a compound with the molecular formula
(A) Cu + dil HNO3 (p) NO C5H10 is
(B) Cu + conc HNO3 (q) NO 2 17. The coordination number of Al in the crystalline state of
(C) Zn + dil HNO3 (r) N 2O AlCl3 is
(D) Zn + conc HNO3 (s) Cu(NO3 ) 2 18. In a constant volume calorimeter, 3.5 g of a gas with molecular
weight 28 was burnt in excess oxygen at 298.0 K. The
(t) Zn(NO3 )2
temperature of the calorimeter was found to increase from
11. Match each of the compounds given in Column-I with the 298.0 K to 298.45 K due to the combustion process. Given
reaction(s), that they can undergo, given in Column-II.
that the heat capacity of the calorimeter is 2.5 kJ K–1, the
Column-I Column-II
numerical value for the enthalpy of combustion of the gas
Br in kJ mol–1 is
(A) (p) Nucleophilic substitution 19. The total number of and particles emitted in the nuclear
O 238 214
reaction 92
U 82
Pb is
OH
(B) (q) Elimination MATHEMATICS
CHO SECTION – I
(C) (r) Nucleophilic addition
OH Single Correct Choice Type
This section contains 4 multiple choice questions. Each question
Br has 4 choices (a), (b), (c) and (d) for its answer, out of which ONLY
(D) (s) Esterification with acetic
ONE is correct.
NO2 anhydride
20. A line with positive direction cosines passes through the
(t) Dehydrogenation point P(2, –1, 2) and makes equal angles with the coordinate
SECTION – IV axes. The line meets the plane
Integer Answer Type 2x + y + z = 9
This section contains 8 X Y Z W at point Q. The length of the line segment PQ equals
questions. The answer to each of 0 0 0 0
(a) 1 (b) 2
the questions is a single-digit 1 1 1 1
integer, ranging from 0 to 9. The (c) 3 (d) 2
2 2 2 2
appropriate bubbles below the 21. The normal at a point P on the ellipse x2 + 4y2 = 16 meets the
respective question numbers in 3 3 3 3
x - axis at Q. If M is the mid point of the line segment PQ,
the ORS have to be darkened. 4 4 4 4 then the locus of M intersects the latus rectums of the
For example, if the correct 5 5 5 5 given ellipse at the points
answers to question numbers X,
6 6 6 6
Y, Z and W (say) are 6, 0, 9 and 2, 3 5 2 3 5 19
(a) , (b) ,
7 7 7 7 2 7 2 4
respectively, then the correct
darkening of bubbles will look 8 8 8 8
like as shown : 9 9 9 9
1 4 3
(c) 2 3, (d) 2 3,
7 7
EBD_780
2009- 12 Target IIT-JEE
22. In the sum of first n terms of an A.P. is cn2, then the sum of 28. An ellipse intersects the hyperbola 2x2 – 2y2 = 1 orthogonally.
squares of these n terms is The eccentricity of the ellipse is reciprocal of that of the
2 2 2 2 hyperbola. If the axes of the ellipse are along the coordinate
n (4n –1) c n (4n +1) c
(a) (b) axes, then
6 3 (a) equation of ellipse is x2 + 2y2 = 2
n (4n2 –1) c 2 n (4n2 +1) c 2 (b) the foci of ellipse are ( 1, 0)
(c) (d)
3 6 (c) equation of ellipse is x2 + 2y2 = 4
23. The locus of the orthocentre of the triangle formed by the
(d) the foci of ellipse are ( 2, 0)
lines
(1+ p) x – py + p (1+ p) = 0, SECTION – III
(1+ q) x – qy + q (1+ q) = 0,
and y = 0, where p q, is Matrix - Match Type
(a) a hyperbola (b) a parabola This section contains 2 questions. Each question contains
(c) an ellipse (d) a straight line statements given in two columns, which have to be matched. The
statements in Column-I are labelled A, B, C and D, while the
SECTION – II statements in Column-II are labelled p, q, r, s and t. Any given
Multiple Correct Choice Type statement in Column-I can have correct matching with ONE OR
This section contains 5 multiple choice questions. Each question MORE statement(s) in Column-II. The appropriate bubbles
has 4 choices (a), (b), (c) and (d) for its answer, out of which ONE corresponding to the answers to these questions have to be
OR MORE is/are correct. darkened as illustrated in the following example:
24. For the function If the correct matches are A—p, s and t; B—q and r; C—p and q;
1 and D—s and t; then the correct darkening of bubbles will look
, x 1,
f (x) = x cos like the following.
x
(a) for at least one x in the interval [1, ), f (x + 2) – f (x) < 2 p q r s t
(b) lim f ' ( x ) 1 A p q r s t
x
B p q r s t
(c) for all x in the interval [1, ), f (x + 2) – f (x) > 2
(d) f ' (x) is strictly decreasing in the interval [1, ) C p q r s t
25. The tangent PT and the normal PN to the parabola y2 = 4ax
D p q r s t
at a point P on it meet its axis at points T and N, respectively.
The locus of the centroid of the triangle PTN is a parabola
whose 29. Match the statements / expressions given in Column-I with
the values given in Column-II.
2a
(a) vertex is ,0 (b) directrix is x = 0 Column-I Column-II
3
2a (A) Root(s) of the equation (p)
6
(c) latus rectum is (d) focus is (a, 0)
3 2 sin2 + sin2 2 = 2

sin nx (B) Points of discontinuity of the (q)


26. If I n dx n = 0, 1, 2, ..., 4
x
(1+ ) sin x
6x 3x
then function f ( x) cos ,
10 where [y] denotes the largest
(a) In In 2 (b) I 2m 1 10 integer less than or equal to y
m=1

10
(C) Volume of the parallelopiped with (r)
3
(c) I 2m 0 (d) In = In + 1 its edges represented by the
m=1
vectors
27. For 0 , the solution (s) of iˆ ˆj, iˆ 2 ˆj and iˆ ˆj kˆ
2
6
(D) Angle between vector a and b (s)
(m ) m 2
cosec cosec 4 2
m=1
4 4 where a , b and c are unit
vectors satisfying
is (are)
a b 3c 0
(a) (b) (c) (d) (t)
4 6 12 12
2009 IIT-JEE Solved Paper 2009-13
30. Match the statements/expressions given in Column-I with 32. Let ABC and ABC' be two non-congruent triangles with
the values given in Column-II.
sides AB = 4, AC = AC' = 2 2 and angle B = 30°. The
Column-I Column-II
(A) The number of solutions of (p) 1 absolute value of the difference between the areas of these
the equation triangles is
x esin x – cos x = 0 33. Let f: R R be a continuous function which satisfies
x
in the interval f ( x) f (t) dt.
0
(B) Value(s) of k for which the planes (q) 2
k x + 4y + z = 0, 4x + k y + 2z = 0 Then the value of f (ln 5) is
and 2x + 2y + z = 0 intersect in a 34. Let (x, y, z) be points with integer coordinates satisfying the
straight line system of homogeneous equations :
(C) Value(s) of k for which (r) 3 3x – y – z = 0
| x – 1 | + | x – 2 | + | x + 1 | + | x + 2 | = 4k – 3x + z = 0
has integer solution(s) – 3x + 2y + z = 0
(D) If y' = y + 1 and y (0) = 1, (s) 4 Then the number of such points for which x2 + y2 + z2 100
then value(s) of y (1n 2) is
(t) 5
x
3 and g (x) = f –1 (x), then the
35. If the function f ( x) x e2
SECTION – IV value of g' (1) is
Integer Answer Type 36. The smallest value of k, for which both the roots of the
This section contains 8 questions. The answer to each of the equation
questions is a single-digit integer, ranging from 0 to 9. The x2 – 8kx + 16 (k2 – k + 1) = 0
appropriate bubbles below the respective question numbers in are real, distinct and have values at least 4, is
the ORS have to be darkened. For example, if the correct answers 37. The maximum value of the function
to question numbers X, Y, Z and W (say) are 6, 0, 9 and 2, f (x) = 2x3 – 15x2 + 36x – 48 on the set
respectively, then the correct darkening of bubbles will look like
A= {x | x2 + 20 9x}is
the following :
38. Let p(x) be a polynomial of degree 4 having extremum at
X Y Z W p ( x)
x = 1, 2 and lim 1 2.
0 0 0 0 x 0 x2
Then the value of p (2) is
1 1 1 1

2 2 2 2
PHYSICS
3 3 3 3
SECTION – I
4 4 4 4 Single Correct Choice Type
5 5 5 5 This section contains 4 multiple choice questions. Each question
has 4 choices (a), (b), (c) and (d) for its answer, out of which ONLY
6 6 6 6 ONE is correct.
39. A piece of wire is bent in the shape of a parabola y = kx2
7 7 7 7
(y-axis vertical) with a bead of mass m on it. The bead can
8 8 8 8 slide on the wire without friction. It stays at the lowest point
of the parabola when the wire is at rest. The wire is now
9 9 9 9 accelerated parallel to the x-axis with a constant acceleration
a. The distance of the new equilibrium position of the bead,
where the bead can stay at rest with respect to the wire,
31. The centres of two circles C1 and C2 each of unit radius are from the y-axis is
at a distance of 6 units from each other. Let P be the mid
point of the line segement joining the centres of C1 and C2 a a
(a) (b)
and C be a circle touching circles C1 and C2 externally. If a gk 2 gk
common tangent to C1 and C passing through P is also a
common tangent to C2 and C, then the radius of the circle C 2a a
(c) (d)
is gk 4 gk
EBD_780
2009- 14 Target IIT-JEE
40. Photoelectric effect experiments are performed using three
different metal plates p, q and r having work functions p =
SECTION – II
2.0 eV, q = 2.5 eV and r = 3.0 eV, respectively. A light beam Multiple Correct Choice Type
containing wavelengths of 550 nm, 450 nm and 350 nm with This section contains 5 multiple choice questions. Each question
equal intensities illuminates each of the plates. The correct
has 4 choices (a), (b), (c) and (d) for its answer, out of which ONE
I-V graph for the experiment is [Take hc = 1240 eV nm]
OR MORE is/are correct.
I
I 43. The figure shows the P-V plot of an ideal gas taken through
p a cycle ABCDA. The part ABC is a semi-circle and CDA is
q p half of an ellipse. Then,
(a) (b)
r q
r
V P
V
3
A
I
I
r 2
D B
q
(c) (d)
p
r q 1
p C
V V
0 1 2 3 V
41. The mass M shown in the figure oscillates in simple harmonic
motion with amplitude A. The amplitude of the point P is

(a) the process during the path A B is isothermal


k1 k2 (b) heat flows out of the gas during the path B C D
M (c) work done during the path A B C is zero
P (d) positive work is done by the gas in the cycle ABCDA
44. Under the influence of the Coulomb field of charge + Q, a
k1A k2 A charge –q is moving around it in an elliptical orbit. Find out
(a) k2 (b) k1 the correct statement(s).
(a) The angular momentum of the charge – q is constant
k1A k2 A
(c) (d) k k (b) The linear momentum of the charge – q is constant
k1 k2 1 2
(c) The angular velocity of the charge –q is constant
42. A uniform rod of length L and mass M is pivoted at the (d) The linear speed of the charge –q is constant
centre. Its two ends are attached to two springs of equal
spring constants k. The springs are fixed to rigid supports 45. Two metallic rings A and B, identical in shape and size but
as shown in the figure, and the rod is free to oscillate in the having different resistivities A and B, are kept on top of
horizontal plane. The rod is gently pushed through a small two identical solenoids as shown in the figure. When current
angle in one direction and released. The frequency of I is switched on in both the solenoids in identical manner,
oscillation is the rings A and B jump to heights hA and hB, respectively,
with hA > hB. The possible relation(s) between their
resistivities and their masses mA and mB is(are)

A B

1 2k 1 k
(a) (b)
2 M 2 M

1 6k 1 24k (a) A > B and mA = mB (b) A< B and mA = mB


(c) (d)
2 M 2 M (c) A > B and mA > mB (d) A < B and mA < mB
2009 IIT-JEE Solved Paper 2009- 15
46. A sphere is rolling without slipping on a fixed horizontal 48. Column-II gives certain systems undergoing a process.
plane surface. In the figure, A is the point of contact, B is the Column-I suggests changes in some of the parameters
centre of the sphere and C is its topmost point. Then,
related to the system. Match the statements in Column-I to
C the approapriate process(es) from Column-II.
Column-I Column-II
(A) The energy of the (p) System : A capacitor,
B
system is increased initially uncharged
(B) Mechanical energy is Process : It is connected
A provided to the system, to a battery
which is converted into (q) System : A gas in an adia-
(a) VC VA 2 VB VC
energy of random motion batic container fitted with
(b) VC VB VB VA of its parts an adiabatic piston
(C) Internal energy of the Process: The gas is com-
(c) VC VA 2 VB VC
system is converted into pressed by pushing the
(d) VC VA 4 VB its mechanical energy piston
47. A student performed the experiment to measure the speed (D) Mass of the system is (r) System: A gas in a rigid
of sound in air using resonance air-column method. Two decreased container
resonances in the air-column were obtained by lowering the
Process : The gas gets
water level. The resonance with the shorter air-column is
the first resonance and that with the longer air-column is cooled due to colder
the second resonance. Then, atmosphere surrounding
(a) the intensity of the sound heard at the first resonance it
was more than that at the second resonance (s) System: A heavy
(b) the prongs of the tuning fork were kept in a horizontal nucleus, initially at rest
plane above the resonance tube
Process: The nucleus fis-
(c) the amplitude of vibration of the ends of the prongs is
typically around 1 cm sions into two fragments
(d) the length of the air-column at the first resonance was of nearly equal masses
somewhat shorter than 1/4th of the wavelength of the and some neutrons are
sound in air emitted
SECTION – III (t) System: A resistive wire
Matrix - Match Type loop
This section contains 2 questions. Each question contains Process: The loop is
statements given in two columns, which have to be matched. The placed in a time varying
statements in Column-I are labelled A, B, C and D, while the magnetic field perpendi-
statements in Column-II are labelled p, q, r, s and t. Any given
statement in Column-I can havecorrect matching with ONE OR cular to its plane
MORE statement(s) in Column-II. The appropriate bubbles 49. Column-I shows four situations of standard Young’s double
corresponding to the answers to these questions have to be slit arrangement with the screen placed far away from the
darkened as illustrated in the following example : slits S1 and S2 . In each of these cases S1 P0 = S2 P0 ,
If the correct matches are A–p, s and t; B–q and r; C–p and q; and S1P1-S2P1= and S1P2 – S2 P2= /3, where is the
D–s and t; then the correct darkening of bubbles will look like
wavelength of the light used. In the cases B, C and D, a
the following.
transparent sheet of refractive index and thickness t is
p q r s t pasted on slit S2. The thicknesses of the sheets are different
A p q r s t in different cases. The phase difference between the light
B p q r s t waves reaching a point P on the screen from the two slits is
C p q r s t denoted by (P) and the intensity by I (P). Match each
situation given in Column-I with the statetment(s) in
D p q r s t
Column-II valid for that situation.
EBD_780
2009- 16 Target IIT-JEE
Column-I Column-II 51. Two soap bubbles A and B are kept in a closed chamber
(A) P2 (p) (P0) = 0 where the air is maintained at pressure 8 N/m2. The radii of
S2 P1 bubbles A and B are 2 cm and 4 cm, respectively. Surface
P0 tension of the soap-water used to make bubbles is 0.04 N/m.
S1 (q) (P1) = 0
Find the ratio nB /nA, where nA and nB are the number of
moles of air in bubbles A and B, respectively. [Neglect the
effect of gravity.]
P2 52. A steady current I goes through a wire loop PQR having
S2 P1 r (P1) = 0 shape of a right angle triangle with PQ = 3x, PR = 4x and
(B) ( – 1) t = /4 P0
QR = 5x. If the magnitude of the magnetic field at P due to
S1
0I
(s) I(P0) > I (P1) this loop is k , find the value of k.
48 x
P2 53. A cylindrical vessel of height 500 mm has an orifice (small
S2
P1 hole) at its bottom. The orifice is initially closed and water is
(C) ( – 1) t = /2 P0 (t) I(P2) > I (P1) filled in it up to height H. Now the top is completely sealed
S1 with a cap and the orifice at the bottom is opened. Some
water comes out from the orifice and the water level in the
vessel becomes steady with height of water column being
P2 200 mm. Find the fall in height (in mm) of water level due to
S2 P1 opening of the orifice.
P0 [Take atmospheric pressure = 1.0 × 105 N/m2, density of
(D) ( – 1) t = 3 /4
S1 water = 1000 kg/m3\ and g = 10 m/s2. Neglect any effect of
surface tension.]
54. A 20 cm long string, having a mass of 1.0 g, is fixed at both
SECTION – IV the ends. The tension in the string is 0.5 N. The string is set
Integer Answer Type into vibrations using an external vibrator of frequency 100
This section contains 8 X Y Z W Hz. Find the separation (in cm) between the successive
questions. The answer to nodes on the string.
0 0 0 0
each of the questions is a 55. A solid sphere of radius R has a charge Q distributed in its
single-digit integer, ranging 1 1 1 1 volumewith a charge density = r a, where and a are
from 0 to 9. The appropriate constants and r is the distance from its centre.
bubbles below the respective 2 2 2 2
question numbers in the R 1
3 3 3 3 If the electric field at r is times that at r = R, find the
ORS have to be darkened. 2 8
For example, if the correct 4 4 4 4 value of a.
answers to question numbers 56. A metal rod AB of length 10x has its one end A in ice at 0.°C,
X, Y, Z and W (say) are 6, 0, 9 5 5 5 5 and the other end B in water at 100 °C. If a point P on the rod
and 2, respectively, then the is maintained at 400 °C, then it is found that equal amounts
correct darkening of bubbles 6 6 6 6
of water and ice evaporate and melt per unit time. The latent
will look like as shown : 7 7 7 7 heat of evaporation of water is 540 cal/g and latent heat of
melting of ice is 80 cal/g. If the point P is at a distance of
8 8 8 8 x from the ice end A, find the value of
9 9 9 9 Neglect any heat loss to the surrounding.]
57. Three objects A, B and C are kept in a straight line on a
50. A light inextensible string that frictionless horizontal surface. These have masses m, 2m
goes over a smooth fixed pulley and m, respectively. The object A moves towards B with a
as shown in the figure speed 9 m/s and makes an elastic collision with it. There
connects two blocks of masses after, B makes completely inelastic collision with C. All
0.36 kg and 0.72 kg. Taking g = motions occur on the same straight line. Find the final speed
10 m/s2, find the work done (in (in m/s) of the object C.
joules) by the string on the
block of mass 0.36 kg during
the first second after the m 2m m
system is released from rest.
A B C
EXPLANATORY NOTES – IIT-JEE 2009

PAPER - 1
1. (c) As Sb2S3 is a negative sol, so Al2(SO4)3 will be the
10. (a,b) 4Na O 2 (limited) 2Na 2O
most effective coagulant due to higher positive charge
on Al (Al3+) – Hardy-Schulze rule.
2Na O 2 (excess) Na 2 O 2
2. (b) Average atomic mass of Fe
11. (b,c) Frenkel defect is a dislocation effect, observed when
(54 5) (56 90) (57 5) the size of the cation and anion differ largely. F-center
55.95
100 is created when an anion is lost from the lattice and
3. (a) Carboxylic acid is stronger acid than phenol. The vacancy is filled by trapping of an electron. Schottky
presence of electron withdrawing group (e.g. Cl) defect changes the density of a crystalline solid.
increases acidic strength, while presence of electron
* *
donating group (e.g. CH3) decreases acidic strength. 12. (a,d) CH3 C H CH CH C H CH 3
4. (b) – CN has highest priority. Further the sum of locants is | |
7 in (b) and 9 in (d). OH OH
5. (b) Correction factor for attractive force for n moles of real Stereoisomer Configuration
gas is given by the term mentioned in (b). I d cis d
6. (d) Nylon and cellulose, both have intermolecular II l cis l
hydrogen bonding, polyvinyl chloride has dipole- III d cis l
dipole interaction, while natural rubber has van der IV d trans d
Waal forces which are weakest. V l trans l
in presence of N VI d trans l
7. (b) P4 3O 2 2 P4 O 6
Enantiomers I and II; IV and V
Here N2 acts as a diluent and thus retards further cis trans
oxidation. Reaction of P4 under other three conditions. Diastereomers I (or II), III (or IV), V and VI
2O Meso III and IV
(a) P4 3O 2 P4 O 6 2 P4 O10 For 13-15
(c) In moist air, P4O6 is hydrolysed to form H3PO3 Reaction of Y indicates that it is Fe3+ salt.
P4 O 6 6H 2O 4H 3PO3 4Fe3 3[Fe(CN)6 ]4 Fe 4 [Fe(CN) 6 ]3
(d) In presence of NaOH, Y hexacyanoferrate (II) Intense blue ppt. soluble in
or ferrocyanide excess of ferrocyanide
P4 3OH 3H 2O PH 3 3H 2 PO 2
Fe3 [Fe(CN)6 ]3 Fe[Fe(CN) 6 ]
8. (a) PN 2 H N2 Y hexacyanoferrate (III) Brown colour
or ferricyanide
0.8 × 5 = 1 × 105 × N2 Further since the product formed (methylene blue) has
sulphur in its structure, it should be supplied by the
N2 = 4 × 10–5 compound X which is thus Na2S.
Solubility in 10 moles = 4 × 10–4. Na 2S 2H H 2S 2Na
Cl Cl
FeCl3 H 2S FeCl2 2HCl S
Cl
Me2N NMe2
9. (c,d) en Pt en Pt en + S +
NH2 H2N
en Cl +
cis trans Me2N S NMe2
H
H3N Cl H3N Cl N
Methylene blue
Pt Pt
Thus
H3N Cl Cl NH3 13. (d) 14. (c) 15. (b)
cis trans
EBD_780
2009- 18 Target IIT-JEE
For 16-18 19. A-p, r, t; B-s, t; C-p, q, r; D-p, r, s
Iodoform test of compound P points out that P has – COCH3
group which shows that it may be either option (a) or (b) of (A) B2 1s 2 , *1s 2 , 2s 2 , *2s 2 , 2 p1y 2 p1z
Q. 16. Further since the dicarbonyl compound R has at least
one -H atom w.r.t to one of the carbonyl groups which is Bond order = 1 Paramagnetic
possible when R is produced from (b) of Q. 18; (a) option of
(B) N2 1s 2 , *1s 2 , 2s 2 , * 2s 2 , 2 p x2 , 2 p 2y 2 p z2
Q. 16 will give dicarbonyl compound having two –CHO,
none of which has -H atom. Bond order = 3 Diamagnetic
Me Me
(i) MeMgBr
Me (C) O2– 1s 2 , *1s 2 ; 2s 2 . * 2s 2 ; 2 px2 , 2 p 2y
(ii) H , H 2O
O Me HO Me
2 pz2 , *2 p 2y * 2 p1z
(B)/P
Bond order = 1.5 Paramagnetic
H 2SO 4
Me
heat Me (D) O2 1s 2 , *1s 2 , 2s 2 , *2s 2 , 2 px2 , 2 p 2y
+ Me
2 pz2 , *2 p1y * 2 p1z
O
Bond order = 2 Paramagnetic
Me O3 Zn H
H 2O 20. A-p, q, s, t; B-s, t; C-p; D-r
COCH3
Me Me Me
Me (A) CH3CH 2CH 2 CN
Pd C / H 2 or
Q SnCl 2 / HCl or
R* DIBAL H

O CH3CH 2CH 2 CH 2 NH 2
OH H
– OH
Intramolecular COCH2 CH3CH 2 CH 2CN CH3CH 2 CH 2 COO
aldol condensation H 2O
Me Me
Pd C / H 2 or
(B) CH 3CH 2 OCOCH 3 DIBAL H
CH 3CH 2 OH
OH

heat OH
CH3CH 2OCOCH3
H 2O
O O
Me Me Me Me
S CH3CH 2 OH CH3COO
* Structure of R would be R' when P is (A)
Pd C / H 2
(C) CH 3 CH CHCH 2 OH

CH3CH 2 CH 2CH 2 OH
O Me
(A)
Me Me
CHCl 3
Q' (D) CH 3 CH 2 CH 2 CH 2 NH 2 KOH

CH 3 CH 2 CH 2 CH 2 NC
CHO Isonitrile (foul smelling)
O3 Zn
H 2O CHO 21. (c) Given that f is a non negative function defined on
Me Me [0, 1] and
R'
x x

16. (b) 17. (a) 18. (b) 1 ( f (t )) 2 dt f (t ) dt , 0 x 1


0 0
2009 IIT-JEE Solved Paper 2009- 19
Differentiating both sides with respect to x, we get Now 0 a b || c d
2
1 [ f ( x )] f ( x) Let a b (c d )
2 2
1 [ f ( x )] [ f ( x )] (a b).c (c d ).c 0
2 2
[ f ( x )] 1 [ f ( x )] and (a b).d (c d ).d 0
d a, b, c are coplanar and a, b, d are coplanar
f ( x) 1 [ f ( x )]2
dx
a, b, c, d are coplanar
d f ( x) Also 90
dx
1 [ f ( x )]2 a b and 90
Integrating both sides with respect to x, we get
c d
d f ( x)
dx 1
But angle between a and c is /3 ( a. c = )
1 [ f ( x )]2 2

sin 1 f ( x) x C So, angle between b and d should also be / 3.


Given that f ( 0 ) = 0 Hence b and d are non parallel.
C=0 24. (c) We have to form 7 digit numbers, using the digits 1,
2 and 3 only, such that the sum of the digits in a
Hence f (x) = sin x
number = 10.
But as f ( x ) is a non negative function on [0, 1] This can be done by taking 2, 2, 2, 1, 1, 1, 1, or by taking
f ( x ) sin x. 2, 3, 1, 1, 1, 1, 1.
Now sin x x, x 0 7! 7!
Number of ways 77.
3!4! 5!
1 1 1 1
f and f . 25. (b) Tangents PA and PB are drawn from the point P (1, 3)
2 2 3 3 2
to circle x y 2 6 x 4 y 11 0 with centre C (3, 2)
22. (a) Given that P (3, 2, 6) is a point in space and Q is a point
on line A
r (iˆ ˆj 2kˆ) ( 3iˆ ˆj 5kˆ)
P (1,8) C
x 1 y 1 z 2 (3,2)
or
3 1 5
Let coordinates of Q be ( 3 1, 1,5 2) B

d.r’s of PQ 3 2, 3, 5 4 Clearly the circumcircle of PAB will pass through C


As PQ is parallel to the plane x – 4 y + 3 z = 1 and as A 90 , PC must be a diameter of the circle.
Equation of required circle is
1.( 3 2) 4.( 3) 3.(5 4) 0 ( x 1)( x 3) ( y 8)( y 2) 0
1
8 2 or x2 y 2 4 x 10 y 19 0
4 Alternative :
23. (c) a, b, c and d are unit vectors,
Let a b (sin )n1 and c d (sin )n2 A
then (a b).(c d ) 1
(sin )n1.(sin )n2 1 P (1,8)
sin sin n1.n2 1
B
sin sin cos 1
where is the angle between n1 and n2 . Equation of chord of contact i.e. AB is
and x.1 y.8 3( x 1) 2( y 8) 11 0
, 0
2 2 or x 3 y 15 0
EBD_780
2009- 20 Target IIT-JEE
Now equation of circle passing through intersection
x y
Equation of AB is 1
point of circle x 2 y 2
6 x 4 y 11 0 and of line 3 1
x 3 y 15 0 is given by S L 0 or x + 3y – 3 = 0 (1)
Also auxillary circle of given ellipse is
i.e. ( x 2 y 2 6 x 4 y 11) ( x 3 y 15) 0
x2 y 2 9 (2)
As this circle passes through P (1, 8) also, we get
Solving equation (1) and (2), we get the point M where
( 1 + 64 – 6 – 32 – 11 ) + (1 – 24 + 15 ) = 0
line AB meets the auxillary circle.
2 n
The required circle is Putting x = 3 – 3y from eq (1) in eq n (2)
( x2 y 2 6 x 4 y 11) 2( x 3 y 15) 0 we get (3 3 y )2 y2 9

or x 2 y2 4 x 10 y 19 0 9 18 y 9 y 2 y2 9
26. (d) z cos i sin 9
2m 1 2m 1 10 y 2 18 y 0 y 0,
z (cos i sin ) 5
cos(2m 1) i sin(2m 1) 12
x 3,
5
using De Moivre's theorem
(cos i sin )n cos n i sin n 12 9
Clearly M ,
5 5
Im( z 2 m 1 ) sin (2 m 1)
15 15 0 0 1
2m 1 1 27
Im( z ) sin(2 m 1) Area of OAM 3 0 1
2 10
m 1 m 1 12 9
1
sin sin 3 sin 5 ...... upto15 terms 5 5
Alternative:
2
sin 15 .sin[ 14 ] 2
2 y2
The given ellipse is x 1, so that
sin 32 12
OA = 3, OB = 1
Using sin sin( ) sin( 2 ) ....n terms
sin(n / 2).sin[ (n 1) / 2]
sin( / 2)

sin15 .sin15 sin 30 .sin 30 M


sin sin 2 B
1
4 sin 2
O A X
x2 y2
27. (d) The given ellipse is x 2 9 y 2 9 or 1
32 12

M Let OAM OMA ( OA = OM )

B 1
In OAB, tan
3
Also in OAM , AOM 180 2
O A X So that sin AOM sin(180 2 ) sin 2

1
2
2 tan 3 3
1 tan 2 1 5
1
So, that A( 3,0) and B (0,1) 9
2009 IIT-JEE Solved Paper 21
2009-

1 30. (b, c) In ABC , given that


Ar( OAM ) OA2 sin AOM
2 A
cos B cos C 4 sin 2
2
1 2
r sin for isosceles triangle B C B C A
2 2 cos cos 4sin 2 0
2 2 2
1 2 3 27 A B C A
3 2sin cos 2sin 0
2 5 10
2 2 2
28. (a) Given z = x + iy where x and y are integer
A B C B C
Also zz 3 zz 3 350 sin 0 or cos 2cos 0
2 2 2
| z |2 ( z 2 z2 ) 350 A
But in a triangle sin 0
2 2 2 2 2
(x y ) (x y ) 175
B C B C
cos 2 cos 0
( x 2 y 2 )( x 2 y 2 ) 25 7 ...(i) 2 2
or (x2 + y2) (x2 – y2) = 35 × 5 ...(ii) B C
x and y are integers, cos
2 1
2
x2 y2 25 and x y2 7 [From eq (i)] B C 2
cos
2 2
x 2 16 and y 9 x 4 and y 3
Applying componendo and dividendo, we get
Vertices of rectangle are
(4, 3) , (4, – 3), (– 4, – 3) , (– 4, 3). B C B C
cos cos
So, area of rectangle = 8 × 6 = 48 sq. units 2 2 1 2
3
Now from eq. (ii) B C B C 1 2
or x2 + y2 = 35 and x2 – y2 = 5 cos cos
2 2
x2 = 20, which is not possible for any integral value
of x B C
2 cos cos
2 2 3
x2 B C
a a2 x2 2sin sin
29. (a,c) Given that L lim 4 ,a 0 2 2
x 0 x4
B C 1
and L is finite. tan tan
2 2 3
x x
( s a)( s c) ( s a)( s b) 1
a 2
x 2 2
Now L lim s ( s b) s ( s c) 3
x 0 4 x3
s a 1
(Using L’Hospital’s rule) or 2 s 3a
s 3
1 1 a + b + c = 3a or b + c = 2a
a x 2 2 2 i.e. AC+ AB = constant
lim ( Base BC = a is given to be constant)
x 0 4 x2
A moves on an ellipse.
L is finite, limiting value of numerator should be
31. (b, c, d) The area bounded by the curve y e x and lines
1 1
zero which is so when 0 x = 0 and y = e is as shown in the graph.
a2 2
x
Y y=e
i.e. a = 2 ( a > 0)
Applying L ‘Hospital’s rule again, we get )
0,e
x B( y=e
(a 2 x 2 )3 / 2 1 C (1,e)
L lim lim
x 0 8x x 0 8(a 2 x 2 )3 / 2
A (0,1)
1 1
(using a = 2) X
8 a3 8 8
1
64
EBD_780
2009- 22 Target IIT-JEE
Required area 34. (b) The given system will have unique solution if | A | 0
1 1 which is so for the matrices.
(e e x )dx [ex ]10 e x dx
0 0
1 0 1 1 1 0
1
x 0 0 1 and 1 0 1 ;
e e dx. 1
0
1 1 0 0 1 0
Also required area
e 0 0 1 0 1 1
x dy (where e x y x ln y )
0 0 1 1 and 1 1 0 ;
e 1 1 0 1 0 0
ln y dy
1 0 1 1 0 1 0
e Using the property 1 0 0 and 1 0 1
ln(e 1 y )dy b b 1 0 1 0 1 1
1 f ( x )dx f (a b x )dx
a a
which are 6 in number.
32. (a, b) Given that 35. (b) For the given system to be inconsistent |A| = 0. The
sin 4 x cos 4 x 1 matrices for which | A | = 0 are
2 3 5 1 0 0 1 0 1 1 1 0
6 0 1 1 , 0 1 0 , 1 1 0
3sin 4 x 2 cos 4 x
5 0 1 1 1 0 1 0 0 1

6 (i) (ii) (iii)


sin 4 x 2[sin 4 x cos 4 x]
5 1 1 1 0 1 0 0 0 1
6 1 0 0 , 1 1 1 , 0 0 1
sin 4 x 2[1 2 sin 2 x cos2 x ] and
5 1 0 0 0 1 0 1 1 1
6 (iv) (v) (vi)
sin 4 x 2 4 sin 2 x (1 sin 2 x )
5
x 1
4 6 2 On solving A y 0
5 sin x 4sin x 2 0
5
z 0
4 2
25sin x 20 sin x 4 0
We find for A = (i)
2 2
(5sin x 2) 0 By Cramer’s rule D1 0 D2 D3
2 3 2 infinite many solution
sin 2 x cos 2 x and tan 2 x
5 5 3 For A = (ii)
By Cramer’s rule D1 0
sin8 x cos8 x 2 3 5 1
Also no solution i.e. inconsistent.
8 27 625 625 625 125
Similarly we find the system as inconsistent in cases
33. (a) Each element of set A
is 3 × 3 symmetric matrix (iii), (v) and (vi).
with five of its entries as 1 and four of its entries as 0, Hence for four cases system is inconsistent.
we can keep in diagonal either 2 zero and one 1 or no 36. (a) P ( X = 3) = (probability of not a six in first chance) ×
zero and three 1 so that the left over zeros and one’s (probability of not a six in second chance)
are even in number. × (probability of a six in third chance)
Hence taking 2 zeros and one 1 in diagonal the possible
5 5 1 25
3! 3!
cases are 9 6 6 6 216
2! 2!
and taking 3 ones in diagonal the possible cases are 37. (b) P( X 3) 1 ( X 3)

3! 1 [ P( X 1) P( X 2)]
1 3
2! 1 5 1 11 25
1 1
Total elements A can have = 9 + 3 = 12 6 6 6 36 36
2009 IIT-JEE Solved Paper 23
2009-

Alternative: 5
(B) ( x 1)( x 2)( x 3)( x 4)( x 5)dx
P ( X 3) P ( X 3) P( X 4) P( X 5) ..... 1
5 5 1 5 5 5 1 5 5 5 5 1 Let ( x 3) t dx dt
.....
6 6 6 6 6 6 6 6 6 6 6 6 Also when x 1, t 2
2 and when x 5, t 2
25 5 5
1 .... Integral becomes
216 6 6 2
(t 2)(t 1)t (t 1)(t 2)dt
25 1 25 6 25 2
216 1 5 216 1 36 2
6 t (t 2 1)(t 2 4)dt 0
2
38. (d) Let us define the events as integrand is an odd function.
A X 6 and B X 3
so that A B X 6 A O is contained by , and ( , )
2 2
5 6
5 1 5 1 ( B) p, t .
Now P( A) ....
6 6 6 6 (C) Let f ( x ) cos2 x sin x
5 2 f ( x) 2sin x cos x cos x
5 1 5 5
1 ....
6 6 6 6 For critical point f ( x ) 0
1
5
5
1 1 5
5 or cos x 0
sin x
= 2
6 6 5 6 5
1 x , , or x ,
6
6 6 2 2
3 4 3
5 1 5 1 5 Now f ( x ) 2 cos 2 x sin x
and P ( B ) ....
6 6 6 6 6 f ( x) x ve
/6
5 f ( x) x 5 / 6 ve
5
P ( A B) P ( A) 6 25 f ( x) x / 2 ve and f ( x) x ve
P( A / B) /2
P( B) P( B) 3 36
5 5
6 and are the points of local maxima.
6 6
39. A-p, q, s; B-p, t; C-p, q, r, t; D-s
Clearly all the intervals given in column II except 0,
(A) ( x 3)2 y y 0 8
contain at least one point of local maxima.
dy
( x 3) 2 y (C ) p, q, r , t
dx
1 1 (D) Let f ( x ) tan 1 (sin x cos x)
dy dx
y ( x 3) 2 1
tan 2 sin x
1 4
or log | y | log c, x 3
x 3 1
f ( x) . 2 cos x
4
y 1 1 2 sin 2 x
log ,x 3 4
c x 3
For f (x) to be an increasing function,
1 1 f ( x) 0
y
ex 3 or y ce x 3, x 3
c cos x 0 x
4 2 4 2
The solution set is ( , ) {3}
3
5 x
, , 0, , , , 0, 4 4
The interval and
2 2 2 8 4 8 3
Clearly only 0, ,
(– ) contained in the domain 8 4 4
( A) p, q, r , s , t ( D) s.
EBD_780
2009- 24 Target IIT-JEE
40. A-p; B-s, t; C-r; D-q, s The velocity of ball as perceived by fish is

(p) As the line hx ky 1, touches the circle x 2 4


y2 4 v' w v 12 16 m/s
3
Length of perpendicular from centre (0 , 0) of circle
A u=0
to line = radius of the circle 2
a = 10 m/s
1 1 s = 7.2 m
2 h2 k2 20 m
2 2 4
h k B
1 v
Locus of (h, k) is x 2 y2 , which is a circle.
4 12.8 m

(q) We know that if | z z1 | | z z2 | k

where | k | | z1 z2 |
fish
then z traces a hyperbola. 42. (b) The charges on the surfaces of the metallic spheres are
shown in the diagram. It is given that the surface charge
Here | z 2 | | z 2 | 3
densities on the outer surfaces of the shells are equal.
Locus of z is a hyperbola. Therefore
+ +
1 t2 2t
(r) We have x 3 ,y +
– –(Q1+ Q2) –
1 t2 1 t2 – (Q1+ Q2) + +
+ – Q1+ Q2+ Q3

x 1 t2 2t – + –
and y – + + – +
3 1 t2 1 t2 –
On squaring and adding, we get + – –Q1

– +
x2 2 (1 t 2 )2 4t 2 + – +
y 1
3 (1 t 2 )2 + – + R Q1 – +
– +
x2 y2 –
or 1
3 1 – + 2R +–
+ –
which is the equation of an ellipse. – + +
(s) We know eccentricity for a parabola = 1 – –
+ 3R

for an ellipse < 1 + – – +


for a hyperbola > 1
+ +
The conics whose eccentricity lies in 1 x are
parabola and hyperbola. Q1 Q1 Q2 Q1 Q2 Q3
x(say)
2 2 2
(t) Let z x iy then 4 R 4 (2 R ) 4 (3R )

2 Q1 4 R2 x
Re [( x 1) iy ] x2 y2 1
Q1 Q2 4 (2 R )2 x 4[4 R 2 x ]
2 2 2 2
( x 1) y x y 1
Q2 4[4 R 2 x ] Q1
y2 x , which is a parabola. 4[4 R 2 x ] 4 R 2 x = 3[4 R2x]
41. (c) Consider the activitiy A to B. Also Q1 Q2 Q3 4 (3R)2 x 9[4 R 2 x]
Applying v 2 u 2 2 as Q3 9[4 R 2 x] Q1 Q2 9[4 R 2 x ] [4 R 2 x ]
v2 02 2 10 7.2 3[4 R 2 x] 5[4 R 2 x ]
v 12 m/s Q1 : Q2 : Q3 1: 3 : 5
2009 IIT-JEE Solved Paper 25
2009-

43. (d) The magnetic field is increasing in the downward y


46. (a) z
direction. Therefore, according to Lenz’s law the current
I1 will flow in the direction ab and I2 in the direction dc.
44. (d) From the graph it is clear that the amplitude is 1 cm and
the time period is 8 second. Therefore, the equation for
the S.H.M. is
2 2
x a sin t 1sin t
T 8
x
x sin t 2C
4 3C
–7 C
The velocity (v) of the particle at any instant of time ‘t’
is
dx d From the figure it is clear that the charge enclosed in
v sin t cos t
dt dt 4 4 4 the cubical surface is 3 C 2 C 7 C 2 C.
Therefore the electric flux through the cube is
The acceleration of the particle is
qin 2C
2
d2x
sin t 0 0
dt 2 4 4 47. (c) Let the radius of the circle be r. Then the two distance
travelled by the two particles before first collision is
4
At t s we get 2 r. Therefore A
3 v 2v
2v t v t 2 r
d2x
2
4 2 where t is the time taken
sin sin for first collision to occur.
dt 2 4 4 3 16 3
2 r
t
3 2 3v
cm / s 2 Distance travelled by particle with velocity
32
45. (b) For the block to slide, the angle of inclination should 2 r 2 r
v= v .
be equal to the angle of repose, i.e., 3v 3
1 1 Therefore, the collision occurs at B.
tan tan 3 60 . Therefore, option (a) is
A
wrong.
For the block to topple, the condition of the block will
be as shown in the figure. 2v
120°

N
B
f
O 15 c
m
v
As the collision is elastic and the particles have equal
cm
10 masses, the velocities will interchange as shown in the
P
M figure. According to the same reasoning as above, the
w 2nd collision will take place at C and the velocities will
again interchange.
A

In POM ,
v
PM 5 cm 2
= tan 120°
OM 7.5 cm 3 B C
For this, 60 . From this we can conclude that the
block will topple at lesser angle of inclination. Thus 2v
the block will remain at rest on the plane up to a certain With the same reasoning the 3rd collision will occur at
anlgle and then it will topple. A. Thus, there will be two elastic collisions before the
particles again reach at A.
EBD_780
2009- 26 Target IIT-JEE
48. (a) The system is made up of five bodies (three circles and
R2 RL 6 1.5 9
two straight lines) of uniform mass distribution. Rp k
R2 RL 6 1.5 7.5
Therefore, we assume the system to be made up of five
point masses where the mass of each body is
24 24 7.5
considered at its geometrical centre. I mA 7.5 mA
9 24
2
7.5
Y Y
option (a) is correct.
(–a, a) (a, a) The potential difference across
(–a, a) (a, a) R L = potential difference across Rp
m m
m m 9
6m m = (7.5 mA) k 9V
m X X 7.5
(0, 0)
(0, 0)
(0, –a) option (b) is incorrect.
m m
(0, –a) (15)
Now,
Power dissipation across R1 2 103
The y-coordinate of the centre of mass is Power dissipation across R2 (9)2
m1 y1 m2 y2 m3 y3 m4 y4 m5 y5 6 103
ycm
m1 m2 m3 m4 m5
15 15 6
8.33
6m 0 m 0 m a m a m ( a ) 2 9 9
ycm
6m m m m m option (c) is incorrect.
The magnitude of power dissipated across RL is
ma a
10 m 10 (9)2
watt.
49. (c, d) Given f = – 24 cm 1.5 103
1 1 1 Now, when R1 and R2 are interchanged, the equivalent
Applying
v u f 2 1.5 3
resistance between R1 and RL k
For (66, 33) 2 1.5 3.5
1 1 1 1 1 66 24 42
v f u 24 66 24 66 24 66
R2 = 6k
24 66
v 37.7
42 24 V
But the value of v = 33. The absolute error is 37.7 – 33
RL = 1.5k

2k R1
= 4.7 cm which is greater than 0.2 cm. Therefore, it is a
wrong reading.
For (78, 39), when u = 78 then
1 1 1 Potential drop across this equivalent resistance
v 78 24
3 103
v = – 34.67
3.5 3
The absolute error is 39 – 34.67 = 4.33 which is greater 24 24 3
3 24
than 0.2 cm. 6 103
3.5
50. (a, d)
Potential difference across
k R1 R1
I I 32 1 92
RL 3
24 V 1.5 10 9 1.5 103
24 V
RL = 1.5 k

6k R2 Rp The magnitude of the power dissipation in RL will


decrease by a factor 9 if R1 and R2 are interchanged.
(d) is the correct option.
2009 IIT-JEE Solved Paper 2009- 27

d psystem 2a
51. (a) Fext Now, n a
dt 2 n
Given Fext 0 psystem = Constant h 2 n2 n 2 h2
E
Due to internal forces acting in the system, the kinetic 2 m 4a 2 8m a 2
and potential energy may change with time.
Also zero external force may create a torque if the line E a 2 for a particular value of n.
of action of forces are along different directions. Thus 57. (b) For ground state n = 1,
the torque will change the angular momentum of the Given m = 1.0 × 10–30 kg, a = 6.6 × 10–9 m
system.
52. (b, d) C p Cv R for all gases 12 (6.6 10 34 2
) 10 68
E J J
30 9 2 48
For monoatomic gas : 8 1 10 (6.6 10 ) 8 10
3 5 Cp 5 20
Cv R ; Cp R ; 10
2 2 Cv 3 J
8
15
C p .Cv ; Cp Cv 4
10 20
100
4 eV meV 8 meV
For diatomic gas : 19 8 1.6
8 1.6 10
5 7 Cp 7
Cv R ; Cp R ; and h h h
58. (d) mv
2 2 Cv 5 p mv
35 n 2a
C p .Cv ; Cp Cv 6 But a
4 2 n
53. (d) The collection of 12 H nuclei and electron is known as
nh nh
plasma which is formed due to high temperature inside mv v v n
2a 2 am
the reactor core.
54. (a) Applying conservation of mechanical energy, we get, 59. A-p, r, s; B-r, s; C-p, q, t; D-r, s
Loss of kinetic energy of two deuteron nuclei (p) + – Q
= Gain in their potential energy.
1 e e –
2 1.5kT +
4 0 r M
P
+ –
9
eV (1.44 10 eVm)
2 1.5 8.6 10 5 T The electric field at M due to the charges at the corners
K 15
4 10 m of regular hexagon is as shown
1.44 10 9 E–
T 15 E+
5
2 1.5 8.6 10 4 10 120°
= 0.0139 × 1011 = 1.4 × 109 K E– 120°
55. (b) For the reading given in option (b), we get, nto > 5 × 1014 E+ 120° E+
which is the Lawson criterion for a reactor to work
successfully. E–

h p2 Here | E | | E | . The symmetry of the situation


56. (a) and E
p 2m shows that E = 0 at M.
Therefore, (A) is the correct option.
n loops
The electric potential due to all the charges at M is
x=0 x=a zero.
Therefore, (B) is incorrect option.
When the system of charges is rotated about line PM,
h2 the net current will be zero.
E Therefore, the magnetic field at M is zero.
2m 2
Hence, (C) is the correct option.
The length in which the particle is restricted to move is
When magnetic field is zero, then = 0
a. This length is a multiple of . (D) is incorrect option.
2
EBD_780
2009- 28 Target IIT-JEE
(s) 2a
P
– + –
– + – + – +
(q) a
M M
P Q
Q – + –
The electric field due to the inner most positive and The electric field at M due to all the charges is zero
because the electric field due to different charges cancel
q out in pairs.
negative charges at M is E1 2 k towards left.
r2 (A) is the correct option.
The electic field due to the next positive and negative The potential at M due to the charges is

q q q q
charges at M is E2 2 k towards right. The V k 4 0
a/2 a/2 5a
(3r )2
4
electric field due to the next positive and negative
(B) is the correct option.
q When the whole system is set into rotation with a
charges at M is E3 2 k towards left. Similarly,, constant angular velocity about the line PQ , we get
(5r )2
three loops in which current is flowing.
it will go on. Clearly the vector sum of these electric The magnetic field due to these currents produce a
fields is not zero. resultant magneic field at M which is not equal to zero.
(A) is incorrect option. Therefore, a net magnetic dipole moment will be
The electric potential due to the charges at M produced.
q q q q q q (C) is an incorrect option.
k ..... 0 (D) is correct option.
r r 3r 3r 5r 5r
(B) is incorrect option. P
+ –
The net current due to the innermost positive and
negative charges is zero. Similarly, the net current due
(t) + + M– –
to other charges in pair is zero. Therefore the magnetic
field at M is zero. Also the magnetic moment is zero. Q
(C) is the correct option
There will be a net electric field due to the arrangement
(D) is incorrect option.
of charges at M towards the right side.
Q
(A) is an incorrect option.
+ – + The electic potential at M will cancel out in pairs by
positive and negative charges, due to symmetrical
(r) – M – arrangement of charges.
(B) is an incorrect option.
P +
When the system of charges rotates about PQ, the net
current is zero due to symmetrical arrangement of
The net electric field due to negative charges in the charges. Therefore, B = 0 and = 0
inner circle is zero. Similarly, the net electric field due to (C) is the correct option.
positive charges in the outer circle is zero. (D) is the incorrect option.
(A) is the correct option. 60. A-p, t; B-q, s, t; C-p, r, t; D-q
The electric potential due to negative charges at M is
different from the electric potential due to positive N
charges at M. Therefore, the electric potential at M is Y f
not equal to zero. v
X
(B) is the correct option. (p)
si n
When the system of charges rotate, we get a current I1 Mg
Mg Mg cos
due to negative charges and another current I due to P
positive charges. The magnitude of the magnetic field
at M due to the currents is different. Therefore, B 0 As the velocity is constant,
and 0 f Mg sin ..... (i)
(C) is incorrect option.
(D) is the correct option. But f N Mg cos ..... (ii)
2009 IIT-JEE Solved Paper 29
2009-

From (i) and (ii), The mechanical energy of the system (X + Y) is


Mg cos Mg sin tan continously decreasing as the system is coming down
and its potential energy is decreasing, the kinetic
The force by X on Y is the resultant of f and N and is energy remaining the same.
equal to Therefore, option (C) is correct and (B) is incorrect.
f2 N2 2
N2 N2 ( 2
1) N The torque of the weight of Y about P is not zero.
(s)
2
= ( tan 1) Mg cos sec Mg cos Mg
= weight of Y.
Therefore, option (A) is correct.
Now, due to the presence of frictional force between Y Y
and X, the mechanical energy of the system ( X Y )
decreases continously as Y slides down.
Therefore, option (C) is correct. X

P The force on Y by X is equal to the wt. of liquid


displaced which cannot be equal to Mg as the density
Z v of Y is greater than density of X (as Y is sinking)
(q) Therefore, option (A) is incorrect.
Y The gravitational potential energy of X increases
Mg continously because as Y moves down, the centre of
X mass of X moves up.
Therefore, option (B) is correct.

As the lift moves up, X also moves up and therefore (t)


the gravitational energy of X is continously increasing.
Option (B) is correct.
The torque of the weight of Y about P is zero as the
perpendicular distance of the line of action of force Y
from the point P is zero.
Option (D) is correct.
The force exerted by X on Y will be equal to
X
Mg Mg 2Mg where Mg is wt. of Y and Mg is the
force on Y due to Z. P
Option (A) is incorrect. Sphere Y is moving with terminal velocity. Therefore,
(r) the net force on Y is zero, i.e.,
B Fv
P
Y

X v
m0g
Mg

Mg B Fv
In this case the force exerted by X on Y is same as the where B = buoyant force and Fv = viscous force.
force exerted by Y on X. The force on X due to Y is B + Fv are exerted by X on Y.
Therefore, option (A) is correct.
R ( Mg ) 2 [(m0 M ) g ]2 Mg The gravitational potential energy of X is continously
increasing because as Y moves down, the centre of
Mg
mass of X moves up.
m0g Option (B) is correct.
The mechanical energy of the system (X + Y) is
R continously decreasing to overcome the viscous
Mg forces.
Therefore, option (A) is incorrect. Option (C) is correct.
EBD_780
2009- 30 Target IIT-JEE

PAPER - 2
1. (a) Chromium in Cr (CO)6 is in zero oxidation state and has 5. (a,b,c) Lower amines like NH3, CH3NH2 and (CH3)2NH break
[Ar]18 3d 54s 1 as the electronic configuration. However, diborane molecule unsymmetrically, while larger amines
CO is a strong ligand, hence pairing up of electrons like (CH3)3N, C5H5N break diborane in symmetrical
takes place leading to following configuration in manner.
Cr(CO)6.
B2 H 6 2NH 3 [H 2 B(NH 3 ) 2 ] [BH 4 ]
3d 4s 4p

B2 H 6 2NH 3 2H 3B N(CH 3 )3
d 2 sp3 Hybridisation
O O
Since the complex has no unpaired electron, its 6. (a,b,c) N N O N–N
magnetic moment is zero. (a) O
2. (d) Migrating tendency of hydride is greater than that of (b)
alkyl group. Further migration of hydride from C–2 gives
more stable carbocation (stabilized by +R effect of OH O O O O
group and +I and hyperconjugative effects of methyl N–N N–O–N
group). O O O O
(c) (d)
H H
1 2 3+ 4 5
7. (a,b,d) The species having less reduction potential with
1, 2 hydride
H3C – C — C– C – CH3
shift
respect to NO3– (E° = + 0.96 V) will be oxidised by
NO3–. These species are V, Fe and Hg.
OH H CH3
8. (b,c) We know that carbohydrates having acetal linkage are
non-reducing while that with hemiacetal linkage are
H H
+
reducing. In the give structure,
H3C – C — C– C – CH3 X has acetal linkage, hence non-reducing.
Y has hemiacetal linkage, hence reducing.
..OH H CH3
..

Further X is -anomer, while Y is -anomer of D-(+)-


More stable carbocation glucose.
9. (a,d) Internal energy and molar enthalpy are state functions.
Ea
3. (d) log k log A …(1) Work (reversible or irreversible) is a path function.
2.303RT
10. A-p, s; B-q, s; C-r, t; D-q, t
1 (A) 3Cu + 8 HNO3 (dil.)
Also given log k 6.0 (2000) …(2)
T
On comparing equations, (1) and (2) 2NO + Cu(NO3 )2 4H 2 O
p s
log A = 6.0 A = 106 s–1
Ea (B) Cu 4HNO3 (conc.)
and 2.303 R 2000 ;
2NO 2 Cu(NO3 ) 2 2H 2 O
Ea = 2000 × 2.303 × 8.314 = 38.29 kJ mol–1 q s

4. (b) H 2C N N H2C N N (C) 4Zn 10HNO3 (dil.)


I III
Octet complete, Octet complete,
6 covalent bond, 6 covalent bond, 4Zn(NO3 )2 N 2 O 5H 2 O
–ve charge on N –ve charge on C t r

(D) Zn 4HNO3 (conc.)


H2C N N H2C N N
II IV
Octet incomplete, Octet incomplete, Zn(NO3 )2 2NO2 2H 2O
5 covalent bond, 5 covalent bond, t q
–ve charge on N –ve charge on C
2009 IIT-JEE Solved Paper 2009-31
11. A-p, q, t; B-p, s, t; C-r, s; D-p
14. 2MnO 2 4KOH O 2 2K 2 MnO 4 2H 2O
– Nu Oxidation number of Mn in K2MnO4 is 6
Nu –

..
+ Br
(p) K2MnO4 ; 2 + x – 8 = 0
O
x=6
Br –
B (–HBr)
..
(A)
(q) 3RTx 2RTy
O O v rms of X = v mp of Y =
15. Mx ; My
Pd–C/ , –H 2 Br

(t) Given vrms = vmp


O
3RTx 2RTy

HX(SN) X + H2O Mx = My
(p)

2RTy M x 2 60 40
O – C – CH3
My 4
OH
(CH 3CO) 2O 3RTx 3 400
(B) (s)
O
16. The seven possible cyclic structural and stereoisomers are
CHO
–H 2
CH3
(t)

Nu
– + C2H5
Nu : H OH H3C CH3
(r)
OH
CHO
(C) H3C H3C H H3C H
CH3
OH
CHO
(CH3CO) 2O
H H H CH3 H CH3
(s)
O – C – CH3 cis, (meso) trans, enantiomers

O
17. Coordination number of Al is 6. It exists in ccp lattice with 6
coordinate layer structure.
Br – Nu
Nu : – 18. Energy released by combustion of 3.5 g gas
(D) + Br :
NO2
(p)
NO2 = 2.5 × (298.45 – 298) kJ

12. The number of water molecules directly bonded to the metal 2.5 0.45 1
Energy released by 1 mole of gas 9 kJmol
centre in CuSO4.5H2O is 4. 3.5 / 28

238 6 214 2 214


H2O OH2 19. 92 U 80 X 82 Pb
2+
SO42– . H2O Hence total number of particles emitted are 2 +6 = 8
Cu
20. (c) The line has +ve and equal direction cosines, these are
H2O OH2
1 1 1
, , or direction ratios are 1, 1, 1. Also the
13. pH of sodium salt of weak acid 3 3 3

1 lines passes through P (2, – 1, 2).


= (pK w pK a log C) Equation of line is
2

1 x 2 y 1 z 2
(14 4 2) 8 (say)
2 1 1 1
EBD_780
2009- 32 Target IIT-JEE

Let Q ( 2, 1, 2) be a point on this line where 22. (c) Given that for an A.P, Sn = cn2
it meets the plane Then Tn Sn Sn 1 cn 2 c (n 1)2
2x y z 9
(2n 1)c
Then Q must satisfy the eqn of plane
Sum of squares of n terms of this A.P
i.e. 2( 2) 1 2 9
Tn2 (2n 1)2 .c 2
1
Q has coordintes (3, 0, 3)
Hence the length of line segments PQ
c2 4 n2 4 n n

(2 3)2 ( 1 0)2 (2 3)2 3 4n(n 1)(2n 1) 4n(n 1)


c2 n
6 2
x2 y2
21. (c) The given ellipse is 1
42 22 2(2n2 3n 1) 6(n 1) 3
c2 n
such that a2 = 16 and b2 = 4 3

4 3 3
e2 1 e 4n2 1 n(4n 2 1)c 2
16 4 2 c2 n
3 3
Let P (4cos , 2sin ) be any point on the ellipse, then
equation of normal at P is 23. (d) The triangle is formed by the lines

4 x sin 2 y cos 12sin cos

x y A (pq,(p+1)(q+1))
1
0
3cos 6sin
(1+
p)=

q)
1+

Q, the point where normal at P meets x –axis, has


x–
p(

qy
+

coordinates (3cos , 0)
py

+q
x–

H(h, k) (1+
)
+p

7 cos q)=
(1

Mid point of PQ is M , sin 0


2
B C
For locus of point M we consider (–p,0) (y = 0) (–q,0)

7 cos
x and y sin
2 AB : (1 p) x py p (1 p) 0
2x AC : (1 q ) x qy q (1 q) 0
cos and sin y
7
BC : y 0
4 x2 So that the vertices are
y2 1 ...(1)
49
A( pq, ( p 1)(q 1)), B ( p, 0), C ( q, 0)
Also the latus rectum of given ellipse is
Let H (h, k ) be the orthocentre of ABC. Then as
3
x ae 4 2 3 or x 2 3 ...(2) AH BC and passes through A( pq, ( p 1)(q 1))
2
Solving equations (1) and (2), we get The eqn of AH is x pq

4 12 1 1 h pq ...(1)
y2 1 y2 or y
49 49 7 Also BH is perpendicular to AC

1 k 0 1 q
The required points are 2 3, . m1m2 1 1
7 h p q
2009 IIT-JEE Solved Paper 2009- 33
Eliminating t from above, we get the locus of centriod
k 1 q
1 (using eqn (1) G as
pq p q
2
3y 4a 2
k pq ...(2) 3x 2a a y2 x a
2a 3 3
From (1) and (2) we observe h k 0
Locus of (h, k) is x y 0 which is a straight 2a
which is a parabola with vertex , 0 , directrix as
3
line.

1 2a a a 4a
24. (b, c, d) We have, f ( x ) x cos , x 1 x or x , latus rectum as and
x 3 3 3 3
focus as (a, 0).
1 1 1 26. (a, b, c) We have
f '( x) cos sin
x x x

lim f '( x) cos 0 (0) × (some finite value) sin n x ....(1)


In dx
x x
(1 )sin x
lim f '( x) 1
x
sin n ( x )
1 1 1 1 1 1 In dx
x
Also f "( x) sin sin cos (1 )sin( x)
2 x 2 x 3 x
x x x

1 1 b b
f "( x) 3
cos 0, x [1, ) Using f ( x) dx f (a b x) dx
x x
a a
f '( x) is strictly decreasing in [1, )
x
f '( x) lim f '( x) sin nx ...(2)
x In dx
x
(1 )sin x
f ( x 2) f ( x)
1 Adding equation (1) and (2), we get
( x 2) x

f ( x 2) f ( x) 2 sin nx sin nx
2I n dx 2 dx
sin x sin x
0
25. (a, d) Let P (at 2 , 2at ) be any point on the parabola
[as integrand is an even function]
y2 4ax.
sin nx
x In dx
Then tangent to parabola at P is y at sin x
t 0

which meets the axis of parabola i.e x-axis at


T (– at2, 0). sin(n 2) x sin nx
Now I n 2 In dx
sin x
Also normal to parabola at P is t x y 2at at 3 0

which meets the axis of parabola at N (2a at 2 , 0)


2 cos( n 1) x sin x
dx 2 cos( n 1) x dx
Let G (x, y) be the centriod of PTN , then sin x
0 0

at 2 at 2 2a at 2 2at
x and y sin(n 1) x
3 3 2 0
n 1 0
2a at 2 2at
x and y In 2 In
3 3
EBD_780
2009- 34 Target IIT-JEE

cot cot 4 cot tan 4


Also I1 1dx and I0 = 0 2
0
cos 2 sin 2 4 sin cos
10
Hence I 2m 1 I3 I5 I7 ...I 21 1 5
sin 2 2 or
m 1 2 6 6

= 10 I (using I n 2 In ) 5
or
= 10 12 12
28. (a, b) The given hyperbola is
10
and I 2m I2 I4 I 6 ... I 20 1
x2 y2 ...(1)
m 1 2

= 20 × I0 (using I n 2 In ) which is a rectangular hyperbola (i.e. a = b)

= 20 × 0 = 0 e 2.
27. (c, d) We have
x2 y2
6
Let the ellipse be 1
( m 1) m a2 b2
cosec cosec 4 2
4 4
m 1 1
Its eccentricity
2
6 sin
4 4 1 a2
(m 1) m b2 a2 1 b2
m 1 sin sin 2 2
4 4
So, the equation of ellipse becomes

m (m 1) x2 2 y2 a2 ...(2)
6 sin
4 4 Let the hyperbola (1) and ellipse (2) intersect each
4
(m 1) m other at P ( x1 , y1 ).
m 1 sin sin
4 4
Then slope of hyperbola (1) at P is given by

dy x1
m1
m (m 1) dx y1
sin cos ( x1, y1 )
4 4
m (m 1) and that of ellipse (2) at P is
6 cos sin
4 4 dy x1
4 m2
(m 1) m dx 2 y1
m 1 sin sin ( x1 , y1 )
4 4
As the two curves intersect orthogonally,

6 m1m2 1
(m 1) m
cot cot 4
4 4 x1 x1
m 1 . 1
y1 2 y1
2
cot cot cot cot
4 4 4 x12 2 y12 ...(i)

5 6 1
... cot cot 4 Also P ( x1 , y1 ) lies on x 2 y2
4 4 2
2009 IIT-JEE Solved Paper 2009- 35

1 The required volume is a. b c


x12 y12 ...(ii)
2
1 1 0
1
Solving (i) and (ii), we get y12 and x12 1 1 2 0
2
1 1
Also P ( x1 , y1 ) lies on ellipse x 2 2 y2 a2
(D) We have a b 3c | a b |2 3 | c |2
x12 2 y12 a 2
1 + 1 = a2 or a2 =2
(a b).(a b) 3 c.c
The required ellipse is x 2 2y 2 whose foci
a.a b.b 2 a.b 3 c.c 1 1 2cos 3
1
are ( ae, 0) 2 ,0 ( 1, 0) (where is the angle between a and b )
2
1
29. A-q, s; B-p, r, s, t; C-t; D-r cos
2 3
(A) The given equation is
30. A-p; B-q, s; C-q, r, s, t; D-r
2sin 2 sin 2 2 2
(A) For the solution of x esin x cos x 0 in 0,
2sin 2 4 sin 2 cos 2 2 0 2

sin 2 2sin 2 (1 sin 2 ) 1 0 Let us consider two functions

y x esin x and y cos x


2sin 4 3sin 2 1 0

2sin 4 2 sin 2 sin 2


e
1 0 The range of y xesin x is 0, , also it is an
2
2sin 2 (sin 2 1) 1(sin 2 1) 0
increasing function on 0, . Their graph are as
(sin 2 1)(2sin 2 1) 0 2
shown in the figure below :
2 2 1
sin 1 or sin
2 y
sinx
y=xe
2 2 2 2
sin sin or sin sin
2 4 y = cos x
x
n or n or
2 4 2 4
(B) We know that [x] is discontinuous at all integral values,

6x
therefore is discontinuous at x , , and Clearly the two curves meet only at one point, therefore
6 3 2

3x the given equation has only one solution in 0, .


. Also cos 0 for any of these values of x. 2
(B) Three given planes are
6x 3x kx 4 y z 0
cos is discontinuous at x , ,
6 3 2
4 x ky 2 z 0
and .
(C) We know that the volume of a parallelopipe with 2x 2 y z 0

coterminus edges as a , b and c is given by [ a b c ]


EBD_780
2009- 36 Target IIT-JEE
Clearly all the planes pass through (0,0,0). (D) Given that
Their line of intersection also pass through (0, 0, 0)
Let a, b, c, be the direction ratios of required line, then dy
y 1 and y (0) 1
we should have dx

ka 4b c 0
dy
dx ln | y 1| x c
4a kb 2c 0 y 1
2a 2b c 0
At x = 0, y = 1 c = ln 2
For the required line to exist the above system of
equations in a, b, c, should have non trivial solution ln |y+1|= x +ln 2 y 1 2e x y 2e x 1
i.e.
y (ln 2) 2eln 2 1 2 2 1 3
k 4 1
4 k 2 0 31. Integer answer : 8
2 2 1 Let r be the radius of required circle.

k (k 4) 4(4 4) 1(8 2 k ) 0 Clearly, in C1CC2 , C1C C2C r 1

k2 6k 8 0 (k 2)(k 4) 0 and P is mid point of C1C2


k = 2 or 4 CP C1C2
(C) We have f ( x) | x 1| | x 2 | | x 1| | x 2 |
Also PM CC1

4x , x 2 Now PMC1 ~ CPC1 (by AA similarity)


2x 4 , 2 x 1
6 , 1 x 1 MC1 PC1
2x 4 , 1 x 2 PC1 CC1
4x , x 2

The graph of the above


function is as given below C1 1 P 1 C2
3 1
1
M
Y
r r
8

–3 –2 –1 0 1 2 3

Clearly, from graph, f ( x ) 6

3
4k 6 k
2
k = 2, 3, 4, 5, 6 ,..... 1 3
r +1= 9 r = 8.
3 r 1
2009 IIT-JEE Solved Paper 2009-37
32. Integer answer : 4 34. Integer answer : 7
The given system of equations is
Let ACC
3x y z 0
then AC C ( AC AC )
3x z 0
and AC B 180 . 3x 2 y z 0
Let x = p where p is an integer
A then y = 0 and z = 3p
But x 2 y2 z2 100 p2 9 p2 100
4 2 2
2 2 p2 10 p 0, 1, 2 3
– i.e. p can take 7 different values.

30° 18 Number of points (x, y, z) are 7.
B C
C' 35. Integer answer : 2

Given that f ( x) x3 e x / 2
Applying sine law in ABC , we get
Let g ( x ) f 1 ( x)
4 2 2
then we should have gof ( x ) x
sin(180 ) sin 30
g ( f ( x)) x
1
sin 45 g ( x3 ex / 2 ) x
2
Differentiating both sides with respect to x, we get
CAC 90
1
g ( x3 e x / 2 ). 3x 2 ex / 2 . 1
So, the required area ar ( ABC) ar ( ABC ) 2

1 1
= ar ( ACC ) AC AC g ( x3 ex / 2 )
2 1
3x 2 ex / 2 .
2
1
2 2 2 2 = 4 sq. units. 1
2 For x = 0 , we get g (1) 2
1/ 2
33. Integer answer : 0
36. Integer answer : 2
x The given equation is
Given that f ( x ) f (t )dt
0
x 2 8kx 16(k 2 k 1) 0
Clearly f (0) 0 . Also f ( x ) f ( x) Both the roots are real and distinct
D>0 (8k )2 4 16(k 2 k 1) 0
f ( x)
1 k> 1 ...(i)
f ( x)
Both the roots are greater than or equal to 4
Integrating both sides with respect to x, we get 8 and f (4) 0
f ( x) k>1 ...(ii)
dx 1 dx
f ( x) and 16 32k 16(k 2 k 1) 0

ln f ( x ) x ln C f ( x) Ce x k2 3k 2 0 (k 1)(k 2) 0
k ( ,1] [2, ) ...(iii)
Now f (0) 0 Ce x 0 C 0
Combining (i), (ii) and (iii), we get k 2 or the smallest
f ( x) 0 x f (ln 5) 0 value of k = 2.
EBD_780
2009- 38 Target IIT-JEE
37. Integer answer : 7 1 4
p( x) x x3 x2
The given function is f ( x) 2 x3 15 x 2 36 x 48 4

16
and A {x | x 2 20 9 x} So, that p (2) 8 4 0
4

A x | x2 9 x 20 0
39. (b) Y
A x | ( x 4)( x 5) 0
Ncos
A = [4, 5] N
2 2
Also f ( x ) 6x 30 x 36 6( x 5x 6) ma Nsin
P
6( x 2)( x 3)
mg
Clearly x A, f ( x ) 0
f is strictly increasing function on A. X
Maximum value of f on A

= f (5) 2 53 15 52 36 5 48 The forces acting on the bead as seen by the observer


in the accelerated frame are : (a) N ; (b) mg ; (c) ma
= 250 – 375 + 180 – 48 = 430 – 423 = 7.
(pseudo force).
38. Integer answer : 0
Let is the angle which the tangent at P makes with the
Let p( x ) ax 4 bx3 cx 2 dx e X- axis. As the bead is in equilibrium with respect to the
wire, therefore
p ( x)
Now lim 1 2 N sin = ma and N cos = mg
x 0 x2
a
tan = … (i)
p( x ) g
lim 1 ...(1)
x 0 x2 But y = k x2. Therefore,
dy
p(0) 0 e 0 2kx tan … (ii)
dx
Applying L’Hospital’s rule to eq n (1), we get From (i) & (ii)
a a
p ( x) 2kx x
lim 1 p (0) 0 g 2kg
x 0 2x
40. (a) The energy possessed by photons of wavelength
d 0
1240
Again applying L ‘Hospital’s rule, we get 550 nm is 2.25 eV
550
p ( x) The energy possessed by photons of wavelength
lim 1 p (0) 2
x 0 2
1240
450 nm is 2.76 eV
2 c = 2 or c = 1 450
p ( x) ax 4 bx 3 x2 The energy possessed by photons of wavelength
1240
350 nm is 3.54 eV
p ( x) 4ax 3 3bx 2 2x 350
As p(x) has extremum at x =1 and 2 For metal plate p :
p (1) 0 and p (2) 0 p = 2 eV.
All the wavelengths are capable of ejecting electrons.
4a 3b 2 0 ...(i)
Therefore, the current is maximum. Also as the work
32a 12b 4 0 or 8a 3b 1 0 ...(ii) function is lowest in p, the kinetic energy of ejected
electron will be highest and therefore, the stopping
1
Solving eq’s (i) and (ii) we get a and b 1 potential is highest.
4
2009 IIT-JEE Solved Paper 2009- 39
For metal plate q : Figure shows the rod at an angle with respect to its
q = 2.5 eV.
equilibrium position. Both the springs are stretched by
Photons of wavelength 550 nm will not be able to eject
electrons and therefore, the current is smaller than p. length .
2
The work function is greater than q therefore the
stopping potential is lower in comparison to p. The restoring torque due to one spring
For metal plate r : = – (Restoring force) × perpendicular distance

r = 3 eV 2
Only wavelength of 350 nm will be able to eject electrons = k k
2 2 4
and therefore, current is minimum. Also the stopping
potential is least. Therefore, the total restoring torque due to both the

2 2
41. (d) springs = 2 k k … (i)
k1 k2 4 2

P M
If I is the moment of inertia of the rod about M then
Case (i)
d2
x1 I I … (ii)
dt 2
A
From (i) & (ii), we get
P M

Case (ii)
d2 2
d2 k 2 k 2
I 2
k
dt 2 dt 2 I 2 M 2
/12 2

In case (ii), the springs are shown in the maximum d2 6k


compressed position. If the spring of spring constant
dt 2 M
k1 is compressed by x1 and that of spring constant k2
is compressed by x2 then Comparing it with the standared equation of rotational
SHM we get
x1 x2 A … (i)

k1 x1 d2 2 2 6k 6k
and k1 x1 k2 x2 x2 …(ii) 2
x2 dt M M

From (i) & (ii)


6k 1 6k
2 v v .
k1 x1 k2 A M 2 M
x1 A x1
k2 k2 k1
43. (b, d) For process B C D:
The temperature at B is greater than the temperature at
42. (c) D. Therefore, U = –ve. Also work done in the process
is negative. Thus heat flows out of the gas during the
2
process.
For the cyclic process ABCDA :

/2 As the process takes place in the clockwise direction.


M Therefore the work done is positive.
• Process during the path A B is circular and therefore
the process is not isothermal (for isothermal process,
the path in P-V graph is a rectangular hyperbola).
2 • During A B C, the work done is positive.
EBD_780
2009- 40 Target IIT-JEE

44. (a) If V 0 is the velocity of centre of the sphere, then


v
VC 2V 0 , V B V 0 and V A 0
–q
VC V B 2V 0 V 0 V0
+Q
VB VA V0 0 V0

VC V B VB VA
The situation is shown in the figure which is similar to (b) is the correct option.
a planet revolving around sun. The distance of – q
from + Q is changing, therefore, force between the Now, | V C V A | | 2V 0 0 | | 2V 0 | 2 | V 0 |
charges will change.
The speed of the charge – q will be greater when the and | V C V A | 2 | V B V C |
charge is nearer to + Q as compared to when it is far. (c) is the correct option.
Therefore, the angular velocity of charge – q is also 47. (a, d) At second resonance the length of air column is more
variable. The direction of the velocity changes
as compared to first resonance. Now, longer the length
continously, therefore, linear momentum is also variable.
of air column, more is the absorption of energy and
The angular momentum of (– q) about Q is constant
lesser is the intensity of sound heard.
because the torque about + Q is zero.
45. (b, d) When current I is switched on in both the solenoids in
identical manner, eddy currents are setup in metallic e
rings A and B in such a way that rings A and B are
repelled.

hA
hB
A B
A B

As shown in the figure, the length of the air column at


1
the first resonance is somewhat shorter than th of
4
the wavelength of the sound in air due to end correction.
Given hA hB . This shows that eddy currents
produced in A are greater than in B. This is possible e e
4 4
when A B (the rate of change of flux is same in
both the rings, therefore induced emf is same). The 48. A-p, q, t; B-q; C-s; D-s
height attained is independent of the masses of rings A (p) When an uncharged capacitor is connected to a battery,
and B. it becomes charged and energy is stored in the
capacitor. (A) is the correct option.
46. (b, c) VC 2V 0 (q) When a gas in an adiabatic container fitted with an
adiabatic piston is compressed by pushing the piston
(i) the internal energy of the system increases
U Q W 0 ( PdV ) PdV
VB V0
(ii) mechanical energy is proceeded to the piston
which is converted into kinetic energy of the gas
VA 0 molecules.
2009 IIT-JEE Solved Paper 2009- 41
(r) None of the options in column I matches. As the gas in
a rigid container gets cooled, the internal energy of the (B)
system will decrease. The average kinetic energy per
molecule will decrease. S2 P2
(s) When a heavy nucleus initially at rest splits into two P1
nuclei of nearly equal masses and some neutrons are P0
emitted then
(i) internal energy of the system is converted into S1
mechanical energy (precisely speaking kinetic energy)
and
(ii) mass of the system decreases which converts into
energy.
(t) When a resistive wire loop is placed in a time varying ( P0 ) , ( P1 ) 0, ( P2 )
magnetic field perpendicular to its palne. 4 12
(i) Induced current shows in the loop due to which
I ( P0 ) I1 I2 2 I1 I 2 cos / 2
the energy of system is increased.
× × × I1 I2 I0 I0 2I0

I ( P1 ) I1 I2 2 I1 I 2 4I 0
× × ×
I ( P2 ) I1 I2 2 I1 I 2 cos / 6
× × ×
I1 I 2 3 I1 I 2
49. A-p, s; B-q; C-t; D-r, s, t
(A) I0 I0 3 I0

S2 P2 (2 3)I 0
P1
Therefore, q matches with (B)
P0
(C)
S1
S2 P2
P1
P0
For path difference / 4 , phase difference is / 2 .
For path difference / 3 , phase difference is 2 / 3 . S1
Here, S1P0 S2 P0 0
( P0 ) 0
Here ( P0 ) / 2; ( P1 ) / 4, ( P2 ) /6
Therefore, (p) matches with (A).
The path difference for P1 and P2 will not be zero. The I ( P0 ) I1 I 2 2 I1 I2 cos(– )
intensities at P0 is maximum.
I ( P0 ) I1 I2 2 I1 I 2 cos 0 I1 I 2 2 I1 I 2 I0 I0 2I0 0

( I1 I 2 )2 ( I0 I0 ) 2 4 I0 I ( P1 ) I1 I 2 2 I1 I 2 cos( / 2)

I ( P1 ) I1 I2 2 I1 I 2 cos I1 I2 I0 I0 2I0
2
I1 I2 I0 I0 2I0 I ( P2 ) I1 I 2 2 I1 I 2 cos
3
I ( P2 ) I1 I2 2 I1 I 2 cos(2 / 3)
I1 I2 I1 I 2 I0 I0 I0 3I 0
I1 I 2 I1 I 2 I0 I0 I0 I0
I ( P0 ) I ( P1 ) I ( P2 ) I ( P1 )
Therefore, (s) matches with (A). (t) matches (C).
EBD_780
2009- 42 Target IIT-JEE
According to ideal gas equation,
(D)
4
S2 P2 PAVA n A RTA 16 (0.02)3 n A RTA … (i)
3
P1
P0 For bubble B :
If PB is the pressure inside the bubble then
S1
4T 4 0.04 PB 12 N / m2
PB 8 4
RB 0.04

Here, ( P0 ) 3 / 4; ( P1 ) / 2; ( P2 ) 5 /12 According to ideal gas equation,


4
3 PBVB nB RTB 12 (0.04) 3 nB RTB … (ii)
I ( P0 ) I1 I2 2 I1 I 2 cos 3
2
Dividing (ii) by (i) we get
I1 I2 I0 I0 2I0
4
I ( P1 ) I1 I2 2 I1 I 2 cos( ) 12 (0.04)3
3 nB
TA TB
I1 I2 2 I1 I 2 I0 I0 2 I0 I0 0 4 nA
16 (0.02)3
3
I ( P2 ) I1 I 2 2 I1 I 2 cos 5 /6
nB
I1 I2 3 I1 I 2 2 3 I0 6
nA
(r), (s), (t) match with (D).
50. (8 J) 52. (k = 7)
Given m = 0.36 kg, M = 0.72 kg. The right angled triangle is shown in the figure. Let us drop
The figure shows the forces on m and a perpendicular from P on QR which cuts QR at M.
M. When the system is released, let The magnatic field due to currents in PQ and RP at P is zero.
the acceleration be a. Then The magnetic field due to current in QR at P is
T – mg = ma
Mg – T = Ma T
Q a
( M m) g a m
a g /3 T
1
M m M
mg M a I (5x – a)
and T = 4 mg/3
For block m :
u = 0, a = g/3, t = 1, s = ? Mg 3x

1 2 1 g 2 2
s = ut + at = 0 + 1 g /6 R
2 2 3 P 4x
Work done by the string on m is
mg g 4 0.36 10 10
T .s Ts 4 8J 0 I …(i)
3 6 3 6 B (cos 1 cos 2)
4 PM
51. (6)
For bubble A : In PQM,
If PA is the pressure inside the bubble then 9x2 = PM2 + a2 … (ii)
In PRM,
16x2 = PM 2 + (5x – a)2 … (iii)
B
A
7 x2 25 x 2 10 xa 10xa = 18x2
2 cm
4 cm a = 1.8 x … (iv)
–2 From (ii) & (iv),
8 Nm
9 x2 PM 2 (1.8 x)2
4T 4 0.04
PA 8 8 PA 16 N / m2
RA 0.02 PM 9 x 2 3.24 x 2 5.76 x 2 2.4 x … (v)
2009 IIT-JEE Solved Paper 2009- 43
54. (5 cm)
a 1.8 x
Also cos 1 0.6 … (vi)
3x 3x T
We know that,
5x a 5 x 1.8 x 3.2
cos 2 0.8 …(vii)
4x 4x 4 mass
where T = tension in the string and
length
From (i), (v), (vi) and (vii),

0 I 0.5
B 0.6 0.8 v 3
10m / s
4 2.4 x 10 / 0.2
The wavelength of the wave established
0 I 0I
1.4 7 v 10
4 2.4 x 48 x 0.1 m 10 cm
f 100
The distance between two successive nodes
0I
Comparing it with B = k , we get, k = 7. 10
48 x = 5cm
2 2
53. (6 mm) 55. (a = 2)
Initially, the pressure of air column above water is P1 = 105 Let us consider a spherical shell
Nm–2 and volume V1 (500 H ) A , where A is the area of of radius x and thickness dx. The
volume of this shell is 4 x2(dx).
cross-section of the vessel. The charge enclosed in this dx
x
Finally, the volume of air column above water is spherical shell is
300 A. If P2 is the pressure of air then
dq (4 x 2 )dx kx a
P2 gh 105
dq 4 kx 2 a
dx .
200 For r = R :
P2 103 10 105 The total charge enclosed in the sphere of radius R is
1000
R
R3 a
Q 4 k x 2 a dx 4 k
3 a .
P1 105 N / m2 P2
0

V1 (500 H ) A The electric field at r = R is


500 mm

V2 300 A
1 4 kR3 a
1 4 k 1 a
E1 R
4 2 4 3 a
0 (3 a ) R 0
H
200 mm

For r = R/2 :
The total charge enclosed in the sphere of radius R/2 is

Initially Finally R/2


4 k ( R / 2)3 a
Q' 4 k x 2 a dx
3 a
0

P2 4
9.8 10 N / m 2 The electric field at r = R/2 is
1 a
As the temperature remains constant, according to Boyle’s 1 4 k ( R / 2)3 a 1 4 k R
E2
law 4 0 3 a ( R / 2) 2 4 0 3 a 2
PV
1 1 P2V2 1
Given, E2 E1
8
105 (500 H ) A (9.8 10 4 ) 300 A
1 a
H = 206 mm 1 4 k R 1 1 4 k 1 a
R
4 (3 a ) 2 3 4 3 a
The fall of height of water level due to the opening of 0 2 0
orifice = 206 – 200 = 6 mm 1+a=3 a=2
EBD_780
2009- 44 Target IIT-JEE
56. ( = 9)
4(10 ) 4
Heat flow from P to A :
3 27
A P B 90 – 9 =
Ice Water
0°C 100°C 10 90
x (10 )x 9
57. (4 m/s)
The heat flow per unit time The velocity of B just after collision with A is
KA(400 0) (mB m A )uB 2m A u A
Q mL fus … (1) vB
x mB mA m A mB
where K = thermal conductivity of rod and A its area of
cross-section, m = mass of ice melting /sec.
Heat flow from P to B : m 2m m
The heat flows per unit time
A B C
KA(400 100)
Q mLvap … (ii) 0 2m 9
(10 ) x 6m / s
m 2m
m = mass of water evaporated/sec
The collision between B and C is completely inelastic.
Dividing (1) by (ii), we get
mB vB = 0(mB + mc) v
400(10 ) L fus 80 6 2m
300 Lvap 540 v 4m/s.
2m m
IIT-JOINT ENTRANCE EXAMINATION (IIT-JEE)
SOLVED PAPER - 2008

PAPER -1
Useful Data: 2. Students I, II and III perform an experiment for measuring
Planck’s constant h = 4.1 × 10–15 eV.s the acceleration due to gravity (g) using a simple pendulum.
Velocity of light c = 3 × 108 m/s They use different lengths of the pendulum and /or record
time for different number of oscillations. The observations
SECTION – I are shown in the table.
Straight Objective Type Least count for length = 0.1 cm
This section contains 6 multiple choice questions. Each question Least count for time = 0.1 s
has 4 choices (a), (b), (c) and (d), out of whichONLY ONE is correct.
Student Length of the No. of Total time Time
1. Figure shows three resistor configurations R1, R2 and R3 pendulum oscillations for (n) period
connected to 3V battery. If the power dissipated by the (cm) (n) oscillations (s)
configuration R1, R2 and R3 is P1, P2 and P3, respectively, (s)
then – I 64.0 8 128.0 16.0
II 64.0 4 64.0 16.0
III 20.0 4 36.0 9.0

If EI, EII and E III are the percentage errors in g, i.e.,

g
3V 3V 100 for students I, II and III, respectively, then
g

(a) EI = 0 (b) EI is minimum


(c) EI = EII (d) EII is maximum
3. Which one of the following statements is WRONG in the
R1 context of X-rays generated from a X-ray tube?
R2
(a) Wavelength of characteristic X-rays decreases when
the atomic number of the target increases.
(b) Cut-off wavelength of the continuous X-rays depends
on the atomic number of the target
(c) Intensity of the characteristic X-rays depends on the
electrical power given to the X-ray tube
3V (d) Cut-off wavelength of the continuous X-rays depends
on the energy of the electrons in the X-ray tube
4. Two beams of red and violet colours are made to pass
separately through a prism (angle of the prism is 60°). In the
position of minimum deviation, the angle of refraction will
be
R3 (a) 30° for both the colours
(b) greater for the violet colour
(a) P1 > P2 > P3 (b) P1 > P3 > P2 (c) greater for the red colour
(c) P2 > P1 > P3 (d) P3 > P2 > P1
(d) equal but not 30° for both the colours
EBD_780
2008- 2 Target IIT-JEE
5. A spherically symmetric gravitational system of particles 8. The balls, having linear momenta p1 pi and p2 pi ,
for r R
has a mass density
0 undergo a collision in free space. There is no external force
0 for r R
acting on the balls. Let p1 and p2 be their final momenta.
where 0 is a constant. A test mass can undergo circular
motion under the influence of the gravitational field of The following option (s) is (are) NOT ALLOWED for any
particles. Its speed v as a function of distance r (0 < r < ) non-zero value of p, a1, a2, b1, b2, c1 and c2.
from the centre of the system is represented by –
v p1 a1iˆ b1ˆj c1kˆ p1 c1kˆ
v (a) (b)
p2 a 2 ˆi b2 ˆj p2 c 2 kˆ

p1 a1ˆi b1ˆj c1kˆ p1 a1ˆi b1ˆj


(c) (d)
(a) (b) p2 a 2 ˆi b 2 ˆj c1kˆ p2 a 2 ˆi b1ˆj
9. In a Young’s double slit experiment, the separation between
R r R r the two slits is d and the wavelength of the light is . The
v v intensity of light falling on slit 1 is four times the intensity of
light falling on slit 2. Choose the correct choice(s).
(a) If d = , the screen will contain only one maximum
(b) If < d < 2 , at least one more maximum (besides the
(c) (d) central maximum) will be observed on the screen
(c) If the intensity of light falling on slit 1 is reduced so
R r R r
that it becomes equal to that of slit 2, the intensities of
6. 2
An ideal gas is expanding such that PT = constant. The the observed dark and bright fringes will increase
coefficient of volume expansion of the gas is –
(d) If the intensity of light falling on slit 2 is increased so
(a) 1/T (b) 2/T
that it becomes equal to that of slit 1, the intensities of
(c) 3/T (d) 4/T
the observed dark and bright fringes will increase
10. A particle of mass m and charge q, moving with velocity v
SECTION - II enters Region II normal to the boundary as shown in the
Multiple Correct Answer Type figure. Region II has a uniform magnetic field B perpendicu-
This section contains 4 multiple correct answer(s) type questions. lar to the plane of the paper. The length of the Region II is
Each question has 4 choices (a), (b), (c) and (d), out of which ONE Choose the correct choice(s).
OR MORE is/are correct.

7. Assume that the nuclear binding energy per nucleon (B/A) Region I Region II Region III
versus mass number (A) is as shown in the figure. Use this
××××××
plot to choose the correct choice(s) given below.
××××××
××××××
B/A v ××××××
××××××
8
6
(a) The particle enters Region III only if its velocity
4
2 q B
v
0 A m
100 200
(b) The particle enters Region III only if its velocity
(a) Fusion of two nuclei with mass numbers lying in the
range of 1 < A < 50 will release energy. q B
v
(b) Fusion of two nuclei with mass numbers lying in the m
range of 51 < A < 100 will release energy
(c) Fission of a nucleus lying in the mass range of 100 < A (c) Path length of the particle in Region II is maximum when
< 200 will release energy when broken into two equal q B
fragments velocity v
m
(d) Fission of a nucleus lying in the mass range of 200 < A
< 260 will release energy when broken into two equal (d) Time spent in Region II is same for any velocity v as
fragments long as the particle returns to Region I
2008 IIT-JEE Solved Paper 3
2008-

14. STATEMENT - 1 : An astronaut in an orbiting space station


SECTION - III above the earth experiences weightlessness.
Reasoning Type because
This section contains 4 reasoning type questions. Each question STATEMENT - 2 : An object moving the earth under the
has 4 choices (a), (b), (c) and (d), out of which ONLY ONE is influence of Earth’s gravitational force is in a state of
correct. “free-fall”.
11. STATEMENT-1 : Two cylinders, one hollow (metal) and the (a) STATEMENT- 1 is True, STATEMENT-2 is True,
other solid (wood) with the same mass and identical STATEMENT-2 is a correct explanation for
dimensions are simultaneously allowed to roll without STATEMENT -1
slipping down an inclined plane from the same height. The (b) STATEMENT -1 is True, STATEMENT -2 is True;
hollow cylinder will reach the bottom of the inclined plane STATEMENT-2 is NOT a correct explanation for
first. STATEMENT - 1
and (c) STATEMENT - 1 is True, STATEMENT- 2 is False
STATEMENT-2 : By the principle of conservation of
(d) STATEMENT -1 is False, STATEMENT -2 is True
energy, the total kinetic energies of both the cylinders are
identical when they reach the bottom of the incline. SECTION - IV
(a) STATEMENT-1 is True, STATEMENT-2 is True; Linked Comprehension Type
STATEMENT-2 is a correct explanation for This section contains 3 paragraphs. Based upon each paragraph,
STATEMENT-1 3 multiple choice questions have to be answered. Each question
(b) STATEMENT-1 is True, STATEMENT-2 is True; has 4 choices (a), (b), (c) and (d), out of which ONLY ONE is
STATEMENT-2 is NOT a correct explanation for STATE- correct.
MENT-1
Paragraph for Question Nos. 15 to 17
(c) STATEMENT -1 is True, STATEMENT-2 is False
A small block of mass M moves on a frictionless surface of an
(d) STATEMENT -1 is False, STATEMENT-2 is True inclined plane, as shown in figure. The angle of the incline suddenly
12. STATEMENT-1 : The stream of water flowing at high speed changes from 60° to 30° at point B. The block is initially at rest at
from a garden hose pipe tends to spread like a fountain A. Assume that collisions between the block and the incline are
when held vertically up, but tends to narrow down when totally inelastic (g = 10 m/s2).
held vertically down.
and A M
STATEMENT-2 : In any steady flow of an incompressible v
fluid, the volume flow rate of the fluid remains constant.
60° B
(a) STATEMENT-1 is True, STATEMENT-2 is True;
STATEMENT-2 is a correct explanation for STATE-
MENT-1
(b) STATEMENT-1 is True, STATEMENT-2 is True; 30° C
STATEMENT-2 is NOT a correct explanation for STATE- 3m 3 3m
MENT-1
(c) STATEMENT -1 is True, STATEMENT-2 is False 15. The speed of the block at point B immediately after it strikes
(d) STATEMENT -1 is False, STATEMENT-2 is True the second incline is –
13. STATEMENT-1 : In a Meter Bridge experiment, null point
A
for an unknown resistance is measured. Now, the unknown
resistance is put inside an enclosure maintained at a higher 3m
temperature. The null point can be obtained at the same 60° B
point as before by decreasing the value of the standard v
resistance. 3m 30°
and
30° C
STATEMENT-2 : Resistance of a metal increases with
increase in temperature. 3m 3 3m
(a) STATEMENT-1 is True, STATEMENT-2 is True;
STATEMENT-2 is a correct explanation for (a) 60 m/s (b) 45 m/s
STATEMENT-1 (c) 30 m/s (d) 15 m/s
(b) STATEMENT-1 is True, STATEMENT-2 is True;
16. The speed of the block at point C, immediately before it
STATEMENT-2 is NOT a correct explanation for leaves the second incline is
STATEMENT-1
(c) STATEMENT -1 is True, STATEMENT-2 is False (a) 120 m/s (b) 105 m/s
(d) STATEMENT -1 is False, STATEMENT-2 is True (c) 90 m/s (d) 75 m/s
EBD_780
2008- 4 Target IIT-JEE
17. If collision between the block and the incline is completely 20. The buoyancy force acting on the gas bubble is (Assume R
elastic, then the vertical (upward) component of the velocity is the universal gas constant)
of the block at point B, immediately after it strikes the second
(P0 gH) 2 / 5
incline is – (a) nRgT0
(P0 gy)7 / 5
(a) 30 m/s (b) 15 m/s
nRgT0
(c) 0 (d) 15 m/s (b) 2/5
(P0 gH) [P0 g (H y)]3/ 5
Paragraph for Question Nos. 18 to 20
A small spherical monoatomic ideal gas bubble ( = 5/3) is trapped
(P0 gH)3 / 5
inside a liquid of density (see figure). Assume that the bubble (c) nRgT0
does not exchange any heat with the liquid. The bubble contains (P0 gy)8 / 5
n moles of gas. The temperature of the gas when the bubble is at
nRgT0
the bottom is T0, the height of the liquid is H and the atmospheric (d)
3/ 5
pressure is P0 (Neglect surface tension). (P0 gH) [P0 g (H y)]2 / 5
Paragraph for Question Nos. 21 to 23
P0 In a mixture of H-He+ gas (He+ is singly ionized He atom), H
atoms and He+ ions are excited to their respective first excited
Liquid states. Subsequently, H atoms transfer their total excitation energy
to He+ ions (by collisions). Assume that the Bohr model of atom is
H exactly valid.
y 21. The quantum number n of the state finally populated in He+
ions is –
(a) 2 (b) 3
18. As the bubble moves upwards, besides the buoyancy force
(c) 4 (d) 5
the following forces are acting on it
22. The wavelength of light emitted in the visible region by He+
(a) Only the force of gravity
ions after collisions with H atoms is –
(b) The force due to gravity and the force due to the
(a) 6.5 × 10–7 m (b) 5.6 × 10–7 m
pressure of the liquid
(c) 4.8 × 10–7 m (d) 4.0 × 10–7 m
(c) The force due to gravity, the force due to the pressure
of the liquid and the force due to viscosity of the liquid 23. The ratio of the kinetic energy of the n = 2 electron for the H
atom to that of He+ ion is –
(d) The force due to gravity and the force due to viscosity
of the liquid (a) 1/4 (b) 1/2
19. When the gas bubble is at a height y from the bottom, its (c) 1 (d) 2
temperature is –

2/5 CHEMISTRY
P0 gH
(a) T0 SECTION – I
P0 gy
Straight Objective Type
2/5 This section contains 6 multiple choice questions. Each question
P0 g (H y) has 4 choices (a), (b), (c) and (d), out of which ONLY ONE is
(b) T0
P0 gH correct.

24. Under the same reaction conditions, initial concentration of


3/5
P0 gH 1.386 mol dm–3 of a substance becomes half in 40 seconds
(c) T0
P0 gy and 20 seconds through first order and zero order kinetics,
respectively. Ratio (k1/k0) of the rate constant for first order
(k1) and zero order (k0) of the reaction is –
3/5
P0 g (H y) (a) 0.5 mol–1 dm3 (b) 1.0 mol dm–3
(d) T0
P0 gH (c) 1.5 mol dm–3 (d) 2.0 mol–1 dm3
2008 IIT-JEE Solved Paper 2008-5
25. 2.5 ml of (2/5) M weak monoacidic base (Kb = 1 × 10–12 at 25°) 31. A gas described by van der Waals equation –
is titrated (2/15) M HCl in water at 25°C. The concentration (a) behave similar to an ideal gas in the limit of large molar
of H+ at equivalence point is (Kw = 1 × 10–14 at 25°C) volumes
(a) 3.7 × 10–14 M (b) 3.2 × 10–7 M (b) behaves similar to an ideal gas is in limit of large
(c) 3.2 × 10–2 M (d) 2.7 × 10–2 M pressures
26. Native silver metal forms a water soluble complex with a (c) is characterised by van der Waals coefficients that are
dilute aqueous solution of NaCN in the presence of dependent on the identity of the gas but are
(a) nitrogen (b) oxygen independent of the temperature.
(c) carbon dioxide (d) argon (d) has the pressure that is lower than the pressure exerted
27. Aqueous solution of Na2S2O3 on reaction with Cl2 gives – by the same gas behaving ideally
(a) Na2S4O6 (b) NaHSO4 32. The correct statements(s) concerning the structures E,F and
G is (are) –
(c) NaCl (d) NaOH
28. Hyperconjugation involves overlap of the following orbitals H3C O H3C OH H3C CH3
(a) - (b) - H3C CH3 H3C CH3 H3C OH
(c) p- p (d) - (E) (F) (G)
29. The major product of the following reaction is –
(a) E,F, and G are resonance structures
Br
(b) E,F and E, G are tautomers
Me
(c) F and G are geometrical isomers
F
Ph S Na
(d) F and G are diastereomers
dimethylformamide 33. The correct statement(s) about the compound given below
is (are)
NO2
Cl H
SPh CH3
Me Me SPh
H3C
F F H
Cl
(a) (b) (a) The compound is optically active
NO2 NO2 (b) The compound possesses centre of symmetry
(c) The compound possesses plane of symmetry
Me Br Me SPh
(d) The compound possesses axis of symmetry
SPh SPh

(c) (d) SECTION - III


NO2 NO2 Reasoning Type
This section contains 4 reasoning questions. Each question has 4
choices (a), (b), (c) and (d) out of which ONLY ONE is correct.
SECTION - II
Multiple Correct Answer Type
34. Statement -1 For every chemical reaction at equilibrium,
This section contains 4 multiple choice answer(s) type questions.
standard Gibbs energy of reaction is zero.
Each question has 4 choices (a), (b), (c) and (d), out of which ONE
Statement -2 At constant temperature and pressure, chemical
OR MORE is/are correct.
reactions are spontaneous in the direction of decreasing
30. A solution of colourless salt H on boiling with excess NaOH Gibbs energy.
produces a non-flammable gas. The gas evolution ceases (a) Statement-1 is True, Statement-2 is True; Statement-2
after sometime. Upon addition of Zn dust to the same is a correct explanation for Statement-1.
solution, the gas evolution restarts. The colourless salt (s) (b) Statement-1 is True, Statement-2 is True; Statement-2
H is (are) is NOT a correct explanation for Statement-1
(a) NH4NO3 (b) NH4NO2 (c) Statement-1 is True, Statement-2 is False
(c) NH4Cl (d) (NH4)2SO4 (d) Statement-1 is False, Statement-2 is True
EBD_780
2008- 6 Target IIT-JEE
35. Statement-1 : The plot of atomic number (y-axis) versus (c) Between nitrates and phosphates, nitrates are less
number of neutrons (x-axis) for stable nuclei shows a abundant in earth’s crust
curvature towards x-axis from the line of 45° slope as the (d) Oxidation of nitrates is possible in soil
atomic number is increased. 39. Among the following, the correct statement is
Statement-2 : Proton-proton electrostatic repulsions begin (a) Between NH3 and PH3, NH3 is a better electron donor
to overcome attractive forces involving protons and because the lone pair of electrons occupies spherical
neutrons in heavier nuclides. s-orbital and is less directional
(a) Statement-1 is True, Statement-2 is True; Statement-2 (b) Between NH3 and PH3 , PH3 is a better electron donor
is a correct explanation for Statement-1. because the lone pair of electrons occupies sp 3 orbital
(b) Statement-1 is True, Statement-2 is True; Statement-2 and is more directional
is NOT a correct explanation for Statement-1 (c) Between NH3 and PH3 , NH3 is a better electron donor
(c) Statement-1 is True, Statement-2 is False because the lone pair of electrons occupies sp 3 orbital
(d) Statement-1 is False, Statement-2 is True and is more directional
36. Statement-1 : Pb+4 compounds are stronger oxidising agents (d) Between NH3 and PH3 , PH3 is a better electron donor
than Sn4+ compounds because the lone pair of electrons occupies spherical
Statement-2 : The higher oxidation states for the group 14 s-orbital and is less directional
elements are more stable for the heavier members of the 40. White phosphorus on reaction with NaOH gives PH3 as one
group due to 'inert pair effect'. of the products. This is a
(a) Statement-1 is True, Statement-2 is True; Statement-2 (a) dimerization reaction
is a correct explanation for Statement-1.
(b) disporportionation reaction
(b) Statement-1 is True, Statement-2 is True; Statement-2
(c) condensation reaction
is NOT a correct explanation for Statement-1
(d) precipitation reaction
(c) Statement-1 is True, Statement-2 is False
(d) Statement-1 is False, Statement-2 is True Paragraph for Question Nos. 41 to 43
37. Statement-1 : Bromobenzene upon reaction with Br2/Fe Properties such as boiling point, freezing point and vapour pressure
gives 1,4-dibromobenzene as the major product. of a pure solvent change when solute molecules are added to get
Statement-2 : In bromobenzene, the inductive effect of the homogeneous solution. These are called colligative properties.
bromo group is more dominant than the mesomeric effect in Application of colligative properties are very useful in day-to-day
directing the incoming electrophile. life. One of its example is the use of ethylene glycol and water
(a) Statement-1 is True, Statement-2 is True; Statement-2 mixture as anti-freezing liquid in the radiator of automobiles.
is a correct explanation for Statement-1. A solution M is prepared by mixing ethanol and water. The mole
(b) Statement-1 is True, Statement-2 is True; Statement-2 fraction of ethanol in the mixture is 0.9
is NOT a correct explanation for Statement-1 Given : Freezing point depression constant of water (Kf water)
(c) Statement-1 is True, Statement-2 is False = 1.86 K kg mol–1
(d) Statement-1 is False, Statement-2 is True Freezing point depression constant of ethanol (K f ethanol)
= 2.0 K kg mol–1
Boiling point elevation constant of water (K b water )
SECTION - IV
= 0.52 K kg mol–1
Linked Comprehension Type Boiling poin t elevation constant of ethan ol (K b ethanol )
This section contains 3 paragraphs. Based upon each paragraph, = 1.2 K kg mol–1
3 multiple choice question have to be answered. Each question Standard freezing point of water = 273 K
has 4 choices (a), (b), (c) and (d), out of which ONLY ONE is
Standard boiling point of water = 155.7 K
correct.
Standard boiling point of water = 373 K
Paragraph for Question Nos. 38 to 40 Standard boiling point of ethanol = 351.5 K
There are some deposits of nitrates and phosphates in earth’s Vapour pressure of pure water = 32.8 mm Hg
crust. Nitrates are more soluble in water. Nitrates are difficult to Vapour pressure of pure ethanol = 40 mm Hg
reduce under the laboratory conditions but microbes do it easily. Molecular weight of water = 18 g mol–1
Ammonia forms large number of complexes with transition metal Molecular weight of ethanol = 46 g mol–1
ions. Hybridization easily explains the ease of sigma donation
capability of NH3 and PH3. Phosphine is a flammable gas and is In answering the following questions, consider the solution to be
prepared from white phosphorous. ideal dilute solutions and solutes to be nonvolatile and non-
38. Among the following, the correct statement is dissociative.
(a) Phosphates have no biological significance in humans 41. The freezing point of the solution M is
(b) Between nitrates and phosphates, phosphates are less (a) 268.7 K (b) 268.5 K
abundant in earth’s crust (c) 234.2 K (d) 150.9 K
2008 IIT-JEE Solved Paper 7
2008-

42. The vapour pressure of the solution M is MATHEMATICS


(a) 39.3 mm Hg (b) 36.0 mm Hg
(c) 29.5 mm Hg (d) 28.8 mm Hg SECTION - I
43. Water is added to the solution M such that the mole fraction Straight Objective Type
of water in the solution becomes 0.9. The boiling point of
This section contains 6 multiple choice questions. Each question
this solution is
has choices (a), (b), (c) and (d), out of which ONLY ONE is correct.
(a) 380.4 K (b) 376.2 K
(c) 375.5 K (d) 354.7 K
47. If 0 < x < 1, then
Paragraph for Question Nos. 44 to 46
In the following reaction sequence, product I, J and L are formed. 1 x 2 [{x cos (cot–1 x) + sin (cot–1 x)}2 – 1]1/2 =
K represents a reagent. x
1.Mg / ether (a) (b) x
1.NaBH4 2.CO 2 K 1 x2
Hex 3 ynal 2.PBr3
I J
3.H3O
(c) x 1 x 2 (d) 1 x2
6
5 2 48. Consider the two curves C1 : y2 = 4x, C2 : x2 + y2 – 6x + 1
7 4 Cl = 0. Then,
Me 1 H2
3 Pd/ BaSO4 quioline
L (a) C1 and C2 touch each other only at one point.
O (b) C1 and C2 touch each other exactly at two points
44. The structure of the product I is – (c) C1 and C2 intersect (but do not touch) at exactly two
points
(d) C1 and C2 neither intersect nor touch each other
(a) Me Br
49. The edges of a parallelopiped are of unit length and are
(b) Me
parallel to non-coplanar unit vectors aˆ, bˆ, cˆ such that
Br
1
aˆ . bˆ bˆ . cˆ cˆ . aˆ .
(c) Me Br 2
(d) Me Br Then, the volume of the parallelopiped is

45. The structures of compound J and K, respectively, are 1 1


(a) (b)
Me 2 2 2
(a) COOH and SOCl2
3 1
OH (c) (d)
2 3
(b) Me and SOCl2
50. Let a and b be non-zero real numbers. Then, the equation
O (ax2 + by2 + c) (x2 – 5xy + 6y2) = 0 represents
(a) four straight lines, when c = 0 and a, b are of the same
(c) Me
sign.
COOH (b) two straight lines and a circle, when a = b, and c is of
(d) Me COOH and CH3SO2Cl sign opposite to that of a
(c) two straight lines and a hyperbola, when a and b are of
46. The structure of product L is the same sign and c is of sign opposite to that of a
Me CHO (d) a circle and an ellipse, when a and b are of the same
(a)
sign and c is of sign opposite to that of a
51. The total number of local maxima and local minima of the
(b) Me CHO
(2 x )3 , – 3 x –1
CHO function f ( x ) is
2/3
(c) x , –1 x 2
Me
(a) 0 (b) 1
(d) Me CHO (c) 2 (d) 3
EBD_780
2008- 8 Target IIT-JEE

n
( x – 1) n n n 1
n
52. Let g ( x) m
; 0 < x < 2, m and n are integers, 56. Let S n 2 2 and Tn 2
for n
log cos ( x – 1) k 1n kn k
k 0n kn k 2
= 1, 2, 3, ............ Then,
m 0, n 0 , and let p be the left hand derivative of |x – 1| at

x = 1. If lim g(x) = p, then (a) Sn (b) Sn


x 1 3 3 3 3
(a) n = 1, m = 1 (b) n = 1, m = – 1
(c) Tn (d) Tn
(c) n = 2, m = 2 (d) n > 2, m = n 3 3 3 3

SECTION - II SECTION - III


Multiple Correct Answer Type Reasoning Type
This section contains 4 multiple correct answer(s) type questions. This section contains 4 reasoning type questions. Each question
Each question has 4 choices (a), (b), (c) and (d), out of which ONE has 4 choices (a), (b), (c) and (d), out of which ONLY ONE is
correct.
OR MORE is/are correct.

57. Consider the system of equations


53. Let P(x1, y1) and Q(x2, y2), y1 < 0, y2 < 0, be the end points of x – 2y + 3z = –1
the latus rectum of the ellipse x2 + 4y2 = 4. The equations of –x + y – 2z = k
parabolas with latus rectum PQ are x – 3y + 4z = 1
STATEMENT-1 : The system of equations has no solution
(a) x2 2 3y 3 3 (b) x2 – 2 3 y 3 3 for k 3
and
(c) x2 2 3y 3 3 (d) x2 2 3y 3 3
1 3 1
54. A straight line through the vertex P of a triangle PQR STATEMENT-2 : The determinant 1 2 k 0 , for
intersects the side QR at the point S and the circumcircle of
1 4 1
the triangle PQR at the point T. If S is not the centre of the
circumcircle, then k 3.
(a) STATEMENT - 1 is True, STATEMENT - 2 is True;
1 1 2 1 1 2 STATEMENT - 2 is a correct explanation for
(a) PS ST (b) PS ST
QS SR QS SR STATEMENT - 1
(b) STATEMENT - 1 is True, STATEMENT - 2 is True;
1 1 4 1 1 4 STATEMENT - 2 is NOT a correct explaination for
(c) (d) STATEMENT - 1
PS ST QR PS ST QR
(c) STATEMENT - 1 is True, STATEMENT - 2 is False
55. Let f (x) be a non-constant twice differentiable function (d) STATEMENT - 1 is False, STATEMENT - 2 is True
58. Consider the system of equations ax + by = 0; cx + dy = 0,
1 where a, b, c, d {0, 1}
definied on ( , ) such that f (x) = f (1 – x) and f ' 0.
4 STATEMENT - 1 : The probability that the system of
Then, 3
equations has a unique solution is .
(a) f "(x) vanishes at least twice on [0, 1] 8
and
1 STATEMENT - 2 : The probability that the system of
(b) f' 0
2 equations has a solution is 1.
(a) STATEMENT - 1 is True, STATEMENT - 2 is True;
1/ 2
1 STATEMENT - 2 is a correct explanation for
(c) f x sin x dx 0 STATEMENT - 1
2
1/ 2 (b) STATEMENT - 1 is True, STATEMENT - 2 is True;
STATEMENT - 2 is NOT a correct explaination for
1/ 2 1 STATEMENT - 1
(d) f (t ) esin t dt f (1 t ) esin t dt (c) STATEMENT - 1 is True, STATEMENT - 2 is False
0 1/ 2 (d) STATEMENT - 1 is False, STATEMENT - 2 is True
2008 IIT-JEE Solved Paper 9
2008-

59. Let f and g be real valued functions defined on interval 62. Let z be any point in A B C.
(–1, 1) such that g" (x) is continuous, g (0) 0. g'(0) = 0,
Then, |z + 1 – i|2 + |z – 5 – i|2 lies between
g ''(0) 0 , and f (x) = g (x) sin x (a) 25 and 29 (b) 30 and 34
STATEMENT-1 : xlim0 [g(x) cot x – g(0) cosec x] = f"(0) (c) 35 and 39 (d) 40 and 44
and 63. Let z be any point A B C and let w be any point
STATEMENT-2 : f '(0) = g(0) satisfying |w – 2 – i| < 3. Then, |z| – |w| + 3 lies between
(a) STATEMENT - 1 is True, STATEMENT - 2 is True;
STATEMENT - 2 is a correct explanation for (a) –6 and 3 (b) –3 and 6
STATEMENT - 1 (c) –6 and 6 (d) –3 and 9
(b) STATEMENT - 1 is True, STATEMENT - 2 is True;
Paragraph for Question Nos. 64 to 66
STATEMENT - 2 is NOT a correct explaination for
STATEMENT - 1 A circle C of radius 1 is inscribed in an equilateral triangle PQR.
(c) STATEMENT - 1 is True, STATEMENT - 2 is False The points of contact of C with the sides PQ, QR, RP are D, E, F,
(d) STATEMENT - 1 is False, STATEMENT - 2 is True respectively. The line PQ is given by the equation 3x y 6 0
60. Consider three planes
P1 : x – y + z = 1
3 3 3
P2 : x + y – z = 1 and the point D is ,
P3 : x – 3y + 3z = 2 2 2 . Further, it is given that the origin
Let L1, L2, L3 be the lines of intersection of the planes P2 and and the centre of C are on the same side of the line PQ.
P3, P3 and P1, P1 and P2, respectively.
64. The equation of circle C is
STATEMENT - 1 : At least two of the lines L1, L2 and L3 are
non-parallel
(a) ( x 2 3)2 ( y 1)2 1
and
STATEMENT - 2 : The three planes doe not have a common 2
point. 1
(b) ( x 2 3) 2 y 1
(a) STATEMENT - 1 is True, STATEMENT - 2 is True; 2
STATEMENT - 2 is a correct explanation for
STATEMENT - 1 (c) (x 3)2 ( y 1) 2 1
(b) STATEMENT - 1 is True, STATEMENT - 2 is True;
STATEMENT - 2 is NOT a correct explanation for (d) (x 3)2 ( y 1) 2 1
STATEMENT - 1
(c) STATEMENT - 1 is True, STATEMENT - 2 is False 65. Points E and F are given by
(d) STATEMENT - 1 is False, STATEMENT - 2 is True
3 3 3 1
(a) , , ( 3, 0) (b) , , ( 3, 0)
2 2 2 2
SECTION - IV
Linked Comprehension Type
This section contains 3 paragraphs. Based upon each paragraph, 3 3 3 1 3 3 3 1
(c) , , , (d) , , ,
3 multiple choice questions have to be answered. Each question 2 2 2 2 2 2 2 2
has 4 choices (a), (b), (c) and (d), out of which ONLY ONE is
correct. 66. Equations of the sides QR, RP are

2 2
Paragraph for Question Nos. 61 to 63 (a) y x 1, y x 1
3 3
Let A, B, C be three sets of complex numbers as defined below
A {z : Im z 1} 1
(b) y x, y 0
B {z :| z 2 i | 3} 3

C {z : Re((1 i) z ) 2}
3 3
61. The number of elements in the set A B C is (c) y x 1, y x 1
2 2
(a) 0 (b) 1
(c) 2 (d) (d) y 3 x, y 0
EBD_780
2008- 10 Target IIT-JEE
Paragraph for Question Nos. 67 to 69 b
x
dx bf (b) af (a)
Consider the functions defined implicitly by the equation (a) ( f ( x))2 1
a3
y 3 – 3y + x = 0 on various intervals in the real line. If
b
x ( , 2) (2, ) , the equation implicitly defines a unique x
dx bf (b) af (a )
(b) ( f ( x )) 2 1
real valued differentiable function y = f (x). If x ( 2, 2) , the a3
equation implicitly defines a unique real valued differentiable func- b
tion y = g(x) satisfying g(0) = 0. x
dx bf (b ) af (a )
(c) ( f ( x)) 2 1
67. If f ( 10 2) 2 2 , then f ''( 10 2) a3

4 2 4 2 b
(a) (b) x
3 3 dx bf (b) af (a )
7 3 7 3 (d) ( f ( x )) 2 1
a3
4 2 4 2 1
(c) (d)
733 733 69. g '( x ) dx
68. The area of the region bounded by the curve y = f (x), the 1
x-axis, and the lines x = a and x = b, where a b 2, (a) 2g (–1) (b) 0
is (c) –2g (1) (d) 2g (1)

PAPER -II

SECTION – I (b) The potential energy of the system is zero


Straight Objective Type (c) The magnitude of the force between the charges at C
This section contains 9 multiple choice questions. Each
question has 4 choices (a), (b), (c) and (d), out of which q2
and B is
2
ONLY ONE is correct. 54 0R

1. A radioactive sample S1 having an activity 5µCi has twice q


the number of nuclei as another sample S2 which has an (d) The potential at point O is
12 0R
activity of 10 µCi. The half lives of S1 and S2 can be
(a) 20 years and 5 years, respectively 3. A transverse sinusoidal wave moves along a string in the
(b) 20 years and 10 years, respectively positive x-direction at a speed of 10 cm/s. The wavelength
(c) 10 years each of the wave is 0.5 m and its amplitude is 10 cm. At a particular
(d) 5 years each time t, the snap–shot of the wave is shown in figure. The
2. Consider a system of three charges q/3,q/3 and 2q/3 placed velocity of point P when its displacement is 5 cm is –
at points A, B and C, respectively, as shown in the figure.
Take O to be the centre of the circle of radius R and angle y
CAB = 60°
y
P
B
x
C
x
60° O

A
3 ˆ 3
(a) jm/s (b) ĵ m / s
q 50 50
(a) The electric field at point O is 2
directed along
8 0R 3 3
the negative x-axis (c) î m / s (d) î m / s
50 50
2008 IIT-JEE Solved Paper 11
2008-

4. A parallel plate capacitor C with plates of unit area and maximum distance y towards wall 2. Displacements x and y
separation d is filled with a liquid of dielectric constant are measured with respect to the equilibrium position of the
K = 2. The level of liquid is d/3 initially. Suppose the liquid block B. The ratio y/x is –
level decreases at a constant speed v, the time constant as a
function of time t is– 2 1
M2 M1
S2 S1
B I
x 1
2
C M2 M1
S2 S1
d d/3 R B II
x
(a) 4 (b) 2
(c) 1/2 (d) 1/4
8. A glass tube of uniform internal radius (r) has a valve
6 0R (15d 9vt ) 0R
(a) (b) separating the two identical ends. Initially, the valve is in a
5d 3vt 2d 2 3dvt 9v t 2 2
tightly closed position. End 1 has a hemispherical soap
bubble of radius r. End 2 has sub-hemispherical soap bubble
6 0R (15d 9vt ) 0R
(c) (d) as shown in figure. Just after opening the valve,
5d 3vt 2 2 2
2d 3dvt 9v t
5. A vibrating string of certain length under a tension T
resonates with a mode corresponding to the first overtone
(third harmonic) of an air column of length 75 cm inside a
tube closed at one end. The string also generates 4 beats
per second when excited along with a tuning fork of
2 1
frequency n. Now when the tension of the string is slightly
increased the number of beats reduces 2 per second.
Assuming the velocity of sound in air to be 340 m/s, the (a) air from end 1 flows towards end 2. No change in the
frequency n of the tuning fork in Hz is volume of the soap bubbles
(b) air from end 1 flows towards end 2. Volume of the soap
(a) 344 (b) 336
bubble at end 1 decreases
(c) 117.3 (d) 109.3
(c) no changes occurs
6. A light beam is travelling from Region IV (figure). The
(d) air from end 2 flows towards end 1. volume of the soap
n 0 n0 bubble at end 1 increases
refractive index in regionals I, II, III and IV are n 0 , ,
9. A bob of mass M is suspended by a massless string of
2 6
length L. The horizontal velocity v at position A is just
n0 sufficient to make it reach the point B. The angle at which
and respectively. The angle of incidence for which
8 the speed of the bob is half of that at A, satisfies –
the beam just misses entering region IV is –
B
Region I Region II Region III Region IV

n0 n0 n0 n0
2 6 8

0 0.2m 0.6m
L
(a) sin–1 (3/4) (b) sin–1 (1/8)
(c) sin–1 (1/4) (d) sin–1 (1/3)
7. A block (B) is attached to two unstretched springs S1 and v
A
S2 with spring constants k and 4k, respectively (see fig. I).
The other ends are attached to identical supports M1 and
M2 not attached to the walls. The springs and supports (a) (b)
4 4 2
have negligible mass. There is no friction anywhere. The
block B is displaced towards wall 1 by a small distance x 3 3
(figure II) and released. The block returns and moves a (c) (d)
2 4 4
EBD_780
2008- 12 Target IIT-JEE
(a) STATEMENT-1 is True, STATEMENT-2 is True;
SECTION – II STATEMENT-2 is a correct explanation for
Reasoning Type STATEMENT-1
This section contains 4 reasoning type questions. Each question (b) STATEMENT-1 is True, STATEMENT-2 is True;
has 4 choices (a), (b), (c) and (d), out of which ONLY ONE is STATEMENT-2 is NOT a correct explanation for
correct. STATEMENT-1
10. STATEMENT-1 : For practical purposes, the earth is used (c) STATEMENT -1 is True, STATEMENT-2 is False
as a reference at zero potential in electrical circuits. (d) STATEMENT -1 is False, STATEMENT-2 is True
and
STATEMENT-2 : The electrical potential of a sphere of
radius R with charge Q uniformly distributed on the surface SECTION-III
Linked Comprehension Type
Q
is given by 4 . This section contains 2 paragraphs. Based upon each paragraph,
0R 3 multiple choice questions have to be answered. Each question
(a) STATEMENT-1 is True, STATEMENT-2 is True; has 4 choices (a), (b), (c) and (d), out of which ONLY ONE is
STATEMENT-2 is a correct explanation for correct.
STATEMENT-1 Paragraph for Question Nos. 14 to 16
(b) STATEMENT-1 is True, STATEMENT-2 is True; The nuclear charge (Ze) is non-uniformly distributed within a
STATEMENT-2 is NOT a correct explanation for nucleus of radius R. The charge density (r) [charge per unit
STATEMENT-1 volume] is dependent only on the radial distance r from the centre
(c) STATEMENT -1 is True, STATEMENT-2 is False of the nucleus as shown in figure The electric field is only along
(d) STATEMENT -1 is False, STATEMENT-2 is True the radial direction.
11. STATEMENT-1 : The sensitivity of a moving coil
galvanometer is increased by placing a suitable magnetic
material as a core inside the coil.
and d
STATEMENT-2 : Soft iron has a high magnetic permeability
and cannot be easily magnetized or demagnetized.
(a) STATEMENT-1 is True, STATEMENT-2 is True;
STATEMENT-2 is a correct explanation for r
O a R
STATEMENT-1 14. The electric field at r = R is
(b) STATEMENT-1 is True, STATEMENT-2 is True; (a) independent of a
STATEMENT-2 is NOT a correct explanation for (b) directly proportional to a
STATEMENT-1 (c) directly proportional to a2
(c) STATEMENT -1 is True, STATEMENT-2 is False (d) inversely proportional to a
(d) STATEMENT -1 is False, STATEMENT-2 is True 15. For a = 0, the value of d (maximum value of ñ as shown in the
12. STATEMENT-1 : For an observer looking out through the figure) is –
window of a fast moving train, the nearby objects appear to
3Ze 3Ze
move in the opposite direction to the train, while the distant (a) (b)
3
objects appear to be stationary. 4 R R3
and 4Ze Ze
STATEMENT-2 : If the observer and the object are moving (c) 3 (d)
3 R 3 R3
at velocities v 1 and v 2 respectively with reference to a 16. The electric field within the nucleus is generally observed
laboratory frame, the velocity of the object with respect to to be linearly dependent on r. This implies.
(a) a = 0 (b) a = R/2
the observer is v 2 – v 1 .
(c) a = R (d) a = 2R/3
(a) STATEMENT-1 is True, STATEMENT-2 is True; Paragraph for Question Nos. 17 to 19
STATEMENT-2 is a correct explanation for A uniform thin cylindrical disk of mass M and radius R is attached
STATEMENT-1 to two identical massless springs of spring constant k which are
(b) STATEMENT-1 is True, STATEMENT-2 is True; fixed to the wall as shown in the figure. The springs are attached
STATEMENT-2 is NOT a correct explanation for to the axle of the disk symmetrically on either side at a distance d
STATEMENT-1 from its centre. The axle is massless and both the springs and the
(c) STATEMENT -1 is True, STATEMENT-2 is False axle are in horizontal plane.
(d) STATEMENT -1 is False, STATEMENT-2 is True The unstretched length of each spring is L. The disk is initially at
13. STATEMENT-1 : It is easier to pull a heavy object than to its equilibrium position with its centre of mass (CM) at a distance
push it on a level ground and L from the wall. The disk rolls without slipping with velocity
STATEMENT-2 : The magnitude of frictional force depends
on the nature of the two surfaces in contact. V V ˆi . The coefficient of friction is µ.
0 0
2008 IIT-JEE Solved Paper 13
2008-

y 18. The centre of mass of the disk undergoes simple harmonic


motion with angular frequency equal to –
2d k 2k
(a) (b)
M M
2k 4k
(c) (d)
3M 3M
d 19. The maximum value of V0 for which the disk will roll without
d
R
V0 slipping is –
x
17. The net external force acting on the disk when its centre of M M
(a) g (b) g
mass is at displacement x with respect to its equilibrium k 2k
position is
3M 5M
(a) – kx (b) – 2 kx (c) g (d) g
(c) – 2 kx/3 (d) – 4 kx/3 k 2k

SECTION – IV
MATRIX-MATCH TYPE QUESTIONS
This section contains 3 questions. Each question contains statements given in two col-
umns which have to be matched. Statements in Column I are labelled as A, B, C and D
whereas statements in Column II are labelled as p, q, r and s. The answers to these ques-
tions have to be appropriately bubbled as illustrated in the following example.
If the correct matches are A-p, A-r, B-p, B-s, C-r, C-s and D-q, then the correctly bubbled
matrix will be look like the following:

20. Column I gives a list of possible set of parameters measured in some experiments. The variations of the parameters in the form of
graphs are shown in Column II. Match the set of parameters given in Column I with the graph given in Column II. Indicate your
answer by darkening the appropriate bubbles of the 4 × 4 matrix given in the ORS.
Column I Column II
(A) Potential energy of a simple pendulum (y axis) as (p) y
a function of displacement (x axis)

x
y

(B) Displacement (y axis) as a function of time (x axis) (q)


for a one dimensional motion at zero or constant
acceleration when the body is moving along the positive
x-direction.
x
O
y
(C) Range of a projectile (y axis) as a function of its (r)
velocity (x axis) when projected at a fixed angle.

x
O
y

(D) The square of the time period (y axis) of a simple (s)


pendulum as a function of its length (x axis)
x
O
EBD_780
2008- 14 Target IIT-JEE
21. Column I contains a list of processes involving expansion of an ideal gas. Match this with Column II describing the thermodynamic
change during this process. Indicate your answer by darkening the appropriate bubbles of the 4 × 4 matrix given in the ORS.
Column I Column II
(A) An insulated container has two chambers (p) The temperature of the gas decreases
separated by a valve. Chamber I contains an ideal gas and the
Chamber II has vacuum. The valve is opened.

I II

Ideal gas vacuum

(B) An ideal monoatomic gas expands to twice its remains (q) The temperature of the gas increases or
original volume such that its pressure P 1/V2 where V constant
is the volume of the gas
(C) An ideal monoatomic gas expands to twice its original volume (r) The gas loses heat
such that its pressure P 1/V4/3 where V is its volume
(D) An ideal monoatomic gas expands such that its (s) The gas gains heat
pressure P and volume V follows the behaviour
shown in the graph
P

V
V1 2V1
22. An optical component and an object S placed along its optic axis are given in Column I. The distance between the object and the
component can be varied. The properties of images are given in Column II. Match all the properties of images from Column II with
the appropriate components given in Column I. Indicate your answer by darkening the appropriate bubbles of the 4 × 4 matrix
given in the ORS.
Column I Column II

S
(A) (p) real image

S
(B) (q) virtual image

S
(C) (r) magnified image

S
(D) (s) image at infinity
2008 IIT-JEE Solved Paper 2008- 15
CHEMISTRY 27. The correct stability order for the following species is

SECTION – I + + +
O O
Straight Objective Type +
(I) (II) (III) (IV)
This section contains 9 multiple choice questions. Each question
has 4 choices (a), (b), (c) and (d), out of which ONLY ONE is (a) (II) > (IV) > (I) > (III) (b) (I) > (II) > (III) > (IV)
correct. (c) (II) > (I) > (IV) > (III) (d) (I) > (III) > (II) > (IV)
23. Among the following, the surfactant that will form micelles 28. Cellulose upon acetylation with excess acetic anhydride/
in aqueous solution at the lowest molar concentration at H2SO4 (catalytic) gives cellulose triacetate whose structure
ambient condition is : – is
(a) (a)
CH3 (CH 2 )15 N (CH 3 ) 3 Br

(b) CH3 (CH 2 )11 OSO3 Na AcO


O
AcO H O
(c) CH3 (CH 2 )6 COO Na H
O
AcO H OAc H H
H O
(d) CH3 (CH 2 )11 N (CH 3 ) 3 Br O
H OAc H H OAc
H O H
24. Solubility product constant (Ksp) of salts of types MX, MX2
and M3X at temperature T are 4.0 × 10–8, 3.2 × 10–14 & 2.7 × O OAc H
H
H OAc
10–15, respectively. Solubilities (mol dm–3) of the salts at
H OAc
temperature 'T' are in the order –
(a) MX > MX2 > M3X (b) M3X > MX2 > MX
(b)
(c) MX2 > M3X > MX (d) MX > M3X > MX2
25. Electrolysis of dilute aqueous NaCl solution was carried out
AcO
by passing 10 milli ampere current. The time required to
O
liberate 0.01 mol of H2 gas at the cathode is (1 Faraday = AcO O
H
96500 C mol–1) H
O
(a) 9.65 × 104 sec (b) 19.3 × 104 sec AcO
H H O
OH H H
4
(c) 28.95 × 10 sec (d) 38.6 × 104 sec O
H OH H H OH
H O H
26. In the following reaction sequence, the correct structures
of E, F and G are O OH H
H
H OH

O O H OH
Heat I2
[E] NaOH
[F] [G]
OH (c)
Ph *
13
[* implies C labelled carbon)
AcO
AcO
O O AcO O
O H
H H H O
O H
(A) E= F= H H
* * – + H
CH3 O Na G = CHI3 O OAc H H
Ph Ph OAc H
O OAc H
O
O O H OAc
H OAc
H OAc
(B) E= F= * – +
* O Na G = CHI3
Ph CH3 Ph (d)
O O
AcO
AcO
AcO
(C) E= F= * H O
* – + H O H H O
CH3 O Na G = CHI3 H
O
H H
Ph Ph H
O H H H
O O H H
O H H
O
OAc
(D) E= F= OAc OAc
OAc
* – + * OAc OAc
CH3 O Na G = CH3I
Ph Ph
EBD_780
2008- 16 Target IIT-JEE
29. Among the following, the coloured compound is (a) STATEMENT-1 is True, STATEMENT-2 is True;
(a) CuCl (b) K3 [Cu (CN)4] STATEMENT-2 is a correct explanation for
(c) CuF2 (d) [Cu (CH3CN)4]BF4 STATEMENT-1.
30. The IUPAC name of [Ni (NH3)4] [NiCl4] is (b) STATEMENT-1 is True, STATEMENT-2 is True;
(a) Tetrachloronickel (II) - tetraamminenickel (II) STATEMENT-2 is NOT a correct explanation for
(b) Tetraamminenickel (II) - tetrachloronickel (II) STATEMENT-1
(c) Tetraamminenickel (II) - tetrachloronickelate (II) (c) STATEMENT-1 is True, STATEMENT-2 is False
(d) Tetrachloronickel (II) - tetrachloronickelate (0) Ans. (C) (d) STATEMENT-1 is False, STATEMENT-2 is True
31. Both [Ni (CO)4] and [Ni (CN)4]2– are diamagnetic. The 35. STATEMENT-1 : [Fe(H2O)5NO]SO4 is paramagnetic.
hybridisations of nickel in these complexes, respectively, and
are STATEMENT-2 : The Fe in [Fe(H2O)5NO]SO4 has three
(a) sp3, sp3 (b) sp3, dsp2 unpaired electrons.
2
(c) dsp , sp 3 (d) dsp2, sp2 (a) STATEMENT-1 is True, STATEMENT-2 is True;
STATEMENT-2 is a correct explanation for
SECTION - II STATEMENT-1.
Assertion - Reason Type (b) STATEMENT-1 is True, STATEMENT-2 is True;
This section contains 4 questions. Each question has 4 choices STATEMENT-2 is NOT a correct explanation for
(a), (b), (c) and (d) out of which ONLY ONE is correct. STATEMENT-1
(c) STATEMENT-1 is True, STATEMENT-2 is False
32. STATEMENT - 1 : Aniline on reaction with NaNO2 / HCl at (d) STATEMENT-1 is False, STATEMENT-2 is True
0oC followed by coupling with -naphthol gives a dark blue SECTION - III
precipitate.
Linked Comprehension Type
and
STATEMENT - 2 : The colour of the compound formed in This section contains 2 paragraphs. Based upon each paragraph,
the reaction of aniline with NaNO2/HCl at 0oC followed by 3 multiple choice question have to be answered. Each question
coupling with -naphthol is due to the extended has 4 choices (a), (b), (c) and (d), out of which ONLY ONE is
conjugation. correct.
(a) STATEMENT-1 is True, STATEMENT-2 is True; Paragraph for Question Nos. 36 to 38
STATEMENT-2 is a correct explanation for In hexagonal systems of crystals, a frequently encountered
STATEMENT-1. arrangement of atoms is described as a hexagonal prism. Here, the
(b) STATEMENT-1 is True, STATEMENT-2 is True; top and bottom of the cell are regular hexagons and three atoms
STATEMENT-2 is NOT a correct explanation for are sandwiched in between them. A space-filling model of this
STATEMENT-1 structure, called hexagonal close-packed (HCP), is constituted of
(c) STATEMENT-1 is True, STATEMENT-2 is False a sphere on a flat surface surrounded in the same plane by six
(d) STATEMENT-1 is False, STATEMENT-2 is True identical spheres as closely as possible. There spheres are then
33. STATEMENT-1 : The geometrical isomers of the complex placed over the first layer so that they touch each other and
[M(NH3)4Cl2] are optically inactive. represent the second layer. Each one of these three spheres
and touches three spheres of the bottom layer. Finally, the second
STATEMENT-2 : Both geometrical isomers of the complex layer is covered with third layer that is identical to the bottom
[M(NH3)4Cl2] possess axis of symmetry. layer in relative position. Assume radius of every sphere to be 'r'.
(a) STATEMENT-1 is True, STATEMENT-2 is True; Answer the following questions -
STATEMENT-2 is a correct explanation for 36. The number of atoms in the HCP unit cell is
STATEMENT-1. (a) 4 (b) 6
(b) STATEMENT-1 is True, STATEMENT-2 is True; (c) 12 (d) 17
STATEMENT-2 is NOT a correct explanation for 37. The volume of this HCP unit cell is –
STATEMENT-1
(c) STATEMENT-1 is True, STATEMENT-2 is False (a) 24 2r 3 (b) 16 2r 3
(d) STATEMENT-1 is False, STATEMENT-2 is True 64
34. STATEMENT - 1 : There is a natural asymmetry between (c) 12 2r 3 (d) r3
3 3
converting work to heat and converting heat to work.and
38. The empty space in this HCP unit cell is
STATEMENT - 2 : No process is possible in which the sole
result is the absorption of heat form a reservoir and its (a) 74% (b) 47.6%
complete conversion into work. (c) 32% (d) 26%
2008 IIT-JEE Solved Paper 17
2008-

Paragraph for Question Nos. 39 to 41 41. The structure of compounds J, K and L respecitvely, are –
A tertiary alcohol H upon acid catalysed dehydration gives a (a) PhCOCH3, PhCH2COCH3 and PhCH2COO–K+
product I. Ozonolysis of I leads to compounds J and K. Compound (b) PhCHO, PhCH2CHO and PhCOO–K+
J upon reaction with KOH gives benzyl alcohol and compound L,
(c) PhCOCH3, PhCH2CHO and CH3COO–K+
whereas K on reaction with KOH gives only M.
(d) PhCHO, PhCOCH3 and PhCOO–K+
O

H3C SECTION- IV
Ph
M= Matrix-Match Type
This section contains 3 questions. Each question contains state-
H
Ph ments given in two columns which have to be matched. State-
ments in Column I are labelled as A, B, C and D whereas state-
39. Compound H is formed by the reaction of ments in Column II are labelled as p, q, r and s. The answers to
these questions have to be appropriately bubbled as illustrated in
O
the following example.
(a) + PhMgBr If the correct matches are A-p, A-s, B-q, B-r, C-p, C-q and D-s, then
Ph CH3 the correctly bubbled matrix will be look like the following:

(b) + PhCH2MgBr
Ph CH3

(c) + PhCH2MgBr
Ph H
42. Match the entries in Column I with the correctly related quan-
tum number(s) in Column II. Indicate your answer by darken-
O O ing the appropriate bubbles of the 4 × 4 matrix given in the
ORS.
(d) +
Ph H Ph MgBr Column I Column II
(A) H 2 N N H3 Cl (p) sodium fusion
extract of the
40. The structure of compound I is compound gives
Ph Prussian blue
CH 3
+ – colour with FeSO4
(a) NH3I
H Ph (B) HO (q) gives positive
COOH FeCl3 test
H 3C Ph
(b) + –
H Ph (C) HO NH3Cl (r) gives white
precipitate with
Ph CH3 AgNO3
(c)
H C H2Ph + –
(D) O2N NH – NH3Br (s) reacts with
H 3C CH3 aldehydes to form
NO2
(d) the corresponding
Ph H hydrazone
derivative
EBD_780
2008- 18 Target IIT-JEE
43. Match the entries in Column I with the correctly related quantum number(s) in Column II. Indicate your answer by darkening the
appropriate bubbles of the 4 × 4 matrix given in the ORS
Column I Column II
(A) Orbital angular momentum of the electron in a (p) Principal quantum number
hydrogen-like atomic orbital
(B) A hydrogen-like one-electron wave function (q) Azimuthal quantum number
obeying Pauli principle
(C) Shape, size and orientation of hydrogen- like (r) Magnetic quantum number
atomic orbitals
(D) Probability density of electron at the nucleus (s) Electron spin quantum number
in hydrogen-like atom
44. Match the conversions in Column I with the type(s) of reaction(s) given in Column II.

Column I Column II

(A) PbS PbO (p) roasting

(B) CaCO3 CaO (q) calcination

(C) ZnS Zn (r) carbon reduction

(D) Cu2S Cu (s) self reduction

MATHEMATICS
47. Consider a branch of the hyperbola
SECTION - I
x2 2 y2 2 2x 4 2 y 6 0
Straight Objective Type
This section contains 9 multiple choice questions. Each question with vertex at the point A. Let B be one of the end points of
has choices (a), (b), (c) and (d), out of which ONLY ONE is cor- its latus rectum. If C is the focus of the hyperbola nearest to
rect. the point A, then the area of the triangle ABC is
2 3
(a) 1 (b) 1
45. Let the function g : ( , ) , be given by 3 2
2 2
2 3
g (u ) 2 tan 1 (eu ) . Then, g is (c) 1 (d) 1
2 3 2
(a) even and is strictly increasing in (0, )
ex e x
(b) odd and is strictly decreasing in ( , ) 48. Let I dx, J dx . Then,
e4 x e2 x 1 e 4x
e 2x
1
(c) odd and is strictly increasing in ( , ) for an arbitrary constant C, the value of J – I equals
(d) neither even nor odd, but is strictly increasing in
( , ) 1 e4 x e2 x 1
(a) log 4 x C
2 e e2 x 1
46. A particle P starts from the point Z0 = 1 + 2i, where i 1.
It moves horizontally away from origin by 5 units and then
vertically away from origin by 3 units to reach a point Z1. 1 e2 x ex 1
(b) log 2 x C
From Z1 the particle moves 2 units in the direction of the
2 e ex 1

vector iˆ ˆj and then it moves through an angle in 1 e2 x ex 1


2 (c) log 2 x C
anticlockwise direction on a circle with centre at origin, to
2 e ex 1
reach a point Z2. The point Z2 is given by
(a) 6 + 7i (b) –7 + 6i 1 e4 x e2 x 1
(d) log 4 x C
(c) 7 + 6i (d) –6 + 7i 2 e e2 x 1
2008 IIT-JEE Solved Paper 2008- 19

49. Let two non-collinear unit vectors â and b̂ form an acute 52. An experiment has 10 equally likely outcomes. Let A and B
be non-empty events of the experiment. If A consists of 4
angle. A point P moves so that at any time t the position
outcomes, the number of outcomes that B must have so
vector OP (where O is the origin) is given by that A and B are independent, is
(a) 2, 4 or 8 (b) 3, 6 or 9
aˆ cos t bˆ sin t. When P is farthest from origin O, let M be
(c) 4 or 8 (d) 5 or 10
the length of OP and û be the unit vector along OP . 53. Consider three points
Then, P ( sin( ), cos ), Q (cos( ),sin ) and
aˆ bˆ R (cos( ),sin( )) , where 0 , , .
(a) uˆ and M (1 aˆ bˆ)1/ 2 4
| aˆ bˆ |
Then,
(a) P lies on the line segment RQ
aˆ bˆ
(b) uˆ and M (1 aˆ bˆ)1/ 2 (b) Q lies on the line segment PR
| aˆ bˆ | (c) R lies on the line segment QP
(d) P, Q, R are non-collinear
aˆ bˆ
(c) uˆ and M (1 2aˆ bˆ)1/ 2
| aˆ bˆ |
SECTION - II
aˆ bˆ Reasoning Type
(d) uˆ and M (1 2aˆ bˆ)1/ 2 This section contains 4 reasoning type questions. Each
| aˆ bˆ |
question has 4 choices (a), (b), (c) and (d), out of which ONLY
50. Let g (x) = log f (x) where f (x) is twice differentible positive ONE is correct.
function on (0, ) such that f (x + 1) = x f (x). Then, for
N = 1, 2, 3, ...........
54. Consider L1 : 2x + 3y + p – 3 = 0
1 1 L2 : 2x + 3y + p + 3 = 0
g '' N g ''
2 2 where p is a real number, and C : x2 + y2 + 6x – 10y + 30 = 0
STATEMENT - 1 : If line L1 is a chord of circle C, then line
1 1 1 L2 is not always a diameter of circle C
(a) 4 1 ..... and
9 25 (2 N 1)2
STATEMENT - 2 : If line L1 is a diameter of circle C, then line
L2 is not a chord of circle C.
1 1 1 (a) STATEMENT - 1 is True, STATEMENT - 2 is True;
(b) 4 1 .....
9 25 (2 N 1)2 STATEMENT - 2 is a correct explanation for
STATEMENT - 1
(b) STATEMENT - 1 is True, STATEMENT - 2 is True;
1 1 1
(c) 4 1 ..... STATEMENT - 2 is NOT a correct explanation for
9 25 (2 N 1)2 STATEMENT - 1
(c) STATEMENT - 1 is True, STATEMENT - 2 is False
1 1 1 (d) STATEMENT - 1 is False, STATEMENT - 2 is True
(d) 4 1 ..... 55. Let a solution y = y(x) of the differential equation
9 25 (2 N 1)2
2
x x 2 1 dy y y 2 1 dx 0 satisfy y (2) .
1 sin x 3
51. The area of the region between the curves y
cos x
1
STATEMENT-1 : y ( x) sec sec x and
1 sin x 6
and y bounded by the lines x = 0 and x
cos x 4 1 2 3 1
STATEMENT-2 : y(x) is given by 1
is y x2
x
2 1 2 1 (a) STATEMENT - 1 is True, STATEMENT - 2 is True;
t 4t
(a) dt (b) dt STATEMENT - 2 is a correct explanation for
0 (1 t 2 ) 1 t 2 0 (1 t 2 ) 1 t 2 STATEMENT - 1
(b) STATEMENT - 1 is True, STATEMENT - 2 is True;
2 1 2 1 STATEMENT - 2 is NOT a correct explanation for
4t t STATEMENT - 1
(c) dt (d) dt
2 2
0 (1 t ) 1 t 0 (1 t ) 1 t 2
2 (c) STATEMENT - 1 is True, STATEMENT - 2 is False
(d) STATEMENT - 1 is False, STATEMENT - 2 is True
EBD_780
2008- 20 Target IIT-JEE
56. Suppose four distinct positive numbers a1, a2, a3, a4 are in 59. Which of the following is true?
G.P. Let b1 = a1, b2 = b1 + a2, b3 = b2 + a3 and b4 = b3 + a4. (a) f (x) is decreasing on (–1, 1) and has a local minimum
STATEMENT - 1 : The numbers b1, b2, b3, b4 are neither in at x = 1
A.P. nor in G.P.
(b) f (x) is increasing on (–1, 1) and has a local minimum
and
at x = 1
STATEMENT - 2 : The numbers b1, b2, b3, b4 are in H.P.
(c) f (x) is increasing on (–1, 1) but has neither a local
(a) STATEMENT - 1 is True, STATEMENT - 2 is True;
STATEMENT - 2 is a correct explanation for maximum nor a local minimum at x = 1
STATEMENT - 1 (d) f (x) is decreasing on (–1, 1) but has neither a local
(b) STATEMENT - 1 is True, STATEMENT - 2 is True; maximum nor a local minimum at x = 1
STATEMENT - 2 is NOT a correct explanation for
ex
STATEMENT - 1 f '(t )
60. Let g ( x ) dt . Which of the following is true?
(c) STATEMENT - 1 is True, STATEMENT - 2 is False 1 t2
0
(d) STATEMENT - 1 is False, STATEMENT - 2 is True
57. Let a, b, c, p, q be real numbers. Suppose , are the roots
1 (a) g'(x) is positive on (– , 0) and negative on (0, )
of the equation x2 + 2px + q = 0 and , are the roots of
(b) g'(x) is negative on (– , 0) and positive on (0, )
the equation ax2 + 2bx + c = 0, where 2
( 1, 0, 1)
(c) g'(x) changes sign on both (– , 0) and (0, )
STATEMENT - 1 : (p2 – q) (b2 – ac) 0
and (d) g'(x) does not change sign on (– , )
STATEMENT - 2 : b pa or c qa
(a) STATEMENT - 1 is True, STATEMENT - 2 is True; Paragraph for Question Nos. 61 to 63
STATEMENT - 2 is a correct explanation for Consider the lines
STATEMENT - 1
(b) STATEMENT - 1 is True, STATEMENT - 2 is True; x 1 y 2 z 1 x 2 y 2 z 3
L1 : L2 :
STATEMENT - 2 is NOT a correct explanation for 3 1 2 1 2 3
STATEMENT - 1
61. The unit vector perpendicular to both L1 and L2 is
(c) STATEMENT - 1 is True, STATEMENT - 2 is False
(d) STATEMENT - 1 is False, STATEMENT - 2 is True iˆ 7 ˆj 7 kˆ iˆ 7 ˆj 5kˆ
(a) (b)
99 5 3

SECTION - III
iˆ 7 ˆj 5kˆ 7iˆ 7 ˆj kˆ
Linked Comprehension Type (c)
5 3
(d)
99
This section contains 2 paragraphs. Based upon each paragraph,
3 multiple choice questions have to be answered. Each question 62. The shortest distance between L1 and L2 is
has 4 choices (a), (b), (c) and (d), out of which ONLY ONE is
correct. 17
(a) 0 (b)
3
Paragraph for Question Nos. 58 to 60
Consider the function f : ( , ) ( , ) defined by 41 17
(c) (d)
5 3 5 3
x2 ax 1
f ( x) ,0 a 2. 63. The distance of the point (1, 1, 1) from the plane passing
x2 ax 1
through the point (–1, –2, –1) and whose normal is
58. Which of the following is true? perpendicular to both the lines L1 and L2 is
(a) (2 a) 2 f ''(1) (2 a) 2 f ''( 1) 0
2 7
(a) (b)
(b) (2 a ) 2 f ''(1) (2 a ) 2 f ''( 1) 0 75 75

(c) f '(1) f '( 1) (2 a ) 2 13 23


(c) (d)
(d) f '(1) f '( 1) (2 a ) 2 75 75
2008 IIT-JEE Solved Paper 2008-21

SECTION - IV
Matrix-Match Type
This section contains 3 questions. Each question contains statements given in two columns, whcih have
to be matched. Statements in Column I are labelled as A, B, C and D whereas statements in Column II are
labelled as p, q, r and s. The answers to these questions have to be appropriately bubbled as illustrted in
the following example. If the correct matches are A-p, A-r, B-p, B-s, C-r, C-s and D-q, then the correctly
bubbled matrix will look like the following :

64. Consider the lines given by


L1 : x + 3y – 5 = 0; L2 : 3x – ky – 1 = 0; L3 : 5x + 2y – 12 = 0
Match the Statements / Expressions in Column I with the Statements / Expressions in Column II and indicate your answer by
darkening the appropriate bubbles in the 4 × 4 matrix given in the ORS.
Column I Column II
(A) L1, L2, L3 are concurrent, if (p) k = –9
6
(B) One of L1, L2, L3 is parallel to at least one of the other two, if (q) k
5
5
(C) L1, L2, L3 from a triangle, if (r) k
6
(D) L1, L2, L3 do not form a triangle, if (s) k = 5
65. Consider all possible permutations of the letters of the word ENDEANOEL. Match the Statements / Expressions in Column I with
the Statements / Expressions in Column II and indicate your answer by darkening the appropriate bubbles in the 4 × 4 matrix
given in the ORS.
Column I Column II
(A) The number of permutations containing the word ENDEA is (p) 5!
(B) The number of permutations in which the letter E occurs in (q) 2 × 5!
the first and the last positions is
(C) The number of permutations in which none of the letters (r) 7 × 5!
D, L, N occurs in the last five positions is
(D) The number of permutations in which the letters A, E, O (s) 21 × 5!
occur only in odd positions is
66. Match the Statements/Expressions in Column I with the Statements / Expressions in Column II and indicate your answer by
darkening the appropriate bubbles in the 4 × 4 matrix given in the ORS.
Column I Column II

x2 2x 4
(A) The minimum value of is (p) 0
x 2
(B) Let A and B be 3 × 3 matrices of real numbers, where A is (q) 1
symmetric, B is skew-symmetric, and
(A + B) (A – B) = (A – B) (A + B). If (AB)t = (–1)k AB, where
(AB)t is the transpose of the matrix AB, then the possible
values of k are
(C) Let a = log3 log3 2. An integer k satisfying (r) 2
a
1 2( k 3 ) 2 , must be less than

1
(D) If sin cos , then the possible values of are (s) 3
2
EBD_780
2008- 22 Target IIT-JEE

EXPLANATORY NOTES – IIT-JEE 2008

PAPER - I
Thus R3 = 2
V2 Since, R2 < R1 < R3
1. (c) We know that P
R P2 > P1 > P3
For constant value of potential difference (V) we have 2. (b) The time period of a simple pendulum is given by
1 2
P T 2 T2 4 2
g 4
R g g T2
Case (i)
g T
100 100 2 100
g T
1 1 Case I
1 1
2 2 1 = 0.1 cm, = 64cm, T = 0.1s, T = 128s
1 1
1
1 1 g
100 0.3125
g
Case II
This is a case of balanced Wheatstone bridge R1 = 1 = 0.1 cm, = 64cm, T = 0.1s, T = 64s
Case (ii)
g
100 0.46875
g
1 1
Case III
2 2
1 1 = 0.1 cm, = 20cm, T = 0.1s, T = 36s
1 1 g
100 1.055
g
Clearly the equivalent resistance (R2) will be less than
1 . g
Clearly, the value of 100 will be least in case (I).
Case (iii) g
3. (b) The continuous spectrum depends on the accelerating
voltage. It has a definite minimum wavelength.
Greater the accelerating voltage for electrons, higher
will be the kinetic energy it attains before striking the
1 1 target, higher will be the frequency of X - rays and
2 2 smaller will be the wavelength. The wavelength of
continuous X - rays is independent of the atomic number
1 of target material.
1 1
4. (a)

60°
60°
1 iv r=
30
2 º

1
Medium 1 Medium 2
2008 IIT-JEE Solved Paper 2008- 23

Spherically symmetrical
60° gravitational system

60° R
ir r=
30º
m
r
Medium 1 Medium 2

For minimum deviation the ray in the prism is parallel to


the base of the prism. This condition does not depend
on the colour (or wave length) of incident radiation. So Therefore in this case
in both the cases, by geometry, r = 30º. So (a) is correct
4 3
option. G 0 r m
3 mv 2
Alternatively :
r2 r
Condition for minimum deviation is r1= r2 = A . v r ...(ii)
2
equation (i) and (ii) are graphically represented by
Here A = 60° r = 30°. graph (c).
1 sin i Alternatively : For r R
Note : 2
sin r Force on the test mass m is F = m × | Eg |
Where Eg is the gravitational field intensity at the point
We know that r v r v (for same pair of
of observation
media)
ir < iv mv 2 GM where M is the total mass of the
m
In this case the angle of incidence of red radiation is r r2
less than that of violet. spherical system.
5. (c) This is a case of a sphere of uniform mass distribution.
1
For r R v
r
The whole mass inside the sphere can be assumed to
be concentrated at the centre of the sphere. Let m be
For r < R Again F ' m Eg'
the mass of test mass. Here the gravitational pull
between the spherically symmetrical gravitational
system and the test mass provides the necessary mv 2 GM
m r
centripetal force required for the circular motion of the r R3
test mass.
v r
Therefore
6. (c) PT2 = constant (given)
4 3 PV
G o R m Also for an ideal gas = constt
3 mv 2 T
r r From the above two equations we get after eliminating
v2 × r = constt P.

1 V
v ...(i) = constt V = k T3 where k = constant
r T3
For r < R dV dT
When the test mass m is inside the spherically 3
V T
symmetric gravitational system at a distance r from its
centre, it will effectively get attracted towards the centre 3
dv VdT ...(i)
by the mass inside the dotted sphere . T
EBD_780
2008- 24 Target IIT-JEE
We know that change in volume due to thermal
expansion is given by dV = V dT ...(ii)
where = coefficient of volume expansion.
From (i) and (ii)

3 4I
V dT = VdT
T
d
3
=
in
T
I ds
7. (b,d) When binding energy per nucleon increases for a
nuclear process, energy is released.
• When two nuclei of mass numbers between 51 to 100
fuse, the mass number of the resulting nuclei will come
out to be between 100 to 200. The graph shows that in
When d =
this process the binding energy per nucleon increases Screen
sin n
and therefore energy is released.
• When nucleus of mass number 200 to 260 breaks; it will sin n
produce nuclei of mass numbers lying between 100 to When n = 0, =0
200 if we assume that the two daughter nuclei are of When n = 1, = 90° (This will be a point on the screen
nearly same mass. This in fact happens practically that which will be at infinity and therefore not practical)
when a heavy nucleus splits into two parts during Other values of n are invalid as –1 sin 1.
nuclear fission, two moderate size nuclei are formed in The screen will have only one maxima.
general. The graph shows that in this process also the
When < d < 2
binding energy per nucleon increases. Therefore energy
is released. n n
2 d
8. (a, d) Use law of conservation of linear momentum. sin sin
The initial linear momentum of the system is piˆ piˆ 0 n
1< 2
Therefore the final linear momentum should also be sin
zero. The possible values of n are 0, + 1, –1.
Option a : There is at least one more maxima (besides the central
maxima, option [b] is correct.
p1' p2'(a1 a2 )iˆ (b1 b2 ) ˆj c1kˆ = Final We know that
momentum. 2
It is given that a1, b1, c1, a2, b2 and c2 have non-zero Imax = I1 I2
values. If a1 = x and a2 = – x. Also if b1 = y and b2 = –
2
y then the î and ĵ components become zero. But the Imin = I1 I2
third term having k̂ component is non-zero. This Initially I1 = 4 I and I2 = I
gives a definite final momentum to the system which Imax = 9 I and Imin = I
violates conservation of linear momentum, so this is When I1 = I2 = I then
an incorrect option. Imax = 4 I and Imin= 0
Option d: i.e., when the intensities become equal, Imin reduces to
zero. Options [c] and [d] are incorrect.
p1' p2' = (a a )iˆ 2b ˆj 0 because b1 0
1 2 1 10. (a, c, d)
Following the same reasoning as above this option is As the particle enters the magnetic field, a force acts on it
also ruled out. due to the magnetic field which moves the particle in a circular
9. (a,b) The condition to obtain maxima in the phenomenon path of radius
observed in young's double slit experiment is
mv
d sin n where n is an integer r
qB
2008 IIT-JEE Solved Paper 2008-25
In the case of rolling there will be no heat losses.
Region I Region III Therefore total mechanical energy remains conserved.
Region II
×××× The potential energy therefore gets converted into
kinetic energy. In both the cases since the initial
×××× potential energy is same, the final kinetic energy will
also be same. Therefore statement -2 is correct.
× r× × × 12. (a) We know that volume flow rate (V) of an incompressible

× ×r × × fluid in steady flow remains constant.


V=a×v
×××× where a = area of cross-section and v = velocity

×××× If v decreases a increases and vice - versa.


When stream of water moves up, its speed (v) decreases
×××× and therefore 'a' increases i.e. the water spreads out as

× v× × a fountain. When stream of water from hose pipe moves


down, its speed increases and therefore area of cross-
×××× section decreases.
Therefore statement-1 is true and statement-2 is the
• For the particle to enter region III, r > (path shown by correct explanation of statement-1.
dotted line) 13. (d) When the temperature of metal increases; its resistance
increases.
mv
Therefore statement - 2 is correct.
qB
(Unknown resistance) (Standard resistance)
q B
v> X S
m
• For maximum path length in region II, r = N
mv
qB
q B 100
v N
m
• The time taken by the particle to move in region II before G
coming back in region I is given by
m For a meter bridge when null point N is obtained we get
t which is independent of v..
qB
X S
11. (d) The acceleration of a body rolling down an inclined
100
plane is given by
When the unknown resistance is put inside an
g sin enclosure, maintained at a high temperature, then X
a
I increases. To maintain the ratio of null point should
1
MR 2 also increase. But if we want to keep the null point at
the initial position (i.e., if we want no change in the
For hollow cylinder
I MR 2 value of ) there to maintain the ratio, S should be
1
MR 2 MR 2 increased.
Therefore statement - 1 is false.
1
MR 2 14. (a) The normal force exerted by the astronaut on orbiting
I 2 1
For solid cylinder space station is zero (until the astronaut exerts some
MR 2 MR 2 2 muscular force). Therefore the apparent weight of
Acceleration of solid cylinder is more than hollow astronaut in an orbiting space station is zero. Astronaut
cylinder and therefore solid cylinder will reach the is called in a state of weightlessness. This is because
bottom of the inclined plane first. astronaut as well as space -ship are freely falling bodies.
Statement -1 is false Statement-1 is true, statement-2 is true and statement-2
• Statement - 2 is the correct explanation of statement-1.
EBD_780
2008- 26 Target IIT-JEE
15. (b) As the inclined plane is frictionless,
The K. E . at B = P.E. at A
1 2 v sin 30°
mv mgh v 2 gh
2
A v
B 30°
v co
30° s 30
h °
30° C
60° B Just after collision
D Also since the collision is elastic, the vertical component
v cos 30°
30° 30° of velocity (v sin 30°) before collision changes in
v sin 30° v 2gh direction, the magnitude remaining the same as shown
30° in the figure. So the rectangular components of velocity
E
C after collision are as shown in the figure. This means
3m 3 3m
that the final velocity of the block should be horizontal
h making an angle 30º with BC. Therefore the vertical
In ADB, tan 60° = component of the final velocity of the block is zero.
3
h=3m 18. (d) The forces acting besides buoyancy force are
• Force of gravity (vertically downwards)
v= 6g 60 m / s • Viscous force (vertically downwards)
This is the velocity of the block just before collision. Force due to pressure of the liquid is the buoyant force.
This velocity makes an angle of 30° with the vertical. 19. (b) It is given that the bubble does not exchange any heat
Also in right angled triangle BEC, EBC= 60°. with the liquid. This means that while the bubble moves
Therefore v makes an angle of 30° with the second up and expand, the process is adiabatic.
inclined plane BC. The component of v along BC is v For adiabatic expansion the pressure -temperature
cos 30°. relationship is
It is given that the collision at B is perfectly inelastic T P1 constt
therefore the impact forces act normal to the plane such
1
that the vertical component of velocity becomes zero. P1
T1 P1 T2 P21 T2 T1
The component of velocity along the incline BC P2
remains unchanged and is equal to v cos 30°
1
3 180 P
= 60 45 m / s T2 T1 1
2 4 P2

BE 1 BE Here T1 = T0, P1 P0 H g,
16. (b) In BCE, tan30° BE 3m
CE 3 3 3
5
P2 P0 (H y) g ,
Applying mechanical energy conservation . 3
Mechanical energy at B = Mechanical energy at C
5
1 5/3
1 2 1 P0 H g 3
M 45 M 10 3 Mvc2 T2 T0
2 2 P0 ( H y ) g

45 + 60 = vc 2 2 3
P0 H g 3 5
vc = 105 m / s T0
P0 ( H y ) g
17. (c) The velocity of the block along BC just before collision
is v cos 30°. The impact forces act perpendicular to the 2
surface so the component of velocity along the incline P0 (H y) g 5
T2 T0
remains unchanged. P0 H g
2008 IIT-JEE Solved Paper 27
2008-

20. (b) Buoyancy force = weight of fluid displaced


= (mass of fluid displaced) g 6.6 10 34 3 108 1 1
13.6 4
16 9
= V g ...(ii)
where V = Volume of fluid displaced
6.6 10 34 3 108 9 16
= Volume of the bubble. 4.68 10 7 m
Now, PV = nRT 7 13.6 4 1.6 10 19
nRT nRT Since only one option is correct, we need not work out
V the case of electron jumping from n = 4 to n = 2 .
P P0 (H y) g
Where P is pressure of the bubble at an arbitrary 13.6 12
23. (a) K. E. for hydrogen atom (for n = 2) = eV
location distant y from the bottom. 4
Substituting the value of tempertaure from equtaion (i)
we get 13.6 22
K. E for He+ (for n = 2) = 13.6 eV
2 22
nR T0 [ P0 ( H y ) g ]5
V 2 1
[ P0 ( H y) g ] Ratio =
[ P0 H g ]5 4
24. (a) The values of rate constants k0, k1 for zero order and
nRT0 first order reaction, respectively, are given by the
...(iii)
3 2 following equation:
[ P0 (H y) g ]5 P0 H g 5
A0
From (ii) and (iii) k0 [where A0 = initial concentration,
2×t1/2
nRT0 g
Buoyancy force = 2
and t1/2 = half-life period]
3
[ P0 (H y) g] 5 P0 H g 5 0.693
and k1
t1/ 2
13.6 Z2
21. (c) For hydrogen like atoms E n eV / atom substituting various given values, we get
n2
For hydrogen atom 1.386 mol litre –1
E1 13.6eV k0 = ... (i)
(Z = 1) 2 20 sec
E2 = – 3.4eV
0.693
E = E2 – E1 = – 3.4 – (– 13.6) = 10.2 eV and k1 = ... (ii)
40sec
i.e., when hydrogen comes to ground state it will release
10.2 eV of energy. Dividing (ii) by (i), we get

For He ion k1 0.693 2 20 –1


E1 = – 13.6 × 4 eV = – 54.4 eV = mol litre
(Z 2) k0 40 1.386
E2 = – 13.6 eV
E3 = – 6.04 eV 0.693
E4 = – 3.4eV = mol–1 litre
1.386
Here He+ ion is in the first excited state i.e., possessing
energy – 13.6 eV. After receiving energy of + 10.2 eV = 0.5 mol–1 litre
from excited hydrogen atom on collision, the energy of = 0.5 mol–1 dm3 [1 litre = 1dm3]
electron will be (–13.6 + 10.2) eV = –3.4 eV. This means Thus the correct answer is (a).
that the electron will jump to n = 4. 25. (d) Let the weak manoacidic base be BOH, then the
22. (c) After collision with hydrogen atom the He+ ion is in its
reaction that occurs during titration is
third excited state (n = 4). After that the electron can
jump into n = 3. BOH + HCl BCl + H2O
hc +
E= h = E4 E3 Equilibrium : B + H 2O BOH + H+
C(1-h) C.h C.h

13.6 4 13.6 4 19 Using the normality equation, N1V1 = N 2 V2


= 1.6 10
16 9 (acid) (base)
EBD_780
2008- 28 Target IIT-JEE
Substituting various given values, we get 28. (b) Alkyl groups with at least one hydrogen atom on the
-carbon atom, attached to an unsaturated carbon
2 2
´ V1 = 2.5´ atom, are able to release electrons in the following way.
15 5
2 15
or V1 = 2.5´ ´ = 2.5 × 3 = 7.5 ml
5 2
Then the concentration of BCl in resulting solution is
given by Note that the delocalisation involves and bond
orbitals (or p orbitals in case of free radicals) ; thus it is
2 also known as – conjugation. This type of electron
´ 2.5 2
[BCl] = 15 = or 0.1 M release due to the presence of the system H–C–C = C
10 10 is known as hyperconjugation
29. (a) The product (a) will be formed.
Kw
Since K h = Nucleophilic substitution of an alkyl halide is easier as
Kb compared to that of an aryl halide.
PhS– is a strong nucleophile and dimethyl formamide
1´10-14
\ Kh = = 10–2 O
1´10-12 ||
is a highly polar aprotic solvent. These
H C NMe 2
2 2
0.1h 0.1h
Thus K h = or 10-2 = reagent favour SN2 reactions at 2° benzylic carbon.
(1- h) (1- h) [Note : In a SN2 reaction, the major product formed is
or 10–2 – 10–2 h = 0.1 h2 inversion product]
or 0.1 h2 + 10–2 h – 10–2 = 0
Me Br Me SPh
(Solving this quadratic equation for h, we get)
F F
-10-2 ± (10-2 )2 + 4´10-1 ´10-2
h= PhSNa +
2´ 0.1 ¾¾¾¾®
DMF

é ù -2 -4 -3
ê -b ± b 2 - 4ac ú
êx = ú = -10 ± 10 + 4´10 NO2 NO2
ê 2a ú 2´ 0.1
ë û 30. (a,b) When ammonium salt NH4NO3 or NH4NO2 (ammonium
salts are colourless) is boiled with excess of NaOH,
-0.01 ± .0001 + 0.004
= ammonia (NH3) gas is evolved as follows:
0.2 NH4NO2 + NaOH ¾¾ ® NaNO2 + NH3 + H2O
NH4NO3 + NaOH ¾¾ ® NaNO3 + NH3 + H2O
0.01 0.0041 The NH3 gas evolved is non-flammable gas.
0.2 When the gas evolution ceases we are left with NaNO2
or NaNO3 in solution.
-0.01 ± 0.64
= These salts get reduced when Zn is added to this
0.2 solution containing salt (NaNO2 or NaNO3) and excess
0.54 NaOH and NH3 gas is evolve.
= Zn / NaOH
0.2 NaNO2 + 6(H) ¾¾¾¾¾
® NaOH + NH 3 + H 2 O
[Neglecting the negative term]
Zn / NaOH
= 0.27 NaNO3 + 8(H) ¾¾¾ ¾¾ ® NaOH + NH 3 + 2H 2 O
[H+] = C. h = 0.1 × 0.27 = 2.7 × 10–2 M Thus the colourless salt [H] is either NH4NO2 or
Thus the correct answer is [d]. NH4NO3.
26. (b) In the presence of oxygen, Ag metal forms a water Thus (a) and (b) are correct answers.
solublecomplex Na [Ag (CN)2] with dilute solution of [Note : NaCl formed has no reaction with NaOH]
NaCN 31. (a,c) Vander Waals equation is
4Ag + 8NaCN + 2H 2 O + O2 æ 2 ö
çç + n a ÷÷ -
çç P ÷ (V nb) = nRT [For n moles of a gas)
¾¾
® 4Na[Ag(CN)2 ]+ 4NaOH è V 2 ø÷÷
(Soluble)
a, b are vander Waals constants
27. (b) The following reaction occurs The ideal gas equation is PV = nRT [For n moles of a
Na2S2O3 + 4Cl2 + 5H2O ¾¾ ® 2 NaHSO4 + 8HCl. gas]
2008 IIT-JEE Solved Paper 2008-29
where P is pressure excerted by ideal gas and V is
volume occupied by ideal gas. 35. (a) Y

æ n 2 a ö÷
ç
In vander Waals equation the term çç P + 2 ÷÷÷
èç V ø÷
No. of protons
represents the pressure exerted by the gas and (V– nb) (atomic number)
the volume occupied by the gas. At low pressure, when Stable nuclei
the gas occoupies large volume the intermolecular
distance between gaseous moleculas is quite large and
X
in such case there is no significant role played by No. of neutrons
intermolecular forces and thus the gas behaves like an
ideal gas thus (a) is correct A look at the above curve shows that for stable nuclei
Under high pressure the intermolecular distance it shows a curvature towards x-axis from the line of 45°
slope (dotted line) as the atomic number (i.e. number of
decreases and the intermolecular forces play a
protons) increases. So statement 1 is true.
significant role and the gas shows a devation from ideal
The proton - proton repulsion would overcome the
behaviour.
attractive force of proton and neutron. Thus statement 2
Thus (b) is not correct. in True. Also this statement 2 is a correct explanation for
a, b i.e. the vander Waals coefficients defined on the statement 1. Therefore the correct answer is option (a).
nature of gas and are independent of temperature so 36. (c) In group 14 elements, the lower (and not higher)
(c) is correct. oxidation states are more stable for heavier members of
the group due to inert pair effect.
æ n2 a ö÷÷ Thus Pb4+ is less stable as compared to Sn 4+ (lead is
ç
The pressure çç P + ÷ is not lower than P so (d) is
èç V ø÷÷ heavier than Tin). Therefore Pb4+ acts as a strong
oxidising agent than Sn4+. Hence statement 1 is false
not correct. and statement 2 is true. Thus the correct answer is
Hence the correct anser is (a, c). option (c).
32. (b,c,d) E and F ; and also E and G differ in position of atom 37. (c) When halogen is present directly on the benzene
nucleus it produces two opposing effects namely + M
(H), so these are tautomers (not resonating structures.
(activating effect) and –I (deactivating)
Geometrical isomers are also diastereomers).
33. (a, d) The given molecule although posseses neither centre •• Br ••
of symmetry nor a plane of symmetry (hence optically
active) but it has an axis of symmetry (Cn).
Note : A Cn axis of symmetry is an axis about which the
molecule can be rotated by 360°/n to produce a molecule
(–I effect, + M effect)
indistinguishable from the original molecule.
However, + M > –I hence halogens are said to be o, p-
H H CH3 directing (due to + M effect) but deactivating group
(due to –I effect) groups giving mainly p-substituted
Cl 180° CH3 Cl Cl Cl H
H3C H3C H
product.
H H Cl 38. (c) We know that phosphates have a biological
Cl CH3
CH3
significance in human, therefore statement (a) is not
Molecule has
correct.
non-superimposable mirror image Since nitrates are more soluble in water so they are less
abundant in earth's crust where as phosphates are less
34. (d) We know that for every chemical reaction at equilibrium, soluble in water and so they are more abundant in
Gibb's free energy ( DG = 0) is zero. However standard earth's crust. Thus statement (b) is False and statement
Gibb's free energy ( DG°) may or may not be zero. Thus (c) is correct.
statement 1 is False. In nitrates (NO –3 ) nitrogen is in + 5 oxidation state
For a spontaneous reaction, at constant temperature
which is the highest oxidation state exhibited by
and pressure, the reaction proceeds in the direction in nitrogen. Because of this nitrates can not be oxidized
which DG is < 0 i.e. in the direction of decreasing Gibb's (oxidation means increase in oxidation state). Hence
energy (G) so the statement 2 is True. statement (d) is not correct.
Thus the only such option is (d) which is correct answer. The correct answer is (c).
EBD_780
2008- 30 Target IIT-JEE
39. (c) In case of group 15 (nitrogen group), on moving down Substituting the given values, we get
the group there occurs a decrease in bond angle of
0.1´1000 520
metal hydrides. This decrease in bond angle of metal DTb = 0.52 ´ = = 3.20 K
0.9 ´18 9´18
hydrides of this group may be attributed to the
increased p- character in the bond pair which results in \ Boiling point of solution = (373 + 3.20) K
more s- character in lone pair orbital. = 376.2 K
The directional character is more for sp3 hybrid orbital i.e. option (b) is correct answer.
than a s- orbital. 44. (d) CH3 - CH 2 - C º C - CH 2 - CHO
Thus the correct answer is (c). Hex-3-ynal
40. (b) The reaction between NaOH and white phosphorus
1.NaBH
(P4) can be represented as follows: ¾¾ ¾¾
4 ® CH - CH - C º C - CH - CH Br
3 2 2 2
2.PBr3
P4 + 3NaOH + 3H 2 O
(white Phosphorus) or Me CH2Br

Product 'I'
¾¾
® 3NaH 2 PO2 + PH3 Sodium borohydride reduces –CHO
(Phosphine) Selectively into –CH2OH
In this reaction Phosphorus is oxidised as well as 45. (a) Me CH2Br
reduced so it is a disproportionation reaction.
The correct answer is (b). “I”
41. (d) The solution M is a mixture of ethanol and water. In it
the mole fraction of ethanol is 0.9 and mole fraction of 1.Mg / Ether
2.CO
water is 0.1 (1.0 – 0.9 = 0.1) Also given are:- ¾¾¾¾¾
2
+
® Me CH2COOH
3.H3O
Standard freezing point of ethanol = 155.7K.
Freezing point depression constant “J”
(Kf) for ethanol = 2.0 kg mol–1 "K"
Molecular weight of ethanol (C2H5 OH) = 46 Me CH 2COCl

0.1´1000 Thus "K" is SOCl2


now molality (m) of solution =
0.9 ´ 46 46. (c) Me CH2COCl
Using the formula :-
“J”
Depression in freezing point ( DTf ) = Kf × m, H
¾¾¾¾¾
2 ® Me CH2CHO
Substituting various values, we get Pd / BaSO 4
(quinoline)
2´ 0.1´1000 2000 H “L” H
DTf = = = 4.83 K.
0.9 ´ 46 414 It is Rosenmund reaction. Simultaneously the reagent
Freezing point of solution 'M' = (155.7 – 4.83) K H2-Pd also reduces carbon-carbon triple bond to double
= 150.9 K bond (syn -addition) giving cis product.
i.e. (d) is the correct answer. 1
47. (c) 2 1 1 2 2
42. (b) o
Given : vapour pressure of pure ethanol (PA )= 40 mmHg 1 x {x cos(cot x) sin(cot x)} 1
Mole fraction (XA) of ethanol in solution = 0.9
Using the formola- 2 1 x
= 1 x x cos cos
Total pressure (P) = P Ao X oA 1 x2
Substituting the given values, we get
P = 40 × 0.9 = 36.0 mm Hg 1
2 2
i.e. (b) is the correct answer. 1
1
43. (b) Given: Standard boiling poing of water = 373K sin sin 1
Boiling point elevation constant of water (Kb) = 0.52 1 x2
Kg mol–1
Molecular weight of water (H2O) = 18 2
Mole fraction of water in solution = 0.1 (1.0 – 0.9 = 0.1) 2 x 1
= 1 x x. 1
0.1´1000 1 x2 1 x2
molality (m) =
0.9 ´18 2
2
using the relation DTb = K b ´ m = 1 x 1 x2 1 = x 1 x2
2008 IIT-JEE Solved Paper 31
2008-

48. (b) The given curves, are 52. (c) As per question,
C1 : y2 = 4x ...(1) and C2 : x2 + y2 – 6 x + 1 = 0...(2) p = left hand derivative of |x –1| at x = 1 p = –1
Solving (1) and (2) we get Also lim g ( x ) p
x2 + 4x – 6x + 1 = 0 x = 1 and y = 2 or –2 x 1
Points of intersection of the two curves are (1, 2) ( x 1) n
and (1, –2). Where g (x) = , 0 < x < 2,
log cos m ( x 1)
dy 2
For C1, m, n are integers, m 0, n > 0
dx y we get,

dy dy ( x 1) n hn
= 1= m1 and = –1 = m1 ' lim m
1 lim 1
dx dx (1, 2) x 1 log cos ( x 1) h 0 log cos m h
(1, 2)

dy 3 x dy hn
1 m2 lim 1 [Using L' Hospital's rule]
For C2, h 0 m(log cos h)
dx y dx (1, 2)
n h n 1 cos h
dy lim 1 [Using L' Hospital's rule]
1 m2 ' h 0 m( sin h)
and dx
(1, 2)
n h n 2 cos h
m1 = m2 and m1 ' m2 ' lim 1 n = 2 and m = 2
h 0 sin h
C1 and C2 touch each other at two points. m
h
49. (a) We know that the volume of a parallelopipe with
y
coterminus edges as the vectors a , b , c is given by
S’(–ae, 0) S(ae, 0)
a.a a.b a .c
2
V= a b c = b.a b .b b .c 53. (b,c) x
c .a c .b c .c
b2 Q b2
1 1/ 2 1/ 2 ae, P ae,
1 1 a a
V2 1/ 2 1 1/ 2 = V=
2 2
1/ 2 1/ 2 1 y
50. (b) x2 – 5xy + 6y2 = 0 represents a pair of straight lines
given by x – 3y = 0 and x – 2y = 0.
Also ax2 + by2 + c = 0 will represent a circle if a = b and
c is of sign opposite to that of a. A’ x
1 ½ 1
51. (c) y 3, Q P 3,
2 R 2
A

O x
Given ellipse is x2 + 4y2 = 4
x2 y2
or 1 a = 2, b = 1
22 1
The given function is 1 3
e= 1 ae = 3
3 4 2
(2 x ) , 3 x 1
f ( x)
1
x2 3 , 1 x 2 As per question P ( ae, b 2 / a ) 3,
2
The graph of y = f (x) is as shown in the figure. From
graph, clearly, there is one local maximum (at x = –1) 1
Q ( ae, b 2 / a) 3,
and one local minima (at x = 0) 2
total number of local maxima or minima = 2. PQ = 2 3
EBD_780
2008- 32 Target IIT-JEE
Now if PQ is the length of latus rectum to be found,
QS SR
Also QS SR ( GM < AM)
3 2
then PQ = 4a = 2 3 a=
2 1 2 2 4
Also as PQ is horizontal, parabola with PQ as latus QS SR QR QS SR QR ...(3)
rectum can be upward parabola (with vertex at A) or
down ward parabola (with vertex at A'). 1 1 4
For upward parabola, From equations (2) and (3) we get
PS ST QR
3 3 1 (d) is also the correct option.
AR = a = Coordinates of A = 0,
2 2
55. (a, b, c, d)
So equation of upward parabola is given by f (x) is a non constant twice differentiable function such
that f (x) = f (1– x)
x2 3 1 or 2
2 3 y x 2 3y 3 3 ...(1) f '(x) = – f ' (1 – x) ...(1)
2
1 1 1
For x , we get f ' f' 1
3 2 2 2
For downward parabola A'R = a =
2 1 1 1
f' f' 0 f' 0 (b)is correct
2 2 2
1 3
Coordinates of A' = 0,
2 1 3
For x = , we get f ' 1 = f'
So equation of downward parabola is given by 4 4 4

x2 2 3 y 1 3 or x 2 2 3y 3 3 ...(2) but given that f ' 1 = 0 f'


1
f'
3
0
2 4 4 4
Hence, f '(x) satisfies all conditions of Rolle's theorem for
the equation of required parabola is given by equation
(1) or (2). 1 1 1 3
x , and , . So there exists at least one point
4 2 2 4
P
1 1 1 3
54. (b, d) C1 , and at least one point C2 , . Such that
4 2 2 4
f "(C1) = 0 and f "(C2 ) 0
f "(x) vanishes at least twice on [0, 1] (a) is correct.
Also using f (x) = f (1 – x)
1 1 1
f x f 1 x f x
Q R 2 2 2
S
1
f x is an even function.
T 2

We know by geometry PS × ST = QS × SR ...(1) sin x. f x 1 is an odd function.


S is not the centre of circulm circle, 2
PS ST 1/ 2
1
And we know that for two unequal real numbers. f x sin x dx 0, (c) is correct.
2
H.M. < G.M. 1/ 2

2 1 1 2 1 12
PS ST PS ST More over f (1 t )esin t
f (u )esin u
du ,
1 1 PS ST
PS ST 12 0

where 1 – t = u
1 1 2
PS ST [using eqn (1)] ...(2) n
n
QS SR
56. (a,d) We have S n 2
(b) is the correct option. k 1n kn k 2
2008 IIT-JEE Solved Paper 2008- 33

n 1
59. (a) We have f (x) = g(x) sin x
n f '(x) = g'(x) sin x + g(x) cos x
and Tn 2
n = 1, 2, 3.........
k 0n kn k 2 f '(0) = g'(0) × 0 + g(0) = g(0) [ g'(0) = 0]
For n = 1 we get Statement 2 is correct.

1 1 1 f '( x ) f '(0)
S1 0.3 and T1 1 Also f "(0) = lim
1 1 1 3 1 0 x 0 x
g ( x) cos x g '( x )sin x g (0)
3 3.14 1.73 = lim
Also = 0.34 × 1.73 = 0.58 x 0 x
3 3 9 9
g ( x ) cos x g (0) g '( x )sin x
S1 T1 , and Tn = lim lim
Sn x 0 x x 0 x
3 3 3 3 3 3
57. (a) The given equations are g ( x) cos x g (0)
x – 2y + 3z = – 1 = lim sin x
lim g '( x)
x 0 x 0
x
– x + y – 2z = k x
x – 3y + 4z =1
g ( x) cos x g (0)
1 2 3 = lim g '(0)
x 0 sin x
Here D= 1 1 2 0
1 3 4 = xlim0 g ( x) cot( x) g (0)cosec x 0

1 1 3 = xlim0 g ( x) cot x g (0)cosec x


and D2 1 k 2 k 3 0 if k 3 Statement 1 is also true and is a correct explanation for
1 1 4 statement 2.
60. (d) The given planes are
If k 3 , the system has no solutions. P1 : x – y + z = 1 ...(1)
Hence statement-1 is true and statement-2 is a correct P2 : x + y – z = –1 ...(2)
explanation for statement - 1. P3 : x – 3y + 3z = 2 ...(3)
58. (b) The given system of equastions is Line L1 is intersection of planes P2 and P3.
ax + by = 0 L1 is parallel to the vector
cx + dy = 0 where a, b, c, d {0, 1}
For the system to have unique solution, iˆ ˆj kˆ
= 1 1 1 4 ˆj 4kˆ
a b a b
i.e. 0 1 3 3
c d c d
This is so in each of the following cases - Line L2 is intersection of P3 and P1
L2 is parallel to the vector
1 1 1 1 0 1 1 0 1 0 0 1
, , , , , ˆj kˆ
1 0 0 1 1 1 1 1 0 1 1 0 iˆ
= 1 1 1 2 ˆj 2 kˆ
Favourable cases for the system to have unique
solution = 6. 1 3 3
a b Line L3 is intersection of P1 and P2
Also total possible cases for =24 = 16
c d L3 is parallel to the vector
( each entry can be either 0 or 1)
iˆ ˆj kˆ
Probability of the given system to have unique solution
2 ˆj 2kˆ
= 1 1 1
6 3
= = 1 1 1
16 8
Statment –1 is true. Clearly lines L1, L2 and L3 are parallel to each other.
Homogeneous system of equations always has a Statement–1 is False
solution (Trival solution x = 0, y = 0) Also family of planes passing through the intersection
The probability that the system of equations has a of P1 and P2 is P1 P2 0 .If plane P3 is represented
solution is 1.
Hence the statement-2 is true but is not a correct explanation by P1 P2 0 for some value of then the three
of statement- 1. planes pass through the same point.
EBD_780
2008- 34 Target IIT-JEE
Here P1 P2 0 But we know that | z | | w | |z w|
x(1 ) y ( 1) z (1 ) ( 1) 0 |z| |w| 6 6 |z| |w| 6
This will be identical to P3 if – 3 <| z | –| w|+3 <9
1 1 1 1 1
...(1) 64. (d) Slope of CD =
1 3 3 2 3
1 1 1 Parametric equation of CD is
Taking , we get and taking
1 2 3
3 3 3
1 1 2 x y
, we get . 2 2 1
1 3 3 3 1
There is no value of which satisfies eq (1). 2 2
The three planes do not have a common point.
Statement 2 is true. Two possible coordinates of C are
(d) is the correct option.
61-63. We have A= {z : Im(z) 1}= {(x, y) : y 1} 3 3 3 1 3 3 3 3 1 3
Clearly A is the set of all points lying on or above the line y , or ,
2 2 2 2 2 2 2 2
= 1 in cartesian plane.
B = {z : | z – 2 – i | = 3} = {(x, y) : (x – 2)2 + (y –1)2 = 9} i.e. 2 3, 2 or 3,1
B is the set of all points lying on the boundary of the
As (0, 0) and C lie on the same side of PQ
circle with centre (2, 1) and radius 3.
C = {z : Re[(1 – i) z ] = 2 } = {(x, y) : x + y = 2 } 3,1 should be the coordinates of C.
C is the set of all points lying on the straight line Remember (x1, y1) and (x2, y2) lie on the same or
represented by x + y = 2 . opposite side of a line ax + by + c = 0 according as
Graphically, the three sets are represented as shown below : ax1 by1 c
0 or < 0.
y ax2 by2 c

Equation of the circle is ( x 3)2 ( y 1) 2 1


(z)P 65. (a) PQR is an equilateral triangle, the incentre C must
S(2+i) R(s+i) coincide with centriod of PQR and D, E, F must
concide with the mid points of sides PQ, QR and RP
(–1, i)Q respectively.
x x
O
Also CPD 30º PD 3
x+

Writing the equation of side PQ in symmetric form we


y=

get,
2

y 3 3 3
61. (b) From graph A B C consists of only one point P x y
2 2 3
[the common point of the region y 1, (x – 2)2 + (y – 1)2 1 3
= 9 and x + y = 2 ] 2 2
n (A B C) = 1
62. (c) As z is a point of A B C 3 3 3 3 3
Coordinates of P = ,
z represents the point P 2 2 2 2
|z + 1 – i |2 + |z – 5 – i|2
|z – (–1 + i)|2 + |z – (5 + i)|2 2 3, 0 and
PQ2 + PR2 = QR2 = 62 = 36
which lies between 35 and 39 coordinates of Q 3 3 3 3 3
, 3,3
(c) is correct option. 2 2 2 2
63. (d) Given that |w – 2 – i| < 3
Let coordinates of R be ( , ) , then using the formula
Distance between w and 2 + i i.e. S is smaller than 3.
w is a point lying inside the circle with centre S and for centriod of we get
radius 3. 3 0
3 2 3
Distance between z (i.e. the point P) and w should 3 and 1
be smaller than 6 (the diameter of the circle) 3 3
i.e. | z – w| < 6 = 0 and =0
2008 IIT-JEE Solved Paper 2008- 35
Coordinates of R = (0, 0)
b b
Hence required area = f ( x )dx 1. f ( x)dx
3 3
Now coordinates of E = mid point of QR = , a a
2 2
b
and coordinates of F = mid point of PR = 3, 0 = x f ( x)
b
x f '( x)dx
a
a
66. (d) Equation of side QR is y = 3x and equation of side
RP is y = 0 b
x .1
Solution (67-69) = b f (b ) a f (a ) dx
a
3 1 ( f ( x)) 2
Given the implicit function y3 – 3y + x = 0

For x (– , –2) (2, ) it is y = f (x) real valued b


x
differentiable function. = 3 b f (b) a f (a)
2
a f ( x) 1
and for x (–2, 2) it is y = g(x) real valued differentiable
function. 69. (d) For y = g(x), we have y3 – 3y + x = 0

dy dy 3
67. (b) We have y3 – 3y + x = 0 3y2 3 1 0 g ( x) 3 g ( x) x 0 ...(1)
dx dx
Putting x = –x, we get
dy 1 1
or f '( x ) 3
dx 3(1 y 2 ) 3 1 [ f ( x)]2 g ( x) 3 g ( x) x 0 ...(2)

Adding equations (1) and (2) we get


2 2
2d y dy d2y
Also 3 y 6y 3 0 3 3 n!
dx 2 dx dx 2 g ( x) g ( x) 3 g ( x) g ( x) 0
r! n r !

2
d2y 2y dy g ( x) g ( x)
dx 2 1 y2 dx
2 2 0
g ( x) g ( x) g ( x) g ( x) 3
2 f ( x)
f "( x ) 3 For g(0) = 0, we should have
9 1 [ f ( x)]2
g(x) + g(–x) = 0

[ From other factor we get g(0) = 3]


2 2 2 4 2
f "( 10 2) g(x) is an odd function
91 83 32 73

68. (a) For x < – 2 1


1
g '( x )dx g ( x) 1
g (1) g ( 1)
we have, 3y – y3 < – 2 y3 – 3y – 2 > 0
–1
(y + 1)2 (y – 2) > 0 y>2 x<–2
g (1) g (1) = 2 g(1).
f (x) is positive x < –2
EBD_780
2008- 36 Target IIT-JEE

PAPER - II
1. (a) Sample S –1 Sample S – 2 Magnitude of force between B and C is
Activity 5 Ci 10 Ci
No. of nuclei N1= 2N N2 = N 1 2q / 3 q / 3 q2
F= =
dN dN o 3R
2 54 o R2
– 1 N1 – 2 N2
dt 1 dt 2 3. (a) Since the wave is sinusoidal moving in positive x-axis
– 5 = 1× 2N ...(i) – 10 = 2×N ...(ii) the point will move parallel to y-axis therefore options
From (i) and (ii) (c) and (d) are ruled out. As the wave moves forward in
positive X-direction, the point should move upwards
5 1 2N 1 1
= = i.e. in the positive Y-direction. Therefore correct option
10 2 N 2 4 is a.
Alternatively-1 :
T1/ 2 2 1 1
= dy dy
T1/ 2 4 T1/2 Velocity of point P = = – (velocity of wave ) ×
1 dt dx
2. (c) The electric field due to A and B at O are equal and
opposite producing a resultant which is zero. The dy
Here is negative . Therefore velocity at point P is
dx
1 2q / 3
electric field at O due to C is E positive and is along y-axis only.
o R2 Alternatively-2 :
Equation of a wave moving in positive x-axis is given
q
= . as y = A sin ( t– ) or vP =A cos ( t– )
6 o R2 Here y = 5 cm, A = 10 cm,
Option [A] is not correct. The electric potential at O is 5 = 10 sin ( t – ) t– = 30º
Substituting this value in the equation of velocity we
get vP = 0.10 × cos 30º
q
0.10
B 3 ow v = v= 0.2
30 º 0.5
2q C R
O = 2 v = 2 × 0.2 = 0.4
3 60º R
q A 3 3
vP= 0.1× 0.4 × = .
3 2 50
It has to be in positive y direction.
4. (a) Let the level of liquid at an instant of time 't' be x. Then
q /3 q /3 2q / 3
VO= K +K +K =0
R R R dx
v=– dx = –vdt
Option [D] is wrong dt
AC AB x t d
In ABC = sin 30º AC= =R dx = – v dt x
AB 2
d /3 0
BC 3AB
Also =sin 60º BC = = 3R d d
AB 2 x– =– vt x – vt
Potential energy of the system 3 3
K= Also the capacitance can be considered as an equivalent
of two capacitances in series such that
q / 3 2/ 3 q/3 2q / 3 q/3 2q / 3
K K 1 1 1
2R R 3R = +
Ceq C1 C 2
kq 2 1 2
= 2 0 1 1 1 d x x
9R 2 3
Ceq o A o AK o A o AK
Option [B] is wrong d –x x
2008 IIT-JEE Solved Paper 2008- 37

1 1 d x x 1 Kd Kx x
overshoot to equilibrium position and move towards
= = wall 2. As this happens, the spring S1 comes to its natural
Ceq o A 1 K o A K length and S2 gets compressed. As there are no frictional
forces involved, the P.E. stored in the spring S1 gets
Kd x 1 K
= stored as the P.E. of spring S2 when the block B reaches
o AK its extreme position after compressing S2 by y.
0 AK 1 1
Ceq = k1 x2 = k2 y2
Kd x(1 K) 2 2
d 1 1
But A = 1, K = 2 and x = – vt × kx2 = 4 ky2
3 2 2

1 2 x2 = 4y2
0
Ceq =
d y 1
2d vt 1 2
3 x 2
8. (b) We know that excess pressure in a soap bubble is
2 0 6 0
= = inversely proportional to its radius. The soap bubble
d 5d 3vt at end 1 has small radius as compared to the soap bubble
2d vt
3 at end 2 (given). Therefore excess pressure at 1 is more.
6R 0
Time constant = RCeq =
5d 3vt
5. (a) The frequency (v) produced by the air column is
2 1
given by
v As the value is opened, air flows from end 1 to end 2
×v=v v= and the volume of soap bubble at end 1 decreases.
9. (d) This is the case of vertical motion when the body just
3 completes the circle. Here
Also, = = 75cm = 0.75 m
4 v = ..... (i)
5gL
4 0.75 340 3
= v= = 340 Hz
3 4 0.75
The frequency of vibrating string = 340. Since this
string produces 4 beats/sec with a tuning fork of O
frequency n therefore n = 340 + 4 or n = 340 – 4. With L
increase in tension, the frequency produced by string L–h v
increases. As the beats/sec decreases therefore n = M 2
P
340 + 4 = 344 Hz. h
v
6. (b) For refraction at parallel interfaces Reference A
Level for PE.
n0 n0 n0
n0 sin sin = sin = sin 90º Applying energy conservation,
2 6 8 Total energy at A = Total energy at P
2
1 2 1 v
mv = m + mgh.
2 2 2

v2
1 v2 = 2gh
sin = 4
8
3v2 3v2 3 15L
= 2gh h= = × 5gL = ...(ii)
7. (c) When the block B is displaced towards wall 1, only 4 8g 8g 8
spring S1 is compressed and S2 is in its natural state.
This happens because the other end of S2 is not attached 15L
L
to the wall but is free. Therefore the energy stored in L h 8 7
In OPM, cos = = =
1 L L 8
the system = k1 x2 . When the block is released, it will
2 3
come back to the equilibrium position, gain momentum, Therefore, the value of lies in the range
4
EBD_780
2008- 38 Target IIT-JEE
10. (a) Both the statements are true and statement-2 is the
r
correct explanation of statement-1 = d–
dr
R
NBA
11. (c) Statement-1 is true. Sensitivity = = . If B The charge in the dotted element shown in Fig (2) is
I C
increases, increases. Statement-2 is wrong because Fig (2)
I
soft iron can be easily magnetised and de magnetized. dq = × 4 r2dr
12. (b) Statement-1 is true. For a moving observer, the near by R R
objects appear to move in the opposite direction at a d 2 d 3
dq = d r 4 r2dr Ze = 4 dr dr – r dr
large speed. This is because the angular speed of the R o R
o
near by object w.r.t observer is large. As the object moves
away the angular velocity decreases and therefore its R3 4 d R4
Ze = d
speed seems to be less. The distant object almost 3 R 4
remains stationary.
Statement-2 is the concept of relative velocity which Ze. 1 1 1 3Ze
= – = d=
states that 4 dR3 3 4 12 R3
v 21 = v 2 G – v1G 16. (c) If the volume charge density is constant then E r.
where G is the laboratory frame. 17. (d) When the disc is at a distance x from the mean position
Thus both the statement are true but statement-2 is not (equilibrium position), the forces acting on the disc are
the correct explanation of statement-1. shown in the figure
13. (b) It is easier to pull a heavy object than to push it on a y
level ground. Statement-1 is true. This is because the
normal reaction in the case of pulling is less as compared kx x
by pushing. (f = N). Therefore the functional force is x
small in case of pulling.
statement-2 is true but is not the correct explanation of
statement-1. C R
14. (a) When the point of observation is on the surface of Mean
Position kx f
sphere then the whole charge inside the sphere (when
distributed symmetrically about the centre) behaves as
a point charge on the centre. Therefore until the charge –2kx + f = –Mac ...(1)
distribution is symmetrical about the centre it does not where ac = acceleration of center of mass. Also the torque
matter what is the ratio a/R. The electric field remains acting on the disc about its center of mass C is
constant and is equal to =f×R=I× c
1 Ze 1
E= . . I
MR 2 ac
4 o R
2
f= =
R R R
P 1
R
[ I= MR2 and ac = R c for rolling without slipping]
a 2
1
f= Ma c ...(ii)
2
From (i) & (ii)

15. (b) For a = 0, the graph is as shown. The equation for the 1
–2kx + Mac= – Mac
graph line is 2
3
r Ma = 2 kx
2 c
d 4kx
r dr
Mac =
3
Net external force acting on the disc when its centre of
mass is at displacement x with respect to the equilibrium
r 4kx
O
Fig (1)
R fig -2 position = directed towards the equilibrium.
3
2008 IIT-JEE Solved Paper 2008-39
18. (d) As derived in ans 17.
u 2 sin 2
4k R= R u2 (for consant and g)
|Fnet| = x g
3
For S.H.M. |Fnet| = M 2x
D q T= T2
4k 4k g
M 2 = = ...(iii)
3 3M 2
T
19. (c) From (i) & (ii)
2k
–2kx + f = –2 f f=×x
3
We see that the frictional force depends on x. As x 21. (A)-(q)
increases, f increases. Also, the frictional force is As the ideal gas expands in vacuum, no work is done (W =
maximum at x = A where A is the amplitude of S.H.M. 0). Also the container is insulated therefore no heat is lost
Therefore the maximum frictional force or gained (Q = 0). According to first law of thermodynamics
2k U=Q+ W
fmax= ×A U= 0
3
There is no change in the temparature of the gas
The force should be utmost equal to the limiting friction (B)-(p, r)
( Mg) for rolling without slipping. Given PV2 = constant ....(i)
2k PV
Mg = ×A ....(iv)
3 Also for an ideal gas = constant ....(ii)
T
For S.H.M. From (i) & (ii) V × T = constant
Velocity amplitude = A As the gas expands its volume increases and temperature
Vo = A decreases
3 Mg (p) is the correct option
Vo = from (iv) To find whether heat is released or absorbed let us find
2k
a relationship between Q and change in temperature T.
3 Mg 4k We know that Q = nC T ...(i)
Vo = from (iii) where C = molar specific heat
2k 3M
Also for a polytropic process we have
3M
Vo = g R
k C =Cv + and PVn = constant
1- n
20. A p, s Here PV2 = Constant. Therefore n = 2
When simple pendulum is displaced from the mean position
towards any of the extreme position, its potential energy R
C = Cv + = Cv – R
increases. In case of a S.H.M. we get a parabola for potential 1- 2
energy versus displacement
3
B q, r, s For mono atomic gas Cv = R
S = ut for zero acceleration. Therefore we get a straight line 2
passing through the origin. Therefore option (q) is correct. 3 R
C= R – R =
1 2 2
S = ut + at2 for constant positive acceleration. In this Substituting this value in (1) we get
2
case we get a part of parabola as a graph line between s R
versus t as shown by graph (s). Q=n × × T..
2
(r) is ruled out because the displacement of the body cannot
In this case the temperature decreases i.e. T is negative.
be non-zero at t = 0. The position of the body may be non-
Therefore Q is negative. This in turn means that heat is lost
zero at t= 0 but not the displacement.
by the gas during the process. (r) is the correct option.
(p) is ruled out because if a is –ve and v is positive.
(C)-(p, s)
R Proceeding in the same way we get in this case
V1/3× T = constant
As the gas expands and volume increases, the
C s
temperature decreases. Therefore (p) is the correct
u option
EBD_780
2008- 40 Target IIT-JEE
4 When the object is at 2F , the image is formed at 2F. The
In this process, x = . image is real, inverted and of the same size.
3
R 3 3R 3 3R When the object is moved still closer from 2F to F , the
C = Cv + = R+ = R–3R =
4 2 1 2 2 image is real, inverted and magnified.
1
3 When the object is at F , the image is real, inverted,
3R highly magnified and is formed at infinity.
Q=n t When the object is between optical centre and F , the
2
As T is negative, Q is positive. This in turn means that image is virtual erect and magnified.
heat is gained by the gas during the process (s) is the correct
option.
(D)-(q, s)
D-p, q, r, s
PV
Also T =
nR
Here (PV) is positive T is positive temperature
increases. So, (q) is the correct option
From the graph it is clear that during the process the R1 R2
pressure of the gas increases which shows that the
internal energy of the gas has increased. Also the volume
increases which means work is done by the system
which needs energy. From these two interpretation we 1 1 R 2 R1
can comfortably conclude that the gas gains heat during
1
= ( – 1) R R = ( – 1) R R
the process. f 1 2 1 2
(s) is the correct option. Here R1 < R2
22. A-p, q, r, s
When the object is at infinity, a real, inverted and f is positive.
diminished image is formed at the focus of the concave Therefore it behaves effectively as a convex lens. So
mirror. the case is same as column I (C).
As the object is brought closer to the mirror, the image 23. (b) We know that surface acting agents (i.e. surfactants)
moves farther, remains real and inverted and increases
such as soaps and detergents belong to the class of
in size (but still it is diminished as compared to the object)
micelles. A miceller system when dissolved in water,
When the object is at C, a real, inverted and same size
image is formed at C. dissociates to give ions. The anion consists of two
When the object is brought still closer, a real, inverted parts. The polar groups such as (COO– or SO 4 ) ion
and magnified image is formed beyond C.
When the object is at focus (F), the image is highly is water loving (i.e. hydrophilic) in nature. It is called
magnified, real and inverted and formed at infinity. head of the species. The hydrocarbon chain which is
When the object is placed between pole and focus, a quite big in size is water repelling (i.e. hydrophobic) in
virtual, erect and magnified image is formed behind the nature. It is called tail of the species. The hydrocarbon
mirror. chain aggregates into the micelle above the critical
Alternatively :
concentration.
f v
v= , m= – It may also be noted that the critical concentration for
f u
1- micelle formation decreases with increase in the
u
molecular weight of the hydrocarbon chain of
for different values of u, we get different values of v. surfactant.
(+ ve, –ve or ). Like wise m comes out to be positive,
negative, greater than 1 etc. Here the two anions that are formed are in case of
B-q "B" (i.e. CH3 (CH2 )11 OSO 3 ) and
In case of a convex mirror we always get virtual,
diminished image between P and F. "C" (i.e. CH3 (CH 2 )6 COO)
C-p, q, r, s
When object is at infinity, a real, inverted and dimished The molecular weight of hydrocarbon chain is more in
image is formed at F. When the object is brought closer case of "B" so it has lower value of critical concentration
to the lens, the image moves away from the lens and for micelle formation in aqueous solution.
increases in size. (Image remains between F & 2F). Hence the correct answer is option (b).
2008 IIT-JEE Solved Paper 2008-41

24. (d) ® M + + X-
MX ¾¾ (Where s is the solubility) O
s s I 2 / NaOH
Ph× (–) (+)
s = Ksp ONa + CH3I
Then Ksp = s 2
or
(F) (G)
® M 2+ + 2X-
Similarly for MX 2 ¾¾ Thus the correct answer is (c).
27. (d) The correct stability order is
1
I > III > II > IV
é K sp ù 3
Ksp = s ´(2s)2 = 4s3 or s = êê ú
ú
êë 4 úû + +
O O
® 3M + + X-3
and for M 3 X ¾¾
3s s I III
+ M effect, 6 hyper-
1 conjugative H’s, + M, 3 hyperconjugative
é K sp ù 4 2 CH 3 groups . H’s, one CH 3 group
Ksp = (3s) ´s = 27s or s = êê
3 4 ú
ú (+ I effect)
ëê 27 ûú
From the given values of K sp for MX, MX2 and +
M3 X, we can find the solubilities of those salts at +
temperature, T. II IV
5 hyper conjugative 5 hyperconjugative H’s
Solubility of MX = 4´10-8 = 2´10-4 CO – H’s 2 R groups one R groups

1 1 28. (a) Cellulose is a polysaccharide composed of only D-


é 3.2 ´10-14 ù 3
Solubility of MX 2 = êê ú or éê 32 ´10-15 ùú 3 glucose units. Every adjacent glucose units are joined
ú êë 4 úû by -glycosidic linkage between C1 of one glucose and
êë 4 úû
C4 of the next.
1 Thus in every glucose units only three –OH groups
= éê8´10-15 ùú 3 or 2´10-15 are free to form triacetate.
ë û
1 HO 6
é 2.7 ´10-15 ù 4
Solubility of M3 X = êê ú 5
HO 6 H O O
ú H
êë 27 úû 5
O 4 OH H
1
HO 6 H O
H 1 3 2 H
1
H 5 4 OH H OH
= éê10-16 ùú 4 or 10–4 O O 2 H H
H 1 3
ë û 4
OH H
O H H OH
Thus the solubilities are in the order MX > M3 X> MX2 3 2
i.e the correct anser is (d). H OH
25. (b) Give : I = 10 milliamperes ; IF = 96500 C mol–1
t = ? ; Moles of H2 produces = 0.01 mol
From the law of electrolysis, we have
I´ t(sec) Cellulose triacetate (CH3CO)2O, H2SOI4
Equivalents of H2 produces =
96500
Substituting given values, we get
(–OH –OCOCH3 )
10´10-3 (amperes)´ t(sec)
0.01´ 2 = 29. (c) Colour is due to d – d transitions. Coloured compounds
96500 contain partly filled d-orbital.
The oxidation state of copper in various compounds is
0.01´ 2´96500
or t= sec + 1 and + 2. In CuF2 it is in + 2 oxidation state. In + 2
10´10-3 state its configuration is
= 19.3 × 104 sec.
i.e. (b) is the correct answer. Cu 2+ = 1s 2 2s 2 2p 6 3s2 3p 6 3d 9

O 3d
O
O i.e.
C Heat C
26. (c) Ph Ph * It has one unpaired electron due to this it is coloured.
* OH CO2 CH3
The colour is due to d-d transitions.
- keto acid) (E) (Note : Cu F2 possesses blue colour in crystalline form)
EBD_780
2008- 42 Target IIT-JEE
30. (c) The correct IUPAC name of the given compound is Statement 2 is true because it is not possible to convert
tetramminenickel (II) - tetrachloronickelate (II) thus (c) the whole of heat absorbed from a reservoir into work.
is the correct answer. Some of the heat is always given to the sink.
31. (b) In carbonyls O.S. of metal is zero Also statement 2 is correct explanation for statement 1.
In [Ni(CO)4], the oxidation state of nickel is zero. Its Thus the correct choice is option (a).
configuration in Ni(CO)4 is 35. (a) In [Fe (H2O)5 NO] SO4, Let the oxidation state of Fe be
3d 4s 4p x. Then for [Fe(H2O)5 NO]2–,
x+1=+2
[Ni(CO)4] ;
or x = + 2 – 1 = + 1
sp3 hybridisation Hence in this complex the oxidation state of Fe is + 1
Electronic configuration of Fe+ can be represented as
In [Ni(CN)4]2– the oxidation state of Ni is 2+ and its Fe + = 1s 2 2s 2 2p 6 3s 2 3p 6 3d 7. This unexpected
configuration is configuration is due to presence of strong ligand field.
3d 4s 4p Due to which 1 electron from 4s1 gets shifted to 3d–
[Ni(CN)4]2 orbitals.
The 3d7 electrons in five 3d– orbitals can be shown as
dsp2 hybridisation 7
3d
3d ;
Thus the hybridisations of nickel in these compounds
are sp3 and dsp2 respectively. In it we find 3 unpaired electrons.
Hence (b) is the correct answer. Because of the presence of unpaired electrons the
32. (d) The colour of the azo dye formed will be orange red but complex is paramagnetic i.e. statement 1 is true.
not blue. However, the colour of dye can said to be due As is clear from above there are three unpaired electrons
to extended conjugation due to presence of azo group. in this complex i.e. statement 2 is true.
Since paramagnetic behaviour is due to presence of
HO unpaired electrons in it so statement 2 is correct
explanation of statement 1.
N N In view of the above facts the correct answer is
option (a)
36. (b) In 1 unit cell of hcp, the number of atoms can be
calculated as follows
Number of atoms in a unit cell of hcp
33. (b) The geometrical isomers of [M(NH3)4Cl 2] can be
represented as follows:- 1 1
12 2 3 =6
6 2
Cl Cl i.e. the correct answer is option (b)
H3N NH3 H3N Cl [Note : in a hexagonal close packing (hcp) the spheres
in the first and third layers are vertically aligned. (See
figure below]
M M

H3N NH3 H3N NH3


120°
Cl NH3 120°
(trans-form) cis - form

These isomers are optically inactive and they posses


axis of symmetry.
Both the statements are thus true. Out of two possible
answers i.e. option (a) and (b) option (b) is correct as c
the statement 2 is not a correct explanation of 4
statement 1.
For a molecule to be optically active it should not c/2
possess alternate axis of symmetry.
34. (a) Statement 1 is true because it is not possible to convert
2
whole of heat to work. For such a conversion we need
an efficiency of 100% but so far we have not been able b
to get such a machine (carnot engine).
3
2008 IIT-JEE Solved Paper 2008- 43
37. (a) The volume of hcp unit cell is given by the formula:- Thus J seems to be C6H5CHO and hence I is
Volume of hexagon = Area of base × height
PhCH C Ph
|
3 2
= 6 (2r) 2 4r CH3
4 3 I
3 Now we will try to answer the questions.
= 24 2 r 39. (b) As can be seen from above reaction sequence
i.e. the correct answer is option (a)
compound (I) is Ph CH C Ph and it is formed
38. (d) In a hcp unit cell the space occupied is 74%, as |
calculated below CH3
Volume of the atoms in a unit cell by catalytic dehydration (acid catalysed) of a tertiary
Packing fraction alcohol (compound H). Therefore compound H is
Volume of a unit cell
OH
4 3 I
6 r 22 1 Ph CH 2 – C Ph
3 I
= 3 = 3 2 = 7 CH3
24 2 r 3 2
= 0.74 or74 % OH
Empty space is HCP unit cell = (100–74)% I H
= 26% Ph CH 2 C Ph Ph CH C Ph
I H 2O I
i.e. the correct answer is option (d). CH3 CH3
For 39-41. Before answering these question let us complete the (H) (I)
sequence of reactions given in data.
O
O
H3C Ph C
The given compound (M) i.e. (H) can be formed by the action of Ph CH3 with
Ph H
PhCH2 Mg Br as follows
is the only product formed by the action of KOH on
compound K. O OMgBr
The compound K is Ph C O C C–CH2Ph
| + PhCH2Mg Br
CH3 Ph CH3 Ph CH3
OH
O Br
Hydrolysis
KOH
II Ph–CH2–C–Ph+Mg
2Ph C O Ph C CH C Ph OH
(Aldol conden.)
I | CH3
CH3 CH3
(H)
Therefore the correct answer is option (b)
Compound K (i.e. Ph C O ) is one of the products
| 40. (a) As can be seen form the above sequence of reactions
CH3 Ph CH 3
of ozonolysis of compound I. Therefore the compound the structure of compound (I) is
I may be H Ph
Therefore the correct answer is option (a).
41. (d) As can be seen from the above sequence of reactions
Ozonolysis
C C Ph CO Ph C O the structures of compounds J, K and L respectively
| | are
CH3 CH3
Ph CHO, Ph C O and PhC OO K
(I) (J) (K) | (L)
(J)
KOH CH3
C O Ph CH 2 OH
(J) Benylalcohol (K)
Thus the correct answer is option (d)
EBD_780
2008- 44 Target IIT-JEE
42. A-r, s; B-p, q; C-p, q, r; D-p, s
compound (A) does not have carbon, hence does not 1 1 b2
Ar ( ABC) = AC BC = (ae a)
responds (p) test. 2 2 a
43. A-q,r; B-p,q,r,s; C-p, q, r; D-p, q
44. A-p; B-q; C-p, r; D-p, s 1 1 3 3
= (e 1) b 2 = 1 2 1
2 2 2 2
1 u
45. (c) Given that g(u) = 2 tan (e )
2
ex
48. (c) Given I dx,
1 1 1 e4 x e2 x 1
g(–u) = 2 tan e u = 2 tan
2 eu 2
x
e e3 x
J 4x 2x
dx = dx
= 2 cot
1
eu =2 tan 1
eu e e 1 e4 x e2 x 1
2 2 2
e x (e 2 x 1)
1 u 1 u J–I= dx
= 2 tan (e ) = 2 tan (e ) e4 x e2 x 1
2 2
= – g(u) g is an odd function. Let ex t e x dx dt
u
2e 1
Also g '(u) 0, u (– , ) 1
1 e 2u t2 1 t2
J I dt dt
g is strictly increasing on (– , ) . t4 t2 1 1
t2 1
46. (d) The initial position of point is Z0 = 1+ 2i t2
Z1 = (1 + 5) + (2 + 3) i = 6 + 5i
1 1
Now Z1 is moved through a distance of 2 units in Let t u 1– dt du
t t2
the direction iˆ ˆj . (i.e. by 1 + i)
du 1 u 1
It becomes Z1 ' Z1 (1 i) 7 6i J I 2
log C
u 1 2 u 1
Now OZ1 ' is r otated through an angle in
2 t2 1
1
anticlockwise direction, therefore Z 2 iZ1 ' 6 7i 1 t 1 e2 x ex 1
= log C log C
47. (b) The given hyperbola is 2 t2 1 2 e2 x ex 1
1
x2 2 y2 2 2x 4 2 y 6 0 t

( x2 2 2 x 2) 2( y 2 2 2 y 2) 6 2 4 49. (a) Given OP aˆ cos t bˆ sin t


2
(x 2) 2 2( y 2) 2 4 OP cos 2 t sin 2 t 2aˆ.bˆ sin t cos t

(x 2) 2 (y 2) 2 2
1 aˆ.bˆ sin 2t 1 aˆ.bˆ (Max . at t )
1 OP 4
2 2
2 2
OP max 1 aˆ.bˆ
2 3
a = 2, b = 2 e= 1
4 2 Also û OP max
Clearly ABC is a right triangle.
Maximum occurs at t =
4
b2
y B ae,
a aˆ bˆ
OP max
2

aˆ bˆ
ˆ max
OP
A C(ae, 0) aˆ bˆ
(a, 0)
aˆ bˆ
Hence uˆ and M = 1 aˆ.bˆ
aˆ bˆ
2008 IIT-JEE Solved Paper 2008- 45
50. (a) Given that g(x) = log f (x) g (x + 1) = log f (x + 1)
g (x + 1) = log x f (x) [ f (x + 1) = x f (x)] x
1 tan
g(x + 1) = logx + log f (x) g(x + 1) – g(x) = log(x) 2
y= x ...(2)
1 1 tan
g '( x 1) g '( x ) 2
x
The area bounded by the above curves, by the lines
1
g "( x 1) g "( x) x = 0 and x = is given by
x2 4

1 x x
Putting, x = x – , we get /4 1 tan 1 tan
2 A 2 2 dx
x x
0 1 tan 1 tan
1 1 2 2
g" x g" x
2 2 x
/4 2 tan
2
1 22 dx
= 0
x 2
1
2
(2 x 1) 2 1 tan
x 2
2
x 1 x 2
Putting x = 1, 2, 3 ........, N we get Let tan =t sec2 dx dt dx dt
2 2 2 1 t2
3 1 22
g" g" ...(1) Also when x 0, t 0 and when x ,t tan
2 2 12 4 8
tan
5 3 22 8
4t
2 1
4t
g" g" 2 ...(2) A dt dt
2 2 3 2 2
0 (1 t ) 1 t 0 (1 t ) 1 t 2
2

7 5 22
g" g" ...(3) 52. (d) We have n (S) = 10
2 2 52 n(A) = 4
Let n(B) = x
1 1 22
g" N g" N ...(N) and n(A B) = y
2 2 (2 N 1) 2 Then for A and B to be independent events
Adding all the above equations, we get P(A B) = P(A) P(B)
y 4 x 5
1 1 x= y
g" N g" 10 10 10 2
2 2
y can be 2 or 4 so that x = 5 or 10
1 1 1 n(B) = 5 or 10
= –4 1 32 5 2
.....
(2 N 1) 2
53. (d) The given points are
P sin( ), – cos ,
51. (b) The given curves are
Q (cos( ),sin )
2
x x x x R (cos( ),sin( ))
cos sin cos sin
1 sin x 2 2 2 2
y
cos x x x x x Where 0 , ,
cos 2 sin 2 cos sin 4
2 2 2 2
1 1 1
x sin( ) cos( ) cos( )
1 tan
y = 2 ...(1) cos sin sin( )
x
1 tan
2 Operating C3 C1 sin C2 cos , we get
1 1 1 sin cos
x x
cos sin sin( ) cos( ) 0
1 sin x 2 2
and y cos sin 0
cos x x x
cos sin
2 2 = (1 sin cos )[cos cos( ) sin sin( )]
EBD_780
2008- 46 Target IIT-JEE

1 (sin cos ) cos(2 ) 56. (c) Given a1, a2 , a3 , a4 are in GP..


Then b1 , b 2 , b3 , b 4 are the numbers
0 , , sin cos 1 a1, a1 a2 , a1 a2 a3 , a1 a2 a3 a4
4
or a, a + ar, a ar ar 2 , a ar ar 2 ar 3
Also 2 cos(2 ) 0 Clearly above numbers are neither in AP nor in G.P. and
4
hence statement 1 is true.
0 the three points are non collinear..
1 1 1 1
Also , , , are
54. (c) The given circle is x 2 y2 6 x 10 y 30 0 a a ar a ar ar a ar ar 2 ar 3
2

Centre (–3, 5), radius = 2 not in A.P.


L1 : 2 x 3 y ( p 3) 0; b1 , b2, b3 , b4 are not in H.P..
L2 : 2 x 3 y p 3 0 Statement 2 is false.
57. (a) As a, b, c, p, q, R and the two given equations have
Clearly L1 || L2 exactly one common root
Either both equations have real roots
Distance between L1 and L2 or both eqations have imaginary roots
p 3 p 3 6 Either 1 0and 2 0 or 1 0 and 2 0
= 2
2 2 2
2 3 13 p2 q 0 and b ac 0
If one line is a chord of the given circle, other line 2
0 and b 2
or p q ac 0
may or may not the diameter of the circle.
Statement 1 is true and statement 2 is false. ( p 2 q)(b 2 ac ) 0
Statement 1 is true.
55. (c) The given differential equation is
c
2 2 Also we have q and
x x 1 dy y y 1 dx 0 a

dy dx q 2 qa
a
/ c c
y y2 1 x x2 1
2 qa
1 1 As 1 or –1 1 1 or c qa
sec y sec x C c
2 Again, as exactly one root is common, and 1
1
y = sec sec x C y (2) =
3 1 2b
2p
a
2 1
= sec sec 2 C b ap
3
Statement 2 is correct.
1 2 1 Also b ap and c aq both equations have
sec sec 2 C
3 exactly one common root.
Statement 2 is a correct explanation of Statement 1.
C y = sec sec 1 x x2 ax 1
6 3 6 6 58. (a) We have f ( x ) ;0<a<2
2
Statement 1 is true. x ax 1

1 1 1 2a( x 2 1)
Also = cos cos f '( x )
y x 6 ( x2 ax 1) 2

11 1 ( x2 ax 1) 2 f '( x) 2a( x 2 1)
= cos cos cos sin cos 1 sin
x 6 x 6
( x2 ax 1)2 f "( x) 2( x 2 ax 1)
1 3 1 1 (2 x a) f '( x) 4ax ...(1)
1 2
y 2x 2 x Putting x = –1 in equation (1), we get
Statement 2 is false. (2 a 2 ) f ''( 1) 4a …(2)
2008 IIT-JEE Solved Paper 47
2008-

Putting x = 1 in equation (1), we get


iˆ 7 ˆj 5kˆ 3 20 17
(2 a)2 f "(1) 4a ...(3) 3iˆ 4kˆ . =
5 3 5 3 5 3
Adding equations (2) and (3), we get
63. (c) The plane passing through (–1, –2, –1) and having
(2 a )2 f "(1) (2 a) 2 f "( 1) 0 normal along n is
– 1(x + 1) – 7(y + 2) + 5(z + 1) = 0
2a ( x 2 1)
59. (a) We have f '( x ) or x 7 y 5 z 10 0
( x2 ax 1) 2 Distance of point (1, 1, 1) from the above plane is
f '( x) 0 x = –1, 1 are the critical points. 1 7 1 5 1 10 13
= =
| |
1 49 25 75
1 1
64. A-s; B-p, q; C-r; D-p, q, s
f (x) is decreasing on (–1, 1) The given lines are
4a L1 : x 3 y 5 0
Also using equation (1), f "( 1) 0
(2 a ) 2
L2 : 3x ky 1 0
4a L3 : 5 x 2 y 12 0
and f "(1) 0
(2 a ) 2
(A) Three lines L1, L2 , L3 are concurrent if
x = – 1 is a point of local maximum
and x = 1 is a point of local minimum.
1 3 5
ex 3 k 1 0 13k 65 0 k 5
f '(t ) f '(e x )
60. (b) g ( x) dt g '( x )
1 t 2
1 e 2x 5 2 12
0

(A) (s)
2 a(e 2 x 1)e x 2 ae x e2 x 1
2x x 2 2x = 2x
. 2x x 2
(e ae 1) (1 e ) 1 e (e ae 1) 1 3
(B) For L1 || L2 k –9
2x 3 k
Now g '( x ) 0 for e 1 0 x>0
and g '( x) 0 for e 1 0 2x
x 0 3 k 6
and L2 || L3 k
g'(x) is negative on (– , 0) and positive on 5 2 5
(0, ) (B) (p), (q)
61. (b) Vector in the direction of L1 n1 3iˆ ˆj 2kˆ (C) Three lines L1, L2, L3 will form a triangle if no two of
them are parallel and no three are concurrent
Vector in the direction of L2 n2 iˆ 2 ˆj 3kˆ
k 5, –9, – 6/5 (C) r
Vector perpendicular to both L1 and L2
(D) L1, L2 , L3 do not form a triangle if either any two of
iˆ ˆj kˆ
these are parallel or the three are concurrent i.e.
= n1 n2 = 3 1 2 iˆ 7 ˆj 5kˆ k = 5, – 9 , – 6/5
1 2 3
(D) (p), (q), (s)
Required unit vector 65. A-p; B-s; C-q; D-q
iˆ 7 ˆj 5kˆ iˆ 7 ˆj 5kˆ (A) For the permutations containing the word ENDEA we
= nˆ consider 'ENDEA' as single letter. Then we have total
1 49 25 5 3
ENDEA, N, O, E, L i.e. 5 letters which can be arranged
62. (d) The shortest distance between L1 and L2 is
in 5! ways.
( a2 a1 ).b1 b2 (A) (p)
= (a2 a1 ).nˆ
b1 b2 (B) If E occupies the first and last position, the middle 7
positions can be filled by N, D, E, A, N, O, L.
where a1 iˆ 2 ˆj kˆ
7!
a2 2iˆ 2 ˆj 3kˆ These can be arranged in 21 5! ways.
2!
a2 a1 3iˆ 4kˆ a2 a1 .nˆ (B) (s)
EBD_780
2008- 48 Target IIT-JEE
(C) If none of the letters D, L, N occur in the last five Now given that
positions then we should arrange D, D, L, N at first
(AB)t ( 1) k AB
four positions and rest five i.e. E, E, E, A,O at last five
positions. This can be done in
(BA)t ( 1)k AB (using equation (2))
4! 5!
ways . At Bt ( 1)k AB
2! 3!
(C) (q) –AB = –1 k AB [using equation(1)]
(D) As per question A, E, E, E, O can be arranged at 1st, k should be an odd number
3rd, 5th, 7th and 9th positions and rest N, D, N, L at (B) (q), (s)
rest 4 positions. This can be done in
(C) Given that a log3 log3 2
5! 4!
ways 2 5!ways
3! 2! 1
log3 2 3a 3a or log 2 3 3 a
log 2 3
(D) (q)
a
66. A-r; B-q, s; C-r, s; D-p, r 3 = 2(3 )
...(1)

x2
a
2x 4 Now 1 2( k 3 )
a
(A) Let y 2 1 2 k .23 2
x 2
1 2 k .3 2 (using eq (1))
2
x (2 y ) x (4 2 y ) 0
1 2 3
For x to be real, we have . 2–k 2k 3
3 3 2
D 0 k=1
(2 – y )2 –4 (4 – 2y) 0 2 k is less than 2 and 3
y 4 y 12 0
(C) (r), (s).
(y + 6) (y – 2) 0 y –6 or y 2
(D) Given that sin cos cos cos
ymin does not exist and local minimum value of 2
y = 2.
(A) r 2n , n Z 2n
2 2
(B) As A is symmetric and B is skew symmetric matrix, we
1
should have –2 n
2
At = A and Bt = – B ...(1)
Also given that 1
Here possible values of are 0 and 2
(A + B) (A – B) = (A– B) (A + B) 2
for n = 0, –1.
A 2 AB BA B2 A 2 AB BA B2
D (p) ,(r).
2BA = 2AB or AB = BA ...(2)
IIT-JOINT ENTRANCE EXAMINATION (IIT-JEE)
SOLVED PAPER - 2007

PAPER - I
PHYSICS
––––––––––––––––––––––––––––––––––––––––––––––––––––––––––––––––––––––––––––––––––––––––––––––––––––
General Instructions
1. Section I : Q. No. 1 to 9 have only one correct answer and carries 3 marks each for correct answer and –1 mark for each wrong
answer.
2. Section II : Q. No. 10 to 13 are Statement-1(Assertion) and Statement-2(Reason) type questions and carries 3 marks each for
correct answer and –1 mark for each wrong answer.
3. Section III : Q. No. 14 to 19 are based on the given paragraph which have only one correct answer and carries 4 marks each
for correct answer and –1 marks for each wrong answer.
4. Section IV : Q. No. 20 to 22 are matching type questions each question has four parts and each question carries 6 marks and
marks will be awarded if all the four parts are correctly matched. No negative mark will be awarded for an incorrectly bubbled
answer.
––––––––––––––––––––––––––––––––––––––––––––––––––––––––––––––––––––––––––––––––––––––––––––––––––––
(b) A potential difference appears between the two
Section - I (Straight Objective Type) cylinders when a charge density is given to the outer
INSTRUCTION : This section contains 9 multiple choice cylinder.
questions numbered 1 to 9. Each question has 4 choices (a), (b), (c) No potential difference appears between the two
(c) and (d), out of which ONLY ONE is correct. cylinders when a uniform line charge is kept along the
axis of the cylinders
(d) No potential difference appears between the two
1. In an experiment to determine the focal length
cylinders when same charge density is given to both
(f ) of a concave mirror by the u - v method, a student places the cylinders.
the object pin A on the principal axis at a distance x from the 4. Two particles of mass m each are tied at the ends of a light
pole P. The student looks at the pin and its inverted image string of length 2a. The whole system is kept on a frictionless
from a distance keeping his/her eye in line with PA. When horizontal surface with the string held tight so that each
the student shifts his/her eye towards left, the image appears mass is at a distance 'a' from the centre P (as shown in the
to the right of the object pin. Then, figure). Now, the mid-point of the string is pulled vertically
(a) x < f (b) f < x < 2f upwards with a small but constant force F. As a result, the
(c) x = 2f (d) x > 2f particles move towards each other on the surface. The
2. A resistance of 2 is connected across one gap of a metre- magnitude of acceleration, when the separation between
bridge (the length of the wire is 100 cm) and an unknown them becomes 2x, is
resistance, greater than 2 , is connected across the other F
gap. When these resistances are interchanged, the balance
point shifts by 20 cm. Neglecting any corrections, the
unknown resistance is m m
(a) 3 (b) 4 P
(c) 5 (d) 6 a a
3. A long, hollow conducting cylinder is kept coaxially inside
another long, hollow conducting cylinder of larger radius. F a F x
(a) (b)
Both the cylinders are initially electrically neutral. 2m a 2 - x 2 2m a 2 - x 2
(a) A potential difference appears between the two
cylinders when a charge density is given to the inner F x F a2 - x2
(c) (d)
cylinder. 2m a 2m x
EBD_780
2007- 2 Target IIT-JEE
5. A circuit is connected as shown in the figure with the switch
Section - II (Assertion-Reason Type)
S open. When the switch is closed, the total amount of
charge that flows from Y to X is INSTRUCTION : This section contains 4 questions numbered 10
to 13. Each question contains STATEMENT–1 (Assertion) and
3 F 6 F
X STATEMENT–2 (Reason). Each question has 4 choices (a), (b),
(c) and (d) out of which ONLY ONE is correct.
10. STATEMENT–1
S In an elastic collision between two bodies, the relative speed
of the bodies after collision is equal to the relative speed
3 6
before the collision.
Y
because
9V STATEMENT–2
(a) 0 (b) 54 C In an elastic collision, the linear momentum of the system is
(c) 27 C (d) 81 C conserved.
6. Consider a neutral conducting sphere. A positive point (a) Statement–1 is True, Statement–2 is True; Statement–
charge is placed outside the sphere. The net charge on the 2 is a correct explanation for Statement–1.
sphere is then (b) Statement–1 is True, Statement–2 is True; Statement–
(a) negative and distributed uniformly over the surface of 2 is NOT a correct explanation for Statement–1
the sphere (c) Statement–1 is True, Statement–2 is False.
(b) negative and appears only at the point on the sphere (d) Statement–1 is False, Statement–2 is True.
closest to the point charge 11. STATEMENT–1 : A block of mass m starts moving on a
(c) negative and distributed non-uniformly over the entire rough horizontal surface with a velocity v. It stops due to
surface of the sphere friction between the block and the surface after moving
(d) zero through a certain distance. The surface is now tilted to an
angle of 30° with the horizontal and the same block is made
7. In the options given below, let E denote the rest mass energy
to go up on the surface with the same initial velocity v. The
of a nucleus and n a neutron.The correct option is
decrease in the mechanical energy in the second situation
is smaller than that in the first situation.
236 137 97 STATEMENT–2 : The coefficient of friction between the
(a) E U E I E Y 2 E (n ) block and the surface decreases with the increase in the
92 53 39
angle of inclination.
(a) Statement–1 is True, Statement–2 is True; Statement–
236 137 97
(b) E U E I E Y 2 E (n ) 2 is a correct explanation for Statement–1
92 53 39 (b) Statement–1 is True, Statement–2 is True; Statement–
2 is NOT a correct explanation for Statement–1
236 140 94 (c) Statement–1 is True, Statement–2 is False
(c) E U E Ba E Kr 2 E ( n)
92 56 36 (d) Statement–1 is False, Statement–2 is True
12. STATEMENT-1
236 140 94
(d) E U E Ba E Kr 2 E ( n) If the accelerating potential in an X-ray tube is increased,
92 56 36
the wavelengths of the characteristic X-rays do not change.
8. A ray of light traveling in water is incident on its surface
open to air. The angle of incidence is , which is less than because
the critical angle. Then there will be
(a) only a reflected ray and no refracted ray STATEMENT-2
(b) only a refracted ray and no reflected ray When an electron beam strikes the target in an X-ray tube,
(c) a reflected ray and a refracted ray and the angle between part of the kinetic energy is converted into X-ray energy.
them would be less than 180° – 2
(d) a reflected ray and a refracted ray and the angle between (a) Statement-1 is True, Statement-2 is True; Statement-2 is
them would be greater than 180° – 2 a correct explanation for Statement-1
9. The largest wavelength in the ultraviolet region of the
hydrogen spectrum is 122 nm. The smallest wavelength in (b) Statement-1 is True, Statement-2 is True; Statement-2
the infrared region of the hydrogen spectrum (to the nearest is NOT a correct explanation for Statement-1
integer) is (c) Statement-1 is True, Statement-2 is False
(a) 802 nm (b) 823 nm
(c) 1882 nm (d) 1648 nm (d) Statement-1 is False, Statement-2 is True
2007 IIT-JEE Solved Paper 2007- 3
13. STATEMENT-1 15. While the piston is at a distance 2L from the top, the hole at
The formula connecting u, v and f for a spherical mirror is the top is sealed. The piston is then released, to a position
valid for mirrors whose sizes are very small compared to where it can stay in equilibrium. In this condition, the
their radii of curvature. distance of the piston from the top is
because æ 2P pR 2 ö÷ æ P pR 2 - Mg ÷ö
çç (b) ççç 0
÷÷÷ (2L) ÷÷ (2L)
(a) 0
çç 2
çè pR 2 P0 ÷÷ø
STATEMENT-2
è pR P0 + Mg ÷ø
Laws of reflection are strictly valid for plane surfaces, but
not for large spherical surfaces.
æ ö æ P pR 2 ö÷
çç P0 pR + Mg ÷÷
2
(a) Statement-1 is True, Statement-2 is True; Statement-2 (c) ç ÷÷ (2L) (d) ççç 0 ÷÷ (2L)
is a correct explanation for Statement-1 çè pR P 2 ÷ø0 çè pR 2 P0 - Mg ÷÷ø
(b) Statement-1 is True, Statement-2 is True; Statement-2 16. The piston is taken completely out of the cylinder. The hole
is NOT a correct explanation for Statement-1 at the top is sealed. A water tank is brought below the cylinder
(c) Statement-1 is True, Statement-2 is False and put in a position so that the water surface in the tank is
(d) Statement-1 is False, Statement-2 is True at the same level as the top of the cylinder as shown in the
figure. The density of the water is . In equilibrium, the height
H of the water column in the cylinder satisfies
Section - III (Linked Comprehension Type)
INSTRUCTION : This section contains 2 paragraphs P14-16 and
P17-19. Based upon each paragraph, 3 multiple choice questions
have to be answered. Each question has 4 choices (a), (b), (c) and
(d), out of which ONLY ONE is correct.
L0
P14-16 : Paragraph for Question Nos. 14 to 16
A fixed thermally conducting cylinder has a radius R and H
height L0. The cylinder is open at its bottom and has a small
hole at its top. A piston of mass M is held at a distance L
from the top surface, as shown in the figure. The atmospheric (a) g(L0 – H)2 + P0(L0 – H) + L0P0 = 0
pressure is P0. (b) g(L0 – H)2 – P0(L0 – H) – L0P0 = 0
(c) g(L0 – H)2 + P0(L0 – H) – L0P0 = 0
(d) g(L0 – H)2 – P0(L0 – H) + L0P0 = 0
2R
P17-19 : Paragraph for Question Nos. 17 to 19
L
Two discs A and B are mounted coaxially on a vertical axle.
The discs have moments of inertia I and 2 I respectively
L0 about the common axis. Disc A is imparted an initial angular
velocity 2 using the entire potential energy of a spring
compressed by a distance xl. Disc B is imparted an angular
velocity by a spring having the same spring constant and
compressed by a distance x2 .Both the discs rotate in the
Piston clockwise direction.
17. The ratio x1/x2 is
1 1
14. The piston is now pulled out slowly and held at a distance (a) 2 (b) (c) 2 (d)
2L from the top. The pressure in the cylinder between its 2 2
top and the piston will then be 18. When disc B is brought in contact with disc A, they acquire
a common angular velocity in time t . The average frictional
P0 torque on one disc by the other during this period is
(a) P0 (b)
2
2I 9I 9I 3I
(a) (b) (c) (d)
P0 Mg P0 Mg 3t 2t 4t 2t
(c) + (d) - 19. The loss of kinetic energy in the above process is
2 pR 2 2 pR 2
2 2 2 2
Therefore the pressure inside the cylinder is P0 throughout I I I I
(a) (b) (c) (d)
the slow pulling process. 2 3 4 6
EBD_780
2007- 4 Target IIT-JEE

Section - IV (Matrix Match Type)


INSTRUCTION : This section contains 3 questions. Each question contains statements given in two columns which have to be
matched. Statements (A, B, C, D ) in Column I have to be matched with statements (p, q, r, s)in Column II. The answers to these
questions have to be appropriately bubbled as illustrated in the following example. If the correct matches are A-p, A-s, B - q,B-r, C-p,
C-q and D-s, then the correctly bubbled 4 × 4 matrix should be as follows :

p q r s
A p q r s
B p q r s
C p q r s
D p q r s

20. Column I gives certain situations in which a straight metallic wire of resistance R is used and Column II gives some resulting
effects. Match the statements in Column I with the statements in Column II and indicate your answer by darkening appropriate
bubbles in the 4 × 4 matrix given in the ORS.
Column I Column II
(A) A charged capacitor is connected to the ends of (p) A constant current flows through the wire
the wire
(B) The wire is moved perpendicular to its length with (q) Thermal energy is generated in the wire
a constant velocity in a uniform magnetic field
perpendicular to the plane of motion
(C) The wire is placed in a constant electric field that (r) A constant potential difference develops between the ends
has a direction along the length of the wire of the wire
(D) A battery of constant emf is connected to the (s) charges of constant magnitude appear at the ends of the wire
ends of the wire.
21. Some physical quantities are given in Column I and some possible SI units in which these quantities may be expressed are given
in Column II. Match the physical quantities in Column I with the units in Column II and indicate your answer by darkening
appropriate bubbles in the 4 × 4 matrix given in the ORS.
Column I Column II
(A) GMeMs (p) (volt) (coulomb)(metre)
G – universal gravitational constant,
Me – mass of the earth, Ms – mass of the Sun
3RT
(B) (q) (kilogram) (metre)3(second)–2
M
R – universal gas constant,
T – absolute temperature, M – molar mass

F2
(C) (r) (metre)2 (second)–2
q2 B 2
F – Force, q – charge, B – magnetic field
GM e
(D) (s) (farad) (volt)2 (kg)–1
Re
G – universal gravitational constant,
Me – mass of the earth, Re – radius of the earth
22. Some laws / processes are given in Column I. Match these with the physical phenomena given in Column II and indicate your
answer by darkening appropriate bubbles in the 4 × 4 matrix given in the ORS.
Column I Column II
(A) Transition between two atomic energy levels (p) Characteristic X-rays
(B) Electron emission from a material (q) Photoelectric effect
(C) Mosley's law (r) Hydrogen spectrum
(D) Change of photon energy into kinetic energy of (s) -decay
electrons
2007 IIT-JEE Solved Paper 5
2007-

CHEMISTRY
––––––––––––––––––––––––––––––––––––––––––––––––––––––––––––––––––––––––––––––––––––––––––––––––––––
General Instructions
1. Section I : Q. No. 23 to 31 have only one correct answer and carries 3 marks each for correct answer and –1 mark for each
wrong answer.
2. Section II : Q. No. 32 to 35 are Statement-1(Assertion) and Statement-2(Reason) type questions and carries 3 marks each for
correct answer and –1 mark for each wrong answer.
3. Section III : Q. No. 36 to 41 are based on the given paragraph which have only one correct answer and carries 4 marks each
for correct answer and –1 marks for each wrong answer.
4. Section IV : Q. No. 42 to 44 are matching type questions each question has four parts and each question carries 6 marks and
marks will be awarded if all the four parts are correctly matched. No negative mark will be awarded for an incorrectly bubbled
answer.
––––––––––––––––––––––––––––––––––––––––––––––––––––––––––––––––––––––––––––––––––––––––––––––––––––

Section - I (Straight Objective Type) 26. Extraction of zinc from zinc blende is achieved by
INSTRUCTION : This section contains 9 multiple choice (a) electrolytic reduction
questions numbered 23 to 31. Each question has 4 choices (a), (b), (b) roasting followed by reduction with carbon
(c) and (d), out of which ONLY ONE is correct. (c) roasting followed by reduction with another metal
23. Among the following, the paramagnetic compound is (d) roasting followed by self-reduction
(a) Na2O2 (b) O3 27. The number of structural isomers for C6H14 is
(c) N2O (d) KO2 (a) 3 (b) 4
(c) 5 (d) 6
24. The species having bond order different from that in CO is
28. The reagent(s) for the following conversion,
(a) NO– (b) NO+
(c) CN – (d) N2 Br ?
H H
Br
25. In the following reaction,
is/are
O (a) alcoholic KOH
Conc . HNO 3
X (b) alcoholic KOH followed by NaNH2
N Conc . H 2SO 4
(c) aqueous KOH followed by NaNH2
H
(d) Zn/CH3OH
the structure of the major product 'X' is 29. When 20 g of naphthoic acid (C11H8O2) is dissolved in 50 g
of benzene (Kf = 1.72 K kg mol–1), a freezing point depression
O of 2K is observed. The Van't Hoff factor (i) is
(a) (a) 0.5 (b) 1
NO 2
N
(c) 2 (d) 3
H
30. The percentage of p-character in the orbitals forming P–P
bonds in P4 is
O2 N O (a) 25 (b) 33
(b) (c) 50 (d) 75
N
31. The value of log10 K for a reaction A B is
H
(Given : r H 298K 54.07 kJ mol 1, r S298K
O = 10 JK–1 mol–1 and R = 8.314 JK–1 mol–1;
(c) 2.303 × 8.314 × 298 = 5705)
N
(a) 5 (b) 10
H
NO 2 (c) 95 (d) 100
Section - II (Assertion-Reason Type)
O INSTRUCTION : This section contains 4 questions numbered 32
(d) to 35. Each question contains STATEMENT-1 (Assertion) and
O2 N N
STATEMENT-2 (Reason). Each question has 4 choices (a), (b), (c)
H
and (d) out of which ONLY ONE is correct.
EBD_780
2007- 6 Target IIT-JEE
32. Statement-1 : In water, orthoboric acid behaves as a weak Section - III (Linked Comprehension Type)
monobasic acid.
INSTRUCTION : This section contains 2 paragraphs C36-38 and
because C39-41. Based upon each paragraph, 3 multiple choice questions
Statement-2 : In water, orthoboric acid acts as a proton have to be answered. Each question has 4 choices (a), (b), (c) and
donor. (d), out of which ONLY ONE is correct.
(a) Statement-1 is True, Statement-2 isTrue; Statement-2
C36-38 : Paragraph for Question Nos. 36 to 38
is a correct explanation for Statement-1
Chemical reactions involve interaction of atoms and molecules. A
(b) Statement-1 is True, Statement-2 is True; large number of atoms/molecules (approximately 6.023 × 1023) are
Statement-2 is NOT a correct explanation for present in a few grams of any chemical compound varying with
Statement-1 their atomic/molecular masses. To handle such large numbers
(c) Statement-1 is True, Statement-2 is False conveniently, the mole concept was introduced. This concept has
implications in diverse areas such as analytical chemistry,
(d) Statement-1 is False, Statement-2 is True.
biochemistry, electrochemistry and radiochemistry. The following
33. Statement-1 : Boron always forms covalent bond. example illustrates a typical case, involving chemical/electrochemical
because reaction, which requires a clear understanding of the mole concept.
Statement-2 : The small size of B3+ favours formation of A 4.0 molar aqueous solution of NaCl is prepared and 500 mL of
covalent bond. this solution is electrolysed. This leads to the evolution of
chlorine gas at one of the electrodes (atomic mass : Na = 23,
(a) Statement-1 is True, Statement-2 isTrue; Statement-2 Hg = 200; 1 Faraday = 96500 coulombs).
is a correct explanation for Statement-1 36. The total number of moles of chlorine gas evolved is
(b) Statement-1 is True, Statement-2 is True; (a) 0.5 (b) 1.0
Statement-2 is NOT a correct explanation for (c) 2.0 (d) 3.0
Statement-1 37. If the cathode is a Hg electrode, the maximum weight (g) of
(c) Statement-1 is True, Statement-2 is False amalgam formed from this solution is
(d) Statement-1 is False, Statement-2 is True. (a) 200 (b) 225
(c) 400 (d) 446
34. STATEMENT-1 : Micelles are formed by surfactant
38. The total charge (coulombs) required for complete
molecules above the critical micellar concentration (CMC).
electrolysis is
because
(a) 24125 (b) 48250
STATEMENT-2 : The conductivity of a solution having (c) 96500 (d) 193000
surfactant molecules decreases sharply at the CMC.
C39-41 : Paragraph for Question Nos. 39 to 41
(a) Statement-1 is True, Statement-2 isTrue; Statement-2
The noble gases have closed-shell electronic configuration and
is a correct explanation for Statement-1 are monoatomic gases under normal conditions. The low boiling
(b) Statement-1 is True, Statement-2 is True; points of the lighter noble gases are due to weak dispersion forces
Statement-2 is NOT a correct explanation for between the atoms and the absence of other interatomic interac-
Statement-1 tions.
(c) Statement-1 is True, Statement-2 is False The direct reaction of xenon with fluorine leads to a series of
compounds with oxidation numbers +2, +4 and +6. XeF4 reacts
(d) Statement-1 is False, Statement-2 is True. violently with water to given XeO3. The compounds of xenon
35. Statement-1 : p-Hydroxybenzoic acid has a lower boiling exhibit rich stereochemistry and their geometries can be deduced
point than o-hydroxybenzoic acid. considering the total number of electron pairs in the valence shell.
because 39. Argon is used in arc welding because of its
Statement-2 : o-Hydroxybenzoic acid has intramolecular (a) low reactivity with metal
hydrogen bonding. (b) ability to lower the melting point of metal
(c) flammability
(a) Statement-1 is True, Statement-2 isTrue; Statement-2
(d) high calorific value
is a correct explanation for Statement-1
40. The structure of XeO3 is
(b) Statement-1 is True, Statement-2 is True; (a) linear (b) planar
Statement-2 is NOT a correct explanation for (c) pyramidal (d) T-shaped
Statement-1 41. XeF4 and XeF6 are expected to be
(c) Statement-1 is True, Statement-2 is False (a) oxidizing (b) reducing
(d) Statement-1 is False, Statement-2 is True. (c) unreactive (d) strongly basic
2007 IIT-JEE Solved Paper 2007-7
Section - IV (Matrix Match Type)
INSTRUCTION : This section contains 3 questions. Each question contains statements given in two columns which have to be
matched. Statements (A, B, C, D) in Column I have to be matched with statements (p, q, r, s) in Column II. The answers to these
questions have to be appropriately bubbled as illustrated in the following example. If the correct matches are A-p, A-s, B-q, B-r, C-p, C-
q and D-s, then the correctly bubbled 4 × 4 matrix should be as follows :

p q r s
A p q r s
B p q r s
C p q r s
D p q r s

42. Match the chemical substances in Column I with type of polymers/type of bonds in Column II. Indicate your answer by
darkening the appropriate bubbles of the 4 × 4 matrix given in the ORS.
Column I Column II
(A) Cellulose (p) Natural polymer
(B) Nylon-6, 6 (q) Synthetic polymer
(C) Protein (r) Amide linkage
(D) Sucrose (s) Glycoside linkage
43. Match the complexes in Column I with their properties listed in Column II. Indicate your answer by darkening the appropriate
bubbles of the 4 × 4 matrix given in the ORS.
Column I Column II
(A) [Co(NH3)4(H2O)2] Cl2 (p) Geometrical isomers
(B) [Pt(NH3)2Cl2] (q) Paramagnetic
(C) [Co(H2O)5Cl]Cl (r) Diamagnetic
(D) [Ni(H2O)6]Cl2 (s) Metal ion with +2 oxidation state
44. Match gases under specified conditions listed in Column I with their properties/laws in Column II. Indicate your answer by
darkening the appropriate bubbles of the 4 × 4 matrix given in the ORS.
Column I Column II
(A) Hydrogen gas (P = 200 atm, T = 273 K) (p) Compressibility factor 1
(B) Hydrogen gas (P ~ 0, T = 273 K) (q) Attractive forces are dominant
(C) CO2 (P = 1 atm, T = 273 K) (r) PV = nRT
(D) Real gas with very large molar volume (s) P(V – nb) = nRT

MATHEMATICS
––––––––––––––––––––––––––––––––––––––––––––––––––––––––––––––––––––––––––––––
General Instructions
1. Section I : Q. No. 45 to 53 have only one correct answer and carries 3 marks each for correct answer and –1 mark for each
wrong answer.
2. Section II : Q. No. 54 to 57 are Statement-1(Assertion) and Statement-2(Reason) type questions and carries 3 marks each for
correct answer and –1 mark for each wrong answer.
3. Section III : Q. No. 58 to 63 are based on the given paragraph which have only one correct answer and carries 4 marks each
for correct answer and –1 marks for each wrong answer.
4. Section IV : Q. No. 64 to 66 are matching type questions each question has four parts and each question carries 6 marks and
marks will be awarded if all the four parts are correctly matched. No negative mark will be awarded for an incorrectly bubbled
answer.
––––––––––––––––––––––––––––––––––––––––––––––––––––––––––––––––––––––––––––––––––––––––––––––––––––
EBD_780
2007- 8 Target IIT-JEE
51. A man walks a distance of 3 units from the origin towards
Section - I (Straight Objective Type) the north-east (N 45° E) direction. From there, he walks a
INSTRUCTION : This section contains 9 multiple choice distance of 4 units towards the north-west (N 45° W)
questions numbered 45 to 53. Each question has 4 choices (a), (b), direction to reach a point P. Then the position of P in the
(c) and (d), out of which ONLY ONE is correct. Argand plane is
(a) 3ei /4 + 4i (b) (3 – 4i)ei /4
45. Let f (x) be differentiable on the interval (0, ) such that (c) (4 + 3i)e i /4 (d) (3 + 4i)ei /4
2 2
t f ( x) x f t 52. The number of distinct real values of , for which the vectors
f (1) = 1, and lim 1 for each x > 0. Then
t x t x 2ˆ
i ˆj kˆ , iˆ 2ˆ
j kˆ and iˆ ˆj 2
kˆ are coplanar, is
f (x) is
(a) zero (b) one
2 2 (c) two (d) three
1 2x 1 4x
(a) (b)
3x 3 3x 3 53. The number of solutions of the pair of equations
2sin2 – cos2 = 0
1 2 1 2cos2 – 3sin = 0
(c) 2 (d)
x x x in the interval [0, 2 ] is
46. Let , be the roots of the equation x2 – px + r = 0 and (a) zero (b) one
(c) two (d) four
, 2 be the roots of the equation x2 – qx + r = 0. Then the
2 Section - II (Assertion-Reason Type)
value of r is INSTRUCTION : This section contains 4 questions numbered 54
2 2 to 57. Each question contains STATEMENT-1 (Assertion) and
(a) ( p q)(2q p) (b) (q p)(2 p q) STATEMENT-2 (Reason). Each question has 4 choices (a), (b), (c)
9 9
and (d) out of which ONLY ONE is correct.
2 2 54. Tangents are drawn from the point (17, 7) to the circle
(c) (q 2 p)(2q p) (d) (2 p q)(2q p)
9 9 x2 + y2 = 169.
47. The tangent to the curve y = e x drawn at the point (c, e c) STATEMENT-1 : The tangents are mutually perpendicular.
intersects the line joining the points (c – 1, e c–1 ) and because
(c + 1, ec + 1) STATEMENT-2 : The locus of the points from which mutually
(a) on the left of x = c (b) on the right of x = c perpendicular tangents can be drawn to the given circle is
(c) at no point (d) at all points x2 + y2 = 338.
48. One Indian and four American men and their wives are to be
(a) Statement-1 is True, statement-2 is True; Statement-2
seated randomly around a circular table. Then the conditional
is a correct explanation for Statement-1.
probability that the Indian man is seated adjacent to his
(b) Statement-1 is True, Statement-2 is True; Statement-2
wife given that each American man is seated adjacent to his
is NOT a correct explanation for Statement-1
wife is
(c) Statement-1 is True, Statement-2 is False
1 1 2 1 (d) Statement-1 is False, Statement-2 is True.
(a) (b) (c) (d)
2 3 5 5 55. Let H1, H2, ... , Hn be mutually exclusive and exhaustive
49. A hyperbola, having the transverse axis of length 2 sin , is events with P(Hi) > 0, i = 1, 2, ..., n. Let E be any other event
confocal with the ellipse 3x2 + 4y2 = 12. Then its equation is
with 0 < P(E) < 1.
(a) x2cosec2 – y2sec2 = 1 (b) x2sec2 – y2cosec2 = 1
STATEMENT-1 :
(c) x2sin2 – y2cos2 = 1 (d) x2cos2 – y2sin2 = 1
P(Hi | E) > P(E | Hi). P(Hi) for i = 1, 2, ..., n because
sec 2 x n
f (t )dt STATEMENT-2 : P( Hi ) 1 .
50. 2 equals i 1
lim
2
x 2
4 x (a) Statement-1 is True, statement-2 is True; Statement-2 is
16
a correct explanation for Statement-1.
8 2 (b) Statement-1 is True, Statement-2 is True; Statement-2
(a) f (2) (b) f (2)
is NOT a correct explanation for Statement-1
(c) Statement-1 is True, Statement-2 is False
2 1
(c) f (d) 4f(2) (d) Statement-1 is False, Statement-2 is True.
2
2007 IIT-JEE Solved Paper 2007-9
56. Let F(x) be an indefinite integral of sin2x. 58. The ratio of the areas of the triangles PQS and PQR is
STATEMENT-1 : The function F(x) satisfies F(x + ) = F(x) (a) 1: 2 (b) 1 : 2
for all real x. because
(c) 1 : 4 (d) 1 : 8
STATEMENT-2 : sin2(x + ) = sin2x for all real x. 59. The radius of the circumcircle of the triangle PRS is
(a) Statement-1 is True, statement-2 is True; Statement-2
(a) 5 (b) 3 3
is a correct explanation for Statement-1.
(b) Statement-1 is True, Statement-2 is True; Statement-2 (c) 3 2 (d) 2 3
is NOT a correct explanation for Statement-1 60. The radius of the incircle of the triangle PQR is
(c) Statement-1 is True, Statement-2 is False (a) 4 (b) 3
(d) Statement-1 is False, Statement-2 is True. 8
(c) (d) 2
3
57. Let the vectors PQ, QR, RS , ST , TU and UP represent
M61-63 : Paragraph for Question Nos. 61 to 63
the sides of a regular hexagon. Let Vr denote the sum of first r terms of an arithmetic
progression (A.P.) whose first term is r and the common
STATEMENT-1 : PQ RS ST 0 . because
difference is (2r – 1). Let Tr = Vr + 1 – Vr – 2 and Qr = Tr + 1
– Tr for r = 1, 2, ...
STATEMENT-2 : PQ RS 0 and PQ ST 0. 61. The sum V1 + V2 + ... + Vn is
(a) Statement-1 is True, statement-2 is True; Statement-2 is 1
a correct explanation for Statement-1. (a) n(n 1)(3n2 n 1)
12
(b) Statement-1 is True, Statement-2 is True; Statement-2
is NOT a correct explanation for Statement-1 1
(b) n(n 1)(3n 2 n 2)
(c) Statement-1 is True, Statement-2 is False 12

(d) Statement-1 is False, Statement-2 is True. 1


(c) n(2n2 n 1)
2

Section - III (Linked Comprehension Type) (d)


1
(2n3 2n 3)
INSTRUCTION : This section contains 2 paragraphs M58-60 and 3
M61-63. Based upon each paragraph, 3 multiple choice questions 62. Tr is always
have to be answered. Each question has 4 choices (a), (b), (c) and (a) an odd number (b) an even number
(d), out of which ONLY ONE is correct. (c) a prime number (d) a composite number
M58-60 : Paragraph for Question Nos. 58 to 60 63. Which one of the following is a correct statement ?
Consider the circle x2 + y2 = 9 and the parabola y2 = 8x. They (a) Q1, Q2, Q3, ... are in A.P. with common difference 5
intersect at P and Q in the first and the fourth quadrants, (b) Q1, Q2, Q3, ... are in A.P. with common difference 6
respectively. Tangents to the curcle at P and Q intersect the (c) Q1, Q2, Q3, ... are in A.P. with common difference 11
x-axis at R and tangents to the parabola at P and Q intersect (d) Q1= Q2 = Q3 = ....
the x-axis at S.

Section - IV (Matrix Match Type)


INSTRUCTION : This section contains 3 questions. Each question contains statements given in two columns which have to be matched.
Statements (A, B, C, D) in Column I have to be matched with statements (p, q, r, s) in Column II. The answers to these questions have to
be appropriately bubbled as illustrated in the following example. If the correct matches are A-p, A-s, B-q, B-r, C-p, C-q and D-s, then the
correctly bubbled 4 × 4 matrix should be as follows :

p q r s
A p q r s
B p q r s
C p q r s
D p q r s
EBD_780
2007- 10 Target IIT-JEE
64. In the following [x] denotes the greatest integer less than or equal to x.
Match the functions in Column I with the properties in Column II and indicate your answer by darkening the appropriate
bubbles in the 4 × 4 matrix given in the ORS.
Column I Column II
(A) x | x | (p) continuous in (–1, 1)

(B) |x| (q) differentiable in (–1, 1)

(C) x + [x] (r) strictly increasing in (–1, 1)


(D) | x – 1 | + | x + 1 | (s) not differentiable at least at one point in (–1, 1)
65. Consider the following linear equations
ax + by + cz = 0
bx + cy + az = 0
cx + ay + bz = 0
Match the conditions/expressions in Column I with statements in Column II and indicate your answer by darkening the
appropriate bubbles in the 4 × 4 matrix given in the ORS.
Column I Column II
(A) a + b + c 0 and a2 + b2 + c2 = ab + bc + ca (p) the equations represent planes meeting only at asingle point
(B) a + b + c = 0 and a2 + b2 + c2 ab + bc + ca (q) the equations represent the line x = y = z.
(C) a + b + c 0 and a2 + b2 + c2 ab + bc + ca (r) the equations represent identical planes.
(D) a + b + c = 0 and a2 + b2 + c2 = ab + bc + ca (s) the equations represent the whole of the threedimensional
space.
66. Match the integrals in Column I with the values in Column II and indicate your answer by darkening the appropriate bubbles in
the 4 × 4 matrix given in the ORS.
Column I Column II

1
dx 1 2
(A) 2
(p) log
1 x 2 3
1

1
dx 2
(B) (q) 2log
2 3
0 1 x

3
dx
(C) (r)
2
1 x2 3

2
dx
(D) (s)
x x2 1 2
1
2007 IIT-JEE Solved Paper 2007-11

PAPER - II
PHYSICS
____________________________________________________________________________________________________
General Instructions
1. Section I : Q. No. 1 to 9 have only one correct answer and carries 3 marks each for correct answer and –1 mark for each wrong
answer.
2. Section II : Q. No. 10 to 13 are Statement-1(Assertion) and Statement-2(Reason) type questions and carries 3 marks each for
correct answer and –1 mark for each wrong answer.
3. Section III : Q. No. 14 to 19 are based on the given paragraph which have only one correct answer and carries 4 marks each
for correct answer and –1 marks for each wrong answer.
4. Section IV : Q. No. 20 to 22 are matching type questions each question has four parts and each question carries 6 marks and
marks will be awarded if all the four parts are correctly matched. No negative mark will be awarded for an incorrectly bubbled
answer.
––––––––––––––––––––––––––––––––––––––––––––––––––––––––––––––––––––––––––––––––––––––––––––––––––––
3. A student performs an experiment to determine the Young's
Section - I (Straight Objective Type)
modulus of a wire, exactly 2 m long, by Searle's method. In a
INSTRUCTION : This section contains 9 multiple choice particular reading, the student measures the extension in
questions numbered 1 to 9. Each question has 4 choices (a), (b), the length of the wire to be 0.8 mm with an uncertainty
(c) and (d), out of which ONLY ONE is correct. of 0.05 mm at a load of exactly 1.0 kg. The student also
measures the diameter of the wire to be 0.4 mm with an
1. A small object of uniform density rolls up a curved surface uncertainty of 0.01 mm. Take g = 9.8 m/s2 (exact). The
Young's modulus obtained from the reading is
with an initial velocity v. It reaches up to a maximum height of
(a) (2.0 0.3) × 1011 N/m2 (b) (2.0 0.2) × 1011 N/m2
3v 2 (c) (2.0 0.1) × 1011 N/m2 (d) (2.0 0.05) × 1011 N/m2
with respect to the initial position. The object is
4g
4. A magnetic field B B0 Jˆ , exists in the region a < x < 2a,

and B = – B0 ˆj , in the region 2a < x < 3a, where B0 is a


positive constant. A positive point charge moving with a
velocity v v0iˆ , where v0 is a positive constant, enters the
v magnetic field at x = a. The trajectory of the charge in this
region can be like
(a) ring (b) solid sphere
(c) hollow sphere (d) disc B0
2. A spherical portion has been removed from a solid sphere
having a charge distributed uniformly in its volume as shown 0 x
in the figure. The electric field inside the emptied space is a 2a 3a

B0

(a) x
a 2a 3a
(a) zero everywhere (b) non-zero and uniform
(c) non-uniform (d) zero only at its center
EBD_780
2007- 12 Target IIT-JEE
8. Positive and negative point charges of equal magnitude are
z
a a
kept at 0, 0, and 0, 0, respectively. The work
2 2
(b) x done by the electric field when another positive point charge
a 2a 3a
is moved from (–a, 0, 0) to (0, a, 0) is
(a) positive
(b) negative
(c) zero
z (d) depends on the path connecting the initial and final
positions
9. Water is filled up to a height h in a beaker of radius R as
(c) x shown in the figure. The density of water is , the surface
a 2a 3a
tension of water is T and the atmospheric pressure is P0.
Consider a vertical section ABCD of the water column
through a diameter of the beaker. The force on water on one
side of this section by water on the other side of this section
has magnitude
z
2R
a 2a 3a
(d) x B

A
h
C
5. In the experiment to determine the speed of sound using a
resonance column,
D
(a) prongs of the tuning fork are kept in a vertical plane
(a) | 2P0Rh + R2 gh – 2RT |
(b) prongs of the tuning fork are kept in a horizontal plane
(b) | 2P0Rh + R gh2 – 2RT |
(c) in one of the two resonances observed, the length of (c) | P0 R2 + R gh2 – 2RT |
the resonating air column is close to the wavelength of (d) | P0 R2 + R gh2 + 2RT |
sound in air
(d) in one of the two resonances observed, the length of
the resonating air column is close to half of the Section - II (Assertion-Reason Type)
wavelength of sound in air INSTRUCTION : This section contains 4 questions numbered 10
to 13. Each question contains STATEMENT-1 (Assertion) and
6. Electrons with de-Broglie wavelength fall on the target in
STATEMENT-2 (Reason). Each question has 4 choices (a), (b), (c)
an X-ray tube. The cut-off wavelength of the emitted X-
and (d) out of which ONLY ONE is correct.
rays is
10. STATEMENT-1
2 The total translational kinetic energy of all the molecules of
2mc 2h
(a) 0 (b) 0 a given mass of an ideal gas is 1.5 times the product of its
h mc
pressure and its volume.
because
2m 2 c 2 3
(c) 0 (d) 0 STATEMENT-2
2
h
The molecules of a gas collide with each other and the
7. A particle moves in the X-Y plane under the influence of a velocities of the molecules change due to the collision.
force such that its linear momentum is (a) Statement-1 is True, Statement-2 is True; Statement-2
is a correct explanation for Statement-1
p(t ) A [iˆ cos(kt ) ˆj sin(kt )], where A and k are constants.
(b) Statement-1 is True, Statement-2 is True; Statement-2
The angle between the force and the momentum is is NOT a correct explanation for Statement-1
(a) 0° (b) 30° (c) Statement-1 is True, Statement-2 is False
(c) 45° (d) 90° (d) Statement-2 is False, Statement-2 is True
2007 IIT-JEE Solved Paper 2007-13
11. STATEMENT-1
Section - III (Linked Comprehension Type)
A vertical iron rod has coil of wire wound over it at the
INSTRUCTION : This section contains 2 paragraphs P14-16 and
bottom end. An alternating current flows in the coil. The rod P17-19. Based upon each paragraph, 3 multiple choice questions
goes through a conducting ring as shown in the figure. The have to be answered. Each question has 4 choices (a), (b), (c) and
ring can float at a certain height above the coil. (d), out of which ONLY ONE is correct.

P14-16 : Paragraph for Question Nos. 14 to 16


Two trains A and B moving with speeds 20 m/s and 30 m/s
respectively in the same direction on the same straight track,
with B ahead of A. The engines are at the front ends. The
engine of train A blows a long whistle.
Assume that the sound of the whistle is composed of
components varying in frequency from f1 = 800 Hz to f2 =
1120 Hz, as shown in the figure. The spread in the frequency
(highest frequency – lowest frequency) is thus 320 Hz. The
speed of sound in still air is 340 m/s.
14. The speed of sound of the whistle is
STATEMENT-2
(a) 340 m/s for passengers in A and 310 m/s for passengers
In the above situation, a current is induced in the ring which
in B
interacts with the horizontal component of the magnetic
field to produce an average force in the upward direction. (b) 360 m/s for passengers in A and 310 m/s for passengers
in B
(a) Statement-1 is True, Statement-2 is True; Statement-2
is a correct explanation for Statement-1 (c) 310 m/s for passengers in A and 360 m/s for passengers
in B
(b) Statement-1 is True, Statement-2 is True; Statement-2
is NOT a correct explanation for Statement-1 (d) 340 m/s for passengers in both the trains
(c) Statement-1 is True, Statement-2 is False 15. The distribution of the sound intensity of the whistle as
(d) Statement-1 is False, Statement-2 is True. observed by the passengers in train A is best represented
by
12. STATEMENT-1
A cloth covers a table. Some dishes are kept on it. The cloth
Intensity

can be pulled out without dislodging the dishes from the


table. (a)
because
f1 f 2 Frequency
STATEMENT-2
For every action there is an equal and opposite reaction.
(a) Statement-1 is True, Statement-2 is True; Statement-2
Intensity

is a correct explanation for Statement-1


(b)
(b) Statement-1 is True, Statement-2 is True; Statement-2
is NOT a correct explanation for Statement-1
f1 f 2 Frequency
(c) Statement-1 is True, Statement-2 is False
(d) Statement-1 is False, Statement-2 is True.
13. STATEMENT-1
Intensity

If there is no external torque on a body about its center of


(c)
mass, then the velocity of the center of mass remains constant.
STATEMENT-2
f1 f 2 Frequency
The linear momentum of an isolated system remains constant.
(a) Statement-1 is True, Statement-2 is True; Statement-2
is a correct explanation for Statement-1
Intensity

(b) Statement-1 is True, Statement-2 is True; Statement-2


is NOT a correct explanation for Statement-1 (d)
(c) Statement-1 is True, Statement-2 is False
(d) Statement-1 is False, Statement-2 is True f1 f 2 Frequency
EBD_780
2007- 14 Target IIT-JEE
16. The spread of frequency as observed by the passengers in 17. Light travels as a
train B is (a) parallel beam in each medium
(a) 310 Hz (b) 330 Hz (b) convergent beam in each medium
(c) 350 Hz (d) 290 Hz (c) divergent beam in each medium
P17-19 : Paragraph for Question Nos. 17 to 19 (d) divergent beam in one medium and convergent beam
The figure shows a surface XY separating two transparent in the other medium.
media, medium-1 and medium-2. The line ab and cd represent 18. The phases of the light wave at c, d, e and f are c, d, e and
waveforms of a light wave travelling in medium-1 and
f respectively. It is given that c f.
incident on XY. The lines ef and gh represent wavefronts of
(a) c cannot be equal to d
the light wave in medium-2 after refraction.
d (b) d can be equal to e
b
(c) ( d – f) is equal to ( c – e)
(d) ( d – c) is not equal to ( f – e)
19. Speed of light is
medium-1
a c (a) the same in medium-1 and medium-2
X Y (b) larger in medium-1 than in medium-2
f h
medium-2 (c) larger in medium-2 than in medium-1
(d) different at b and d.
e g

Section - IV (Matrix Match Type)


INSTRUCTION : This section contains 3 questions. Each question contains statements given in two columns which have to be
matched. Statements (A, B, C, D) in Column I have to be matched with statement (p, q, r, s) in Column II. The answers to these questions
have to be appropriately bubbled as illustrated in the following example. If the correct matches are A-p, A-s, B-q, B-r C-p, C-q and
D-s, then the correctly bubbled 4 × 4 matrix should be as follows:
p q r s
A p q r s
B p q r s
C p q r s
D p q r s
20. Column I describes some situations in which a small object moves. Column II describes some characteristics of these motions.
Match the situations in Column I with the characteristics in Column II and indicate your answer by darkening appropriate
bubbles in 4 × 4 matrix given in the ORS.
Column I Column II
(A) The object moves on the x -axis under a (p) The object executes a simple harmonic motion.
conservative force in such a way that its
''speed " and position'' satisfy v c c2 x 2
1
where c1 and c2 are positive constants.
(B) The object moves on the x- axis in such a way that (q) The object does not change its direction.
its velocity and its displacement from the origin
satisfy v = – kx,where k is a positive constant.
(C) The object is attached to one end of a mass-less (r) The kinetic energy of the object keeps on
spring of a given spring constant. The other decreasing.
end of the spring is attached to the ceiling of an
elevator. Initially everything is at rest. The elevator
starts going upwards with a constant acceleration a.
The motion of the object is observed from the elevator
during the period it maintains this acceleration.
(D) The object is projected from the earth's surface (s) The object can change its direction only once.
vertically upwards with a speed 2 GM e / R e ,
where, Me is the mass of the earth and Re is the
radius of the earth, Neglect forces from objects other
than the earth.
2007 IIT-JEE Solved Paper 15
2007-

21. Two wires each carrying a steady current I are shown in four configurations in Column I. Some of the resulting effects are
described in Column II. Match the statements in Column I with the statements in column II and indicate your answer by
darkening appropriate bubbles in the 4 × 4 matrix given in the ORS.
Column I Column II
(A) Point P is situated midway between (p) The magnetic fields (B) at P due to the currents
the wires. in the wires are in the same direction.

P
(B) Point P is situated at the mid-point of the (q) The magnetic fields (B) at P due to the currents
line joining the centers of the circular in the wires are in opposite directions.
wires, which have same radii.

(C) Point P is situated at the mid-point of the (r) There is no magnetic field at P.
line joining the centers of the circular
wires, which have same radii.

(D) Point P is situated at the common center (s) The wires repel each other.
of the wires.

22. Column I gives some devices and Column II gives some processes on which the functioning of these devices depend. Match
the devices in Column I with the processes in Column II and indicate your answer by darkening appropriate bubbles in the 4 ×
4 matrix given in the ORS.
Column I Column II
(A) Bimetallic strip (p) Radiation from a hot body
(B) Steam engine (q) Energy conversion
(C) Incandescent lamp (r) Melting
(D) Electric fuse (s) Thermal expansion of solids

CHEMISTRY
––––––––––––––––––––––––––––––––––––––––––––––––––––––––––––––––––––––––––––––––––––––––––––––––––––
General Instructions
1. Section I : Q. No. 23 to 31 have only one correct answer and carries 3 marks each for correct answer and –1 mark for each
wrong answer.
2. Section II : Q. No. 32 to 35 are Statement-1(Assertion) and Statement-2(Reason) type questions and carries 3 marks each for
correct answer and –1 mark for each wrong answer.
3. Section III : Q. No. 36 to 41 are based on the given two paragraphs which have only one correct answer and carries 4 marks
each for correct answer and –1 marks for each wrong answer.
4. Section IV : Q. No. 42 to 44 are matching type questions each question has four parts and each question carries 6 marks and
marks will be awarded if all the four parts are correctly matched. No negative mark will be awarded for an incorrectly bubbled
answer.
––––––––––––––––––––––––––––––––––––––––––––––––––––––––––––––––––––––––––––––––––––––––––––––––––––
EBD_780
2007- 16 Target IIT-JEE
27. Consider a titration of potassium dichromate solution with
Section - I (Straight Objective Type)
acidified Mohr's salt solution using diphenylamine as
INSTRUCTION : This section contains 9 multiple choice indicator. The number of moles of Mohr's salt required per
questions numbered 23 to 31. Each question has 4 choices (a), (b), mole of dichromate is
(c) and (d), out of which ONLY ONE is correct.
(a) 3 (b) 4
23. Cyclohexene on ozonolysis followed by reaction with zinc (c) 5 (d) 6
dust and water gives compound E. Compound E on further 28. Consider a reaction aG + bH Products. When
treatment with aqueous KOH yields compound F. concentration of both the reactants G and H is doubled, the
Compound F is rate increases by eight times. However, when concentration
of G is doubled keeping the concentration of H fixed, the
rate is doubled. The overall order of the reaction is
(a) CHO (b) CHO
(a) 0 (b) 1
(c) 2 (d) 3
CO 2 H 29. A solution of a metal ion when treated with KI gives a red
(c) COOH (d) precipitate which dissolves in excess KI to give a colourless
CO 2 H
solution. Moreover, the solution of metal ion on treatment
24. Among the following, the least stable resonance structure with a solution of cobalt (II) thiocyanate gives rise to a deep
blue crystalline precipitate. The metal ion is
is
(a) Pb2+ (b) Hg2+
(c) Cu2+ (d) Co2+
O
(a) 30. A positron is emitted from 23
. The ratio of the atomic
N 11 Na
mass and atomic number of the resulting nuclide is
O
(a) 22/10 (b) 22/11
(c) 23/10 (d) 23/12
31. The number of stereoisomers obtained by bromination of
O trans-2-butene is
(b) N (a) 1 (b) 2
(c) 3 (d) 4
O
Section - II (Assertion-Reason Type)
INSTRUCTION : This section contains 4 questions numbered
32 to 35. Each question contains STATEMENT-1 (Assertion) and
O
(c) STATEMENT-2 (Reason). Each question has 4 choices (a), (b), (c)
N and (d) out of which ONLY ONE is correct.
O
32. STATEMENT-1 : Band gap in germanium is small.
because
STATEMENT-2 : The energy gap of each germanium atomic
energy level is infinitesimally small.
O (a) Statement-1 is True, Statement-2 is True; Statement-2
(d) is a correct explanation for Statement-1
N
(b) Statement-1 is True, Statement-2 is True; Statement-2
O is not a correct explanation for Statement-1
(c) Statement-1 is True, Statement-2 is False
(d) Statement-1 is False, Statement-2 is True.
25. Among the following metal carbonyls, the C–O bond order 33. Statement -1: Molecules that are not superimpossable on
is lowest in their mirror images are chiral.
(a) [Mn(CO)6]+ (b) [Fe(CO)5] Statement -2: All chiral molecules have chiral centres.
(c) [Cr(CO)6] (d) [V(CO)6]– (a) Statement-1 is True, Statement-2 is True; Statement-2
26. For the process H2O(l) (1 bar, 373 K) H2O(g) (1 bar, 373 is a correct explanation for Statement-1
K), the correct set of thermodynamic parameters is (b) Statement-1 is True, Statement-2 is True; Statement-2
is not a correct explanation for Statement-1
(a) G = 0, S = +ve (b) G = 0, S = –ve (c) Statement-1 is True, Statement-2 is False
(c) G = +ve, S = 0 (d) G = –ve, S = +ve (d) Statement-1 is False, Statement-2 is True.
2007 IIT-JEE Solved Paper 17
2007-

34. STATEMENT-1 : Glucose gives a reddish-brown precipitate 38. Sodium fusion extract, obtained from aniline, on treatment
with Fehling's solution. with iron (II) sulphate and H2SO4 in presence of air gives a
because Prussian blue precipitate. The blue colour is due to the
STATEMENT-2 : Reaction of glucose with Fehling's solution formation of
give CuO and gluconic acid.
(a) Statement-1 is True, Statement-2 is True; Statement-2 (a) Fe4[Fe(CN)6]3 (b) Fe3[Fe(CN)6]2
is a correct explanation for Statement-1 (c) Fe4[Fe(CN)6]2 (d) Fe3[Fe(CN)6]3
(b) Statement-1 is True, Statement-2 is True; Statement-2 C39-41 : Paragraph for Question Nos. 39 to 41
is not a correct explanation for Statement-1
Riemer-Tiemann reaction introduces an aldehyde group, on
(c) Statement-1 is True, Statement-2 is False
to the aromatic ring of phenol, ortho to the hydroxyl group.
(d) Statement-1 is False, Statement-2 is True.
This reaction involves electrophilic aromatic substitution.
35. STATEMENT-1 : Alkali metals dissolve in liquid ammonia
to give blue solutions. because This is a general method for the synthesis of substituted
STATEMENT-2 : Alkali metals is liquid ammonia give salicylaldehyde as depicted below.
solvated species of the type [M(NH3)n]+ (M = alkali metals).
(a) Statement-1 is True, Statement-2 is True; Statement-2
is a correct explanation for Statement-1 OH O Na OH
(b) Statement-1 is True, Statement-2 is True; Statement-2 CHO CHO
is a correct explanation for Statement-1 aq. HCl
[I]
(c) Statement-1 is True, Statement-2 is False (Intermediate)
(d) Statement-1 is False, Statement-2 is True.
CH3 CH 3 CH 3
(I) (II) (III)
Section - III (Linked Comprehension Type)
INSTRUCTION : This section contains 2 paragraphs C36-38 and
C39-41. Based upon each paragraph, 3 multiple choice questions 39. Which one of the following reagents is used in the above
have to be answered. Each question has 4 choices (a), (b), (c) and reaction ?
(d), out of which ONLY ONE is correct. (a) aq.NaOH + CH3Cl (b) aq.NaOH + CH2Cl2
C36-38 : Paragraph for Question Nos. 36 to 38 (c) aq.NaOH + CHCl3 (d) aq.NaOH + CCl4
Redox reactions play a pivotal role in chemistry and biology.
40. The electrophile in the reaction is
The values of standard redox potential (E°) of two half-cell
(a) : CHCl (b) +CHCl
reactions decide which way the reaction is expected to 2
proceed. A simple example is a Daniel cell in which zinc goes (c) : CCl2 (d) CCl3
into solution and copper gets deposited. Given below are a
set of half-cell reactions (acidic medium) along with their E° 41. The structure of the intermediate I is
(V with respect to normal hydrogen electrode) values. Using
this data obtain the correct explanations to questions given.
I2 + 2e– 2I– E° = 0.54 O Na O Na
Cl2 + 2e– 2Cl– E° = 1.36 CH 2 Cl CHCl 2
Mn3+ + e– Mn2+ E° = 1.50 (a) (b)
Fe3+ + e– Fe2+ E° = 0.77
O2 + 4H+ + 4e– 2H2O E° = 1.23
36. Among the following, identify the correct statement. CH 3 CH 3
(a) Chloride ion is oxidised by O2
(b) Fe2+ is oxidised by iodine
(c) Iodide ion is oxidised by chlorine
(d) Mn2+ is oxidised by chlorine O Na O Na
37. While Fe3+ is stable, Mn 3+ is not stable in acid solution CCl 3 CH 2 OH
because
(c) (d)
(a) O2 oxideses Mn2+ to Mn3+
(b) O2 oxideses both Mn2+ to Mn3+ and Fe2+ to Fe3+
(c) Fe3+ oxideses H2O to O2 CH 3 CH3
(d) Mn3+ oxideses H2O to O2
EBD_780
2007- 18 Target IIT-JEE

Section - IV (Matrix Match Type)


INSTRUCTION : This section contains 3 questions. Each question contains statements given in two columns which have to be
matched. Statements (A, B, C, D) in Column I have to be matched with statements (p, q, r, s) in Column II. The answers to these
questions have to be appropriately bubbled as illustrated in the following example. If the correct matches are A-p, A-s, B-q, B-r, C-p,
C-q and D-s, then the correctly bubbled 4 × 4 matrix should be as follows :

p q r s
A p q r s
B p q r s
C p q r s
D p q r s

42. Match the crystal system/unit cells mentioned in Column I with their characteristic features mentioned in Column II. Indicate
your answer by darkening the appropriate bubbles of the 4 × 4 matrix given in the ORS.

Column I Column II
(A) Simple cubic and face-centered cubic parameters (p) have these parameters, a = b = c and
(B) cubic and rhombohedral (q) are two crystal systems
(C) cubic and tetragonal (r) have only two crystallo-graphic angles of 90°
(D) hexagonal and monoclinic (s) belong to same crystal system
43. Match the reactions in Columns I with nature of the reactions/type of the products in Column II. Indicate your answer by
darkening the appropriate bubbles of the 4 × 4 matrix given in the ORS.
Column I Column II
(A) O 2 O2 O 22 (p) redox reaction

(B) CrO24 H (q) one of the products has trigonal planar structure
(C) MnO4 NO2 + H+ (r) dimeric bridged tetrahedral metal ion
(D) NO3 H 2SO 4 + Fe2+ (s) disproportionation
44. Match the compounds/ions in column I with their properties/reactions in Column II. Indicate your answer by darkening the
appropriate bubbles of the 4 × 4 matrix given in the ORS.
Column I Column II
(A) C6H5CHO (p) gives precipitate with 2, 4-dinitrophenylhydrazine
(B) CH3C CH (q) gives precipitate with AgNO3
(C) CN– (r) is a nucleophile
(D) I– (s) is involved in cyanohydrin formation

MATHEMATICS
––––––––––––––––––––––––––––––––––––––––––––––––––––––––––––––––––––––––––––––––––––––––––––––––––––
General Instructions
1. Section I : Q. No. 45 to 53 have only one correct answer and carries 3 marks each for correct answer and –1 mark for each
wrong answer.
2. Section II : Q. No. 54 to 57 are Statement-1(Assertion) and Statement-2(Reason) type questions and carries 3 marks each for
correct answer and –1 mark for each wrong answer.
3. Section III : Q. No. 58 to 63 are based on the given paragraph which have only one correct answer and carries 4 marks each
for correct answer and –1 marks for each wrong answer.
4. Section IV : Q. No. 64 to 66 are matching type questions each question has four parts and each question carries 6 marks and
marks will be awarded if all the four parts are correctly matched. No negative mark will be awarded for an incorrectly bubbled
answer.
–––––––––––––––––––––––––––––––––––––––––––––––––––––––––––––––––––––––––––––––––––––––––––––––––––
2007 IIT-JEE Solved Paper 2007-19
50. The letters of the word COCHIN are permuted and all the
Section - I (Straight Objective Type)
permutations are arranged in an alphabetical order as in an
INSTRUCTION : This section contains 9 multiple choice
English dictionary. The number of words that appear before
questions numbered 45 to 53. Each question has 4 choices (a), (b),
(c) and (d), out of which ONLY ONE is correct. the word COCHIN is
(a) 360 (b) 192
d2x (c) 96 (d) 48
45. equals
dy 2 51. Let Ec denote the complement of an event E. Let E, F, G be
1 1
pairwise independent events with P(G) > 0 and
3
d2y d2y dy P(E F G) = 0. Then P(Ec Fc | G) equals
(a) 2
(b)
dx 2 dx
dx (a) P(Ec) + P(Fc) (b) P(Ec) – P(Fc)
2 3 (c) P(Ec) – P(F) (d) P(E) – P(Fc)
d2y dy d2y dy
(c) (d) 52. Let ABCD be a quadrilateral with area 18, with side AB parallel
dx 2 dx dx 2 dx
to the side CD and 2AB = CD. Let AD be perpendicular to
AB and CD. If a circle is drawn inside the quadrilateral ABCD
46. Let a , b , c be unit vectors such that a b c 0 . Which
touching all the sides, then its radius is
one of the following is correct ?
(a) 3 (b) 2
(a) a b b c c a 0
3
(c) (d) 1
(b) a b b c c a 0 2

(c) a b b c a c 0 dy 1 y2
53. The differential equation determines a family
dx y
(d) a b, b c, c a are muturally perpendicular
of circles with
z
47. If | z | = 1 and z 1, then all the values of lie on (a) variable radii and a fixed centre at (0, 1)
1 z2
(a) a line not passing through the origin (b) variable radii and a fixed centre at (0, –1)

(b) | z | = 2 (c) fixed radius 1 and variable centres along the x–axis.
(c) the x-axis (d) the y-axis (d) fixed radius 1 and variable centres along the y–axis.
x
48. Let f ( x) for n 2 and
(1 x n )1/ n Section - II (Assertion-Reason Type)
INSTRUCTION : This section contains 4 questions numbered 54
g(x) = ( fofo ...of ) ( x ). Then xn 2
g ( x )dx equals. to 57. Each question contains STATEMENT-1 (Assertion) and
f occurs n times STATEMENT-2 (Reason). Each question has 4 choices (a), (b), (c)
1 1
and (d) out of which ONLY ONE is correct.
1 1 1 1
(a) (1 nx n ) n K (b) (1 nx n ) n K 54. Lines L1 : y – x = 0 and L2 : 2x + y = 0 intersect the line L3 : y
n( n 1) n 1
+ 2 = 0 at P and Q, respectively. The bisector of the acute
1 1
1 angle between L1 and L2 intersects L3 at R.
1 1 1
(c) (1 nxn ) n K (d) (1 nx n ) n K
n(n 1) n 1 STATEMENT-1 : The ratio PR : RQ equals 2 2 : 5 .
49. Let O(0, 0), P(3, 4), Q(6, 0) be the vertices of the triangles because
OPQ. The point R inside the triangle OPQ is such that the STATEMENT-2 : In any triangle, bisector of an angle divides
triangles OPR, PQR, OQR are of equal area. The coordinates the triangle into two similar triangles.
of R are
(a) Statement-1 is True, Statement-2 is True; Statement-2
4 2 is not a correct explanation for Statement-1
(a) ,3 (b) 3,
3 3 (b) Statement-1 is True, Statement-2 is True; Statement-2
is NOT a correct explanation for Statement-1
4 4 2
(c) 3, (d) , (c) Statement-1 is True, Statement-2 is False
3 3 3
(d) Statement-1 is False, Statement-2 is True.
EBD_780
2007- 20 Target IIT-JEE
55. Let f (x) = 2 + cos x for all real x. 58. Which one of the following statements is correct ?
Statement - 1 : For each real t, there exists a point c in (a) G1 > G2 > G3 > ...
[t, t + ] such that f '(c) = 0 because
(b) G1 < G2 < G3 < ...
Statement - 2 : f(t) = f(t + 2 ) for each real t.
(c) G1 = G2 = G3 = ...
(a) Statement-1 is True, Statement-2 is True; Statement-2
is a correct explanation for Statement-1 (d) G1 < G3 < G5 < ... and G2 > G4 > G6 > ....
(b) Statement-1 is True, Statement-2 is True; Statement-2 59. Which one of the following statements is correct ?
is NOT a correct explanation for Statement-1
(a) A1 > A2 > A3 > ...
(c) Statement-1 is True, Statement-2 is False
(b) A1 < A2 < A3 < ...
(d) Statement-1 is False, Statement-2 is True.
(c) A1 > A3 > A5 > ... and A2 < A4 < A6 < ...
x2
56. STATEMENT-1 : The curve y x 1 is symmetric (d) A1 < A3 < A5 < ... and A2 > A4 > A6 > ...
2
with respect to the line x = 1. because 60. Which one of the following statements is correct ?

STATEMENT-2 : A parabola is symmetric about its axis. (a) H1 > H2 > H3 > ...
(a) Statement-1 is True, Statement-2 is True; Statement-2 (b) H1 < H2 < H3 < ...
is a correct explanation for Statement-1 (c) H1 > H3 > H5 > ... and H2 < H4 < H6 < ...
(b) Statement-1 is True, Statement-2 is True; Statement-2
(d) H1 < H3 < H6 < ... and H2 > H4 > H6 > ...
is NOT a correct explanation for Statement-1
(c) Statement-1 is True, Statement-2 is False
(d) Statement-1 is False, Statement-2 is True. M61-63 : Paragraph for Question Nos. 61 to 63
57. Consider the planes 3x – 6y – 2z = 15 and 2x + y – 2z = 5. If a continuous function f defined on the real line R, assumes
STATEMENT-1 : The parametric equations of the line of positive and negative values in R then the equation f(x) = 0
intersection of the given planes are x = 3 + 14t, y = 1 + 2t, z = has a root in R. For example, if it is known that a continuous
15t. because function f on R is positive at some point and its minimum
value is negative then the equation f(x) = 0 has a root in R.
STATEMENT-2 : The vector 14iˆ 2 ˆj 15kˆ is parallel to
the line of intersection of given planes. Consider f(x) = kex – x for all real x where k is a real constant.

(a) Statement-1 is True, Statement-2 is True; Statement-2 61. The line y = x meets y = kex for k 0 at
is a correct explanation for Statement-1
(a) no point (b) one point
(b) Statement-1 is True, Statement-2 is True; Statement-2
(c) two points (d) more than two points
is NOT a correct explanation for Statement-1
(c) Statement-1 is True, Statement-2 is False 62. The positive value of k for which kex – x = 0 has only one
root is
(d) Statement-1 is False, Statement-2 is True.

1
(a) (b) 1
e
Section - III (Linked Comprehension Type)
INSTRUCTION : This section contains 2 paragraphs M58-60 and (c) e (d) loge2
M61-63. Based upon each paragraph, 3 multiple choice questions 63. For k > 0, the set of all values of k for which kex – x = 0 has
have to be answered. Each question has 4 choices (a), (b), (c) and
two distinct roots is
(d), out of which ONLY ONE is correct.
M58-60 : Paragraph for Question Nos. 58 to 60 1 1
(a) 0, (b) ,1
Let A1, G1, H1 denote the arithmetic, geometric and harmonic e e
means, respectively, of two distinct positive numbers. For n
2, Let An – 1 and Hn – 1 have arithmetic, geometric and 1
(c) , (d) (0, 1)
harmonic means as An, Gn, Hn respectively. e
2007 IIT-JEE Solved Paper 21
2007-

Section - IV (Matrix Match Type)


INSTRUCTION : This section contains 3 questions. Each question contains statements given in two columns which have to be
matched. Statements (A, B, C, D) in Column I have to be matched with statements (p, q, r, s) in Column II. The answers to these
questions have to be appropriately bubbled as illustrated in the following example. If the correct matches are A-p, A-s, B-q, B-r, C-p,
C-q, and D-s, then the correctly bubbled 4 × 4 matrix should be as follows :

p q r s
A p q r s
B p q r s
C p q r s
D p q r s

64. Match the statements in Column I with the properties in Column II and indicate your answer by darkening the appropriate
bubbles in the 4 × 4 matrix given in the ORS.
Column I Column II
(A) Two intersecting circles (p) have a common tangent
(B) Two mutually external circles (q) have a common normal
(C) Two circles, one strictly inside the other (r) do not have a common tangent
(D) Two branches of a hyperbola (s) do not have a common normal

x2 6x 5
65. Let f ( x)
2
x 5x 6

Match of expressions/statements in Column I with expressions/statements in Column II and indicate your answer by darkening
the appropriate bubbles in the 4 × 4 matrix given in the ORS.
Column I Column II
(A) If –1 < x < 1, then f(x) satisfies (p) 0 < f(x) < 1
(B) If 1 < x < 2, then f(x) satisfies (q) f(x) < 0
(C) If 3 < x < 5, then f(x) satisfies (r) f(x) > 0
(D) If x > 5, then f(x) satisfies (s) f(x) < 1

66. Let (x, y) be such that sin 1 (ax) cos 1 ( y ) cos 1 (bxy ) .
2

Match the statements in Column I with statements in Column II and indicate your answer by darkening the appropriate bubbles
in the 4 × 4 matrix given in the ORS.
Column I Column II
(A) If a = 1 and b = 0, then (x, y) (p) lies on the circle x2 + y2 = 1
(B) If a = 1 and b = 1, they (x, y) (q) lies on (x2 – 1)(y2 – 1) = 0
(C) If a = 1 and b = 2, then (x, y) (r) lies on y = x
(D) If a = 2 and b = 2, then (x, y) (s) lies on (4x2 – 1) (y2 – 1) = 0
EBD_780
2007- 22 Target IIT-JEE

EXPLANATORY NOTES – IIT-JEE 2007

PAPER - I
1. (b) As shown in the figure, when the object (O) is placed or, 80 X = (X + 2) + 40
between F and C, the image (I) is formed beyond C. It
is in this condition that when the student shifts his 200
eyes towards left, the image appears to the right of the or, 80 X = (X 2) 40
X 2
object pin.
240
or, X= =3 .
Movement towards left 80
3. (a) When a charge density is given to the inner cylinder,
F the potential developed at its surface is different from
I C O that on the outer cylinder. This is because the potential
decreases with distance for a charged conducting
cylinder when the point of consideration is outside the
cylinder.
But when a charge density is given to the outer cylinder,
2. (a) Given X is greater than 2 when the bridge is balanced it will change its potential by the same amount as that
of the inner cylinder. Therefore no potential difference
R X
will be produced between the cylinders in this case.
100
4. (b) The acceleration of mass m is due to the force T cos
T cos = ma
R=2 X
T cos
a= ... (i)
m

100- also, F = 2T sin


G F
T= ... (ii)
2 sin
B Switch
F
or, 100 R – R = X
or, 200 – 2 = X T T
a
200 Tsin Tsin a
or, = T
X 2
Tcos
When the resistances are interchanged the jockey shifts
x x
20 cm. Therefore
From (i) and (ii)
X 2
20 80 F cos F F x
a=
2sin m 2m tan 2m a 2
x2
80 X – X = 2 + 40

X R= 2 a2 x2
tan
x

5. (c) When steady state is reached, the current I coming


l+20 80 l
from the battery is
G
9 = I (3 + 6) I = 1A
2007 IIT-JEE Solved Paper 2007-23
9. (b) The smallest frequency and largest wavelength in
3 F 6 F ultraviolet region will be for transition of electron from
+Q1 –Q1 +Q2 –Q2
orbit 2 to orbit 1.

3 6 1 1 1
R –
n12 n22

9V 1 1 1 1 3R
R – R 1–
–9 2 2 4 4
122 10 m 1 2
Potential difference across 3 resistance = 3V and
potential difference across 6 resistance = 6V 4
R m –1
p.d. across 3 µF capacitor = 3V 3 122 10–9
and p.d. across 6 µF capacitor = 6V The highest frequency and smallest wavelength for
Charge on 3 µF capacitor Q1 = 3 × 3 = 9 µC infrared region will be for transition of electron from
Charge on 6 µF capacitor Q2 = 6 × 6 = 36 µC to 3rd orbit.

Charge (– Q1) is shifted from the positive plate of


1 1 1
6 µF capacitor. The remaining charge on the positive R –
n12 n22
plate of 6 µF capacitor is shifted through the switch.
Charge passing the switch = 36 – 9 = 27 µC
1 4 1 1
6. (d) When a positive point charge is placed outside a –
conducting sphere, a rearrangement of charge takes 3 122 10 –9 32
place on the surface. But the total charge on the sphere
is zero as no charge has left or entered the sphere. 3 122 9 10 –9
= 823.5nm
net charge = 0 4

+
10. (b) Statement 1 : For an elastic collision, the coefficient of
+ restitution = 1
+
+q +
+
| v2 v1 |
e= | v2 – v1| = | u1 – u2 |
7. (a) Iodine and Yttrium are medium sized nuclei and | u1 u2 |
therefore, have more binding energy per nucleon as Relative velocity after collision is equal to relative
compared to Uranium which has a big nuclei and less velocity before collision. Therefore the statement is
B.E./nucleon. true.
In other words, Iodine and Yttrium are more stable and
Statement 2 : Linear momentum remains conserved in
therefore possess less energy and less rest mass. Also
when Uranium nuclei explodes, it will convert into I an elastic collision. This statement is true. But statement
and Y nuclei having kinetic energies. 2 alone does not lead us to the conclusion of statement
8. (c) The ray is partly reflected and partly refracted 1. Therefore statement 2 is NOT a correct explanation
MOB = 180 – 2 of statement 1.
To prove e = 1, we need to apply conservation of kinetic
ray energy also in an elastic collision.
ed
M ac t
re fr P 11. (c) Statement 1 : In the first case the mechanical energy is
tly
Par
completely converted into heat becuase of fiction. While
O
is second case, a part of mechanical energy is

converted into heat due to fiction but another part of
Pa

mechanical energy is retained in the form of potential


rtl
ay

energy of the block.


tr

re
fle
en
cid

c
t ed

Therefore statement 1 is correct.


In

ra

A B
y

Statement 2 : This is a wrong statement because the


But the angle between refracted and reflected rays is coefficient of friction between the block and the surface
POB. Clearly is POB is less than MOB. does not depend on the angle of inclination.
EBD_780
2007- 24 Target IIT-JEE
12. (b) Statement-1 : The wavelength of characteristic X-rays
depends on the type of atoms of which the target
material is made. It does not depend on the accelerating P0
potential. Therefore, statement 1 is true.
Statement-2 : When an electric beam strikes the target p
in an X-ray tube, part of the kinetic energy is converted L0
into X-ray energy. This statement is true. But P H
statement-2 does not explain statement-1. P

13. (c) Statement-1 :

1 1 1
L0 P0
The mirror (spherical) formula is valid only p= ... (ii)
u v f L0 H

for mirrors of small apertures where the size of aperture From (i) and (ii)
is very small as compared to the radius of curvature of
L0 P0
the mirror. This statement is true. P0 ( L0 H) g
L0 H
Statement-2 :
L0P0 = P0 (L0 – H) + (L0 – H)2 g
Laws of mirror are valid for plane as well as large
g (L0 – H)2 + P0 (L0 – H) – L0P0 = 0
spherical surfaces.
Therefore, statement 2 is wrong. 17. (c) For disc A
14. (a) When the piston is pulled out slowly, the pressure
1 2 1
drop produced inside th e cylin der is almost kx1 I (2 )2
2 2
instantaneously neutralised by the air entering from
outside into the cylinder (through the small hole at the kx12 2I 2
... (i)
top).
For disc B
Therefore, the pressure inside the cylinder is P 0
throughout the slow pulling process. 1 2 1 2 A
kx2 2I
15. (d) The condition for equilibrium of the piston is 2 2

Mg = (P0 – p) R2 Seal
kx22 I 2 ... (ii)
y 2L
Mg p On dividing (i) and (ii), we get B
p= 2
P0
R
P0 k x12 2I 2
x1
2
Since the cylinder is thermally conducting, the kx22 I 2 x2
temperature remains the same.
18. (a) When disc B is brought in contact with disc A
Therefore
Let ' be the final angular velocity of both the disc
P0 × (2L × R2) = py × R2 rotating together. Apply conservation of angular
momentum for the two disc system.
P0
y 2L I (2 ) + 2I ( ) = (I + 2I) '
p
4
'=
3
P0 P0 R2
2L 2L Torque on disc A
P0
Mg P0 R 2 Mg
R2 4
LA Lf Li I I 2
3 2I
16. (c) At equilibrium, p = P A= t t t 3t
p = P0 + (L0 – H) g ... (i) The negative sign represents that the torque creates
Also P0 × ( R2 L0) = p [ R2 (L0 – H)] angular retardation.
2007 IIT-JEE Solved Paper 25
2007-

19. (b) Loss in kinetic energy = (K.E.)initial – (K.E.)final C : r, s

1 1 1 4
2 F2
= I (2 ) 2 (2 I ) 2 (I 2I ) Reason : F = qvB v2 =
2 2 2 3 q 2 B2
Unit of v2 is m2s–2 which is further equal to FV2 kg–1.
3 4 4 2 D : r, s
= [2I 2 + I 2] – I
2 3 3
2GM
Reason : Escape velocity ve =
2 R
2 8 2 I
= 3I I
3 3 2GM
ve2
20. A : q R
Reason : When a charged capacitor is connected to the GM
ends of the wire, a variable current (decreasing in magnitude The unit of is m2 s–2.
R
with time) passes through the wire (shown as resistor) and
thermal energy is generated. The potential difference across 22. A p, r
the wire also decreases with time. The charge on the Reason : Characteristic X-ray are produced due to transition
capacitor plate also decreases with time. of electrons from one energy level to another.
Similarly the lines in the hydrogen spectrum is obtained due
B : r. s
to transition of electrons from one energy level to another.
Reason : e = B v B q, s
When B, , v are constant, e is constant Reason : In photoelectric effect electrons from the metal
A constant potential difference develops across the surface are emitted out upon the incidence of light of
ends of the wire and charges of constant magnitude appear appropriate frequency.
at the ends of the wire. In -decay, electrons are emitted from the nucleus of an
atom.
C:s
C p
Reason : The free electrons move under the influence of Moseley gave a law which related frequency of emitted X-
electric field opposite to the direction of electric field. This ray with the atomic number of the target material
movement of e– continues till the electric field inside the
wire is zero. a( Z – b)
Charges of constant magnitude appear at the ends of D q
the wire. In photoelectric effect, energy of photons of incident ray
D : p, q, r gets converted into kinetic energy of emitted electrons.
Reason : Since, E, R are constant, a constant current flows 23. (d) (a) In Na2O2, we have O 22 ion. Number of valence
in the wire. Due to heating effect of current, thermal energy
is generated in the wire. Also a constant potential difference elctrons of the two oxygen in O 22 ion = 8 × 2 + 2
develops between the ends of the wire. =18 which are present as follows
21. A : p, q
2
s2, *1s2, 2s2, *2s2, 2 px2 { 2 p 2y = 2 pz ,
m
Reason : Unit of GMeMs = Fr2 = Nm 2 = kg 2 × m2
s { *2 p 2y = *2 pz2
= kg m3s–2
Also (volt) (coulomb) (metre) = (joule) (metre) Number of unpaired electrons = 0, hence, O 22
= (N - m) (m) = Nm2 = kg m3s–2 is diamagnetic.
B : r, s (b) No. of valence electrons of all atoms in
O3 = 6 × 3 = 18.
3RT Thus, it also, does not have any unpaired electron,
Reason : vrms = hence it is diamagnetic.
M
(c) No. of valence electrons of all atom in N2O
2 3RT = 2 × 5 + 6 = 16. Hence, here also all electrons are
vrms
M paired. So it is diamagnetic.

3RT (d) In KO2, we have O2 . No. of valence electrons of


Unit of is m2 s–2
M all atoms in O2 = 2 × 6 + 1 = 13,
Also (farad) (volt)2 (kg)–1 = (joule) kg–1 = N-m kg–1
Thus it has one unpaired electron, hence it is
= kg ms–2 m kg–1 = m2s–2 paramagnetic.
EBD_780
2007- 26 Target IIT-JEE
24. (a) Molecular electronic configuration of Van't Hoff factor,
CO: 1s 2 , *1s 2 , 2s 2 , * 2s 2 ,{ 2 p y 2 2 pz 2 , 2 px2 Observed value of colligative property
i=
Nb Na 10 4
Theoretical value of colligative property
Therefore, bond order 3
2 2
2
= 0.5 .
NO : 1s 2 , *1s2 , 2s2 , *2s 2 , 2 px 2 ,{ 2 p y 2 2 pz 2 4
30. (d) In P4 the P–P linkage is formed by sp3 -sp3 hybridized
10 4
Bond order 3 orbital overlapping So, the percentage of p-character
2
will be 75%.
CN 1s 2 , *1s 2 , 2 s 2 , * 2s 2 ,
31. (b) A B
2 2 2
{ 2 py 2 pz , 2 px G° = H° – T S°
G° = – 2.303 RTlog10 K
10 4
Bond order 3 – 2.303 RT log10 K = H° – T S°
2
2.303 RT log10 K = T S° – H°
N2 : 1s2 , *1s 2 , 2s 2 , *2s 2 ,{ 2 p2y 2 pz2 , 2 p2x
T S H 298×10 + 54.07×1000
log10 K = =10
10 4 2.303RT 2.303×8.314× 298
Bond order 3
2 32. (c) Statement-1 is correct but statement-2 incorrect.
– 2 2 2 2 Orthoboric acid (H3BO3) is soluble in water and behaves
NO : 1s , *1s , 2s , * 2s , 2 px2 ,
as weak monobasic acid. It does not donate protons
{ 2 p 2y 2 pz2 , { * 2 p1y * 2 p1z like most acids, but rather it accepts OH– ions. It is,
therefore, Lewis acid, and is better written as B(OH)3.
10 6 B(OH)3 + 2H2O H3O+ + [B(OH)4]–; pKa = 9.25
Bond order 2
2 33. (a) Both statements are true. Boron forms only covalent
NO– has different bond order from that in CO. compounds (bonds) because small sized B ion polarizes
25. (b) The ring to which –NH group is attached is activated the corresponding anion largely.
due to the lone pairs on N (+M and +E effects); while 34. (b) Statement-1 is correct because the surfactant molecules
the ring to which –C = O is attached is deactivated. aggregate to form micelles only at or above the critical
Hene, the electrophile would go to the para-position
micellar concentration (CMC). Although statement-2
of the activated ring.
is also correct, i.e., the conductivity of the solution
26. (b) Extraction of Zn from ZnS (Zinc blende) is achieved by
having surfactant molecules decreases sharply at CMC.
roasting followed by reduction with carbon.
However, statement-2 is not the explanation for
2ZnS + 3O2 2ZnO + 2SO2
statement-1. The explantion is that "each micelle
ZnO + C Zn + CO contains at least 100 molecules" and thus, with the
27. (c) The five possible isomers are: formation of miscelles, the number of ions in solution
(i) n-Hexane (ii) 2-Methylpentane decreases and mobility of the bulkier micelle particles
(iii) 3- Methylpentane (iv) 2,2-Dimethylbutane decreases. This, finally leads to decrease in conductivity
(v) 2, 3-Dimethylbutane of the solution.
Alc.KOH 35. (d) p-Hydroxybenzoic acid has higher boiling point than
28. (b) BrCH2 – CH2Br CH2 = CHBr
o-hydroxybenzoic acid due to intermolecular hydrogen
NaNH 2 bonding. Thus, statement-1 is false. o-Hydroxybenzoic
CH CH
Elimination of HBr from CH2 = CHBr requires a stronger acid shows intramolecular H-bonding thus, statement-
base because here, Br acquires partial double bond 2 is true.
character due to resonance.
29. (a) Molecular weight of naphthoic acid O
C11H8O2 = 172 gmol–1. OH C H
The theoretical value of depression in freezing point O
H
20×1000
= Kf × molality = 1.72× 4K
172×50
2007 IIT-JEE Solved Paper 2007- 27

36. (b) Reaction at anode: 2Cl– Cl2 + 2e–


moles of Cl– = 4 × 500 × 10–3 = 2
1
moles Cl2 = ×2 1
2
37. (d) 500 ml of 4.0 molar NaCl has 2 mole of NaCl.
By electrolysis we can get a maximum of 2 moles of
sodium which can combine with exactly 2 moles of
mercury to give amalgam.
The maximum weight of amalgam which can be (B) : (p), (r) and (s)
formed from this solution In [Pt (NH3)Cl2], Pt is in + 2 state with configuration 5d 8.
= weight of 2 mole of sodium + weight of 2 mole of mercury Since NH3 is a strong field ligand, it will pair all the electrons
= 2 × 23 + 2 × 200 = 446g making the complex diamagnetic. Moreover, it is a square
38. (d) Na+ + e– Na planar complex showing cis-trans isomerism.
Total number of moles of Na+ discharged at cathode
= 2mole
The number of electron required for this purpose
= 2 mole
Total charge required
= 2 faraday = 2 × 96500 = 193000 coulombs.
39. (a) Argon, being a noble gas, will not react with the metals,
thus, can be used in arc welding. (C) : (q) and (s) In [Co(H2O)5Cl]Cl, Co is in + 2 state with
40. (c) In XeO3 there are total of 4 electron pairs around central 3d 7 configuration making it paramagnetic.
atom. Out of which 3 are bonding electron pair and one (D) : (q) and (s) In [Ni(H2O)6]Cl2, Ni is in + 2 state with
is non-bonding electron pair. This combination
3d 8 configuration. It is attached with weak
provides sp3-hybridization and pyramidal shape.
field ligands, therefore it is paramagnetic.
44. (A) : (p) and (s) Because 200 atm pressure is very large.
For H2 gas, at very high pressure Z > 1.
Xe
O (B) : (r) Since P ~ 0, it means very low presure,
O
O so ideal behaviour is observed.
41. (a) All xenon fluorides are strongly oxidizing, XeF4 can (C) : (p) and (q) Since P is 1 atm, Z for CO2 would be less
act as reducing agent (with F2) as well as oxidizing than 1.
agent but XeF6 can only function as an oxidizing agent. (D) : (r) In real gas with very high molar volume,
6XeF4 + 12H2O 4Xe + 2XeO3 + 24HF + 3O2 molecules will be very far apart from
XeF6 + 3H2O XeO3 + 6HI each other due to which van der Waal's
42. (A) : (p) and (s) Cellulose is a natural polymer and has a forces as well as actual volume occupied
C1 – C4 -glycosidic linkage. by molecules will be negligible.
(B) : (q) and (r) Nylon-6, 6 is a synthetic polymer of 45. (a) Given that f (x) is differentiable on (0, ) with
hexamethylenediamine and adipic acid
and has amide linkages. t 2 f ( x) x 2 f (t )
f (1) = 1 and lim 1 for each x > 0
(C) : (p) and (r) Proteins are natural polymers of amino t x t x
acids joined by amide linkages (peptide
bonds). 2t f ( x) x2 f '(t )
(D) : (s) Sucrose is a disaccharide of -D glucose lim 1
t x 1
and -D-fructose and has an
, -glycosidic linkage. [Using L' Hospital rule]
43. (A) : (p), (q) and (s) 2xf (x) – x2 f ' (x) = 1
In [Co(NH3)4(H2O)2]2+, Co is in + 2 state having 3d7
2 1
configuration, which makes it paramagentic due to odd f ' (x) – f ( x)
electrons. Moreover, it is an octahedral complex showing x x2
cis-trans isomerism w.r.t., H2O. [Linear differential equation]
EBD_780
2007- 28 Target IIT-JEE
Integrating factor
1
1 e e
2 1 e
dx 2 1 2 1
e x e 2 log x e log1/ x 2 e e
2 x c
x e e 1
2 e e 1
2
2
1 1 1
Solution is f (x) × dx
2 2
x x x2
e e 1 2
.
2 (e e 1 )
f ( x) 1 1
2 3
C f (x) = Cx2 +
x 3x 3x
e e 1
Also f (1) = 1 1
2 ve
ve
1 e e 1 ve
1= C+ C = 2/3 1
3 2

x–c<0 x<c
2 2 1
f (x) = x The two lines meet on the left of line x = c.
3 3x
48. (c) Let E1 The Indian man is seated adjacent to his wife.
46. (d) As , are the roots of x2 – px + r = 0
E2 Each American man is seated adjacent to
=p ....(1) his wife.
and =r ....(2)
P ( E1 E2 )
Then P ( E1 / E2 )
P ( E2 )
Also ,2 are the roots of x2 – qx + r = 0
2
Now E1 E2 All men are seated adjacent to their
wives.
2 = q or + 4 = 2q ....(3)
2 We can consider the 5 couples as single-single
Solving (1) and (3) for and ,we get objects which can be arranged in a circle in 4! ways.
But for each couple, husband and wife can interchange
1 2 their places in 2! ways.
(2q p ) and (2q q )
3 3 Number of ways when all men are seated adjacent
to their wives = 4! × (2!)5
Substituting values of and , in equation (2),
Also in all 10 persons can be seated in a circle in 9!
2 ways.
we get (2 p q)(2q p) r.
9
4! (2!)5
47. (a) The equation of tangent to the curve y = ex at (c, ec) is P (E1 E2 )
9!
y ec ec ( x c) … (1) Similarly if each American man is seated adjacent to his
wife, considering each American couple as single object
and equation of line joining (c –1, ec–1) and (c +1, ec+1)
and Indian woman and man as seperate objects there
is are 6 different objects which can be arranged in a circle
in 5! ways. Also for each American couple, husband
ec 1 ec 1
y ec 1
[ x (c 1)] and wife can interchange their places in 2 ! ways.
(c 1) (c 1) So the number of ways in which each American man is
seated adjacent to his wife.
e c (e e 1 ) = 5! × (2!)4
y ec 1
[ x c 1] … (2)
2
5! (2!) 4
Subtracting equation (1) from (2), we get P ( E2 )
9!
1 1
e e 2 e e (4! (2!)5 ) / 9!
ec ec 1 ec ( x c) ec So P ( E1 / E2 )
2
2 2
(5! (2!) 4 ) / 9! 5
2007 IIT-JEE Solved Paper 2007- 29
49. (a) The length of transverse axis = 2 sin = 2a 51. (d) OP OA AP
a = sin OP OA OB
Also for ellipse 3x2 + 4y2 = 12
z 2)
x2 y2 P(
or 1, a2 = 4, b2 = 3
4 3 B 4
z1
2 4 3 A
b 3 1 /4
e 1– 1–
a 2 4 2 O

1
Focus of ellipse 2 ,0 (1, 0) OP 3ei /4
4ei( /2 / 4)
2
3ei /4
4ei /2 i /4
.e
As hyperbola is confocal with ellipse, focus of
hyperbola = (1, 0) 3ei /4
4iei /4

ae = 1 sin ×e=1 ei / 4 (3 4i ).
e = cosec 52. (c) We know that three vector are coplanar if their scalar
triple product is zero.
b2 = a2 (e2 – 1) = sin2 (cosec2 – 1) = cos2
Equation of hyperbola is 2
1 1
2
1 1 0
x2 y2
– 1 2
sin 2 cos2 1 1

or, x2cosec2 – y2 sec2 =1 R1 R1 R2 R3

2 2 2
sec2 x 2 2 2
f (t )dt 0
2 2
50. (a) Let L = lim form 1 1 0
2 0
x
4 x2 1 1 2
16

On applying L' Hospital's rule, we get 1 1 1


2 2
(2 )1 1 0
d sec 2 x 2
f (t )dt 1 1
dx 2
L = lim
2
x d 1 1 1
4 x2
dx 16 (2 2
) 0 (1 2
) 0 0
2
0 0 (1 )
Using
( R2 R1, R3 R1 )
d h ( x)
f (t ) dt f (h( x )) h '( x ) f ( g ( x)). g '( x), we get 2 2 2
dx g ( x) (2 )(1 ) 0

2
f (sec 2 x ).2sec 2 x tan x Two real solutions.
L = lim
2x 53. (c) 2 sin2 cos 2 0 1 2cos 2 0
x
4
1 5 7 11
cos 2 2 , , ,
2 3 3 3 3
f (2) 2 2 1 8
= f (2)
5 7 11
2 , , , ....(1)
4 6 6 6 6
EBD_780
2007- 30 Target IIT-JEE
where [0, 2 ] Now given that 0 < P (E) < 1

P( E / Hi )P(Hi )
Also 2 cos 2 3sin 0 0 1
2 P( Hi / E)
2 sin + 3 sin – 2 = 0
P (E/Hi) P (Hi) < P (Hi/E)
1
(2 sin – 1) (sin + 2) = 0 sin
2 But if P(Hi E) 0 then
[ sin 2] P (Hi/E) = P (E/Hi) = 0
Then P (E/Hi) P (Hi) < P (Hi/E) is not true.
5
, .... (2) Statement -1 is not always true.
6 6
Also as H1 , H2 , …Hn are mutually exclusive and
where [0, 2 ] n

exhaustive events, therefore P( Hi ) 1.


5
Combining (1) and (2), we get , i 1
6 6
Two solutions are there. Statement -2 is true.
54. (a) Equation of director circle of the given circle 1
x2 + y2 = 169 is x2 + y2 = 2 × 169 = 338.
56. (d) F ( x) sin 2 x dx (1 cos 2 x )dx
2
We know from every point on director circle, the
tangents drawn to given circle are perpendicular to 1
(2 x sin 2 x) C
each other. 4
Here (17, 7) lies on director circle.
1
The tangent from (17, 7) to given circle are mutually Now, F ( x ) (2 x 2 sin (2 x 2 )) C
perpendicular. 4
Alternatively :
1
Let (x1, y1) be the point from which the tangents drawn [2 x 2 sin 2 x] C F ( x)
4
to the circle x2 + y2 = 169 are mutually perpendicular.
Then combined equation of tangents is given by Statement –1is false.
SS1 = T2 Also sin2 (x + ) = sin2 x, x R

(x2 + y2 + 169) ( x12 Statement –2 is true.


y12 169)
= (xx1 + yy1 – 169)2 57. (c) PQ ( RS ST ) PQ RT
As lines represented by this equation are mutually
perpendicular. = | PQ | | RT | sin150 n 0
Coeff. of x2 + Coeff. of y2 = 0
0
x12 y12 169 x12 x12 y12 169 y12 0 Statement-1 is true.

x12 y12 338 Also, PQ RS | PQ | | RS | sin120 n1 0


Locus of (x1, y1) is
And PQ ST PQ | | ST | sin180 n2 0
x2 + y2 = 338
Also (7, 17) satisfies it, therefore the tangents drawn Statement-2 is false.
from (7, 17) to the given circle are mutually
perpendicular. S
P(Hi E) P(E / Hi )P(Hi ) R 120°
55. We know P(Hi/E) T
P(E) P(E)

P (Hi/E) P (E) = P (E/Hi) P (Hi)


U Q
P(E / Hi )P(Hi )
P(E)
P(Hi / E)
P
2007 IIT-JEE Solved Paper 2007-31
For Qs. 58-60 : 62. (d) Tr Vr 1 Vr 2
y
P(1, 2 2) (r 1)2 r 1 r2 r
(r 1)3 r3 2
2 2 2 2

3r 2
2r 1
Tr = (r + 1) (3r – 1)
S O T R x For each r, Tr has two different factors other than 1 and
(–1,0) (9,0)
itself.
Q Tr is always a composite number.
(1, 2 2) 63. (b) Qr 1 Qr Tr 2 Tr 1 (Tr 1 Tr )

y' = Tr 2 2Tr 1 Tr
1 (r 3)(3r 5) 2(r 2)(3r 2) (r 1)(3r 1)
PQ ST ST 2 1
Ar PQS 2 Qr + 1 – Qr = 6 (r + 1) + 5 – 6r – 5 = 6 (constant)
58. (c)
Ar PQR 1 TR 8 4 Q1, Q2, Q3, .... are in AP with common difference 6.
PQ TR
2 64. A-p, q, r; B-p, s; C-r, s; D-p, q
59. (b) For PRS,
x 2 if x 0
1 1 (A) y x| x|
Ar ( PRS) SR PT 10 2 2 x 2 if x 0
2 2
Y
10 2, a PS 2 3
b = PR = 6 2 , c = SR = 10
Radius of circumference
–1
abc 2 3 6 2 10 Graph is as follows : X' X
R 3 3 O
4 4 10 2 1
60. (d) Radius of incircle
area of PQR
semi perimeter of PQR s Y'
From graph y = x | x | is continuous in (– 1, 1) (p)
We have a = PR 6 2 , b = QP =PR 6 2 differentiable in (– 1, 1) (q)
Strictly increasing in (–1, 1). (r)
c = PQ 4 2
1 x if x 0
and PQ TR 16 2 (B) y |x|
2 x if x 0
6 2 6 2 4 2 y2 = – x, x < 0
s 8 2 {where y can take only + ve values}
2
2
and y = x, x 0
16 2 Graph is as follows :
r 2
8 2 We get
Y
n n
r2 r
61. (b) V1 + V2 + .... + Vn = Vr r3
r 1 r 1
2 2

n2 n
= n3
2 2 X' X
–1 O 1
n 2 (n 1)2 n (n 1) (2 n 1) n( n 1)
=
4 12 4
n ( n 1) 2n 1
= n( n 1) 1
4 3 Y'
From graph y | x | is continuous in (– 1, 1) (p)
n ( n 1) (3n 2 n 2)
= not differentiable at x = 0 (s)
12
EBD_780
2007- 32 Target IIT-JEE

(A) a b c 0 and a 2 b2 c2 ab bc ca 0
x 1, 1 x 0 2 2 2
(a b) (b c ) (c a ) 0
(C) y x [ x] x, 0 x 1 a b c (but 0as a b c 0)
x 1, 1 x 2 This equation represent identical planes.
(B) a b c 0
Graph of y = x + [x] is as follows : and a 2 b 2 c 2 ab bc ca 0
0 and a, b, c are not all equal.
Y
All equations are not identical but have infinite many
3 solutions.
ax by ( a b) z (using a+b+c=0)
2
and bx cy (b c) z
1 (b2 ac) y (b2 ac ) z y=z
X' X ax by cy 0 ax ay x y
–2 –1 O 1 2 x y z
–1
The equations represent the line x y z
–2 0 and a 2 b 2 c 2 ab bc ca 0
(C) a b c
0 Equations have only trivial solution
i.e., x y z 0
Y' the equations represents the three planes meeting at a
From graph, y = x + [x] is neither continuous, nor single point namely origin.
differentiable at x = 0 and hence in ( – 1, 1). (s)
(D) a b c 0 and a 2 b2 c2 ab bc ca 0
Also it is strictly increasing in (– 1, 1) (r)
a b c and 0
2 x, x 1
a b c 0
(D) y | x 1| | x 1| 2, 1 x 1 All equations are satisfied by all x, y, and z.
2 x, x 1 The equations represent the whole of the three
Graph of function is as follows : dimensional space (all points in 3–D)
66. A-s; B-s; C-p; D-r
Y
1 dx 1
(A) [tan x]1 1 tan 1 (1) tan 1 ( 1)
2 11 x2

1 2
4 4 4 2
X' X
–1 O 1 1 dx 1
1
–1 (B) sin x sin 1 (1) sin 1 (0)
0 2 0
1 x
–2

Y' 0
2 2
From graph, y = f (x) is continuous (p) and differentiable
(q) in (– 1, 1) but not strictly increasing in (– 1, 1). 3
3 dx 1 1 x 1
65. A-r; B-q; C-p; D-s (C) log [log 2 log 3]
21 2 2 1 x 2
Here we have, the determinant of the coefficient matrix of x 2
given equation, as
1
log 2 / 3
a b c 2
b c a
(a b c )(a 2 b2 c2 ab bc ca ) 2 dx 1 2 1
c a b (D) sec x sec 2 sec 1 1
1 2 1
x x 1
1
(a b c)[(a b)2 (b c ) 2
(c a ) ] 2
2 0
3 3
2007 IIT-JEE Solved Paper 2007-33

PAPER - II
1. (d) By the concept of energy conservation
Now Y 2 D
2 Y D
1 2 1 2 3v
mv I mg
2 2 4g [ the value of m, g and L are exact]

For rolling motion v = R 0.01 0.05


=2 = 2 × 0.025 + 0.0625
0.4 0.8
1 2 1 v2 3 2 = 0.05 + 0.0625 = 0.1125
mv I mv
2 2 R2 4 Y = 2 × 1011 × 0.1125 = 0.225 × 1011

1 v2 3 2 1 2 1 2 0.2 1011 N/m 2


I mv mv mv We can also take the value of Y from options given
2 R2 4 2 4
without calculating it as it is same in all options.
1 v2 1 2 1 2 Y 2 0.2 1011 N/m 2
I mv I = mR
2 R2 4 2 4. (a) For a < x < 2a;
This is the formula of the moment of inertia of the disc.
2. (b) Let us consider a uniformly charged solid sphere B = B0 ˆj
without any cavity. Let the charge per unit volume be
The initial velocity is v v0 i
and O be the centre of the sphere. Let us consider a
uniformly charged sphere of negative charged density
Z
having its centre at O'. Also let OO' be equal to a. Y'
Let us consider an arbitrary point P in the small sphere.
F B0
The electric field due to charge on big sphere

E1 OP
3 X
0 x=a V0
From the diagram it is clear that the force on the particle
is towards positive Z-axis (apply Fleming's left hand
O rule) at x = a, which shifts the particle as shown in the
a X-Z plane.
P O For 2a < x < 3a
B B0 j
Also the electric field due to small sphere The direction of velocity is shown at x = 2a. Again
using Fleming's left hand rule, we get the direction of
E2 PO ' force. This changes the profile of the particle as shown
3 0 by the dotted line.
The total electric field Alternatively :
Use the vector form of B and v in the formulae
E E1 E 2 [OP PO] OO '
3 0 3 0 F q(v B) to get the instantaneous direction of
Thus electric field will have a finite value which will be force at x = a and x = 2a.
uniform. 5. (a) As shown in the figure, the prongs of the tuning fork
are kept in a vertical plane.
mg L
3. (b) We know that Y = 2
D
4

4mgL 4 1 9.8 2
Y 2
D 3 2 3
0.4 10 0.8 10

2.0 1011 N/m 2


EBD_780
2007- 34 Target IIT-JEE
6. (a) The cut off wavelength is given by In the first part the force is created due to pressure and
in the second part the force is due to surface tension T.
hc
0 ...(i) Force = 2P0Rh + R gh2 – 2RT
eV
10. (b) Statement 1 : The total kinetic energy of n moles of gas
According to de Broglie equation
3
h h is K = nRT
2
p 2meV But PV = nRT
K = 1.5 PV
2 h2 h2 ..(ii)
V Statement one is true.
2meV 2
2me Statement 2 :
From (i) and (ii), The molecules of a gas collide with each other and the
2 2
velocities of the molecules change due to collision.
hc 2me 2mc But statement 2 is not a correct explanation of
0
eh 2 h statement 1.
11. (a) As shown in the figure the horizontal component of
7. (d) p (t ) A[i cos (kt ) j sin (kt )] the magnetic field interacts with the induced current
produced in the conducting ring which produces an
dp average force in the upward direction. (Fleming's left
F Ak [ i sin (kt ) j cos (kt )]
dt hand rule).
F .P 0 F I
I
F. p Fp cos
BH
But F . p 0 cos =0
= 90°.
Conducting ring
8. (c) The charges make an electric dipole. A and B points lie
on the equatorial plane of the dipole. 12. (b) Statement 1 : Cloth can be pulled out without
Therefore, potential at A = potential at B = 0 dislodging the dishes from the table because of inertia.
W = q (VA – VB) = q × 0 = 0 Therefore, statement – 1 is true.
Statement 2 : This is Newton's third law and hence
true. But statement 2 is not a correct explanation of
statement 1.
13. (b) Statement 1 : For velocity of centre of mass to remain
constant the net force acting on a body must be zero.
Therefore the statement 1 is false.

Statement 2 : The linear momentum of an isolated


system remains constant. This statement is true.
14. (b) The speed of sound depends on the frame of reference
of the observer.
h g 15. (a) Since all the passengers in train A are moving with a
9. (b) The force is P0 (2 R h) 2 RT
2 velocity of 20 m/s therefore the distribution of sound
intensity of the whistle by the passengers in train A is
P0 uniform.

2R v v0 340 30 31
16. (a) ' 1 800 800
FT v vs 340 20 32
h/2
h
P0 + g v v0 31
2
'' 2 1120
v vs 32

31 31
'' ' = (1120 – 800) × 320 310 Hz.
32 32
2007 IIT-JEE Solved Paper 2007-35
17. (a) For plane wave fronts the beam of light is parallel.
A B
18. (c) Since points c and d are on the same wavefront,
therefore d = c P
Similarly, e = f
d– f = c– f
19. (b) The gap between consecutive wavefronts in medium 2 C : q, r
is less than that is medium 1. Therefore, wavelength of Reason : The magnetic field due to current in loop A at P is
light in medium 2 is less than that in medium 1. Therefore, equal and opposite to the magnetic field due to current in
speed of light is more in medium 1 and less in medium 2. loop B at P.
20. A p A B
2
Reason : For a simple harmonic motion v = a x 2 . On P

comparing it with v = c1 c2 x 2 we find the two


comparable.
B q, r D : q, s
Reason : The direction of magnetic field at P due to current
Reason : v = – k x
in loop A is perpendicular to the plane of paper directed
when x is positive; v is – ve, and as x decreases, v decreases. vertically upwards.
Therefore kinetic energy will decreases. When x = 0, v = 0. The direction of magnetic field at P due to current in loop B
Therefore the object does not change its direction. is perpendicular to the plane of paper directed vertically
When x is negative, v is positive. But as x decreases in downward.
magnitude, v also decreases. Therefore kinetic energy Since the current are in opposite direction the wires repel
decreases. When x = 0, v = 0. Therefore the object does not each other.
change its direction. B
c p
A
Reason : When a = 0, let the spring have an extension x.
Then k x = mg. P
When the elevator starts going upwards with a constant
acceleration, as seen by the observer in the elevator, the
object is at rest. 22. A s
ma + mg = k x' Reason : Bimetallic strip is based on thermal expansion of
solids.
ma = k (x' – x) (Since a is constant)
B q
D q, r Steam engine is based on energy conversion.
The speed is 2 times the escape speed. Therefore the C p, q
Incandescent lamp is based on energy conversion and
object will leave the earth. It will therefore not change the
radiation from a hot body.
direction and its kinetic energy will keep on decreasing.
D q, r
21. A : q, r Electric fuse is based on melting point of the fuse material
Reason : The magnetic field at P due to current flowing in which is turn depends on the heating effect of current.
AB is perpendicular to the plane of paper acting vertically
downward. And the magnetic field at P due to current flowing 1.O3 CHO
23. (a)
in CD is perpendicular to the plane of paper acting vertically 2.H2O/Zn CHO
upwards. [E]
A B
Therefore, q is correct. P KOH(aq),
C D
CHO
As P is the mid point, the two magnetic fields, cancel out – H 2O
each other. Therefore, r is correct. aldol condensation
[F]
B:p
Reason : The magnetic field at P due to current in loop A is 24. (a) Due to similar charges on adjacent atoms, the structure
along the axial line towards right. Similarly, the magnetic (a) is least stable.
25. (d) An anionic carbonyl complex can delocalise more
field at P due to current in loop B is also along the axial line
electron density to antibonding pi-orbital of CO and
towards right.
hence, lowers the bond order.
EBD_780
2007- 36 Target IIT-JEE
26. (a) Since, liquid is passing into gaseous phase so 34. (c) Statement-1 is correct, but statement-2 is incorrect
entropy will increase and at 373 K during the phase because glucose on reaction with Fehling solution
transformation it remains at equilibrium. So, G = 0. gives Cu2O and not CuO.
27. (d) The following reaction occurs: 35. (b) Statement-1 is correct. Statement-2 is also correct but
2 not the correct explanation becuase blue colour of the
6Fe Cr2 O 27 14H 6Fe 3
2Cr 3 + 7H2O
solution is due to the solvated electrons.
From the above equation, we find that Mohr's salt
(FeSO4.(NH4)2SO4.6H2O) and dichromate reacts in 6 : 1 36. (c) 2I Cl2 I2 2Cl
molar ratio.
o
28. (d) Overall order = sum of orders w.r.t each reactant. E° = E I Eo 0.54 1.36
/ I2 Cl2 / Cl
Let the order be x and y for G and H respectively
E° = 0.82V
[G]mole [H]mole rate(mole E° is positive hence, iodide ion is oxidized by chlorine.
Exp.No. 1 1
litre litre litre time 1 ) 37. (d) 4Mn 3 2H 2 O 4Mn 2 O2 4H
1 a b r
2 2a 2b 8r Eo EoH = 1.50 + (– 1.23) = 0.27 V
Mn 3 / Mn 2 2 O / O2
3 2a b 2r
Reaction is feasible. [ E o is positive]
Applying r = k [G]x [H]y we get, 38. (a) The precipitate formed in this reaction is of
x = 1, y = 2 Fe4 [ Fe(CN)6]3
39. (c) Reagents for Reimer - Tiemann reaction are
For (1) and (3), the rate is doubled when conc. of G
aq. NaOH + CHCl3.
is doubled keeping that of H constant i.e., rate [G]
-
x=1 40. (c) OH- + CHCl3 : C Cl3 + H 2O
From (2) and (3), y = 2
Overall order is 3. CCl3 Cl : CCl2 (Dichlorocarbene)

29. (b) Hg 2 2KI HgI2 2K


(red ppt)

HgI 2 2KI K 2 [HgI 4 ]


41. (b)
Hg 2 Co2 4SCN Co[Hg(SCN) 4 ]
(deep blue crystalline)

23 23 0
30. (c) 11
Na 10
X 1

31. (a) Anti addition of Br2 on trans alkene provides meso


compound.
CH3
CH 3 H H Br
CCl4
C C + Br2
H CH 3 H Br 42. A-p, s; B-p,q; C-q; D-q, r.
CH3 A p, s; Parameters of a cubic system are a = b = c and
= = = 90°
32. (b) Statement-1 is correct because as we go down a group,
energy gap between successive orbits decreases which There are three types of lattices in cubic system
causes decrease in the energy gap betwen valence These are simple, Face centred and body centred
band and conduction band. Statement-2 is also correct B p, q ; The parameters of a Rhombohedral system are
because each band is a collection of closely spaced a = b = c; = = = 90°
large number of atomic energy levels. But this is not Cubic and Rhombohedral are two crystal systems. There
correct explanation of statement-1. are seven crystal systems in all
33. (c) Statement-1 is correct. Statement-2 is incorrect because C q; These are two crystal systems.
compound can be chiral even in the absence of chiral D q, r; Hexagonal and monoclinic are two crystal systems.
atoms. The parameters of these are
2007 IIT-JEE Solved Paper 2007-37
Hexagonal; a = b c and = = 90°, r = 120°
Also since a, b, c are non parallel (these form an equilateral
Monoclinic ; a b c and = = 90°, 90° ).
43. (A - p, s); (B - r); (C - p, q); (D - p).
A p, s; The reaction is redox reaction because the O.N. of a b b c c a 0
47. (d) Given | z | = 1 and z 1
O in O2– is – 0.5 and that in O2 is zero. In O 22 is –1.0.
z
It involves reduction oxidation reaction. Since here a To find locus of
1 z2
part of molecule is oxidised and a part is reduced so it
is disproportionation. z z
We have
B r; The structure of Cr2O72– is given below 1 z 2
zz z2

2– [ |z| 1 | z |2 zz 1]
O
O 1
purely imaginary number
z z
Cr must lie on y – axis.
O O Cr
O
Alternatively :
O O | z | 1 let z ei

[Note : In any solution dichromate ions and chromate ions ei ei . e i i


then
exist in equilibrium. In alkali solution, dichromate ions are 1 e 2i i i 2 sin
e e2i . e
converted into chromate ions and on acidification chromate
ions are converted back into dichromate ion.] Re ( ) = 0 x=0 locus of is y - axis.
C p, q; The reaction is x
48. (a) Given f (x) for n 2
2MnO4– 6H 5NO2 –
2Mn 2
3H 2 O 5NO3– (1 x n )1/ n
In involves change in O.N of Mn (from + 7 in MnO4– ) to x
+ 2(in Mn2+), So Mn is reduced and NO2– is oxidised to f o f (x) =f [f (x)] = f
NO3– it is a redox reaction. (1 x n )1/ n
The structure of NO3– (one of the products is trigonal
x
planar)
D p, It is a redox reaction. (1 x n )1/ n x
1/ n
44. A-p, s; B-q; C-q, r, s; D-q, r x n (1 2 x n )1/ n
1
2
d x d dx d dx dx 1 xn
45. (d)
dy 2 dy dy dx dy dy
x
Further, f o f o f (x) =
d 1 1 (1 3 x n )1/ n
= dx dy dy Proceeding in the similar manner, we get
dx dx x
g (x) = f o f o f ..... o f (x)
3 (1 nx n )1/ n
1 d2y 1 dy d2 y (f occurs n times)

dy
2
dx 2 dy dx dx 2
x n –1
dx dx Now, x n – 2 g ( x ) dx dx
(1 nx n )1/ n
46. (b) Since, a b c 0 and a, b, c are unit vectors, Let 1 + nxn = t
n2xn–1 dx = dt
therefore a, b, c form an equilateral triangle.
1
a (a b c ) 0 Integral becomes t –1/ n dt
n2
a a a b a c 0
1
a b c a –
n
1
1 t
Similarly, b c c a . K
n2 – 1 1
a b b c c a n
EBD_780
2007- 38 Target IIT-JEE
51. (c) We have
1 t1–1/ n
. K
n n –1

(1 nx n )1–1/ n
K G
n(n –1)
49. (c) Ar ( OPR) = Ar ( PQR) = Ar ( OQR) F
E
P (3, 4)

F G=
P (E c Fc G)
R P (E c F c / G)
P (G)

P (G) P ( E G) P(G F)
By simply geometry P (G)
R should be the centroid of PQO
[From venn diagram Ec Fc G= G–E G–F G]
3 6 0 4 0 0 4
R , 3, P (G) P (E ) P ( G) P(G) P ( F )
3 3 3
P (G)
Alternatively :
Here OP = PQ = 5 [ E, F, G are pairwise independent]
OPQ is an isosceles . = 1– P (E) – P (F) = P (Ec) – P (F)
R must lie on angle bisector of P. 52. (b) Given AB || CD, CD = 2AB
Let AB = a then CD = 2a
Y Let radius of circle be r.
Let circle touches AB at P, BC at Q, AD at R and CD at S.
P (3, 4) Then AR = AP = r, BP = BQ = a – r
DR = DS = r and CQ = CS = 2a – r
In BEC
BC2 = BE2 + EC2
(a – r + 2a – r)2 = (2r)2 + (a)2
R (3, k)
X 9a2 + 4r2 – 12ar = 4r2 + a2
O
3
r a ....(1)
Let R (3, k) 2
Also Ar (quad. ABCD) = 18
1 2a
Now Ar ( ORQ) = Ar ( OPQ)
3 a
r 2a-r
1 1 1 4 D S E C
6 k 6 4 k
2 3 2 3
4
R 3,
3
–r
2a

50. (c) The letter of word COCHIN in alphabetic order are C, R O


C, H, I, N, O.
Fixing first letter C and keeping C at second place, rest
Q

4 can be arranged in 4! ways.


r
a–

Similarly the words starting with CH, CI, CN are 4! in


each case. A P a-r B
r
Then fixing first two letters as CO next four places a
when filled in alphabetic order give the word COCHIN. 1
Numbers of words coming before COCHIN are a 2r a 2r 18
2
4 × 4! = 4 × 24 = 96
ar = 6
2007 IIT-JEE Solved Paper 39
2007-

Solving the above equations using cross product


3r 2
6 (using equation (1)) method, we get a : b : c 14 : 2 :15
2 Eqn. of line is
r2 = 4 r=2 x 3 y 1 z
t
1 y 2 14 2 15
dy
53. (c) whose parametric form is
dx y
x 3 14t , y 1 2t, z 15t
2y Statement-I is false (value of y is not matching).
dy 2dx 0 Since dr’s of line intersection of given planes are
1 y2
14, 2,15

2 1 y2 2x 2c 14i 2 j 15k is parallel to this line.


Statement 2 is true.
1 y2 x c ( x c) 2 y 2 1
a b 2ab
which is a circle of fixed radius 1 and variable centre (c, 58. (c) Given A1 , G1 ab, H1
2 a b
0) lying on x-axis.
54. (c) Point of intersection of L1 and L2 is A (0, 0). An 1 Hn 1
also An , Gn An 1 H n 1
Also P (– 2, – 2), Q (1, – 2) 2
A 2 An 1 H n 1
(0, 0) Hn
An 1 H n 1
L2
=0

Gn2 An H n An H n An 1 H n 1
:2
–x

x+

Similarly we can prove


:y

y=

AnHn = An–1Hn–1 = An–2 Hn–2 = .... = A1H1


1
L

R AnHn = ab
P L3 : y + 2 = 0 Q
G12 G22 G32 .... ab
(–2, –2) (1, –2)
AR is the bisector of PAQ, therefore R divides G1 G2 G3 .... ab
PQ in the same ratio as AP : AQ. An Hn
1 1
Thus PR : RQ = AP : AQ = 2 2 : 5 59. (a) We have An
2
Statement-1 is true.
Statement-2 is clearly false. An– AAn-1 An 1 Hn 1
n An 1
55. (b) Given that f (x) = 2 + cos x which is continuous and 2
differentiable every where. Hn An
1
1
Also f ' (x) = – sin x = 0 ( An 1 H n 1 )
f ' (x) = 0 x=n 2
An < An–1 or An–1 > An
There exists c [t, t + ] for t R We can conclude that A1 > A2 > A3 > ....
Such that f ' (c) = 0
Statements-1 is true. ab
Also f (x) being periodic of period 2 , statement-2 is 60. (b) We have An Hn = ab Hn
An
true, but statement-2 is not a correct explanation of
statement-1. 1 1
Hn–1 < Hn
2 An 1 An
x
56. (a) The given curve is y – x 1 H1 < H2 < H3
2
or (x – 1)2 = – 2 (y – 3/2) 61. (b) For k = 0, line y = x meets y = 0, i.e., x-axis only at one
which is a parabola, so should be symmetric with point.
respect to its axis x – 1 = 0 For k < 0, y = kex meets y = x only once as shown in the
Both the statements are true and statement 2 is a graph.
correct explanation for statement 1. y
57. (d) The line of intersection of given plane is y=x
3 x 6 y 2 z 15 0 2 x y 2 z 5
For z 0 , we obtain x 3 and y 1 O
Line passes through (3, –1, 0). x' x
Let a, b, c be the d’rs of line of intersection, then
x
3a 6b 2c 0 y = ke , k < 0
2a b 2c 0 y'
EBD_780
2007- 40 Target IIT-JEE
62. (a) Let f (x) = kex – x –x –1 ( x 1)
Now for f (x) = 0 to have only one root means the line y Also f (x) – 1 –
2 ( x – 2)( x – 3)
= x must be tangential to the curve y = kex. x – 5x 6
Let it be so at (x1, y1) then –( ve)
dy dy For – 1 < x < 1, f (x) – 1 –ve
1 ke x1 (–ve)(–ve)
dx curve1 dx curve 2 f (x) – 1 < 0 f (x) < 1(s)
0 < f (x) < 1 (p)
1
e x1 also y1 ke x1 and y1 = x1 (–ve)( ve)
k (B) If 1 < x < 2 then f (x) –ve
x1 = 1 1 = ke k = 1/e (–ve)(–ve)
63. (a) For y = x to be tangent to the curve f (x) < 0 (q) and so f (x) < 1 (s)
y = kex, k = 1/e (–ve)( ve)
For y = kex to meet y = x at two points we should (C) If 3 < x < 5 then f (x) –ve
( ve)( ve)
1 1
have k k 0, as k > 0. f (x) < 0 (q) and so f (x) < 1 (s)
e e (D) For x > 5, f (x) > 0 (r)
64. A-p, q; B-p, q; C-q, r; D-q, r
–( x 1)
Also f (x) – 1 0
( x – 2)( x – 3)
For x > 5, f (x) < 1(s)
0 < f (x)< 1(p)
(A) - p, q 66. A-p, B-q, C-p, D-s
C1 C2

sin–1(ax) + cos– 1 y + cos– 1(bxy) =


2
It is clear from the figure that two intersecting circles have a
common tangent and a common normal joining the centres
cos–1 y + cos–1 (bxy) = – sin –1(ax) = cos–1(ax)
2
Let cos–1 y = , cos–1 (bxy) = , cos–1 (ax) =
(B) - p, q C1 C2 y = cos , bxy = cos , ax = cos
We get + = and cos = bxy
(C) – q, r cos ( – ) = bxy
Two circle when one is completely inside the other have a cos y cos + sin sin = bxy
common normal C1C2 but no common tangent.
axy + sin sin = bxy
(a – b) xy = – sin sin
(a – b)2 x2y2 = – sin2 sin2
C1 C2
= (1– cos2 ) (1– cos2 )
(a – b)2 x2y2 = (1– a2x2 (1– y2
(D) – q, r (A) For a = 1, b = 0, equation (1) reduces to
Two branches of hyperbola have no common tangent but x2y2 = (1 – x2) (1– y2) x2 + y2 = 1
have a common normal joining S1S2.
(B) For a = 1, b = 1 equation (1) becomes
(1– x2) (1– y2) = 0
(x2 – 1) (y2 – 1) = 0
S2 S1
(C) For a = 1, b = 2 equation (1) reduces to
x2y2 = (1– x2) (1– y2)
65. A-p, r, s; B-q, s; C-q, s; D-p, r, s
x2 + y2 = 1
2
x – 6x 5 ( x – 5)( x –1)
We have f (x) (D) For a = 2, b = 2 equation (1) reduces to
x2 – 5x 6 ( x – 2)( x – 3)
0 = (1– 4x2) (1– y2)
(–ve)(–ve) (4x2 – 1) (y2 – 1) = 0
(A) If – 1 < x < 1 then f (x) ve
(–ve)(–ve)
f (x) > 0 (r) . . .
MEMORY BASED

IIT-JOINT ENTRANCE EXAMINATION (IIT-JEE)


SOLVED PAPER - 2006
PHYSICS
––––––––––––––––––––––––––––––––––––––––––––––––––––––––––––––––––––––––––––––––––––––––––––––––––––
General Instructions
1. Q. No. 1 to 12 have only one correct answer and carries 3 marks each for correct answer and –1 mark for each wrong answer.
2. Q. No. 13 to 20 have one or more than one correct answer and carries 5 marks each for correct answer and –1 mark for each
wrong answer.
3. Q. No. 21 to 32 are based on the given paragraph which have only one correct answer and carries 5 marks each for correct
answer and –2 marks for each wrong answer.
4. Q. No. 33 to 36 are numerical based which has answers as numerical value and each question carries 6 marks each for correct
answer and there is no negative marking.
5. Q. No. 37 to 40 are matching type questions each question has four parts and each question carries 6 marks and marks will be
awarded if all the four parts are correctly matched. No marks will be given for any wrong match in any question.
––––––––––––––––––––––––––––––––––––––––––––––––––––––––––––––––––––––––––––––––––––––––––––––––––––
4. If a steady current I is flowing through a cylindrical element
Section - A (Multiple Choice Questions with ABC. Choose the correct relationship
one correct option)
A 2r B
221 r C
1. 87 Ra is a radioactive substance having half life of 4 days.
I
Find the probability that a nucleus undergoes decay after
two half lives l/2
l/2
1 3 1
(a) 1 (b) (c) (d) (a) V AB 2VBC
2 4 4
(b) Power across BC is 4 times the power across AB
2. A solid sphere of mass M and radius R having moment of
(c) Current densities in AB and BC are equal
inertia I about its diameter is recast into a solid disc of radius
r and thickness t. The moment of inertia of the disc about an (d) Electric field due to current inside AB and BC are equal
axis passing the edge and perpendicular to the plane remains 5. Focal length of the plano-convex lens is 15 cm. A small object
I. Then R and r are related as is placed at A as shown in the figure. The plane surface is
silvered. The image will form at
2 2
(a) r R (b) r R
15 15

2 2
(c) r R (d) r R
15 15
3. The string between blocks of mass m and 2m is massless
and inextensible. The system is suspended by a massless
spring as shown. If the string is cut find the magnitudes of A 20 cm
accelerations of mass 2m and m (immediately after cutting) (a) 60 cm to the left of lens (b) 12 cm to the left of lens
(a) g, g (c) 60 cm to the right of lens(d) 30 cm to the left of lens
g 6. A binary star system consists of two stars A and B which
(b) g, have time period TA and TB, radius RA and RB and mass MA
2
and MB. Then
g (a) if TA > TB then RA > RB (b) if TA > TB then MA > MB
(c) ,g
2
2 3
2m TA RA
g g (c) (d) TA = TB
(d) , TB RB
2 2 m
EBD_780
2006-2 Target IIT-JEE
7. A massless rod of length L is suspended by two identical 10. Find the time constant (in s) for the given RC circuits in the
strings AB and CD of equal length. A block of mass m is given order respectively
suspended from point O such that BO is equal to ‘x’. Further
it is observed that the frequency of 1st harmonic in AB is
equal to 2nd harmonic frequency in CD. ‘x’ is
L
(a)
5 A C

4L
(b)
5
3L O
(c) B D
4 x L
L Vm V V
(d) R C
4 1 1

8. The graph shows relationship between object distance and R1 C1


image distance for a equiconvex
R2 lens. Then, focal length
C2 of
R1
the lens is
R2
31
v cm R2 C2
30 R1 1 , R2 2 , C1 4 F , C2 2 F

8 8
(a) 18, 4, (b) 18, ,4
10 9 9

8 8
0 (–9, +9) (c) 4, 18, (d) 4, , 18
u cm –31 –30 –20 –10 9 9

(a) 0.50 ± 0.05 cm (b) 0.50 ± 0.10 cm 11. A student performs an experiment for determination of
(c) 5.00 ± 0.05 cm (d) 5.00 ± 0.10 cm
9. In a screw gauge, the zero of mainscale coincides with fifth 4 2
division of circular scale in figure (i). The circular division of g . The error in length is and in time T is T
T2
screw gauge are 50. It moves 0.5 mm on main scale in one
rotation. The diameter of the ball in figure (ii) is and n is number of times the reading is taken. The
measurement of g is most accurate for
0 10
5 T n
0
(a) 5 mm 0.2 sec 10
(b) 5mm 0.2 sec 20
(c) 5 mm 0.1 sec. 10
Figure (i) (d) 1mm 0.1 sec 50
12. Rays of light from Sun falls on a biconvex lens of focal
0 30 length f and the circular image of Sun of radius r is formed
25
20 on the focal plane of the lens. Then
(a) Area of image is r2 and area is directly proportional of f
(b) Area of image is r2 and area is directly proportional to f 2
(c) Intensity of image increases if f is increased
Figure (ii)
(d) If lower half of the lens is covered with black paper
(a) 2.25 mm (b) 2.20 mm area will become half
(c) 1.20 mm (d) 1.25 mm
2006 IIT-JEE Solved Paper 2006-3
17. A small ball starts moving from A over a fixed track as shown
Section - B (Multiple Choice Questions with in the figure. Surface AB has friction. From A to B the ball rolls
one or more than one correct option) without slipping. Surface BC is frictionless. KA, KB and KC
are kinetic energies of the ball at A, B and C, respectively.
13. A long current carrying wire, carrying current I1 such that I1 Then
is flowing out from the plane of paper is placed at O. A C
steady state current I2 is flowing in the loop ABCD A

C
B hC
I2 hA
O
O'
I1
B
A (a) hA hC ; K B KC (b) hA hC ; KC KA
D
(c) hA hC ; K B KC (d) hA hC ; K B KC
(a) the net force is zero
18. A black body of temperature T is inside chamber of T0
(b) the net torque is zero temperature initially. Sun rays are allowed to fall from a hole
(c) as seen from O, the loop will rotate in clockwise along in the top of chamber. If the temperature of black body (T)
OO' axis and chamber (T0) remains constant, then
(d) as seen from O, the loop will rotate in anticlockwise
direction along OO' axis
14. A solid cylinder is rolling down a rough inclined plane of T0
inclination . Then T
(a) The friction force is dissipative
(b) The friction force is necessarily changing
(c) The friction force will aid rotation but hinder translation (a) Black body will absorb more radiation
(d) The friction force is reduced if is reduced (b) Black body will absorb less radiation
15. The following field line can never represent (c) Black body emit more energy
(a) induced electric field (d) Black body emit energy equal to energy absorbed by it
(b) magnetostatic field
19. The function x A sin 2 t B cos2 t C sin t cos t
(c) gravitational field of a mass at rest
represent SHM for which of the option(s)
(d) electrostatic field
(a) for all value of A, B and C (C 0)
1
16. The graph between the stopping potential (V0) and is (b) A = B, C = 2B
(c) A = –B, C = 2B
shown in the figure. 1 , 2 and 3 are work functions, (d) A = B, C = 0
20. A spherical symmetric charge system is centered at origin.
which of the following is/are correct Given, Electric potential
V0 Q Q
V r R0 , V r R0
4 0 R0 4 0r
Metal 1 Metal 2 Metal 3
v
1 2 3

1 (nm–1)
0.001 0.002 0.004

(a) 1: 2: 3 1: 2 : 4
(b) 1: 2: 3 4 : 2 :1 R0 r
hc (a) Within r = 2R0 total enclosed net charge is Q
(c) tan
e (b) Electric field is discontinued at r = R0
(d) ultravioletlight can be used to emit photoelectrons from (c) Change is only present at r = R0
metal 2 and metal 3 only (d) Electrostatic energy is zero for r < R0
EBD_780
2006- 4 Target IIT-JEE

Section - C (Passage based Questions) PASSAGE II


A cylindrical tank has a hole of diameter 2r in its bottom. The hole
INSTRUCTION : Read the passages given below and answer the is covered wooden cylindrical block of diameter 4r, height h and
questions that follows. density /3.
PASSAGE I
h1 /3
In the given circuit the capacitor (C) may be charged through 4r
resistance R by a battery V by closing switch S1. Also when S1 is h2 h
opened and S2 is closed the capacitor is connected in series with
inductor (L).

2r
Situation I : Initially, the tank is filled with water of density to a
height such that the height of water above the top of the block is
h1 (measured from the top of the block).
Situation II : The water is removed from the tank to a height h2
(measured from the bottom of the block), as shown in the figure.
The height h2 is smaller than h (height of the block) and thus the
block is exposed to the atmosphere.

21. At the start, the capacitor was uncharged. When switch S1 24. Find the minimum value of height h1 (in situation 1), for
is closed and S2 is kept open, the time constant of this which the block just starts to move up?
circuit is . Which of the following is correct 2h 5h
(a) (b)
CV 3 4
(a) after time interval , charge on the capacitor is 5h 5h
2 (c) (d)
3 2
(b) after time interval 2 , charge on the capacitor of CV
(1 – e–2) 25. Find the height of the water level h2 (in situation 2), for
which the block remains in its original position without the
(c) the work done by the voltage source will be half of the application of any external force
heat dissipated when the capacitor is fully charged
h 4h
(d) after time interval 2 , charge on the capacitor is (a) (b)
3 9
CV (1 – e–1)
2h
22. When the capacitor gets charged completely, S1 is opened (c) (d) h
3
and S2 is closed. Then,
26. In situation 2, if h2 is further decreased, then
(a) at t = 0, energy stored in the circuit is purely in the form
(a) cylinder will not move up and remains at its original
of magnetic energy
position
(b) at any time t > 0, current in the circuit is in the same
direction h
(b) for h2 = , cylinder again starts moving up
(c) at t > 0, there is no exchange of energy between the 3
inductor and capacitor
h
(d) at any time t > 0, instantaneous current in the circuit (c) for h2 = , cylinder again starts moving up
4
C
may be V h
L (d) for h2 = , cylinder again starts moving up
5
23. Given that the total charge stored in the LC circuit is Q0, for
PASSAGE III
t 0, the charge on the capacitor is
Waves y1 = A cos(0.5 x 100 t ) and y2 = A cos(0.46 x 92 t )
t t are travelling along x-axis. (Here x is in m and t is in second)
(a) Q Q0 cos (b) Q Q0 cos
2 LC 2 LC
27. Find the number of times intensity is maximum in time interval
of 1 sec.
d 2Q 1 d 2Q
(c) Q LC (d) Q (a) 4 (b) 6
dt 2 LC dt 2
(c) 8 (d) 10
2006 IIT-JEE Solved Paper 2006-5
28. The wave velocity of louder sound is shaded half region of the plate. The collision of the balls
(a) 100 m/s (b) 192 m/s with the plate is elastic. What is v?
(c) 200 m/s (d) 96 m/s b
29. The number of times y1 + y2 = 0 at x = 0 in 1 sec is
(a) 100 (b) 46
a
(c) 192 (d) 96

PASSAGE IV
v
Advanced countries are making use of powerful electromagnets
to move trains at very high speed. These trains are called maglev (Given n = 100, M = 3 kg, m = 0.01 kg; b = 2 m; a = 1 m;
trains (abbreviated from magnetic levitation). These trains float g = 10 m/s2).
on a guideway and do not run on steel rail tracks. 34. 0.05 kg steam at 373 K and 0.45 kg of ice at 253K are mixed in
Instead of using a engine based on fossil fuels, they make use of
an insulated vessel. Find the equilibrium temperature of the
magnetic field forces. The magnetized coils are arranged in the
guide way which repels the strong magnets placed in the train's mixture. Given, Lfusion = 80 cal/g = 336 J/g, Lvaporization = 540
under carriage. This helps train move over the guideway , a technic cal/g = 2268 J/g, Sice = 2100 J/Kg K = 0.5 cal/gK and Swater =
called electro-dynamic suspension. When current passes in the 4200 J/Kg K = 1 cal/gK
coils of guideway , a typical magnetic field is set up between the 35. In hydrogen-like atom (Z = 11), nth line of Lyman series has
undercarriage of train and guideway which pushes and pull the wavelength . The de-Broglie's wavelength of electron in
train along the guideway depending on the requirement. the level from which it originated is also . Find the value of
The lack of friction and its aerodynamic style allows the train to n?
move at very high speed. 36. A circular disc with a groove along its diameter is placed
30. The levitation of the train is due to horizontally on a rough surface. A block of mass 1 kg is
(a) Mechanical force (b) Electrostatic attraction placed as shown. The co-efficient of friction between the
(c) Electrostatic repulsion (d) Magnetic repulsion block and all surfaces of groove and horizontal surface in
31. The disadvantage of maglev trains is that 2
(a) More friction (b) Less pollution contact is = . The disc has an acceleration of 25 m/s2
5
(c) Less wear & tear (d) High initial cost towards left. Find the acceleration of the block with respect
32. The force which makes maglev move 4 3
(a) Gravitational field (b) Magnetic field to disc. Given cos , sin
5 5
(c) Nuclear forces (d) Air drag

Section - D (Numerical based Questions)


2
33. A rectangular plate of mass M and dimension a × b is held in 25 m/s
horizontal position by striking n small balls (each of mass
m) per unit area per second. The balls are striking in the

Section - E (Column matching type questions)

INSTRUCTION : Given below are certain matching type questions, where two columns (each having 4 items) are given.
Immediately after the columns the matching grid is given, where each item of Column I has to be matched with the items of Column
II, by encircling the correct match(es). Note that an item of column I can match with more than one item of column II. All the items of
column II must be matched.

37. Heat given to process is positive, match the following option of Column I with the corresponding option of column II :
Column I Column II
(A) JK (p) W > 0
(B) KL (q) Q < 0
(C) LM (r) W< 0
(D) MJ (s) Q>0
The matching grid :
(A) p q r s (B) p q r s (C) p q r s (D) p q r s
EBD_780
2006- 6 Target IIT-JEE
38. Given below are certain matching type questions, where two columns (each having 4 items) are given. Immediately after the
columns the matching grid is given, where each item of Column I has to be matched with the items of Column II, by encircling
the correct match(es). Note that an item of column I can match with more than one item of column II. All the items of column II
must be matched. Match the following :
Column I Column II
(A) Nuclear fusion (p) Converts some matter into energy
(B) Nuclear fission (q) Generally possible for nuclei with low atomic
number
(C) -decay (r) Generally possible for nuclei with higher atomic
number
(D) Exothermic nuclear reaction (s) Essentially proceeds by weak nuclear forces
The matching grid :
(A) p q r s (B) p q r s (C) p q r s (D) p q r s
39. Match the following columns :
Column I Column II Column
(A) Dielectric ring uniformly charged (p) Constant electrostatic field out of system
(B) Dielectric ring uniformly charged (q) Magnetic field strength
rotating with angular velocity
(C) Constant current in ring i (r) Electric field (induced)
(D) i = i0cos t (s) Magnetic dipole moment
The matching grid :
(A) p q r s (B) p q r s (C) p q r s (D) p q r s
40. A simple telescope used to view distant objects has eyepiece and objective lens of focal lengths fe and fo, respectively. Then
Column I Column II
(A) Intensity of light received by lens (p) Radius of aperture
(B) Angular magnification (q) Dispersion of lens
(C) Length of telescope (r) Focal length of objective lens and eyepiece lens
(D) Sharpness of image (s) Spherical aberration
The matching grid :
(A) p q r s (B) p q r s (C) p q r s (D) p q r s

CHEMISTRY
––––––––––––––––––––––––––––––––––––––––––––––––––––––––––––––––––––––––––––––––––––––––––––––––––––
General Instructions
1. Q. No. 41 to 52 have only one correct answer and carries 3 marks each for correct answer and –1 mark for each wrong answer.
2. Q. No. 53 to 60 have one or more than one correct answer and carries 5 marks each for correct answer and –1 mark for each
wrong answer.
3. Q. No. 61 to 72 are based on the given paragraph which have only one correct answer and carries 5 marks each for correct
answer and –2 marks for each wrong answer.
4. Q. No. 73 to 76 are numerical based which has answers as numerical value and each question carries 6 marks each for correct
answer and there is no negative marking.
5. Q. No. 77 to 80 are matching type questions each question has four parts and each question carries 6 marks and marks will be
awarded if all the four parts are correctly matched. No marks will be given for any wrong match in any question.
––––––––––––––––––––––––––––––––––––––––––––––––––––––––––––––––––––––––––––––––––––––––––––––––––––
2006 IIT-JEE Solved Paper 2006-7

Section - A (Multiple Choice Questions with 48. Ag NH 3 [Ag(NH3 )] ; k1 6.8 10 3


one correct option)
[Ag ( NH 3 )] NH 3 [Ag ( NH 3 ) 2 ] ; k 2 1.6 10 3
41. How can the following reaction be made to proceed in
forward direction? then the formation constant of [Ag(NH3)2]+ is
B(OH)3 + NaOH NaBO2 + Na[B(OH)4] + H2O (a) 6.8 × 10–6 (b) 1.08 × 10–5
(a) addition of borax (c) 1.08 × 10–6 (d) 6.8 × 10–5
(b) addition of cis -1,2-diol
49. In the following reaction,
(c) addition of Na2HPO4
(d) addition of trans -1,2-diol CH3 NH 2 CHCl3 KOH
42. A solution when diluted with H2O and boiled, gives a white Nitrogen containing compound + KCl + H2O.
precipitate. On addition of excess NH4Cl/NH4OH, the volume
The nitrogen containing compound is
of precipitate decreases leaving behind a white gelatinous
precipitate. Identify the precipitate which dissolves in (a) CH 3 NH CH 3 (b) CH 3 C N
NH4OH/NH4Cl
(a) Al (OH)3 (b) Zn(OH)2 (c) CH 3 N C (d) CH 3 N C
(c) Ca(OH)2 (d) Mg(OH)2 50. CuSO4 decolourises on addition of KCN, the product formed
43. When benzenesulfonic acid and p-nitrophenol are treated is
with NaHCO3, the gases released respectively are (a) Cu2+ get reduced to form [Cu(CN)4]3–
(a) SO2, NO (b) SO2, NO2 (b) [Cu(CN)4]2–
(c) CO2, CO2 (d) SO2, CO2 (c) CuCN (d) Cu(CN)2
44. A mono-atomic ideal gas undergoes a process in which the 51. The direct conversion of A to B is difficult, hence it is carried
ratio of P to V at any instant is constant and equals to 1. out by the following shown path :
What is the molar heat capacity of the gas C D
3R
(a) (b) 2 R
2

5R A B
(c) 0 (d) Given
2
45. The increasing order of boiling points of the below S( A C) 50 e.u. , S(C D) 30 e.u. , S( B D) 20 e.u. ,
mentioned alcohols is
(I) 1,2-dihydroxybenzene (II) 1,3-dihydroxybenzene where e.u. is the entropy unit, then S(A B) is
(III) 1,4-dihydroxybenzene (IV) Hydroxybenzene (a) +60 e.u. (b) +100 e.u.
(a) I < II < IV < III (b) I < II < III < IV (c) –60 e.u. (d) –100 e.u.
(c) IV < II < I < III (d) IV < I < II < III
52. The Haber’s process for the formation of NH3 at 298K is
46. Identify the product, P in the following reaction :
N 2 3H 2 2NH3 ; H = –46.0 kJ; Which of the
CH 3 CH CH 2 NOCl P
following is the correct statement
(a) CH 3 CH CH 2 (b) CH 3 CH C H 2 (a) The condition for equilibrium is
| | | |
NO Cl Cl NO G N2 3G H 2 2G NH3

NO where G is Gibbs free energy per mole of the gaseous


|
(c) C H 2 CH 2 CH 2 (d) species measured at that partial pressure.
CH 3 CH 2 CH
| | | (b) On adding N2, the equilibrium will shift to forward
NO Cl Cl
direction because according to IInd law of
47. The IUPAC name of C6H5COCl is thermodynamics the entropy must increase in the
(a) Benzene chloro ketone direction of spontaneous reaction
(b) Benzoyl chloride (c) The catalyst will increase the rate of forward reaction
(c) Chloro phenyl ketone by 2 times and that of backward reaction by 1.5 times
(d) Benzene carbonyl chloride (d) None of these
EBD_780
2006- 8 Target IIT-JEE
57. Refer to the figure given :
Section - B (Multiple Choice Questions with
Which of the following statements is wrong?
one or more than one correct option) gas C
gas A
53. If the bond length of CO bond in carbon monoxide is 1.128Å,
then what is the value of CO bond length in Fe(CO) 5? Z
(a) 1.15Å (b) 1.128Å 1 ideal gas
(c) 1.13Å (d) 1.118Å
54. The species present in solution when CO2 is dissolved in gas B
water are
(a) CO2, H2CO3, HCO3–, CO32–
(b) H2CO3, CO32– P
(c) CO32–, HCO3– (a) For gas A, a = 0 and Z will linearly depend on pressure
(d) CO2, H2CO3 (b) For gas B, b = 0 and Z will linearly depend on pressure
55. Which of the following reactants on reaction with conc. (c) Gas C is a real gas and we can find ‘a’ and ‘b’ if
NaOH followed by acidification gives following lactone as intersection data is given
the product? (d) All van der Waal gases will behave like gas C and give
positive slope at high pressure
O
58. The smallest ketone and its next homologue are reacted
C with NH2OH to form oxime
O (a) Two different oximes are formed
CH2 (b) Three different oximes are formed
(c) Two oximes formed are optically active
COOCH3 COOH
(d) All oximes formed are optically active
(a) (b)
COOH CHO
COOH CHO
(c) (d) 59. N(isomeric products) ;
COOH CHO C5H11Cl

56. Cl CH 2CH 2 CH3 fractional distillati on


M (isomeric products)
AlCl3 (i) O 2 / Identify N and M
P Q Phenol (a) 6, 4 (b) 6, 6
(ii) H3O
(c) 4, 4 (d) 3, 3
The major products P and Q are 60. MgSO4 on reaction with NH4OH and Na2HPO4 forms a white
crystalline precipitate. What is its formula?
(a) Mg( NH 4 )PO 4 (b) Mg 3 (PO 4 ) 2
(a) and CH3CH2CHO
(c) MgCl 2 .MgSO 4 (d) MgSO 4

Section - C (Passage based Questions)


(b) and CH3COCH3 INSTRUCTION : Read the passages given below and answer the
questions that follows.
PASSAGE I
The conversion of an amide to an amine with one carbon atom
(c) and CH3COCH3
less by the action of alkaline hydrohalite is known as Hofmann
bromamide degradation.
O O

(d) NH2 NH–Br


and CH3CH2CHO (i) (ii)
2006 IIT-JEE Solved Paper 9
2006-

O PASSAGE II
O C The coordination number of Ni2+ is 4.
N NiCl 2 KCN(excess ) A (cyano complex)
N–Br
.. NiCl 2 Conc . HCl (excess ) B (chloro complex )
(iii) (iv)
64. The IUPAC name of A and B are
HO (a) Potassium tetracyanonickelate (II), potassium
O tetrachloronickelate (II)
N NH2 (b) Tetracyanopotassiumnickelate (II),
tetrachloropotassiumnickelate (II)
H
(v) (vi) (c) Tetracyanonickel (II), tetrachloronickel (II)
In this reaction, RCONHBr is formed from which the reaction has (d) Potassium tetracyanonickel (II), potassium
derived its name. Hofmann reaction is accelerated if the migrating tetrachloronickel (II)
group is more electron-releasing. Hofmann degradation reaction is 65. Predict the magnetic nature of A and B
an intramolecular reaction. (a) Both are diamagnetic
61. How can the conversion of (i) to (ii) be brought about? (b) A is diamagnetic and B is paramagnetic with one
(a) KBr (b) KBr + CH3ONa unpaired electron
(c) KBr + KOH (d) Br2 + KOH
(c) A is diamagnetic and B is paramagnetic with two
62. Which is the rate determining step in Hofmann bromamide
unpaired electrons
degradation?
(a) Formation of (i) (b) Formation of (ii) (d) Both are paramagnetic
(c) Formation of (iii) (d) Formation of (iv) 66. The hybridization of A and B are
63. What are the constituent amines formed when the mixture (a) dsp2, sp3 (b) sp3, sp3
of (i) and (ii) undergoes Hofmann bromamide degradation? 2
(c) dsp , dsp 2 (d) sp3d2, d2sp3
15
CONH2 CONH2 PASSAGE III
Several short-lived radioactive species have been used to
&
determine the age of wood or animal fossils. One of the most
D interesting substances is 6C14 (half-life 5760 years) which is used
(i) (ii)
in determining the age of carbon-bearing materials (e.g. wood,
animal fossils, etc.). Carbon-14 is produced by the bombardment
15
NH2 NH2
15 of nitrogen atoms present in the upper atmosphere with neutrons
NH2 NH2
(from cosmic rays).
14 1 14 1
(a) , , & 7N 0n 6C 1H

D D Thus carbon-14 is oxidised to CO2 and eventually ingested by


plants and animals. The death of plants or animals put an end to
15 the intake of C14 from the atmosphere. After this the amount of
NH2 NH2 C14 in the dead tissues starts decreasing due to its disintegration
as per the following reaction :
(b) and 6C
14
7N
4
1
0

D
The C14 isotope enters the biosphere when carbon dioxide is taken
15 up in plant photosynthesis. Plants are eaten by animals, which
NH2 NH2 exhale C14 as CO2. Eventually, C14 participates in many aspects of
the carbon cycle. The C14 lost by radioactive decay is constantly
(c) and replenished by the production of new isotopes in the atmosphere.
In this decay-replenishment process, a dynamic equilibrium is
established whereby the ratio of C14 to C12 remains constant in
15
living matter. But when an individual plant or an animal dies, the C14
NHD
isotope in it is no longer replenished, so the ratio decreases as C14
decays. So, the number of C14 nuclei after time t (after the death of
(d) and living matter) would be less than in a living matter. The decay
constant can be calculated using the following formula,
EBD_780
2006-10 Target IIT-JEE
70. Calculate (ln K) for
0.693
t1/ 2
C 6 H12 O 6 2 Ag H 2O C 6 H12 O 7 2H 2 Ag
The intensity of the cosmic rays have remain the same for 30,000
(a) 55.6 (b) 29.6
years. But since some years the changes in this are observed due
to excessive burning of fossil fuel and nuclear tests. (c) 66 (d) 58.38
67. Why do we use the carbon dating to calculate the age of the 71. On adding NH3, pH of the solution increases to 11 then,
fossil? identify the effect on potential of half-cell
(a) Rate of exchange of carbon between atmosphere and
(a) Eox increased from E 0ox by 0.65 V
living is slower than decay of C14
(b) It is not appropriate to use C14 dating to determine age
(b) Eox decreased from E 0ox by 0.65 V
(c) Rate of exchange of C14 between atmosphere and living
organism is so fast that an equilibrium is set up between
the intake of C14 by organism and its exponential decay (c) Ered increased from E 0red by 0.65 V
(d) none of the above
68. What should be the age of the fossil for meaningful (d) Ered decreased from E 0red by 0.65 V
determination of its age? 72. NH3 is used in this reaction rather than any other base.
(a) 6 years Select the correct statement out of the following
(b) 6000 years
(a) [Ag(NH3)2]+ is a weaker oxidizing agent than Ag+
(c) 60,000 years
(b) To dissolve the insoluble silver oxide formed under
(d) can be used to calculate any age
the reaction conditions
69. A nuclear explosion has taken place leading to increase in
concentration of C14 in nearby areas. C14 concentration is (c) Ag precipitates gluconic acid as its silver salt
C1 in nearby areas and C2 in areas far away. If the age of the (d) NH3 changes the standard reduction potential of
fossil is determined to be T1 and T2 at the respective places [Ag(NH3)2]+
then
(a) The age of the fossil will increase at the place where
1 C1
Section - D (Subjective Problems)
explosion has taken place and T1 T2 ln
C2
(b) The age of the fossil will decrease at the place where 73. 75.2 g of C6H5OH (phenol) is dissolved in a solvent of
Kf = 14. If the depression in freezing point is 7 K then find
1 C1 the % of phenol that dimerises.
explosion has taken place and T1 T2 ln
C2
74. For the reaction, 2CO O 2 2CO 2 ; H 560kJ. Two
(c) The age of fossil will be determined to be same
moles of CO and one mole of O2 are taken in a container of
T1 C1 volume 1 L. They completely form two moles of CO2, the
(d) T2 C2 gases deviate appreciably from ideal behaviour. If the
PASSAGE IV pressure in the vessel changes from 70 to 40 atm, find the
magnitude(absolutevalue) of U at 500 K. (1 L atm = 0.1 kJ)
Tollen’s test is given by aldehydes. 75. We have taken a saturated solution of AgBr. Ksp of AgBr is
12 × 10–14. If 10–7 mole of AgNO3 are added to 1 litre of this
Ag e Ag ; E 0red 0.800 V solution find conductivity (specific conductance) of this
solution in terms of 10–7 S m –1 units. Given, Molar
C 6 H12 O 6 H 2O C 6 H12O 7 2H 2e ; E 0ox 0.05V conductance of Ag+, Br– and NO3– are 6×10–3 Sm2mol–1,
Gluconic acid
8×10–3 Sm2mol–1 and 7×10–3 Sm2mol–1.
[ Ag ( NH 3 ) 2 ] e Ag 2 NH 3 ; E 0red 0.373V 76. The edge length of unit cell of a metal having molecular
weight 75 g/mol is 5Å which crystallizes in cubic lattice. If
2.303RT F 1 the density is 2 g/cc then find the radius of metal atom
Given 0.0591 & 38.9 2 V
F RT (NA = 6 × 1023). Give the answer in pm.
2006 IIT-JEE Solved Paper 11
2006-

Section - E (Column matching type questions)


INSTRUCTION : Given below are certain matching type questions, where two columns (each having
4 items) are given. Immediately after the columns the matching grid is given, where each item of Column I
has to be matched with the items of Column II, by encircling the correct match(es). Note that an item of
column I can match with more than one item of column II. All the items of column II must be matched.
77. Match the extraction processes listed in Column I with metals listed in Column II :
Column I Column II
(A) Self reduction (p) Lead
(B) Carbon reduction (q) Silver
(C) Complex formation and displacement by metal (r) Copper
(D) Decomposition of iodide (s) Boron
The matching grid :
(A) p q r s (B) p q r s (C) p q r s (D) p q r s
78. Match the following :
Column I Column II
3
(A) Bi (BiO) (p) Heat
(B) [AlO2 ] Al(OH) 3 (q) Hydrolysis

(C) [SiO 4 ]4 [Si 2 O 7 ]6 (r) Acidification

(D) [B 4 O7 ]2 [B(OH)3 ] (s) Dilution by water


The matching grid :
(A) p q r s (B) p q r s (C) p q r s (D) p q r s
79. According to Bohr’s theory,
En = Total energy, Kn = Kinetic energy, Vn = Potential energy, rn = Radius of nth orbit
Match the following :
Column I Column II
(A) Vn / Kn = ? (p) 0
(B) If radius of nth orbit Enx , x = ? (q) –1
(C) Angular momentum in lowest orbital (r) –2
1
(D) Z y, y ? (s) 1
rn
The matching grid :
(A) p q r s (B) p q r s (C) p q r s (D) p q r s
80. Match the following :
Column I Column II
(A) C6H5CH2CD2Br on reaction with C2H5O– gives (p) E1 reaction
C6H5–CH=CD2
(B) PhCHBrCH2 and PhCHBrCD3, both react with (q) E2 reaction
the same rate
(C) C6H5CH2CH2Br on treatment with C2H5O– and (r) E1cB reaction
C2H5OD gives C6H5CD=CH2
(D) C6H5CH2CH2Br reacts faster than C6H5CD2CH2Br (s) First order reaction
on reaction with C2H5O– in ethanol
The matching grid :
(A) p q r s (B) p q r s (C) p q r s (D) p q r s
EBD_780
2006- 12 Target IIT-JEE

MATHEMATICS
––––––––––––––––––––––––––––––––––––––––––––––––––––––––––––––––––––––––––––––––––––––––––––––––––––
General Instructions
1. Q. No. 81 to 92 have only one correct answer and carries 3 marks each for correct answer and –1 mark for each wrong answer.
2. Q. No. 93 to 100 have one or more than one correct answer and carries 5 marks each for correct answer and –1 mark for each
wrong answer.
3. Q. No. 101 to 112 are based on the given paragraph which have only one correct answer and carries 5 marks each for correct
answer and –2 marks for each wrong answer.
4. Q. No. 113 to 116 are numerical based which has answers as numerical value and each question carries 6 marks each for correct
answer and there is no negative marking.
5. Q. No. 117 to 120 are matching type questions each question has four parts and each question carries 6 marks and marks will
be awarded if all the four parts are correctly matched. No marks will be given for any wrong match in any question.
––––––––––––––––––––––––––––––––––––––––––––––––––––––––––––––––––––––––––––––––––––––––––––––––––––

86. Let a, b, c be the sides of a triangle where a b c and


Section - A (Multiple Choice Questions with
R. If the roots of the equation
one correct option) x2 + 2(a + b + c)x + 3 (ab + bc + ca) = 0 are real, then

81. The value of lim (sin x )1 / x (1 x ) sin x , where x > 0 is 4 5


x 0
(a) (b)
3 3
(a) 0 (b) –1
1 5 4 5
(c) 1 (d) 2 (c) , (d) ,
3 3 3 3
x2 1 2 2
82. dx x x
x3 2 x 4 2x2 1 87. If F ( x) f g where f"(x) = –f(x) and
2 2

2 x4 2 x2 1 2 x4 2 x2 1 g(x) = f '(x) and given that F(5) = 5, then F(10) is equal to


(a) 2
c (b) 3
c (a) 5 (b) 10
x x
(c) 0 (d) 15
2 x4 2 x2 1 2 x4 2 x2 1 88. If the LCM of p, q is r2t4s2, where r, s, t are prime numbers
(c) c (d) c and p, q are the positive integers then the number of ordered
x 2 x2
pair (p, q) is
83. One angle of an isosceles is 120º and radius of its incircle
(a) 252 (b) 254
3 . Then the area of the triangle in sq. units is (c) 225 (d) 224
(a) 7 12 3 (b) 12 7 3
89. Let 0, an d t 1 = (tan ) tan , t 2 = (tan ) cot ,
(c) 12 7 3 (d) 4 4
t3 = (cot )tan and t4 = (cot )cot , then
84. The values of (0, 2 ) for which 2 sin 2 – 5 sin + 2 > 0,
(a) t1 > t2 > t3 > t4 (b) t4 > t3 > t1 > t2
are (c) t3 > t1 > t2 > t4 (d) t2 > t3 > t1 > t4
5 5 90. The axis of a parabola is along the line y = x and the distances
(a) 0, ,2 (b) ,
6 6 8 6 of its vertex and focus from origin are 2 and 2 2
respectively. If vertex and focus both lie in the first quadrant,
5 41 then the equation of the parabola is
(c) 0, , (d) , (a) (x + y)2 = (x – y – 2) (b) (x – y)2 = (x + y – 2)
8 6 6 48
2
(c) (x – y) = 4 (x + y – 2) (d) (x – y)2 = 8 (x + y – 2)
w wz 91. A plane which is perpendicular to two planes 2x – 2y + z = 0
85. If is purely real where w = +i , 0 and z 1, and x – y + 2z = 4, passes through (1, –2, 1). The distance of
1 z
then the set of the values of z is the plane from the point (1, 2, 2) is
(a) 0 (b) 1
(a) {z : |z| = 1} (b) {z : z = z }
(c) {z : z 1} (d) {z : |z| = 1, z 1} (c) 2 (d) 2 2
2006 IIT-JEE Solved Paper 2006- 13
98. f(x) is cubic polynomial with f(2) = 18 and f(1) = –1. Also
92. Let a iˆ 2 ˆj kˆ, b iˆ ˆj kˆ and c iˆ ˆj kˆ . A f(x) has local maxima at x = –1 and f '(x) has local minima at
x = 0, then
vector in the plane of a and b whose projection on c is
(a) the distance between (–1, 2) and (a f(a)), where x = a is
1 the point of local minima is 2 5
, is
3
(b) f(x) is increasing for x [1, 2 5 ]
(a) 4iˆ ˆj 4kˆ (b) 3iˆ ˆj 3kˆ
(c) f(x) has local minima at x = 1
ˆj 2kˆ ˆj 4kˆ (d) the value of f(0) = 15
(c) 2iˆ (d) 4iˆ
99. Let A be vector parallel to line of intersection of planes
Section - B (Multiple Choice Questions with P 1 and P 2 . Plane P 1 is parallel to the vectors
one or more than one correct option(s). 2 ˆj 3kˆ and 4 ˆj 3kˆ and that P2 is parallel to
ˆj kˆ and 3iˆ 3 ˆj , then the angle between vector A and a
93. The equations of the common tangents to the parabola y =
x2 and y = – (x – 2)2 is/are given vector 2iˆ ˆj 2kˆ is
(a) y = 4 (x – 1) (b) y = 0
(c) y = –4 (x – 1) (d) y = –30x – 50
(a) (b)
94. If f(x) = min {1, x2, x3}, then 2 4
(a) f(x) is continuous x R
(b) f'(x) > 0, x > 1 3
(c) (d)
(c) f(x) is continuous but not differentiable x R 6 4
(d) f(x) is not differentiable at two points
95. A curve y = f (x) passes through (1, 1) and at P(x, y), tangent ex , 0 x 1 x
cuts the x–axis and y–axis at A and B respectively such that 100. Let f ( x ) 2 ex 1
, 1 x 2 and g( x) f (t ) dt, x [1, 3]
BP : AP = 3 : 1, then x e, 2 x 3 0
(a) equation of curve is xy' – 3y = 0
(b) normal at (1, 1) is x + 3y = 4 then g(x) has
(c) curve passes through (2, 1/8) (a) local maxima at x = 1 + ln 2 and local minima at x = e
(d) equation of curve is xy' + 3y = 0
(b) local maxima at x = 1 and local minima at x = 2
96. Let a hyperbola passes through the focus of the ellipse
(c) no local maxima
x2 y 2 (d) no local minima
1 . The transverse and conjugate axes of this
25 16
hyperbola coincide with the major and minor axes of the
Section - C (Passage based Questions)
given ellipse, also the product of eccentricities of given
ellipse and hyperbola is 1, then INSTRUCTION : Read the passages given below and answer the
questions that follows.
x2 y2
(a) the equation of hyperbola is 1 PASSAGE I
9 16
There are n urns, each of these contain n + 1 balls. The ith urn
2 2 contains i white balls and (n + 1 – i) red balls. Let ui be the event
x y
(b) the equation of hyperbola is 1 of selecting ith urn, i = 1, 2, 3 .........., n and w the event of getting
9 25
a white ball.
(c) focus of hyperbola is (5, 0) 101. If P(ui) i, where i = 1, 2, 3,......., n, then lim P( w)
n
(d) vertex of hyperbola is (5 3 , 0)
(a) 1 (b) 2/3
97. In ABC, internal angle bisector of A meets side BC in D. (c) 3/4 (d) 1/4
DE AD meets AC in E and AB in F. Then 102. If P(ui) = c, (a constant) then P(un/w) =
2bc A 1
(a) AE is HM of b & c (b) AD cos (a) (b)
b c 2 n 1 n 1

4bc A n 1
(c) EF sin (d) AEF is isosceles (c) (d)
b c 2 n 1 2
EBD_780
2006- 14 Target IIT-JEE
107. If P is any point of C1 and Q is another point on C2, then
1
103. Let P(ui ) , if n is even and E denotes the event of
n PA2 PB 2 PC 2 PD 2
is equal to
choosing even n umbered urn, then the value of QA2 QB 2 QC 2 QD 2
P(w / E) is
(a) 0.75 (b) 1.25
n 2 n 2 n 1
(a) (b) (c) (d) (c) 1 (d) 0.5
2n 1 2(n 1) n 1 n 1
108. If a circle is such that it touches the line L and the circle C1
externally, such that both the circles are on the same side of
PASSAGE II the line, then the locus of centre of the circle is
(a) ellipse (b) hyperbola
Let the definite integral be defined by the formula (c) parabola (d) pair of straight line
b 109. A line L' through A is drawn parallel to BD. Point S moves
b a
f ( x )dx ( f (a) f (b )) . For more accurate result for such that its distances from the line BD and the vertex A are
2
a equal. If locus of S cuts L' at T2 and T3 and AC at T1, then
b c b area of T1T2T3 is
c (a, b), we can use f ( x ) dx f ( x )dx f ( x )dx F (c ) so
a a c 1 2
b (a) sq. units (b) sq. units
a b b a 2 3
that for c , we get f ( x )dx f (a ) f (b) 2 f (c ) .
2 4
a (c) 1 sq. units (d) 2 sq. units
/2
104. sin x dx =
0 PASSAGE IV

(a) 1 2 (b) 1 2 1 0 0
8 4
Let A 2 1 0 , and U1, U2 and U3 are columns of a 3 × 3 matrix
3 2 1
(c) (d)
8 2 4 2
U. If column matrices U 1 , U 2 and U 3 satisfying
x
x a
f ( x) dx f ( x) f (a)
2 1 2 2
a
105. If lim 3
0 , then f(x) is
AU1 0 , AU 2 3 , AU 3 3 evaluate as directed in the
x a ( x a)
of maximum degree 0 0 1
(a) 4 (b) 3
following questions.
(c) 2 (d) 1
106. If f "(x) < 0 x (a, b) and c is a point such that a < c < b,
and (c, f (c)) is the point lying on the curve for which F(c) is 110. The value |U| is
maximum, then f '(c) is equal to
3
f (b ) f ( a ) 2 f (b) f (a) (a) 3 (b) –3 (c) (d) 2
(a) (b) 2
b a b a
111. The sum of the elements of the matrix U–1 is
2 f (b) f (a)
(c) (d) 0 (a) –1 (b) 0 (c) 1 (d) 3
2b a

3
PASSAGE III
112. The value of [3 2 0] U 2 is
0
ABCD is a square of side length 2 units. C1 is the circle touching
all the sides of the square ABCD and C2 is the circumcircle of
square ABCD. L is a fixed line in the same plane and R is a fixed 5 3
point. (a) 5 (b) (c) 4 (d)
2 2
2006 IIT-JEE Solved Paper 15
2006-

Section - D (Subjective Problems) 3 æ 3ö æ 3ö


2 3
n-1 æ 3 ö
n
115. If an = -ççç ÷÷÷ + ççç ÷÷÷ + .....(-1) ççç ÷÷÷ and
113. Let a and b be the roots of the equation x2 – 10cx – 11d = 0 4 è 4ø è 4ø è4ø
and those of x2 – 10ax – 11b = 0 are c, d then the value of
bn = 1 – an, then find the least natural number n0 such that
a + b + c + d, when a b c d, is.
bn > an " n ³ n0 .
1
116. For a twice differentiable function f (x), g(x) is defined as
(1 x 50 )100 dx
g(x) = (f '(x) 2 + f"(x)) f(x) on [a, e]. If for
114. The value of 5050 01 is. a < b < c < d < e, f (a) = 0, f (b) = 2, f (c) = –1, f (d) = 2,
50 101 f (e) = 0 then find the minimum number of zeros of g(x).
(1 x ) dx
0

Section - E (Column matching type questions)


INSTRUCTION : Given below are certain matching type questions, where two columns (each having 4 items) are given. Immediately
after the columns the matching grid is given, where each item of Column I has to be matched with the items of Column II, by
encircling the correct match(es). Note that an item of column I can match with more than one item of column II. All the items of
column II must be matched.
117. Match the following :
(3, 0) is the pt. from which three normals are drawn to the parabola y2 = 4x which meet the parabola in the points P, Q and R. Then
Column I Column II
(A) Area of PQR (p) 2
(B) Radius of circumcircle of PQR (q) 5/2
(C) Centroid of PQR (r) (5/2, 0)
(D) Circumcentre of PQR (s) (2/3, 0)
The matching grid :

(A) p q r s (B) p q r s (C) p q r s (D) p q r s

118. Match the following : Colu


Column I Column II

/2
(A) (sin x)cos x cos x cot x log(sin x)sin x dx (p) 1
0

(B) Area bounded by –4y2 = x and x – 1 = –5y2 (q) 0


(C) Cosine of the angle of intersection of (r) 6 ln 2
curves y = 3x – 1 log x and y = xx – 1is

dy 6 4
(D) Let where y(0) = 0 then value (s)
dx x y 3

of y when x + y = 6 is
The matching grid :
(A) p q r s (B) p q r s (C) p q r s (D) p q r s
EBD_780
2006- 16 Target IIT-JEE
119. Match the following :
Column I Column II
(A) Two rays x + y = |a| and ax – y = 1 intersects (p) 2
each other in the first quadrant in the
interval a (a0, ), the value of a0 is
4
(B) Point ( , , ) lies on the plane x + y + z = 2. (q)
3

Let a iˆ ˆj kˆ, kˆ (kˆ a ) 0, then =

1 0 1 0
(C) 1 y 2 dy y 2 1 dy (r) 1 xdx 1 xdx
0 1 0 1

(D) If sinA sinB sinC + cosA cosB = 1, (s) 1


then the value of sinC =

The matching grid :

(A) p q r s (B) p q r s (C) p q r s (D) p q r s

120. Match the following


Column I Column II
1
(A) tan 1 t , then tan t = (p) 1
i 1 2i 2

5
(B) Sides a, b, c of a triangle ABC are in AP and (q)
3
a b c
cos 1 , cos 2 , cos 3 ,
b c a c a b

then tan 2 1
tan 3
2 2

2
(C) A line is perpendicular to x + 2y + 2z = 0 and (r)
3
passes through (0, 1, 0). The perpendicular
distance of this line from the origin is

The matching grid :

(A) p q r (B) p q r (C) p q r (D) p q r


EXPLANATORY NOTES – IIT-JEE 2006
When the string is cut :
1. (b) Radioactive decay is a random process. Each decay is
For mass m :
a completely independent event.
Therefore, which particular nucleus will decay at a given Fnet = mam mg = mam am = g (downwards)
instant of time cannot be predicted. In other words For mass 2m :
when a particular nucleus will decay cannot be Fnet = 2ma2m
predicted. Each nucleus has same probability of 2mg – T = 2ma2m
disintegration.
g
2mg – 3mg = 2ma2m a2m =
2. (b) A' Y 2
The negative sign indicates that the acceleration is in
M B upwards direction.
M Alternatively :
r
In situation 1, the tension T has to hold both the masses
R 2m and m therefore,
A
T = 3mg
B' Y' In situation 2, when the string is cut, the mass m is a
freely falling body and its acceleration due to gravity
For solid sphere is g.
2 For mass 2m, just after the string is cut, T remains 3mg
IAB = MR 2 I (given) ... (i) because of the extension of string (or otherwise it would
5
quickly changed to 2mg).
For solid disc
3mg – 2mg = 2m × a
1 3
IA'B' = IYY' + Mr2 = Mr 2 Mr 2 Mr 2 g
2 2 a
IAB = IA'B' (given) ... (ii) 2
From (i) and (ii),
4. (b) A 2r B
2 3
C
MR 2 Mr 2 I
5 2 r
l/2
2 l/2
r= R
15
3. (c) Just before the string is cut by equilibrium of mass m,
T ' = mg ... (i) (a)
VAB I AB R AB R AB 2[ 4r 2 ] 1
By equilibrium of mass 2m, T = 2mg + T' ... (ii) VBC I BC RBC RBC 4
From (i) and (ii), T = 2mg + mg = 3mg ... (iii) 2[ r 2 ]

[IAB = IBC, wire is of same material]


Therefore option (a) is incorrect.

PBC I 2 RBC 2[ 4r 2 ] 1
(b) 2
T PAB I RAB 4
T
2m 2m 2[ r 2 ]
2mg
T' PAB = 4PBC; Therefore (b) is correct.
2mg
T' I
m m
(c)
J AB 4r 2 1
; Therefore (c) is incorrect.
J BC I 4
mg mg
r2
Situation 1 Situation 2
EBD_780
2006- 18 Target IIT-JEE
7. (a) Frequency of Ist harmonic of AB
V AB
E AB /2 1 1 TAB
(d) ; Therefore (d) is incorrect. =
E BC VBC 4 2 m
/2
1 TCD
5. (b) The focal length f of the equivalent mirror is Frequency of 2nd harmonic of CD =
m
1 2 1 2 1 15 Given that the two frequencies are equal.
f f1 fm 15 f cm
2 1 TAB 1 TCD
2 m m

TAB
= TCD
4
TAB = 4TCD ... (i)

A B
20 cm

AB CD
2
TAB TCD
Since f has a positive value, the combination behaves
as a converging mirror. O
The sign convention for the above formula is B x L–x D
(a) Focal length of converging lens or mirror is
positive. m
(b) Focal length of diverging lens or mirror is negative. L

15 For rotational equilibrium of massless rod, taking torque


Here u = – 20cm, f cm
2 about point O.
[– ve sign because the equivalent mirror is converging], TAB × x = TCD (L – x) ... (ii)
v=? For translational equilibrium,
According to mirror formula TAB + TCD = mg ... (iii)
1 1 1 On solving, (i) and (iii), we get
v u f mg
TCD = ;
1 1 1 5
v = – 12 cm
v 20 15 / 2 4mg
Negative sign indicates that the image is 12 cm in front TAB =
5
of mirror. Substituting these values in (ii), we get
6. (d) The gravitational force of attraction between the stars
will provide the necessary centripetal forces. 4mg mg
x ( L x)
In this case angular velocity of both stars is the same. 5 5
2 4x = L – x
Therefore time period remains the same. .
T L
x=
5
8. (c) We know that in case of a convex lens when object is
placed at C ', the image is obtained at C. This situation
RA is represented in the graph by the point corresponding
MB MA
RB to u = –10 cm, v = 10 cm.
R
Therefore R = 10 cm 5cm f
2
2006 IIT-JEE Solved Paper 19
2006-

31 Parallel
v cm Rays from
Focal object placed
30 plane at infinity
f
A

on focal plane
2r C

Image formed
10
B

0 (–9, +9) Biconvex lens


u cm –31 –30 –20 –10

Lens formula is 13. (a, c) Net force on the loop :


Force on AB : The magnetic field due to current I1 is
1 1 1 f v u
2 2 2
along AB.
f v u f v u
dF = I (d B sin 0 ) 0
(for maximum error in f)
f 0.1 0.1
C
2
25 (10) (10) 2
B
f = 25 × 0.1 × 2 × 0.01 = 0.05 I2
Therefore, the focal length = (5.00 ± 0.05) cm.
O
0.5 O'
9. (c) Least count = 0.01mm
50 I1
Zero error = 5 × L.C
= 5 × 0.01 mm A
= 0.05 mm
D
Diameter of ball = [Reading on main scale] + [Reading
on circular scale × L . C] – Zero error
= 0.5 × 2 + 25 × 0.01 – 0.05 Force on CD : Similarly the magnetic field due to current I1
is along DC. Because = 180° here, therefore force on DC is
= 1.20 mm
zero.
10. (b) Time constant of R – C circuit is = Req Ceq
(i) 1 = (2 + 1) (2 + 4) = 18 µs
B1 I2
2 1 2 4 8 C
(ii) 2 µs
2 1 2 4 9
d
2 1 B
(iii) 3 (4 2) 4µs
2 1
Force on BC : Consider a small element d .
g T
11. (d) 2 dF = I 2 d B1sin 90 dF = I 2 d B1
g T
By Fleming's left hand rule, the direction of this force is
and T are least and number of readings are perpendicular to the plane of the paper directed outwards.
maximum in option (d), therefore the measurement of g
is most accurate with data used in this option. Force on AD : dF I 2 d B1sin 90 I 2 d B1

AB
12. (b) From the figure in ABC, tan B1
AC D
AB = AC tan
2r = f tan d
Area of image = r2 f 2 A I2
EBD_780
2006- 20 Target IIT-JEE
By Fleming's left hand rule, the direction of this force is Comparing equation (i) by y = mx + c, we get the slope
perpendicular to the plane of paper directed inwards. Since
hc
the current elements are located symmetrical to current I1, of the line m tan
e
therefore force on BC will cancel out the effect of force on
AD. Option (c) is correct.
Net force on loop ABCD is zero. From the graph it is clear that,
Net Torque on the loop : The force on BC and AD will create 1 –1
a torque on ABCD in clockwise direction about OO' as seen 0.001 nm
01
by the observer at O.
14. (c, d) As shown in the figure, the component of weight
1
mg sin tends to slide the point of contact (of the cylinder 01 1000mn
0.001
with inclined plane) along its direction. The sliding friction
acts in the opposite direction to oppose this relative motion.
1 –1
Because of frictional force the cylinder rolls. Also 0.002 nm
02
N
02 500nm
f
and 03 250nm
Violet colour light will have wavelength less than
sin mg cos 400 nm.
mg mg
Therefore, this light will be unable to show photoelectric
effect on plate 3 Option (d) is wrong.
Thus frictional force adds rotation but hinders translational
motion. 17. (a, b) At point A, potential energy of the ball = mghA
Applying Fnet = ma along the direction of inclined plane, At point B, potential energy of the ball = 0
weget mg sin – f = mac, At point C, potential energy of the ball = mghC
where ac = acceleration of centre of mass of the cylinder Total energy at point A, EA = KA + mghA
f = mg sin – mac … (i) Total energy at point B, EB = KB
Total energy at point C, EC = KC + mghC
g sin g sin 2 According to the law of conservation of energy.
But ac = g sin ... (ii)
Ic 2
mR / 2 3 EA = EB = EC ... (i)
1 1
mR 2 mR 2 EA = EB EC > KC ...(ii)
EA = EC
mg sin KA + mghA = KB + mghC
From (i) and (ii), f =
3
KC KA
If is reduced, frictional force is reduced. or, hA hC ...(iii)
mg
15. (c, d) Out of the given options only induced electric field
and magnetostatic field form closed loops of field lines. hA hC ; KC K A ...(iv)
16. (a, c) : : eV01 : eV02 : eV03 Option (b) is correct
1 2 3
From (i), (ii) and (iv), we get hA > hC; KB > KC Option (a) is correct
V01 : V02 : V03 0.001: 0.002 : 0.004 1: 2 : 4 18. (a, c, d) Since sun rays fall on the black body, it will absorb
Therefore option (a) is correct more radiation and since, its temperature is constant it will
V0 emit more radiation. The temperature will remain same only
when energy emitted is equal to energy absorbed.
Metal 1 Metal 2 Metal 3
19. (a, b, c) The given equation is
x = A sin2 t + B cos2 t + C sin t cos t
–1 Rearranging the equation in a meaningful form (for
1/ (nm )
0.001 0.002 0.004 interpretation of SHM)
hc A B C
By Einstein’s photoelectric equation, – eV x= (2sin 2 t ) (2 cos 2 t ) (2sin t cos t )
2 2 2
hc A B C
V – ...(i) = [1 cos 2 t ] [1 cos 2 t ] [sin 2 t ]
e e 2 2 2
2006 IIT-JEE Solved Paper 21
2006-

C V
(a) For A = 0 and B = 0, x = sin (2 t )
2
C R
The above equation is that of SHM with amplitude C S1
2
and angular frequency 2 . Thus option (a) is correct.
(b) If A = B and C = 2B then x = B + B sin 2 t
S2
This is equation of SHM. The mean position of the
particle executing SHM is not at the origin. L
Option (b) is correct. [x = B = x' = b sin 2 t] Therefore, Q0 = CV.
(c) A = – B, C = 2B; Therefore 2
x = B cos 2 t + B sin 2 t When t = 2 , Q = CV 1 e CV [1 e 2 ]
Let B = X cos = X sin then
x = X sin 2 t cos + X cos 2 t sin
This represents equation of SHM. 22. (d) The instantaneous charge on plates at any time t during
(d) A = B, C = 0 and x = A. This equation does not represents discharging is
Q = Q0 cos t
SHM.
Instantaneous current,
20. (a,b,d)
(a) The whole charge Q will be enclosed in a sphere of dQ
I= Q0 sin t
diameter 2R0. dt
(b) Electric field E = zero inside the sphere. Hence electric _ C
+ I
field is discontinued at r = R0.
(c) Charges in and E are continuously present for r > R0.
Option (c) is incorrect. S2
(d) For r < R0, the potential is constant and the electric
intensity is zero. Obviously, the electrostatic energy is
zero for r < R0.
L
The magnitude of maximum current
Imax = Q0
1
Here Q0 = CV and =
LC

R0 1 C
r Imax = CV × =V
LC L
+ ++++++ +
+ 23. (c) Apply Kirchhoff 's law in the circuit
++
++++++

Q dI Q dI
L
++++++

R0 L 0
C dt C dt

d dQ d 2Q
++

Q = LC LC
dt 2
+
+
+++++
++ dt dt
24. (c) Consider the equilibrium of wooden block.
E Forces acting in the downward direction are

P1 (2r )2

R0 r

21. (b) For charging of R – C circuit, Q = Q0 [1 – e–t/ ] when


the charging is complete, the potential difference
between the capacitor plates will be V. The charge
stored in this case will be maximum. P1 P0 W P2
EBD_780
2006- 22 Target IIT-JEE
(a) Weight of wooden cylinder 27. (a); 28. (c); 29. (d)
The equations are y 1 = A cos (0.5 x – 100 t) and
h
= (2r )2 h g 4r 2 g y2 = A cos (0.46 x – 92 t) represents two progressive wave
3 3
travelling in the same direction with slight difference in the
(b) Force due to pressure (P1) created by liquid of height frequency. This will give the phenomenon of beats.
h1 above the wooden block is
Comparing it with the equation
= P1 × (2r)2 = [P0 + h1 g] × (2r)2
y = A cos (kx – t), we get
Force acting on the upward direction due to pressure
P 2 exerted from below the wooden block and 1 = 100 2 f1 = 100 f1 = 50 Hz and
atmospheric pressure is
2
K1 = 0.5 0.5 1=4m
2 2 2
P2 (2r ) r P0 (r ) 1

P0 ( h1 h ) g 3r 2 P0 r 2 Wave velocity = 1f1 = 200 m/s [Alternatively use v =


K
]
At the verge of rising
2 = 92 2 f2 = 92 f2 = 46 Hz
2 2
P0 (h1 h) g 3r r P0 Therefore beat frequency = f1 – f2 = 4 Hz and

2 200
4r 2 h g K2 = 0.46 0.46 2=
2 46
P0 h1 g 4 r 2
3

5h 200
or, h1 Wave velocity = ×46 = 200 m/s
3 46

25. (b) Considering equilibrium of wooden block. Wave velocity is same because it depends on the medium in
Total downward force = Total force upwards which the wave is travelling.

Wt. of block + force due to atmospheric pressure = Now, at x = 0,


Force due to pressure of liquid + Force due to y1 + y2 = (A cos 10 t) + (A cos 92 t) = 0
atmospheric pressure cos 100 t = – cos 92 t = cos (– 92 t)
= cos [(2n + 1) – 92 t
(16r 2 ) g P0 16r 2
3
2n 1
= [h2 g + P0] [(16 – 4)r2] + P0 × 4r2 t=
192

(16r2) h g = h2 g × × 12r2 1
3 when t = 0, n = and when t = 1,
2
h
16 12h2
3 191
n= 95.2
2
4
h h2 net amplitude is zero for n = 96 times (the nearest
9
answer).
26. (a) When the height h2 of water level is further decreased, 30. (d) The magnetised coils running along the track repel large
then the upward force acting on the wooden block magnets on the train's under carriage.
decreases. The total force downward remains the same.
31. (d) Initial cost will be more.
This difference will be compensated by the normal
32. (b) The magnetic force will pull the vehicle.
reaction by the tank wall on the wooden block. Thus
33. Since the plate is held horizontal therefore net torque acting
the block does not moves up and remains at its original
on the plate is zero.
position.
2006 IIT-JEE Solved Paper 2006-23
34. (1) Heat lost by steam at 100°C to change to 100°C water
m
F mLvap = 0.05 × 2268 × 1000 = 1,13,400 J
3b/4 (2) Heat lost by 100°C water to change to 0°C water
= 0.05 × 4200 × 100 = 21,000 J
(3) Heat required by 0.45 kg of ice to change its temperature
a from 253 K to 273 K
= m × Sice × T = 0.45 × 2100 × 20 = 18,900 J
(4) Heat required by 0.45 kg ice at 273 K to convert into
b/2 0.45 kg water at 273 K
Mg = mLfusion = 0.45 × 336 × 1000 = 151,200 J
m'
From the above data it is clear that the amount of heat
b 3b required by 0.45 kg of ice at 253 K to convert into 0.45 kg of
Mg × F ... (i)
2 4 water at 273 K (1,70,100 J) cannot be provided by heat lost
by 0.05 kg of steam at 373 K to convert into water at 273 K.
dp b
F= n (Area) = n × (2mv) × a × ... (ii) Therefore the final temperature will be 273 K or 0°C.
dt 2 35. nth line of Lyman series means electron jumping from
From (i) and (ii) (n + 1)th orbit to 1st orbit.
b b 3b For an electron to revolve in (n + 1)th orbit.
Mg × = n × (2mv) × a × ×
2 2 4 2 r = (n + 1)
3 2 2 2 2
3 × 10 = 100 × 2 × 0.01 × v × 1 × r –10 ( n 1)
4 (n 1) (n 1) 0.529 10
v = 10 m/s Z
Alternatively : 1 Z
b 2 0.529 10 –10 n 1 ..(i)
Torque due to weight of plate, 1 = Mg ×
2
torque due to small element shown dotted in the figure Also we know that when electron jumps from (n + 1)th orbit
dp to 1st orbit.
= x × dF = x n a dx
dt 1 1 1 1
RZ 2 – 1.09 107 Z 2 1–
2 2
x dx 1 (n 1) (n 1)2
...(ii)
From (i) and (ii)

Z 1
–10
1.09 107 Z 2 1–
2 (0.529 10 )( n 1) (n 1)2
On solving, we get n = 24
Total torque, 36. Applying pseudo force ma and resolving it.
b a 2 b2 Applying Fnet = max for x-direction
2=
n(2mv) ax dx 2mnv b
b/2 2 4 ma cos – (f1 + f2) = max
ma cos – µN1 –µN2 = max
a 3b2
= 2nmv ma cos – µma sin – µ mg = max
2 4
As 1 = 2 ax = a cos – µa sin – µg

a 3b2 4 2 3 2
Mg
b
2nmv = 25 25 10 10 m / s 2
2 2 4 5 5 5 5
37. (A) (q)
a 3b
Mg = 2nm v JK is a isovolumic process. Therefore work done is zero.
2 2
But there is decrease in pressure. Now Q = U + W.
4 Mg 4 3 10 Therefore Q = U. In this case U = nCv T and P T.
v= Since pressure has decreased means temperature has
2nma 3b 2 100 0.01 1 3 2
= 10 m/s decreased. Therefore U is negative and so is Q.
EBD_780
2006- 24 Target IIT-JEE
(B) (p, s) 43. (NA)None of the given options is correct. Benzene sulphonic
KL is a isobaric process. Pressure is constant. The volume acid, being stronger acid than carbonic acid, would
is increasing therefore W > 0. Also there is an increase in liberate CO2 when treated with sodium bicarbonate,
temperature. For both the case heat is absorbed. Therefore but p-nitrophenol, being less acidic than carbonic acid,
Q > 0. will not liberate CO2.
(C) (s) 44. (b) In general, the molar heat capacity for any process is
LM is a isovolumic process. Therefore work done is zero. given by
The process is accompanied by increases in pressure. In
this case, the temperature has increased and therefore U > R
C Cv , when PVn = constant
0. Therefore Q > 0. 1 n
(D) (q, r) P
The process MJ is accompained with decrease in volume. Here 1 , i.e. PV–1 = constant
V
Therefore W < 0. Also from the graph we can conclude
that the temperature in the process decreases. Therefore 3
For monoatomic gas, Cv R
U is also negative 2
Q < 0.
3 R 3 R 4R
38. A-p, q; B-p, r; C-p, s; D-p, q, r C R R 2R .
In a nuclear fusion reaction matter is converted into energy 2 1 ( 1) 2 2 2
and nuclei of low atomic number generally given this reaction. 45. (d) Among the given compounds, hydroxybenzene (IV)
In a nuclear fission reaction matter is converted into energy has least molar mass and therefore possess least boiling
and nuclei of high atomic number generally given this point. Among the three isomeric dihydroxybenzenes,
reaction. 1,2-dihydroxybenzene (I) forms intramolecular
39. A-p; B-r, s; C-q, s; D-q, r, s H-bonding with the result it will not form intermolecular
(A) Charge on ring will create electric field which is time H-bonding leading to lowest boiling point. On
independent. the other hand 1,3-dihydroxybenzene (II) and 1,
(B) The rotating charge is like a current. This will create a 4-dihydroxybenzene (III) do not undergo chelation,
magnetic field and a magnetic moment. hence they will involve extensive intermolecular
(C) Since net charge is zero there will be no time H-bonding leading to higher boiling point. Further
independent electric field. The current produces intermolecular hydrogen bonding is stronger in the
magnetic field and magnetic moment.
p-isomer than the m-isomer hence former has highest
(D) A changing magnetic field will be produced. This will
b.p. Thus the decreasing order of boiling points is
create an induced electric field. Also a changing
III > II > I > IV.
magnetic moment will be produced.
40. (A) (p). 46. (b) Nitrosyl chloride adds on olefins according to
Markovnikof’s rule, where NO+ constitutes the positive
More the radius of aperture more is the amount of light
part of the addendum.
entering the telescope.
f0 CH3 CH CH 2 NOCl CH 3 CH CH 2
(B) (q). M= | |
fe Cl NO
(C) (r). L = f0 + fe
47. (b) RCOX like derivative of carboxylic acids are named
(D) (p), (q), (s). as oyl chlorides.
Depends on dispersion of lens, spherical aberration and
48. (b) The required reaction is
radius of aperture.
41. (b) cis-1,2-diol forms chelated complex ion with the Ag 2 NH 3 [Ag( NH 3 ) 2 ] ; K ?
product, [B(OH)4]– causing the reaction to proceed in
From the given equations, we have
forward direction.
CH2–OH

HO–CH2
– [Ag(NH3 )] [Ag(NH 3 ) 2 ]
+
HO OH
+
CH2–O O–CH2 k1 ; k2
B B [Ag ][NH3 ] [Ag(NH 3 )] [NH 3 ]
CH2–OH HO OH HO–CH2 CH2–O O–CH2
Stable chelated complex ion The value of K is given by
42. (b) Precipitate of Zn(OH)2 formed at initial stage dissolves K = k1 × k2 = 6.8 × 10–3 × 1.6 × 10–3 = 1.08 × 10–5.
in excess of NH4OH due to the formation of tetrammine 49. (c) This is an example of carbylamine reaction
Zn (II) complex.
CHCl3 RNH 2 KOH
Zn 2 2 NH 4 OH Zn ( OH ) 2 2 NH 4
R N C 3KCl 3H 2 O
Zn(OH) 2 4NH 4 [Zn(NH 3 ) 4 ]2 2H 2 O 2H
2006 IIT-JEE Solved Paper 2006- 25

50. (a) CuSO 4 2KCN Cu(CN) 2 K 2SO 4 Pb


57. (b) For gas A, a = 0, Z = 1 implies Z varies linearly
RT
1 with pressure.
Cu(CN) 2 CuCN (CN)2
unstable 2
a
3 For gas B, b = 0, Z = 1 . Hence, Z does not vary
CuCN + 3 KCN K 3 [Cu (CN ) 4 ] VRT
colourless linearly with pressure.
Given the intersection data for gas C, it is possible to
51. (a) S( A B) S( A C) S(C D) S( B D) find the values of ‘a’ and ‘b’. All vander Waal gases,
= 50 + 30 – 20 = 60 e.u. like gas C, give positive slope at high pressures.
52. (a) In a reversible reaction, catalyst speeds up both the
forward and backward reactions to the same extent, so O
(c) is wrong. At equilibrium, 58. (b) C + NH2OH
G G products G reactants 0 H3C CH3

2G NH3 (G N 2 3G H 2 ) 0 OH HO
O N N
or 2G NH3 G N2 3G H 2
+ NH OH C C
53. (d) The metal to ligand bonding creates a synergic effect
which strengthens the bond between CO and theH 3C
metal CH3 H3C CH3 H3C CH3
(Fe) leading to contraction of bond between C and O (same compound)
of CO.
54. (a) CO 2 H2O H2CO3 O
+ NH2OH
H HCO 3 H CO 32 C
– H3C C2H5
CHO – COO
OH
55. (d) intramolecular
Cannizzaro reaction OH HO
CHO CH2OH
O N N
O + NH OH +
C C
+ COOH
H + C H3C H3C C2H5 H5C2 CH3
H O + H2O
CH2OH (syn) (anti)
CH2 optically active oximes

Cl2
59. (a)
AlCl3
56. (c) + CH3CH2CH2Cl

Isopropylbenzene
CH2Cl Cl
Cl
(cumene), (P) Cl
, , ,
O–O–H d, l d, l CH2Cl
I II + III IV + V VI
O2 H2O
H CH CH Cl So, the value of N will be 1 + 2 + 2 + 1 = 6.
Since enantiomers have nearly same physical
Isopropylbenzene Cumene
hydroperoxide properties, II and III as well as IV and V can’t be
separated, hence the number of isomers (M) will be
OH 1 + 1 + 1 + 1 = 4.
O –O–H
60. (a) Mg 2 NH 3 HPO 24 Mg ( NH 4 ) PO 4
+
O 61. (d) The reagent used in Hofmann bromamide reaction is
Phenol Acetone (Q) alkaline halogen (NaOH or KOH + X2).
EBD_780
2006- 26 Target IIT-JEE
62. (d) Conversion of (iii) to (iv) involving rearrangement is 68. (b) As the half-life of C14 is 5760 years, so a 6 year old
the slowest step. Species (iii) is electron deficient (N fossil’s age can’t be determined. Further this technique
has only 6 electrons), hence it has a tendency to get its cannot be used to date objects older than 30,000 years.
octet completed by migration of alkyl group. After this length of time the radioactivity is too low to
63. (b) Since the reaction is intramolecular, no cross product be measured.
will be formed.
69. (a) 1 1
T1 ln C1 , T2 ln C2
CONH2 NH2
Let the concentration of C14 in the fossil be C. In nearby
Br 2,
; areas concentration of C14 in living beings will be C1
KOH
D D and in far off places C2, obviously C1 > C2.
(i) Hence, age of fossil in nearby areas,

15 15 1 C1
CONH2 NH2 T1 ln ....... (i)
C

; 1 C2
And age in far off places, T2 ln ..... (ii)
C
(ii)
1 C1
From (i) and (ii), T1 T2 ln
2 KCN 2 KCl C2
64. (a) Ni K 2 [Ni(CN) 4 ]; Ni K 2 [NiCl4 ]
excess (excess)

65. (c) For [Ni(CN)4]2+; Ni = [Ar]3d 84s2; Ni2+ = [Ar]3d 8 Since, C1 > C2, R.H.S. is positive i.e., T1 > T2.
3d 4s 4p 70. (d) In the given reaction,
Ni2+
Ag ions are reduce to Ag and Glucose is oxidised to
However, CN– is a strong field ligand so it forces the 3d gluconic acid as per the given reactions,
electrons to pair up and hence the effective
configuration in this case will be Ag e Ag; E ored 0.800 V and
Ni2+ in presence of CN–
3d 4s 4p C6 H12O6 H 2O C6 H12 O7 2H 2e ;
Gluconic acid
× × × ×

dsp 2 E oox 0.05V


Thus [Ni (CN)4]2– exhibits dsp 2 hybridization and
square planar shape. Since here number of unpaired Hence, E ocell 0.8 0.05 0.75 V
electrons is zero the complex will be diamagnetic.
In case of [NiCl4]2–, Cl– is a weak field ligand, so the G ocell nFE 2F 0.75 RT ln K
effective configuration of Ni2+ in this complex will be
as follows : 2F
ln K (0.75) 2 38.92 0.75 58.38
Ni2+ in presence of Cl– RT
3s 4s 4d 71. (a) For the reaction,
× × × ×
C 6 H12O 6 H2O C 6 H12O 7 2H 2e
sp 3 Gluconic acid

So here Ni2+ is sp3 hybridised and thus tetrahedral in


shape. Since the complex has two unpaired electrons, 0.0591 [P] 0.0591
E E ln E ln[H ]2
it will be paramagnetic. n [R ] 2
66. (a) Discussed above.
67. (c) It is clear from the 3rd paragraph, which states that in 0.0591
E E 2 ln( pH) 0.0591 11 0.65
living organisms a dynamic equilibrium is established 2
whereby the ratio of C14 to C12 remains constant. The
C14 which decays into N14 is replenished by the So, Eoxidation increases over E ooxidation by 0.65 V..
production of new isotopes.
2006 IIT-JEE Solved Paper 27
2006-

72. (b) During Tollen’s test, oxidation of silver ion requires an So the magnitude is 557 kJ mol–1.
alkaline medium. Under these conditions it forms G° = – 2.303RT log Kp at equilibrium G° = 0
insoluble silver oxide, hence to dissolve this oxide a – 2.303RT log Kp = 0
complexing agent, ammonia is added, which brings log Kp = 0 or Kp = 1
silver ion as diamminosilver (I) ion, [Ag(NH3)2]+. It is a
75. Given : m (Ag ) 6 10 3 ; m (Br ) 8 10 3 ;
soluble complex.
m (NO3 ) 7 10 3 and K sp (AgBr) 12 10 14
73.
To find the specific conductivity ( ) of the final solution of
AgBr in which AgNO3 (10–7 M) is mixed we must find the
individual of the ions.

Total number of moles at equilibrium = 1 1 or so ln Ag Br NO3


2 2

T f =iK f ×(molality) Again, m molar concentration


Calculation of molar concentration of ions :
75.2 weight of phenol = 75.2g Concentration,
7 14 1
94 2 mol.wt of phenol = 94 [ NO3 ] 10 7
moles / l 10 4
moles / m 3
0.75 Let x be the molar concentration of Ag+ from AgBr
So the percentage of phenol that dimerises = 75%. (x 10 7 )x 12 10 14
Alternately :
or x 2 10 7 x 12 10 14 0
Phenol dimerises in the solvent (organic)as :
or, ( x 4 10 7 7 7
)(x 3 10 ) 0 x 3 10 M
2C6 H5OH (C 6 H5OH) 2
7 4
[Br ] 3 10 M 3 10 moles / m 3 and
1mol 0 mol (Initial)
1 – mol / 2 mol (Equilibrium) [Ag ] 3 10 7 10 7 4 10 7 M

4 10 4 moles / m3
van't Hoff factor, i = 1 – + 1
2 2
6×10-3 × 4 ×10-4
Ag
where is the degree of dimerisation.
T f (obs) Kf molality i 24 10 7 (Sm 2 mol 1 mol / m3 ) 24 10 7 S / m
3 4
Similarly, 8 10 3 10 24 × 10-7 S/m and
Br
or 7 = 14 × m × 1
2 3 4 7
7 10 10 7 10 S/ m
In the given problem, amount of the solvent used to dis- NO3

solve 75.2 g of phenol is not mentioned. Hence molality m, 7 7


(24 24 7) 10 S/ m 55 10 S/ m
can not be calculated and so also the value of .
If amount of solution is presumed to be 1 kg ( = 100g) then, So the correct answer is 55.

75.2 3
m 0.8 (mol. mass of phenol = 9.4) 76. For bcc ; r a;
94 2

75.2 n M d N Av a3
And 7 = 14 × (1 – ) d or n
94 2
N Av a3 M
or 0.75 75%
74. H U ( PV ) U V P ( V 0) 2 6 1023 (5 10 8 )3
n 2
75
or U H V P 560 [1(40 70) 0.1]
Therefore Metal crystallizes in BCC structure and for a BCC
1
560 3 557 kJ mol lattice 3a 4r
EBD_780
2006- 28 Target IIT-JEE
(C) Angular momentum of electron in lowest (1s) orbital
3 3 5
r a 2.165 Å 216.5 pm h h
4 4 = ( 1) 0(0 1) 0; (C) (p)
2 2
So the required answer is 217 pm.
1
77. A-p, r; B-p, r; C-q; D-s (D) Z 1; (D) (s)
rn
The oxides and sulphides of less active metals like Hg, Cu &
80. A-q; B-p, s; C-r, s; D-q
Pb are unstable to heat and hence no reducing agent is
E 1 mechanisms are encountered only with tertiary or
required. They undergo self reduction.
secondary substrates and in presence of either a weak base
(A) 2Cu 2 S 3O 2 2Cu 2 O 2SO 2 ; or a base in low concentration. So primary substrates will
follow E2 mechanism,
Cu 2S 2Cu 2 O 6Cu SO 2
i.e. (A) E2 and (D) E2.
Further E1 mechanism (similar to SN1) proceeds by first order
(B) kinetics and is determined by the slower (first) step of the
2PbS 3O 2 2 PbO 2SO 2 ;
formation of carbocation. Hence (B) E1 and first order
reaction.
PbS 2PbO 3Pb SO 2
Reaction of C6H5CH2CH2Br on treatment with C2H5O– in
Hence (A) (p), (r) presence of C2H5OD gives C6H5CD=CH2. This reaction
The oxides of less electropositive metals like Pb, Zn, Fe, Sn, follows E1CB (Elimination unimolecular conjugate base)
Cu, etc. are reduced by strongly heating them with coke or mechanism. This 2 step mechanism follows the following
path :
coal.
H –
C2H5O
PbO C Pb CO 2 ; –
C6H5 – CH – CH2 C6H5 – CH – CH2
fast
2Cu 2 O C 4Cu CO 2 Br Br

Hence (B) (p), (r) H D D


C HO C2H 5OD –
Extraction from argentite (Ag2S) C H – CH – CH C6H5 – CH – CH2Br C6H5 – C – CH2 – Br
fast fast
Br
Ag2S + 2NaCN Na2S + 2AgCN;
slow –
C6H5 – CD = CH2 + Br
AgCN NaCN Na[Ag(CN)2 ]
Sod. argentocyanide soluble (formation of carbanion and elimination
of Br from carbanion)
Zn, being more electropositive than Ag, displaces Ag from Although this mechanism involves 2 steps the overall rate
the complex. of the reaction is limited to the slower second step and hence
the rate of reaction depends only on the concentration of
2 Na[ Ag ( CN ) 2 ] Zn Na 2 [ Zn ( CN ) 4 ] 2 Ag the carbanion, i.e. first order reaction. Hence, (C) (r), (s).
Hence (C) (q)
81. (c) lim (sin x)1/ x (1/ x)sin x
Among the halides of Boron, BI3 is unstable because of the x 0
large size of Iodine and small size of Boron atom. Hence it
sin x
decomposes to give Boron. Thus, (D) (s). 1
lim (sin x)1/ x lim
78. A-q, Bi3+ hydrolyses to yield BiO+ ion x 0 x 0 x
B-s, AlO2– on dilution yields a white ppt. of Al(OH)3
1
C-p, When heated (SiO4)4– changes to (Si2O7)6– lim sin x log
x 0 x [ |sin x| < 1 when x 0]
0 e
D-r, When acidified (B4O7)2– gives B(OH)3 (or H3BO3)
79. A-r; B-q; C-p; D-s logx 1/x
lim lim
x 0 cosec x 0 cosecx cotx
e e
Vn Kze 2 / r i
(A) 2; where K (A) – (r)
Kn Kze2 / 2r 4 0
[Using L' Hospital rule]
sin x
lim . tan x
(B) rn ( En ) 1; (B) (q) e x 0 x
e0 1
2006 IIT-JEE Solved Paper 2006- 29

x2 1 x2 1 85. (d)
w - wz
is purely real
82. (d) dx dx
3 4 2 2 1 1- z
x 2x 2x 1 x5 2
x2 x4 w – wz w wz w wz w wz
4 4 1– z 1 z 1 z 1 z
3
1 x x5 dx w wz wz wzz w wz wz wzz
4 2 1
2 w w ( w w ) | z |2
x2 x4
| z |2 = 1 ( w i and 0)
2 1
Put 2 t | z | = 1 also given z 1
x2 x4
The required set is {z : | z | =1, z 1}
4 4 1 dt 2 t
dx dt c 3 ( 1)
x3 x5 4 t 4
86. (a) a, b, c are sides of a s.t. a b c
2 1
2 |a b| |c| a2 b2 2ab c 2
x2 x4 2 x4 2 x2 1
c c Similarly we have b2 + c2 – 2bc < a2; c2 + a2 – 2ca < b2
2 2 x2 On adding, we get
83. (c) By Sine law in ABC
x a
a x 3 a 2 b2 c2
a2 + b2 + c2 < 2(ab + bc + ca) 2
sin 30 sin120 ab bc ca
A
....(1)
120° Roots of the given equation are real
x x
(a b c) 2 3 (ab bc ca ) 0

a 2 b2 c2
B C 3 2 ....(2)
a ab bc ca
1 3 2
x x sin120 x 4
2 4 From (1) & (2) we get, 3 2 2 .
3
(2 x a ) 3 2
Also, 3 3 x 2 2
s 2 4 x x
87. (a) F (x) f g
3 2 2
x = 2 (2 + 3) 4 (4 3 4 3)
4 x x 1 x x 1
F '(x) = 2 f .f ' . 2g .g ' .
7 3 12sq.units. 2 2 2 2 2 2
84. (a) 2 sin2 5 sin 2 0 x x x x
(sin 2) (2sin 1) 0 f .f ' f' . f ''
2 2 2 2
1 [ 1 sin 1]
sin [ g (x) = f ' (x) g' (x) = f '' (x)]
2
y x x x x
f .f ' –f' f
2 2 2 2
=0 [ f ''(x) = – f (x)]
/6 F (x) is a constant function.
x= x = 5 /6
y
1
F (x) = F (5) = 5 x R
2
F (10) = 5
0 2 x 88. (c) According to question lcm (p, q) = r2t4s2. For lcm (p, q)
to contain r2 following are the possible powers of r in p
From graph, we get and q. (0, 2); (1, 2); (2, 2)
Out of these all except (2, 2) can be interchanged for p
5
x 0, ,2 & q.
6 6
EBD_780
2006- 30 Target IIT-JEE
Total ways for p, q to contain different powers of r 93. (a,b) if y = mx + c is tangent to y = x2 then
are = 2 × 3 – 1 = 5 x2 – mx – c = 0 has equal roots
Similarly different powers of t that can be given to p &
q are (0, 4), (1, 4), (2, 4), (3, 4), (4, 4) out of which except m2
m2 + 4c = 0 c –
(4, 4) all others can be interchanged for p and q. 4
Total ways for p, q to contain different powers of t
are = 2 × 5 – 1 = 9 m2
y mx – is tangent to y = x2
Similarly total ways for p, q to contain different powers 4
of S are = 2 × 3 – 1 = 5 This is also tangent to y = – (x – 2)2
Total ordered pairs of p, q can be = 5 × 9 × 5 = 225.
m2
mx – – x2 4x – 4
89. (b) 0, tan 1 and cot 1 4
4
Let tan 1 x and cot 1 y m2
x2 + (m – 4)x + 4 – 4 = 0 has equal roots
Where x, y > 0 and are very small, then
t1= (1 – x)1– x, t2 = (1– x)1+ y, t3
= (1+ y)1– x, t4 = (1 + y)1+ y m2 – 8m + 16 = – m2 + 16 m = 0, 4
Clearly, t4 > t3 and t1 > t2 also, t3 > t1 y = 0 or y = 4x – 4 are the tangents.
Thus t4 > t3 > t1> t2. 94. (a,b,c) From graph, f (x) is continuous everywhere but not
90. (d) Vertex is (1, 1) and focus is (2, 2), directrix x + y = 0 differentiable at x = 1.
Equation of parabola is
2
x y y
( x – 2)2 ( y – 2) 2 y = x3
2 y = x2
x2 + y2 – 2xy = 8 (x + y – 2)
(x – y)2 = 8 (x + y – 2)
91. (d) The equation of plane through the point (1, –2, 1) y=1
(1, 1)
and perpendicular to the planes 2 x 2 y z 0 and
x y 2z 4 is given by x' x
O
x 1 y 2 z 1
2 2 1 0
1 1 2

x y 1 0
It’s distance from the point (1, 2, 2) is y'
y = f (x)
1 2 1
2 2.
2
95. (c, d) Tangent to the curve y f ( x) at (x, y) is
92. (a) A vector in the plane of a and b is dy
Y y ( X x)
u a b (1 )i (2 ) j (1 )k dx
dy
1 x y
u.c 1 dx dy
Projection of u on c A ,0 ; B 0, x y
3 |c| 3 dy dx
dx
u.c 1
dy
|1 2 1 | 1 3 x y
dx
|2 | 1 1 or 3 1 0
dy
u 2i j 2k or 4i j 4k BP : PA 3 :1 x dx
4
2006 IIT-JEE Solved Paper 2006- 31
dy A
x 3y 0
dx Also AD = AE cos
2
dy dx
3
y x 2bc
AE HM of b and c.
c b c
log y 3log x log c y
x3
As curve passes through (1, 1), c = 1 A
4bc sin
A 2
curve is x3 y 1 , which also passes through Again EF = 2DE = 2. AD tan
2 b c
1
2, . 98. (b, c) Let f (x) = ax3 + bx2 + cx + d
18
Then, f (2) = 18 8a + 4b + 2c + d = 18 … (1)
x2 y2 f (1) = – 1 a+b +c+d=–1 … (2)
96. (a,c) For the given ellipse 1
25 16 f (x) has local max. at x = – 1
16 3 3a – 2b + c = 0 … (3)
e = 1–
25 5 f ( x) has local min. at x = 0 b=0 … (4)
5 Solving (1), (2), (3) and (4), we get
Eccentricity of hyperbola
3 1 17
f ( x) (19 x3 57 x 34) f (0)
x2 y2 4 2
Let the hyperbola be – 1 then
A2 B2 57 2
Also f ( x) ( x 1) 0, x 1
25 16 2 4
B2 A2 –1 A
9 9 Also f ( x) = 0 x = 1, – 1
2 2
x 9y f ( 1) 0, f (1) 0 x 1 is a point of local max.
2
– 1 As it passes through focus of ellipse
A 16 A2 and x = 1 is a point of local min. Distance between (– 1,
i.e. (3, 0)
2) and (1, f (1)), i.e. (1, – 1) is 13 2 5
we get
99. (b, d) Normal to plane P1 is
A2 = 9 B2 = 16
2 2
n1 (2i 3k ) (4 j 3k ) 18i
Equation of hyperbola is x – y 1 , focus of Normal to plane P2 is
9 16
hyperbola is (5, 0), vertex of hyperbola is (3, 0). n2 ( j k ) (3i 3 j ) 3i 3 j 3k

A is parallel to (n1 n 2 ) ( 54 j 54 k )
97. (a,b,c,d) By simple geometry in AFE, AF = AE
AFE is an isosceles . Now, angle between A and 2i j 2k is given by
Now Ar ( ABC) = Ar ( ABD) + Ar ( ADC)
1 1 A 1 A ( 54 j 54k ).(2i j 2k ) 1
cos
bc sin A cAD sin bAD sin 54 2.3 2
2 2 2 2 2
A 3
2bc cos or
2 4 4
AD
b c x
100. (a, b) g (x) = f (t ) dt
0
A
b
A/2 A/2 ex , 0 x 1
x 1
c C g ( x) f ( x) 2 e , 1 x 2
F E x e, 2 x 3
D

B g ( x) 0 ex 1
2 or x e 0
EBD_780
2006- 32 Target IIT-JEE

x 1 log 2 or x = e n
c
x = 1 + ln 2 or e n 1
1 2 n
c
x
e , 0 x 1 n 1 n 1 n 1
x 1 n n 1 2
g ( x) e , 1 x 2
1, 2 x 3 n 1 n ( n 1) n 1
2

g (1 ln 2) 2 and g (e) 1 g ( x) has local max. P( w E )


103. (b) P(w/E)
at x = 1 + ln 2 and local min. at x = e. P (E)

1 2
1 14 6 1 n
n n 1 n n 1 n n 1 n n 1
3
1 1 1 n
times
n n n 2
2
2 n
1 2 3
1 n(n 1) 2 (n being even)
1 2 3 1 n
e X n 2
(1+ ln 2)
Graph of g'(x) n n
1
4 2 2 n 2
n(n 1) 2 2(n 1)
Also graph of g ( x) suggests, g (x) has local max. at x
= 1 and local min. at x = 2
101. (b) P (ui ) i P(ui ) ki, But P (ui ) 1
/2
0
2
2 104. (a) sin x dx sin 0 sin 2sin
ki 1 k i 1 k 0 4 2 4
n ( n 1)
= (1 2)
2i 8
P(ui )
n(n 1) x x a
By total prob. theorem f ( x)dx ( f (x) f (a))
a 2
105. (d) lim 0
n x a ( x a )3
P ( w) P (ui ) P ( w / ui )
i 1 a h h
f ( x)dx ( f (a h) f (a ))
n lim a 2 0
2i i h 0 h3
n( n 1) n 1
i 1 1 h
f (a h) [ f (a) f ( a h) ] ( f '(a h ))
lim 2 2 0
2 n( n 1) (2n 1) 2n 1
2
. h 0 3h 2
n ( n 1) 6 3n 3
[Using L’Hospital rule]
2n 1 2 1/ n 2 1 1 h
lim P ( w) lim lim f (a h) f (a) f '(a h)
n n 3n 3 n 3 3/ n 3 lim 2 2 2 0
102. (a) P(ui) = c h 0 3h 2
Using Baye’s thm. 1 1 h
f '( a h) f '( a h) f ''( a h)
P (w / un ) P (un ) lim 2 2 2 0
P(un/w) n h 0 6h
P (w / ui ) P(ui )
i 1 [Using L’Hospital rule]
2006 IIT-JEE Solved Paper 2006- 33
f ''( a h) Then the centre of C' is equidistant from the centre of
lim 0 f "( x) 0, a R C1 and from line T.
h 0 12
locus of centre of C' is a parabola.
f (x) must be of max. degree 1.
106. (a) f "( x) 0, x (a, b), for c (a, b)
c a b c
F (c ) ( f (a) f (c )) ( f (b) f (c ))
2 2
O L
b a c a b c C'
f ( c) f (a) f (b )
2 2 2 T
b a 1 1 C1
F '(c) f '(c) f (a) f (b )
2 2 2
1
[(b a) f '(c ) f (a ) f (b)]
2
1
F "(c ) (b a) f "(c ) 0
2
[ f "( x ) 0, x (a, b) and b > a]
F (c) is max. at the point (c, f (c)) where 109. (c) Since S is equidistant form A and line BD, it traces a
F’ (c) = 0 parabola. Clearly, AC is the axis, A (1, 1) is the focus

f (b) f (a) 1 1
f ’ (c) and T1 , is the vertex of parabola.
b a 2 2
107. (a) Without loss of generality we can assume the square 1
ABCD with its vertices A (1, 1), B (–1, 1), C (–1, –1), D AT 1 .
(1, –1) 2
T2 T3 = latus rectum of parabola
P to be the point (0, 1) and Q as ( 2, 0 ).

Y Y

T2

P (0,–1) (–1,1)B A(1,1)


B A(1,1)
(–1,1) T3
L T1
Q ( 2, 0)
O X' O X
X' X'
C2
C1
C D(1,–1) C D
(–1,–1) (–1,–1) (1,–1)

Y'
Y'

PA2 PB 2 PC 2 PD 2 1
Then, 4 2 2
2 2 2
QA QB QC QD 2 2
1 1 5 5 12 1 1 1
0.75 Area ( T1T2T3) = 2 2 1 sq. units
2[( 2 1) 2
1] 2(( 2 1) 2
1] 16 2 2 2

108. (b) Let C' be the said circle touching C1 and L, so that C1
a 1 0 0 a 1
and C' are on the same side of L. Let us draw a line T
parallel to L at a distance equal to the radius of circle 110. (a) Let U1 b then 2 1 0 b 0
C1, on opposite side of L. c 3 2 1 c 0
EBD_780
2006- 34 Target IIT-JEE

1 1
a 1 x (1 x50 )101 101 50 x50 (1 x50 )100 dx
0 0
2a b 0
3a 2b c 0 1
5050 x50 (1 x50 )100 dx
0
a = 1, b = – 2, c = 1
1
1 I' 5050 x50 (1 x50 )100 dx
0
U1 –2
1
1 5050 I I ' 5050 (1 x 50 )100 dx
0

1
2 2 5050 x50 (1 x50 )100 dx
Similarly, U2 –1 , U3 –1 0

–4 –3 1
5050 (1 x50 )101 dx 5050 I '
0
1 2 2
I
U –2 –1 –1 |U|=3 5050 I 5051 I ' 5050 5051
I'
1 –4 –3
2 3 n
3 3 3 3
115. an .... ( 1) n 1

–1 –2 0 4 4 4 4
1
111. (b) U–1 –7 –5 –3
n
3 3 3
9 6 3 1 n
4 4 3 3
= 1
3 7 4
1 1
Sum of elements of U–1 (0) 0 4
3
bn = 1– an and bn > an n n0
1 2 2 3 7 1– an > an 2an < 1
112. (a) [3 2 0] –2 –1 –1 2 = [3 2 0] –8 5 n n
6 3 3 1
1 –4 –3 0 –5 1 1
7 4 4 6
113. Roots of x2 – 10cx – 11d = 0 are a and b (– 3)n +1 < 22n–1
a + b = 10c and ab = – 11d For n to be even, inequality always holds. For n to be odd,
Similarly c and d are the roots of x2 – 10ax – 11b = 0 it holds for n 7.
c + d = 10a and cd = – 11b The least natural no., for which it holds is 6
a + b + c + d = 10 (a + c) and abcd = 121 bd ( it holds for every even natural no.)
b + d = 9(a + c) and ac = 121 d
116. g (x) = (f ' (x))2 + f '' (x) f (x) = ( f ( x) f '( x))
Also we have a2 – 10 ac – 11d = 0 and c2 – 10ac – 11b = 0 dx
a2 + c2 – 20ac – 11 (b + d) = 0 Let h (x) = f (x) f ' (x)
(a + c)2 – 22 × 121 – 99 (a + c) = 0 Then, f (x) = 0 has four roots namely a, , , e
a + c = 121 or – 22 where b < < c and c < < d.
For a + c = – 22, we get a = c And f ' (x) = 0 at three points k1, k2, k3
rejecting this value we have a + c = 121 where a < k1 < , < k2 < , < k3 < e
[ Between any two roots of a polynomial function
a + b + c + d =10 (a + c) = 1210
f (x) = 0 there lies atleast one root of f ' (x) = 0]
1 1 There are atleast 7 roots of f (x) . f ' (x) = 0
114. Let I (1 x 50 )100 dx and I ' (1 x50 )101 dx
0 0
d
There are atleast 6 roots of ( f ( x) f '( x)) 0
1 dx
Then, I ' 1.(1 x50 )101 dx
0 i.e. zeros of g (x) = 6
2006 IIT-JEE Solved Paper 35
2006-

117. Let y = mx – 2m – m3 be the equation of normal to y2 = 4x.


dy
As it passes through (3, 0), we get m = 0, 1, – 1 1 m1
Then three points on parabola are given by (m2, – 2m) for dx (1,0)
m = 0, 1, – 1
P (0, 0), Q (1, 2), R (1, – 2) dy
For second curve x x (1 log x)
dx
0 0 1
1
Area of PQR 1 2 1 2 sq. units dy
2 1 m2
1 –2 1 dx (1,0)

Radius of circum-circle, m1 = m2 Two curves touch each other


abc 5 5 4 5 Angle between them is 0°
R cos = 1,
4 4 2 2
(where, a, b, c are the sides of PQR) (C) (p)

2 dy 6
Centroid of PQR ,0 (D)
3 dx x y

5 dx 1 y
Circumcentre ,0 x
2 dy 6 6
Thus, A-p; B-q; C-s; D-r I.F. = e–y/6
118. A-p, B-s, C-p, D-r
Solution is xe–y/6 = – ye–y/6 – 6e–y/6 + c
/2 x + y + 6 = cey/6
(A) (sin x )cos x (cos x cot x – log(sin x )sin x ) dx
0
x + y + 6 = 6ey/6 (y (0) = 0)
1 12 = 6ey/6 (using x + y = 6)
= du where (sin x)cos x = u = 1
0 y = 6 ln 2 (D) (r)
(A) (p) 119. A-s; B-p; C-q, r; D-s
(A) On solving the given equations
(B) (–4, 1) Y x + y = |a| and ax – y = 1, we get

1 |a| a | a | –1
x and y
a 1 a 1
X
(1, 0) Rays intersect each other in I quad.
x, y > 0 a + 1 > 0 and a|a| – 1 > 0 a>1
a0 = 1(A) (s)
(–4, –1)
(B) ( , , ) lies on the plane x + y + z = 2
+ + =2
2 1 1
Solving y – x and y 2 – ( x –1) , we get Also k (k a) (k.a)k – (k.k ) a
4 5
intersection points as (– 4, + 1) k– i– j– k 0
Required area
i j 0
1 1 4
[(1 – 5 y 2 ) 4 y 2 ]dy 2 (1 – y 2 )dy , =0= =2 ( + + = 2)
–1 0 3
(B) (p)
(B) (s)
(C) By inspection, the point of intersection of two 1 1
(C) (1 – y 2 )dy ( y 2 – 1)dy
curves y = 3x–1 log x and y = xx – 1 is (1, 0) 0 0

dy 3x –1 1 4
For first curve 3x –1 log 3 log x 2 (1 – y 2 )dy
dx x 0 3
EBD_780
2006- 36 Target IIT-JEE

1
1 – x dx
0
1 x dx
1 t lim tan 1 (2n 1) tan 1 1
Also, 2 1 – x dx n
0 –1 0

1 2n 1 1
lim tan lim tan
n 1 (2 n 1) n 1 1/ n
1 –1
0
0 t tan 1 (1) tan t 1 (A)-(p)
4
(B) a, b, c are in AP 2b = a + c
[ y 1 – x , i.e., y2 = – (x – 1) and y 1 x
a 1 tan 2 1 / 2 a
2
i.e., y = (x + 1) represent same area under the given cos 1
limits] b c 1 tan 2 / 2 b c

1 a a b c a
2
0
x dx Using
0
f ( x ) dx
0
f ( a – x )dx tan 2 1
2 b c a
1
2 4 a b c
2. x3/ 2 , (C) (r) and (q) Similarly tan 2 3
3 0 3 2 a b c
(D) Given : sin A sin B sin C + cos A cos B = 1 1 tan 2 3 2b 2b 2
But sin A sin B sin C + cos A cos B tan (B)-(r)
2 2 a b c 3b 3
sin A sin B + cos A cos B = cos (A – B) (C) Equation of line through (0, 1, 0) & perpendicular to x +
cos (A – B) 1 cos (A – B) =1
A–B =0 A=B x y 1 z
2y + 2z = 0 is
Given relation becomes sin2A sin C + cos2 A = 1 1 2 2
sin C = 1, For some value of , the foot of perpendicular from
(D) (s) origin to line is ( , 2 + 1, 2 )
120. A-p; B-r; C-q dr's of this from origin are , 2 + 1, 2

1 (2i 1) (2i 1) 2
(A) t tan 1 tan 1 1. 2.( 2 1) 2.2 0
2 9
i 1 2i i 1 1 4i 2 1
2 5 4
Foot of perpendicular is , ,
9 9 9
[tan 1 (2i 1) tan 1 (2i 1)]
i 1 4 25 16 45 5
Required distance
t = tan–13 – tan–11 + tan–15 – tan–13 +........ 81 81 81 81 3
+ tan–1(2n + 1) – tan–1(2n – 1) +........ (C)-(q)

You might also like